Download as pdf or txt
Download as pdf or txt
You are on page 1of 317

CONSTITUTIONAL LAW REVIEW

3. Qualities of a good written constitution (Bar 2012)


a. Broad. Not just because it provides for the organization of the entire government
CONSTITUTIONAL LAW REVIEW and covers all persons and things within the territory of the State but because it must
be comprehensive enough to provide for every contingency.
1st Semester, School Year 2018-2019 b. Brief. It must confine itself to basic principles to be implemented with legislative
details more adjustable to change and easier to amend.
Under Atty. Enan Flores
c. Definite. To prevent ambiguity in its provisions which could result in confusion and
divisiveness among the people.

CHAPTER 1: CONSTITUTIONAL LAW 4. Essential Parts of a written constitution

a. Constitution of Liberty. The series of prescriptions setting forth the fundamental


civil and political rights of the citizens and imposing limitations on the powers of
I. The Philippine Constitution government as a means of securing the enjoyment of those rights, e.g. Art. III, Art. IV.
b. Constitution of Government. The series of provisions outlining the organization of
1. Definition, nature, concept and purpose of Constitution the government, enumerating its powers, laying down certain rules relative to its
administration, and defining the electorate, e.g., Arts. VI, VII, VIII and IX.
a. Definition: That body of rules and maxims in accordance with which the powers of
sovereignty are habitually exercised [Cooley]. With particular reference to the c. Constitution of Sovereignty. The provisions pointing out the mode or procedure in
Constitution of the Philippines: That written instrument enacted by direct action of the accordance with which formal changes in the fundamental law may be brought about,
people by which the fundamental powers of the government are established, limited e.g., Art. XVII.
and defined, and by which those powers are distributed among the several departments
for their safe and useful exercise for the benefit of the body politic [Malcolm].
5. Effectivity of the 1987 Philippine Constitution (Art. XVIII, Sec. 27)
b. Purpose: To prescribe the permanent framework of a system of government, to
assign to the several departments their respective powers and duties, and to establish NOTE: The 1987 Constitution’s date of effectivity is February 2, 1987, the date of the
certain first principles on which the government is founded. plebiscite when the people ratified the Constitution. (De Leon v. Esguerra)

a. De Leon vs. Esguerra, 153 SCRA 602, No. L-78059, 31 August 1987

2. Classification FACTS: Petitioners pray that the subject Memoranda of February 8, 1987 be
declared null and void and that respondents be prohibited from taking over their
a. Written or unwritten. A written constitution is one whose precepts are embodied
positions. And maintains that pursuant to Section 3 of the Barangay Election Act of
in one document or set of documents; while an unwritten constitution consists of rules
1982 (BP Blg. 222), their terms of office "shall be six (6) years which shall
which have not been integrated into a single, concrete form but are scattered in various
commence on June 7, 1982 and shall continue until their successors shall have
sources, such as statutes of a fundamental character, judicial decisions, commentaries
of publicists, customs and traditions, and certain common law principles. elected and shall have qualified," or up to June 7, 1988.

b. Enacted (Conventional) or Evolved (Cumulative). A conventional constitution is ISSUE: Whether or not the designation of respondents to replace petitioners was
enacted, formally struck off at a definite time and place following a conscious or proper?
deliberate effort taken by a constituent body or ruler; while a cumulative constitution is
RULING: No. The 1987 Constitution was ratified in a plebiscite on Feb 2, 1987,
the result of political evolution, not inaugurated at any specific time but changing by
accretion rather than by any systematic method. therefore, the Provisional Constitution must be deemed to have superseded. Having
become inoperative, respondent OIC Gov could no longer rely on Sec 2, Art 3,
c. Rigid or Flexible. A rigid Constitution is one that can be amended only by a formal thereof to designate respondents to the elective positions occupied by petitioners.
and usually difficult process; while a flexible Constitution is one that can be changed by
ordinary legislation. NOTE: Article XVII needs qualification. Petitioners must now be held to have acquired security of tenure, Sec 8, Art 1 of
the 1987 Constitution further provides in part:

M.R.A.D.C. LUMBRE 1
CONSTITUTIONAL LAW REVIEW

"The term of office of elective local officials, except barangay officials, which Lambino v. Comelec enumerates the distinctions between revision and amendment,
shall be determined by law, shall be three years xxx." as follows: Revision broadly implies a change that alters a basic principle in the
Constitution, like altering the principle of separation of powers or the system of
NOTE: Applicable rule: Article XVIII, Section 3. Where the old law is: (1) checks and balances. There is also revision if the change alters the substantial
inconsistent; or (2) amended/repealed consequently, respondents can no longer entirety of the Constitution. On the other hand, amendment broadly refers to a
rely on the old law. change that adds, reduces, deletes, without altering the basic principle involved.
Revision generally affects several provisions of the Constitution; while amendment
b. Laws and international agreements prior to the effectivity of the 1987 generally affects only the specific provision being amended.
Constitution – Art. XVIII, Secs. 3 and 4
i. Lambino vs. Comelec, GR No. 174153, October 25, 2006
Section 3. All existing laws, decrees, executive orders, proclamations, letters of
instructions, and other executive issuances not inconsistent with this Constitution FACTS: On 25 August 2006, Lambino et al filed a petition with the COMELEC
shall remain operative until amended, repealed, or revoked. to hold a plebiscite that will ratify their initiative petition to change the 1987
Constitution under Section 5(b) and (c) and Section 73 of Republic Act No.
Section 4. All existing treaties or international agreements which have not been 6735 or the Initiative and Referendum Act.
ratified shall not be renewed or extended without the concurrence of at least two-
thirds of all the Members of the Senate. The Lambino Group alleged that their petition had the support of 6,327,952
individuals constituting at least twelve per centum (12%) of all registered
voters, with each legislative district represented by at least three per centum
6. Amendments and revision (Art. XVII, Secs. 1 to 4) (3%) of its registered voters. The Lambino Group also claimed that COMELEC
election registrars had verified the signatures of the 6.3 million individuals.
Section 1. Any amendment to, or revision of, this Constitution may be proposed by:
The Lambino Group’s initiative petition changes the 1987 Constitution by
The Congress, upon a vote of three-fourths of all its Members; or a constitutional modifying Sections 1-7 of Article VI (Legislative Department) and Sections 1-
convention. 4 of Article VII (Executive Department) and by adding Article XVIII entitled
Section 2. Amendments to this Constitution may likewise be directly proposed by the “Transitory Provisions.” These proposed changes will shift the present
people through initiative upon a petition of at least twelve per centum of the total Bicameral-Presidential system to a Unicameral-Parliamentary form of
number of registered voters, of which every legislative district must be represented by government.
at least three per centum of the registered voters therein. No amendment under this RULING: The Initiative Petition Does Not Comply with Section 2,
section shall be authorized within five years following the ratification of this Constitution Article XVII of the Constitution on Direct Proposal by the People
nor oftener than once every five years thereafter.
Section 2, Article XVII of the Constitution is the governing constitutional
The Congress shall provide for the implementation of the exercise of this right. provision that allows a people’s initiative to propose amendments to the
Section 3. The Congress may, by a vote of two-thirds of all its Members, call a Constitution.
constitutional convention, or by a majority vote of all its Members, submit to the The framers of the Constitution intended that the “draft of the proposed
electorate the question of calling such a convention. constitutional amendment” should be “ready and shown” to the people “before”
Section 4. Any amendment to, or revision of, this Constitution under Section 1 hereof they sign such proposal. The framers plainly stated that “before they sign there
shall be valid when ratified by a majority of the votes cast in a plebiscite which shall be is already a draft shown to them.” The framers also “envisioned” that the
held not earlier than sixty days nor later than ninety days after the approval of such people should sign on the proposal itself because the proponents must
amendment or revision. “prepare that proposal and pass it around for signature.”

Any amendment under Section 2 hereof shall be valid when ratified by a majority of the The essence of amendments “directly proposed by the people through initiative
votes cast in a plebiscite which shall be held not earlier than sixty days nor later than upon a petition” is that the entire proposal on its face is a petition by the
ninety days after the certification by the Commission on Elections of the sufficiency of people. This means two essential elements must be present. First, the people
the petition. must author and thus sign the entire proposal. No agent or representative can
sign on their behalf. Second, as an initiative upon a petition, the proposal must
a. Difference be embodied in a petition.

M.R.A.D.C. LUMBRE 2
CONSTITUTIONAL LAW REVIEW

These essential elements are present only if the full text of the proposed members of Congress with the question of whether or not to call a
amendments is first shown to the people who express their assent by signing Convention to be resolved by the people in a plebiscite [Sec. 3, Art. XVII]
such complete proposal in a petition. Thus, an amendment is “directly
a. Imbong v. COMELEC, 35 SCRA 28, (1970)
proposed by the people through initiative upon a petition” only if the people
sign on a petition that contains the full text of the proposed amendments. FACTS: Two separate but related petitions for declaratory relief were
filed pursuant to Sec. 19 of R.A. No. 6132 by petitioners Manuel B.
There is no presumption that the proponents observed the constitutional Imbong and Raul M. Gonzales, both members of the Bar, taxpayers
requirements in gathering the signatures. The proponents bear the burden and interested in running as candidates for delegates to the
gathering the signatures – that the petition contained, or incorporated by Constitutional Convention. Both impugn the constitutionality of R.A.
attachment, the full text of the proposed amendments. No. 6132, claiming during the oral argument that it prejudices their
The Lambino Group did not attach to their present petition with this Court a rights as such candidates.
copy of the paper that the people signed as their initiative petition. The On March 16, 1967, Congress, acting as a Constituent Assembly
Lambino Group submitted to this Court a copy of a signature sheet after the pursuant to Art. XV of the Constitution, passed Resolution No. 2 which
oral arguments of 26 September 2006 when they filed their Memorandum on among others called for a Constitutional Convention to propose
11 October 2006. constitutional amendments to be composed of two delegates from
each representative district who shall have the same qualifications as
1. The Two-Part Test
those of Congressmen, to be elected on the second Tuesday of
In determining whether the Lambino proposal involves an amendment or November, 1970 in accordance with the Revised Election Code. On
a revision, the Court considered the two-part test. First, the quantitative June 17, 1969, Congress, also acting as a Constituent Assembly,
test asks whether the proposed change is so extensive in its provisions as passed Resolution No. 4 amending the aforesaid Resolution No. 2 of
to change directly the “substance entirety” of the Constitution by the March 16, 1967 by providing that the convention “shall be composed
deletion or alteration of numerous provisions. The court examines only the of 320 delegates apportioned among the existing representative
number of provisions affected and does not consider the degree of the districts according to the number of their respective inhabitants:
change. Second, the qualitative test, which inquires into the qualitative Provided, that a representative district shall be entitled to at least two
effects of the proposed change in the Constitution. The main inquiry is delegates, who shall have the same qualifications as those required
whether the change will “accomplish such far-reaching changes in the of members of the House of Representatives,” 1 “and that any other
nature of our basic governmental plan as to amount to a revision”. details relating to the specific apportionment of delegates, election of
delegates to, and the holding of, the Constitutional Convention shall
2. Provisions that needed to be changed to effect change from be embodied in an implementing legislation: Provided, that it shall not
unitary to federal form of government, and from presidential to be inconsistent with the provisions of this Resolution.”
parliamentary form of government
ISSUE: Whether RA 6132 was constitutional because it had to do with
The Lambino Group’s initiative petition changes the 1987 Constitution by the calling of a Constitutional Convention but was not passed by 2/3
modifying Sections 1-7 of Article VI (Legislative Department) and Sections of all the members of the Senate and House of Representatives,
1-4 of Article VII (Executive Department) and by adding Article XVIII voting separately.
entitled “Transitory Provisions.”
RULING: The Supreme Court upheld the validity of the law, declaring
b. Procedure that after Congress had exercised its constituent power by adopting
RBH 2 and RBH 4, with the requisite 2/3 vote as required by the 1935
i. Proposal
Constitution, it may, by simply exercising legislative power, pass a
1. By Congress law providing for the details for the implementation of the resolutions
passed in the exercise of its constituent power.
By a vote of 3/4 of all its members. Majority of authorities opine that this
is to be understood as 3/4 of the Senate and 3/4 of the House of 3. By People’s Initiative; Requirements and limitations
Representatives.
Initiative: The power of the people to propose amendments to the
2. By a Constitutional Convention Constitution or to propose and enact a legislation through an election
called for the purpose. There are three systems of initiative, namely:
Which may be called into existence either by a 3/4 vote of all the members initiative on the Constitution which refers to a petition proposing
of Congress, or (if such vote is not obtained) by a majority vote of all the amendments to the Constitution; initiative on statutes which refers

M.R.A.D.C. LUMBRE 3
CONSTITUTIONAL LAW REVIEW

to a petition proposing to enact a national legislation; and initiative Constitution through initiative must be more rigorous and difficult
on local legislation which refers to a petition proposing to enact a than the initiative on legislation.
regional, provincial, city, municipal or bararigay law, resolution or
ordinance [Sec. 2(a), R.A. 6735]. Indirect Initiative is exercise of RA 6735 is Inadequate to cover the System of Initiative on the
initiative by the people through a proposition sent to Congress or the Constitution.
local legislative body for action [Sec. 2(b) R.A. 6735].
RA 6735 was, as its history reveals, intended to cover initiative
Requirements: A petition of at least 12% of the total number of to propose amendments to the Constitution. However, RA 6735
registered voters, of which every legislative district must be is incomplete, inadequate, or wanting in essential terms and
represented by at least 3% of the registered voters therein. conditions insofar as initiative on amendments to the Constitution
Limitation: No amendment in this manner shall be authorized within is concerned.
five years following the ratification of this Constitution nor more often Although Section 3 (Definition of Terms) of RA 6753 defines
than once every five years thereafter.
initiative on amendments to the Constitution, it does not provide
a. Defensor-Santiago vs. COMELEC, GR No. 127325, 19 for the contents of a petition for initiative on the Constitution.
March 1997 Other provisions of the Act cannot be made to apply to fill in the
gap since these provisions expressly refer to “proposed laws
FACTS: Under the 1987 Constitution, in Section 2 of Article XVII,
sought to be enacted, approved or rejected, amended or
the system of initiative was introduced through which the people
repealed.”
may directly propose amendments to the Constitution. In 1996,
Atty. Jesus Delfin filed with COMELEC a "Petition to Amend the While RA 6735 provides separate Subtitles for initiative and
Constitution, to Lift Term Limits of Elective Officials, by People's referendum on laws and ordinances, no subtitle is provided for
Initiative" (Delfin Petition). initiative on the Constitution. This omission means that the main
thrust of RA 6753 is initiative and referendum on national and
Senator Miriam Defensor Santiago and others filed an action for
local laws. If Congress intended RA 6735 to fully provide for the
prohibition raising the argument that: (1) The constitutional
implementation of the initiative on amendments to the
provision on people's initiative to amend the Constitution can only
Constitution, it could have provided for a subtitle therefor.
be implemented by law and no such law has been passed. While
Republic Act 6735 provides for three systems of initiative (on the This deliberate omission indicates that the matter of people's
Constitution, on statutes, and on local legislation), however, said initiative to amend the Constitution was left to some separate
law is inadequate with respect to a system of initiative on the and future law.
Constitution. (2) The people's initiative is limited to amendments
b. Resolution on the MR filed in Lambino vs. COMELEC
to the Constitution, not to revision thereof. Extending or lifting of
dated November 21, 2006
term limits constitutes a revision and is, therefore, outside the
power of the people's initiative. The Court held that R.A. No. 6735 is sufficient.

RULING: The Delfin petition was not granted. The system of ii. Ratification (Sec. 4, Art. XVII)
initiative on the Constitution under Section 2 of Article XVII of
The proposed amendment shall become part of the Constitution when
the Constitution is not self-executing. ratified by a majority of the votes cast in a plebiscite held not earlier
While the Constitution has recognized or granted the right of the than 60 nor later than 90 days after the approval of the proposal by
people to directly propose amendments to the Constitution Congress or the Constitutional Convention, or after the certification
by the Commission on Elections of the sufficiency of the petition for
through the system of initiative, the people cannot exercise it if
initiative under Sec. 2, Art. XVII.
Congress, for whatever reason, does not provide for its
implementation by way of a law or statute. 1. Doctrine of Proper Submission

Based on the interpellations of the 1986 Constitutional It is the condition and limitation that all the amendments to be
Commission which drafted the 1987 Constitution, initiative can proposed by the same Convention must be submitted to the people
only relate to "amendments" not "revision” of the Constitution. in a single “election” or plebiscite.
Moreover, the process of proposing amendments to the a. Tolentino vs. COMELEC, 41 SCRA 702

M.R.A.D.C. LUMBRE 4
CONSTITUTIONAL LAW REVIEW

FACTS: The herein petitioner, Arturo Tolentino, filed the instant (3) Whether the people had acquiesced in the new Constitution (with or without
petition, to prohibit the respondents from implementing the Organic valid ratification) - 4 justices said the people had already accepted the new
Resolution No. 1 and the other resolutions passed by the Convention. Constitution, 2 said that there can be no free expression by the people qualified
He argued that the said resolutions were null and void on the ground to vote of their acceptance or repudiation of the proposed Constitution under
that the calling and holding of plebiscite is lodged exclusively to the martial law, one said he is not prepared to state that a new Constitution once
Congress. Moreover, under Section 1 of Article XV of the Constitution, accepted by the people must be accorded recognition independently of valid
the proposed amendment cannot be presented to the people for ratification, and three expressed their lack of knowledge or competence to rule
ratification separately from each and all of the amendments to be on the question because under a regime of martial law with the free expression
drafted and proposed by the Constitution. of opinions restricted, they have no means of knowing, to the point of judicial
certainty, whether the people have accepted the Constitution. (4) Whether the
ISSUE: Is there any limitation or condition in Section 1 of Article XV petitioners are entitled to relief - 6 justices voted to dismiss the petitions, while
of the Constitution which is violated by the act of the Convention for 4 were for giving due course to the petitions.
the plebiscite on the sole amendment contained in Organic Resolution
1? (5) Whether the new Constitution is already in force - 4 said yes by virtue of
the people’s acceptance of the same, 4 said they could not with judicial
RULING: Yes. It is the condition and limitation that all amendments certainty whether or not the people had accepted the Constitution, and 2
to be proposed by the same Convention must be submitted to the declared that the new Constitution is not in force, “with the result that there
people in a single “election” or plebiscite. This is the doctrine of are not enough votes to declare tha the new Constitution is not in force”.
proper submission. The Court said that the Constitution
unequivocably says “an election,” which means only one and thus, The SC decision concluded: “Accordingly, by virtue of the majority 6 votes x x
leaving no room for doubt on how many “elections” or plebiscites may x. with 4 dissenting votes x x x all of the aforementioned cases are hereby
be held by the Convention. dismissed. This being the vote of the majority, there is no further judicial
obstacle to the new Constitution being considered in force and effect. ”
NOTE: Piecemeal ratification of amendment is not allowed.
c. Judicial review of amendments

The question is now regarded as subject to judicial review, because


invariably, the issue will boil down to whether or not the constitutional
provisions had been followed. (Javellana v. Executive Secretary) That duty is
part of the judicial power vested in the courts by an express grant under Sec.
1, Art. VIII of the Constitution which states: “Judicial power includes the duty
of the courts of justice to settle actual controversies involving rights which are
legally demandable and enforceable, and to determine whether or not there
has been a grave abuse of discretion amounting to lack or excess of jurisdiction
on the part of any branch or instrumentality of Government.”
i. Javellana vs. Executive Secretary, 50 SCRA 30

The validity of the ratification of the 1973 Constitution was challenged in


Javellana v. Executive Secretary, 50 SCRA 30, and companion cases
(collectively known as the Ratification Cases).
The basic issues and the votes of the SC justices were:
(1) Whether the validity of Proclamation 1102 is a political or a justiciable
question - 6 justices said it is justiciable, 3 said it is political, and 1 justice
qualified his vote.

(2) Whether the new Constitution was validly ratified (with substantial if not
strict compliance) conformably with the 1935 Constitution - 6 justices said no,
3 said there was substantial compliance, and 1 qualified his vote.

M.R.A.D.C. LUMBRE 5
CONSTITUTIONAL LAW REVIEW

II. The Philippines as a State the immunity of the sovereign is recognized only with regard to public acts or
acts jure imperii of a state, but not with regard to private acts or acts jure
gestionis. If the act is in pursuit of a sovereign activity, or an incident thereof,
1. National Territory (Art. I) then it is an act jure imperii, especially when it is not undertaken for gain or
profit.
The national territory comprises the Philippine archipelago, with all the islands and
waters embraced therein, and all other territories over which the Philippines has In the case at bench, if petitioner has bought and sold lands in the ordinary
sovereignty or jurisdiction, consisting of its terrestrial, fluvial and aerial domains, course of a real estate business, surely the said transaction can be categorized
including its territorial sea, the seabed, the subsoil, the insular shelves, and other as an act jure gestionis. However, petitioner has denied that the
submarine areas. The waters around, between, and connecting the islands of the acquisition and subsequent disposal of Lot 5-A were made for profit
archipelago, regardless of their breadth and dimensions, form part of the internal waters but claimed that it acquired said property for the site of its mission or
of the Philippines. the Apostolic Nunciature in the Philippines. Private respondent failed
to dispute said claim. Lot 5-A was acquired by petitioner as a donation from
a. Territorial waters
the Archdiocese of Manila. The donation was made not for commercial purpose,
A belt of coastal waters extending at most 12 nautical miles (22.2 km; 13.8 mi) but for the use of petitioner to construct thereon the official place of residence
from the baseline (usually the mean low-water mark) of a coastal state. The of the Papal Nuncio. The right of a foreign sovereign to acquire property, real
territorial sea is regarded as the sovereign territory of the state, although foreign or personal, in a receiving state, necessary for the creation and maintenance
ships (civilian) are allowed innocent passage through it, or transit passage for of its diplomatic mission, is recognized in the 1961 Vienna Convention on
straits; this sovereignty also extends to the airspace over and seabed below. Diplomatic Relations (Arts. 20-22). This treaty was concurred in by the
Adjustment of these boundaries is called, in international law, maritime Philippine Senate and entered into force in the Philippines on November 15,
delimitation. (Art. III, UNCLOS III) 1965.
b. Archipelagic Doctrine
ii. Liang vs. People, GR No. 125865, 28 January 2000
“The waters around, between, and connecting the islands of the archipelago,
FACTS: Petitioner is an economist working with the Asian Development Bank
regardless of their breadth and dimensions, form part of the internal waters of the
(ADB). He was charge for allegedly uttering defamatory words against fellow
Philippines.” Based on the principle that an archipelago, which consists of a number
ADB worker before the MeTC of Mandaluyong. He was arrested and
of islands separated by bodies of water, should be treated as one integral unit.
subsequently released in jail after fixing bail. The next day, the MeTC judge
received an “office of protocol” from the DFA stating that petitioner is covered
by immunity from legal process under Section 45 of the Agreement between
2. Doctrine of State Immunity (Art. XVI, Section 3) – “The State cannot be sued the ADB and the Philippine Government regarding the Headquarters of the ADB
without its consent.” in the country. Based on the said protocol communication that petitioner is
NOTE: It is originally an international law doctrine, which, in relation to Art. II, Sec. 2 immune from suit, the MeTC judge without notice to the prosecution dismissed
of the 1987 Constitution, is deemed incorporated into the law of our land. (USA v. the two criminal cases.
Guinto)
RULING: The petitioner is not immune from legal process. The DFA’s
a. Entitlement of immunity, justiciable or political question? determination that a certain person is covered by immunity is only preliminary
which has no binding effect in courts. In receiving ex-parte the DFA’s advice
It is a political question. and in motu proprio dismissing the two criminal cases without notice to the
i. The Holy See v. Rosario, 238 SCRA 524, Dec. 1, 1994 prosecution, the latter’s right to due process was violated. It should be noted
that due process is a right of the accused as much as it is of the prosecution.
FACTS: Petitioner in its petition invokes the privilege of sovereign immunity The needed inquiry in what capacity petitioner was acting at the time of the
and denied that the acquisition and its subsequent disposal were made for alleged utterances requires for its resolution evidentiary basis that has yet to
profit but for its Apostolic mission in the Philippines. be presented at the proper time. At any rate, it has been ruled that the mere
invocation of the immunity clause does not ipso facto result in the dropping of
RULING: There are two conflicting concepts of sovereign immunity, each the charges.
widely held and firmly established. According to the classical or absolute
theory, a sovereign cannot, without its consent, be made a respondent in the NOTE: In the case of Holy See v. Rosario, the issue was the immunity of State,
courts of another sovereign. According to the newer or restrictive theory, whereas in Liang v. People, the issue was the immunity of an individual.

M.R.A.D.C. LUMBRE 6
CONSTITUTIONAL LAW REVIEW

b. When a suit is against a state and when it is not c. Immunity of International Organizations and Agencies
i. Arigo vs. Swift, G.R. No. 206501, 16 September 2014 i. SEAFDEC vs. NLRC, 241 SCRA 580

FACTS: On January 15, 2013, the USS Guardian departed Subic Bay for FACTS: Two labor cases were filed by the herein private respondents
its next port of call in Makassar, Indonesia. On January 17, 2013 at 2:20 against the petitioner, Southeast Asian Fisheries Development Center
a.m. while transiting the Sulu Sea, the ship ran aground on the northwest (SEAFDEC), before the National Labor Relations Commission (NLRC),
side of South Shoal of the Tubbataha Reefs, about 80 miles east-southeast Regional Arbitration Branch, Iloilo City. In these cases, the private
of Palawan. No cine was injured in the incident, and there have been no respondents claim having been wrongfully terminated from their
reports of leaking fuel or oil. employment by the petitioner.
ISSUE: Whether the case filed against the respondent is a case against
RULING: Petitioner is immune from suit. It is beyond question that
the State.
petitioner SEAFDEC is an international agency enjoying diplomatic
RULING: As applied to the local state, the doctrine of state immunity immunity. It has already been held in Southeast Asian Fisheries
is based on the justification given by Justice Holmes that "there can be Development Center-Aquaculture Department vs. National Labor
no legal right against the authority which makes the law on which the Relations Commission (G.R. No. 86773, 206 SCRA 283/1992). Petitioner
right depends." [Kawanakoa v. Polybank, 205 U.S. 349] There are Southeast Asian Fisheries Development Center-Aquaculture Department
other practical reasons for the enforcement of the doctrine. In the case (SEAFDEC-AQD) is an international agency beyond the jurisdiction of
of the foreign state sought to be impleaded in the local jurisdiction,
public respondent NLRC.
the added inhibition is expressed in the maxim par in parem, non
habet imperium. All states are sovereign equals and cannot ii. Callado vs. IRRI, 244 SCRA 210
assert jurisdiction over one another. A contrary disposition would,
in the language of a celebrated case, "unduly vex the peace of ISSUE: May the immunity of the IRRI as an international organization be
nations." [De Haber v. Queen of Portugal, 17 Q. B. 171] invoked in this case inasmuch as it waived the same by virtue of its
Memorandum on "Guidelines on the handling of dismissed employees in
While the doctrine appears to prohibit only suits against the state without relation to P.D. 1620."?
its consent, it is also applicable to complaints filed against officials
of the state for acts allegedly performed by them in the discharge RULING: Yes. IRRI's immunity from suit is undisputed. Presidential
of their duties. The rule is that if the judgment against such officials will Decree No. 1620, Article 3 provides:
require the state itself to perform an affirmative act to satisfy the same, Art. 3. Immunity from Legal Process. The Institute shall enjoy immunity
such as the appropriation of the amount needed to pay the damages from any penal, civil and administrative proceedings, except insofar as
awarded against them, the suit must be regarded as against the state that immunity has been expressly waived by the Director-General
itself although it has not been formally impleaded. [Garcia v. Chief of Staff, xxxxx
16 SCRA 120] In such a situation, the state may move to dismiss the
complaint on the ground that it has been filed without its consent. (NOTE: Petitioner’s reliance on the Memorandum with "Guidelines in handling
Create box here.) cases of dismissal of employees in relation to P.D. 1620" dated July 26,
1983, is misplaced. The Memorandum reads, in part:
The US respondents in this case were sued in their official capacity as
commanding officers of the US Navy who had control and supervision over “If the plaintiff's attorney or the arbiter, asks if IRRI will waive its
the USS Guardian and its crew. The alleged act or omission resulting in immunity we may reply that the Institute will be happy to do so,
the unfortunate grounding of the USS Guardian on the TRNP was as it has in the past in the formal manner required thereby reaffirming our
committed while they were performing official military duties. Considering commitment to abide by the laws of the Philippines and our full faith in
that the satisfaction of a judgment against said officials will require the integrity and impartially of the legal system.”
remedial actions and appropriation of funds by the US government,
From the last paragraph of the foregoing quotation, it is clear that in
the suit is deemed to be one against the US itself. The principle of State
cases involving dismissed employees, the Institute may waive its
immunity therefore bars the exercise of jurisdiction by this Court over the
immunity, signifying that such waiver is discretionary on its part.
persons of respondents Swift, Rice and Robling.
We agree with private respondent IRRI that this memorandum cannot,
by any stretch of the imagination, be considered the express waiver
by the Director-General. May" rather than the mandatory term "shall" in
the last paragraph of the memo indicates discretion.

M.R.A.D.C. LUMBRE 7
CONSTITUTIONAL LAW REVIEW

iii. DFA vs. NLRC, 18 September 1996 1. Governmental function – Rule: No suit without its consent.
ISSUE: Whether by entering into service contracts with different private a. Farolan vs. CTA, 217 SCRA 298
companies, ADB has descended to the level of an ordinary party to a
ISSUE: Whether the Collector of Customs may be held liable for
commercial transaction giving rise to a waiver of its immunity from suit.
the bales of fabric lost by the private respondent.
RULING: Under the Charter and Headquarters Agreement, the ADB RULING: No. The Supreme Court said that the Bureau of
enjoys immunity from legal process of every form, except in the specified Customs, being an unincorporated agency without a separate
cases of borrowing and guarantee operations, as well as the purchase, juridical personality, enjoys immunity from suit. It is invested
sale and underwriting of securities. The Bank’s officers, on their part, with an inherent power of sovereignty, namely the power of
enjoy immunity in respect of all acts performed by them in their official taxation; it performs governmental functions.
capacity. The Charter and the Headquarters Agreement granting these
2. Proprietary function – Rule: Suit will lie against the agency.
immunities and privileges are treaty covenants and commitments
voluntarily assumed by the Philippine government which must be a. China National Machinery and Equipment Corp. (Group)
respected. vs. Judge Santamaria, GR No. 185572, 7 February 2012

The logical question is whether the foreign state is engaged in the activity ISSUE: Whether CNMEG is entitled to immunity, as it was an
in the regular course of business. If the foreign state is not engaged agent of the Chinese government, precluding it from being sued
regularly in a business or trade, the particular act or transaction must then before a local court.
be tested by its nature. If the act is in pursuit of a sovereign activity, or RULING: No. CNMEG is engaged in a proprietary activity. While
an incident thereof, then it is an act jure imperii, especially when it is not the Loan Agreement entered into between EXIM Bank and the
undertaken for gain or profit. Philippine government, the Contract Agreement was between
Northrail and CNMEG. Although the Contract Agreement was
d. Immunity of government agencies
silent on the classification of the legal nature of the transaction,
i. Incorporated (has an original charter) the foregoing provisions of the Loan Agreement, which is an
inextricable part of the entire undertaking, nonetheless reveal the
If the charter provides that the agency can sue and be sued, then the suit intention of the parties to the Northrail Project to classify the
will lie, including one for tort. The provision in the charter constitutes whole venture as commercial or proprietorial in character.
express consent on the part of the State to be sued.
Moreover, although CNMEG claims to be a government-owned
NOTE: If the charter is silent, apply the rule on unincorporated agencies corporation, it failed to adduce evidence that is has not consented
(whether governmental or proprietary). to be sued under Chinese law. Thus, following the Court’s ruling
in Deutsche Gesellscharft, in the absence of evidence to the
1. Fontanilla vs. Maliaman, 194 SCRA 486
contrary, CNMEG is presumed to be a government-owned and
ISSUE: Whether NIA, a government agency, may be held for controlled corporation without an original charter. As a result, it
damages caused by the negligent act by its driver who was not its has the capacity to sue and be sued under Sec. 36 of the
special agent. Corporation Code. (NOTE: The case is unique because of this.)

RULING: Yes. NIA is a government agency with a juridical CNMEG also failed to present a certification from the DFA, whose
personality and not a mere agency of the government. Since it is a authority to determine immunity from suit is exclusive to such
corporate body performing non-governmental functions. Therefore, it agency.
may be held liable for the damages caused by the accident resulting An agreement to submit any dispute to arbitration may also be
from the tortious act of its driver-employee. In this case, NIA assumes construed as an implicit waiver of immunity from suit.
the responsibility of an ordinary employer, and as such, it becomes
answerable for damages, under paragraph 5 of Article 2180. b. Civil Aeronautics Administration vs. CA, GR No. L-51806,
8 November 1968
ii. Unincorporated
ISSUE: Whether the suit against CAA is really a suit against the
An unincorporated agency has no juridical personality independent of the Republic, which cannot be sued without its consent, which was
government. To determine its suability, one has to inquire into the not given in this case.
principal functions of the agency.

M.R.A.D.C. LUMBRE 8
CONSTITUTIONAL LAW REVIEW

RULING: No. From Republic Act 776, Sec. 32 (24) and (25), it General Hospital was not a special agent; thus the Government is not
can be seen that the CAA is tasked with private or non- liable.
governmental functions which operate to remove it from the
f. Waiver of state immunity
purview of the rule on State immunity from suit.
NOTE: Waiver can only be given by the Legislature. If ever there is a waiver,
In the recent case of Malong v. Philippine National Railways it was
the same must be strictly interpreted or strictissimis juris.
held that the Philippine National Railways, although owned and
operated by the government, was not immune from suit as it i. Express consent (Suit against the Philippine government)
does not exercise sovereign but purely proprietary and business
Read: Republic vs. Feliciano, 148 SCRA 424
functions. Accordingly, as the CAA was created to undertake the
management of airport operations which primarily involve FACTS: On January 22, 1970, Feliciano filed a complaint with CFI of
proprietary functions, it cannot avail of the immunity from suit Camarines Sur against the Republic of the Philippines, represented by the
accorded to government agencies performing strictly Land Authority, for the recovery of ownership and possession of a parcel
governmental functions. of land, consisting of four (4) lots situated in the Barrio of Salvacion,
Municipality of Tinambac, Camarines Sur. He alleged that he bought the
e. Suability not outright liability property in question from Victor Gardiola by virtue of a Contract of Sale
dated May 31, 1952, followed by a Deed of Absolute Sale on October 30,
i. Meritt vs. Gov’t. Of the Phil. Islands, 34 Phil 311, No. 11154, 21 1954; that Gardiola had acquired the property by purchase from the heirs
March 1916) of Francisco Abrazado whose title to the said property was evidenced by
FACTS: Merrit was riding a motorcycle along Padre Faura Street when he an informacion posesoria that upon plaintiff's purchase of the property, he
was bumped by the ambulance of the General Hospital. Merrit sustained took actual possession of the same, introduced various improvements
therein and caused it to be surveyed in July 1952, which survey was
severe injuries rendering him unable to return to work. The legislature
approved by the Director of Lands on October 24, 1954.
later enacted Act 2457 authorizing Merritt to file a suit against the
Government in order to fix the responsibility for the collision between his ISSUE: Whether the State can be sued for recovery and possession of a
motorcycle and the ambulance of the General Hospital, and to determine parcel of land.
the amount of the damages, if any, to which he is entitled. After trial, the
RULING: The doctrine of non-suability of the State has proper application
lower court held that the collision was due to the negligence of the driver
in this case. The plaintiff has impleaded the Republic of the Philippines as
of the ambulance. It then determined the amount of damages and ordered
defendant in an action for recovery of ownership and possession of a
the government to pay the same.
parcel of land, bringing the State to court just like any private person who
ISSUES: Whether the government, in enacting Act 2457, waived its is claimed to be usurping a piece of property. A suit for the recovery of
immunity from suit, as well as conceding its liability to plaintiff? property is not an action in rem, but an action in personam. It is an action
directed against a specific party or parties, and any judgment therein
RULING: By consenting to be sued a state simply waives its immunity
binds only such party or parties. The complaint filed by plaintiff, the
from suit. It does not thereby concede its liability to plaintiff, or create any
private respondent herein, is directed against the Republic of the
cause of action in his favor, or extend its liability to any cause not
Philippines, represented by the Land Authority, a governmental agency
previously recognized. It merely gives a remedy to enforce a preexisting
created by Republic Act No. 3844.
liability and submits itself to the jurisdiction of the court, subject to its
right to interpose any lawful defense. The failure of the petitioner to assert the defense of immunity from suit
when the case was tried before the court a quo, as alleged by private
Under the Civil Code, the state is liable when it acts through a special
respondent, is not fatal. It is now settled that such defense "may be
agent, but not when the damage should have been caused by the official
invoked by the courts suasponte at any stage of the proceedings."
to whom properly it pertained to do the act performed. A special agent is
one who receives a definite and fixed order or commission, foreign to the Private respondent contends that the consent of petitioner may be read
exercise of the duties of his office if he is a special official. This concept from the Proclamation itself, when it established the reservation "subject
does not apply to any executive agent who is an employee of the acting to private rights, if any there be." We do not agree. No such consent can
administration and who on his own responsibility performs the functions be drawn from the language of the Proclamation. The exclusion of existing
which are inherent in and naturally pertain to his office and which are private rights from the reservation established by Proclamation No. 90
regulated by law and the regulations. The driver of the ambulance of the cannot be construed as a waiver of the immunity of the State from suit.

M.R.A.D.C. LUMBRE 9
CONSTITUTIONAL LAW REVIEW

Waiver of immunity, being a derogation of sovereignty, will not be decision of COA is reviewable by SC on the ground of grave abuse
inferred lightly, but must be construed in strictissimi of discretion, likewise if there is no COA decision within 30 days,
juris. Moreover, the Proclamation is not a legislative act. The file certiorari within 30 days; (4) If SC grants certiorari against
consent of the State to be sued must emanate from statutory COA, go to DBM; if disapproved, file mandamus;
authority. Waiver of State immunity can only be made by an act of c. UP vs. Dizon, GR No. 171182, August 23, 2012
the legislative body.
ISSUE: Whether UP’s funds may be validly garnished for failure
The inscription in the property registry of an informacion posesoria under to pay respondent for the construction and renovation of the
the Spanish Mortgage Law was a means provided by the law then in force former’s buildings.
in the Philippines prior to the transfer of sovereignty from Spain to the
RULING: No. UP's funds, being government funds, are not
United States of America, to record a claimant's actual possession of a
subject to garnishment. Despite its establishment as a body
piece of land, established through an ex parte proceeding conducted in
corporate, the UP remains to be a "chartered institution"
accordance with prescribed rules. Such inscription merely furnishes, at
performing a legitimate government function. Irrefragably, the
best, prima facie evidence of the fact that at the time the proceeding was
UP is a government instrumentality, performing the State’s
held, the claimant was in possession of the land under a claim of right as
constitutional mandate of promoting quality and accessible
set forth in his application. The possessory information could ripen into a
education. As a government instrumentality, the UP administers
record of ownership after the lapse of 20 years (later reduced to 10 years),
special funds sourced from the fees and income enumerated
upon the fulfillment of the requisites prescribed in Article 393 of the
under Act No. 1870 and Section 1 of Executive Order No. 714,
Spanish Mortgage Law.
and from the yearly appropriations, to achieve the purposes laid
There is no showing in the case at bar that the informacion posesoria held down by Section 2 of Act 1870, as expanded in Republic Act No.
by the respondent had been converted into a record of ownership. Such 9500. All the funds going into the possession of the UP, including
possessory information, therefore, remained at best mere prima any interest accruing from the deposit of such funds in any
facie evidence of possession. banking institution, constitute a "special trust fund," the
disbursement of which should always be aligned with the UPs
1. General law
mission and purpose, and should always be subject to auditing
a. Act No. 3083 in relation with CA 327, as amended by by the COA. The funds of the UP are government funds that are
Secs. 49-50, PD 1445 public in character. They include the income accruing from the
use of real property ceded to the UP that may be spent only for
Requires that all money claims against the government must first
the attainment of its institutional objectives.
be filed with the Commission on Audit (COA) before suit is
instituted in court. The Department of Agriculture may be sued A marked distinction exists between suability of the State and its
for money claims based on a contract entered into in its liability. As the Court succinctly stated in Municipality of San
governmental capacity, because of the express consent Fernando, La Union v. Firme: A distinction should first be made
contained in Act No. 3038, provided that the claim be first
between suability and liability. "Suability depends on the consent
brought to the Commission on Audit in accordance with CA 327,
of the state to be sued, liability on the applicable law and the
as amended.
established facts. The circumstance that a state is suable does
b. Procedure to collect money claims arising from not necessarily mean that it is liable; on the other hand, it can
contracts never be held liable if it does not first consent to be sued. Liability
NOTE: (1) Seek claim from COA; is not conceded by the mere fact that the state has allowed itself
to be sued. When the state does waive its sovereign immunity, it
(2) COA will submit claim to DBM; is only giving the plaintiff the chance to prove, if it can, that the
(3) DBM will ask Congress to include claim in its GAA for the defendant is liable.
following year; The Constitution strictly mandated that, "No money shall be paid
(4) Congress will include claim in its GAA. out of the Treasury except in pursuance of an appropriation made
by law." The execution of the monetary judgment against the UP
Remedies: (1) If Congress will not give due course, file was within the primary jurisdiction of the COA, pursuant to PD
mandamus; (2) If not approved by the COA, certiorari to SC as

M.R.A.D.C. LUMBRE 10
CONSTITUTIONAL LAW REVIEW

1445. It was of no moment that a final and executory decision failure of the Mayor, the Municipal Board, or any other city officer,
already validated the claim against the UP. The settlement of the to enforce the provisions of this chapter, or any other law or
monetary claim is still subject to the primary jurisdiction of the ordinance, or from negligence of said Mayor, Municipal Board, or
COA; as such, claimants have no alternative except to first seek other officers while enforcing or attempting to enforce said
the approval of their monetary claim by COA. provisions.

2. Special Law RULING: Article 2189 applies. Section 4 of the said Act refers to
liability arising from negligence, in general, regardless of the
(NOTE: Remedy is to go to regular courts.)
object thereof, whereas Article 2189 governs liability due to
a. Arts. 2180 and 2189 of the new Civil Code "defective streets," in particular. Since the present action is
based upon the alleged defective condition of a road, said Article
Article 2180. The obligation imposed by article 2176 is
2189 is decisive thereon.
demandable not only for one's own acts or omissions, but also
for those of persons for whom one is responsible. At any rate, under Article 2189 of the Civil Code, it is not
xxxx necessary for the liability therein established to attach that the
defective roads or streets belong to the province, city or
The State is responsible in like manner when it acts through a municipality from which responsibility is exacted. What said
special agent; but not when the damage has been caused by the article requires is that the province, city or municipality have
official to whom the task done properly pertains, in which case either "control or supervision" over said street or road. Even if P.
what is provided in article 2176 shall be applicable.
Burgos Avenue were, therefore, a national highway, this
Article 2189. Provinces, cities and municipalities shall be liable circumstance would not necessarily detract from its "control or
for damages for the death of, or injuries suffered by, any person supervision" by the City of Manila, under Republic Act 409. Said
by reason of the defective condition of roads, streets, bridges, Act governs the disposition or appropriation of the highway funds
public buildings, and other public works under their control or and the giving of aid to provinces, chartered cities and
supervision. municipalities in the construction of roads and streets within their
i. Manila vs. Teotico, G.R. No. L-23052, 29 January 1968 respective boundaries.

FACTS: January 27 1958, 8:00pm: Genaro Teotico was at the The determination of whether or not P. Burgos Avenue is
corner of Old Luneta and P. Burgos Ave., Manila (in a loading and under the control or supervision of the City of Manila and
unloading zone) waiting for a jeepney. He hailed a jeep and as whether the latter is guilty of negligence, in connection
he stepped down the curb to board the jeepney, he fell inside an with the maintenance of said road, which were decided by
uncovered and unlighted catch basin or manhole on P. Burgos the Court of Appeals in the affirmative, is one of fact, and
Avenue. Due to the fall, his head hit the rim of the manhole the findings of said Court thereon are not subject to our
breaking his eyeglasses and causing broken pieces thereof to review.
pierce his left eyelid. As blood flowed therefrom, impairing his
NOTE: In relation to Republic v. Feliciano, the waiver must be
vision, several persons came to his assistance and pulled him out
express.
of the manhole. One of them brought Teotico to the Philippine
General Hospital, where his injuries were treated, after which he b. How to claim?
was taken home. In addition to the lacerated wound in his left
Go to regular courts.
upper eyelid, Teotico suffered contusions on the left thigh, the
left upper arm, the right leg and the upper lip apart from an ii. Implied consent
abrasion on the right infra-patella region.
1. By entering into a business contract (Suit against foreign
ISSUE: Whether the present case is governed by Section 4 of government)
Republic Act No. 409 (Charter of the City of Manila) OR Article NOTE: Why suit against foreign government only? Act 3083, by
2189 of the Civil Code. which money claims based on contract may be filed, is a form of
express consent by the State.
(Sec. 4 of RA 409): The city shall not be liable or held for
damages or injuries to persons or property arising from the

M.R.A.D.C. LUMBRE 11
CONSTITUTIONAL LAW REVIEW

a. Restrictive Doctrine of State Immunity – Such execution will require another waiver, because the power of the court
ends when the judgment is rendered, since government funds and properties
See Holy See v. Rosario. may not be seized under writs of execution or garnishment, unless such
b. USA vs. Ruiz, 136 SCRA 487 disbursement is covered by the corresponding appropriation as required by
law. (NOTE: "No money shall be paid out of the Treasury except in pursuance
ISSUE: Whether the US Naval Base in bidding for said contracts of an appropriation made by law.")
exercise governmental functions so as to be able to invoke state
immunity from suit. i. Municipality of San Miguel vs. Fernandez, 130 SCRA 56

RULING: Yes. The correct test for the application of State ISSUE: Whether the funds of the petitioner Municipality of San Miguel in
immunity is not the conclusion of a contract by a State but the the hands of the provincial and municipal treasurers of Bulacan and San
legal nature of the act. The contract for the repair of wharves was Miguel, respectively, are public funds which are exempt from execution for
a contract in jure imperii, because the wharves were to be used the satisfaction of the money judgment for payment of unpaid rentals.
in national defense, which is a governmental function. RULING: The court held that properties of municipal corporations are
2. By commencing a suit (Suit against private individual) exempt as they are held in trust for the people, intended and used for the
accomplishment of the purposes for which municipal corporations are
a. Froilan vs. Pan Oriental Shipping and Republic of the created (although it is an incorporated agency whose charter provides that
Philippines, GR No. L-6060, 30 September 1950
it can sue and be sued). Also, Presidential Decree No. 477: "No money
ISSUE: Whether the lower court erred in dismissing the shall be paid out of the treasury except in pursuance of a lawful
counterclaim on the ground of alleged lack of jurisdiction over the appropriation or other specific statutory authority". As such, there must
intervenor Republic of the Philippines. be a corresponding appropriation in the form of an ordinance duly passed
by the Sangguniang Bayan before any money of the municipality may be
RULING: The other ground for dismissing the defendant's
counterclaim is that the State is immune from suit. This is paid out. In the case at bar, it has not been shown that the Sangguniang
untenable, because by filing its complaint in intervention the Bayan has passed an ordinance to this effect.
Government in effect waived its right of non-suability. ii. Municipality of Makati vs. CA, 190 SCRA 206
The immunity of the state from suits does not deprive it of the ISSUE: Whether the funds of the Municipality of Makati is exempt from
right to sue private parties in its own courts. The state as plaintiff garnishment and levy upon execution?
may avail itself of the different forms of actions open to private
litigants. In short, by taking the initiative in an action against a RULING: Yes. Public funds are not subject to levy and execution, unless
private party, the state surrenders its privileged position and otherwise provided for by statute. Municipal revenues derived from taxes,
comes down to the level of the defendant. The latter licenses and market fees, and which are intended primarily and exclusively
automatically acquires, within certain limits, the right to set up for the purpose of financing the governmental activities and functions of
whatever claims and other defenses he might have against the the municipality, are exempt from execution.
state. The United States Supreme Court thus explains:
Absent a showing that the municipal council of Makati has passed an
"No direct suit can be maintained against the United States. But
ordinance appropriating from its public funds an amount corresponding to
when an action is brought by the United States to recover money
the balance due under the RTC decision, no levy under execution may be
in the hands of a party who has a legal claim against them, it
would be a very rigid principle to deny to him the right of setting validly effected on the public funds of petitioner.
up such claim in a court of justice, and turn him around to an However, where the municipality fails or refuses, without justifiable
application to Congress." reason, to effect payment of a final money judgment rendered against it,
It is however, contended for the intervenor that, if there was at the claimant may avail of the remedy of mandamus in order to compel the
all any waiver, it was in favor of the plaintiff against whom the enactment and approval of the necessary appropriation ordinance and the
complaint in intervention was directed. This contention is corresponding disbursement of municipal funds to satisfy the money
untenable. As already stated, the complaint in intervention was judgment.
in a sense in derogation of the defendant's claim over the h. Suit against public officers
possession of the vessel in question.
i. Arigo vs. Swift, G.R. No. 206501, 16 September 2014
g. Consent to be sued does not include consent to execution

M.R.A.D.C. LUMBRE 12
CONSTITUTIONAL LAW REVIEW

ISSUE: Whether the US respondents may be held liable for damages


caused by the USS Guardian to the Tubbataha Reefs?

RULING: No. The US respondents were sued in their official capacity as


commanding officers of the US Navy who had control and supervision over
the USS Guardian and its crew. The alleged act or omission resulting in
the unfortunate grounding of the USS Guardian on the TRNP was
committed while they were performing official military duties. Considering
that the satisfaction of a judgment against said officials will require
remedial actions and appropriation of funds by the US government, the
suit is deemed to be one against the US itself. The principle of State
immunity therefore bars the exercise of jurisdiction by this Court over the
persons of respondents Swift, Rice and Robling.

The waiver of State immunity under the VFA pertains only to criminal
jurisdiction and not to special civil actions such as the present petition for
issuance of a writ of Kalikasan. In fact, it can be inferred from Section 17,
Rule 7 of the Rules that a criminal case against a person charged with a
violation of an environmental law is to be filed separately.

i. Immunity cannot be used to perpetrate an injustice on a citizen


i. Wylie vs. Rarang, 209 SCRA 357
ISSUE: Whether the questioned publication is in the performance of
petitioners’ official functions, and were therefore, immune from suit.

RULING: There is no question that the two (2) petitioners actively


participated in screening the features and articles in the POD as part of
their official functions. Under the rule that U.S. officials in the performance
of their official functions are immune from suit, then it should follow that
the petitioners may not be held liable for the questioned publication.

It is to be noted, however, that the petitioners were sued in their personal


capacities for their alleged tortious acts in publishing a libelous article.

The question, therefore, arises - are American naval officers who commit
a crime or tortious act while discharging official functions still covered by
the principle of state immunity from suit? Pursuing the question further,
does the grant of rights, power, and authority to the United States under
the RP-US Bases Treaty cover immunity of its officers from crimes and
torts? Our answer is No.

The case of Chavez v. Sandiganbayan gives the law on immunity from suit
of public officials:

“The general rule is that public officials can be held personally accountable
for acts claimed to have been performed in connection with official duties
where they have acted ultra vires or where there is showing of bad faith.”

M.R.A.D.C. LUMBRE 13
CONSTITUTIONAL LAW REVIEW

III. The Fundamental Powers of the State most demanding of the three powers. The justification is found in the Latin maxims:
salus populi est suprema lex, and sic utere tuo ut alienum non laedas.

i. Requisites for valid exercise


Similarities and differences – As to: Purpose, Delegation, Persons affected,
Effects of the exercise, Benefits of the persons affected, Amount of imposition, a) Lawful Subject: The interests of the public in general, as distinguished
and Constitutional limitations from those of a particular class, require the exercise of the power. This means
that the activity or property sought to be regulated affects the general welfare;
1. The inherent powers of the State are: if it does, then the enjoyment of the rights flowing therefrom may have to yield
(a) Police Power; to the interests of the greater number.

(b) Power of Eminent Domain; and b) Lawful Means: The means employed are reasonably necessary for the
accomplishment of the purpose, and not unduly oppressive on individuals.
(c) Power of Taxation.
Additional Limitations [When exercised by delegate]:
2. Similarities:
a) Express grant by law, e.g., Secs. 16, 391,447, 458 and 468, R. A. 7160,
a) Inherent in the State, exercised even without need of express constitutional for local government units;
grant.
b) Within territorial limits for local government units, except when exercised
b) Necessary and indispensable; State cannot be effective without them. to protect water supply; and
c) Methods by which State interferes with private property. c) Must not be contrary to law. Activity prohibited by law cannot, in the
guise of regulation, be allowed; an activity allowed by law may be regulated,
d) Presuppose equivalent compensation
but not prohibited.
e) Exercised primarily by the Legislature. ii. Cases
3. Distinctions:
1. MMDA v. Bel-Air Village Association, Inc., 328 SCRA 836, March 27, 2000
a) Police power regulates both liberty and property; eminent domain and taxation affect ISSUE: Whether the MMDA has the authority to open Neptune Street of Bel-Air
only property rights.
Viilage to the public pursuant to its regulatory and police power?
b) Police power and taxation are exercised only by government; eminent domain may RULING: No. Petitioner claims that it has the authority to open Neptune Street to
be exercised by private entities.
public traffic because it is an agent of the state endowed with police power in the
c) Property taken in police power is usually noxious or intended for a noxious purpose delivery of basic services In Metro Manila, and from the premise that it has police
and may thus be destroyed; while in eminent domain and taxation, the property is power, there is no need for the City of Makati to enact ordinance opening the
wholesome and devoted to public use or purpose. Neptune Street.
d) Compensation in police power is the intangible, altruistic feeling that the individual Police Power is an inherent attribute of sovereignty. It has been defined as the
has contributed to the public good; in eminent domain, it is the full and fair equivalent power vested by the Constitution in the legislature to make, ordain, and establish
of the property taken; while in taxation, it is the protection given and/or public all manner of wholesome and reasonable laws, statutes and ordinances, either with
improvements instituted by government for the taxes paid. 5
penalties or without, not repugnant to the Constitution as they shall judge to be for
4. Limitations: Generally, the Bill of Rights, although in some cases the exercise of the good and welfare of the commonwealth, and for the subjects of the same. It
the power prevails over specific constitutional guarantees. The courts may annul the bears stressing that the police power is lodged in the National Legislature. It cannot
improvident exercise of police power, e.g., in Quezon City v. Ericta, 122 SCRA 759 and be exercised by any group or body of individuals nor possessing legislative power.
in Philippine Press Institute v. Comelec, 244 SCRA 272. The National Legislature, however, may delegate this power to the President and
administrative boards as well as the lawmaking bodied of municipal corporations or
the LGUs. Once delegated, the agents can exercise only such legislative power as
1. Police Power are conferred on them by the national lawmaking body.
Definition: The power of promoting public welfare by restraining and regulating the use It must be noted that the powers of MMDA are limited to the following acts:
of liberty and property. Police power is the most pervasive, the least limitable, and the formulation, coordination, regulation, implementation, preparation,
management, monitoring, setting of policies, installation of a system and

M.R.A.D.C. LUMBRE 14
CONSTITUTIONAL LAW REVIEW

administration. There is nothing in RA 7924 that grants the MMDA police power, 3. Valentin Tio v. Videogram Regulatory Board, 151 SCRA 208 (1987)
let alone legislative power.
FACTS: Petitioner assails the constitutionality of Presidential Decree No. 1987
NOTE: In a litany of cases, it has been ruled that MMDA really has no police power. entitled “An Act Creating the Videogram Regulatory Board” with broad powers to
regulate and supervise the videogram industry, on the grounds: (1) the tax
2. PASEI v. Drilon, 163 SCRA 386 (1988)
imposed is harsh, confiscatory, oppressive and/or in lawful restraint or trade in
FACTS: In this petition for certiorari and prohibition, the Philippine Association of violation of the due process clause of the Constitution; that there is overregulation
Service Exporters, Inc. (PASEI), a firm engaged in the recruitment of Filipino of the video industry as if it were a nuisance.
workers, male and female, for overseas placement, challenges the Constitutional
validity of the Department of Labor and Employment’s Department Order No. 1, RULING: The tax imposed by the DECREE is not only a regulatory but also a
revenue measure prompted by the realization that earnings of videogram
Series of 1988, which serves as guidelines for the temporary suspension of
establishments of around P600 million per annum have not been subjected to tax,
deployment of Filipino domestic and household workers. PASEI assails the subject
thereby depriving the Government of an additional source of revenue. It is an end-
measure for: (1) discrimination against males or females since it does not apply to user tax, imposed on retailers for every videogram they make available for public
all Filipino workers but only to domestic helpers and females with similar skills; viewing. It is similar to the 30% amusement tax imposed or borne by the movie
(2)violation of the right to travel; (3) invalid exercise of the lawmaking power, industry which the theater-owners pay to the government, but which is passed on
police power being legislative, and not executive, in character; and (4) violation of to the entire cost of the admission ticket, thus shifting the tax burden on the buying
non-impairment clause. or the viewing public. It is a tax that is imposed uniformly on all videogram
operators.
RULING: DOLE’s DO No. 1 is a valid police power measure. It has been defined as
the "state authority to enact legislation that may interfere with personal liberty or The levy of the 30% tax is for a public purpose. It was imposed primarily to answer
property in order to promote the general welfare." As defined, it consists of (1) an the need for regulating the video industry, particularly because of the rampant film
imposition of restraint upon liberty or property, (2) in order to foster the common piracy, the flagrant violation of intellectual property rights, and the proliferation of
good. It is not capable of an exact definition but has been, purposely, veiled in pornographic video tapes. And while it was also an objective of the DECREE to
general terms to underscore its all-comprehensive embrace. protect the movie industry, the tax remains a valid imposition.
Taxation has been made the implement of the state’s police power. At bottom, the
Based on the concept of police power and since the Court was satisfied that the
rate of tax is a matter better addressed to the taxing legislature.
classification made – the preference for female workers — rests on substantial
distinction, it was held that the PASEI failed to show the why the contested measure We do not share petitioner’s fears that the video industry is being over-regulated
should be nullified. and being eased out of existence as if it were a nuisance. Being a relatively new
industry, the need for its regulation was apparent. While the underlying objective
Moreover, the consequence the deployment ban has on the right to travel does not
of the DECREE is to protect the moribund movie industry, there is no question that
impair the right. The right to travel is subject, among other things, to the
public welfare is at bottom of its enactment, considering “the unfair competition
requirements of "public safety," "as may be provided by law. The Court is well
posed by rampant film piracy; the erosion of the moral fiber of the viewing public
aware of the unhappy plight that has befallen our female labor force abroad,
brought about by the availability of unclassified and unreviewed video tapes
especially domestic servants, which is compelling motive for urgent Government
containing pornographic films and films with brutally violent sequences; and losses
action. As precisely the caretaker of Constitutional rights, the Court is called upon
in government revenues due to the drop in theatrical attendance, not to mention
to protect victims of exploitation. In fulfilling that duty, the Court sustains the
the f act that the activities of video establishments are virtually untaxed since mere
Government's efforts.
payment of Mayor’s permit and municipal license fees are required to engage in
The Court did not find merit in the contention that Department Order No. 1 business.”
constitutes an invalid exercise of legislative power. It is true that police power is
4. Acebedo Optical Company, Inc. v. CA, 329 SCRA 314 (2000)
the domain of the legislature, but it does not mean that such an authority may not
be lawfully delegated. As we have mentioned, the Labor Code itself vests the ISSUE: Whether the City Mayor has the authority to impose special conditions in
Department of Labor and Employment with rulemaking powers in the enforcement the grant of the business permit?
whereof.
RULING: No. Police power is essentially regulatory in nature and the power
It was also held that the non-impairment clause of the Constitution, invoked by the to issue license or grant business permits, if for a regulatory purpose, is within
petitioner, must yield to the loftier purposes targeted by the Government. The the ambit of this power. This power necessarily includes the power to revoke and
concern of the Government, however, is not necessarily to maintain profits of to impose conditions. However, the power to grant or issue licenses or business
business firms. The interest of the State is to provide a decent living to its citizens. permits must always be exercised in accordance with law, with utmost
observance of the rights of all concerned to due process and equal protection of

M.R.A.D.C. LUMBRE 15
CONSTITUTIONAL LAW REVIEW

the law. What is sought by Acebedo from the City Mayor is a permit to engage in As enunciated in the preambular clauses of the challenged BOT Circular, the
the business of running an optical shop. It does not purport to seek a license to overriding consideration is the safety and comfort of the riding public from the
engage in the practice of optometry. A business permit is issued primarily to dangers posed by old and dilapidated taxis.
regulate the conduct of business and the City Mayor cannot, through the issuance
of such permit, regulate the practice of a profession. Such a function is The State, in the exercise of its police power, can prescribe regulations to promote
within the exclusive domain of the administrative agency specifically the health, morals, peace, good order, safety and general welfare of the people. It
empowered by law to supervise the profession, in this case the Professional can prohibit all things hurtful to comfort, safety and welfare of society. It may also
Regulations Commission and the Board of Examiners in Optometry. regulate property rights.

5. Taxicab Operators of MM v. BOT, 117 SCRA 597 (1982) NOTE: Remember this case as it may be asked in the Bar. This would be a good
answer as to the issue of the phasing out of the jeepneys.
ISSUE: Whether the Bureau of Transportation’s Memorandum Circular 77-42 which
declared that “no car beyond six years shall be operated as taxi” is a valid exercise 6. Del Mar v. PAGCOR, 346 SCRA 485 (2000)
of police power.
FACTS: PAGCOR's argues that under Section 11 of P.D. 1869 they were granted
RULING: Yes. PD 101 grants to the Board of Transportation the power: "4. To fix the rights, privileges, and authority to operate and maintain gambling casinos,
just and reasonable standards, classification, regulations, practices, clubs, and other recreation or amusement places, sports, gaming pools, i.e.,
measurements, or service to be furnished, imposed, observed, and followed by basketball, football, lotteries, etc. Therefore it contends that the extent and nature
operators of public utility motor vehicles." of PAGCORs franchise is so broad that literally all kinds of sports and gaming pools,
Section 2 of said Decree provides procedural guidelines for said agency to follow in including jai-alai, are covered therein. It concluded that since under Section 11 of
the exercise of its powers: P.D. No. 1869, games of skill like basketball and football have been lumped
together with the word lotteries just before the word etc. and after the words
"Sec. 2. Exercise of powers. - In the exercise of the powers granted in the preceding gaming pools, it may be deduced from the wording of the law that when bets or
section, the Board shall proceed promptly along the method of legislative inquiry. stakes are made in connection with the games of skill, they may be classified as
Apart from its own investigation and studies, the Board, in its discretion, may
games of chance under the coverage of PAGCORs franchise.
require the cooperation and assistance of the Bureau of Transportation among
others that may be able to furnish useful information or data in the formulation of RULING: No, PAGCOR is not validly vested with the authority to operate jai alai.
the Board of any policy, plan or program in the implementation of this Decree. The PAGCOR is engaged in business affected with public interest. The phrase affected
Board may also call conferences, require the submission of position papers or other with public interest means that an industry is subject to control for the public
documents, information, or data by operators or other persons that may be affected good; it has been considered as the equivalent of subject to the exercise of the
by the implementation of this Decree, or employ any other suitable means of
police power.
inquiry."
A legislative franchise to operate jai-alai is imbued with public interest and involves
It is clear from the provision aforequoted, the Board gives it a wide range of choice
an exercise of police power. The familiar rule is that laws which grant the right to
in gathering necessary information or data in the formulation of any policy, plan or
program. It is not mandatory that it should first call a conference or require the exercise a part of the police power of the state are to be construed strictly and any
submission of position papers or other documents from operators or persons who doubt must be resolved against the grant. The legislature is regarded as the
may be affected, this being only one of the options open to the Board, which is guardian of society, and therefore is not presumed to disable itself or abandon the
given wide discretionary authority. discharge of its duty. Thus, courts do not assume that the legislature intended to
part away with its power to regulate public morals.
Dispensing with a public hearing prior to the issuance of the Circulars is
neither violative of procedural due process. As held in Central Bank vs. Hon. The manner of granting the franchise, to whom it may be granted must always
Cloribel and Banco Filipino, 44 SCRA 307 (1972): defined in a clear and unequivocal language. In the absence of these defining
terms, any claim to a legislative franchise to operate a game played for bets and
"Previous notice and hearing as elements of due process, are constitutionally
denounced as a menace to morality ought to be rejected.
required for the protection of life or vested property rights, as well as of liberty,
when its limitation or loss takes place in consequence of a judicial or quasi-judicial A statute which legalizes a gambling activity or business should be strictly
proceeding, generally dependent upon a past act or event which has to be construed and every reasonable doubt must be resolved to limit the powers and
established or ascertained. It is not essential to the validity of general rules or rights claimed under its authority.
regulations promulgated to govern future conduct of a class or persons or
enterprises, unless the law provides otherwise." 7. Taxation or Police Power? Angeles University Foundation vs. City of
Angeles, et. al., GR No. 189999, June 27, 2012

M.R.A.D.C. LUMBRE 16
CONSTITUTIONAL LAW REVIEW

ISSUE: Whether petitioner AUF is exempt from payment of building permit and and such action of the State amounts to taking for which it should be confiscated,
related fees imposed under Sec. 209 of the National Building Code. as a tax deduction does not offer full reimbursement of the SC discount.

RULING: No! Building permit fees refer to the basic permit fee and other charges RULING: It is in the exercise of its police power that the Congress enacted R.A.
imposed under the National Building Code. Exempted from the payment of building Nos. 9257 and 9442, the laws mandating a 20% discount on purchases of
permit fees are: (1) public buildings and (2) traditional indigenous family dwellings. medicines made by senior citizens and PWDs. It is also in further exercise of this
Not being expressly included in the enumeration of structures to which the power that the legislature opted that the said discount be claimed as tax deduction,
building permit fees do not apply, petitioner’s claim for exemption rests rather than tax credit, by covered establishments.
solely on its interpretation of the term "other charges imposed by the
National Government" in the tax exemption clause of R.A. No. 6055. The issue of just compensation finds no relevance in the instant case as it had
already been made clear in Carlos Superdrug that the power being exercised by
R.A. No. 6055 granted tax exemptions to educational institutions like petitioner
the State in the imposition of senior citizen discount was its police power. Unlike in
which converted to non-stock, non-profit educational foundations. Section 8 of said
law provides: the exercise of the power of eminent domain, just compensation is not required in
wielding police power. This is precisely because there is no taking involved, but
SECTION 8. The Foundation shall be exempt from the payment of all taxes, import only an imposition of burden.
duties, assessments, and other charges imposed by the Government on all
income derived from or property, real or personal, used exclusively for the NOTE: In this case, the petitioners were actually arguing that if the government
educational activities of the Foundation. insists on requiring them to give 20% discounts to SCs and PWDs, it is as if the
government is taking private property so they must at least be given just
That a building permit fee is a regulatory imposition is highlighted by the fact
compensation. The Supreme Court said that the State, in requiring petitioners to
that in processing an application for a building permit, the Building Official shall see
give discounts, is exercising police power. Hence, just compensation is not
to it that the applicant satisfies and conforms with approved standard requirements
on zoning and land use xxxxx since building permit fees are not charges on required. Moreover, what was imposed here is a regulation on the expected profits,
property, they are not impositions from which petitioner is exempt. A not earned profits. Because if it was imposed on earned profits, then it is as if the
"charge" is broadly defined as the "price of, or rate for, something," while the word State took away something that was already in the possession of the petitioners;
"fee" pertains to a "charge fixed by law for services of public officers or for use of thus, in that case, just compensation is required to be paid.
a privilege under control of government."
Petitioner failed to demonstrate that the above bases of assessment were arbitrarily
determined or unrelated to the activity being regulated. Neither has petitioner 2. Eminent Domain (Section 9, Article III, see also Sec, 18, Art, XII, Secs. 4 &
adduced evidence to show that the rates of building permit fees imposed and 9, Art. XIII)
collected by the respondents were unreasonable or in excess of the cost of As Distinguished from Police Power: Police power is the power of the State to
regulation and inspection. promote public welfare by restraining and regulating the use of liberty and property.
NOTE: Payment without annotation of the protest is considered a waiver of protest. The power of eminent domain is the inherent right of the State to condemn private
Imposition of building permit fees is an exercise of police power, not taxation. property to public use upon payment of just compensation.

8. Eminent Domain or Police Power? Southern Luzon Drug Corporation vs. Although both police power and eminent domain have the general welfare for their
DSWD, G.R. No. 199669, 25 April 2017 object, and recent trends show a mingling of the two with the latter being used as an
implement of the former, there are still traditional distinctions between the two.
FACTS: This is a petition for review on certiorari assailing the Decision of the CA
which dismissed the petition for prohibition filed by petitioner against the DSWD, Property condemned under police power is usually noxious or intended for a noxious
the National Council on Disability Affairs or NCDA, the DOF and the BIR, which purpose, hence no compensation is paid. Likewise in the exercise of police power,
sought to prohibit the implementation of Section 4(a) of R.A. No. 9257, otherwise property rights of individuals are subjected to restraints and burdens in order to secure
known as the "Expanded Senior Citizens Act of 2003" and Section 32 of R.A. No. the general comfort, health and prosperity of the State. Where a property interest is
9442, which amends the "Magna Carta for Disabled Persons," particularly the merely restricted because the continued use thereof would be injurious to public
provisions granting 20% discount on the purchase of medicines by senior citizens interest, there is no compensable taking. However, when a property interest is
and persons with disability (PWD), insofar as these provisions only allow tax appropriated and applied to some public purpose, there is need to pay just
deduction on the gross income based on the net cost of goods sold or services compensation. In the exercise of police power, the State restricts the use of private
rendered as compensation to private establishments. Petitioner argues that the law property, but none of the property interests in the bundle of rights which constitute
is confiscatory in a sense that the State takes away a portion of its supposed profits ownership is appropriated for use by or for the benefit of the public.

M.R.A.D.C. LUMBRE 17
CONSTITUTIONAL LAW REVIEW

Use of the property by the owners is limited, but no aspect of the property is used by Constitutional sense, the property owner need not file a claim for just compensation
or for the benefit of the public. The deprivation of use can, in fact, be total, and it will with the COA; he may go directly to the RTC to demand payment.
not constitute compensable taking if nobody else acquires use of the property or any
interest therein. If, however, in the regulation of the use of the property, somebody d) Public Use: It is the general concept of meeting public need or public exigency.
else acquires the use or interest thereof, such restriction constitutes compensable It is not confined to actual use by the public in its traditional sense.
taking. e) Just Compensation: The full and fair equivalent of the property taken; it is the
It is well settled that eminent domain is an inherent power of the State that need fair market value of the property. It is settled that the market value of the
not be granted even by the fundamental law. Sec. 9, Art. Ill of the Constitution, in property is “that sum of money which a person, desirous but not compelled to buy,
mandating that “private property shall not be taken for public use without just and an owner, willing but not compelled to sell, would agree on as a price to be
compensation”, merely imposes a limit on the government’s exercise of this given and received therefor.” Just compensation means not only the correct amount
power and provides a measure of protection to the individual’s right to property. An to be paid to the owner of the land but also payment within a reasonable time
ejectment suit should not ordinarily prevail over the State’s power of eminent domain. from its taking.

NOTE: Eminent domain is an inherent power of the State. Once a State is created, it Reckoning Point: Compensation is determined as of the date of the filing of the
can already exercise such power. This is also practiced by other States. The purpose of complaint for eminent domain, but where the filing of the complaint occurs after
Art. III, Sec. 9 is providing for the limitation of the exercise. It is an international law the actual taking of the property and the owner would be given undue incremental
concept that, as provided for in Art. II, Sec. 2 of the 1987 Constitution, is automatically advantages arising from the use to which the government devotes the property
incorporated as part of the law of the land. Even if Article III, Section 9 was not expropriated, just compensation is determined as of the date of the taking.
incorporated in the Constitution, the government can still exercise the power of eminent ii. Cases
domain.
1. Republic v. Salem Investment Corporation, G.R. No. 137569, June 23,
Who May Exercise Such Power: 2000
The Congress and, by delegation, the President, administrative bodies, local ISSUE: Whether the CA wrongly declared that the spouses Dela Rama could still
government units, and even private enterprises performing public services. sell in 1988 their property as title thereto had not yet passed to the government in
i. Requisites/Limitations for valid exercise 1983, thereby conveying to respondent Guerrero the right to receive the just
compensation for the expropriated area.
a) Necessity: The foundation of the right to exercise eminent domain is genuine
necessity and that necessity must be of a public character. When the power is RULING: No. The power of eminent domain is an inherent power of the State. No
exercised by the Legislature, the question of necessity is generally a political constitutional conferment is necessary to vest it in the State. The constitutional
question; but when exercised by a delegate, the determination of whether there is provision on eminent domain, Art. III, §9, provides a limitation rather than a basis
genuine necessity for the exercise is a justiciable question. for the exercise of such power by the government.

b) Private Property: Private property already devoted to public use or is already It is only upon the completion of these two stages that expropriation is said to have
owned by the State or another agency cannot be expropriated by a delegate of been completed. Moreover, it is only upon payment of just compensation that title
legislature acting under a general grant of authority. (City of Manila v. Chinese over the property passes to the government. Therefore, until the action for
Community). All private property capable of ownership may be expropriated, expropriation has been completed and terminated, ownership over the property
except money and choses in action. Even services may be subject to eminent being expropriated remains with the registered owner. Consequently, the latter can
domain. exercise all rights pertaining to an owner, including the right to dispose of his
property, subject to the power of the State ultimately to acquire it through
NOTE: In this case, the local government unit can no longer expropriate the expropriation.
property already devoted to another public purpose, which is the cemetery, for the
purpose of erecting a barangay hall. In the case at hand, the first stage of expropriation was completed when B.P. Blg.
340 was enacted providing for the expropriation of 1,380 square meters of the land
c) Taking: The requisites for valid taking: (1) the expropriator must enter a private in question. The constitutionality of this law was upheld in the case of Republic v.
property; (2) entry must be for more than a momentary period; (3) entry must be De Knecht. In 1990, the government commenced the second stage of expropriation
under warrant or color of authority; (4) property must be devoted to public use or through the filing of a petition for the determination of just compensation. This
otherwise informally appropriated or injuriously affected; and (5) utilization of the stage was not completed, however, because of the intervention of Guerrero which
property must be in a such a way as to oust the owner and deprive him of beneficial gave rise to the question of ownership of the subject land. Therefore, the title to
use. (Republic v. Castelvi) However, it may include trespass without actual eviction the expropriated property of the De la Ramas remained with them and did not at
of the owner, material impairment of the value of the property or prevention of the that point pass to the government.
ordinary uses for which the property was intended. Where there is taking in the

M.R.A.D.C. LUMBRE 18
CONSTITUTIONAL LAW REVIEW

The De la Ramas are mistaken in arguing that the two stages of expropriation cited 113498 of the estate of defendant Salud Jimenez is considered expropriated in
above only apply to judicial, and not to legislative, expropriation. Although favor of the government based on the Order of the Honorable Court dated July 11,
Congress has the power, to determine what land to take, it cannot do so arbitrarily. 1991." It is crystal clear from the contents of the agreement that the parties limited
Judicial determination of the propriety of the exercise of the power, for instance, in the compromise agreement to matter of just compensation to petitioner. Said
view of allegations of partiality and prejudice by those adversely affected, and the expropriate order is not closely intertwined with the issue of payment such that
just compensation for the subject property is provided in our constitutional system. failure to pay by respondent will also nullify the right of respondent to expropriate.
No statement to this effect was mentioned in the agreement. The Order was
We see no point in distinguishing between judicial and legislative expropriation as mentioned in the agreement only to clarify what was subject to payment.
far as the two stages mentioned above are concerned. Both involve these stages
and in both the process is not completed until payment of just compensation is This court therefore finds that the Court of Appeals did not err in interpreting
made. The Court of Appeals was correct in saying that B.P. Blg. 340 did not "original demand" to mean the fixing of just compensation. The authority of
effectively expropriate the land of the De la Ramas. As a matter of fact, it merely respondent and the nature of the purpose thereof have been put to rest when the
commenced the expropriation of the subject property. Expropriation Order dated July 11, 1991 became final and was duly admitted by
petitioner in the compromise agreement. The only issue for consideration is the
Thus, in 1988, the De la Ramas still had authority to transfer ownership of their manner and amount of payment due to petitioner. In fact, aside from the
land and convey all rights, including the right to receive just compensation, to withdrawal of private respondent's appeal to the Court of Appeals concerning Lot
Guerrero. 1406-A, the matter of payment of just compensation was the only subject of the
2. Estate of Salud Jimenez v. PEZA, 349 SCRA 240, Jan. 16, 2001 compromise agreement dated January 4, 1993. Under the compromise agreement,
petitioner was supposed to receive respondent's Lot No. 434 in exchange for Lot
ISSUE: Whether the CA wrongly interpreted the phrase “original demand” 1406-B. When respondent failed to fulfill its obligation to deliver Lot 434, petitioner
contained in Article 2041 of petitioner estate is the return of the subject lot (Lot can again demand for the payment but not the return of the expropriated Lot 1406-
1406-B) which is sought to be expropriated and not the determination of just B. This interpretation by the Court of Appeals is in according with Section 4 to 8,
compensation for the lot, and ordering the trial judge to proceed with the hearing Rule 67 of the Rules of Court.
of the expropriation proceedings regarding the determination of just compensation.
3. Mactan Cebu International Airport Authority v. Lozada, Jr., February 25,
RULING: No. Expropriation proceedings involve two (2) phases. The first phase 2010
ends either with an order of expropriation (when the right of plaintiff to take the
land and the public purpose to which they are to be devoted are upheld) or an order ISSUE: Whether the CA erred in ruling that respondents may initiate a claim for
of dismissal. Either order would be a final one since if finally disposes of the case. the recovery of possession and reconveyance of ownership of a lot that was
The second phase concerns the determination of just compensation to be previously expropriated, but whose purpose was never actually initiated, realized,
ascertained by three (3) commissioners. It ends with an order fixing the amount to or implemented.
be paid to the defendant. Inasmuch as it leaves nothing more to be done, this order RULING: No. The Court held that it is apparent that the acquisition by the Republic
finally disposes of the second stage. To both orders the remedy therefrom is an
of the expropriated lots was subject to the condition that the Lahug Airport would
appeal.
continue its operation. The condition not having materialized because the airport
In the case at bar, the first phase was terminated when the July 11, 1991 order of had been abandoned, the former owner should then be allowed to reacquire the
expropriation became final and the parties subsequently entered into a compromise expropriated property.
agreement regarding the mode of payment of just compensation. When respondent
failed to abide by the terms of the compromise agreement, petitioner filed and It is well settled that the taking of private property by the Governments power of
action to partially rescind the same. Obviously, the trial could only validly order the eminent domain is subject to two mandatory requirements: (1) that it is for a
rescission of the compromise agreement anent the payment of just compensation particular public purpose; and (2) that just compensation be paid to the property
inasmuch as that was the subject of the compromise. However, on August 4, 1991, owner. These requirements partake of the nature of implied conditions that should
the trial court gravely abused its discretion when it ordered the return of Lot 1406- be complied with to enable the condemner to keep the property expropriated. More
B. It, in effect, annulled the Order of Expropriation dated July 11, 1991 which was particularly, with respect to the element of public use, the expropriator should
already final and executory. commit to use the property pursuant to the purpose stated in the petition for
Contrary to petitioner's contention, the incorporation of the expropriation order in expropriation filed, failing which, it should file another petition for the new purpose.
the compromise agreement did not subject said to rescission but instead If not, it is then incumbent upon the expropriator to return the said property to its
constituted an admission by petitioner of respondent's authority to expropriate the private owner, if the latter desires to reacquire the same. Otherwise, the judgment
subject parcel of land and the public purpose for which it was expropriated. This is of expropriation suffers an intrinsic flaw, as it would lack one indispensable element
evident from paragraph three (3) of the compromise agreement which states that for the proper exercise of the power of eminent domain, namely, the particular
the "swap arrangement recognizes the fact that Lot 1406-B covered by TCT No. T- public purpose for which the property will be devoted. Accordingly, the private

M.R.A.D.C. LUMBRE 19
CONSTITUTIONAL LAW REVIEW

property owner would be denied due process of law, and the judgment would Thus, SC ruled that the special circumstances prevailing in this case entitle
violate the property owners right to justice, fairness, and equity. Besides, the Court respondent to recover possession of the expropriated lot from the Republic.
expressly hold that the taking of private property, consequent to the Governments
While the prevailing doctrine is that "the non-payment of just compensation does
exercise of its power of eminent domain, is always subject to the condition that the
not entitle the private landowner to recover possession of the expropriated lots,
property be devoted to the specific public purpose for which it was taken. If this
however, in cases where the government failed to pay just compensation within
particular purpose or intent is not initiated or not at all pursued, and is peremptorily
five (5) years from the finality of the judgment in the expropriation proceedings,
abandoned, then the former owners, if they so desire, may seek the reversion of
the owners concerned shall have the right to recover possession of their property.
the property, subject to the return of the amount of just compensation received.
After all, it is the duty of the government, whenever it takes property from private
In such a case, the exercise of the power of eminent domain has become improper
persons against their will, to facilitate the payment of just compensation. In
for lack of the required factual justification.
Cosculluela v. Court of Appeals, we defined just compensation as not only the
Furthermore, the right of respondents to repurchase Lot No. 88 may be enforced correct determination of the amount to be paid to the property owner but also the
based on a constructive trust constituted on the property held by the government payment of the property within a reasonable time. Without prompt payment,
in favor of the former. compensation cannot be considered "just."

4. Republic vs. Lim, GR No. 161656, June 29, 2005 NOTE: The Supreme Court stated in this case that just compensation also pertains
to payment within a reasonable period, or within 5 years. Beyond this period, it can
ISSUE: Whether the Republic has retained ownership of Lot 932 despite its failure
no longer be considered just. In this case, the owner of the property can recover
to pay respondent’s predecessors-in-interest the just compensation therefor
the expropriated property.
pursuant to the judgment of the CFI rendered as early as May 14, 1940.
5. Heirs of Alberto Suguitan v. City of Mandaluyong, 328 SCRA 137, March
RULING: One of the basic principles enshrined in our Constitution is that no person
14, 2000
shall be deprived of his private property without due process of law; and in
expropriation cases, an essential element of due process is that there must be just ISSUE: Whether the Sangguniang Panlungsod of Mandaluyong City validly
compensation whenever private property is taken for public use. Accordingly, exercised its power of eminent domain by merely issuing a Resolution authorizing
Section 9, Article III, of our Constitution mandates: "Private property shall not be the City mayor to institute expropriation proceedings over the properties of
taken for public use without just compensation." The Republic disregarded the petitioner.
foregoing provision when it failed and refused to pay respondent’s predecessors- RULING: No. The power of eminent domain is essentially legislative in nature. It
in-interest the just compensation for Lots 932 and 939. is firmly settled, however, that such power may be validly delegated to local
government units, other public entities and public utilities, although the scope of
The Court of Appeals is correct in saying that Republic’s delay is contrary to the
this delegated legislative power is necessarily narrower than that of the delegating
rules of fair play. In jurisdictions similar to ours, where an entry to the expropriated authority and may only be exercised in strict compliance with the terms of the
property precedes the payment of compensation, it has been held that if the delegating law.
compensation is not paid in a reasonable time, the party may be treated as a
trespasser ab initio. The basis for the exercise of the power of eminent domain by local government
units is section 19 of RA 7160 which provides that:
… It is tantamount to confiscation of private property. While it is true that all private
“A local government unit may, through its chief executive and acting
properties are subject to the need of government, and the government may take
pursuant to an ordinance, exercise the power of eminent domain for public
them whenever the necessity or the exigency of the occasion demands, however
use, purpose, or welfare for the benefits of the poor and the landless, upon
from the taking of private property by the government under the power of eminent
payment of just compensation, pursuant to the provisions of the
domain, there arises an implied promise to compensate the owner for his loss. Constitution and pertinent laws; Provided, however, That the power of
There is a recognized rule that title to the property expropriated shall pass from eminent domain may not be exercised unless a valid and definite offer has
been previously made to the owner, and such offer was not accepted;
the owner to the expropriator only upon full payment of the just compensation. So,
Provided, further, That the local government unit may immediately take
how could the Republic acquire ownership over Lot 932 when it has not paid its
possession of the property upon the filing of the expropriation proceedings
owner the just compensation, required by law, for more than 50 years? Clearly,
and upon making a deposit with the proper court of at least fifteen percent
without full payment of just compensation, there can be no transfer of title from (15%) of the fair market value of the property based on the current tax
the landowner to the expropriator. declaration of the property to be expropriated; Provided, finally, That the
amount to be paid for the expropriated property shall be determined by

M.R.A.D.C. LUMBRE 20
CONSTITUTIONAL LAW REVIEW

the proper court, based on the fair market value at the time of the taking RULING: No. The necessity for taking petitioner's property for public use upon
of the property.” payment of just compensation was alleged in the said Complaint. The allegation
stressing that the property would be used to improve the delivery of health services
Despite the existence of this legislative grant in favor of local governments, it is
still the duty of the courts to determine whether the power of eminent domain is satisfied the requirements of necessity and public use. Needless to state,
being exercised in accordance with the delegating law. In fact, the courts have respondent has the burden of proving the elements of eminent domain during the
adopted a more censorious attitude in resolving questions involving the proper continuation of the proceedings in the trial court, and the petitioner the right to
exercise of this delegated power by local bodies, as compared to instances when it rebut such proof.
is directly exercised by the national legislature.
As pointed out by the Solicitor General, the current effective law on delegated
The courts have the obligation to determine whether the following requisites have authority to exercise the power of eminent domain is found in Section 12, Book III
been complied with by the local government unit concerned: of the Revised Administrative Code, which provides:
1. An ordinance is enacted by the local legislative council authorizing the local "SEC. 12. Power of Eminent Domain -- The President shall determine when it
chief executive, in behalf of the local government unit, to exercise the power is necessary or advantageous to exercise the power of eminent domain in
of eminent domain or pursue expropriation proceedings over a particular behalf of the National Government, and direct the Solicitor General, whenever
private property.
he deems the action advisable, to institute expropriation proceedings in the
2. The power of eminent domain is exercised for public use, purpose or welfare, proper court."
or for the benefit of the poor and the landless.
The foregoing provision does not require prior unsuccessful negotiation as a
3. There is payment of just compensation, as required under Section 9, Article condition precedent for the exercise of eminent domain. In Iron and Steel Authority
III of the Constitution, and other pertinent laws. v. Court of Appeals, the President chose to prescribe this condition as an additional
requirement instead. In the instant case, however, no such voluntary restriction
4. A valid and definite offer has been previously made to the owner of the
property sought to be expropriated, but said offer was not accepted. was imposed.

In the present case, the City of Mandaluyong seeks to exercise the power of In an eminent domain proceeding, a motion to dismiss filed under Rule 67 prior to
eminent domain over petitioners' property by means of a resolution, in the 1997 amendments partakes of the nature of an answer. Hence, its allegations
contravention of the first requisite. The law in this case is clear and free from of facts must be proven. On the other hand, under the 1997 Rules, upon the
ambiguity. government's deposit of an amount equivalent to the assessed value of the
property, a writ of possession shall be issued by the trial court without need of any
6. SMI Development Corporation v. Republic, 323 SCRA 862, Jan. 28, 2000
hearing as to the amount to be deposited.
FACTS: The Republic of the Philippines represented by the Department of Health
Sec. 2, Rule 67 of the 1997 Rules of Court, states:
thru the National Children’s Hospital filed a complaint for Eminent Domain against
SMI Development Corporation for the purpose of expropriating three parcels of land Sec. 2. Entry of plaintiff upon depositing value with authorized government
belonging to said corporation which are adjacent to the premises of the hospital. depositary. — Upon the filing of the complaint or at any time thereafter, and
In its Motion to Dismiss, the defendant alleged that the complaint lacked after due notice to the defendant, the plaintiff shall have the right to take or
or had insufficient cause of action; that the taking of the property would enter upon the possession of the real or personal property involved if he
not serve the purpose for which it was intended; that the plaintiff failed to deposits with the authorized government depositary an amount equivalent to
negotiate with it for the purchase of the property which reflects against the assessed value of the property for purposes of taxation to be held by such
bank subject to the orders of the court. Such deposit shall be in money, unless
the urgency and necessity of the plaintiffs need of the property and implies lack
in lieu thereof the court authorizes the deposit of a certificate of deposit in a
of intention to pay its true and fair market value; and that [the] necessity to
government bank of the Republic of the Philippines payable on demand to the
expropriate the property is negated by the fact that less than a kilometer from the
authorized government depositary.
plaintiffs premises was the Quezon Institute which is presently not put to its
optimum use and is a better place for putting up the frontline services for which If personal property is involved, its value shall be provisionally ascertained and
the property is needed with less costs and less prejudice to private rights. the amount to be deposited shall be promptly fixed by the court.

ISSUE: Whether a prior unsuccessful negotiation is a condition sine qua non for After such deposit is made the court shall order the sheriff or other proper
the valid exercise of eminent domain. officer to forthwith place the plaintiff in possession of the property involved
and promptly submit a report thereof to the court with service of copies to the
parties.

M.R.A.D.C. LUMBRE 21
CONSTITUTIONAL LAW REVIEW

Under the foregoing Section, the Republic is entitled to a writ of possession, once Comelec. Section 2 does not constitute a valid exercise of the police power of the
the provisional compensation mentioned therein is deposited. State.

7. PPI v. COMELEC, G.R. No. 119694. May 22, 1995 8. TELEBAP, Inc. v. COMELEC, 289 SCRA 337, April 21, 1998
FACTS: In this Petition for Certiorari and Prohibition with prayer for the issuance ISSUE: Whether Sec. 92 of BP 881 authorizing COMELEC to procure radio and
of a Temporary Restraining Order, PPI asks us to declare Comelec Resolution No. television time to be known as “COMELEC Time” free of charge during the period
2772 unconstitutional and void on the ground that it violates the prohibition of campaign, is valid, as it takes property without due process of law and without
imposed by the Constitution upon the government, and any of its agencies, against just compensation.
the taking of private property for public use without just compensation. Petitioner
also contends that the 22 March 1995 letter directives of Comelec requiring RULING: Yes. In truth, radio and television broadcasting companies, which are
publishers to give free "Comelec Space" and at the same time process raw data to given franchises, do not own the airwaves and frequencies through which they
make it camera-ready, constitute impositions of involuntary servitude, contrary to transmit broadcast signals and images. They are merely given the temporary
the provisions of Section 18 (2), Article III of the 1987 Constitution. Finally, PPI privilege of using them. Since a franchise is a mere privilege, the exercise of the
argues that Section 8 of Comelec Resolution No. 2772 is violative of the privilege may reasonably be burdened with the performance by the grantee of some
constitutionally guaranteed freedom of speech, of the press and of expression. form of public service. In the allocation of limited resources, relevant conditions
may validly be imposed on the grantees or licensees. The reason for this is that, as
RULING: Sec. 2 of Resolution No. 2772 is null and void. The taking of print space already noted, the government spends public funds for the allocation and regulation
here sought to be effected may first be appraised under the rubric of expropriation of the broadcast industry, which it does not do in the case of the print media. To
of private personal property for public use. The threshold requisites for a lawful require the radio and television broadcast industry to provide free air time for the
taking of private property for public use need to be examined here: one is the COMELEC Time is a fair exchange for what the industry gets.
necessity for the taking; another is the legal authority to effect the taking. The
element of necessity for the taking has not been shown by respondent Comelec. It NOTE: Difference between PPI v. COMELEC and TELEBAP v. COMELEC is that in PPI
has not been suggested that the members of PPI are unwilling to sell print space the newspapers are owned by private persons, whereas in TELEBAP, the broadcast
at their normal rates to Comelec for election purposes. Indeed, the unwillingness media companies’ frequencies are owned by the government, and were only
or reluctance of Comelec to buy print space lies at the heart of the problem. 3 granted by virtue of a franchise, subject to control by the latter.
Similarly, it has not been suggested, let alone demonstrated, that Comelec has 9. Heirs of Juancho Ardona vs. Reyes, 125 SCRA 220 (1983)
been granted the power of eminent domain either by the Constitution or by the
legislative authority. A reasonable relationship between that power and the FACTS: The Philippine Tourism Authority filed 4 complaints with the CFI of Cebu
enforcement and administration of election laws by Comelec must be shown; it is City for the expropriation of 282 hectares of rolling land situated in barangays
not casually to be assumed. Malubog and Babag, Cebu City for the development into integrated resort
complexes of selected and well-defined geographic areas with potential tourism
As earlier noted, the Solicitor General also contended that Section 2 of Resolution
value. The PTA will construct a sports complex, club house, golf course, playground
No. 2772, even if read as compelling publishers to "donate" "Comelec space" may
be sustained as a valid exercise of the police power of the state. This argument and picnic area on said land. An electric power grid will also be established by NPC
was, however, made too casually to require prolonged consideration on our part. as well as deep well and drainage system. Complimentary support facilities (malls,
Firstly, there was no effort (and apparently no inclination on the part of Comelec) coffee shops, etc) will also be created. The defendants alleged that the taking is
to show that the police power — essentially a power of legislation — has been allegedly not impressed with public use under the Constitution. Also, assuming that
constitutionally delegated to respondent Commission. Secondly, while private PTA has such power, the intended use cannot be paramount to the determination
property may indeed be validly taken in the legitimate exercise of the police power of the land as a land reform area; that limiting the amount of compensation by
of the state, there was no attempt to show compliance in the instant case with the legislative fiat is constitutionally repugnant; and that since the land is under the
requisites of a lawful taking under the police power. land reform program, it is the Court of Agrarian Relations and not the Court of First
Section 2 of Resolution No. 2772 is a blunt and heavy instrument that purports, Instance, that has jurisdiction over the expropriation cases.
without a showing of existence of a national emergency or other imperious public RULING: There are three provisions of the Constitution which directly provide for
necessity, indiscriminately and without regard to the individual business condition
the exercise of the power of eminent domain.
of particular newspapers or magazines located in differing parts of the country, to
take private property of newspaper or magazine publishers. No attempt was made Section 2, Article IV states that “private property shall not be taken for public use
to demonstrate that a real and palpable or urgent necessity for the taking of print without just compensation.”
space confronted the Comelec and that Section 2 of Resolution No. 2772 was itself
the only reasonable and calibrated response to such necessity available to the

M.R.A.D.C. LUMBRE 22
CONSTITUTIONAL LAW REVIEW

Section 6, Article XIV allows the State, in the interest of national welfare or defense generally invoking police power, since said prohibition amounts to a taking of
and upon payment of just compensation to transfer to public ownership, utilities respondents' property without payment of just compensation.
and other private enterprises to be operated by the government.
11. When Entitled to Compensation? Mosqueda vs. Pilipino Banana
Section 13, Article XIV states that the Batasang Pambansa may authorize upon Growers & Exporters Association, Inc., G.R. No. 189185, 16 August 2016
payment of just compensation the expropriation of private lands to be subdivided FACTS: Davao City enacted Ordinance No. 0309-07 which banned against aerial
into small lots and conveyed at cost to deserving citizens. spraying as an agricultural practice by all agricultural entities. The Pilipino Banana
Growers and Exporters Association, Inc. (PBGEA) then filed a petition to challenge
The concept of public use is not limited to traditional purposes for the construction
the constitutionality of the ordinance. They alleged that the ordinance exemplified
of roads, bridges, and the like. The idea that "public use" means "use by the public"
the unreasonable exercise of police power; violated the equal protection clause;
has been discarded. As long as the purpose of the taking is public, then the power amounted to the confiscation of property without due process of law. The petition
of eminent domain comes into play. It is accurate to state then that at present argues that the total ban against aerial spraying, coupled with the inadequate time
whatever may be beneficially employed for the general welfare satisfies the to shift to truck-mounted boom spraying, effectively deprives the farmers with an
requirement of public use. The petitioners have not shown that the area being efficient means to control the spread of diseases that threatens the banana
developed is land reform area and that the affected persons have been given plantations. As such, the ordinance is unreasonable, unfair, oppressive, and
emancipation patents and certificates of land transfer. The contract clause has tantamount to a restriction or prohibition of trade.
never been regarded as a barrier to the exercise of the police power and likewise
The petition also submits that the maintenance of the 30-meter buffer zone under
eminent domain.
Section 5 of the ordinance constitutes an improper exercise of police power; that
10. The Office of the Solicitor General vs. Ayala Land Incorporated, G.R. the ordinance will require all landholdings to maintain the buffer zone, thereby
No. 177056, September 18, 2009 diminishing to a mere 1,600 square meters of usable and productive land for every
hectare of the plantation bounding residential areas, with the zone being reserved
ISSUE: Whether the CA seriously erred in affirming the ruling of the RTC that for planting "diversified trees;" that this requirement amounts to taking without
respondents are not obliged to provide free parking spaces to their customers or just compensation or due process; and that the imposition of the buffer zone unduly
the public, and that the practice of charging parking fees is not in violation of the deprives all landowners within the City of Davao the beneficial use of their property.
National Building Code.
RULING: The respondents posit that the requirement of maintaining a buffer zone
RULING: No. The term "parking fees" cannot even be found at all in the entire under Section 6 of the ordinance violates due process for being confiscatory; and
National Building Code and its IRR. that the imposition unduly deprives all agricultural landowners within Davao City of
the beneficial use of their property that amounts to taking without just
Without using the term outright, the OSG is actually invoking police power to justify
compensation.
the regulation by the State, through the DPWH Secretary and local building officials,
of privately owned parking facilities, including the collection by the The position of the respondents is untenable.
owners/operators of such facilities... of parking fees from the public for the use
thereof. In City of Manila v. Laguio, Jr., we have thoroughly explained that taking only
becomes confiscatory if it substantially divests the owner of the beneficial use of
When there is a taking or confiscation of private property for public use, the State its property, viz.:
is no longer exercising police power, but another of its inherent powers, namely,
eminent domain. Eminent domain enables the State to forcibly acquire private lands An ordinance which permanently restricts the use of property that it cannot be used
intended for public use upon... payment of just compensation to the owner. for any reasonable purpose goes beyond regulation and must be recognized as a
taking of the property without just compensation. It is intrusive and violative of the
Although in the present case, title to and/or possession of the parking facilities private property rights of individuals.
remain/s with respondents, the prohibition against their collection of parking fees
from the public, for the use of said facilities, is already tantamount to a taking or The Constitution expressly provides in Article III, Section 9 that, "private property
confiscation of their properties. shall not be taken for public use without just compensation." The provision is the
most important protection of property rights in the Constitution. This is a restriction
The State is not only requiring that respondents devote a portion of the latter's on the general power of the government to take property. The constitutional
properties for use as parking spaces, but is also mandating that they give the public provision is about ensuring that the government does not confiscate the property
access to said parking spaces for free. of some to give it to others. In part too, it is about loss spreading. If the government
takes away a person's property to benefit society, then society should pay. The
In conclusion, the total prohibition against the collection by respondents of parking
principal purpose of the guarantee is "to bar the Government from forcing some
fees from persons who use the mall parking facilities has no basis in the National
Building Code or its IRR. The State also cannot impose the same prohibition by

M.R.A.D.C. LUMBRE 23
CONSTITUTIONAL LAW REVIEW

people alone to bear public burdens which, in all fairness and justice, should be The establishment of the buffer zone is required for the purpose of minimizing the
borne by the public as a whole. effects of aerial spraying within and near the plantations. Although Section 3(e) of
the ordinance requires the planting of diversified trees within the identified buffer
There are two different types of taking that can be identified. A "possessory" taking zone, the requirement cannot be construed and deemed as confiscatory requiring
occurs when the government confiscates or physically occupies property. A payment of just compensation. A landowner may only be entitled to compensation
"regulatory" taking occurs when the government's regulation leaves no reasonable if the taking amounts to a permanent denial of all economically beneficial or
economically viable use of the property. productive uses of the land. The respondents cannot be said to be permanently and
completely deprived of their landholdings because they can still cultivate or make
other productive uses of the areas to be identified as the buffer zones.
In the landmark case of Pennsylvania Coal v. Mahon, it was held that a taking also
could be found if government regulation of the use of property went "too far." When 12. When Just Compensation Shall Accrue? Interest? Secretary of the
regulation reaches a certain magnitude, in most if not in all cases there must be an DPWH vs. Spouses Tecson, G.R. No. 179334, April 21, 2015
exercise of eminent domain and compensation to support the act. While property ISSUE: Whether the just compensation, as well as interest, should be computed
may be regulated to a certain extent, if regulation goes too far it will be recognized from 1940, at the time of the taking (0.70 per sqm) or from 2002, when the RTC
as a taking. rendered a Decision directing DPWH to pay respondents just compensation (1.5K
No formula or rule can be devised to answer the questions of what is too far and per sqm)?
when regulation becomes a taking. In Mahon, Justice Holmes recognized that it RULING: The Court ruled that the purpose of just compensation is not to reward
was "a question of degree and therefore cannot be disposed of by general the owner for the property taken but to compensate him for the loss thereof. As
propositions." On many other occasions as well, the U.S. Supreme Court has said such, the true measure of the property, as upheld by a plethora of cases, is the
that the issue of when regulation constitutes a taking is a matter of considering the market value at the time of the taking, when the loss resulted.
facts in each case. The Court asks whether justice and fairness require that the
economic loss caused by public action must be compensated by the government "Just compensation" is the sum equivalent to the market value of the property,
and thus borne by the public as a whole, or whether the loss should remain broadly described as the price fixed by the seller in open market in the usual and
concentrated on those few persons subject to the public action. ordinary course of legal action and competition, or the fair value of the property as
between the one who receives and the one who desires to sell, it being fixed at the
What is crucial in judicial consideration of regulatory takings is that government time of the actual taking by the government. Just compensation is defined as the
regulation is a taking if it leaves no reasonable economically viable use of property full and fair equivalent of the property taken from its owner by the expropriator.
in a manner that interferes with reasonable expectations for use. A regulation that
permanently denies all economically beneficial or productive use of land is, from Just compensation due respondents-movants in this case should, therefore, be
the owner's point of view, equivalent to a "taking" unless principles of nuisance or fixed not as of the time of payment but at the time of taking in 1940.
property law that existed when the owner acquired the land make the use
prohibitable. When the owner of real property has been called upon to sacrifice all NOTE: If the taking of the property preceded the filing of the case, then just
economically beneficial uses in the name of the common good, that is, to leave his compensation should be determined from the time the property was taken, and not
property economically idle, he has suffered a taking. from the time of filing of the case. As to the imposition of interest, one must take
note that there are 2 kinds of interest: (1) legal interest; and (2) interest for
A regulation which denies all economically beneficial or productive use of land will forbearance of money.
require compensation under the takings clause. Where a regulation places
limitations on land that fall short of eliminating all economically beneficial use, a 13. Prescriptive Period? What is Inverse Condemnation? National Power
taking nonetheless may have occurred, depending on a complex of factors including Corporation vs. Heirs of Macabangkit Sangkay, 656 SCRA 60 (2011)
the regulation's economic effect on the landowner, the extent to which the FACTS: NAPOCOR undertook the Agus River Hydroelectric Power Plant Project in
regulation interferes with reasonable investment-backed expectations and the the 1970s to generate electricity for Mindanao. The project included the
character of government action. These inquiries are informed by the purpose of the construction of several underground tunnels to be used in diverting the water flow
takings clause which is to prevent the government from forcing some people alone
from the Agus River to the hydroelectric plants. On November 21, 1997, the
to bear public burdens which, in all fairness and justice, should be borne by the
respondents, the heirs of Macabangkit Sangkay, as the owners of land with an area
public as a whole.
of 221,573 square meters situated in Ditucalan, Iligan City, sued NPC in the RTC
A restriction on use of property may also constitute a "taking" if not reasonably for the recovery of damages and of the property, with the alternative prayer for
necessary to the effectuation of a substantial public purpose or if it has an unduly the payment of just compensation.
harsh impact on the distinct investment-backed expectations of the owner.
ISSUE: Whether the Heirs of Macabangkit Sangkay’s right to claim just
compensation had prescribed under Section 3(i) of RA No. 6395.

M.R.A.D.C. LUMBRE 24
CONSTITUTIONAL LAW REVIEW

RULING: No. The 5-year prescriptive period under Sec. 3(i) of RA No. 6395 does of the petitioner to delay the inevitable performance of its obligation to pay just
not apply to claims for just compensation, as it is applicable only to an action for compensation for Lot 1406-B. Indeed, the Court pronounced there that "the
damages. compromise agreement was only about the mode of payment by swapping of lots
xxx, only the originally agreed form of compensation that is by [lot12] payment,
The action to recover just compensation from the State or its expropriating agency was rescinded."
differs from the action for damages. The former, also known as inverse
condemnation, has the objective to recover the value of property taken in fact by That pronouncement became the law of the case, anything to the contrary
the governmental defendant, even though no formal exercise of the power of of which the petitioner could not validly rely upon. The doctrine of the law of
eminent domain has been attempted by the taking agency. Just compensation is the case means that whatever is irrevocably established as the controlling legal rule
the full and fair equivalent of the property taken from its owner by the expropriator. between the same parties in the same case, whether correct on general principles
or not, continues to be the law of the case for as long as the facts on which the
The measure is not the takers gain, but the owners’ loss. The word just is used to
legal rule was predicated continue to be the facts of the case before the court. It
intensify the meaning of the word compensation in order to convey the idea that
applies in a situation where an appellate court has made a ruling on a question on
the equivalent to be rendered for the property to be taken shall be real, substantial, appeal and thereafter remands the case to the lower court for further proceedings;
full, and ample. On the other hand, the latter action seeks to vindicate a legal wrong the question then settled by the appellate court becomes the law of the case binding
through damages, which may be actual, moral, nominal, temperate, liquidated, or the lower court and any subsequent appeal, and questions necessarily involved and
exemplary. When a right is exercised in a manner not conformable with the norms dealt with in a former appeal will be regarded as the law of the case in a subsequent
enshrined in Article 19 and like provisions on human relations in the Civil Code, and appeal, although the questions are not expressly treated in the opinion of the court,
the exercise results to the damage of another, a legal wrong is committed and the inasmuch as the presumption is that all the facts in the case bearing on the point
wrongdoer is held responsible. decided have received due consideration whether all or none of them are mentioned
in the opinion.
NOTE: As to the issue of inverse condemnation, the term used is “inverse” because,
as opposed to expropriation where the entity exercising the power is the one who To reiterate, in G.R. No. 137285, the Court upheld the annulment of the
files the complaint, here, the owners of the expropriated property are the ones who Compromise Agreement and recognized that the agreed upon mode of payment of
initiated the complaint for expropriation for the purpose of claiming just the just compensation for Lot 1406-B with Lot 434 was cancelled. It is notable that
compensation. the Court mentioned nothing therein about the invalidation of the amount of just
compensation corresponding to the mode of payment, which was the value of Lot
14. Applying the Doctrine of the Law of the Case in Just Compensation. 434 at the time, which silence was the Court’s acknowledgment that the parties
EPZA (now PEZA) vs. Pulido, 656 SCRA 315 (2011) understood and accepted, by entering into the Compromise Agreement in 1993,
that the just compensation for Lot 1406-B was Lot 434 (or the value of Lot 434,
FACTS: In G.R. No. 137285 the Court explained that the Estate of Salud Jimenez
which at the time of the swap in 1993 was definitely much higher than Lot 434’s
had already acknowledged the expropriation of Lot 1406-B by entering into the
value in 1981).
Compromise Agreement; and that the provisions of the Compromise Agreement
had consequently related only to the form or mode of payment of the just Accordingly, we completely agree with the RTC’s observation that "when the parties
compensation for Lot 1406-B, that is, in lieu of cash, another lot (Lot 434) was to signed the compromise agreement and the same was approved, they had in fact
be delivered as just compensation to the Estate of Salud Jimenez, stating: settled between themselves the question of what is just compensation and that
both of them had intended that defendant would be compensated on the basis of
Considering that the decision in G.R. No.137285 became final and executory, the
prevailing values at the time of the agreement." We further completely agree with
RTC conducted proceedings to determine the just compensation for Lot 1406-B.
the CA’s conclusion that "by agreeing to a land swap in 1993 in the ill-fated
During the trial, however, the petitioner raised the issue of whether the just
compromise agreement, [PEZA] had impliedly agreed to paying just compensation
compensation should be based on the value or assessment rate prevailing in 1981
using the market values in 1993."
or in 1993, insisting that the just compensation for Lot 1406-B should be equivalent
to its fair market value in 1981, the time of the filing of its expropriation complaint, With the annulment of the Compromise Agreement, the payment of just
which was the time of the taking. The Estate of Salud Jimenez contended, in compensation for Lot 1406-B now has to be made in cash. In that regard, the order
contrast, that the just compensation should be reckoned as of August 4, 1997, of the Court to remand to the RTC for the determination of just compensation was
when the Compromise Agreement was annulled and set aside. indubitably for the sole objective of ascertaining the equivalent monetary value in
1993 of Lot 1406-B or Lot 434.
RULING: What would have been an easy and straightforward implementation of
the decision promulgated on January 16, 2001 in G.R. No. 137285 was delayed by Accordingly, the Court hereby imposed 12% interest per annum on the unpaid
the petitioner’s interposition of the issue on the proper reckoning point for gross value of ₱81,331,600.00 for Lot 1406-B (i.e., 13,118 square meters x
computing the just compensation for Lot 1406-B. A reading of the decision in G.R. ₱6,200.00/square meter) from August 23, 1993, the date of the approval of the
No. 137285 exposes the interposition as nothing more than an insincere attempt failed Compromise Agreement, until the full amount of the just compensation is

M.R.A.D.C. LUMBRE 25
CONSTITUTIONAL LAW REVIEW

paid, as a way of making the compensation just. This accords with a long line of Depreciated replacement cost approach is the "method of valuation which provides
pertinent jurisprudence, whereby the Court has imposed interest at 12% per the current cost of replacing an asset with its modern equivalent asset less
annum in eminent domain whenever the expropriator has not immediately deductions for all physical deterioration and all relevant forms of obsolescence and
delivered the just compensation. The imposition of interest was in the nature of optimisation."
damages for delay in payment which, in effect, makes the obligation on the part of
the government one of forbearance. Under this method, the appraiser assesses the current gross replacement of the
assets, usually comprised of the land and the building. If the asset is an
15. When Other Methods of Valuation is Allowed. Republic vs. Judge Mupas improvement, the appraiser assesses the cost of its replacement with a modern
and PIATCO, G.R. No. 181892, 8 September 2015 equivalent and deducts depreciation to reflect the differences between the
hypothetical modern equivalent and the actual asset. The appraiser has to
FACTS: On 2004, the Republic filed a complaint for expropriation of the NAIA IPT "establish the size and specification that the hypothetical buyer ideally requires at
III before the RTC. Notably, the property to be expropriated did not include the the date of valuation in order to provide the same level of productive output or an
land which the Republic already owns – only the NAIA-IPT structures therein. equivalent service."
Pending the expropriation case, the SC ruled that the Republic is barred from taking
over the NAIA-IPT III until just compensation is paid to PIATCO as the builder and Construction costs are "the costs that are normally and directly incurred in the
owner of the structure. It also ruled that RA 8974 applies insofar as it: 1) provides purchase and installation of an asset, or group of assets, into functional use." On
valuation standards in determining the amount of just compensation; and 2) the other hand, attendant costs are "the costs that are normally required to
required the Republic to immediately pay PIATCO at least the proffered value of purchase and install a property but that are not usually included in the vendor
the NAIA-IPT III for purposes of determining the effectivity of the WOP. invoice."
For purposes of computing just compensation, the SC held that PIATCO should only In these consolidated cases, we rule that the depreciated replacement cost method,
be paid the value of the improvements and/or structures using the replacement rather than the new replacement cost method, is the more appropriate method to
cost method under Section 10 of RA 8974 IRR. The SC added, however, that the use in appraising NAIA-IPT III.
replacement cost method is only one of the factors to be considered in determining
just compensation; equity should also be considered. Injustice would result if we award PIATCO just compensation based on the new
replacement cost of the NAIA-IPT III, and disregard the fact that the Government
RTC rendered a computation of just compensation at USD116M, and further expropriated a terminal that is not brand new; the NAIAIPT III simply does not
directed the Republic and the team of Takenaka and Asahikosan to pay their have the full economic and functional utility of a brand new airport.
respective shares in the BOC expenses. On appeal, the CA ordered the Republic of
the Philippines to pay the PIATCO the amount of USD371M with interest at 6% Adjustments for depreciation should be made to reflect the differences between the
annum as just compensation for the expropriation of the NAIA-IPT III. MR was modern equivalent asset and the actual asset or the NAIA-IPT III. The reason is
denied. Hence, this petition. that depreciation involves the loss of value caused by the property's reduced utility
as a result of damage, advancement of technology, current trends and tastes, or
RULING: In computing the just compensation, the SC applied the depreciated environmental changes.
replacement cost method consistent with Section 10 of RA 8974 IRR and the
principle that the property owner of the expropriated property shall be Just compensation must not extend beyond the property owner's loss or injury.
compensated for his actual loss. SC therefore agreed with the Gleeds' deduction of This is the only way for the compensation paid to be truly just, not only to the
depreciation and deterioration from the construction cost. individual whose property is taken, but also to the public who will shoulder the cost
of expropriation.
There are various methods of appraising a property using the cost approach.
NOTE: Just compensation on the appropriation of specialized properties such as
Among them are the reproduction cost, the replacement cost new, and the airports and schools, may be computed using other methods.
depreciated replacement cost. Reproduction cost is the "estimated current cost to
construct an exact replica of the subject building, using the same materials, 16. Determination of Just Compensation a Judicial Function. LBP vs.
construction standards, design, layout, and quality of workmanship; and Dalauta, G.R. No. 190004, 8 August 2017
incorporating all the deficiencies, superadequacies, and obsolescence of the subject FACTS: Dalauta was the registered owner of a 25.2160-hectare agricultural land
building." It is the cost of duplicating the subject property at current price or the in Butuan City, which was placed by DAR under compulsory acquisition of CARP.
current cost of reproducing a new replica of the property being appraised using the LBP offered ₱192,782.59 as compensation for the land, but Dalauta rejected such
same, or closely similar, materials.
valuation for being too low. The case was referred to the DAR Adjudication
Replacement cost new is "the estimated cost to construct a building with utility Board (DARAB) through the Provincial Agrarian Reform Adjudicator (PARAD) of
equivalent to the appraised building using modern materials and current standards, Butuan City, which affirmed the valuation made by LBP after a summary
design, and layout" administrative proceeding was conducted.

M.R.A.D.C. LUMBRE 26
CONSTITUTIONAL LAW REVIEW

Dalauta filed a petition for determination of just compensation with the RTC, acting appeal the decision to the SAC. This is because the DARAB decision can never attain
as Special Agrarian Court (SAC, sitting as SAC, alleging that LBP’s valuation of the finality as it is not a decision promulgated by the judicial courts. Also, determination
land was inconsistent with DAR Administrative Order (A.O.) No. 06, series of 1992 of just compensation is a judicial function. However, the period to assail the DARAB
for determining the just compensation of lands covered by CARP’s compulsory decision is not imprescriptible. Citing Article 1144(2) of the Civil Code, the action
acquisition scheme. Giving weight to the report of the board of commissioners, SAC must be brought within 10 years from the time the right of action accrues upon an
ruled in favor of Dalauta, ordering DAR and LBP to pay Dalauta ₱2,639,557.00. LBP obligation created by law. The accrual of the right of action shall be counted not
moved for reconsideration but was denied SAC. Hence, LBP elevated the case to from receipt of the DARAB decision but from the time the property was taken by
the CA, arguing that SAC erred in fixing the just compensation and in taking the expropriating authority, because in that situation there was an obligation that
cognizance of the case since the DARAB decision already attained finality when was created by law. Take note, as this might be asked in the Bar.
respondent failed to appeal said decision within 15 days.
17. Judicial Determination of Just Compensation. Alfonso vs. LBP and DAR,
However, the CA ruled that the SAC correctly took cognizance of the case and G.R. No. 181912, 29 November 2016
reiterated that the SAC had original and exclusive jurisdiction over all petitions for
the determination of just compensation. The appellate court stated that the original FACTS: Upon the effectivity of RA 6657, the DAR sought to acquire Palomar’s San
and exclusive jurisdiction of the SAC would be undermined if the DAR would vest Juan and Bibincahan properties at a valuation of P36,066.27 and P792,869.06,
in administrative officials the original jurisdiction in compensation cases and make respectively. Palomar, however, rejected the valuations.
the SAC an appellate court for the review of administrative decisions.
Land Valuation Case Nos. 68-01 and 70-01 were consequently filed before the DAR
ISSUE: Whether the action to question the valuation of the DARAB has prescribed Provincial Adjudication Board (Board) for summary determination of just
for failure to assail the same within 15 days. compensation. In the meantime, or on April 16, 2001, Palomar sold her rights over
the two properties to petitioner Ramon M. Alfonso. No agreement has been reached
RULING: Yes. The Court declared that the final determination of just
until the case reached the Special Agrarian Court.
compensation is a judicial function; that the juris diction of the Regional Trial Court,
sitting as Special Agrarian Court, is original and exclusive, not appellate. ISSUE: Whether the courts are obliged to apply the DARAB formula in cases where
The Court stated that in agrarian reform cases, primary jurisdiction is vested in the they are asked to determine just compensation for property covered by RA 6657.
DAR, more specifically, in the DARAB as provided for in Section 50 of R.A. No. 6657. RULING: The factors listed under Section 17 of RA 6657 and its resulting formulas
On the other hand, the SACs are the Regional Trial Courts expressly granted by law provide a uniform framework or structure for the computation of just compensation
with original and exclusive jurisdiction over all petitions for the determination of which ensures that the amounts to be paid to affected landowners are not arbitrary,
just compensation to landowners. absurd or even contradictory to the objectives of agrarian reform. Until and unless
Citing the case of LBP v. Heirs of Vda. de Arieta, the Court held that DAR-awarded declared invalid in a proper case, the DAR formulas partake of the nature of
compensation, LBP's valuation of lands covered by CARL is considered only as statutes, which under the 2009 amendment became law itself, and thus have in
an initial determination, which is not conclusive, as it is the RTC, sitting as a Special their favor the presumption of legality, such that courts shall consider, and not
Agrarian Court, which should make the final determination of just compensation. disregard, these formulas in the determination of just compensation for properties
covered by the CARP. When faced with situations which do not warrant the
In maintaining the original and exclusive jurisdiction of SAC on the final formula's strict application, courts may, in the exercise of their judicial discretion,
determination of just compensation on landowners, the Court ruled that the relax the formula's application to fit the factual situations before them, subject only
valuation of property in eminent domain is essentially a judicial function which to the condition that they clearly explain in their Decision their reasons (as borne
cannot be vested in administrative agencies. by the evidence on record) for the deviation undertaken. It is thus entirely allowable
for a court to allow a landowner's claim for an amount higher than what would
The Court also added that Congress expressly granted the RTC, acting as SAC, the otherwise have been offered (based on an application of the formula) for as long
original and exclusive jurisdiction over all petitions for the determination of just as there is evidence on record sufficient to support the award.
compensation to landowners. Only the legislature can recall that power. The DAR
has no authority to qualify or undo that. NOTE: This case is similar to Dalauta as it involves determination of just
compensation, but the issue here is the DARAB Formula and whether the court
NOTE: This is a very important case as it overruled three (3) SC rulings. Before should follow the same. If the court (SAC) is obliged to apply the DARAB Formula,
the promulgation of Dalauta, the rule is that period to appeal the DARAB valuation then it would be merely following the formula or decision of an administrative body,
to the SAC is 15 days from receipt of the decision, as according to procedural law, which runs counter to the principle that determination of just compensation is a
the decision of the DARAB, which is an administrative agency, may be appealed to judicial function. However, the Court added that the DARAB should be accorded
the RTC acting as a Special Agrarian Court. After the lapse of 15 days the DARAB great respect for their expertise and SACs should consider or follow primarily the
decision shall attain finality. In Dalauta, the new rule is that even if the 15-day DARAB Formula, but they are not required to strictly follow the same. They may
period to appeal the DARAB decision has lapsed, the aggrieved party may still deviate from the Formula, provided that: (1) the decision to deviate must be

M.R.A.D.C. LUMBRE 27
CONSTITUTIONAL LAW REVIEW

stated; and (2) the deviation should be supported by evidence on record. Thus, to
reiterate: determination of just compensation is a judicial function. However,
because of the Court’s respect for the expertise of the DARAB, the SACs are ordered
to follow first the DARAB Formula, but ultimately they can deviate from the said
Formula, provided that they comply with the requirements stated above.

3. Taxation
NOTE: The primary source of the power of taxation lies with the people and is delegated
to the legislature.
Limitations:

Due Process of Law: Tax should not be confiscatory.


Equal Protection Clause: Taxes should be uniform and equitable. (Sec. 28(1), Art.
VI)
Public Purpose: (Tax for special purpose: [Sec. 29 (3), Art. VI]: Treated as a special
fund and paid out for such purpose only; when purpose is fulfilled, the balance, if any,
shall be transferred to the general funds of the Government.

Double Taxation: Additional taxes are laid on the same subject by the same taxing
jurisdiction during the same taxing period and for the same purpose.
Police Power v. Taxation. In Gerochi v. Department of Energy, G.R. No. 159796, July
17, 2007, the Court made a conservative and pivotal distinction between police power
and taxation, holding that the distinction rests in the purpose for which the charge is
made. If generation of revenue is the primary purpose and regulation is merely
incidental, the imposition is a tax; but if regulation is the primary purpose, the fact that
revenue is incidentally raised does not make the imposition a tax. Thus, the Supreme
Court concluded that the Universal Charge imposed under Sec. 34 of the EPIRA is an
exaction that invokes the State’s police power, particularly its regulatory dimension,
gleaned from Sec. 34 itself which enumerates the purposes of the Universal Charge
which can be amply discerned as regulatory in character.
License fee v. Tax

1) License fee is a police measure; tax is a revenue measure.


2) Amount collected for a license fee is limited to the cost of permit and reasonable
police regulation [except when the license fee is imposed on a non-useful occupation,
as in Physical Therapy Organization v. Municipal Board of Manila, infra.]; amount of tax
may be unlimited provided it is not confiscatory.
3) License fee is paid for the privilege of doing something, and may be revoked when
public interest so requires; Tax is imposed on persons or property for revenue. See:
Compania General de Tabacos v. City of Manila, 8 SCRA 367.

i. Requisites for valid exercise

M.R.A.D.C. LUMBRE 28
CONSTITUTIONAL LAW REVIEW

IV. State Policies, Principles and Other General RULING: The enforcement of the national budget, as primarily contained in the
Considerations GAA, is indisputably a function both constitutionally assigned and properly
entrusted to the Executive branch of government.
In Guingona, Jr. v. Hon. Carague, the Court explained that the phase of budget
1. Republicanism (Art. II, Sec. 1) execution "covers the various operational aspects of budgeting" and accordingly
includes "the evaluation of work and financial plans for individual activities," the
Section 1. The Philippines is a democratic and republican State. Sovereignty resides in "regulation and release of funds" as well as all "other related activities" that
the people and all government authority emanates from them. comprise the budget execution cycle. This is rooted in the principle that the
allocation of power in the three principal branches of government is a grant of all
a. Separation of powers –
powers inherent in them. Thus, unless the Constitution provides otherwise, the
The principle of separation of powers and its concepts of autonomy and Executive department should exclusively exercise all roles and prerogatives which
independence stem from the notion that the powers of government must be divided go into the implementation of the national budget as provided under the GAA as
to avoid concentration of these powers in any one branch; the division, it is hoped, well as any other appropriation law.
would avoid any single branch from lording its power over the other branches or
In view of the foregoing, the Legislative branch of government, much more any of
the citizenry. To achieve this purpose, the divided power must be wielded by co-
its members, should not cross over the field of implementing the national budget
equal branches of government that are equally capable of independent action in
since, as earlier stated, the same is properly the domain of the Executive. Again,
exercising their respective mandates.
in Guingona, Jr., the Court stated that "Congress enters the picture when it
MMDA v. Concerned Residents of Manila Bay, G.R. Nos. 171947-48, deliberates or acts on the budget proposals of the President.
February 15, 2011;
Thereafter, Congress, "in the exercise of its own judgment and wisdom, formulates
ISSUE: Whether or not the setting of time frame for the concerned government an appropriation act precisely following the process established by the Constitution,
agencies to perform their assigned task is an encroachment over the powers which specifies that no money may be paid from the Treasury except in accordance
and functions of the Executive Branch headed by the President of the with an appropriation made by law." Upon approval and passage of the GAA,
Philippines? Congress’ law-making role necessarily comes to an end and from there the
Executive‘s role of implementing the national budget begins. So as not to blur the
RULING: No. The case is now in the execution phase of the final and executory constitutional boundaries between them, Congress must "not concern itself with
December 18, 2008 Decision. The Manila Bay Advisory Committee was created to details for implementation by the Executive.
receive and evaluate the quarterly progressive reports on the activities undertaken
by the agencies in accordance with said decision and to monitor the execution The foregoing cardinal postulates were definitively enunciated in Abakada where
phase. In the absence of specific completion periods, the Committee recommended the Court held that "from the moment the law becomes effective, any provision of
that time frames be set for the agencies to perform their assigned tasks. This may law that empowers Congress or any of its members to play any role in the
be viewed as an encroachment over the powers and functions of the Executive implementation or enforcement of the law violates the principle of separation of
Branch headed by the President of the Philippines. powers and is thus unconstitutional."

This view is misplaced. The issuance of subsequent resolutions by the Court is It must be clarified, however, that since the restriction only pertains to "any role in
simply an exercise of judicial power under Art. VIII of the Constitution, because the the implementation or enforcement of the law," Congress may still exercise its
execution of the Decision is but an integral part of the adjudicative function of the oversight function which is a mechanism of checks and balances that the
Court. None of the agencies ever questioned the power of the Court to implement Constitution itself allows. But it must be made clear that Congress’ role must be
the December 18, 2008 Decision nor has any of them raised the alleged confined to mere oversight. Any post-enactment-measure allowing legislator
encroachment by the Court over executive functions. While additional activities are participation beyond oversight is bereft of any constitutional basis and hence,
required of the agencies like submission of plans of action, data or status reports, tantamount to impermissible interference and/or assumption of executive
these directives are but part and parcel of the execution stage of a final decision functions.
under Rule 39 of the Rules of Court.
As the Court ruled in Abakada: Any post-enactment congressional measure x x x
Belgica vs. Ochoa, Jr, G.R. No. 208566, November 19, 2013 (November 11 should be limited to scrutiny and investigation. In particular, congressional
in SC website) oversight must be confined to the following:

ISSUE: Whether or not the 2013 PDAF Article and all other Congressional Pork (1) Scrutiny based primarily on Congress’ power of appropriation and the budget
Barrel Laws similar thereto are unconstitutional considering that they violate the hearings conducted in connection with it, its power to ask heads of departments to
principles of constitutional provisions on (a) separation of powers. appear before and be heard by either of its Houses on any matter pertaining to
their departments and its power of confirmation; and

M.R.A.D.C. LUMBRE 29
CONSTITUTIONAL LAW REVIEW

(2) Investigation and monitoring of the implementation of laws pursuant to the 6770, militates against an interpretation that would insulate the Deputy
power of Congress to conduct inquiries in aid of legislation. Any action or step Ombudsman from the disciplinary authority of the OP and yet expose the
beyond that will undermine the separation of powers guaranteed by the Special Prosecutor to the same ills that a grant of independence to the
Constitution. Office of the Ombudsman was designed for.
i. Principle of checks and balances Congress recognized the importance of the Special Prosecutor as a
necessary adjunct of the Ombudsman, aside from his or her deputies, by
This allows one department to resist encroachments upon its prerogatives or making the Office of the Special Prosecutor an organic component of the
to rectify mistakes or excesses committed by the other departments, e.g., veto Office of the Ombudsman and by granting the Ombudsman control and
power of the President as check on improvident legislation, etc. supervision over that office. This power of control and supervision includes
1. Gonzales III vs. Office of the President, G.R. No. 196231, vesting the Office of the Ombudsman with the power to assign duties to
January 28, 2014 the Special Prosecutor as he/she may deem fit. Thus, by constitutional
design, the Special Prosecutor is by no means an ordinary subordinate but
RULING: Section 8(2) of RA No. 6770 vesting disciplinary authority in the one who effectively and directly aids the Ombudsman in the exercise of
President over the Deputy Ombudsman violates the independence of the his/her duties, which include investigation and prosecution of officials in
Office of the Ombudsman and is thus unconstitutional. the Executive Department.
Our discussions, particularly the Court’s expressed caution against Under Section 11(4) of RA No. 6770, the Special Prosecutor handles the
presidential interference with the constitutional commissions, on one prosecution of criminal cases within the jurisdiction of the Sandiganbayan
hand, and those expressed by the framers of the 1987 Constitution, on and this prosecutorial authority includes high-ranking executive officials.
the other, in protecting the independence of the Constitutional For emphasis, subjecting the Special Prosecutor to disciplinary and
Commissions, speak for themselves as overwhelming reasons to invalidate removal powers of the President, whose own alter egos and officials in the
Section 8(2) of RA No. 6770 for violating the independence of the Office Executive Department are subject to the prosecutorial authority of the
of the Ombudsman. Special Prosecutor, would seriously place the independence of the Office
of the Ombudsman itself at risk.
In more concrete terms, we rule that subjecting the Deputy Ombudsman
to discipline and removal by the President, whose own alter egos and Thus, even if the Office of the Special Prosecutor is not expressly made
officials in the Executive Department are subject to the Ombudsman’s part of the composition of the Office of the Ombudsman, the role it
disciplinary authority, cannot but seriously place at risk the independence performs as an organic component of that Office militates against a
of the Office of the Ombudsman itself. The Office of the Ombudsman, by differential treatment between the Ombudsman’s Deputies, on one hand,
express constitutional mandate, includes its key officials, all of them and the Special Prosecutor himself, on the other. What is true for the
tasked to support the Ombudsman in carrying out her mandate. Ombudsman must be equally true, not only for her Deputies but, also for
Unfortunately, intrusion upon the constitutionally-granted independence other lesser officials of that Office who act directly as agents of the
is what Section 8(2) of RA No. 6770 exactly did. By so doing, the law Ombudsman herself in the performance of her duties.
directly collided not only with the independence that the Constitution
guarantees to the Office of the Ombudsman, but inevitably with the Pursuing the present line of reasoning, when one considers that by express
principle of checks and balances that the creation of an Ombudsman office mandate of paragraph 8, Section 13, Article XI of the Constitution, the
seeks to revitalize. Ombudsman may "exercise such other powers or perform functions or
duties as may be provided by law," it is indubitable then that Congress
What is true for the Ombudsman must be equally and necessarily true for has the power to place the Office of the Special Prosecutor under the Office
her Deputies who act as agents of the Ombudsman in the performance of of the Ombudsman.
their duties. The Ombudsman can hardly be expected to place her
complete trust in her subordinate officials who are not as independent as Thus, under the present Constitution, there is every reason to treat the
she is, if only because they are subject to pressures and controls external Special Prosecutor to be at par with the Ombudsman's deputies, at least
to her Office. This need for complete trust is true in an ideal setting and insofar as an extraneous disciplinary authority is concerned, and must also
truer still in a young democracy like the Philippines where graft and enjoy the same grant of independence under the Constitution.
corruption is still a major problem for the government. For these reasons, ii. Principle of blending of powers
Section 8(2) of RA No. 6770 (providing that the President may remove a
Deputy Ombudsman) should be declared void. Instances when powers are not confined exclusively within one department but
are assigned to or shared by several departments, e.g., enactment of general
Given this legislative history, the present overall legal structure of the appropriations law.
Office of the Ombudsman, both under the 1987 Constitution and RA No.

M.R.A.D.C. LUMBRE 30
CONSTITUTIONAL LAW REVIEW

iii. Political question vs. Justiciable question delegate. On the other hand, the second test is called the "sufficient
standard test." Jurisprudence holds that a law lays down a sufficient standard
“A purely justiciable question implies a given right, legally demandable and
when it provides adequate guidelines or limitations in the law to map out the
enforceable, an act or omission violative of such right, and a remedy granted
and sanctioned by law for said breach of right. In Tatad v. Secretary of Energy, boundaries of the delegate‘s authority and prevent the delegation from running
the Supreme Court ruled that what the petitioners raised were justiciable riot. To be sufficient, the standard must specify the limits of the delegate‘s
questions, considering that the “statement of facts and definition of issues authority, announce the legislative policy, and identify the conditions under
clearly show that the petitioners are assailing R.A. 8180 because its provisions which it is to be implemented.
infringe the Constitution and not because the law lacks wisdom”. In Tanada v.
In view of the foregoing, the Court agrees with petitioners that the phrase
Angara, supra., the petition seeking the nullification of the Senate concurrence
of the President’s ratification of the Agreement establishing the World Trade "and for such other purposes as may be hereafter directed by the
Organization (WTO), was held to present a justiciable controversy, because President" under Section 8 of PD 910 constitutes an undue delegation
where an action is alleged to infringe the Constitution, it becomes not only the of legislative power insofar as it does not lay down a sufficient
right but the duty of the judiciary to settle the dispute. standard to adequately determine the limits of the President‘s
authority with respect to the purpose for which the Malampaya Funds
The term 'political question’ connotes what it means in ordinary parlance,
may be used. As it reads, the said phrase gives the President wide latitude to
namely a question of policy. It refers to those questions which, under the
use the Malampaya Funds for any other purpose he may direct and, in effect,
Constitution, are to be decided by the people in their sovereign capacity, or in
allows him to unilaterally appropriate public funds beyond the purview of the
regard to which full discretionary authority has been delegated to the
legislative or executive branch of government. It is concerned with issues law. That the subject phrase may be confined only to "energy resource
dependent upon the wisdom, not legality, of a particular measure.” development and exploitation programs and projects of the government" under
the principle of ejusdem generis, meaning that the general word or phrase
b. Principle of non-delegation of powers is to be construed to include – or be restricted to – things akin to, resembling,
“Potestas delegata non potest delegare”, based on the ethical principle that or of the same kind or class as those specifically mentioned, is belied by three
delegated power constitutes not only a right but a duty to be performed by the (3) reasons: first, the phrase "energy resource development and exploitation
delegate through the instrumentality of his own judgment and not through the programs and projects of the government" states a singular and general class
intervening mind of another. and hence, cannot be treated as a statutory reference of specific things from
which the general phrase "for such other purposes" may be limited; second,
i. Two (2) fundamental tests for adequate legislative guidelines for
the said phrase also exhausts the class it represents, namely energy
delegated rule-making –
development programs of the government; and, third, the Executive
Belgica vs. Ochoa, Jr, G.R. No. 208566, November 19, 2013 department has, in fact, used the Malampaya Funds for non-energy related
purposes under the subject phrase, thereby contradicting respondents‘ own
ISSUE: Whether or not Section 8 of PD 910 creating a special fund, known as
the Malampaya Funds, constitutes an undue delegation of legislative power position that it is limited only to "energy resource development and exploitation
since the phrase "and for such other purposes as may be hereafter directed by programs and projects of the government."
the President" gives the President "unbridled discretion to determine for what Thus, while Section 8 of PD 910 may have passed the completeness test since
purpose the funds will be used.”
the policy of energy development is clearly deducible from its text, the phrase
RULING: Yes. While the designation of a determinate or determinable amount "and for such other purposes as may be hereafter directed by the
for a particular public purpose is sufficient for a legal appropriation to exist, President" under the same provision of law should nonetheless be
the appropriation law must contain adequate legislative guidelines if stricken down as unconstitutional as it lies independently unfettered
the same law delegates rule-making authority to the Executive either by any sufficient standard of the delegating law. This notwithstanding, it
for the purpose of (a) filling up the details of the law for its must be underscored that the rest of Section 8, insofar as it allows for the use
enforcement, known as supplementary rule-making, or (b) of the Malampaya Funds "to finance energy resource development and
ascertaining facts to bring the law into actual operation, referred to as exploitation programs and projects of the government," remains legally
contingent rule-making. effective and subsisting. Truth be told, the declared unconstitutionality of the
aforementioned phrase is but an assurance that the Malampaya Funds would
There are two (2) fundamental tests to ensure that the legislative guidelines be used – as it should be used – only in accordance with the avowed purpose
for delegated rule-making are indeed adequate. The first test is called the and intention of PD 910.
"completeness test." Case law states that a law is complete when it sets
forth therein the policy to be executed, carried out, or implemented by the

M.R.A.D.C. LUMBRE 31
CONSTITUTIONAL LAW REVIEW

2. Adherence to International law (Art. II, Sec. 2) petitioner. The Confirmation of Sentence further states that pursuant to the 48th
and 49th Articles of War the sentence on MG Carlos Flores Garcia, AFP shall not be
Section 2. The Philippines … adopts the generally accepted principles of international remitted/mitigated by any previous confinement.
law as part of the law of the land and adheres to the policy of peace, equality, justice,
freedom, cooperation, and amity with all nations. Aggrieved, petitioner filed with this Court the present petition for certiorari and
petition for habeas corpus, alternatively. Garcia argued that the confirmation issued
a. Doctrine of Incorporation by the Office of the President directing his two-year detention in a penitentiary had
already been fully served following his preventive confinement subject to Article 29
By virtue of this clause, our Courts have applied the rules of international law in a
of the RPC (Revised Penal Code).
number of cases even if such rules had not previously been subject of statutory
enactments, because these generally accepted principles of international law are RULING: The Court upheld the authority of the President, as Commander-in-Chief,
automatically part of our own laws. to confirm the sentence. It held that the General Court Martial had jurisdiction over
the case since it was indisputable that Garcia was an officer in the active service of
AFP when he committed the violations until his arraignment. Garcia’s mandatory
3. Renunciation of war as an instrument of national policy (Art. II, Sec. 2) retirement on November 18, 2004 did not divest the General Court Martial of its
jurisdiction. Having established the jurisdiction of the General Court Martial over
Section 2. The Philippines renounces war as an instrument of national policy, …. the case and the person of the petitioner, the President, as Commander-in-Chief,
a. Relate with Art. VI, Sec. 23 (1) therefore acquired the jurisdiction to confirm petitioner's sentence as mandated
under Article 47 of the Articles of War The Court stressed that Article 48 of the
Section 23. The Congress, by a vote of two-thirds of both Houses in joint session Articles of War vests on the President, as Commander-in-Chief, the power to
assembled, voting separately, shall have the sole power to declare the existence of approve or disapprove the entire or any part of the sentence given by the court
a state of war. martial, also Article 49 of the same grants the President the power to mitigate or
remit a sentence.
b. Policy of freedom from nuclear weapons (Art. II, Sec. 8)
“Thus, the power of the President to confirm, mitigate and remit a sentence
Section 8. The Philippines, consistent with the national interest, adopts and
of erring military personnel is a clear recognition of the superiority of
pursues a policy of freedom from nuclear weapons in its territory. civilian authority over the military. However, although the law (Articles of War)
NOTE: Nuclear weapons made by the Philippines, but installed outside its territorial which conferred those powers to the President is silent as to the deduction of the
jurisdiction, is allowed. period of preventive confinement to the penalty imposed, as discussed earlier, such
is also the right of an accused provided for by Article 29 of the RPC.”

4. Civilian supremacy clause (Art. II, Sec. 3)


5. Doctrine of Separation of the Church and the State (Art. II, Sec. 6)
Section 3. Civilian authority is, at all times, supreme over the military. The Armed
Section 6. The separation of Church and State shall be inviolable.
Forces of the Philippines is the protector of the people and the State. Its goal is to
secure the sovereignty of the State and the integrity of the national territory. a. Relate with Art. XIV, Sec. 3 (3)
a. Garcia vs. Executive Secretary, GR No. 198554, 30 July 2012 Section 3. xxxxx At the option expressed in writing by the parents or guardians,
religion shall be allowed to be taught to their children or wards in public elementary
FACTS: Major General Carlos F. Garcia (ret) was convicted for violations of the
96th Article of War (Conduct Unbecoming an Officer and Gentleman) and 97th and high schools within the regular class hours by instructors designated or
Article of War (Conduct Prejudicial to Good Order and Military Discipline) for approved by the religious authorities of the religion to which the children or wards
knowingly, wrongfully and unlawfully fail to disclose/declare and make untruthful belong, without additional cost to the Government.
statements under oath of all his existing assets in his Sworn Statement of Assets
and Liabilities and Net worth (SALN) for the years 2003 and 2002 as required by
RA 3019, as amended in relation to RA 6713. 6. Social Justice
After six (6) years and two (2) months of preventive confinement, on December a. New or Expanded meaning (Art II, Sec. 10 and Art. XIII, Sec. 1)
16, 2010, petitioner was released from the Camp Crame Detention Center.
Section 10. The State shall promote social justice in all phases of national
On September 9, 2011, The Office of the President, or the President as development. (Article II)
Commander-in-Chief of the AFP and acting as the Confirming Authority under the
Articles of War, confirmed the sentence imposed by the Court Martial against

M.R.A.D.C. LUMBRE 32
CONSTITUTIONAL LAW REVIEW

Section 1. The Congress shall give highest priority to the enactment of measures 8. Equal access to opportunity for public service and prohibition against
that protect and enhance the right of all the people to human dignity, reduce social, political dynasties (Art. II, Section 26)
economic, and political inequalities, and remove cultural inequities by equitably
Section 26. The State shall guarantee equal access to opportunities for public service
diffusing wealth and political power for the common good. and prohibit political dynasties as may be defined by law.
To this end, the State shall regulate the acquisition, ownership, use, and disposition
of property and its increments. (Article XIII)
9. Education

a. Free and compulsory elementary education (Art. XIV, Sec. 2[2])


7. The right of the people to a balanced and healthful ecology (Art. II, Sec. 16)
Section 2. The State shall:
Section 16. The State shall protect and advance the right of the people to a balanced
and healthful ecology in accord with the rhythm and harmony of nature. xxxx

a. Oposa vs. Factoran, Jr., 224 SCRA 792 [1993] Establish and maintain, a system of free public education in the elementary and
high school levels. Without limiting the natural rights of parents to rear their
FACTS: A taxpayer’s class suit was filed by minors Juan Antonio Oposa, et al., children, elementary education is compulsory for all children of school age;
representing their generation and generations yet unborn, and represented by their
parents against Fulgencio Factoran Jr., Secretary of DENR. They prayed that b. Optional study of religion in public elementary and high schools (Art.
judgment be rendered ordering the defendant, his agents, representatives and XIV, Sec. 3[3])
other persons acting in his behalf to: 1. Cancel all existing Timber Licensing
Agreements (TLA) in the country; 2. Cease and desist from receiving, accepting, Section 3. At the option expressed in writing by the parents or guardians, religion
processing, renewing, or appraising new TLAs; and granting the plaintiffs “such shall be allowed to be taught to their children or wards in public elementary and
other reliefs just and equitable under the premises.” They alleged that they have a high schools within the regular class hours by instructors designated or approved
clear and constitutional right to a balanced and healthful ecology and are entitled by the religious authorities of the religion to which the children or wards belong,
to protection by the State in its capacity as parens patriae. Furthermore, they claim without additional cost to the Government.
that the act of the defendant in allowing TLA holders to cut and deforest the
remaining forests constitutes a misappropriation and/or impairment of the natural c. Academic Freedom (Art. XIV, Sec. 5[2])
resources property he holds in trust for the benefit of the plaintiff minors and i. Cadet 1CL Cudia vs. The Superintendent of the PMA, G.R. No. 211362,
succeeding generations. 24 February 2015
RULING: The Supreme Court ruled that they can, for themselves, for others of ISSUE: Whether the PMA can validly dismiss Cudia based on its findings.
their generation, and for the succeeding generation, file a class suit. Their
personality to sue in behalf of succeeding generations is based on the concept of RULING: Yes. It is within PMA’s right to academic freedom to decide whether
intergenerational responsibility insofar as the right to a balanced and healthful or not a cadet is still worthy to be part of the institution.
ecology is concerned. Such a right considers the “rhythm and harmony of nature” Cudia would argue that there is no law providing that a guilty finding by the
which indispensably include, inter alia, the judicious disposition, utilization, HC may be used by the PMA to dismiss or recommend the dismissal of a cadet
management, renewal and conservation of the country’s forest, mineral, land, from the PMA; that Honor Code violation is not among those listed as
waters, fisheries, wildlife, offshore areas and other natural resources to the end justifications for the attrition of cadets considering that the Honor Code and
that their exploration, development, and utilization be equitably accessible to the the Honor System (manner which PMA conducts investigation of Honor Code
present as well as the future generations. violations) do not state that a guilty cadet is automatically terminated or
dismissed from service.
Needless to say, every generation has a responsibility to the next to preserve that
rhythm and harmony for the full enjoyment of a balanced and healthful ecology. Such argument is not valid. Even without express provision of a law, the PMA
has regulatory authority to administratively dismiss erring cadets. Further,
Put a little differently, the minor’s assertion of their right to a sound environment
there is a law (Commonwealth Act No. 1) authorizing the President to dismiss
constitutes at the same time, the performance of their obligation to ensure the
cadets. Such power by the President may be delegated to the PMA
protection of that right for the generations to come. Superintendent, who may exercise direct supervision and control over the
cadets.

Further, as stated earlier, such power by the PMA is well within its academic
freedom. Academic freedom or, to be precise, the institutional autonomy of

M.R.A.D.C. LUMBRE 33
CONSTITUTIONAL LAW REVIEW

universities and institutions of higher learning has been enshrined in the The prescription shall be interrupted when proceedings are instituted against
Constitution. the guilty person and shall begin to run again if the proceedings are dismissed
for reasons not constituting double jeopardy.
The essential freedoms of academic freedom on the part of schools are as
follows: a. the right to determine who may teach; b. the right to determine This simply means that if the commission of the crime is known, the prescriptive
what may be taught; c. the right to determine how it shall be taught; d. the period shall commence to run on the day it was committed.
right to determine who may be admitted to study.
In the present case, it was well-nigh impossible for the State, the aggrieved party,
The Honor Code is just but one way for the PMA to exercise its academic to have known the violations of R.A. No. 3019 at the time the questioned
freedom. If it determines that a cadet violates it, then it has the right to dismiss transactions were made because, as alleged, the public officials concerned connived
said cadet. In this case, based on its findings, Cudia lied – which is a violation or conspired with the "beneficiaries of the loans." Thus, we agree with the
of the Honor Code. COMMITTEE that the prescriptive period for the offenses with which the
respondents in OMB-0-96-0968 were charged should be computed from the
But Cudia’s lie is not even that big; is dismissal from the PMA really warranted? discovery of the commission thereof and not from the day of such commission.
The PMA Honor Code does not distinguish between a big lie and a minor lie. It Since the computation of the prescriptive period for the filing of the criminal action
punishes any form of lying. It does not have a gradation of penalties. In fact, should commence from the discovery of the offense, the OMBUDSMAN clearly acted
it is the discretion of the PMA as to what penalty may be imposed. When Cudia with grave abuse of discretion in dismissing outright Case No. OMB-0-96-0968. It
enrolled at PMA, he agreed to abide by the Honor Code and the Honor System. should have first received the evidence from the complainant and the respondents
Thus, while the punishment may be severe, it is nevertheless reasonable and to resolve the case on its merits and on the issue of the date of discovery of the
not arbitrary, and, therefore, not in violation of due process - also considering offense.
that Cudia, as a cadet, must have known all of these.

11. Regalian Doctrine (Art. XII, Sec. 2 relate with Art. XII, Sec. 5 and Art. II,
10. The right of the States to recover properties unlawfully acquired by public Sec. 22)
officials and employees (Art. XI, Sec. 15)
Section 2. All lands of the public domain, waters, minerals, coal, petroleum, and other
Section 15. The right of the State to recover properties unlawfully acquired by public mineral oils, all forces of potential energy, fisheries, forests or timber, wildlife, flora and
officials or employees, from them or from their nominees or transferees, shall not be fauna, and other natural resources are owned by the State. With the exception of
barred by prescription, laches, or estoppel. agricultural lands, all other natural resources shall not be alienated. The exploration,
a. Presidential Ad Hoc Fact-Finding Committee on Behest Loans, et. al. v. development, and utilization of natural resources shall be under the full control and
Desierto, G.R. No. 130140, Oct. 25, 1999 supervision of the State. The State may directly undertake such activities, or it may
enter into co-production, joint venture, or production-sharing agreements with Filipino
ISSUE: Whether public respondent Ombudsman Aniano A. Desierto committed citizens, or corporations or associations at least 60 per centum of whose capital is owned
grave abuse of discretion in holding that the offenses with which the other by such citizens. Such agreements may be for a period not exceeding twenty-five years,
respondents were charged in OMB-0-96-0968 had already prescribed. renewable for not more than twenty-five years, and under such terms and conditions
as may provided by law. In cases of water rights for irrigation, water supply, fisheries,
RULING: The upshot of the foregoing discussion is that the prosecution of offenses
or industrial uses other than the development of waterpower, beneficial use may be the
arising from, relating or incident to, or involving ill-gotten wealth contemplated in
measure and limit of the grant.
Section 15, Article XI of the Constitution may be barred by prescription.
The State shall protect the nation’s marine wealth in its archipelagic waters, territorial
Since the law alleged to have been violated, i.e., paragraphs (e) and (g) of Section
sea, and exclusive economic zone, and reserve its use and enjoyment exclusively to
3, R.A. No. 3019, as amended, is a special law, the applicable rule in the
Filipino citizens.
computation of the prescriptive period is Section 2 of Act No. 3326, [Entitled "An
Act to Establish Periods of Prescription for Violations Penalized by Special Acts and The Congress may, by law, allow small-scale utilization of natural resources by Filipino
Municipal Ordinances to Provide When Prescription Shall Begin to Run."] as citizens, as well as cooperative fish farming, with priority to subsistence fishermen and
amended, which provides: fish workers in rivers, lakes, bays, and lagoons.
Sec. 2. Prescription shall begin to run from the day of the commission of the The President may enter into agreements with foreign-owned corporations involving
violation of the law, and if the same be not known at the time, from the either technical or financial assistance for large-scale exploration, development, and
discovery thereof and institution of judicial proceedings for its investigation utilization of minerals, petroleum, and other mineral oils according to the general terms
and punishment. and conditions provided by law, based on real contributions to the economic growth and
general welfare of the country. In such agreements, the State shall promote the

M.R.A.D.C. LUMBRE 34
CONSTITUTIONAL LAW REVIEW

development and use of local scientific and technical resources. The Congress may found within said areas, Sections 3(a) and 3(b) violate the rights of private
provide for the applicability of customary laws governing property rights or relations in landowners.
determining the ownership and extent of ancestral domain.
In addition, petitioners question the provisions of the IPRA defining the powers and
The President shall notify the Congress of every contract entered into in accordance jurisdiction of the NCIP and making customary law applicable to the settlement of
with this provision, within thirty days from its execution. (Article XII) disputes involving ancestral domains and ancestral lands on the ground that these
provisions violate the due process clause of the Constitution.
Section 5. The State, subject to the provisions of this Constitution and national
development policies and programs, shall protect the rights of indigenous cultural RULING: Seven (7) voted to dismiss the petition. Justice Kapunan filed an opinion,
communities to their ancestral lands to ensure their economic, social, and cultural well- which the Chief Justice and Justices Bellosillo, Quisumbing, and Santiago join,
being. (Article XII) sustaining the validity of the challenged provisions of R.A. 8371. Justice Puno also
filed a separate opinion sustaining all challenged provisions of the law with the
Section 22. The State recognizes and promotes the rights of indigenous cultural exception of Section 1, Part II, Rule III of NCIP Administrative Order No. 1, series
communities within the framework of national unity and development. (Article II) of 1998, the Rules and Regulations Implementing the IPRA, and Section 57 of the
a. Isagani Cruz v. Sec. of DENR, G.R. No. 135385, Dec. 6, 2000 IPRA which he contends should be interpreted as dealing with the large-scale
exploitation of natural resources and should be read in conjunction with Section 2,
FACTS: Petitioners assail the constitutionality of the following provisions of the Article XII of the 1987 Constitution. On the other hand, Justice Mendoza voted to
IPRA and its Implementing Rules on the ground that they amount to an unlawful dismiss the petition solely on the ground that it does not raise a justiciable
deprivation of the State's ownership over lands of the public domain as well as controversy and petitioners do not have standing to question the constitutionality
minerals and other natural resources therein, in violation of the regalian doctrine of R.A. 8371.
embodied in Section 2, Article XII of the Constitution:
Seven (7) other members of the Court voted to grant the petition. Justice
"(1) Section 3(a) which defines the extent and coverage of ancestral domains, Panganiban filed a separate opinion expressing the view that Sections 3 (a)(b), 5,
and Section 3(b) which, in turn, defines ancestral lands; 6, 7 (a)(b), 8, and related provisions of R.A. 8371 are unconstitutional. He reserves
judgment on the constitutionality of Sections 58, 59, 65, and 66 of the law, which
"(2) Section 5, in relation to section 3(a), which provides that ancestral
he believes must await the filing of specific cases by those whose rights may have
domains including inalienable public lands, bodies of water, mineral and other
been violated by the IPRA. Justice Vitug also filed a separate opinion expressing
resources found within ancestral domains are private but community property
the view that Sections 3(a), 7, and 57 of R.A. 8371 are unconstitutional. Justices
of the indigenous peoples;
Melo, Pardo, Buena, Gonzaga-Reyes, and De Leon join in the separate opinions of
"(3) Section 6 in relation to section 3(a) and 3(b) which defines the Justices Panganiban and Vitug.
composition of ancestral domains and ancestral lands;
As the votes were equally divided (7 to 7) and the necessary majority was not
"(4) Section 7 which recognizes and enumerates the rights of the indigenous obtained, the case was redeliberated upon. However, after redeliberation, the
peoples over the ancestral domains; voting remained the same. Accordingly, pursuant to Rule 56, Section 7 of the Rules
of Civil Procedure, the petition is DISMISSED. The subject IPRA provisions are
(5) Section 8 which recognizes and enumerates the rights of the indigenous constitutional.
peoples over the ancestral lands;

"(6) Section 57 which provides for priority rights of the indigenous peoples in
the harvesting, extraction, development or exploration of minerals and other 12. Nationalist provisions: (Art. II, Sec. 19)
natural resources within the areas claimed to be their ancestral domains, and
Section 19. The State shall develop a self-reliant and independent national economy
the right to enter into agreements with nonindigenous peoples for the
effectively controlled by Filipinos.
development and utilization of natural resources therein for a period not
exceeding 25 years, renewable for not more than 25 years; and a. Utilization of natural resources (Art. XII, Sec. 2)
"(7) Section 58 which gives the indigenous peoples the responsibility to Section 2. xxx The exploration, development, and utilization of natural resources
maintain, develop, protect and conserve the ancestral domains and portions shall be under the full control and supervision of the State. The State may directly
thereof which are found to be necessary for critical watersheds, mangroves, undertake such activities, or it may enter into co-production, joint venture, or
wildlife sanctuaries, wilderness, protected areas, forest cover or reforestation.” production-sharing agreements with Filipino citizens, or corporations or
associations at least 60 per centum of whose capital is owned by such citizens.
Petitioners also contend that, by providing for an all-encompassing definition of
Such agreements may be for a period not exceeding twenty-five years, renewable
"ancestral domains" and "ancestral lands" which might even include private lands
for not more than twenty-five years, and under such terms and conditions as may
provided by law. In cases of water rights for irrigation, water supply, fisheries, or

M.R.A.D.C. LUMBRE 35
CONSTITUTIONAL LAW REVIEW

industrial uses other than the development of waterpower, beneficial use may be Petitioners filed the present Petitions for Certiorari, Mandamus, and Injunction
the measure and limit of the grant. to enjoin respondents from implementing SC-46 and to have it nullified for
willful and gross violation of the 1987 Constitution and certain international
The State shall protect the nation’s marine wealth in its archipelagic waters, and municipal laws.
territorial sea, and exclusive economic zone, and reserve its use and enjoyment
exclusively to Filipino citizens. RULING: In La Bugal-B’laan Tribal Association, Inc. v. Ramos, we held that
the deletion of the words “service contracts” in the 1987 Constitution did not
The Congress may, by law, allow small-scale utilization of natural resources by amount to a ban on them per se. In fact, in that decision, we quoted in length,
Filipino citizens, as well as cooperative fish farming, with priority to subsistence portions of the deliberations of the members of the Constitutional Commission
fishermen and fish workers in rivers, lakes, bays, and lagoons. (ConCom) to show that in deliberating on paragraph 4, Section 2, Article XII,
The President may enter into agreements with foreign-owned corporations they were actually referring to service contracts as understood in the 1973
involving either technical or financial assistance for large-scale exploration, Constitution, albeit with safety measures to eliminate or minimize the abuses
development, and utilization of minerals, petroleum, and other mineral oils prevalent during the martial law regime.
according to the general terms and conditions provided by law, based on real The phrase “agreements involving either technical or financial assistance”,
contributions to the economic growth and general welfare of the country. In such referred to in paragraph 4, Section 2, Article XII of the 1987 Constitution are
agreements, the State shall promote the development and use of local scientific in fact service contracts. But unlike those of the 1973 variety, the new ones
and technical resources. The Congress may provide for the applicability of are between foreign corporations acting as contractors on the one hand; and
customary laws governing property rights or relations in determining the ownership on the other, the government as principal or “owner” of the works. In the new
and extent of ancestral domain. service contracts, the foreign contractors provide capital, technology and
The President shall notify the Congress of every contract entered into in accordance technical know-how, and managerial expertise in the creation and operation of
with this provision, within thirty days from its execution. large-scale mining/extractive enterprises; and the government, through its
agencies (DENR, MGB), actively exercises control and supervision over the
i. Resident Marine Mammals vs. Sec. Angelo Reyes, G.R. No. 180771, entire operation.
21 April 2015
Such service contracts may be entered into only with respect to minerals,
FACTS: On June 13, 2002, the Government of the Philippines, acting through petroleum and other mineral oils. The grant thereof is subject to several
the Department of Energy (DOE), entered into a Geophysical Survey and safeguards, among which are these requirements:
Exploration Contract-102 (GSEC-102) with Japan Petroleum Exploration Co.,
Ltd. (JAPEX), a Japanese company. This contract involved geological and (i) The service contract shall be crafted in accordance with a general law that
geophysical studies of the Tañon Strait. will set standard or uniform terms, conditions and requirements, presumably
to attain a certain uniformity in provisions and avoid the possible insertion of
On December 21, 2004, DOE and JAPEX formally converted GSEC-102 into terms disadvantageous to the country.
Service Contract No. 46 (SC-46), which allowed the exploration, development,
and exploitation of petroleum resources in a block covering approximately (ii) The President shall be the signatory for the government because,
2,850 square kilometers offshore the Tañon Strait. supposedly before an agreement is presented to the President for signature, it
will have been vetted several times over at different levels to ensure that it
From May 9 to 18, 2005, JAPEX conducted seismic surveys in and around the conforms to law and can withstand public scrutiny.
Tañon Strait. JAPEX committed to drill one exploration well during the second
sub-phase of the project. Since the well was to be drilled in the marine waters (iii) Within thirty days of the executed agreement, the President shall report it
of Aloguinsan and Pinamungajan, where the Tañon Strait was declared a to Congress to give that branch of government an opportunity to look over the
protected seascape in 1988, JAPEX agreed to comply with the Environmental agreement and interpose timely objections, if any.
Impact Assessment (EIA) requirements pursuant to Presidential Decree No. SC-46 is null and void for noncompliance with the requirements of the 1987
1586. Constitution.
Having obtained the required Environmental Compliance Certificate (ECC) from While the Court finds that Presidential Decree No. 87 is sufficient to satisfy the
the Environmental Management Bureau (EMB) of the Department of requirement of a general law, the absence of the two other conditions, that the
Environment and Natural Resources (DENR), JAPEX began to drill an President be a signatory to SC-46, and that Congress be notified of such contract,
exploratory well, with a depth of 3,150 meters, near Pinamungajan town in the renders it null and void.
western Cebu Province. The drilling began on November 16, 2007 and lasted
until February 8, 2008. The disposition, exploration, development, exploitation, and utilization of
indigenous petroleum in the Philippines are governed by Presidential Decree No.
87 or the Oil Exploration and Development Act of 1972. This was enacted by then

M.R.A.D.C. LUMBRE 36
CONSTITUTIONAL LAW REVIEW

President Ferdinand Marcos to promote the discovery and production of indigenous While the requirements in executing service contracts in paragraph 4, Section 2
petroleum through the utilization of government and/or local or foreign private of Article XII of the 1987 Constitution seem like mere formalities, they, in reality,
resources to yield the maximum benefit to the Filipino people and the revenues to take on a much bigger role. As we have explained in La Bugal, they are the
the Philippine Government. PD No. 87, although enacted in 1972, before the safeguards put in place by the framers of the Constitution to “eliminate or
adoption of the 1987 Constitution, remains to be a valid law unless otherwise minimize the abuses prevalent during the martial law regime.” Thus, they are not
repealed. just mere formalities, which will only render a contract unenforceable but not void,
if not complied with. They are requirements placed, not just in an ordinary statute,
Likewise, the Court could not simply assume that PD No. 87 had been impliedly but in the fundamental law, the non -observance of which will nullify the contract.
repealed. Implied repeals are not lightly presumed. It is a settled rule that when
laws are in conflict with one another, every effort must be exerted to reconcile Our Constitution requires that the President himself be the signatory of service
them. Moreover, in cases where the statute seems to be in conflict with the agreements with foreign-owned corporations involving the exploration,
Constitution, but a construction that it is in harmony with the Constitution is also development, and utilization of our minerals, petroleum, and other mineral oils.
possible, that construction should be preferred. This power cannot be taken lightly. In contrast, under PD No. 87, it is required
that the Petroleum Board, now the DOE, obtain the President’s approval for the
Consequently, we find no merit in petitioners’ contention that SC-46 is prohibited execution of any contract under said statute. Even if we were inclined to relax the
on the ground that there is no general law prescribing the standard or uniform requirement in La Bugal, it must be shown that the government agency or
terms, conditions, and requirements for service contracts involving oil exploration subordinate official has been authorized by the President to enter into such service
and extraction. contract for the government. Otherwise, it should be at least shown that the
However, while PD No. 87 may serve as the general law upon which a service President subsequently approved of such contract explicitly. None of these
contract for petroleum exploration and extraction may be authorized, the circumstances is evident in the case at bar.
exploitation and utilization of this energy resource in the present case may be NOTE: As provided by the Constitution, only the State and its citizens (natural or
allowed only through a law passed by Congress, since the Tañon Strait is a NIPAS juridical) may exploit the country’s natural resources. However, a 100% foreign-
area. owned corporation may be allowed to explore or exploit jointly with the State, but
As SC-46 was executed in 2004, its terms should have conformed not only to the the following requisites (the first 3 of which are stated in the La Bugal case) must
provisions of Presidential Decree No. 87, but also to those of the 1987 be complied with:
Constitution. It is basic that the law is deemed written into every contract. 1. There must be a general law granting such;
Although a contract is the law between the parties, the provisions of positive law
which regulate contracts are deemed written therein and shall limit and govern 2. The President should be the signatory (or the same must be signed by
the relations between the parties. his Secretary or alter-ego, with his consent, based on the doctrine of qualified
agency or the alter-ego doctrine, which provides: “All acts of the heads of
Paragraph 4, Section 2, Article XII of the 1987 Constitution requires that the executive departments, unless disapproved by the President, are deemed to
President himself enter into any service contract for the exploration of petroleum. be acts of the President.”);
SC -46 appeared to have been entered into and signed only by the DOE through
its then Secretary, Vicente S. Perez, Jr., contrary to the said constitutional 3. The contract/agreement must be reported to Congress within 30 days
requirement. Moreover, public respondents have neither shown nor alleged that from its execution;
Congress was subsequently notified of the execution of such contract.
4. The Contract must be a Financial and Technical Assistance Agreement
The argument that, based on the “alter ego principle,” the [Energy Secretary's] (FTAA);
acts are also that of then President Macapagal-Arroyo’s, cannot apply in this case.
5. The same must only refer to:
The alter ego principle or the doctrine of qualified political agency recognizes the
establishment of a single executive, all executive and administrative organizations a. Minerals;
are adjuncts of the Executive Department, the heads of the various executive b. Petroleum;
departments are assistants and agents of the Chief Executive, and, except in cases
where the Chief Executive is required by the Constitution or law to act in person c. Other mineral oils.
or the exigencies of the situation demand that he act personally, the multifarious
As to the validity of P.D. No. 87 (i.e. that it was not repealed by the promulgation
executive and administrative functions of the Chief Executive are performed by
of the 1987 Constitution), the same must be related to Art. XVIII, Sec. 3, that:
and through the executive departments, and the acts of the Secretaries of such
“All existing laws, decrees, executive orders, proclamations, letters of instructions,
departments, performed and promulgated in the regular course of business, are,
and other executive issuances not inconsistent with this Constitution shall remain
unless disapproved or reprobated by the Chief Executive presumptively the acts
operative until amended, repealed, or revoked.”
of the Chief Executive.

M.R.A.D.C. LUMBRE 37
CONSTITUTIONAL LAW REVIEW

b. Franchise, certificate and authority for public utilities (Art.XII, Sec.11) (2) participation of foreign investors in its board of directors is limited to
their proportionate share in its capital; and
i. Roy III vs. Chairperson Herbosa, G.R. No. 207246, 22 November
2016 (3) all its executive and managing officers must be citizens of the
Philippines.
FACTS: Roy, as a lawyer and taxpayer, filed the Petition, assailing the validity
of SEC-MC No. 8 for not conforming to the letter and spirit of the Gamboa In the exhaustive review made by the Court in the Gamboa Resolution of the
Decision and Resolution and for having been issued by the SEC with grave deliberations of the Constitutional Commission, the opinions of the framers of
abuse of discretion. Petitioner Roy seeks to apply the 60-40 Filipino ownership the 1987 Constitution, the opinions of the SEC and the DOJ as well as the
requirement separately to each class of shares of a public utility corporation, provisions of the FIA, its implementing rules and its predecessor statutes, the
whether common, preferred nonvoting, preferred voting or any other class of intention to apply the voting control test and the beneficial ownership test was
shares. Petitioner Roy also questions the ruling of the SEC that respondent not mentioned in reference to "each class of shares." Even the Gamboa
Philippine Long Distance Telephone Company ("PLDT") is compliant with the Decision was silent on this point.
constitutional rule on foreign ownership. He prays that the Court declare SEC-
MC No. 8 unconstitutional and direct the SEC to issue new guidelines regarding 60% pertains only to shares with voting rights.
the determination of compliance with Section 11, Article XII of the Constitution c. Ownership / acquisition of lands (Art. XII, Secs. 7 and 8)
in accordance with Gamboa.
Section 7. Save in cases of hereditary succession, no private lands shall be
RULING: Petition denied. To reiterate, the "beneficial owner or beneficial transferred or conveyed except to individuals, corporations, or associations
ownership" definition in the SRC-IRR is understood only in determining the qualified to acquire or hold lands of the public domain.
respective nationalities of the outstanding capital stock of a public utility
corporation in order to determine its compliance with the percentage of Filipino Section 8. Notwithstanding the provisions of Section 7 of this Article, a natural-
ownership required by the Constitution. born citizen of the Philippines who has lost his Philippine citizenship may be a
transferee of private lands, subject to limitations provided by law.
The restrictive re-interpretation of "capital" as insisted by the petitioners is
unwarranted. i. Philippine National Oil Company (PNOC) vs. Keppel Philippines
Holdings, Inc., G.R. No. 202050, 25 July 2016
Petitioners' insistence that the 60% Filipino equity requirement must be
applied to each class of shares is simply beyond the literal text and FACTS: Keppel filed a complaint for specific performance which was countered
contemplation of Section 11, Article XII of the 1987 Constitution, viz: by PNOC stating that Keppel's claims by contending that the agreement was
illegal for circumventing the constitutional prohibition against aliens holding
Sec. 11. No franchise, certificate, or any other form of authorization for lands in the Philippines. It further asserted that the option contract was void,
the operation of a public utility shall be granted except to citizens of the as it was unsupported by a separate valuable consideration. It also claimed
Philippines or to corporations or associations organized under the laws of that it was not privy to the agreement. RTC rendered in favor of Keppel and
the Philippines at least sixty per centum or whose capital is owned by such PNOC was ordered to execute a deed of absolute sale upon payment of Keppel.
citizens, nor shall such franchise, certificate or authorization be exclusive PNOC elevated the case to the CA which affirmed the decision of the RTC.
in character or for a longer period than fifty years. Neither shall any such Failure to secure a reconsideration, hence, this petition.
franchise or right be granted except under the condition that it shall be
subject to amendment, alteration, or repeal by the Congress when the ISSUE: Whether the terms of the Agreement amounted to a virtual sale of the
common good so requires. The State shall encourage equity participation land to Keppel that was designed to circumvent the constitutional prohibition
in public utilities by the general public. The participation of foreign on aliens owning lands in the Philippines.
investors in the governing body of any public utility enterprise shall be
RULING: The agreement was executed to enable Keppel to use the land for
limited to their proportionate share in its capital, and all the executive and
its shipbuilding and ship repair business. The industrial/commercial purpose
managing officers of such corporation or association must be citizens of
behind the agreement differentiates the present case from Lui She where the
the Philippines.
leased property was primarily devoted to residential use.Undoubtedly, the
As worded, effective control by Filipino citizens of a public utility is already establishment and operation of a shipyard business involve significant
assured in the provision. With respect to a stock corporation engaged in the investments. Keppel's uncontested testimony showed that it incurred P60
business of a public utility, the constitutional provision mandates three million costs solely for preliminary activities to make the land suitable as a
safeguards: shipyard, and subsequently introduced improvements worth P177 million.
Taking these investments into account and the nature of the business that
(1) 60% of its capital must be owned by Filipino citizens; Keppel conducts on the land, we find it reasonable that the agreement's terms

M.R.A.D.C. LUMBRE 38
CONSTITUTIONAL LAW REVIEW

provided for an extended duration of the lease and a restriction on the rights Section 6. The national language of the Philippines is Filipino. As it evolves, it
of Lusteveco. shall be further developed and enriched on the basis of existing Philippine and
other languages.
We observe that, unlike in Lui She, Lusteveco was not completely denied its
ownership rights during the course of the lease. It could dispose of the lands ii. Official language (Art. XIV, Sec. 7 and Sec. 6, 2nd par.)
or assign its rights thereto, provided it secured Keppel's prior written consent.
That Lusteveco was able to convey the land in favour of PNOC during the Section 6. xxxx Subject to provisions of law and as the Congress may deem
pendency of the lease should negate a finding that the agreement's terms appropriate, the Government shall take steps to initiate and sustain the use of
amounted to a virtual transfer of ownership of the land to Keppel. Filipino as a medium of official communication and as language of instruction
in the educational system.
NOTE: Keppel followed the resolution in Gamboa v. Teves, decreeing that the
60% Filipino ownership requirement applies separately to each class of shares, Section 7. For purposes of communication and instruction, the official
whether with or without voting rights. Months following the promulgation of languages of the Philippines are Filipino and, until otherwise provided by law,
the Keppel decision, the Supreme Court reverted to the original definition of English.
“capital” in the original Gamboa v. Teves ruling, thus rendering the Keppel iii. Auxiliary official language (Art. XIV, Sec. 7, 2nd par.)
ruling useless.
Section 7. The regional languages are the auxiliary official languages in the
d. Practice of profession (Art. XII, Sec. 14, 2nd par.) regions and shall serve as auxiliary media of instruction therein.
Section 14. xxxx The State shall encourage appropriate technology and regulate iv. Voluntary or optional language (Art. XIV, Sec. 7, 3rd par.)
its transfer for the national benefit. The practice of all professions in the Philippines
shall be limited to Filipino citizens, save in cases prescribed by law. Section. 7. xxxx Spanish and Arabic shall be promoted on a voluntary and
optional basis.
e. Ownership, control, administration and establishment of educational
institutions (Art XIV, Sec. 4[2]) b. Language used in the promulgation of the Philippine Constitution (Art.
XIV, Sec. 8)
Section 4. xxxx Educational institutions, other than those established by religious
groups and mission boards, shall be owned solely by citizens of the Philippines or Section 8. This Constitution shall be promulgated in Filipino and English and shall
corporations or associations at least sixty per centum of the capital of which is be translated into major regional languages, Arabic, and Spanish.
owned by such citizens. The Congress may, however, require increased Filipino
equity participation in all educational institutions. The control and administration of
educational institutions shall be vested in citizens of the Philippines. 14. Philippine flag (Art. XVI, Sec. 1)
f. Ownership and management of mass media (Art. XVI, Sec. 11[1]) Section 1. The flag of the Philippines shall be red, white, and blue, with a sun and
Section 11. The ownership and management of mass media shall be limited to three stars, as consecrated and honored by the people and recognized by law.
citizens of the Philippines, or to corporations, cooperatives or associations, wholly-
owned and managed by such citizens.
15. Country’s name, national anthem and national seal (Art. XVI, Sec. 2)
g. Ownership of and control over businesses engaged in advertising
industry (Art. XVI, Sec. 11[2]) Section 2. The Congress may, by law, adopt a new name for the country, a national
anthem, or a national seal, which shall all be truly reflective and symbolic of the ideals,
The advertising industry is impressed with public interest, and shall be regulated history, and traditions of the people. Such law shall take effect only upon its ratification
by law for the protection of consumers and the promotion of the general welfare. by the people in a national referendum.
Only Filipino citizens or corporations or associations at least seventy per centum of
the capital of which is owned by such citizens shall be allowed to engage in the
advertising industry. 16. Military (Art. XVI, Secs. 4 and 5)
Section 4. The Armed Forces of the Philippines shall be composed of a citizen armed
force which shall undergo military training and serve as may be provided by law. It shall
13. Language keep a regular force necessary for the security of the State.
a. Classifications Section 5. All members of the armed forces shall take an oath or affirmation to uphold
i. National Language (Art. XIV, Sec. 6, 1st par.) and defend this Constitution.

M.R.A.D.C. LUMBRE 39
CONSTITUTIONAL LAW REVIEW

The State shall strengthen the patriotic spirit and nationalist consciousness of the
military, and respect for people's rights in the performance of their duty.

Professionalism in the armed forces and adequate remuneration and benefits of its
members shall be a prime concern of the State. The armed forces shall be insulated
from partisan politics. No member of the military shall engage, directly or indirectly, in
any partisan political activity, except to vote.

No member of the armed forces in the active service shall, at any time, be appointed
or designated in any capacity to a civilian position in the Government, including
government-owned or controlled corporations or any of their subsidiaries.
Laws on retirement of military officers shall not allow extension of their service.

The officers and men of the regular force of the armed forces shall be recruited
proportionately from all provinces and cities as far as practicable.

The tour of duty of the Chief of Staff of the armed forces shall not exceed three years.
However, in times of war or other national emergency declared by the Congress, the
President may extend such tour of duty.
a. Private armies – Art. XVIII, Sec. 24

Section 24. Private armies and other armed groups not recognized by duly
constituted authority shall be dismantled. All paramilitary forces including Civilian
Home Defense Forces not consistent with the citizen armed force established in this
Constitution, shall be dissolved or, where appropriate, converted into the regular
force.

17. Police force (Art. XVI, Sec. 6)

Section 6. The State shall establish and maintain one police force, which shall be
national in scope and civilian in character, to be administered and controlled by a
national police commission. The authority of local executives over the police units in
their jurisdiction shall be provided by law.

M.R.A.D.C. LUMBRE 40
CONSTITUTIONAL LAW REVIEW

V. Legislative Department Section 5. Each local government unit shall have the power to create its own
sources of revenues and to levy taxes, fees and charges subject to such
guidelines and limitations as the Congress may provide, consistent with the
1. Who may exercise legislative power basic policy of local autonomy. Such taxes, fees, and charges shall accrue
exclusively to the local governments.
NOTE: Contrary to what most political law books say, legislative power is primarily
exercised by, or vested in, the PEOPLE. However, when the people ratified the piece of d. Delegated Tariff Powers to the President (Art. VI, Sec. 28(2)
paper called the 1987 Constitution, they are deemed to have relinquished such power Section 28. xxxx The Congress may, by law, authorize the President to fix
to the Congress, except to the extent reserved by the provision on initiative and within specified limits, and subject to such limitations and restrictions as it
referendum. As such, it is not proper to say that the power of initiative and referendum may impose, tariff rates, import and export quotas, tonnage and wharfage
is a delegated power, but merely a reservation of the people’s original power. Moreover, dues, and other duties or imposts within the framework of the national
administrative issuances are not considered as an exercise of delegated power, as these development program of the Government.
are merely interpretations of an existing legislation, and thus have force and effect of
law. e. Delegated Emergency Powers (Art. VI, Sec. 23(2))

As a rule, what has been delegated cannot be delegated further, or “potestas delegata Section 23. xxxx In times of war or other national emergency, the Congress
non potest delegare,” and the exceptions are provided by the Constitution, as follows: may, by law, authorize the President, for a limited period and subject to such
restrictions as it may prescribe, to exercise powers necessary and proper to
1. Emergency Powers granted to the President under Art. VI, Sec. 23(2). When
carry out a declared national policy. Unless sooner withdrawn by resolution of
such powers are granted by Congress (through a law), the President may enact
the Congress, such powers shall cease upon the next adjournment thereof.
legislations to the extent that the purpose for which the powers are granted are
achieved. f. Lagman vs. Executive Secretary, G.R. No. 231658, 4 July 2017
2. Tariff powers under Art. VI, Sec. 28(2). FACTS: Effective May 23, 2017, and for a period not exceeding 60 days,
3. Local Government Units or the Sanggunians have the power to enact President Rodrigo Roa Duterte issued Proclamation No. 216 declaring a state
legislations, through ordinances, which affect the locality. of martial law and suspending the privilege of the writ of habeas corpus in the
whole of Mindanao.
4. Those delegated to the President by Congress:
Within the timeline set by Section 18, Article VII of the Constitution, the
a. Fixing tariff rates; President submitted to Congress on May 25, 2017, a written Report on the
factual basis of Proclamation No. 216.
b. Emergency powers (supra);
The Report pointed out that for decades, Mindanao has been plagued with
c. Martial law powers (legislative powers necessary and proper to carry out
rebellion and lawless violence which only escalated and worsened with the
the declared national policy of Congress.
passing of time.
a. Art VI, Sec. 1
The President went on to explain that on May 23, 2017, a government
Section 1. The legislative power shall be vested in the Congress of the operation to capture the high-ranking officers of the Abu Sayyaff Group (ASG),
Philippines which shall consist of a Senate and a House of Representatives, Senior Leader Isnilon Hapilon, and the Maute Group was conducted. These
except to the extent reserved to the people by the provision on initiative and groups, which have been unleashing havoc in Mindanao, however, confronted
referendum. the government operation by intensifying their efforts at sowing violence aimed
not only against the government authorities and its facilities but likewise
b. Art VI, Sec. 1 relate with Sec. 32 against civilians and their properties.
Section 32. The Congress shall, as early as possible, provide for a system of After the submission of the Report and the briefings, the Senate issued P.S.
initiative and referendum, and the exceptions therefrom, whereby the people Resolution No. 3888 expressing full support to the martial law proclamation
can directly propose and enact laws or approve or reject any act or law or part and finding Proclamation No. 216 "to be satisfactory, constitutional and in
thereof passed by the Congress or local legislative body after the registration accordance with the law". In the same Resolution, the Senate declared that it
of a petition therefor signed by at least ten per centum of the total number of found "no compelling reason to revoke the same".
registered voters, of which every legislative district must be represented by at
least three per centum of the registered voters thereof. The Senate's counterpart in the lower house shared the same sentiments. The
House of Representatives likewise issued House Resolution No. 105010
c. Delegated Legislative power to LGUs (Art. X, Sec. 5) "EXPRESSING THE FULL SUPPORT OF THE HOUSE OF REPRESENTATIVES TO

M.R.A.D.C. LUMBRE 41
CONSTITUTIONAL LAW REVIEW

PRESIDENT RODRIGO DUTERTE AS IT FINDS NO REASON TO REVOKE These extraordinary powers are conferred by the Constitution with the
PROCLAMATION NO. 216, ENTITLED 'DECLARING A STATE OF MARTIAL LAW President as Commander-in-Chief; it therefore necessarily follows that the
AND SUSPENDING THE PRIVILEGE OF THE WRIT OF HABEAS CORPUS IN THE power and prerogative to determine whether the situation warrants a mere
WHOLE OF MINDANAO"'. exercise of the calling out power; or whether the situation demands suspension
of the privilege of the writ of habeas corpus; or whether it calls for the
RULING: The President as the Commander-in-Chief wields the extraordinary declaration of martial law, also lies, at least initially, with the President. The
powers of: a) calling out the armed forces; b) suspending the privilege of the power to choose, initially, which among these extraordinary powers to wield in
writ of habeas corpus; and c) declaring martial law. These powers may be a given set of conditions is a judgment call on the part of the President. As
resorted to only under specified conditions. Commander-in-Chief, his powers are broad enough to include his prerogative
The framers of the 1987 Constitution reformulated the powers of the to address exigencies or threats that endanger the government, and the very
Commander-in-Chief by revising the "grounds for the activation of emergency integrity of the State.
powers, the manner of activating them, the scope of the powers, and review It is thus beyond doubt that the power of judicial review does not extend to
of presidential action." calibrating the President's decision pertaining to which extraordinary power to
Among the three extraordinary powers, the calling out power is the most avail given a set of facts or conditions. To do so would be tantamount to an
benign and involves ordinary police action. The President may resort to this incursion into the exclusive domain of the Executive and an infringement on
extraordinary power whenever it becomes necessary to prevent or suppress the prerogative that solely, at least initially, lies with the President.
lawless violence, invasion, or rebellion. "[T]he power to call is fully The elimination by the framers of the 1987 Constitution of the requirement of
discretionary to the President;" the only limitations being that he acts within prior concurrence of the Congress in the initial imposition of martial law or
permissible constitutional boundaries or in a manner not constituting grave suspension of the privilege of the writ of habeas corpus further supports the
abuse of discretion. In fact, "the actual use to which the President puts the conclusion that judicial review does not include the calibration of the
armed forces is x x x not subject to judicial review." President's decision of which of his graduated powers will be availed of in a
The extraordinary powers of suspending the privilege of the writ of habeas given situation. Voting 28 to 12, the framers of the 1987 Constitution removed
corpus and/or declaring martial law may be exercised only when there is actual the requirement of congressional concurrence in the first imposition of martial
invasion or rebellion, and public safety requires it. The 1987 Constitution law and suspension of the privilege.
imposed the following limits in the exercise of these powers: "(1) a time limit A plain reading of Section 18, Article VII of the Constitution shows that the
of sixty days; (2) review and possible revocation by Congress; [and] (3) review President's power to declare martial law is not subject to any condition except
and possible nullification by the Supreme Court." for the requirements of actual invasion or rebellion and that public safety
The framers of the 1987 Constitution eliminated insurrection, and the phrase requires it. Besides, it would be contrary to common sense if the decision of
"imminent danger thereof' as grounds for the suspension of the privilege of the the President is made dependent on the recommendation of his mere alter ego.
writ of habeas corpus or declaration of martial law. They perceived the phrase Rightly so, it is only on the President and no other that the exercise of the
"imminent danger" to be "fraught with possibilities of abuse;" besides, the powers of the Commander-in-Chief under Section 18, Article VII of the
calling out power of the President "is sufficient for handling imminent danger." Constitution is bestowed.

The powers to declare martial law and to suspend the privilege of the writ of At this juncture, it must be stressed that prior to Proclamation No. 216 or the
habeas corpus involve curtailment and suppression of civil rights and individual declaration of martial law on May 23, 2017, the President had already issued
freedom. Thus, the declaration of martial law serves as a warning to citizens Proclamation No. 55 on September 4, 2016, declaring a state of national
that the Executive Department has called upon the military to assist in the emergency on account of lawless violence in Mindanao. This, in fact, is extant
maintenance of law and order, and while the emergency remains, the citizens in the first Whereas Clause of Proclamation No. 216. Based on the foregoing
must, under pain of arrest and punishment, not act in a manner that will render presidential actions, it can be gleaned that although there is no obligation or
it more difficult to restore order and enforce the law. As such, their exercise requirement on his part to use his extraordinary powers on a graduated or
requires more stringent safeguards by the Congress, and review by the Court. sequential basis, still the President made the conscious and deliberate effort to
first employ the most benign from among his extraordinary powers. As the
It must be stressed, however, that the graduation refers only to hierarchy initial and preliminary step towards suppressing and preventing the armed
based on scope and effect. It does not in any manner refer to a sequence, hostilities in Mindanao, the President decided to use his calling out power first.
arrangement, or order which the Commander-in-Chief must follow. This so- Unfortunately, the situation did not improve; on the contrary, it only worsened.
called "graduation of powers" does not dictate or restrict the manner by which Thus, exercising his sole and exclusive prerogative, the President decided to
the President decides which power to choose. impose martial law and suspend the privilege of the writ of habeas corpus on
the belief that the armed hostilities in Mindanao already amount to actual
rebellion and public safety requires it.

M.R.A.D.C. LUMBRE 42
CONSTITUTIONAL LAW REVIEW

Section 18, Article VII of the Constitution states that "[i]n case of invasion or of the Philippines or any part thereof; (ii) any body of land, naval, or armed
rebellion, when the public safety requires it, [the President] may x x x suspend forces; or (b) to deprive the Chief Executive or Congress, wholly or partially,
the privilege of writ of habeas corpus or place the Philippines or any part of any of their powers and prerogatives.
thereof under martial law." Clearly, the Constitution grants to the President
the discretion to determine the territorial coverage of martial law and the In determining what crime was committed, we have to look into the main
suspension of the privilege of the writ of habeas corpus. He may put the entire objective of the malefactors. If it is political, such as for the purpose of severing
Philippines or only a part thereof under martial law. the allegiance of Mindanao to the Philippine Government to establish a wilayat
therein, the crime is rebellion. If, on the other hand, the primary objective is
This is both an acknowledgement and a recognition that it is the Executive to sow and create a condition of widespread and extraordinary fear and panic
Department, particularly the President as Commander-in-Chief, who is the among the populace in order to coerce the government to give in to an unlawful
repository of vital, classified, and live information necessary for and relevant demand, the crime is terrorism. Here, we have already explained and ruled
in calibrating the territorial application of martial law and the suspension of the that the President did not err in believing that what is going on in Marawi City
privilege of the writ of habeas corpus. It, too, is a concession that the President is one contemplated under the crime of rebellion.
has the tactical and military support, and thus has a more informed
understanding of what is happening on the ground. Thus, the Constitution 2. Houses of Congress
imposed a limitation on the period of application, which is 60 days, unless a. Senate
sooner nullified, revoked or extended, but not on the territorial scope or area
of coverage; it merely stated "the Philippines or any part thereof," depending i. Composition (Art. VI, Sec. 2)
on the assessment of the President.
Section 2. The Senate shall be composed of twenty-four Senators who shall
In fine, it is difficult, if not impossible, to fix the territorial scope of martial law be elected at large by the qualified voters of the Philippines, as may be
in direct proportion to the "range" of actual rebellion and public safety simply provided by law.
because rebellion and public safety have no fixed physical dimensions. Their
1. Antonio F. Trillanes IV vs. Hon. Oscar Pimentel, Sr., in His Capacity
transitory and abstract nature defies precise measurements; hence, the
as Presiding Judge, RTC – Branch 148, Makati City et al., GR No.
determination of the territorial scope of martial law could only be drawn from
179817, 27 June 2008
arbitrary, not fixed, variables. The Constitution must have considered these
limitations when it granted the President wide leeway and flexibility in FACTS: In the aftermath of this eventful episode dubbed as the "Oakwood
determining the territorial scope of martial law. Incident," petitioner Antonio F. Trillanes IV was charged, along with his
comrades, with coup d’etat defined under Article 134-A of the Revised Penal
Moreover, the President's duty to maintain peace and public safety is not
Code before the Regional Trial Court (RTC) of Makati.
limited only to the place where there is actual rebellion; it extends to other
areas where the present hostilities are in danger of spilling over. It is not Close to four years later, petitioner, who has remained in detention, threw his
intended merely to prevent the escape of lawless elements from Marawi City, hat in the political arena and won a seat in the Senate with a six-year term
but also to avoid enemy reinforcements and to cut their supply lines coming commencing at noon on June 30, 2007.
from different parts of Mindanao. Thus, limiting the proclamation and/or
suspension to the place where there is actual rebellion would not only defeat The present petition for certiorari to set aside the two Orders of the trial court,
the purpose of declaring martial law, it will make the exercise thereof and for prohibition and mandamus to (i) enjoin respondents from banning the
ineffective and useless. Senate staff, resource persons and guests from meeting with him or
transacting business with him in his capacity as Senator; and (ii) direct
It is also of judicial notice that the insurgency in Mindanao has been ongoing respondents to allow him access to the Senate staff, resource persons and
for decades. While some groups have sought legal and peaceful means, others guests and permit him to attend all sessions and official functions of the
have resorted to violent extremism and terrorism. Rebellion may be subsumed Senate. Petitioner preliminarily prayed for the maintenance of the status quo
under the crime of terrorism, which has a broader scope covering a wide range ante of having been able hitherto to convene his staff, resource persons and
of predicate crimes. In fact, rebellion is only one of the various means by which guests at the Marine Brig.
terrorism can be committed.299 However, while the scope of terrorism may
be comprehensive, its purpose is distinct and well-defined. The objective of a RULING: An attempting to strike a distinction between his case and that of
"'terrorist" is to sow and create a condition of widespread fear among the Jalosjos, petitioner chiefly points out that former Rep. Romeo Jalosjos
populace in order to coerce the government to give in to an unlawful demand. (Jalosjos) was already convicted, albeit his conviction was pending appeal,
This condition of widespread fear is traditionally achieved through bombing, when he filed a motion similar to petitioner’s Omnibus Motion, whereas he is a
kidnapping, mass killing, and beheading, among others. In contrast, the mere detention prisoner. He asserts that he continues to enjoy civil and
purpose of rebellion, as previously discussed, is political, i.e., (a) to remove political rights since the presumption of innocence is still in his favor.
from the allegiance to the Philippine Government or its laws: (i) the territory

M.R.A.D.C. LUMBRE 43
CONSTITUTIONAL LAW REVIEW

Further, petitioner illustrates that Jalosjos was charged with crimes involving b. House of Representatives
moral turpitude, i.e., two counts of statutory rape and six counts of acts of
lasciviousness, whereas he is indicted for coup d’etat which is regarded as a i. Apportionment of legislative districts
"political offense. a. Gerrymandering
Furthermore, petitioner justifies in his favor the presence of noble causes in i. Restraint - (Art. VI, Sec. 5[3], 1st sentence)
expressing legitimate grievances against the rampant and institutionalized
practice of graft and corruption in the AFP. Section 5. xxxx Each legislative district shall comprise, as far as
practicable, contiguous, compact, and adjacent territory. xxxx
In the present case, it is uncontroverted that petitioner’s application for bail
and for release on recognizance was denied. The determination that the NOTE: Gerrymandering is NOT unconstitutional, provided that a
evidence of guilt is strong, whether ascertained in a hearing of an application newly-formed legislative district (thru an act of Congress), conforms
for bail or imported from a trial court’s judgment of conviction, justifies the to the Constitutional requirement that it be “contiguous, compact, and
detention of an accused as a valid curtailment of his right to provisional liberty. adjacent territory.”
This accentuates the proviso that the denial of the right to bail in such cases
is "regardless of the stage of the criminal action." Such justification for b. Representation of cities and provinces (Art. VI, Sec. 5[3], 2nd
confinement with its underlying rationale of public self-defense applies equally sentence)
to detention prisoners like petitioner or convicted prisoners-appellants like
Section 5. xxxx Each city with a population of at least two hundred fifty
Jalosjos.
thousand, or each province, shall have at least one representative.
As the Court observed in Alejano v. Cabuay, it is impractical to draw a line
between convicted prisoners and pre-trial detainees for the purpose of i. Aquino vs. COMELEC, GR No. 189793, 7 April 2010
maintaining jail security; and while pre-trial detainees do not forfeit their FACTS: Following the enactment of Republic Act No. 9716, the first
constitutional rights upon confinement, the fact of their detention makes their and second districts of Camarines Sur were reconfigured in order to
rights more limited than those of the public. create an additional legislative district for the province. Hence, the
Petitioner’s contention hinges on the doctrine in administrative law that "a first district municipalities of Libmanan, Minalabac, Pamplona,
public official cannot be removed for administrative misconduct committed Pasacao, and San Fernando were combined with the second district
during a prior term, since his re-election to office operates as a condonation municipalities of Milaor and Gainza to form a new second legislative
of the officer’s previous misconduct to the extent of cutting off the right to district.
remove him therefor." Petitioners contend that the reapportionment introduced by Republic
The assertion is unavailing. The case against petitioner is not administrative in Act No. 9716, runs afoul of the explicit constitutional standard that
nature. And there is no "prior term" to speak of. In a plethora of cases, the requires a minimum population of two hundred fifty thousand
Court categorically held that the doctrine of condonation does not apply to (250,000) for the creation of a legislative district.5 The petitioners
criminal cases. Election, or more precisely, re-election to office, does not claim that the reconfiguration by Republic Act No. 9716 of the first
obliterate a criminal charge. Petitioner’s electoral victory only signifies and second districts of Camarines Sur is unconstitutional, because the
pertinently that when the voters elected him to the Senate, "they did so with proposed first district will end up with a population of less than
full awareness of the limitations on his freedom of action [and] x x x with the 250,000 or only 176,383.
knowledge that he could achieve only such legislative results which he could The petitioners posit that the 250,000 figure appearing in the above-
accomplish within the confines of prison." cited provision is the minimum population requirement for the
The performance of legitimate and even essential duties by public officers has creation of a legislative district. The petitioners theorize that, save in
never been an excuse to free a person validly in prison. The duties imposed by the case of a newly created province, each legislative district created
the "mandate of the people" are multifarious. The accused-appellant asserts by Congress must be supported by a minimum population of at least
that the duty to legislate ranks highest in the hierarchy of government. The 250,000 in order to be valid. Under this view, existing legislative
accused-appellant is only one of 250 members of the House of districts may be reapportioned and severed to form new districts,
Representatives, not to mention the 24 members of the Senate, charged with provided each resulting district will represent a population of at least
the duties of legislation. Congress continues to function well in the physical 250,000. On the other hand, if the reapportionment would result in
absence of one or a few of its members. x x x Never has the call of a particular the creation of a legislative seat representing a populace of less than
duty lifted a prisoner into a different classification from those others who are 250,000 inhabitants, the reapportionment must be stricken down as
validly restrained by law. invalid for non-compliance with the minimum population requirement.

M.R.A.D.C. LUMBRE 44
CONSTITUTIONAL LAW REVIEW

ISSUE: Whether a population of 250,000 is an indispensable each district would elect their own representative to Congress as well
constitutional requirement for the creation of a new legislative district as eight members of the Sangguniang Panglungsod. On March 13,
in a province. 2007, the COMELEC en Banc promulgated Resolution No. 78376
implementing R.A. No. 9371.
RULING: No. We start with the basics. Any law duly enacted by
Congress carries with it the presumption of constitutionality. Before a In asking for the nullification of R.A. No. 9371 and Resolution No.
law may be declared unconstitutional by this Court, there must be a 7837 on constitutional grounds, the petitioner argued that the
clear showing that a specific provision of the fundamental law has COMELEC cannot implement R.A. No. 9371 without providing for the
been violated or transgressed. When there is neither a violation of a rules, regulations and guidelines for the conduct of a plebiscite which
specific provision of the Constitution nor any proof showing that there is indispensable for the division or conversion of a local government
is such a violation, the presumption of constitutionality will prevail unit. He prayed for the issuance of an order directing the respondents
and the law must be upheld. To doubt is to sustain. to cease and desist from implementing R.A. No. 9371 and COMELEC
Resolution No. 7837, and to revert instead to COMELEC Resolution
There is no specific provision in the Constitution that fixes a 250,000 No. 7801 which provided for a single legislative district for Cagayan
minimum population that must compose a legislative district. de Oro.’
As already mentioned, the petitioners rely on the second sentence of ISSUE: Whether R.A. No. 9371 merely provides for the legislative
Section 5(3), Article VI of the 1987 Constitution, coupled with what reapportionment of Cagayan de Oro City, or involves the division and
they perceive to be the intent of the framers of the Constitution to conversion of a local government unit, which in this case requires a
adopt a minimum population of 250,000 for each legislative district. plebiscite.
The second sentence of Section 5(3), Article VI of the Constitution, RULING: This argument essentially proceeds from a
succinctly provides: "Each city with a population of at least two misunderstanding of the constitutional concepts of apportionment of
hundred fifty thousand, or each province, shall have at least one legislative districts and division of local government units.
representative."
Legislative apportionment is defined by Black's Law Dictionary as the
The provision draws a plain and clear distinction between the determination of the number of representatives which a State, county
entitlement of a city to a district on one hand, and the entitlement of or other subdivision may send to a legislative body. It is the allocation
a province to a district on the other. For while a province is entitled of seats in a legislative body in proportion to the population; the
to at least a representative, with nothing mentioned about population, drawing of voting district lines so as to equalize population and voting
a city must first meet a population minimum of 250,000 in order to power among the districts. Reapportionment, on the other hand, is
be similarly entitled. the realignment or change in legislative districts brought about by
changes in population and mandated by the constitutional
The use by the subject provision of a comma to separate the phrase
requirement of equality of representation.
"each city with a population of at least two hundred fifty thousand"
from the phrase "or each province" point to no other conclusion than Article VI of the 1987 Constitution lays down the rules on legislative
that the 250,000 minimum population is only required for a city, but apportionment under its Section 5 which provides:
not for a province.
Sec. 5(1). (1) The House of Representatives shall be composed
Plainly read, Section 5(3) of the Constitution requires a 250,000 of not more than two hundred fifty members unless otherwise
minimum population only for a city to be entitled to a representative, fixed by law, who shall be elected from legislative districts
but not so for a province. apportioned among the provinces, cities, and the Metropolitan
Manila area in accordance with the number of their respective
ii. Bagabuyo vs. COMELEC, 8 December 2008
inhabitants, and on the basis of a uniform and progressive ratio,
FACTS: On October 10, 2006, Cagayan de Oro's then Congressman and those who, as provided by law, shall be elected through a
Constantino G. Jaraula filed and sponsored House Bill No. 5859: "An party-list system of registered national, regional and sectoral
Act Providing for the Apportionment of the Lone Legislative District of parties or organizations.
the City of Cagayan De Oro." This law eventually became Republic Act
xxx
(R.A.) No. 9371. It increased Cagayan de Oro's legislative district
from one to two. For the election of May 2007, Cagayan de Oro's (3) Each legislative district shall comprise, as far as practicable,
voters would be classified as belonging to either the first or the second continuous, compact, and adjacent territory. Each city with a
district, depending on their place of residence. The constituents of

M.R.A.D.C. LUMBRE 45
CONSTITUTIONAL LAW REVIEW

population of at least two hundred fifty thousand, or each In contrast with the equal representation objective of Article VI,
province, shall have at least one representative. Section 5, Article X, Section 10 expressly speaks of how local
government units may be "created, divided, merged, abolished, or its
(4) Within three years following the return of every census, the boundary substantially altered." Its concern is the commencement,
Congress shall make a reapportionment of legislative districts the termination, and the modification of local government units'
based on the standards provided in this section. corporate existence and territorial coverage; and it speaks of two
Separately from the legislative districts that legal apportionment or specific standards that must be observed in implementing this
reapportionment speaks of, are the local government units that the concern, namely, the criteria established in the local government code
Constitution itself classified into provinces, cities, municipalities and and the approval by a majority of the votes cast in a plebiscite in the
barangays. In its strict and proper sense, a municipality has been political units directly affected. Under the Local Government Code
defined as "a body politic and corporate constituted by the (R.A. No. 7160) passed in 1991, the criteria of income, population
incorporation of the inhabitants of a city or town for the purpose of and land area are specified as verifiable indicators of viability and
local government thereof." The creation, division, merger, abolition capacity to provide services. The division or merger of existing units
or alteration of boundary of local government units, i.e., of provinces, must comply with the same requirements (since a new local
cities, municipalities, and barangays, are covered by the Article on government unit will come into being), provided that a division shall
Local Government (Article X). Section 10 of this Article provides: not reduce the income, population, or land area of the unit affected
to less than the minimum requirement prescribed in the Code.
“No province, city, municipality, or barangay may be created,
divided, merged, abolished, or its boundary substantially altered, A pronounced distinction between Article VI, Section 5 and, Article X,
except in accordance with the criteria established in the local Section 10 is on the requirement of a plebiscite. The Constitution and
government code and subject to approval by a majority of the the Local Government Code expressly require a plebiscite to carry out
votes cast in a plebiscite in the political unit directly affected.” any creation, division, merger, abolition or alteration of boundary of
a local government unit. In contrast, no plebiscite requirement exists
Under both Article VI, Section 5, and Article X, Section 10 of the under the apportionment or reapportionment provision.
Constitution, the authority to act has been vested in the Legislature.
The Legislature undertakes the apportionment and reapportionment In Tobias v. Abalos, a case that arose from the division of the
of legislative districts, and likewise acts on local government units by congressional district formerly covering San Juan and Mandaluyong
setting the standards for their creation, division, merger, abolition and into separate districts, we confirmed this distinction and the fact that
alteration of boundaries and by actually creating, dividing, merging, no plebiscite is needed in a legislative reapportionment. The plebiscite
abolishing local government units and altering their boundaries issue came up because one was ordered and held for Mandaluyong in
through legislation. Other than this, not much commonality exists the course of its conversion into a highly urbanized city, while none
between the two provisions since they are inherently different was held for San Juan. In explaining why this happened, the Court
although they interface and relate with one another. ruled that no plebiscite was necessary for San Juan because the
objective of the plebiscite was the conversion of Mandaluyong into a
The concern that leaps from the text of Article VI, Section 5 is political highly urbanized city as required by Article X, Section 10 the Local
representation and the means to make a legislative district sufficiently Government Code; the creation of a new legislative district only
represented so that the people can be effectively heard. As above followed as a consequence. In other words, the apportionment alone
stated, the aim of legislative apportionment is "to equalize population and by itself did not call for a plebiscite, so that none was needed for
and voting power among districts." Hence, emphasis is given to the San Juan where only a reapportionment took place.
number of people represented; the uniform and progressive ratio to
be observed among the representative districts; and accessibility and xxxx After it became constitutionally entrenched, a plebiscite was also
commonality of interests in terms of each district being, as far as always identified with the creation, division, merger, abolition and
practicable, continuous, compact and adjacent territory. In terms of alteration of boundaries of local government units, never with the
the people represented, every city with at least 250,000 people and concept of legislative apportionment.
every province (irrespective of population) is entitled to one iii. Sema vs. COMELEC, 16 July 2008
representative. In this sense, legislative districts, on the one hand,
and provinces and cities, on the other, relate and interface with each FACTS: The Ordinance appended to the 1987 Constitution
other. To ensure continued adherence to the required standards of apportioned two legislative districts for the Province of Maguindanao.
apportionment, Section 5(4) specifically mandates reapportionment The first legislative district consists of Cotabato City and eight
as soon as the given standards are met. municipalities. Maguindanao forms part of the Autonomous Region in
Muslim Mindanao (ARMM), created under its Organic Act, Republic Act

M.R.A.D.C. LUMBRE 46
CONSTITUTIONAL LAW REVIEW

No. 6734, as amended by Republic Act No. 9054. Although under the “Sec. 10. No province, city, municipality, or barangay may be
Ordinance, Cotabato City forms part of Maguindanao’s first legislative created, divided, merged, abolished or its boundary substantially
district, it is not part of the ARMM but of Region XII, having voted altered except in accordance with the criteria established in the
against its inclusion in the ARMM in the plebiscite held in November local government code and subject to approval by a majority of
1989. the votes cast in a plebiscite in the political units directly
affected.”
On 28 August 2006, the ARMM’s legislature, the ARMM Regional
Assembly, exercising its power to create provinces under Section 19, Thus, the creation of any of the four local government units –
Article VI of RA 9054,5 enacted Muslim Mindanao Autonomy Act No. province, city, municipality or barangay – must comply with three
201 (MMA Act 201) creating the Province of Shariff Kabunsuan conditions. First, the creation of a local government unit must follow
composed of the eight municipalities in the first district of the criteria fixed in the Local Government Code. Second, such creation
Maguindanao. The voters of Maguindanao ratified Shariff Kabunsuan’s must not conflict with any provision of the Constitution. Third, there
creation in a plebiscite held on 29 October 2006. must be a plebiscite in the political units affected.
On 10 May 2007, the COMELEC issued Resolution No. 7902, subject There is neither an express prohibition nor an express grant of
of these petitions, amending Resolution No. 07-0407 by renaming the authority in the Constitution for Congress to delegate to regional or
legislative district in question as "Shariff Kabunsuan Province with local legislative bodies the power to create local government units.
Cotabato City (formerly First District of Maguindanao with Cotabato However, under its plenary legislative powers, Congress can delegate
City)." to local legislative bodies the power to create local government units,
subject to reasonable standards and provided no conflict arises with
In G.R. No. 177597, Sema, who was a candidate in the 14 May 2007 any provision of the Constitution. In fact, Congress has delegated to
elections for Representative of "Shariff Kabunsuan with Cotabato provincial boards, and city and municipal councils, the power to create
City," prayed for the nullification of COMELEC Resolution No. 7902 barangays within their jurisdiction, subject to compliance with the
and the exclusion from canvassing of the votes cast in Cotabato City criteria established in the Local Government Code, and the plebiscite
for that office. Sema contended that Shariff Kabunsuan is entitled to requirement in Section 10, Article X of the Constitution. However,
one representative in Congress under Section 5 (3), Article VI of the under the Local Government Code, "only x x x an Act of Congress"
Constitution and Section 3 of the Ordinance appended to the can create provinces, cities or municipalities.
Constitution. Thus, Sema asserted that the COMELEC acted without
or in excess of its jurisdiction in issuing Resolution No. 7902 which Under Section 19, Article VI of RA 9054, Congress delegated to the
maintained the status quo in Maguindanao’s first legislative district ARMM Regional Assembly the power to create provinces, cities,
despite the COMELEC’s earlier directive in Resolution No. 7845 municipalities and barangays within the ARMM. Congress made the
designating Cotabato City as the lone component of Maguindanao’s delegation under its plenary legislative powers because the power to
reapportioned first legislative district. Sema further claimed that in create local government units is not one of the express legislative
issuing Resolution No. 7902, the COMELEC usurped Congress’ power powers granted by the Constitution to regional legislative bodies. In
to create or reapportion legislative districts. the present case, the question arises whether the delegation to the
ARMM Regional Assembly of the power to create provinces, cities,
ISSUES: municipalities and barangays conflicts with any provision of the
(1) Whether Section 19, Article VI of RA 9054, delegating to the Constitution.
ARMM Regional Assembly the power to create provinces, cities, There is no provision in the Constitution that conflicts with the
municipalities and barangays, is constitutional; and delegation to regional legislative bodies of the power to create
(2) If in the affirmative, whether a province created by the ARMM municipalities and barangays, provided Section 10, Article X of the
Regional Assembly under MMA Act 201 pursuant to Section 19, Article Constitution is followed. However, the creation of provinces and cities
VI of RA 9054 is entitled to one representative in the House of is another matter. Section 5 (3), Article VI of the Constitution
Representatives without need of a national law creating a legislative provides, "Each city with a population of at least two hundred fifty
district for such province. thousand, or each province, shall have at least one representative" in
the House of Representatives. Similarly, Section 3 of the Ordinance
RULING: appended to the Constitution provides, "Any province that may
hereafter be created, or any city whose population may hereafter
(1) The creation of local government units is governed by Section 10,
increase to more than two hundred fifty thousand shall be entitled in
Article X of the Constitution, which provides:
the immediately following election to at least one Member x x x."

M.R.A.D.C. LUMBRE 47
CONSTITUTIONAL LAW REVIEW

Clearly, a province cannot be created without a legislative district Indeed, the office of a legislative district representative to Congress
because it will violate Section 5 (3), Article VI of the Constitution as is a national office, and its occupant, a Member of the House of
well as Section 3 of the Ordinance appended to the Constitution. For Representatives, is a national official. It would be incongruous for a
the same reason, a city with a population of 250,000 or more cannot regional legislative body like the ARMM Regional Assembly to create
also be created without a legislative district. Thus, the power to create a national office when its legislative powers extend only to its regional
a province, or a city with a population of 250,000 or more, requires territory. The office of a district representative is maintained by
also the power to create a legislative district. Even the creation of a national funds and the salary of its occupant is paid out of national
city with a population of less than 250,000 involves the power to funds. It is a self-evident inherent limitation on the legislative powers
create a legislative district because once the city’s population reaches of every local or regional legislative body that it can only create local
250,000, the city automatically becomes entitled to one or regional offices, respectively, and it can never create a national
representative under Section 5 (3), Article VI of the Constitution and office.
Section 3 of the Ordinance appended to the Constitution. Thus, the
power to create a province or city inherently involves the power to To allow the ARMM Regional Assembly to create a national office is to
create a legislative district. allow its legislative powers to operate outside the ARMM’s territorial
jurisdiction. This violates Section 20, Article X of the Constitution
For Congress to delegate validly the power to create a province or which expressly limits the coverage of the Regional Assembly’s
city, it must also validly delegate at the same time the power to create legislative powers "[w]ithin its territorial jurisdiction x x x."
a legislative district. The threshold issue then is, can Congress validly
delegate to the ARMM Regional Assembly the power to create The ARMM Regional Assembly itself, in creating Shariff Kabunsuan,
legislative districts for the House of Representatives? The answer is recognized the exclusive nature of Congress’ power to create or
in the negative. reapportion legislative districts by abstaining from creating a
legislative district for Shariff Kabunsuan. Section 5 of MMA Act 201
(2) Section 5(1), Article VI of the Constitution vests in Congress the provides that:
power to increase, through a law, the allowable membership in the
House of Representatives. Section 5(4) empowers Congress to Except as may be provided by national law, the existing legislative
reapportion legislative districts. The power to reapportion legislative district, which includes Cotabato City as a part thereof, shall remain.
districts necessarily includes the power to create legislative districts However, a province cannot legally be created without a legislative
out of existing ones. Congress exercises these powers through a law district because the Constitution mandates that "each province shall
that Congress itself enacts, and not through a law that regional or have at least one representative." Thus, the creation of the Province
local legislative bodies enact. The allowable membership of the House of Shariff Kabunsuan without a legislative district is unconstitutional.
of Representatives can be increased, and new legislative districts of
Congress can be created, only through a national law passed by xxxx What Felwa teaches is that the creation of a legislative district
Congress. In Montejo v. COMELEC, we held that the "power of by Congress does not emanate alone from Congress’ power to
redistricting x x x is traditionally regarded as part of the power (of reapportion legislative districts, but also from Congress’ power to
Congress) to make laws," and thus is vested exclusively in Congress. create provinces which cannot be created without a legislative district.
Thus, when a province is created, a legislative district is created by
This textual commitment to Congress of the exclusive power to create operation of the Constitution because the Constitution provides that
or reapportion legislative districts is logical. Congress is a national "each province shall have at least one representative" in the House of
legislature and any increase in its allowable membership or in its Representatives. This does not detract from the constitutional
incumbent membership through the creation of legislative districts principle that the power to create legislative districts belongs
must be embodied in a national law. Only Congress can enact such a exclusively to Congress. It merely prevents any other legislative body,
law. It would be anomalous for regional or local legislative bodies to except Congress, from creating provinces because for a legislative
create or reapportion legislative districts for a national legislature like body to create a province such legislative body must have the power
Congress. An inferior legislative body, created by a superior legislative to create legislative districts. In short, only an act of Congress can
body, cannot change the membership of the superior legislative body. trigger the creation of a legislative district by operation of the
The creation of the ARMM, and the grant of legislative powers to its Constitution. Thus, only Congress has the power to create, or trigger
Regional Assembly under its organic act, did not divest Congress of the creation of, a legislative district.
its exclusive authority to create legislative districts. This is clear from Moreover, if as Sema claims MMA Act 201 apportioned a legislative
the Constitution and the ARMM Organic Act, as amended. district to Shariff Kabunsuan upon its creation, this will leave Cotabato
City as the lone component of the first legislative district of
Maguindanao. However, Cotabato City cannot constitute a legislative

M.R.A.D.C. LUMBRE 48
CONSTITUTIONAL LAW REVIEW

district by itself because as of the census taken in 2000, it had a at legislative sessions and committee meetings despite his having been
population of only 163,849. To constitute Cotabato City alone as the convicted in the first instance of a non-bailable offense.
surviving first legislative district of Maguindanao will violate Section 5
(3), Article VI of the Constitution which requires that "[E]ach city with ISSUE: Whether membership in Congress exempt an accused from statutes
a population of at least two hundred fifty thousand x x x, shall have and rules which apply to validly incarcerated persons in general.
at least one representative." RULING: No. We start with the incontestable proposition that all top officials
NOTE: Reapportionment refers to a realignment or change in of Government-executive, legislative, and judicial are subject to the majesty
legislative districts brought about by changes in population and of law. There is an unfortunate misimpression in the public mind that election
mandated by the Constitutional requirement of equality of or appointment to high government office, by itself, frees the official from the
representation. common restraints of general law. Privilege has to be granted by law, not
inferred from the duties of a position. In fact, the higher the rank, the greater
ii. Qualifications (Art. VI, Sec. 6) is the requirement of obedience rather than exemption.
Section 6. No person shall be a Member of the House of Representatives The immunity from arrest or detention of Senators and members of the House
unless he is a natural-born citizen of the Philippines and, on the day of the of Representatives, the latter customarily addressed as Congressmen, arises
election, is at least twenty-five years of age, able to read and write, and, from a provision of the Constitution. The history of the provision shows that
except the party-list representatives, a registered voter in the district in privilege has always been granted in a restrictive sense. The provision granting
which he shall be elected, and a resident thereof for a period of not less an exemption as a special privilege cannot be extended beyond the ordinary
than one year immediately preceding the day of the election. meaning of its terms. It may not be extended by intendment, implication or
equitable considerations.
a. Qualifications of Party-List representatives (Sec. 9 of RA
7941) However, the accused-appellant has not given any reason why he should be
exempted from the operation of Section 11, Article VI of the Constitution. The
Section 9. Qualifications of Party-List Nominees. No person shall be members of Congress cannot compel absent members to attend sessions if the
nominated as party-list representative unless he is a natural-born reason for the absence is a legitimate one. The confinement of a Congressman
citizen of the Philippines, a registered voter, a resident of the charged with a crime punishable by imprisonment of more than six months is
Philippines for a period of not less than one (1) year immediately not merely authorized by law, it has constitutional foundations.
preceding the day of the election, able to read and write, a bona fide
member of the party or organization which he seeks to represent for It will be recalled that when a warrant for accused-appellant's arrest was
at least ninety (90) days preceding the day of the election, and is at issued, he fled and evaded capture despite a call from his colleagues in the
least twenty-five (25) years of age on the day of the election. House of Representatives for him to attend the sessions and to surrender
voluntarily to the authorities. Ironically, it is now the same body whose call he
In case of a nominee of the youth sector, he must at least be twenty- initially spurned which accused-appellant is invoking to justify his present
five (25) but not more than thirty (30) years of age on the day of the motion. This cannot be countenanced because, to reiterate, aside from its
election. Any youth sectoral representative who attains the age of being contrary to well-defined Constitutional restrains, it would be a mockery
thirty (30) during his term shall be allowed to continue in office until of the aims of the State's penal system.
the expiration of his term.
What the accused-appellant seeks is not of an emergency nature. Allowing
3. Parliamentary immunities and privileges accused-appellant to attend congressional sessions and committee meeting for
a. Freedom from arrest – Art. VI, Sec. 11 five (5) days or more in a week will virtually make him free man with all the
privilege appurtenant to his position. Such an aberrant situation not only
Section 11. A Senator or Member of the House of Representatives shall, in all elevates accused-appellant's status to that of a special class, it also would be
offenses punishable by not more than six years imprisonment, be privileged from a mockery of the purposes of the correction system.
arrest while the Congress is in session.
When the voters of his district elected the accused-appellant to Congress, they
i. People v. Jalosjos, 324 SCRA 689, Feb. 3, 2000 did so with full awareness of the limitations on his freedom of action. They did
so with the knowledge that he could achieve only such legislative results which
FACTS: The accused-appellant, Romeo F. Jalojos is a full-pledged member of
he could accomplish within the confines of prison. To give a more drastic
Congress who is now confined at the national penitentiary while his conviction
illustration, if voters elect a person with full knowledge that he suffering from
for statutory rape on two counts and acts of lasciviousness on six counts is
a terminal illness, they do so knowing that at any time, he may no longer serve
pending appeal. The accused-appellant filed this motion asking that he be
his full term in office.
allowed to fully discharge the duties of a Congressman, including attendance

M.R.A.D.C. LUMBRE 49
CONSTITUTIONAL LAW REVIEW

In the ultimate analysis, the issue before us boils down to a question of the province of Cebu, took the floor of this chamber on the one hour
constitutional equal protection. privilege to deliver a speech, entitled 'A Message to Garcia;

The performance of legitimate and even essential duties by public officers has WHEREAS, in the course of said speech, the Congressman from the
never been an excuse to free a person validly in prison. The duties imposed by Second District of Cebu stated the following: xxx xxx xxx
the "mandate of the people" are multifarious. The accused-appellant asserts
that the duty to legislative ranks highest in the hierarchy of government. The The people, Mr. President, have been hearing of ugly reports that under
accused-appellant is only one of 250 members of the House of your unpopular administration the free things they used to get from the
Representatives, not to mention the 24 members of the Senate, charged with government are now for sale at premium prices. They say that even
the duties of legislation. Congress continues to function well in the physical pardons are for sale, and that regardless of the gravity or seriousness of
absence of one or a few of its members. Depending on the exigency of a criminal case, the culprit can always be bailed out forever from jail as
Government that has to be addressed, the President or the Supreme Court can long as he can come across with a handsome dole. I am afraid, such an
also be deemed the highest for that particular duty. The importance of a anomalous situation would reflect badly on the kind of justice that your
function depends on the need to its exercise. The duty of a mother to nurse administration is dispensing. . . . . xxx xxxx xxx”
her infant is most compelling under the law of nature. A doctor with unique Although some members of the court expressed doubts of petitioner's cause
skills has the duty to save the lives of those with a particular affliction. An of action and the Court's jurisdiction, the majority decided to hear the matter
elective governor has to serve provincial constituents. A police officer must further, and required respondents to answer, without issuing any preliminary
maintain peace and order. Never has the call of a particular duty lifted a injunction. Evidently aware of such circumstance with its implications, and
prisoner into a different classification from those others who are validly pressed for time in view of the imminent adjournment of the legislative session,
restrained by law. the special committee continued to perform its talk, and after giving
A strict scrutiny of classifications is essential lest wittingly or otherwise, Congressman Osmeña a chance to defend himself, submitted its reports on
insidious discriminations are made in favor of or against groups or types of July 18, 1960, finding said congressman guilty of serious disorderly behaviour;
individuals. and acting on such report, the House approved on the same day—before
closing its session—House Resolution No. 175, declaring him guilty as
The Court cannot validate badges of inequality. The necessities imposed by recommended, and suspending him from office for fifteen months.
public welfare may justify exercise of government authority to regulate even if
thereby certain groups may plausibly assert that their interests are Osmeña contended in his petition that: (1) the Constitution gave him complete
disregarded. parliamentary immunity, and so, for words spoken in the House, he ought not
to be questioned; (20 that his speech constituted no disorderly behaviour for
b. Speech and debate clause –Art. VI, Sec. 11 which he could be punished; and (3) supposing he could be questioned and
discipline therefor, the House had lost the power to do so because it had taken
Section 16. No Member shall be questioned nor be held liable in any other up other business before approving House Resolution No. 59. Now, he takes
place for any speech or debate in the Congress or in any committee thereof. the additional position (4) that the House has no power, under the Constitution,
i. Osmena v. Pendatun, 109 Phil. 863 (1960) to suspend one of its members.

FACTS: On July 14, 1960, Congressman Sergio Osmeña, Jr., submitted to this ISSUE: Whether petitioner Osmeña may invoke parliamentary immunity.
Court a verified petition for "declaratory relief, certiorari and prohibition with RULING: Section 15, Article VI of our Constitution provides that "for any
preliminary injunction" against Congressman Salapida K. Pendatun and speech or debate" in Congress, the Senators or Members of the House of
fourteen other congressmen in their capacity as members of the Special Representative "shall not be questioned in any other place." This section was
Committee created by House Resolution No. 59. He asked for annulment of taken or is a copy of sec. 6, clause 1 of Art. 1 of the Constitution of the United
such Resolution on the ground of infringement of his parliamentary immunity; States. In that country, the provision has always been understood to mean
he also asked, principally, that said members of the special committee be that although exempt from prosecution or civil actions for their words uttered
enjoined from proceeding in accordance with it, particularly the portion in Congress, the members of Congress may, nevertheless, be questioned in
authorizing them to require him to substantiate his charges against the Congress itself. Observe that "they shall not be questioned in any other place"
President with the admonition that if he failed to do so, he must show cause than Congress.
why the House should not punish him.
Furthermore, the Rules of the House which petitioner himself has invoked (Rule
The petition attached a copy of House Resolution No. 59, the pertinent portions XVII, sec. 7), recognize the House's power to hold a member responsible "for
of which reads as follows: words spoken in debate."
“WHEREAS, on the 23rd day of June, 1960, the Honorable Sergio Osmeña,
Jr., Member of the House of Representatives from the Second District of

M.R.A.D.C. LUMBRE 50
CONSTITUTIONAL LAW REVIEW

On the question whether delivery of speeches attacking the Chief Executive country of this nature. I am nauseated. I spit on the face of Chief Justice
constitutes disorderly conduct for which Osmeña may be disciplined, many Artemio Panganiban and his cohorts in the Supreme Court, I am no longer
arguments pro and con have been advanced. We believe, however, that the interested in the position [of Chief Justice] if I was to be surrounded by
House is the judge of what constitutes disorderly behaviour, not only because idiots. I would rather be in another environment but not in the Supreme
the Constitution has conferred jurisdiction upon it, but also because the matter Court of idiots x x x.
depends mainly on factual circumstances of which the House knows best but
which cannot be depicted in black and white for presentation to, and To Pobre, the foregoing statements reflected a total disrespect on the part of
adjudication by the Courts. For one thing, if this Court assumed the power to the speaker towards then Chief Justice Artemio Panganiban and the other
determine whether Osmeña conduct constituted disorderly behaviour, it would members of the Court and constituted direct contempt of court. Accordingly,
thereby have assumed appellate jurisdiction, which the Constitution never Pobre asks that disbarment proceedings or other disciplinary actions be taken.
intended to confer upon a coordinate branch of the Government. The theory of The immunity Senator Santiago claims is rooted primarily on the provision of
separation of powers fastidiously observed by this Court, demands in such Article VI, Section 11 of the Constitution, which provides: "A Senator or
situation a prudent refusal to interfere. Each department, it has been said, had Member of the House of Representative shall, in all offenses punishable by not
exclusive cognizance of matters within its jurisdiction and is supreme within its more than six years imprisonment, be privileged from arrest while the
own sphere. (Angara vs. Electoral Commission, 63 Phil., 139.) Congress is in session. No member shall be questioned nor be held liable in
Under our form of government, the judicial department has no power to revise any other place for any speech or debate in the Congress or in any committee
even the most arbitrary and unfair action of the legislative department, or of thereof."
either house thereof, taking in pursuance of the power committed exclusively ISSUE: Whether the Court may discipline Senator Santiago for her speech
to that department by the Constitution. It has been held by high authority that, delivered on the Senate floor.
even in the absence of an express provision conferring the power, every
legislative body in which is vested the general legislative power of the state RULING: No. Explaining the import of the underscored portion of the
has the implied power to expel a member for any cause which it may deem provision, the Court, in Osmeña, Jr. v. Pendatun, said:
sufficient. In Hiss. vs. Barlett, 3 Gray 473, 63 Am. Dec. 768, the supreme court
Our Constitution enshrines parliamentary immunity which is a fundamental
of Mass. says, in substance, that this power is inherent in every legislative
privilege cherished in every legislative assembly of the democratic world. As
body; that it is necessary to the to enable the body 'to perform its high
old as the English Parliament, its purpose "is to enable and encourage a
functions, and is necessary to the safety of the state;' 'That it is a power of
representative of the public to discharge his public trust with firmness and
self-protection, and that the legislative body must necessarily be the sole judge
success" for "it is indispensably necessary that he should enjoy the fullest
of the exigency which may justify and require its exercise. '. . . There is no
liberty of speech and that he should be protected from resentment of every
provision authority courts to control, direct, supervise, or forbid the exercise
one, however, powerful, to whom the exercise of that liberty may occasion
by either house of the power to expel a member. These powers are functions
offense."
of the legislative department and therefore, in the exercise of the power this
committed to it, the senate is supreme. An attempt by this court to direct or As American jurisprudence puts it, this legislative privilege is founded upon
control the legislature, or either house thereof, in the exercise of the power, long experience and arises as a means of perpetuating inviolate the functioning
would be an attempt to exercise legislative functions, which it is expressly process of the legislative department. Without parliamentary immunity,
forbidden to do. parliament, or its equivalent, would degenerate into a polite and ineffective
debating forum. Legislators are immune from deterrents to the uninhibited
In any event, petitioner's argument as to the deprivation of the district's
discharge of their legislative duties, not for their private indulgence, but for
representation cannot be more weighty in the matter of suspension than in the
the public good. The privilege would be of little value if they could be subjected
case of imprisonment of a legislator; yet deliberative bodies have the power in
to the cost and inconvenience and distractions of a trial upon a conclusion of
proper cases, to commit one of their members to jail.
the pleader, or to the hazard of a judgment against them based upon a judge’s
ii. Pobre vs. Santiago, A.C. No. 7399, 25 August 2009 speculation as to the motives.

FACTS: In his sworn letter/complaint dated December 22, 2006, with This Court is aware of the need and has in fact been in the forefront in
enclosures, Antero J. Pobre invites the Court’s attention to the following upholding the institution of parliamentary immunity and promotion of free
excerpts of Senator Miriam Defensor-Santiago’s speech delivered on the speech. Neither has the Court lost sight of the importance of the legislative
Senate floor: and oversight functions of the Congress that enable this representative body
to look diligently into every affair of government, investigate and denounce
x x x I am not angry. I am irate. I am foaming in the mouth. I am anomalies, and talk about how the country and its citizens are being served.
homicidal. I am suicidal. I am humiliated, debased, degraded. And I am Courts do not interfere with the legislature or its members in the manner they
not only that, I feel like throwing up to be living my middle years in a perform their functions in the legislative floor or in committee rooms. Any claim

M.R.A.D.C. LUMBRE 51
CONSTITUTIONAL LAW REVIEW

of an unworthy purpose or of the falsity and mala fides of the statement uttered of her own chamber. It is unfortunate that her peers bent backwards and
by the member of the Congress does not destroy the privilege. The disciplinary avoided imposing their own rules on her.
authority of the assembly and the voters, not the courts, can properly
discourage or correct such abuses committed in the name of parliamentary
immunity. 4. Disqualifications
For the above reasons, the plea of Senator Santiago for the dismissal of the a. Incompatible offices – Art. VI, Sec. 13, 1st sentence
complaint for disbarment or disciplinary action is well taken. Indeed, her
privilege speech is not actionable criminally or in a disciplinary proceeding Section 13. No Senator or Member of the House of Representatives may hold any
under the Rules of Court. It is felt, however, that this could not be the last other office or employment in the Government, or any subdivision, agency, or
word on the matter. instrumentality thereof, including government-owned or controlled corporations or
their subsidiaries, during his term without forfeiting his seat. xxxx
To the Court, the lady senator has undoubtedly crossed the limits of decency
and good professional conduct. It is at once apparent that her statements in NOTE: Effect of holding incompatible offices: the Senator or Representative shall
question were intemperate and highly improper in substance. To reiterate, she automatically lose his/her seat in Congress.
was quoted as stating that she wanted "to spit on the face of Chief Justice
i. Liban vs. Gordon, G.R. No. 175352, 15 July 2009;
Artemio Panganiban and his cohorts in the Supreme Court," and calling the
Court a "Supreme Court of idiots." FACTS: Petitioners Dante V. Liban, Reynaldo M. Bernardo, and Salvador M.
Viari filed with this Court a Petition to Declare Richard J. Gordon as Having
In this case, the lady senator clearly violated Canon 8, Rule 8.01 and Canon
Forfeited His Seat in the Senate. Petitioners are officers of the Board of
11 of the Code of Professional Responsibility, which respectively provide:
Directors of the Quezon City Red Cross Chapter while respondent is Chairman
Canon 8, Rule 8.01.––A lawyer shall not, in his professional dealings, use of the Philippine National Red Cross (PNRC) Board of Governors.
language which is abusive, offensive or otherwise improper.
During respondent’s incumbency as a member of the Senate of the Philippines,
Canon 11.––A lawyer shall observe and maintain the respect due to the he was elected Chairman of the PNRC during the 23 February 2006 meeting of
courts and to the judicial officers and should insist on similar conduct by the PNRC Board of Governors. Petitioners allege that by accepting the
others. chairmanship of the PNRC Board of Governors, respondent has ceased to be a
member of the Senate as provided in Section 13, Article VI of the Constitution,
The Court is not hesitant to impose some form of disciplinary sanctions on which reads:
Senator/Atty. Santiago for what otherwise would have constituted an act of
utter disrespect on her part towards the Court and its members. The factual SEC. 13. No Senator or Member of the House of Representatives may hold
and legal circumstances of this case, however, deter the Court from doing so, any other office or employment in the Government, or any subdivision,
even without any sign of remorse from her. Basic constitutional consideration agency, or instrumentality thereof, including government-owned or
dictates this kind of disposition. controlled corporations or their subsidiaries, during his term without
forfeiting his seat. Neither shall he be appointed to any office which may
It is imperative on our part to re-instill in Senator/Atty. Santiago her duty to have been created or the emoluments thereof increased during the term
respect courts of justice, especially this Tribunal, and remind her anew that for which he was elected.
the parliamentary non-accountability thus granted to members of Congress is
not to protect them against prosecutions for their own benefit, but to enable Petitioners cite Camporedondo v. NLRC, which held that the PNRC is a
them, as the people’s representatives, to perform the functions of their office government-owned or controlled corporation. Petitioners claim that in
without fear of being made responsible before the courts or other forums accepting and holding the position of Chairman of the PNRC Board of
outside the congressional hall. It is intended to protect members of Congress Governors, respondent has automatically forfeited his seat in the Senate,
against government pressure and intimidation aimed at influencing the pursuant to Flores v. Drilon, which held that incumbent national legislators lose
decision-making prerogatives of Congress and its members. their elective posts upon their appointment to another government office.

The Rules of the Senate itself contains a provision on Unparliamentary Acts ISSUE: Whether the office of the PNRC Chairman is a government office or an
and Language that enjoins a Senator from using, under any circumstance, office in a government-owned or controlled corporation for purposes of the
"offensive or improper language against another Senator or against any public prohibition in Section 13, Article VI of the Constitution.
institution." But as to Senator Santiago’s unparliamentary remarks, the Senate
RULING: No. The PNRC is not government-owned but privately owned. The
President had not apparently called her to order, let alone referred the matter
vast majority of the thousands of PNRC members are private individuals,
to the Senate Ethics Committee for appropriate disciplinary action, as the Rules
including students. Under the PNRC Charter, those who contribute to the
dictates under such circumstance. The lady senator clearly violated the rules
annual fund campaign of the PNRC are entitled to membership in the PNRC for

M.R.A.D.C. LUMBRE 52
CONSTITUTIONAL LAW REVIEW

one year. Thus, any one between 6 and 65 years of age can be a PNRC member even originally a party to this case, was being compelled, as a consequence of
for one year upon contributing ₱35, ₱100, ₱300, ₱500 or ₱1,000 for the year.20 the Decision, to suddenly reorganize and incorporate under the Corporation
Even foreigners, whether residents or not, can be members of the PNRC. Code, after more than sixty (60) years of existence in this country.
Section 5 of the PNRC Charter, as amended by Presidential Decree No. 1264,
reads: Its existence as a chartered corporation remained unchallenged on ground of
unconstitutionality notwithstanding that R.A. No. 95 was enacted on March 22,
SEC. 5. Membership in the Philippine National Red Cross shall be open to 1947 during the effectivity of the 1935 Constitution, which provided for a
the entire population in the Philippines regardless of citizenship. Any proscription against the creation of private corporations by special law, to wit:
contribution to the Philippine National Red Cross Annual Fund Campaign
shall entitle the contributor to membership for one year and said SEC. 7. The Congress shall not, except by general law, provide for the
contribution shall be deductible in full for taxation purposes. formation, organization, or regulation of private corporations, unless such
corporations are owned and controlled by the Government or any
Thus, the PNRC is a privately owned, privately funded, and privately run subdivision or instrumentality thereof. (Art. XIV, 1935 Constitution.)
charitable organization. The PNRC is not a government-owned or controlled
corporation. However, the PNRC Charter is violative of the Constitutional Similar provisions are found in Article XIV, Section 4 of the 1973 Constitution
proscription against the creation of private corporations by special law. Section and Article XII, Section 16 of the 1987 Constitution. The latter reads:
16, Article XII of the Constitution provides: SECTION 16. The Congress shall not, except by general law, provide for
Sec. 16. The Congress shall not, except by general law, provide for the the formation, organization, or regulation of private corporations.
formation, organization, or regulation of private corporations. Government- Government-owned or controlled corporations may be created or
owned or controlled corporations may be created or established by special established by special charters in the interest of the common good and
charters in the interest of the common good and subject to the test of economic subject to the test of economic viability.
viability. Since its enactment, the PNRC Charter was amended several times,
The Constitution emphatically prohibits the creation of private corporations particularly on June 11, 1953, August 16, 1971, December 15, 1977, and
except by general law applicable to all citizens. The purpose of this October 1, 1979, by virtue of R.A. No. 855, R.A. No. 6373, P.D. No. 1264, and
constitutional provision is to ban private corporations created by special P.D. No. 1643, respectively. The passage of several laws relating to the PNRC’s
charters, which historically gave certain individuals, families or groups special corporate existence notwithstanding the effectivity of the constitutional
privileges denied to other citizens. proscription on the creation of private corporations by law, is a recognition that
the PNRC is not strictly in the nature of a private corporation contemplated by
In sum, we hold that the office of the PNRC Chairman is not a government the aforesaid constitutional ban.
office or an office in a government-owned or controlled corporation for
purposes of the prohibition in Section 13, Article VI of the 1987 Constitution. A closer look at the nature of the PNRC would show that there is none like it
However, since the PNRC Charter is void insofar as it creates the PNRC as a not just in terms of structure, but also in terms of history, public service and
private corporation, the PNRC should incorporate under the Corporation Code official status accorded to it by the State and the international community.
and register with the Securities and Exchange Commission if it wants to be a There is merit in PNRC’s contention that its structure is sui generis.
private corporation. The PNRC succeeded the chapter of the American Red Cross which was in
Resolution on the Motion for Clarification and/or for Reconsideration existence in the Philippines since 1917. It was created by an Act of Congress
dated 18 January 2011, GR No. 175352 after the Republic of the Philippines became an independent nation on July 6,
1946 and proclaimed on February 14, 1947 its adherence to the Convention of
As correctly pointed out in respondent’s Motion, the issue of constitutionality Geneva of July 29, 1929 for the Amelioration of the Condition of the Wounded
of R.A. No. 95 was not raised by the parties, and was not among the issues and Sick of Armies in the Field (the "Geneva Red Cross Convention"). By that
defined in the body of the Decision; thus, it was not the very lis mota of the action the Philippines indicated its desire to participate with the nations of the
case. We have reiterated the rule as to when the Court will consider the issue world in mitigating the suffering caused by war and to establish in the
of constitutionality in Alvarez v. PICOP Resources, Inc. Philippines a voluntary organization for that purpose and like other volunteer
organizations established in other countries which have ratified the Geneva
Thus, the Court should not have declared void certain sections of R.A. No. 95, Conventions, to promote the health and welfare of the people in peace and in
as amended by Presidential Decree (P.D.) Nos. 1264 and 1643, the PNRC war.
Charter. Instead, the Court should have exercised judicial restraint on this
matter, especially since there was some other ground upon which the Court So must this Court recognize too the country’s adherence to the Geneva
could have based its judgment. Furthermore, the PNRC, the entity most Convention and respect the unique status of the PNRC in consonance with its
adversely affected by this declaration of unconstitutionality, which was not treaty obligations. The Geneva Convention has the force and effect of law.

M.R.A.D.C. LUMBRE 53
CONSTITUTIONAL LAW REVIEW

Under the Constitution, the Philippines adopts the generally accepted principles nor private in character. However, Sir said that there is no such animal as
of international law as part of the law of the land. This constitutional provision neither-public-nor-private corporation.
must be reconciled and harmonized with Article XII, Section 16 of the
Constitution, instead of using the latter to negate the former. b. Forbidden offices - Art. VI, Sec. 13 (2nd sentence)

By requiring the PNRC to organize under the Corporation Code just like any Section 13. xxxx Neither shall he be appointed to any office which may have been
other private corporation, the Decision of July 15, 2009 lost sight of the PNRC’s created or the emoluments thereof increased during the term for which he was
special status under international humanitarian law and as an auxiliary of the elected.
State, designated to assist it in discharging its obligations under the Geneva NOTE: Effect of holding forbidden offices: the Senator or Representative shall
Conventions. Although the PNRC is called to be independent under its forfeit BOTH offices.
Fundamental Principles, it interprets such independence as inclusive of its duty
to be the government’s humanitarian partner. To be recognized in the 5. Election of officers – Art. VI, Sec. 16 (1)
International Committee, the PNRC must have an autonomous status, and
Section 16. The Senate shall elect its President and the House of Representatives, its
carry out its humanitarian mission in a neutral and impartial manner.
Speaker, by a majority vote of all its respective Members. Each House shall choose such
The PNRC, as a National Society of the International Red Cross and Red other officers as it may deem necessary.
Crescent Movement, can neither "be classified as an instrumentality of the
a. Rep. Baguilat vs. Speaker Alvarez, G.R. No. 227757, 25 July 2017
State, so as not to lose its character of neutrality" as well as its independence,
nor strictly as a private corporation since it is regulated by international RULING: After a judicious study of this case, the Court finds that petitioners have
humanitarian law and is treated as an auxiliary of the State. no clear legal right to the reliefs sought. Records disclose that prior to the
Speakership Election held on July 25, 2016, then-Acting Floor Leader Rep. Farinas
Based on the above, the sui generis status of the PNRC is now sufficiently
responded to a parliamentary inquiry from Rep. Atienza as to who would elect the
established. Although it is neither a subdivision, agency, or instrumentality of
Minority Leader of the House of Representatives. Rep. Farinas then articulated
the government, nor a government-owned or -controlled corporation or a
that: (a) all those who vote for the winning Speaker shall belong to the
subsidiary thereof, as succinctly explained in the Decision of July 15, 2009, so
Majority and those who vote for other candidates shall belong to the
much so that respondent, under the Decision, was correctly allowed to hold his
Minority; (b) those who abstain from voting shall likewise be considered
position as Chairman thereof concurrently while he served as a Senator, such
part of the Minority; and (c) the Minority Leader shall be elected by the
a conclusion does not ipso facto imply that the PNRC is a "private corporation"
members of the Minority.
within the contemplation of the provision of the Constitution, that must be
organized under the Corporation Code. As correctly mentioned by Justice Thereafter, the election of the Speaker of the House proceeded without
Roberto A. Abad, the sui generis character of PNRC requires us to approach any objection from any member of Congress, including herein petitioners. Notably,
controversies involving the PNRC on a case-to-case basis. the election of the Speaker of the House is the essential and formative step
conducted at the first regular session of the 17 th Congress to determine the
In sum, the PNRC enjoys a special status as an important ally and auxiliary of
constituency of the Majority and Minority (and later on, their respective leaders),
the government in the humanitarian field in accordance with its commitments
considering that the Majority would be comprised of those who voted for the
under international law. This Court cannot all of a sudden refuse to recognize
winning Speaker and the Minority of those who did not. The unobjected procession
its existence, especially since the issue of the constitutionality of the PNRC
of the House at this juncture is reflected in its Journal No. 1 dated July 25, 2016,
Charter was never raised by the parties. It bears emphasizing that the PNRC
which, based on case law, is conclusive as to what transpired in Congress.
has responded to almost all national disasters since 1947, and is widely known
to provide a substantial portion of the country’s blood requirements. Its After Speaker Alvarez took his oath of office, he administered the oath of office to
humanitarian work is unparalleled. The Court should not shake its existence to all Members of the House of the 17th Congress. On the same day, the Deputy
the core in an untimely and drastic manner that would not only have negative Speakers, and other officers of the House (among others, the Majority Leader) were
consequences to those who depend on it in times of disaster and armed elected and all took their respective oaths of office.
hostilities but also have adverse effects on the image of the Philippines in the
international community. The sections of the PNRC Charter that were declared During his privilege speech delivered on July 26, 2016, which was a full day after
void must therefore stay. all the above-mentioned proceedings had already been commenced and completed,
NOTE: The Court belatedly realized its mistake in declaring the Philippine Rep. Lagman questioned Rep. Fariñas' interpretation of the Rules. Aside from the
National Red Cross as a private, unincorporated entity. This is because in belated timing of Rep. Lagman's query, Rep. Suarez aptly points out that the
declaring the PNRC as such, the PNRC would become a private corporation, Journal for that session does not indicate any motion made, seconded and carried
capable of being sued, as well as losing its tax exempt status. Thus, the Court to correct the entry in the Journal of the previous session (July 25, 2016) pertinent
rectified this by decreeing the sui generis character of the PNRC, neither public

M.R.A.D.C. LUMBRE 54
CONSTITUTIONAL LAW REVIEW

to any recording error that may have been made, as to indicate that in fact, a it is bound to protect and uphold x x x. Constitutional respect and a becoming
protest or objection was raised. regard for the sovereign acts of a coequal branch prevents the Court from prying
into the internal workings of the [House of Representatives]."
Logically speaking, the foregoing circumstances would show that the House of
Representatives had effectively adopted Rep. Farinas' proposal anent the new rules 6. Electoral Tribunals - Art. VI, Sec. 17
regarding the membership of the Minority, as well as the process of determining
Section 17. The Senate and the House of Representatives shall each have an Electoral
who the Minority Leader would be. More significantly, this demonstrates the
Tribunal which shall be the sole judge of all contests relating to the election, returns,
House's deviation from the "legal bases" of petitioners' claim for entitlement to the and qualifications of their respective Members. Each Electoral Tribunal shall be
reliefs sought before this Court, namely: (a) the "long-standing tradition" of composed of nine Members, three of whom shall be Justices of the Supreme Court to
automatically awarding the Minority Leadership to the second placer in the be designated by the Chief Justice, and the remaining six shall be Members of the
Speakership Elections, i.e., Rep. Baguilat; and (b) the rule21 that those who Senate or the House of Representatives, as the case may be, who shall be chosen on
abstained in the Speakership Elections should be deemed as independent Members the basis of proportional representation from the political parties and the parties or
of the House of Representatives, and thus, they could not have voted for a Minority organizations registered under the party-list system represented therein. The senior
Leader in the person of Rep. Suarez. As will be explained hereunder, the deviation Justice in the Electoral Tribunal shall be its Chairman.
by the Lower House from the aforesaid rules is not averse to the Constitution.
a. Cases:
Section 16 (1), Article VI of the 1987 Constitution reads: i. Pimentel vs. HRET, 393 SCRA 227 (2002)

Section 16. (1) The Senate shall elect its President and the House of ISSUES: Whether the present compositions of the HRET and COA (Commission
Representatives, its Speaker, by a majority vote of all its respective Members. on Appointments) violate the Constitutional requirement of proportional
representation because there are no party-list representatives in the HRET and
Each house shall choose such other officers as it may deem necessary. COA.

Under this provision, the Speaker of the House of Representatives shall be elected RULING: No. The Constitution expressly grants to the House of
by a majority vote of its entire membership. Said provision also states that the Representatives the prerogative, within constitutionally defined limits, to
House of Representatives may decide to have officers other than the Speaker, and choose from among its district and party-list representatives those who may
that the method and manner as to how these officers are chosen is something occupy the seats allotted to the House in the HRET and the CA.
within its sole control. In the case of Defensor-Santiago v. Guingona, which Section 18, Article VI of the Constitution explicitly confers on the Senate and
involved a dispute on the rightful Senate Minority Leader during the 11th Congress on the House the authority to elect among their members those who would fill
(1998-2001), this Court observed that "[w]hile the Constitution is explicit on the the 12 seats for Senators and 12 seats for House members in the Commission
manner of electing x x x [a Speaker of the House of Representative,] it is, however, on Appointments.
dead silent on the manner of selecting the other officers [of the Lower House]. All
that the Charter says is that ' [e]ach House shall choose such other officers as it Under Section 17, Article VI of the Constitution, each chamber of Congress
may deem necessary.' [As such], the method of choosing who will be such other exercises the power to choose, within constitutionally defined limits, who
officers is merely a derivative of the exercise of the prerogative conferred by the among their members would occupy the allotted 6 seats of each chamber’s
aforequoted constitutional provision. Therefore, such method must be prescribed respective electoral tribunal.
by the [House of Representatives] itself, not by [the] Court." Thus, even assuming that party-list representatives comprise a sufficient
number and have agreed to designate common nominees to the HRET and the
Corollary thereto, Section 16 (3), Article VI of the Constitution vests in the House
CA, their primary recourse clearly rests with the House of Representatives and
of Representatives the sole authority to, inter alia, "determine the rules of its
not with this Court. Under Sections 17 and 18, Article VI of the Constitution,
proceedings." These "legislative rules, unlike statutory laws, do not have the
party-list representatives must first show to the House that they possess the
imprints of permanence and obligatoriness during their effectivity. In fact, they 'are
required numerical strength to be entitled to seats in the HRET and the CA.
subject to revocation, modification or waiver at the pleasure of the body adopting
Only if the House fails to comply with the directive of the Constitution on
them.' Being merely matters of procedure, their observance are of no concern to
proportional representation of political parties in the HRET and the CA can the
the courts, for said rules may be waived or disregarded by the legislative body at
party-list representatives seek recourse to this Court under its power of judicial
will, upon the concurrence of a majority [of the House of Representatives]. " Hence,
review. Under the doctrine of primary jurisdiction, prior recourse to the House
as a general rule, "[t]his Court has no authority to interfere and unilaterally intrude
into that exclusive realm, without running afoul of [C]onstitutional principles that

M.R.A.D.C. LUMBRE 55
CONSTITUTIONAL LAW REVIEW

is necessary before petitioners may bring the instant case to the court. nullity of the grant of naturalization of Limkaichong’s father which, however,
Consequently, petitioners’ direct recourse to this Court is premature. is not allowed as it would constitute a collateral attack on the citizenship of the
father. In our jurisdiction, an attack on a person's citizenship may only be done
The discretion of the House to choose its members to the HRET and the CA is through a direct action for its nullity.
not absolute, being subject to the mandatory constitutional rule on
proportional representation. However, under the doctrine of separation of True, the HRET has jurisdiction over quo warranto petitions, specifically over
powers, the Court may not interfere with the exercise by the House of this cases challenging ineligibility on the ground of lack of citizenship. No less than
the 1987 Constitution vests the HRET the authority to be the sole judge of all
constitutionally mandated duty, absent a clear violation of the Constitution or
contests relating to the election, returns and qualifications of its Members. This
grave abuse of discretion amounting to lack or excess of jurisdiction.
constitutional power is likewise echoed in the 2004 Rules of the HRET. Rule 14
Otherwise, ‘the doctrine of separation of powers calls for each branch of thereof restates this duty, thus:
government to be left alone to discharge its duties as it sees fit. Neither can
the Court speculate on what action the House may take if party-list Rule 14. Jurisdiction. – The Tribunal is the sole judge of all contests
representatives are duly nominated for membership in the HRET and the CA relating to the election, returns, and qualifications of the Members of the
House of Representatives.
The instant petitions are bereft of any allegation that respondents prevented
Time and again, this Court has acknowledged this sole and exclusive
the party-list groups in the House from participating in the election of members
jurisdiction of the HRET. The power granted to HRET by the Constitution is
of the HRET and the CA. Neither does it appear that after the May 11, 1998
intended to be as complete and unimpaired as if it had remained originally in
elections, the House barred the party-list representatives from seeking
the legislature. Such power is regarded as full, clear and complete and excludes
membership in the HRET or the CA. Rather, it appears from the available facts the exercise of any authority on the part of this Court that would in any wise
that the party-list groups in the House at that time simply refrained from restrict it or curtail it or even affect the same.
participating in the election process. The party-list representatives did not
designate their nominees even up to the time they filed the instant petitions, Such power of the HRET, no matter how complete and exclusive, does not
with the predictable result that the House did not consider any party-list carry with it the authority to delve into the legality of the judgment of
naturalization in the pursuit of disqualifying Limkaichong. To rule otherwise
representative for election to the HRET or the CA. As the primary recourse of
would operate as a collateral attack on the citizenship of the father which, as
the party-list representatives lies with the House of Representatives, ‘the Court
already stated, is not permissible.
cannot resolve the issues presented by petitioners at this time.
Unfortunately, much as the Tribunal wants to resolve said issue, it cannot do
b. Functions – Art. VI, Sec. 17 so because its jurisdiction is limited to the qualification of the proclaimed
Section 17. xxxx Electoral Tribunal which shall be the sole judge of all contests respondent Limkaichong, being a sitting Member of the Congress.
relating to the election, returns, and qualifications of their respective Members. Evidently, there is no basis to oblige the Tribunal to reopen the naturalization
xxxx proceedings for a determination of the citizenship of the ascendant of
i. Vilando vs. HRET, 656 SCRA 17 (2011) respondent. A petition for quo warranto is not a means to achieve that purpose.
To rule on this issue in this quo warranto proceeding will not only be a clear
ISSUE: Whether the HRET, having the plenary, absolute, and exclusive grave abuse of discretion amounting to a lack or excess of jurisdiction, but also
jurisdiction to determine, among others, the qualifications of members of the a blatant violation of due process on the part of the persons who will be
House of Representatives, can look into the eligibility of Limkaichong even if, affected or who are not parties in this case.
as an incident thereto, it would mean looking into the validity of the Certificate
of Naturalization. The HRET, therefore, correctly relied on the presumption of validity of the July
9, 1957 and September 21, 1959 Orders of the Court of First Instance (CFI)
RULING: No. Citizenship, being a continuing requirement for Members of the Negros Oriental, which granted the petition and declared Julio Sy a naturalized
House of Representatives, however, may be questioned at any time. For this Filipino absent any evidence to the contrary.
reason, the Court deems it appropriate to resolve the petition on the merits.
c. Jurisdiction
This position finds support in the rule that courts will decide a question,
otherwise moot and academic, if it is "capable of repetition, yet evading NOTE: The earliest time to invoke the jurisdiction of the HRET is “at noon on the
review." The question on Limkaichong’s citizenship is likely to recur if she thirtieth (30th) day of June next following their election.” (Section 7, Art. VI)
would run again, as she did run, for public office, hence, capable of repetition.
i. Reyes vs. COMELEC, G.R. No. 207264, June 25, 2013
In this petition, Vilando seeks to disqualify Limkaichong on the ground that she
is a Chinese citizen. To prove his point, he makes reference to the alleged

M.R.A.D.C. LUMBRE 56
CONSTITUTIONAL LAW REVIEW

FACTS: The COMELEC First Division found that, contrary to the declarations Section 17. The Senate and the House of Representatives shall each have
that she made in her COC, petitioner is not a citizen of the Philippines because an Electoral Tribunal which shall be the sole judge of all contests relating
of her failure to comply with the requirements of Republic Act (R.A.) No. 9225 to the election, returns, and qualifications of their respective Members.
or the Citizenship Retention and Re-acquisition Act of 2003, namely: (1) to
take an oath of allegiance to the Republic of the Philippines; and (2) to make The next inquiry, then, is when is a candidate considered a Member of the
a personal and sworn renunciation of her American citizenship before any House of Representatives?
public officer authorized to administer an oath. In addition, the COMELEC First
Division ruled that she did not have the one year residency requirement under To be considered a Member of the House of Representatives, there must be a
Section 6, Article VI of the 1987 Constitution. Thus, she is ineligible to run for concurrence of the following requisites:
the position of Representative for the lone district of Marinduque.
(1) a valid proclamation,
On 14 May 2013, the COMELEC En Banc, promulgated a Resolution denying
(2) a proper oath, and
petitioner’s Motion for Reconsideration for lack of merit.
(3) assumption of office.
Four days thereafter or on 18 May 2013, petitioner was proclaimed winner of
the 13 May 2013 Elections. Here, the petitioner cannot be considered a Member of the House of
On 5 June 2013, the COMELEC En Banc issued a Certificate of Finality16 Representatives because, primarily, she has not yet assumed office. To repeat
declaring the 14 May 2013 Resolution of the COMELEC En Banc final and what has earlier been said, the term of office of a Member of the House of
executory, considering that more than twenty-one (21) days have elapsed Representatives begins only "at noon on the thirtieth day of June next following
from the date of promulgation with no order issued by this Court restraining their election." Thus, until such time, the COMELEC retains jurisdiction.
its execution.
In her attempt to comply with the second requirement, petitioner attached a
On same day, petitioner took her oath of office before Feliciano R. Belmonte purported Oath of Office taken before Hon. Feliciano Belmonte Jr. on 5 June
Jr., Speaker of the House of Representatives. Petitioner has yet to assume 2013. However, this is not the oath of office which confers membership to the
office, the term of which officially starts at noon of 30 June 2013. House of Representatives.

Thus, this Petition for Certiorari with Prayer for Temporary Restraining Order Section 6, Rule II (Membership) of the Rules of the House of Representatives
and/or Preliminary Injunction and/or Status Quo Ante Order. provides:

ISSUE: Whether the COMELEC is without jurisdiction over petitioner who is a Section 6. Oath or Affirmation of Members. – Members shall take their
duly-proclaimed winner and who has already taken her oath for the position of oath or affirmation either collectively or individually before the Speaker in
Member of the House of Representatives. open session.

RULING: According to petitioner, the COMELEC was ousted of its jurisdiction Consequently, before there is a valid or official taking of the oath it must be
when she was duly proclaimed because pursuant to Section 17, Article VI of made (1) before the Speaker of the House of Representatives, and (2) in open
the 1987 Constitution, the HRET has the exclusive jurisdiction to be the "sole session. Here, although she made the oath before Speaker Belmonte, there is
judge of all contests relating to the election, returns and qualifications" of the no indication that it was made during plenary or in open session and, thus, it
Members of the House of Representatives. remains unclear whether the required oath of office was indeed complied with.

Contrary to petitioner’s claim, however, the COMELEC retains jurisdiction for More importantly, we cannot disregard a fact basic in this controversy – that
the following reasons: before the proclamation of petitioner on 18 May 2013, the COMELEC En Banc
had already finally disposed of the issue of petitioner’s lack of Filipino
First, the HRET does not acquire jurisdiction over the issue of petitioner’s citizenship and residency via its Resolution dated 14 May 2013. After 14 May
qualifications, as well as over the assailed COMELEC Resolutions, unless a 2013, there was, before the COMELEC, no longer any pending case on
petition is duly filed with said tribunal. Petitioner has not averred that she has petitioner’s qualifications to run for the position of Member of the House of
filed such action. Representative. We will inexcusably disregard this fact if we accept the
Second, the jurisdiction of the HRET begins only after the candidate is argument of the petitioner that the COMELEC was ousted of jurisdiction when
considered a Member of the House of Representatives, as stated in Section 17, she was proclaimed, which was four days after the COMELEC En Banc decision.
Article VI of the 1987 Constitution: The Board of Canvasser which proclaimed petitioner cannot by such act be
allowed to render nugatory a decision of the COMELEC En Banc which affirmed
a decision of the COMELEC First Division.

M.R.A.D.C. LUMBRE 57
CONSTITUTIONAL LAW REVIEW

NOTE: The jurisdiction of the COMELEC remains from the time of elections (2 nd The Supreme Court has spoken. Its pronouncements must be respected. Being
Monday of May) and before noon of the 30 th of June next following the day of the ultimate guardian of the Constitution, and by constitutional design, the
elections. Thus, the earliest time that one can invoke the jurisdiction of the Supreme Court is "supreme in its task of adjudication; x x x. As a rule, all
HRET is at noon of the 30th of June next following the day of elections, which decisions and determinations in the exercise of judicial power ultimately go to
is the day that the elected official assumes office. and stop at the Supreme Court whose judgment is final." This Tribunal, as all
other courts, must take their bearings from the decisions and rulings of the
Remember the requisites to become a Member of the House of Supreme Court.
Representatives: (1) a valid proclamation; (2) a proper oath; and (3)
assumption of office. If the requisites have been met, only then can the elected Reyes argues in essence that this Court is devoid of original jurisdiction to
official invoke the jurisdiction of the HRET. In this case, petitioner’s COC was annul her proclamation. Instead, it is the HRET that is constitutionally
cancelled; thus, there can be no valid proclamation of one who was disqualified mandated to resolve any questions regarding her election, the returns of such
and deemed to have never been a candidate. election, and her qualifications as a Member of the House of Representatives
especially so that she has already been proclaimed, taken her oath, and started
ii. Velasco vs. Speaker Belmonte, G.R. No. 211140, 12 January 2016 to discharge her duties as a Member of the House of Representatives
FACTS: In the present cases, before respondent Regina Reyes was proclaimed representing the Lone District of the Province of Marinduque. But the
on May 18, 2013, the COMELEC En Banc, in its Resolution of May 14, 2013 in confluence of the three acts in this case - her proclamation, oath and
SPA No. 13-053 (DC), had already resolved that the COMELEC First Division assumption of office - has not altered the legal situation between Velasco and
correctly cancelled her COC on the ground that she lacked the Filipino Reyes.
citizenship and residency requirements. Thus, the COMELEC nullified her The important point of reference should be the date the COMELEC finally
proclamation. When Regina Reyes challenged the COMELEC actions, the decided to cancel the Certificate of Candidacy (COC) of Reyes which was on
Supreme Court En Banc, in its Resolution of June 25, 2013 in G.R. No. 207246, May 14, 2013. The most crucial time is when Reyes's COC was cancelled due
upheld the same. to her non-eligibility to run as Representative of the Lone District of the
RULING: With the COMELEC's cancellation of respondent Regina Reyes' COC, Province of Marinduque - for without a valid COC, Reyes could not be treated
resulting in the nullification of her proclamation, the Tribunal, much as we as a candidate in the election and much less as a duly proclaimed winner. That
would want to, cannot assume jurisdiction over the present petitions. The particular decision of the COMELEC was promulgated even before Reyes's
jurisdiction of the HRET begins only after the candidate is considered a Member proclamation, and which was affirmed by this Court's final and executory
of the House of Representatives. Resolutions dated June 25, 2013 and October 22, 2013.

And to be considered a Member of the House of Representatives, there must As we have previously stated, the administration of oath and the registration
be a concurrence of the following requisites: (1) a valid proclamation, (2) a of Velasco in the Roll of Members of the House of Representatives for the Lone
proper oath, and (3) assumption of office, so the Supreme Court pronounced District of the Province of Marinduque are no longer a matter of discretion or
in its Resolution of June 25, 2013 in G.R. No. 207264, thus: judgment on the part of Speaker Belmonte, Jr. and Sec. Gen. Barua-Yap. They
are legally duty-bound to recognize Velasco as the duly elected Member of the
x x x, the jurisdiction of the HRET begins only after the candidate is House of Representatives for the Lone District of Marinduque in view of the
considered a Member of the House of Representatives, as stated in Section ruling rendered by this Court and the COMELEC'S compliance with the said
17, Article VI of the 1987 Constitution: x x x x ruling, now both final and executory.
Based on the above-quoted ruling of the Supreme Court, a valid proclamation iii. Abayon vs. COMELEC, 11 February 2010
is the first essential element before a candidate can be considered a Member
of the House of Representatives over which the Tribunal could assume FACTS: These two cases are about the authority of the House of
jurisdiction. Such element is obviously absent in the present cases as Regina Representatives Electoral Tribunal (HRET) to pass upon the eligibilities of the
Reyes' proclamation was nullified by the COMELEC, which nullification was nominees of the party-list groups that won seats in the lower house of
upheld by the Supreme Court. On this ground alone, the Tribunal is without Congress.
power to assume jurisdiction over the present petitions since Regina Reyes In G.R. 189466, petitioner Daryl Grace J. Abayon is the first nominee of the
"cannot be considered a Member of the House of Representatives," as declared Aangat Tayo party-list organization that won a seat in the House of
by the Supreme Court En Banc in G.R. No. 207264. It further stresses: Representatives during the 2007 elections.
"x x x there was no basis for the proclamation of petitioner [Regina Reyes] Respondents Perfecto C. Lucaban, Jr., Ronyl S. Dela Cruz, and Agustin C.
on 18 May 2013. Without the proclamation, the petitioner's oath of office Doroga, all registered voters, filed a petition for quo warranto with respondent
is likewise baseless, and without a precedent oath of office, there can be HRET against Aangat Tayo and its nominee, petitioner Abayon, in HRET Case
no valid and effective assumption of office." 07-041. They claimed that Aangat Tayo was not eligible for a party-list seat in

M.R.A.D.C. LUMBRE 58
CONSTITUTIONAL LAW REVIEW

the House of Representatives, since it did not represent the marginalized and subject to the same term limitation of three years for a maximum of three
underrepresented sectors. consecutive terms.

In G.R. 189506, petitioner Jovito S. Palparan, Jr. is the first nominee of the As this Court also held in Bantay Republic Act or BA-RA 7941 v. Commission
Bantay party-list group that won a seat in the 2007 elections for the members on Elections, a party-list representative is in every sense "an elected member
of the House of Representatives. of the House of Representatives." Although the vote cast in a party-list election
is a vote for a party, such vote, in the end, would be a vote for its nominees,
Shortly after the elections, respondent Lesaca and the others with him filed who, in appropriate cases, would eventually sit in the House of
with respondent HRET a petition for quo warranto against Bantay and its Representatives.
nominee, petitioner Palparan, in HRET Case 07-040. Lesaca and the others
alleged that Palparan was ineligible to sit in the House of Representatives as Both the Constitution and the Party-List System Act set the qualifications and
party-list nominee because he did not belong to the marginalized and grounds for disqualification of party-list nominees. Section 9 of R.A. 7941,
underrepresented sectors that Bantay represented, namely, the victims of echoing the Constitution, states:
communist rebels, Civilian Armed Forces Geographical Units (CAFGUs), former
rebels, and security guards. Lesaca and the others said that Palparan Sec. 9. Qualification of Party-List Nominees. – No person shall be nominated
committed gross human rights violations against marginalized and as party-list representative unless he is a natural-born citizen of the
underrepresented sectors and organizations. Philippines, a registered voter, a resident of the Philippines for a period of not
less than one (1) year immediately preceding the day of the election, able to
ISSUE: Whether HRET has jurisdiction over the question of qualifications of read and write, bona fide member of the party or organization which he seeks
petitioners Abayon and Palparan as nominees of Aangat Tayo and Bantay to represent for at least ninety (90) days preceding the day of the election,
party-list organizations, respectively, who took the seats at the House of and is at least twenty-five (25) years of age on the day of the election.
Representatives that such organizations won in the 2007 elections.
In case of a nominee of the youth sector, he must at least be twenty-five (25)
RULING: Yes. but not more than thirty (30) years of age on the day of the election. Any
youth sectoral representative who attains the age of thirty (30) during his term
Although it is the party-list organization that is voted for in the elections, it is shall be allowed to continue until the expiration of his term.
not the organization that sits as and becomes a member of the House of
Representatives. Section 5, Article VI of the Constitution, identifies who the In the cases before the Court, those who challenged the qualifications of
"members" of that House are: petitioners Abayon and Palparan claim that the two do not belong to the
marginalized and underrepresented sectors that they ought to represent. The
Sec. 5. (1). The House of Representatives shall be composed of not more than Party-List System Act provides that a nominee must be a "bona fide member
two hundred and fifty members, unless otherwise fixed by law, who shall be of the party or organization which he seeks to represent."
elected from legislative districts apportioned among the provinces, cities, and
the Metropolitan Manila area in accordance with the number of their respective It is for the HRET to interpret the meaning of this particular qualification of a
inhabitants, and on the basis of a uniform and progressive ratio, and those nominee—the need for him or her to be a bona fide member or a representative
who, as provided by law, shall be elected through a party‑list system of of his party-list organization—in the context of the facts that characterize
registered national, regional, and sectoral parties or organizations. petitioners Abayon and Palparan’s relation to Aangat Tayo and Bantay,
respectively, and the marginalized and underrepresented interests that they
Clearly, the members of the House of Representatives are of two kinds: presumably embody.
"members x x x who shall be elected from legislative districts" and "those who
x x x shall be elected through a party-list system of registered national, NOTE: COMELEC determines the qualifications of party lists. HRET determines
regional, and sectoral parties or organizations." This means that, from the qualification of the party list nominees.
Constitution’s point of view, it is the party-list representatives who are
"elected" into office, not their parties or organizations. These representatives 7. The Commission on Appointments – (will be discussed in the President’s
are elected, however, through that peculiar party-list system that the Power of Appointment)
Constitution authorized and that Congress by law established where the voters 8. Powers of Congress
cast their votes for the organizations or parties to which such party-list
representatives belong. a. Legislative oversight functions

Once elected, both the district representatives and the party-list i. Three kinds –
representatives are treated in like manner. They have the same deliberative
ABAKADA Guro vs. Purisima, GR No. 166715, 14 August 2008
rights, salaries, and emoluments. They can participate in the making of laws
that will directly benefit their legislative districts or sectors. They are also

M.R.A.D.C. LUMBRE 59
CONSTITUTIONAL LAW REVIEW

FACTS: Petitioners assail, among others, the creation of a congressional Congress exercises supervision over the executive agencies through its veto
oversight committee on the ground that it violates the doctrine of separation power. It typically utilizes veto provisions when granting the President or an
of powers. While the legislative function is deemed accomplished and executive agency the power to promulgate regulations with the force of law.
completed upon the enactment and approval of the law, the creation of the These provisions require the President or an agency to present the proposed
congressional oversight committee permits legislative participation in the regulations to Congress, which retains a "right" to approve or disapprove any
implementation and enforcement of the law. regulation before it takes effect. Such legislative veto provisions usually
provide that a proposed regulation will become a law after the expiration of a
RULING: Yes. Broadly defined, the power of oversight embraces all activities certain period of time, only if Congress does not affirmatively disapprove of
undertaken by Congress to enhance its understanding of and influence over the regulation in the meantime. Less frequently, the statute provides that a
the implementation of legislation it has enacted. Clearly, oversight concerns proposed regulation will become law if Congress affirmatively approves it.
post-enactment measures undertaken by Congress: (a) to monitor
xxxx From the foregoing, it is clear that congressional oversight is not
bureaucratic compliance with program objectives, (b) to determine whether
unconstitutional per se, meaning, it neither necessarily constitutes an
agencies are properly administered, (c) to eliminate executive waste and
encroachment on the executive power to implement laws nor undermines the
dishonesty, (d) to prevent executive usurpation of legislative authority, and
constitutional separation of powers. Rather, it is integral to the checks and
(d) to assess executive conformity with the congressional perception of public
balances inherent in a democratic system of government. It may in fact even
interest.
enhance the separation of powers as it prevents the over-accumulation of
The power of oversight has been held to be intrinsic in the grant of legislative power in the executive branch.
power itself and integral to the checks and balances inherent in a democratic
However, to forestall the danger of congressional encroachment "beyond the
system of government.
legislative sphere," the Constitution imposes two basic and related constraints
The acts done by Congress purportedly in the exercise of its oversight powers on Congress. It may not vest itself, any of its committees or its members with
may be divided into three categories, namely: scrutiny, investigation and either executive or judicial power. And, when it exercises its legislative power,
supervision. it must follow the "single, finely wrought and exhaustively considered,
procedures" specified under the Constitution, including the procedure for
a. Scrutiny enactment of laws and presentment.
Congressional scrutiny implies a lesser intensity and continuity of attention to Thus, any post-enactment congressional measure such as this should be
administrative operations. Its primary purpose is to determine economy and limited to scrutiny and investigation. In particular, congressional oversight
efficiency of the operation of government activities. In the exercise of must be confined to the following:
legislative scrutiny, Congress may request information and report from the
other branches of government. It can give recommendations or pass (1) scrutiny based primarily on Congress’ power of appropriation and the
resolutions for consideration of the agency involved. budget hearings conducted in connection with it, its power to ask heads
of departments to appear before and be heard by either of its Houses on
b. Congressional investigation any matter pertaining to their departments and its power of confirmation
While congressional scrutiny is regarded as a passive process of looking at the and
facts that are readily available, congressional investigation involves a more (2) investigation and monitoring of the implementation of laws pursuant
intense digging of facts. The power of Congress to conduct investigation is to the power of Congress to conduct inquiries in aid of legislation.
recognized by the 1987 Constitution under section 21, Article VI.
Any action or step beyond that will undermine the separation of powers
c. Legislative supervision guaranteed by the Constitution. Legislative vetoes fall in this class.
The third and most encompassing form by which Congress exercises its Legislative veto is a statutory provision requiring the President or an
oversight power is thru legislative supervision. "Supervision" connotes a administrative agency to present the proposed implementing rules and
continuing and informed awareness on the part of a congressional committee regulations of a law to Congress which, by itself or through a committee formed
regarding executive operations in a given administrative area. While both by it, retains a "right" or "power" to approve or disapprove such regulations
congressional scrutiny and investigation involve inquiry into past executive before they take effect. As such, a legislative veto in the form of a
branch actions in order to influence future executive branch performance, congressional oversight committee is in the form of an inward-turning
congressional supervision allows Congress to scrutinize the exercise of delegation designed to attach a congressional leash (other than through
delegated law-making authority, and permits Congress to retain part of that scrutiny and investigation) to an agency to which Congress has by law initially
delegated authority. delegated broad powers. It radically changes the design or structure of the

M.R.A.D.C. LUMBRE 60
CONSTITUTIONAL LAW REVIEW

Constitution’s diagram of power as it entrusts to Congress a direct role in 1. Constitutional requisites / limitations
enforcing, applying or implementing its own laws.
a. Garcilliano vs. House of Representatives, 23 December 2008
Congress has two options when enacting legislation to define national policy
within the broad horizons of its legislative competence. It can itself formulate FACTS: Tapes ostensibly containing a wiretapped conversation
the details or it can assign to the executive branch the responsibility for making purportedly between the President (GMA) and a high-ranking official
necessary managerial decisions in conformity with those standards. In the of the COMELEC surfaced. Notoriously referred to as the "Hello Garci"
latter case, the law must be complete in all its essential terms and conditions tapes, these allegedly contained the President’s instructions to
when it leaves the hands of the legislature. Thus, what is left for the executive COMELEC Commissioner Virgilio Garcillano to manipulate in her favor
branch or the concerned administrative agency when it formulates rules and results of the 2004 presidential elections. These recordings were to
regulations implementing the law is to fill up details (supplementary rule- become the subject of heated legislative hearings conducted
making) or ascertain facts necessary to bring the law into actual operation separately by committees of both Houses of Congress.
(contingent rule-making). Alarmed by these developments, petitioner Virgilio O. Garcillano
Administrative regulations enacted by administrative agencies to implement (Garcillano) filed with this Court a Petition for Prohibition and
and interpret the law which they are entrusted to enforce have the force of law Injunction, with Prayer for Temporary Restraining Order and/or Writ
and are entitled to respect. Such rules and regulations partake of the nature of Preliminary Injunction docketed as G.R. No. 170338. He prayed
of a statute and are just as binding as if they have been written in the statute that the respondent House Committees be restrained from using
itself. As such, they have the force and effect of law and enjoy the presumption these tape recordings of the "illegally obtained" wiretapped
of constitutionality and legality until they are set aside with finality in an conversations in their committee reports and for any other purpose.
appropriate case by a competent court. He further implored that the said recordings and any reference
thereto be ordered stricken off the records of the inquiry, and the
Congress, in the guise of assuming the role of an overseer, may not pass upon respondent House Committees directed to desist from further using
their legality by subjecting them to its stamp of approval without disturbing the recordings in any of the House proceedings.
the calculated balance of powers established by the Constitution. In exercising
discretion to approve or disapprove the IRR based on a determination of After more than two years of quiescence, Senator Panfilo Lacson
whether or not they conformed to the provisions of RA 9335, Congress roused the slumbering issue with a privilege speech, "The Lighthouse
arrogated judicial power unto itself, a power exclusively vested in this Court That Brought Darkness." In his discourse, Senator Lacson promised
by the Constitution. to provide the public "the whole unvarnished truth – the what’s,
when’s, where’s, who’s and why’s" of the alleged wiretap, and sought
Following this rationale, Section 12 of RA 9335 should be struck down as an inquiry into the perceived willingness of telecommunications
unconstitutional. While there may be similar provisions of other laws that may providers to participate in nefarious wiretapping activities.
be invalidated for failure to pass this standard, the Court refrains from
invalidating them wholesale but will do so at the proper time when an On motion of Senator Francis Pangilinan, Senator Lacson’s speech
appropriate case assailing those provisions is brought before us. was referred to the Senate Committee on National Defense and
Security, chaired by Senator Rodolfo Biazon.
1. Concept of “legislative veto”
In the Senate’s plenary session the following day, a lengthy debate
Legislative veto is a statutory provision requiring the President or an ensued when Senator Richard Gordon aired his concern on the
administrative agency to present the proposed implementing rules and possible transgression of Republic Act (R.A.) No. 4200 if the body
regulations of a law to Congress which, by itself or through a committee were to conduct a legislative inquiry on the matter. On August 28,
formed by it, retains a "right" or "power" to approve or disapprove such 2007, Senator Miriam Defensor-Santiago delivered a privilege speech,
regulations before they take effect. (AKBAYAN v. Hon. Purisima, supra) It articulating her considered view that the Constitution absolutely bans
is a means whereby the legislature can block or modify administrative the use, possession, replay or communication of the contents of the
action taken under a statute. It is a form of legislative control in the "Hello Garci" tapes. However, she recommended a legislative
implementation of particular executive action. investigation into the role of the Intelligence Service of the AFP
(ISAFP), the Philippine National Police or other government entities in
ii. The power of inquiry (Art. VI, Sec. 21) the alleged illegal wiretapping of public officials.
Section 21. The Senate or the House of Representatives or any of its On October 26, 2007, Maj. Lindsay Rex Sagge, a member of the ISAFP
respective committees may conduct inquiries in aid of legislation in accordance and one of the resource persons summoned by the Senate to appear
with its duly published rules of procedure. The rights of persons appearing in, and testify at its hearings, moved to intervene as petitioner in G.R.
or affected by, such inquiries shall be respected. No. 179275.

M.R.A.D.C. LUMBRE 61
CONSTITUTIONAL LAW REVIEW

RULING: The Court dismissed the first petition, G.R. No. 170338, However, in the conduct of its day-to-day business the Senate of each
and granted the second, G.R. No. 179275. Congress acts separately and independently of the Senate of the
Congress before it.
The Senate cannot be allowed to continue with the conduct of the
questioned legislative inquiry without duly published rules of The language of Section 21, Article VI of the Constitution requiring
procedure, in clear derogation of the constitutional requirement. that the inquiry be conducted in accordance with the duly published
rules of procedure is categorical. It is incumbent upon the Senate to
Section 21, Article VI of the 1987 Constitution explicitly provides that publish the rules for its legislative inquiries in each Congress or
"[t]he Senate or the House of Representatives, or any of its respective otherwise make the published rules clearly state that the same shall
committees may conduct inquiries in aid of legislation in accordance be effective in subsequent Congresses or until they are amended or
with its duly published rules of procedure." The requisite of publication repealed to sufficiently put public on notice.
of the rules is intended to satisfy the basic requirements of due
process. Publication is indeed imperative, for it will be the height of If it was the intention of the Senate for its present rules on legislative
injustice to punish or otherwise burden a citizen for the transgression inquiries to be effective even in the next Congress, it could have easily
of a law or rule of which he had no notice whatsoever, not even a adopted the same language it had used in its main rules regarding
constructive one. What constitutes publication is set forth in Article 2 effectivity.
of the Civil Code, which provides that "[l]aws shall take effect after
15 days following the completion of their publication either in the The absence of any amendment to the rules cannot justify the
Official Gazette, or in a newspaper of general circulation in the Senate’s defiance of the clear and unambiguous language of Section
Philippines." 21, Article VI of the Constitution. The organic law instructs, without
more, that the Senate or its committees may conduct inquiries in aid
The respondents in G.R. No. 179275 admit in their pleadings and even of legislation only in accordance with duly published rules of
on oral argument that the Senate Rules of Procedure Governing procedure, and does not make any distinction whether or not these
Inquiries in Aid of Legislation had been published in newspapers of rules have undergone amendments or revision. The constitutional
general circulation only in 1995 and in 2006. With respect to the mandate to publish the said rules prevails over any custom, practice
present Senate of the 14th Congress, however, of which the term of or tradition followed by the Senate.
half of its members commenced on June 30, 2007, no effort was
undertaken for the publication of these rules when they first opened The invocation by the respondents of the provisions of R.A. No. 8792,
their session. otherwise known as the Electronic Commerce Act of 2000, to support
their claim of valid publication through the internet is all the more
Recently, the Court had occasion to rule on this very same question. incorrect. R.A. 8792 considers an electronic data message or an
In Neri v. Senate Committee on Accountability of Public Officers and electronic document as the functional equivalent of a written
Investigations, we said: document only for evidentiary purposes. In other words, the law
merely recognizes the admissibility in evidence (for their being the
Fourth, we find merit in the argument of the OSG that respondent original) of electronic data messages and/or electronic documents. It
Committees likewise violated Section 21 of Article VI of the does not make the internet a medium for publishing laws, rules and
Constitution, requiring that the inquiry be in accordance with the regulations.
"duly published rules of procedure."
Very recently, the Senate caused the publication of the Senate Rules
The phrase "duly published rules of procedure" requires the Senate of of Procedure Governing Inquiries in Aid of Legislation in the October
every Congress to publish its rules of procedure governing inquiries 31, 2008 issues of Manila Bulletin and Malaya. While we take judicial
in aid of legislation because every Senate is distinct from the one notice of this fact, the recent publication does not cure the infirmity
before it or after it. Since Senatorial elections are held every three (3) of the inquiry sought to be prohibited by the instant petitions. Insofar
years for one-half of the Senate’s membership, the composition of the as the consolidated cases are concerned, the legislative investigation
Senate also changes by the end of each term. Each Senate may thus subject thereof still could not be undertaken by the respondent
enact a different set of rules as it may deem fit. Not having published Senate Committees, because no published rules governed it, in clear
its Rules of Procedure, the subject hearings in aid of legislation contravention of the Constitution.
conducted by the 14th Senate, are therefore, procedurally infirm.
NOTE: Three (3) requirements in order for legislative inquiry to be
On the nature of the Senate as a "continuing body," this Court sees valid:
fit to issue a clarification. Certainly, there is no debate that the Senate
as an institution is "continuing," as it is not dissolved as an entity with (1) The inquiry should be in aid of legislation;
each national election or change in the composition of its members.

M.R.A.D.C. LUMBRE 62
CONSTITUTIONAL LAW REVIEW

(2) The Rules of Procedure should be duly published; and RULING: E.O. 464, to the extent that it bars the appearance of executive
officials before Congress, deprives Congress of the information in the
(3) The rights of persons appearing in, or affected by, such inquiries possession of these officials. To resolve the question of whether such
shall be respected. withholding of information violates the Constitution, consideration of the
2. Executive Privilege; Kinds – general power of Congress to obtain information, otherwise known as the
power of inquiry, is in order.
Senate of the Phil. vs. Ermita, GR No. 169777, 20 April 2006
The Congress power of inquiry is expressly recognized in Section 21 of
FACTS: The present consolidated petitions for certiorari and prohibition Article VI of the Constitution which reads:
proffer that the President has abused such power by issuing Executive
Order No. 464 (E.O. 464) last September 28, 2005. They thus pray for its SECTION 21. The Senate or the House of Representatives or any of
declaration as null and void for being unconstitutional. its respective committees may conduct inquiries in aid of legislation
in accordance with its duly published rules of procedure. The rights of
In the exercise of its legislative power, the Senate of the Philippines, persons appearing in or affected by such inquiries shall be respected.
through its various Senate Committees, conducts inquiries or
investigations in aid of legislation which call for, inter alia, the attendance As discussed in Arnault, the power of inquiry, "with process to enforce it"
of officials and employees of the executive department, bureaus, and is grounded on the necessity of information in the legislative process. If
offices including those employed in Government Owned and Controlled the information possessed by executive officials on the operation of their
Corporations, the Armed Forces of the Philippines (AFP), and the Philippine offices is necessary for wise legislation on that subject, by parity of
National Police (PNP). The subject matters of the inquiries include the reasoning, Congress has the right to that information and the power to
following: (1) the railway project of the North Luzon Railways Corporation compel the disclosure thereof.
with the China National Machinery and Equipment Group (North Rail That this power of inquiry is broad enough to cover officials of the
project); the so-called “Gloria-gate Scandal;” as well as (3) the wire- executive branch may be deduced from the same case. The power of
tapping of the President. inquiry, the Court therein ruled, is co-extensive with the power to
Thus, on September 28, 2005, the President issued E.O. 464, "Ensuring legislate. The matters which may be a proper subject of legislation and
Observance of the Principle of Separation of Powers, Adherence to the those which may be a proper subject of investigation are one. It follows
Rule on Executive Privilege and Respect for the Rights of Public Officials that the operation of government, being a legitimate subject for
Appearing in Legislative Inquiries in Aid of Legislation Under the legislation, is a proper subject for investigation.
Constitution, and For Other Purposes," which, pursuant to Section 6 Even where the inquiry is in aid of legislation, there are still recognized
thereof, took effect immediately. exemptions to the power of inquiry, which exemptions fall under the rubric
Senate President Drilon received from Executive Secretary Ermita a copy of "executive privilege." Since this term figures prominently in the
of E.O. 464, and another letter informing him "that officials of the challenged order, it being mentioned in its provisions, its preambular
Executive Department invited to appear at the meeting [regarding the clauses, and in its very title, a discussion of executive privilege is crucial
North Rail project] will not be able to attend the same without the consent for determining the constitutionality of E.O. 464.
of the President, pursuant to [E.O. 464]" and that "said officials have not Schwartz defines executive privilege as "the power of the Government to
secured the required consent from the President." withhold information from the public, the courts, and the Congress."
On even date which was also the scheduled date of the hearing on the Similarly, Rozell defines it as "the right of the President and high-level
alleged wiretapping, Gen. Senga sent a letter to Senator Biazon, executive branch officers to withhold information from Congress, the
Chairperson of the Committee on National Defense and Security, informing courts, and ultimately the public."
him "that per instruction of [President Arroyo], thru the Secretary of Executive privilege is, nonetheless, not a clear or unitary concept. It has
National Defense, no officer of the [AFP] is authorized to appear before encompassed claims of varying kinds. Tribe, in fact, comments that while
any Senate or Congressional hearings without seeking a written approval it is customary to employ the phrase "executive privilege," it may be more
from the President" and "that no approval has been granted by the accurate to speak of executive privileges "since presidential refusals to
President to any AFP officer to appear before the public hearing of the furnish information may be actuated by any of at least three distinct kinds
Senate Committee on National Defense and Security scheduled [on] 28 of considerations, and may be asserted, with differing degrees of success,
September 2005." in the context of either judicial or legislative investigations."
ISSUE: Whether E.O. 464 contravenes the power of inquiry vested in One variety of the privilege, Tribe explains, is the state secrets privilege
Congress; thus, unconstitutional. invoked by U.S. Presidents, beginning with Washington, on the ground

M.R.A.D.C. LUMBRE 63
CONSTITUTIONAL LAW REVIEW

that the information is of such nature that its disclosure would subvert mere fact of being executive officials. Indeed, the extraordinary character
crucial military or diplomatic objectives. Another variety is the informer’s of the exemptions indicates that the presumption inclines heavily against
privilege, or the privilege of the Government not to disclose the identity executive secrecy and in favor of disclosure.
of persons who furnish information of violations of law to officers charged
with the enforcement of that law. Finally, a generic privilege for internal Validity of Section 1
deliberations has been said to attach to intra-governmental documents Section 1 is similar to Section 3 in that both require the officials covered
reflecting advisory opinions, recommendations and deliberations by them to secure the consent of the President prior to appearing before
comprising part of a process by which governmental decisions and policies Congress. There are significant differences between the two provisions,
are formulated. however, which constrain this Court to discuss the validity of these
This privilege, based on the constitutional doctrine of separation of provisions separately.
powers, exempts the executive from disclosure requirements applicable to Section 1 specifically applies to department heads. It does not, unlike
the ordinary citizen or organization where such exemption is necessary to Section 3, require a prior determination by any official whether they are
the discharge of highly important executive responsibilities involved in covered by E.O. 464. The President herself has, through the challenged
maintaining governmental operations, and extends not only to military order, made the determination that they are. Further, unlike also Section
and diplomatic secrets but also to documents integral to an appropriate 3, the coverage of department heads under Section 1 is not made to
exercise of the executive’ domestic decisional and policy making functions, depend on the department heads’ possession of any information which
that is, those documents reflecting the frank expression necessary in might be covered by executive privilege. In fact, in marked contrast to
intra-governmental advisory and deliberative communications. Section 3 vis-à-vis Section 2, there is no reference to executive privilege
That a type of information is recognized as privileged does not, however, at all. Rather, the required prior consent under Section 1 is grounded on
necessarily mean that it would be considered privileged in all instances. Article VI, Section 22 of the Constitution on what has been referred to as
For in determining the validity of a claim of privilege, the question that the question hour.
must be asked is not only whether the requested information falls within SECTION 22. The heads of departments may upon their own initiative,
one of the traditional privileges, but also whether that privilege should be with t he consent of the President, or upon the request of either
honored in a given procedural setting. House, as the rules of each House shall provide, appear before and
In Chavez v. PCGG, the Court held that this jurisdiction recognizes the be heard by such House on any matter pertaining to their
common law holding that there is a "governmental privilege against public departments. Written questions shall be submitted to the President of
disclosure with respect to state secrets regarding military, diplomatic and the Senate or the Speaker of the House of Representatives at least
other national security matters." The same case held that closed-door three days before their scheduled appearance. Interpellations shall
Cabinet meetings are also a recognized limitation on the right to not be limited to written questions, but may cover matters related
information. thereto. When the security of the State or the public interest so
requires and the President so states in writing, the appearance shall
Similarly, in Chavez v. Public Estates Authority, the Court ruled that the be conducted in executive session.
right to information does not extend to matters recognized as "privileged
information under the separation of powers,” by which the Court meant Determining the validity of Section 1 thus requires an examination of the
Presidential conversations, correspondences, and discussions in closed- meaning of Section 22 of Article VI. Section 22 which provides for the
door Cabinet meetings. It also held that information on military and question hour must be interpreted vis-à-vis Section 21 which provides for
diplomatic secrets and those affecting national security, and information the power of either House of Congress to "conduct inquiries in aid of
on investigations of crimes by law enforcement agencies before the legislation."
prosecution of the accused were exempted from the right to information. Sections 21 and 22, therefore, while closely related and complementary
From the above discussion on the meaning and scope of executive to each other, should not be considered as pertaining to the same power
privilege, both in the United States and in this jurisdiction, a clear principle of Congress. One specifically relates to the power to conduct inquiries in
emerges. Executive privilege, whether asserted against Congress, the aid of legislation, the aim of which is to elicit information that may be used
courts, or the public, is recognized only in relation to certain types of for legislation, while the other pertains to the power to conduct a question
information of a sensitive character. hour, the objective of which is to obtain information in pursuit of Congress’
oversight function.
While executive privilege is a constitutional concept, a claim thereof may
be valid or not depending on the ground invoked to justify it and the When Congress merely seeks to be informed on how department heads
context in which it is made. Noticeably absent is any recognition that are implementing the statutes which it has issued, its right to such
executive officials are exempt from the duty to disclose information by the information is not as imperative as that of the President to whom, as Chief

M.R.A.D.C. LUMBRE 64
CONSTITUTIONAL LAW REVIEW

Executive, such department heads must give a report of their performance relation to specific categories of information and not to categories of
as a matter of duty. In such instances, Section 22, in keeping with the persons.
separation of powers, states that Congress may only request their
appearance. Nonetheless, when the inquiry in which Congress requires Section 3 of E.O. 464, therefore, cannot be dismissed outright as invalid
their appearance is "in aid of legislation" under Section 21, the appearance by the mere fact that it sanctions claims of executive privilege. This Court
is mandatory for the same reasons stated in Arnault. must look further and assess the claim of privilege authorized by the Order
to determine whether it is valid.
In fine, the oversight function of Congress may be facilitated by
compulsory process only to the extent that it is performed in pursuit of While the validity of claims of privilege must be assessed on a case to case
legislation. This is consistent with the intent discerned from the basis, examining the ground invoked therefor and the particular
deliberations of the Constitutional Commission. circumstances surrounding it, there is, in an implied claim of privilege, a
defect that renders it invalid per se. By its very nature, and as
Ultimately, the power of Congress to compel the appearance of executive demonstrated by the letter of respondent Executive Secretary quoted
officials under Section 21 and the lack of it under Section 22 find their above, the implied claim authorized by Section 3 of E.O. 464 is not
basis in the principle of separation of powers. While the executive branch accompanied by any specific allegation of the basis thereof (e.g., whether
is a co-equal branch of the legislature, it cannot frustrate the power of the information demanded involves military or diplomatic secrets, closed-
Congress to legislate by refusing to comply with its demands for door Cabinet meetings, etc.). While Section 2(a) enumerates the types of
information. information that are covered by the privilege under the challenged order,
Congress is left to speculate as to which among them is being referred to
When Congress exercises its power of inquiry, the only way for department by the executive. The enumeration is not even intended to be
heads to exempt themselves therefrom is by a valid claim of privilege. comprehensive, but a mere statement of what is included in the phrase
They are not exempt by the mere fact that they are department heads. "confidential or classified information between the President and the public
Only one executive official may be exempted from this power — the officers covered by this executive order."
President on whom executive power is vested, hence, beyond the reach of
Congress except through the power of impeachment. It is based on her Certainly, Congress has the right to know why the executive considers the
being the highest official of the executive branch, and the due respect requested information privileged. It does not suffice to merely declare that
accorded to a co-equal branch of government which is sanctioned by a the President, or an authorized head of office, has determined that it is
long-standing custom. so, and that the President has not overturned that determination. Such
declaration leaves Congress in the dark on how the requested information
Section 1, in view of its specific reference to Section 22 of Article VI of the could be classified as privileged. That the message is couched in terms
Constitution and the absence of any reference to inquiries in aid of that, on first impression, do not seem like a claim of privilege only makes
legislation, must be construed as limited in its application to appearances it more pernicious. It threatens to make Congress doubly blind to the
of department heads in the question hour contemplated in the provision question of why the executive branch is not providing it with the
of said Section 22 of Article VI. The reading is dictated by the basic rule information that it has requested.
of construction that issuances must be interpreted, as much as possible,
in a way that will render it constitutional. A claim of privilege, being a claim of exemption from an obligation to
disclose information, must, therefore, be clearly asserted. Upon the other
The requirement then to secure presidential consent under Section 1, hand, Congress must not require the executive to state the reasons for
limited as it is only to appearances in the question hour, is valid on its the claim with such particularity as to compel disclosure of the information
face. For under Section 22, Article VI of the Constitution, the appearance which the privilege is meant to protect.
of department heads in the question hour is discretionary on their part.
The claim of privilege under Section 3 of E.O. 464 in relation to Section
Section 1 cannot, however, be applied to appearances of department 2(b) is thus invalid per se. It is not asserted. It is merely implied. Instead
heads in inquiries in aid of legislation. Congress is not bound in such of providing precise and certain reasons for the claim, it merely invokes
instances to respect the refusal of the department head to appear in such E.O. 464, coupled with an announcement that the President has not given
inquiry, unless a valid claim of privilege is subsequently made, either by her consent. It is woefully insufficient for Congress to determine whether
the President herself or by the Executive Secretary. the withholding of information is justified under the circumstances of each
Validity of Sections 2 and 3 case. It severely frustrates the power of inquiry of Congress.

En passant, the Court notes that Section 2(b) of E.O. 464 virtually states In fine, Section 3 and Section 2(b) of E.O. 464 must be invalidated.
that executive privilege actually covers persons. Such is a misuse of the No infirmity, however, can be imputed to Section 2(a) as it merely
doctrine. Executive privilege, as discussed above, is properly invoked in provides guidelines, binding only on the heads of office mentioned in

M.R.A.D.C. LUMBRE 65
CONSTITUTIONAL LAW REVIEW

Section 2(b), on what is covered by executive privilege. It does not purport him not to accept the bribe. However, when probed further on
to be conclusive on the other branches of government. It may thus be President Arroyo and petitioner’s discussions relating to the NBN
construed as a mere expression of opinion by the President regarding the Project, petitioner refused to answer, invoking "executive privilege."
nature and scope of executive privilege. To be specific, petitioner refused to answer questions on: (a) whether
or not President Arroyo followed up the NBN Project, (b) whether or
On Necessity of Publication: While E.O. 464 applies only to officials of not she directed him to prioritize it, and (c) whether or not she
the executive branch, it does not follow that the same is exempt from the directed him to approve it.
need for publication.
Respondent Committees found petitioner’s explanations
NOTE: The 3 kinds of executive privilege: (1) state secrets; (2) informer’s unsatisfactory. Without responding to his request for advance notice
privilege; and (3) generic privilege on internal deliberations. State secrets of the matters that he should still clarify, they issued the Order citing
refer to military, diplomatic, and other national security matters, i.e. petitioner in contempt of respondent Committees and ordering his
closed-door cabinet meetings, presidential conversations and arrest and detention at the Office of the Senate Sergeant-at-Arms
correspondence). until such time that he would appear and give his testimony.
Question Hour Inquiries in Aid of Petitioner, after filing a Motion for Reconsideration, then filed his
(Sec. 22) Legislation Supplemental Petition for Certiorari (with Urgent Application for
(Sec. 21) TRO/Preliminary Injunction), which the Court granted. Thus, this
Attendance Attendance is Attendance is present Motion for Reconsideration.
discretionary; Congress compulsory.
may only request XPN: Executive ISSUE: Whether there is a recognized presumptive presidential
attendance. Privilege. communications privilege.
Purpose Obtain information in Elicit information that RULING: Yes. Senate v. Ermita expounds on the premise of the
pursuit of may be used for foregoing ruling in this wise:
Congressional legislation.
oversight. Section 2(b) in relation to Section 3 virtually provides that, once the
head of office determines that a certain information is privileged, such
a. Presumptive Presidential Communication Privilege – determination is presumed to bear the President’s authority and has
the effect of prohibiting the official from appearing before Congress,
Neri vs. Senate, 5 September 2008
subject only to the express pronouncement of the President that it is
FACTS: On November 20, 2007, petitioner did not appear before allowing the appearance of such official. These provisions thus allow
respondent Committees upon orders of the President invoking the President to authorize claims of privilege by mere silence.
executive privilege. On November 22, 2007, the respondent
Such presumptive authorization, however, is contrary to the
Committees issued the show-cause letter requiring him to explain why
he should not be cited in contempt. On November 29, 2007, in exceptional nature of the privilege. Executive privilege, as already
discussed, is recognized with respect to information the confidential
petitioner’s reply to respondent Committees, he manifested that it
was not his intention to ignore the Senate hearing and that he thought nature of which is crucial to the fulfillment of the unique role and
responsibilities of the executive branch, or in those instances where
the only remaining questions were those he claimed to be covered by
executive privilege. He also manifested his willingness to appear and exemption from disclosure is necessary to the discharge of highly
important executive responsibilities. The doctrine of executive
testify should there be new matters to be taken up. He just requested
that he be furnished "in advance as to what else" he "needs to clarify." privilege is thus premised on the fact that certain information must,
as a matter of necessity, be kept confidential in pursuit of the public
On September 26, 2007, petitioner appeared before respondent interest. The privilege being, by definition, an exemption from the
Committees and testified for about eleven (11) hours on matters obligation to disclose information, in this case to Congress, the
concerning the National Broadband Project (the "NBN Project"), a necessity must be of such high degree as to outweigh the public
project awarded by the Department of Transportation and interest in enforcing that obligation in a particular case.
Communications ("DOTC") to Zhong Xing Telecommunications
Equipment ("ZTE"). Petitioner disclosed that then Commission on In light of this highly exceptional nature of the privilege, the Court
finds it essential to limit to the President the power to invoke the
Elections ("COMELEC") Chairman Benjamin Abalos offered him P200
Million in exchange for his approval of the NBN Project. He further privilege. She may of course authorize the Executive Secretary to
invoke the privilege on her behalf, in which case the Executive
narrated that he informed President Gloria Macapagal Arroyo
Secretary must state that the authority is "By order of the President",
("President Arroyo") of the bribery attempt and that she instructed

M.R.A.D.C. LUMBRE 66
CONSTITUTIONAL LAW REVIEW

which means that he personally consulted with her. The privilege essence of substance. On the other hand, "non-delegable" means that
being an extraordinary power, it must be wielded only by the highest a power or duty cannot be delegated to another or, even if delegated,
official in the executive hierarchy. In other words, the President may the responsibility remains with the obligor. The power to enter into
not authorize her subordinates to exercise such power. There is even an executive agreement is in essence an executive power. This
less reason to uphold such authorization in the instant case where the authority of the President to enter into executive agreements without
authorization is not explicit but by mere silence. Section 3, in relation the concurrence of the Legislature has traditionally been recognized
to Section 2(b), is further invalid on this score. in Philippine jurisprudence. Now, the fact that the President has to
secure the prior concurrence of the Monetary Board, which shall
The constitutional infirmity found in the blanket authorization to submit to Congress a complete report of its decision before
invoke executive privilege granted by the President to executive contracting or guaranteeing foreign loans, does not diminish the
officials in Sec. 2(b) of E.O. No. 464 does not obtain in this case. executive nature of the power.
In this case, it was the President herself, through Executive Secretary The "doctrine of operational proximity" was laid down precisely to
Ermita, who invoked executive privilege on a specific matter involving limit the scope of the presidential communications privilege but, in
an executive agreement between the Philippines and China, which any case, it is not conclusive.
was the subject of the three (3) questions propounded to petitioner
Neri in the course of the Senate Committees’ investigation. Thus, the In the case at bar, the danger of expanding the privilege "to a large
factual setting of this case markedly differs from that passed upon in swath of the executive branch" (a fear apparently entertained by
Senate v. Ermita. respondents) is absent because the official involved here is a member
of the Cabinet, thus, properly within the term "advisor" of the
Senate v. Ermita expounds on the constitutional underpinning of the President; in fact, her alter ego and a member of her official family.
relationship between the Executive Department and the Legislative Nevertheless, in circumstances in which the official involved is far too
Department to explain why there should be no implied authorization remote, this Court also mentioned in the Decision the organizational
or presumptive authorization to invoke executive privilege by the test laid down in Judicial Watch, Inc. v. Department of Justice. This
President’s subordinate officials, as follows: goes to show that the operational proximity test used in the Decision
When Congress exercises its power of inquiry, the only way for is not considered conclusive in every case. In determining which test
department heads to exempt themselves therefrom is by a valid claim to use, the main consideration is to limit the availability of executive
of privilege. They are not exempt by the mere fact that they are privilege only to officials who stand proximate to the President, not
department heads. Only one executive official may be exempted from only by reason of their function, but also by reason of their positions
this power - the President on whom executive power is vested, hence, in the Executive’s organizational structure. Thus, respondent
beyond the reach of Congress except through the power of Committees’ fear that the scope of the privilege would be
impeachment. It is based on he being the highest official of the unnecessarily expanded with the use of the operational proximity test
executive branch, and the due respect accorded to a co-equal branch is unfounded.
of governments which is sanctioned by a long-standing custom. The President’s claim of executive privilege is not merely based on a
Thus, if what is involved is the presumptive privilege of presidential generalized interest; and in balancing respondent Committees’ and
communications when invoked by the President on a matter clearly the President’s clashing interests, the Court did not disregard the
within the domain of the Executive, the said presumption dictates that 1987 Constitutional provisions on government transparency,
the same be recognized and be given preference or priority, in the accountability and disclosure of information.
absence of proof of a compelling or critical need for disclosure by the It must be stressed that the President’s claim of executive privilege is
one assailing such presumption. Any construction to the contrary will not merely founded on her generalized interest in confidentiality. The
render meaningless the presumption accorded by settled Letter dated November 15, 2007 of Executive Secretary Ermita
jurisprudence in favor of executive privilege. In fact, Senate v. Ermita specified presidential communications privilege in relation to
reiterates jurisprudence citing "the considerations justifying a diplomatic and economic relations with another sovereign nation as
presumptive privilege for Presidential communications." the bases for the claim.
The power to enter into an executive agreement is a “quintessential The context in which executive privilege is being invoked is that the
and non-delegable presidential power.” information sought to be disclosed might impair our diplomatic as well
The fact that a power is subject to the concurrence of another entity as economic relations with the People’s Republic of China. Given the
does not make such power less executive. "Quintessential" is defined confidential nature in which this information were conveyed to the
as the most perfect embodiment of something, the concentrated President, he cannot provide the Committee any further details of

M.R.A.D.C. LUMBRE 67
CONSTITUTIONAL LAW REVIEW

these conversations, without disclosing the very thing the privilege is the courts to determine on a case by case basis whether the matter
designed to protect. at issue is of interest or importance, as it relates to or affects the
public.
This Court did not rule that the Senate has no power to investigate
the NBN Project in aid of legislation. There is nothing in the assailed From the nature of the JPEPA as an international trade agreement, it
Decision that prohibits respondent Committees from inquiring into the is evident that the Philippine and Japanese offers submitted during
NBN Project. They could continue the investigation and even call the negotiations towards its execution are matters of public concern.
petitioner Neri to testify again. He himself has repeatedly expressed This, respondents do not dispute. They only claim that diplomatic
his willingness to do so. Our Decision merely excludes from the scope negotiations are covered by the doctrine of executive privilege, thus
of respondents’ investigation the three (3) questions that elicit constituting an exception to the right to information and the policy of
answers covered by executive privilege and rules that petitioner full public disclosure.
cannot be compelled to appear before respondents to answer the said
questions. We have discussed the reasons why these answers are It is well-established in jurisprudence that neither the right to
covered by executive privilege. That there is a recognized public information nor the policy of full public disclosure is absolute, there
interest in the confidentiality of such information is a recognized being matters which, albeit of public concern or public interest, are
principle in other democratic States. To put it simply, the right to recognized as privileged in nature.
information is not an absolute right. The documents on the proposed JPEPA as well as the text which is
b. Offers in treaty negotiation – subject to negotiations and legal review by the parties fall under the
exceptions to the right of access to information on matters of public
AKBAYAN, et al. vs. Thomas Aquino in His Capacity as concern and policy of public disclosure. They come within the
Undersecretary of the Department of Trade, et al., GR No. coverage of executive privilege. At the time when the Committee was
170516, 16 July 2008 requesting for copies of such documents, the negotiations were
ongoing as they are still now and the text of the proposed JPEPA is
FACTS: Petitioners – non-government organizations, still uncertain and subject to change. Considering the status and
Congresspersons, citizens and taxpayers – seek via the present nature of such documents then and now, these are evidently covered
petition for mandamus and prohibition to obtain from respondents the by executive privilege consistent with existing legal provisions and
full text of the Japan-Philippines Economic Partnership Agreement settled jurisprudence.
(JPEPA) including the Philippine and Japanese offers submitted during
the negotiation process and all pertinent attachments and annexes Practical and strategic considerations likewise counsel against the
disclosure of the "rolling texts" which may undergo radical change or
thereto.
portions of which may be totally abandoned. Furthermore, the
Petitioners assert, first, that the refusal of the government to disclose negotiations of the representatives of the Philippines as well as of
the documents bearing on the JPEPA negotiations violates their right Japan must be allowed to explore alternatives in the course of the
to information on matters of public concern and contravenes other negotiations in the same manner as judicial deliberations and working
constitutional provisions on transparency, such as that on the policy drafts of opinions are accorded strict confidentiality.
of full public disclosure of all transactions involving public interest.
Applying the principles adopted in PMPF v. Manglapus, it is clear that
Second, they contend that non-disclosure of the same documents
while the final text of the JPEPA may not be kept perpetually
undermines their right to effective and reasonable participation in all
confidential – since there should be "ample opportunity for discussion
levels of social, political, and economic decision-making. Lastly, they
before [a treaty] is approved" – the offers exchanged by the parties
proffer that divulging the contents of the JPEPA only after the
during the negotiations continue to be privileged even after the JPEPA
agreement has been concluded will effectively make the Senate into
is published. It is reasonable to conclude that the Japanese
a mere rubber stamp of the Executive, in violation of the principle of
representatives submitted their offers with the understanding that
separation of powers.
"historic confidentiality" would govern the same. Disclosing these
RULING: Petition dismissed. In determining whether or not a offers could impair the ability of the Philippines to deal not only with
particular information is of public concern there is no rigid test which Japan but with other foreign governments in future negotiations.
can be applied. ‘Public concern’ like ‘public interest’ is a term that
A ruling that Philippine offers in treaty negotiations should now be
eludes exact definition. Both terms embrace a broad spectrum of
open to public scrutiny would discourage future Philippine
subjects which the public may want to know, either because these
representatives from frankly expressing their views during
directly affect their lives, or simply because such matters naturally
negotiations. While, on first impression, it appears wise to deter
arouse the interest of an ordinary citizen. In the final analysis, it is for
Philippine representatives from entering into compromises, it bears

M.R.A.D.C. LUMBRE 68
CONSTITUTIONAL LAW REVIEW

noting that treaty negotiations, or any negotiation for that matter, the principle that it is not a trier of facts at first instance, is averse to
normally involve a process of quid pro quo, and oftentimes making any authoritative findings of fact, for that function is first for the
negotiators have to be willing to grant concessions in an area of lesser court-martial court to fulfill.
importance in order to obtain more favorable terms in an area of
greater national interest. The vitality of the tenet that the President is the commander-in-chief of
the Armed Forces is most crucial to the democratic way of life, to civilian
Diplomatic negotiations, therefore, are recognized as privileged in this
supremacy over the military, and to the general stability of our
jurisdiction, the JPEPA negotiations constituting no exception. It bears
representative system of government. The Constitution reposes final
emphasis, however, that such privilege is only presumptive. For as
authority, control and supervision of the AFP to the President, a civilian
Senate v. Ermita holds, recognizing a type of information as privileged
who is not a member of the armed forces, and whose duties as
does not mean that it will be considered privileged in all instances.
commander-in-chief represent only a part of the organic duties imposed
Only after a consideration of the context in which the claim is made
upon the office, the other functions being clearly civil in nature. Civilian
may it be determined if there is a public interest that calls for the
supremacy over the military also countermands the notion that the
disclosure of the desired information, strong enough to overcome its
military may bypass civilian authorities, such as civil courts, on matters
traditionally privileged status.
such as conducting warrantless searches and seizures.
Going back to the present case, the Court recognizes that the
Pursuant to the maintenance of civilian supremacy over the military, the
information sought by petitioners includes documents produced and
Constitution has allocated specific roles to the legislative and executive
communicated by a party external to the Philippine government,
branches of government in relation to military affairs. Military
namely, the Japanese representatives in the JPEPA negotiations, and
appropriations, as with all other appropriations, are determined by
to that extent this case is closer to the factual circumstances of CIEL
Congress, as is the power to declare the existence of a state of war.
than those of Fulbright.
Congress is also empowered to revoke a proclamation of martial law or
3. Commander-in-Chief Clause – the suspension of the writ of habeas corpus. The approval of the
Commission on Appointments is also required before the President can
Gudani vs. Senga, GR No. 170561, 15 August 2006 promote military officers from the rank of colonel or naval captain.
FACTS: Petitioners were called by the Senate Committee to testify in its Otherwise, on the particulars of civilian dominance and administration
over the military, the Constitution is silent, except for the commander-in-
28 September 2005 hearing (related to the Senate v. Ermita case).
Petitioners attended such hearing and testified before the Committee, chief clause which is fertile in meaning and implication as to whatever
inherent martial authority the President may possess.
despite the fact that the day before, there was an order from Gen. Senga
(which in turn was sourced "per instruction" from President Arroyo) The commander-in-chief provision in the Constitution is denominated as
prohibiting them from testifying without the prior approval of the Section 18, Article VII, which begins with the simple declaration that "[t]he
President. President shall be the Commander-in-Chief of all armed forces of the
Philippines x x x" Outside explicit constitutional limitations, such as those
Petitioners seek the annulment of a directive from President Gloria
Macapagal-Arroyo enjoining them and other military officers from found in Section 5, Article XVI, the commander-in-chief clause vests on
the President, as commander-in-chief, absolute authority over the persons
testifying before Congress without the President’s consent. Petitioners also
pray for injunctive relief against a pending preliminary investigation and actions of the members of the armed forces. Such authority includes
the ability of the President to restrict the travel, movement and speech of
against them, in preparation for possible court-martial proceedings,
initiated within the military justice system in connection with petitioners’ military officers, activities which may otherwise be sanctioned under
civilian law.
violation of the aforementioned directive.
RULING: Petition denied. Notably, the guilt or innocence of petitioners in Critical to military discipline is obeisance to the military chain of command.
Willful disobedience of a superior officer is punishable by court-martial
violating Articles 65 and 97 of the Articles of War is not an issue before
under Article 65 of the Articles of War. "An individual soldier is not free to
this Court, especially considering that per records, petitioners have not
ignore the lawful orders or duties assigned by his immediate superiors.
yet been subjected to court martial proceedings. Owing to the absence of For there would be an end of all discipline if the seaman and marines on
such proceedings, the correct inquiry should be limited to whether board a ship of war [or soldiers deployed in the field], on a distant service,
respondents could properly initiate such proceedings preparatory to a were permitted to act upon their own opinion of their rights [or their
formal court-martial, such as the aforementioned preliminary opinion of the President’s intent], and to throw off the authority of the
investigation, on the basis of petitioners’ acts surrounding their testimony commander whenever they supposed it to be unlawfully exercised."
before the Senate on 28 September 2005. Yet this Court, consistent with

M.R.A.D.C. LUMBRE 69
CONSTITUTIONAL LAW REVIEW

Further traditional restrictions on members of the armed forces are those prerogatives of the President as commander-in-chief. Congress holds
imposed on free speech and mobility. Kapunan is ample precedent in significant control over the armed forces in matters such as budget
justifying that a soldier may be restrained by a superior officer from appropriations and the approval of higher-rank promotions, yet it is on the
speaking out on certain matters. As a general rule, the discretion of a President that the Constitution vests the title as commander-in-chief and
military officer to restrain the speech of a soldier under his/her command all the prerogatives and functions appertaining to the position. Again, the
will be accorded deference, with minimal regard if at all to the reason for exigencies of military discipline and the chain of command mandate that
such restraint. It is integral to military discipline that the soldier’s speech the President’s ability to control the individual members of the armed
be with the consent and approval of the military commander. forces be accorded the utmost respect. Where a military officer is torn
between obeying the President and obeying the Senate, the Court will
The necessity of upholding the ability to restrain speech becomes even without hesitation affirm that the officer has to choose the President. After
more imperative if the soldier desires to speak freely on political matters. all, the Constitution prescribes that it is the President, and not the Senate,
The Constitution requires that "[t]he armed forces shall be insulated from who is the commander-in-chief of the armed forces.
partisan politics," and that ‘[n]o member of the military shall engage
directly or indirectly in any partisan political activity, except to vote." At the same time, the refusal of the President to allow members of the
Certainly, no constitutional provision or military indoctrination will military to appear before Congress is still subject to judicial relief. The
eliminate a soldier’s ability to form a personal political opinion, yet it is Constitution itself recognizes as one of the legislature’s functions is the
vital that such opinions be kept out of the public eye. For one, political conduct of inquiries in aid of legislation. Inasmuch as it is ill-advised for
belief is a potential source of discord among people, and a military torn by Congress to interfere with the President’s power as commander-in-chief,
political strife is incapable of fulfilling its constitutional function as it is similarly detrimental for the President to unduly interfere with
protectors of the people and of the State. For another, it is ruinous to Congress’s right to conduct legislative inquiries. The impasse did not come
military discipline to foment an atmosphere that promotes an active dislike to pass in this petition, since petitioners testified anyway despite the
of or dissent against the President, the commander-in-chief of the armed presidential prohibition. Yet the Court is aware that with its
forces. Soldiers are constitutionally obliged to obey a President they may pronouncement today that the President has the right to require prior
dislike or distrust. This fundamental principle averts the country from consent from members of the armed forces, the clash may soon loom or
going the way of banana republics. actualize.
Indeed, the military practice is to require a soldier to obtain permission We believe and hold that our constitutional and legal order sanctions a
from the commanding officer before he/she may leave his destination. A modality by which members of the military may be compelled to attend
soldier who goes from the properly appointed place of duty or absents legislative inquiries even if the President desires otherwise, a modality
from his/her command, guard, quarters, station, or camp without proper which does not offend the Chief Executive’s prerogatives as commander-
leave is subject to punishment by court-martial. It is even clear from the in-chief. The remedy lies with the courts.
record that petitioners had actually requested for travel authority from the
PMA in Baguio City to Manila, to attend the Senate Hearing. Even The fact that the executive branch is an equal, coordinate branch of
petitioners are well aware that it was necessary for them to obtain government to the legislative creates a wrinkle to any basic rule that
permission from their superiors before they could travel to Manila to persons summoned to testify before Congress must do so. There is
attend the Senate Hearing. considerable interplay between the legislative and executive branches,
informed by due deference and respect as to their various constitutional
As earlier noted, we ruled in Senate that the President may not issue a functions. Reciprocal courtesy idealizes this relationship; hence, it is only
blanket requirement of prior consent on executive officials summoned by as a last resort that one branch seeks to compel the other to a particular
the legislature to attend a congressional hearing. In doing so, the Court mode of behavior. The judiciary, the third coordinate branch of
recognized the considerable limitations on executive privilege, and government, does not enjoy a similar dynamic with either the legislative
affirmed that the privilege must be formally invoked on specified grounds. or executive branches. Whatever weakness inheres on judicial power due
However, the ability of the President to prevent military officers from to its inability to originate national policies and legislation, such is balanced
testifying before Congress does not turn on executive privilege, but on the by the fact that it is the branch empowered by the Constitution to compel
Chief Executive’s power as commander-in-chief to control the actions and obeisance to its rulings by the other branches of government.
speech of members of the armed forces. The President’s prerogatives as
commander-in-chief are not hampered by the same limitations as in Following these principles, it is clear that if the President or the Chief of
executive privilege. Staff refuses to allow a member of the AFP to appear before Congress, the
legislative body seeking such testimony may seek judicial relief to compel
Our ruling that the President could, as a general rule, require military the attendance. Such judicial action should be directed at the heads of the
officers to seek presidential approval before appearing before Congress is executive branch or the armed forces, the persons who wield authority
based foremost on the notion that a contrary rule unduly diminishes the and control over the actions of the officers concerned. The legislative

M.R.A.D.C. LUMBRE 70
CONSTITUTIONAL LAW REVIEW

purpose of such testimony, as well as any defenses against the same — the alleged immateriality of the information sought by the legislative body from
whether grounded on executive privilege, national security or similar a witness is relied upon to contest its jurisdiction, the court is in duty bound to
concerns — would be accorded due judicial evaluation. pass upon the contention. The fact that the legislative body has jurisdiction or
the power to make the inquiry would not preclude judicial intervention to
a. Remedy to compel attendance? correct a clear abuse of discretion in the exercise of that power.
Via judicial relief. (Gudani v. Senga, supra) Applying the criterion laid down in the last two preceding paragraphs to the
iii. Legislative contempt resolution of the issue under consideration, we find that the question for the
refusal to answer which the petitioner was held in contempt by the Senate is
1. Arnault vs. Nazareno, GR No. L-3820, 18 July 1950 pertinent to the matter under inquiry. In fact, this is not and cannot be
disputed. Senate Resolution No. 8, the validity of which is not challenged by
FACTS: This is an original petition for habeas corpus to relieve the petitioner
the petitioner, requires the Special Committee, among other things, to
from his confinement in the New Bilibid Prison to which he has been committed
determine the parties responsible for the Buenavista and Tambobong estates
by virtue of a resolution adopted by the Senate on May 15, 1950, which reads deal, and it is obvious that the name of the person to whom the witness gave
as follows: the P440,000 involved in said deal is pertinent to that determination — it is in
fact the very thing sought to be determined. The contention is not that the
Whereas, Jean L. Arnault refused to reveal the name of the person to whom question is impertinent to the subject of the inquiry but that it has no relation
he gave the P440,000, as well as answer other pertinent questions related to or materiality to any proposed legislation. We have already indicated that it is
the said amount; Now, therefore, be it. not necessary for the legislative body to show that every question propounded
to a witness is material to any proposed or possible legislation; what is required
Resolved, that for his refusal to reveal the name of the person to whom he is that is that it be pertinent to the matter under inquiry.
gave the P440,000 Jean L. Arnault be committed to the custody of the It is said that the Senate has already approved the three bills recommended
Sergeant-at-Arms and imprisoned in the New Bilibid Prison, Muntinlupa, Rizal, by the Committee as a result of the uncompleted investigation and that there
until discharged by further order of the Senate or by the special committee is no need for it to know the name of the person to whom the witness gave
created by Senate Resolution No. 8, such discharge to be ordered when he the P440,000. But aside from the fact that those bills have not yet been
shall have purged the contempt by revealing to the Senate or to the said approved by the lower house and by the President and that they may be
special committee the name of the person to whom he gave the P440,000, as withdrawn or modified if after the inquiry is completed they should be found
well as answer other pertinent questions in connection therewith. unnecessary or inadequate, there is nothing to prevent the Congress from
approving other measures it may deem necessary after completing the
RULING: Petition denied. Once an inquiry is admitted or established to be investigation. We are not called upon, nor is it within our province, to
within the jurisdiction of a legislative body to make, we think the investigating determine or imagine what those measures may be. And our inability to do so
committee has the power to require a witness to answer any question pertinent is no reason for overruling the question propounded by the Senate to the
to that inquiry, subject of course to his constitutional right against self- witness.
incrimination. The inquiry, to be within the jurisdiction of the legislative body
to make, must be material or necessary to the exercise of a power in it vested It may be contended that the determination of the parties responsible for the
by the Constitution, such as to legislate, or to expel a Member; and every deal is incumbent upon the judicial rather than upon the legislative branch. But
question which the investigator is empowered to coerce a witness to answer we think there is no basis in fact or in law for such assumption. The petitioner
must be material or pertinent to the subject of the inquiry or investigation. So has not challenged the validity of Senate Resolution No. 8, and that resolution
a witness may not be coerced to answer a question that obviously has no expressly requires the committee to determine the parties responsible for the
relation to the subject of the inquiry. But from this it does not follow that every deal. We are bound to presume that the Senate has acted in the due
question that may be propounded to a witness must be material to any performance of its constitutional function in instituting the inquiry, if the act is
proposed or possible legislation. In other words, the materiality of the question capable of being so construed. On the other hand, there is no suggestion that
must be determined by its direct relation to any proposed or possible the judiciary has instituted an inquiry to determine the parties responsible for
legislation. The reason is, that the necessity or lack of necessity for legislative the deal. Under the circumstances of the case, it appearing that the questioned
action and the form and character of the action itself are determined by the transaction was affected by the head of the Department of Justice himself, it
sum total of the information to be gathered as a result of the investigation, is not reasonable to expect that the Fiscal or the Court of First Instance of
and not by a fraction of such information elicited from a single question. Manila will take the initiative to investigate and prosecute the parties
responsible for the deal until and unless the Senate shall determined those
It is necessary deduction from the decision in Re Chapman, 41 L. ed., 1154, parties are and shall taken such measures as may be within its competence to
that where the questions are not pertinent to the matter under inquiry a take the redress the wrong that may have been committed against the people
witness rightfully may refuse to answer. So we are of the opinion that where

M.R.A.D.C. LUMBRE 71
CONSTITUTIONAL LAW REVIEW

as a result of the transaction. As we have said, the transaction involved no less of the inquiry. The Senate has empowered the committee to continue the
than P5,000,000 of public funds. That certainly is a matter of a public concern investigation during the recess. By refusing to answer the questions, the
which it is the duty of the constitutional guardian of the treasury to investigate. witness has obstructed the performance by the Senate of its legislative
function, and the Senate has the power to remove the obstruction by
If the subject of investigation before the committee is within the range of compelling the witness to answer the questions thru restraint of his liberty until
legitimate legislative inquiry and the proposed testimony of the witness called he shall have answered them. That power subsists as long as the Senate, which
relates to that subject, obedience, to its process may be enforced by the is a continuing body, persists in performing the particular legislative function
committee by imprisonment. (Sullivan vs. Hill, 73 W. Va., 49; 79 S.E., 670; involved. To hold that it may punish the witness for contempt only during the
40 Ann. Cas. [1916 B.], 1115.) session in which investigation was begun, would be to recognize the right of
It is next contended for the petitioner that the Senate lacks authority to commit the Senate to perform its function but at the same time to deny to it an
him for contempt for a term beyond its period of legislative session, which essential and appropriate means for its performance. Aside from this, if we
ended on May 18, 1950. should hold that the power to punish for contempt terminates upon the
adjournment of the session, the Senate would have to resume the investigation
Like the Senate of the United States, the Senate of the Philippines is a at the next and succeeding sessions and repeat the contempt proceedings
continuing body whose members are elected for a term of six years and so against the witness until the investigation is completed-an absurd,
divided that the seats of only one-third become vacant every two years, two- unnecessary, and vexatious procedure, which should be avoided.
thirds always continuing into the next Congress save as vacancies may occur
thru death or resignation. Members of the House of Representatives are all As against witness's inconsistent and unjustified claim to a constitutional right,
elected for a term of four years; so that the term of every Congress is four is his clear duty as a citizen to give frank, sincere, and truthful testimony before
years. The Second Congress of the Philippines was constituted on December a competent authority. The state has the right to exact fulfillment of a citizen's
30, 1949, and will expire on December 30, 1953. The resolution of the Senate obligation, consistent of course with his right under the Constitution. The
committing the Petitioner was adopted during the first session of the Second witness in this case has been vociferous and militant in claiming constitutional
Congress, which began on the fourth Monday of January and ended in May 18, rights and privileges but patently recreant to his duties and obligations to the
1950. Government which protects those rights under the law. When a specific right
and a specific obligation conflict with each other, and one is doubtful or
Had said resolution of commitment been adopted by the House of uncertain while the other is clear and imperative, the former must give way to
Representatives, we think it could be enforced until the final adjournment of the latter.
the last session of the Second Congress in 1953. We find no sound reason to
limit the power of the legislative body to punish for contempt to the end of NOTE: The Arnault doctrine has recently been overturned by the case of Balag
every session and not to the end of the last session terminating the existence v. Senate (July 3, 2018, G.R. No. 234608).
of that body. The very reason for the exercise of the power to punish for Going back to the case of Romero v. Estrada, the Court held that the Senate,
contempt is to enable the legislative body to perform its constitutional function as opposed to the House of Representatives, is a continuing body because even
without impediment or obstruction. Legislative functions may be and in during the time of elections there are still 12 senators remaining and will still
practice are performed during recess by duly constituted committees charged occupy their seats. Thus, there is no situation where the Senate will have no
with the duty of performing investigations or conducting hearing relative to Senator sitting. Even if the Senate is a continuing institution, all the legislative
any proposed legislation. To deny to such committees the power of inquiry with measures passed, submitted, or filed before the Senate, after the end of the
process to enforce it would be to defeat the very purpose for which that the Session, shall be deemed to have been determined. Hence, there is need of
power is recognized in the legislative body as an essential and appropriate refiling in the next Session of Congress. For example, if a Senator’s bill was in
auxiliary to is legislative function. It is but logical to say that the power of self- the 2nd reading during the termination of the previous Session, he or she will
preservation is coexistent with the life to be preserved. have to refile said bill, and it will have to undergo 1 st reading – back to square
But the resolution of commitment here in question was adopted by the Senate, one.
which is a continuing body and which does not cease exist upon the periodical However, in the case of Arnault v. Nazareno, the Court held that for the
dissolution of the Congress or of the House of Representatives. There is no purpose of legislative contempt, the Senate is a continuing institution. That is,
limit as to time to the Senate's power to punish for contempt in cases where if before the end of the Session, a person is held in contempt, he will remain
that power may constitutionally be exerted as in the present case. in detention and will not be set free even if the Session has ended. Unless the
Mere reflection upon the situation at hand convinces us of the soundness of Senate has ordered the release, he will remain detained there, based on the
this proposition. The Senate has ordered an investigation of the Buenavista premise that the Senate is a continuing body.
and Tambobong estates deal, which we have found it is within its competence Now comes Balag v. Senate. Here, the Court held that even if Senate is a
to make. That investigation has not been completed because of the refusal of continuing institution, it cannot hold a person cited in contempt indefinitely.
the petitioner as a witness to answer certain questions pertinent to the subject

M.R.A.D.C. LUMBRE 72
CONSTITUTIONAL LAW REVIEW

After the end of the Session, a person must be immediately released. However, There being no provision in the Local Government Code explicitly granting local
a person cited in contempt may still be detained, provided there is still a legislative bodies, the power to issue compulsory process and the power to
legitimate inquiry. punish for contempt, the Sanggunian Panlungsod of Dumaguete is devoid of
power to punish the petitioners Torres and Umbac for contempt. The Ad-Hoc
2. NORECO vs. Sang. Panlunsod of Dumaguete, 155 SCRA 421 (1987) Committee of said legislative body has even less basis to claim that it can
FACTS: Assailed is the validity of a subpoena sent by the respondent exercise these powers.
Committee to the petitioners Paterio Torres and Arturo Umbac, Chairman of Even assuming that the respondent Sangguniang Panlungsod and the
the Board of Directors and the General Manager, respectively, of petitioner respondent Ad-Hoc Committee had the power to issue the subpoena and the
Negros Oriental II Electric Cooperative NORECO II, requiring their attendance order complained of, such issuances would still be void for being ultra vires.
and testimony at the Committee's investigation. Specifically, the inquiry was The contempt power (and the subpoena power) if actually possessed, may only
to focus on the alleged installation and use by the petitioner NORECO II of be exercised where the subject matter of the investigation is within the
inefficient power lines in that city. Similarly under fire is the Order issued by jurisdiction of the legislative body. As admitted by the respondents in their
the same Committee on the latter date directing said petitioners to show cause Comment, the investigation to be conducted by the Ad-Hoc Committee was to
why they should not be punished for legislative contempt due to their failure look into the use by NORECO II of inefficient power lines "of pre-war vintage"
to appear at said investigation. which the latter had acquired from the Visayan Electric Com. company, and "to
RULING: Petition granted. A line should be drawn between the powers of hear the side of the petitioners" (Comment, Rollo, p. 50). It comes evident
Congress as the repository of the legislative power under the Constitution, and that the inquiry would touch upon the efficiency of the electric service of
those that may be exercised by the legislative bodies of local government unit, NORECO II and, necessarily, its compliance with the franchise. Such inquiry is
e.g. the Sangguniang Panlungsod of Dumaguete which, as mere creatures of beyond the jurisdiction of the respondent Sangguniang Panlungsod and the
law, possess delegated legislative power. While the Constitution does not respondent committee.
expressly vest Congress with the power to punish non-members for legislative There is no doubt that a city government has the power to enact ordinances
contempt, the power has nevertheless been invoked by the legislative body as regulating the installation and maintenance of electric power lines or wires
a means of preserving its authority and dignity (Arnault v. Nazareno) in the within its territorial jurisdiction. The power subsists notwithstanding the
same way that courts wield an inherent power to "enforce their authority, creation of the National Electrification Administration (NEA), to which body the
preserve their integrity, maintain their dignity, and ensure the effectiveness of franchise powers of local government units were transferred by Presidential
the administration of justice." Decree No. 269.
The exercise by the legislature of the contempt power is a matter of self- The Sangguniang Panlungsod of Dumaguete may, therefore, enact ordinances
preservation as that branch of the government vested with the legislative to regulate the installation and maintenance of electric power lines, e.g.
power, independently of the judicial branch, asserts its authority and punishes prohibit the use of inefficient power lines, in order to protect the city residents
contempt thereof. The contempt power of the legislature is, therefore, sui from the hazards these may pose. In aid of this ordinance making power, said
generis, and local legislative bodies cannot correctly claim to possess it for the body or any of its committees may conduct investigations similar to, but not
same reasons that the national legislature does. The power attaches not to the the same as, the legislative investigations conducted by the national
discharge of legislative functions per se but to the character of the legislature legislature. As already discussed, the difference lies in the lack of subpoena
as one of the three independent and coordinate branches of government. The power and of the power to punish for contempt on the part of the local
same thing cannot be said of local legislative bodies which are creations of law. legislative bodies. They may only invite resource persons who are willing to
To begin with, there is no express provision either in the 1973 Constitution or supply information which may be relevant to the proposed ordinance. The type
in the Local Government Code (Batas Pambansa Blg. 337) granting local of investigation which may be conducted by the Sangguniang Panlungsod does
legislative bodies, the power to subpoena witnesses and the power to punish not include within its ambit an inquiry into any suspected violation by an
non-members for contempt. Absent a constitutional or legal provision for the electric cooperative of the conditions of its electric franchise.
exercise of these powers, the only possible justification for the issuance of a b. Power of impeachment
subpoena and for the punishment of non-members for contumacious behavior
would be for said power to be deemed implied in the statutory grant of i. Subject officials – Art. XI, Sec. 2
delegated legislative power. But, the contempt power and the subpoena power
partake of a judicial nature. They cannot be implied in the grant of legislative Section 2. The President, the Vice-President, the Members of the Supreme
power. Neither can they exist as mere incidents of the performance of Court, the Members of the Constitutional Commissions, and the Ombudsman
legislative functions. To allow local legislative bodies or administrative agencies may be removed from office on impeachment for, and conviction of, culpable
to exercise these powers without express statutory basis would run afoul of violation of the Constitution, treason, bribery, graft and corruption, other high
the doctrine of separation of powers. crimes, or betrayal of public trust. All other public officers and employees may
be removed from office as provided by law, but not by impeachment.

M.R.A.D.C. LUMBRE 73
CONSTITUTIONAL LAW REVIEW

1. Immunity vis-à-vis impeachability offense that carries the penalty of removal from office, would be
violative of the clear mandate of the fundamental law.
a. President –
NOTE: Petitioners cannot file a case against the President which
David vs. Macapagal-Arroyo, GR No. 171396, 3 May 2016 would result in the removal. Settled is the rule that the President may
FACTS: These seven (7) consolidated petitions for certiorari and NOT be sued in any civil or criminal case. He/she remains
prohibition allege that in issuing Presidential Proclamation No. 1017 accountable, but only in the mode and manner provided by law, and
(PP 1017) and General Order No. 5 President Gloria Macapagal-Arroyo that is by impeachment.
committed grave abuse of discretion. ii. RE: EM No. 03-010 – Order of the First Division of the
RULING: Incidentally, it is not proper to implead President Arroyo as Commission - on Elections Dated August 15, 2003, A.M. No.
respondent. Settled is the doctrine that the President, during his 03-8-22-SC, September 16, 2003
tenure of office or actual incumbency, may not be sued in any civil or FIRST, as already stated in our foregoing 26 August 2003 Resolution,
criminal case, and there is no need to provide for it in the Constitution the Commission has no jurisdiction to hold the Court or any of its
or law. It will degrade the dignity of the high office of the President, Members in contempt for any, decision, order or official action they
the Head of State, if he can be dragged into court litigations while issue.
serving as such. Furthermore, it is important that he be freed from
any form of harassment, hindrance or distraction to enable him to True, the COMELEC -- along with the Commission on Audit, the
fully attend to the performance of his official duties and functions. Commission on Civil Service and the, Ombudsman -- is a
Unlike the legislative and judicial branch, only one constitutes the constitutionally created body with constitutionally mandated
executive branch and anything which impairs his usefulness in the functions. However, as already stated, the actions of all such
discharge of the many great and important duties imposed upon him constitutional, bodies are subject to" certiorari review by the Supreme
by the Constitution necessarily impairs the operation of the Court as was done in G.R. Nos. 147589 and 147613. Thus, the Court
Government. However, this does not mean that the President is not may intervene, strike down or modify COMELEC's actions without
accountable to anyone. Like any other official, he remains accountable itself incurring any liability for contempt; whether its Justices happen
to the people68 but he may be removed from office only in the mode to be impeachable officers or not if the Supreme Court (or its
provided by law and that is by impeachment. Members) can be held liable for contempt for official actions, then it
would cease to be "supreme" in its task of interpreting the law and
b. Other impeachable officers – would become subordinate to whichever agency claims the power to
i. Lecaroz vs. Sandiganbayan, 128 SCRA 324 [1984] (Focus on cite the Court or its Members for contempt.
the question of whether an impeachable officer can be charged In short, the fact that Supreme Court Justices are impeachable
criminally while holding office) officers should not be the ground for the COMELEC's dismissal of the
RULING: On the issue of whether a public officer can be held contempt charges. Rather, they cannot be held liable for contempt,
criminally liable while holding their office, the Court held that the because their herein questioned Decision, Resolution, and Order that
broad power of the New Constitution vests the respondent court with have allegedly interfered with, proceedings of the COMELEC were
jurisdiction over "public officers and employees, including those in made pursuant to their constitutional function. To stop or impede
government-owned or controlled corporations." There are exceptions, COMELEC's proceedings when these have been conducted without or
however, like constitutional officers, particularly those declared to be in excess of jurisdiction or with grave abuse of discretion is not merely
removed by impeachment. Section 2, Article XIII of the 1973 a judicial prerogative; the Constitution mandates such move as a
Constitution provides: judicial duty." The performance of this duty cannot subject the Court
or its Members to contempt of the COMELEC; otherwise, they would
"SEC. 2. The President, the Members of the Supreme Court, and not be able to reverse or modify its abusive actions.
the Members of the Constitutional Commissions shall be removed
from office on impeachment for, and conviction of, culpable The COMELEC's notion that impeachable officers cannot be held in
violation of the Constitution, treason, bribery, other high crimes, contempt is palpably incorrect or at least misleading. Maliciously
or graft and corruption." implied in this notion is that the Supreme Court erred in holding the
Chairman and Members of the COMELEC in contempt via its
Thus, the provision proscribes removal from office of the Resolution dated 18 February 2003 in the same G.R. Nos. 147589 and
aforementioned constitutional officers by any other method; 147613.
otherwise, to allow a public officer who may be removed solely by
impeachment to be charged criminally while holding his office with an

M.R.A.D.C. LUMBRE 74
CONSTITUTIONAL LAW REVIEW

As mentioned earlier, this Court has undisputed certiorari powers over RULING: Yes. Under Section 5, Article VIII of the Constitution, the
the actions of the Commission on Elections. As an incident of such Supreme Court shall have the power to exercise original jurisdiction
prerogative, the Court has the inherent authority to enforce its orders over cases affecting ambassadors, other public ministers and consuls,
and to hold the COMELEC's Chairman and Commissioners in contempt and over petitions for certiorari, prohibition, mandamus, quo
when they impede, obstruct, or degrade its proceedings or orders, or warranto, and habeas corpus. Section 5 of Article VIII does not limit
disobey, ignore or otherwise offend its dignity. the Court's quo warranto jurisdiction only to certain public officials or
that excludes impeachable officials therefrom.
Clearly, the COMELEC has no reciprocal constitutional power to pass
upon the actions of this Court or its Members Hence, the Commission This is not the first time the Court took cognizance of a quo warranto
has absolutely no authority to hold them in contempt as an incident petition against an impeachable officer. In the consolidated cases of
of its inexistent power of review. Even more clearly, it has no right to Estrada v. Macapagal-Arroyo and Estrada v. Desierto, the Court
recriminate or sulk when its imprudent actions are reversed, or its assumed jurisdiction over a quo warranto petition that challenged
Members held in contempt for their rash actions. Gloria Macapagal-Arroyo's title to the presidency.
While the COMELEC is given specific powers and functions by the Accordingly, the Court could, as it did in Estrada, assume jurisdiction
Constitution, the Commission does not have the same level and over the instant quo warranto petition against an impeachable officer.
standing as the three great branches of government.
Quo warranto and impeachment are two distinct proceedings,
FIFTH, the COMELEC'S First Division ruled that in the same cases although both may result in the ouster of a public officer. Strictly
(G.R. Nos. 147589 and 1 4761 3), the Court had allegedly degraded speaking, quo warranto grants the relief of "ouster", while
the Commission by making the latter a mere recommendatory body" impeachment affords "removal."
and thus deprived it of its constitutional powers to enforce election
laws. Again, this is pure legal heresy. A quo warranto proceeding is the proper legal remedy to determine
a person's right or title to a public office and to oust the holder from
In our 26 June 2001 Decision in those cases, a fact-finding task was its enjoyment. It is the proper action to inquire into a public officer's
delegated to the COMELEC: to determine which of the party-list eligibility or the validity of his appointment. Under Rule 66 of the
candidates had complied with the eight-point guideline we had issued. Rules of Court, a quo warranto proceeding involves a judicial
This task had to be delegated because the Court is not a trier of facts, determination of the right to the use or exercise of the office.
and' the Corn mission is' precisely the constitutional agency that is
supposedly knowledgeable of election matters and the principal trier Impeachment, on the other hand, is a political process undertaken
thereof. by the legislature to determine whether the public officer committed
any of the impeachable offenses, namely, culpable violation of the
Clearly delineated in our Decision was the specific work remanded to Constitution, treason, bribery, graft and corruption, other high
the COMELEC fact-findings It did not involve, much less impair, the crimes, or betrayal of public trust. It does not ascertain the officer's
normal powers and duties of the poll body. To stress, its task of fact- eligibility for appointment or election, or challenge the legality of his
finding was specific and limited, one that accrued only as a direct assumption of office. Conviction for any of the impeachable offenses
result of the disposition" in the said cases. In other words, its shall result in the removal of the impeachable official from office.
authority in this specific instance was coextensive only with that
which, was delegated to it to implement the Decision. The OSG's quo warranto petition challenged respondent's right and
title to the position of Chief Justice.
iii. Republic v. Sereno, G.R. No. 237428, June 19, 2018
Clearly, the OSG questioned the respondent's eligibility for
FACTS: Respondent filed a Motion for Reconsideration of the appointment as Chief Justice and sought to invalidate such
Supreme Court’s decision dated May 11, 2018 which found appointment. The OSG's petition, therefore, is one for quo warranto
respondent guilty of unlawfully holding and exercising the Office if the over which the Court exercises original jurisdiction.
Chief Justice.
As the Court previously held, "where the dispute is on the eligibility
Respondent argued that the Court is without jurisdiction to oust an to perform the duties by the person sought to be ousted or disqualified
impeachable officer through quo warranto; that the official acts of the a quo warranto is the proper action."
Judicial and Bar Council (JBC) and the President involves political
questions that cannot be annulled absent any allegation of grave ii. Limitation – Art. XI, Sec. 3(5)
abuse of discretion; that the petition for quo warranto is time-barred; Section 3. xxxx No impeachment proceedings shall be initiated against the
and that respondent was and is a person of proven integrity. same official more than once within a period of one year.

M.R.A.D.C. LUMBRE 75
CONSTITUTIONAL LAW REVIEW

iii. Procedure (1) Approval by the President;


1. Initiation stage (2) Inaction of the President within 30 days;

a. If initiated by less than 1/3 (Art. XI, Sec. 3[2] and [3]) (3) Legislative override by 2/3 of Congress (with Yeas and Nays entered into
the Journal of each House).
Section 3. xxx A verified complaint for impeachment may be filed by
any Member of the House of Representatives or by any citizen upon ii. Presidential veto and congressional override – Art. VI, Sec. 27
a resolution or endorsement by any Member thereof, which shall be
included in the Order of Business within ten session days, and referred Section 27. Every bill passed by the Congress shall, before it becomes a law,
to the proper Committee within three session days thereafter. The be presented to the President. If he approves the same he shall sign it;
Committee, after hearing, and by a majority vote of all its Members, otherwise, he shall veto it and return the same with his objections to the House
shall submit its report to the House within sixty session days from where it originated, which shall enter the objections at large in its Journal and
such referral, together with the corresponding resolution. The proceed to reconsider it. If, after such reconsideration, two-thirds of all the
resolution shall be calendared for consideration by the House within Members of such House shall agree to pass the bill, it shall be sent, together
ten session days from receipt thereof. with the objections, to the other House by which it shall likewise be
reconsidered, and if approved by two-thirds of all the Members of that House,
A vote of at least one-third of all the Members of the House shall be it shall become a law. In all such cases, the votes of each House shall be
necessary either to affirm a favorable resolution with the Articles of determined by yeas or nays, and the names of the Members voting for or
Impeachment of the Committee, or override its contrary resolution. against shall be entered in its Journal. The President shall communicate his
The vote of each Member shall be recorded. veto of any bill to the House where it originated within thirty days after the
date of receipt thereof, otherwise, it shall become a law as if he had signed it.
b. If initiated by 1/3 (Art. XI, Sec. 3[4])
The President shall have the power to veto any particular item or items in an
Section 3. In case the verified complaint or resolution of appropriation, revenue, or tariff bill, but the veto shall not affect the item or
impeachment is filed by at least one-third of all the Members of the items to which he does not object.
House, the same shall constitute the Articles of Impeachment, and
trial by the Senate shall forthwith proceed. 1. Message veto vs. Pocket veto
2. Trial stage (Art. XI, Sec. 3[6]) Message Veto: the power or right vested in one branch of the
government to cancel or postpone the decisions, enactments, etc. of
The Senate shall have the sole power to try and decide all cases of another branch, especially the right of the President or Chief Executive to
impeachment. When sitting for that purpose, the Senators shall be on oath reject bills passed by the legislature. Also called a “veto message” a
or affirmation. When the President of the Philippines is on trial, the Chief document exercising such right and setting forth the reasons for such
Justice of the Supreme Court shall preside, but shall not vote. No person action.
shall be convicted without the concurrence of two-thirds of all the
Members of the Senate. Pocket Veto: An indirect veto of a legislative bill by the executive through
retention of the bill unsigned until after adjournment of the legislature.
iv. Consequences of conviction – Art. XI, Sec. 3(7) Such veto is not applicable in our jurisdiction because inaction by the
Section 3. xxx Judgment in cases of impeachment shall not extend further President never produces a veto even if Congress is in recess. The
than removal from office and disqualification to hold any office under the President must still act to veto the bill and communicate the same to
Republic of the Philippines, but the party convicted shall nevertheless be liable Congress by returning the vetoed bill with his veto message.
and subject to prosecution, trial, and punishment, according to law. 2. General veto power vs. Item or line-veto power
c. Limitations on legislative power General Veto Power is the power of the President to reject a bill
i. Bills that shall originate exclusively in the House of Representatives proposed by legislature by refusing to sign it into law. The President
– Art. VI, Sec. 24 actually writes the word “veto” (Latin for “I forbid”) on the bill and sends
it back to the legislature with a statement of his or her objections.
Section 24. All appropriation, revenue or tariff bills, bills authorizing increase
of the public debt, bills of local application, and private bills, shall originate Item or Line-Veto Power, sometimes called the “partial veto,” is a type
exclusively in the House of Representatives, but the Senate may propose or of veto that would give the President the power to cancel an individual
concur with amendments. provision – line-items – in spending, or “appropriations” bills, without

NOTE: Three (3) ways by which a bill becomes a law:

M.R.A.D.C. LUMBRE 76
CONSTITUTIONAL LAW REVIEW

vetoing the entire bill. This is allowed ONLY in Appropriations, Revenue, . . . Legislative control cannot be exercised in such a manner as to
and Tariff (ART) bills. encumber the general appropriation bill with veto-proof "logrolling
measures", special interest provisions which could not succeed if
a. Doctrine of Inappropriate Provision separately enacted, or "riders", substantive pieces of legislation
As the Constitution is explicit that the provision which Congress can incorporated in a bill to insure passage without veto . . .
include in an appropriations bill must "relate specifically to some
particular appropriation therein" and "be limited in its operation to the
appropriation to which it relates," it follows that any provision which
does not relate to any particular item, or which extends in its
operation beyond an item of appropriation, is considered "an
inappropriate provision" which can be vetoed separately from an item.
Also to be included in the category of "inappropriate provisions" are
unconstitutional provisions and provisions which are intended to
amend other laws, because clearly these kind of laws have no place
in an appropriations bill. These are matters of general legislation more
appropriately dealt with in separate enactments. Former Justice Irene
Cortes, as Amicus Curiae, commented that Congress cannot by law
establish conditions for and regulate the exercise of powers of the
President given by the Constitution for that would be an
unconstitutional intrusion into executive prerogative.

The doctrine of "inappropriate provision" was well elucidated in Henry


v. Edwards, supra, thus:

Just as the President may not use his item-veto to usurp constitutional
powers conferred on the legislature, neither can the legislature
deprive the Governor of the constitutional powers conferred on him
as chief executive officer of the state by including in a general
appropriation bill matters more properly enacted in separate
legislation. The Governor's constitutional power to veto bills of general
legislation . . . cannot be abridged by the careful placement of such
measures in a general appropriation bill, thereby forcing the Governor
to choose between approving unacceptable substantive legislation or
vetoing "items" of expenditures essential to the operation of
government. The legislature cannot by location of a bill give it
immunity from executive veto. Nor can it circumvent the Governor's
veto power over substantive legislation by artfully drafting general
law measures so that they appear to be true conditions or limitations
on an item of appropriation. Otherwise, the legislature would be
permitted to impair the constitutional responsibilities and functions of
a co-equal branch of government in contravention of the separation
of powers doctrine . . . We are no more willing to allow the legislature
to use its appropriation power to infringe on the Governor's
constitutional right to veto matters of substantive legislation than we
are to allow the Governor to encroach on the Constitutional powers of
the legislature. In order to avoid this result, we hold that, when the
legislature inserts inappropriate provisions in a general appropriation
bill, such provisions must be treated as "items" for purposes of the
Governor's item veto power over general appropriation bills.
xxx xxx xxx

M.R.A.D.C. LUMBRE 77
CONSTITUTIONAL LAW REVIEW

VI. Executive Department The Congress shall, by law, provide who shall serve as President in case of
death, permanent disability, or resignation of the Acting President. He shall
serve until the President or the Vice-President shall have been elected and
1. The President qualified, and be subject to the same restrictions of powers and
disqualifications as the Acting President.
a. Qualifications of President – Art. VII, Sec. 2
1. Estrada vs. Disierto, GR Nos. 146710-15, 2 March 2001
Section 2. No person may be elected President unless he is a natural-born citizen
of the Philippines, a registered voter, able to read and write, at least forty years of RULING: As to the first issue on whether Estrada resigned, the Court held
age on the day of the election, and a resident of the Philippines for at least ten that, to be considered resign there must be an intent to resign and the
years immediately preceding such election. intent must be coupled by acts of relinquishment. The validity of a
resignation is not governed by any formal requirement as to form. It can
b. Presidential succession be oral. It can be written. It can be express. It can be implied. As long as
the resignation is clear, it must be given legal effect. Using the totality
i. If there is vacancy BEFORE the beginning of the term – Art. VII, Sec.
test, by the totality of prior, contemporaneous and posterior facts and
7 circumstantial evidence bearing a material relevance on the issue, the
Section 7. The President-elect and the Vice President-elect shall assume office court held that President resigned.
at the beginning of their terms. In sum, it was held that the resignation of the petitioner cannot be
If the President-elect fails to qualify, the Vice President-elect shall act as doubted. It was confirmed by his leaving Malacaang. In the press release
President until the President-elect shall have qualified. containing his final statement, (1) he acknowledged the oath-taking of the
respondent as President of the Republic albeit with the reservation about
If a President shall not have been chosen, the Vice President-elect shall act as its legality; (2) he emphasized he was leaving the Palace, the seat of the
President until a President shall have been chosen and qualified. presidency, for the sake of peace and in order to begin the healing process
of our nation. He did not say he was leaving the Palace due to any kind of
If at the beginning of the term of the President, the President-elect shall have
inability and that he was going to re-assume the presidency as soon as
died or shall have become permanently disabled, the Vice President-elect shall
the disability disappears; (3) he expressed his gratitude to the people for
become President.
the opportunity to serve them. Without doubt, he was referring to the past
Where no President and Vice-President shall have been chosen or shall have opportunity given him to serve the people as President; (4) he assured
qualified, or where both shall have died or become permanently disabled, the that he will not shirk from any future challenge that may come ahead in
President of the Senate or, in case of his inability, the Speaker of the House of the same service of our country. Petitioners reference is to a future
Representatives, shall act as President until a President or a Vice-President challenge after occupying the office of the president which he has given
shall have been chosen and qualified. up; and (5) he called on his supporters to join him in the promotion of a
constructive national spirit of reconciliation and solidarity. Certainly, the
The Congress shall, by law, provide for the manner in which one who is to act national spirit of reconciliation and solidarity could not be attained if he
as President shall be selected until a President or a Vice-President shall have did not give up the presidency. The press release was his valedictory, his
qualified, in case of death, permanent disability, or inability of the officials final act of farewell. His presidency is now in the past tense.
mentioned in the next preceding paragraph.
On the second issue of whether Estrada is only temporarily unable to act
NOTE: Failure to qualify includes: failure to elect; no elections; or non- as President:
completion of canvass by the noon of 30th of June. Failure to choose includes:
failure of the NBOC to proclaim a winner before the noon of 30th of June. Upon receipt of the letter, the HoR and Senate issued a resolution
expressing support for the presidency of Glorai Macapagal Arroyo although
ii. If there is PERMANENT vacancy DURING the incumbency – Art. VII, the constitution provides that the congress shall convene to decide on such
Sec. 8 inability both houses of Congress have recognized respondent Arroyo as
Section 8. In case of death, permanent disability, removal from office, or the President. Implicitly clear in that recognition is the premise that the
resignation of the President, the Vice-President shall become the President to inability of petitioner Estrada is no longer temporary. Congress has clearly
serve the unexpired term. In case of death, permanent disability, removal from rejected petitioner’s claim of inability. In such case, the question is political
office, or resignation of both the President and Vice-President, the President of in nature and addressed solely to Congress by constitutional fiat even if
the Senate or, in case of his inability, the Speaker of the House of the petitioner can prove that he did not resign, still, he cannot successfully
Representatives, shall then act as President until the President or Vice- claim that he is a President on leave on the ground that he is merely
President shall have been elected and qualified. unable to govern temporarily. That claim has been laid to rest by Congress

M.R.A.D.C. LUMBRE 78
CONSTITUTIONAL LAW REVIEW

and the decision that respondent Arroyo is the de jure President made by controlled bank or financial institution to the President, the Vice-President, the Members
a co-equal branch of government cannot be reviewed by this Court. of the Cabinet, the Congress, the Supreme Court, and the Constitutional Commissions,
the Ombudsman, or to any firm or entity in which they have controlling interest, during
iii. If there is TEMPORARY vacancy DURING the incumbency – Art. VI, their tenure. (Article XI)
Sec. 11
a. Rule on additional positions:
Section 11. Whenever the President transmits to the President of the Senate
and the Speaker of the House of Representatives his written declaration that (1) Elective – Art. IX-B, Sec. 7, 1st par.
he is unable to discharge the powers and duties of his office, and until he
transmits to them a written declaration to the contrary, such powers and duties Section 7. No elective official shall be eligible for appointment or designation in
shall be discharged by the Vice-President as Acting President. any capacity to any public office or position during his tenure.

Whenever a majority of all the Members of the Cabinet transmit to the (2) Appointive - Art. IX-B, Sec. 7, 2nd par.
President of the Senate and to the Speaker of the House of Representatives Section 7. xxxx Unless otherwise allowed by law or by the primary functions of his
their written declaration that the President is unable to discharge the powers position, no appointive official shall hold any other office or employment in the
and duties of his office, the Vice-President shall immediately assume the Government or any subdivision, agency or instrumentality thereof, including
powers and duties of the office as Acting President. Government-owned or controlled corporations or their subsidiaries.
Thereafter, when the President transmits to the President of the Senate and to (3) Elective (President and VP) and appointive (Cabinet Secretaries,
the Speaker of the House of Representatives his written declaration that no Usecs. and Asecs.) - Art. VII, Sec. 13
inability exists, he shall reassume the powers and duties of his office.
Meanwhile, should a majority of all the Members of the Cabinet transmit within Section 13. The President, Vice-President, the Members of the Cabinet, and their
five days to the President of the Senate and to the Speaker of the House of deputies or assistants shall not, unless otherwise provided in this Constitution, hold
Representatives, their written declaration that the President is unable to any other office or employment during their tenure. They shall not, during said
discharge the powers and duties of his office, the Congress shall decide the tenure, directly or indirectly, practice any other profession, participate in any
issue. For that purpose, the Congress shall convene, if it is not in session, business, or be financially interested in any contract with, or in any franchise, or
within forty-eight hours, in accordance with its rules and without need of call. special privilege granted by the Government or any subdivision, agency, or
instrumentality thereof, including government-owned or controlled corporations or
If the Congress, within ten days after receipt of the last written declaration, their subsidiaries. They shall strictly avoid conflict of interest in the conduct of their
or, if not in session, within twelve days after it is required to assemble, office.
determines by a two-thirds vote of both Houses, voting separately, that the
President is unable to discharge the powers and duties of his office, the Vice- The spouse and relatives by consanguinity or affinity within the fourth civil degree
President shall act as President; otherwise, the President shall continue of the President shall not, during his tenure, be appointed as Members of the
exercising the powers and duties of his office. Constitutional Commissions, or the Office of the Ombudsman, or as Secretaries,
Undersecretaries, chairmen or heads of bureaus or offices, including government-
2. Prohibitions - Art. VII, Sec. 13; relate with Art. XI, Sec. 16 owned or controlled corporations and their subsidiaries.
Section 13. The President, Vice-President, the Members of the Cabinet, and their i. Art. VII, Sec. 3
deputies or assistants shall not, unless otherwise provided in this Constitution, hold any
other office or employment during their tenure. They shall not, during said tenure, Section 3. There shall be a Vice-President who shall have the same
directly or indirectly, practice any other profession, participate in any business, or be qualifications and term of office and be elected with, and in the same manner,
financially interested in any contract with, or in any franchise, or special privilege as the President. He may be removed from office in the same manner as the
granted by the Government or any subdivision, agency, or instrumentality thereof, President.
including government-owned or controlled corporations or their subsidiaries. They shall
The Vice-President may be appointed as a Member of the Cabinet. Such
strictly avoid conflict of interest in the conduct of their office.
appointment requires no confirmation.
The spouse and relatives by consanguinity or affinity within the fourth civil degree of
ii. Art. VIII, Sec. 8 (1)
the President shall not, during his tenure, be appointed as Members of the Constitutional
Commissions, or the Office of the Ombudsman, or as Secretaries, Undersecretaries, Section 8. A Judicial and Bar Council is hereby created under the supervision
chairmen or heads of bureaus or offices, including government-owned or controlled of the Supreme Court composed of the Chief Justice as ex officio Chairman,
corporations and their subsidiaries. (Article VII) the Secretary of Justice, and a representative of the Congress as ex officio
Members, a representative of the Integrated Bar, a professor of law, a retired
Section 16. No loan, guaranty, or other form of financial accommodation for any
Member of the Supreme Court, and a representative of the private sector.
business purpose may be granted, directly or indirectly, by any government-owned or

M.R.A.D.C. LUMBRE 79
CONSTITUTIONAL LAW REVIEW

iii. Art. XII, Sec. 9 The Court had occasion to explain the meaning of an ex-officio position in
Rafael vs. Embroidery and Apparel Control and Inspection Board, thus: "An
Section 9. The Congress may establish an independent economic and planning examination of section 2 of the questioned statute (R.A. 3137) reveals that for
agency headed by the President, which shall, after consultations with the the chairman and members of the Board to qualify they need only be
appropriate public agencies, various private sectors, and local government designated by the respective department heads. With the exception of the
units, recommend to Congress, and implement continuing integrated and representative from the private sector, they sit ex-officio. In order to be
coordinated programs and policies for national development. designated they must already be holding positions in the offices mentioned in
Until the Congress provides otherwise, the National Economic and the law.
Development Authority shall function as the independent planning agency of The term "primary" used to describe "functions" refers to the order of
the government. importance and thus means chief or principal function. The term is not
iv. Doctrine of Qualified Political Agency restricted to the singular but may refer to the plural. The additional duties must
not only be closely related to, but must be required by the official's primary
Under this doctrine, which recognizes the establishment of a single executive, functions. If the functions required to be performed are merely incidental,
"all executive and administrative organizations are adjuncts of the Executive remotely related, inconsistent, incompatible, or otherwise alien to the primary
Department, the heads of the various executive departments are assistants function of a cabinet official, such additional functions would fall under the
and agents of the Chief Executive, and, except in cases where the Chief purview of "any other office" prohibited by the Constitution.
Executive is required by the Constitution or law to act in person on the
exigencies of the situation demand that he act personally, the multifarious It bears repeating though that in order that such additional duties or functions
executive and administrative functions of the Chief Executive are performed may not transgress the prohibition embodied in Section 13, Article VII of the
by and through the executive departments, and the acts of the Secretaries of 1987 Constitution, such additional duties or functions must be required by the
such departments, performed and promulgated in the regular course of primary functions of the official concerned, who is to perform the same in an
business, are, unless disapproved or reprobated by the Chief Executive ex-officio capacity as provided by law, without receiving any additional
presumptively the acts of the Chief Executive." (Ocampo et al v. Rear compensation therefor.
Admiral Enriquez) The ex-officio position being actually and in legal contemplation part of the
v. Civil Liberties Union vs. Executive Secretary, 194 SCRA 317 (1991) principal office, it follows that the official concerned has no right to receive
additional compensation for his services in the said position. The reason is that
RULING: In deciding the issue whether the prohibition in Section 13, Article these services are already paid for and covered by the compensation attached
VII of the 1987 Constitution insofar as Cabinet members, their deputies or to his principal office. It should be obvious that if, say, the Secretary of Finance
assistants are concerned, admit of the broad exceptions made for appointive attends a meeting of the Monetary Board as an ex-officio member thereof, he
officials in general under Section 7, par. (2), Article I-XB which, for easy is actually and in legal contemplation performing the primary function of his
reference is quoted anew, thus: "Unless otherwise allowed by law or by the principal office in defining policy in monetary and banking matters, which come
primary functions of his position, no appointive official shall hold any other under the jurisdiction of his department. For such attendance, therefore, he is
office or employment in the Government or any subdivision, agency or not entitled to collect any extra compensation, whether it be in the form of a
instrumentality thereof, including government-owned or controlled corporation per diem or an honorarium or an allowance, or some other such euphemism.
or their subsidiaries,” the Court held: By whatever name it is designated, such additional compensation is prohibited
by the Constitution.
To reiterate, the prohibition under Section 13, Article VII is not to be
interpreted as covering positions held without additional compensation in ex- As earlier clarified in this decision, ex-officio posts held by the executive official
officio capacities as provided by law and as required by the primary functions concerned without additional compensation as provided by law and as required
of the concerned official's office. The term ex-officio means "from office; by by the primary functions of his office do not fall under the definition of "any
virtue of office." It refers to an "authority derived from official character other office" within the contemplation of the constitutional prohibition.
merely, not expressly conferred upon the individual character, but rather
annexed to the official position." Ex-officio likewise denotes an "act done in an It being clear that the 1987 Constitution seeks to prohibit the President, Vice-
official character, or as a consequence of office, and without any other President, members of the Cabinet, their deputies or assistants from holding
appointment or authority than that conferred by the office." An ex-officio during their tenure multiple offices or employment in the government, except
member of a board is one who is a member by virtue of his title to a certain in those cases specified in the Constitution itself and as above clarified with
office, and without further warrant or appointment. To illustrate, by express respect to posts held without additional compensation in an ex-officio capacity
provision of law, the Secretary of Transportation and Communications is the as provided by law and as required by the primary functions of their office, the
ex-officio Chairman of the Board of the Philippine Ports Authority, and the Light citation of Cabinet members (then called Ministers) as examples during the
Rail Transit Authority. debate and deliberation on the general rule laid down for all appointive officials

M.R.A.D.C. LUMBRE 80
CONSTITUTIONAL LAW REVIEW

should be considered as mere personal opinions which cannot override the 'The solicitor General submits that 'the sweeping limitations or conditions
constitution's manifest intent and the people' understanding thereof. attached to the 'Cabinet Prohibition' do not obtain in the class of other
appointive officials, who by the Decision are members of the civil service in
In the light of the construction given to Section 13, Article VII in relation to general. He is correct. Other appointive officials below the rank of assistant
Section 7, par. (2), Article IX-B of the 1987 Constitution, Executive Order No. secretary are not covered by the constitutional prohibition under consideration.
284 dated July 23, 1987 is unconstitutional. Ostensibly restricting the number Since E.O. 824 includes in its coverage 'other appointive officials' aside from
of positions that Cabinet members, undersecretaries or assistant secretaries members of the Cabinet, their undersecretaries and assistant secretaries, it
may hold in addition to their primary position to not more than two (2) should be upheld insofar as these 'other appointive officials; are concerned.
positions in the government and government corporations, Executive Order
No. 284 actually allows them to hold multiple offices or employment in direct "Thus, it is clear that E.O. 284 is valid insofar as it authorizes 'other appointive
contravention of the express mandate of Section 13, Article VII of the 1987 officials', other than the members of the Cabinet, undersecretaries and
Constitution prohibiting them from doing so, unless provided in the 1987 assistant secretaries, to hold multiple positions in government, but not to
Constitution itself. exceed two (2) positions.
NOTE: There are Cabinet secretaries that are called in-rank secretaries or [G.R. No. 138489. November 29, 2001]
undersecretaries (because they are the same in rank as with the former).
These in-rank secretaries can be appointed to other positions as they are not The Decision of the Supreme Court in Civil Liberties Union and Anti-Graft
one of those regular secretaries. League of the Philippines, Inc. was clarified in the Resolution of the Court En
Banc on August 1, 1991, in that the constitutional ban against dual or multiple
vi. Clarificatory en banc Resolution in GR No. 83896 (Civil Liberties positions applies only to the members of the Cabinet, their deputies or
Union vs. Executive Secretary) and GR No. 83815 (Anti-Graft League assistants. It does not cover other appointive officials with equivalent rank or
of the Philippines, Inc., et al. vs. Juico, as Secretary of Agrarian Reform those lower than the position of Assistant Secretary.
et al.) dated August 1, 1991
[G.R. No. 138965. June 30, 2006]
RULING: "In the case of Civil Liberties Union vs. Executive Secretary, GR No.
83896, promulgated on February 22, 1991, the Supreme Court declared E.O. Citing the Resolution in Civil Liberties Union v. Executive Secretary,
No. 284 null and void, being contrary to the express provisions of the respondents allege that the strict prohibition against holding multiple positions
fundamental law, specifically Section 13, Article VII in relation to Section 7, provided under Section 13, Article VII of the 1987 Constitution applies only to
par. (2), Article IX-B. Section 13, Article VII, reads as follows: heads of executive departments, their undersecretaries and assistant
secretaries; it does not cover other public officials given the rank of Secretary,
"The Supreme Court ruled that while E.O. No. 284 ostensibly prohibits a Undersecretary, or Assistant Secretary.
member of the Cabinet, undersecretary, assistant secretary or other appointive
officials from holding more than two (2) positions, it, in effect, allows these [G.R. No. 191644. February 19, 2013]
officials to hold more that one (1) but not to exceed two (2) positions. This is According to Public Interest Center, Inc. v. Elma, the only two exceptions
in direct contravention of Section 13, Article VII of the 1987 Constitution which against the holding of multiple offices are: (1) those provided for under the
is categorical in its prohibition. Although said Section authorizes certain Constitution, such as Section 3, Article VII, authorizing the Vice President to
exceptions, these exceptions refer only to positions held in an ex officio become a member of the Cabinet; and (2) posts occupied by Executive officials
capacity or as required by the primary functions of the first office. specified in Section 13, Article VII without additional compensation in ex officio
"The ruling, however, of the Supreme Court in the abovementioned case was capacities as provided by law and as required by the primary functions of the
clarified by the High Court itself in its En Banc Resolution dated August 1, 1991. officials’ offices. In this regard, the decision in Public Interest Center, Inc. v.
Pertinent portions of said Resolution read as follows: Elma adverted to the resolution issued on August 1, 1991 in Civil Liberties
Union v. The Executive Secretary, whereby the Court held that the phrase "the
'Another point of clarification raised by Solicitor General refers to the persons Members of the Cabinet, and their deputies or assistants" found in Section 13,
affected by the constitutional prohibition. The persons cited in the supra, referred only to the heads of the various executive departments, their
constitutional provision are the 'Members of the Cabinet, their deputies or undersecretaries and assistant secretaries, and did not extend to other public
assistants.' These terms must be given their common and general acceptation officials given the rank of Secretary, Undersecretary or Assistant Secretary.
as referring to the heads of the executive departments, their undersecretaries
and assistant secretaries. Public officials given the rank equivalent to a The clarification was the Court’s action on the motion for clarification filed in
Secretary, Undersecretary or Assistant Secretary are not covered by the Civil Liberties Union v. The Executive Secretary, and revises the main opinion
prohibition, not is the Solicitor General affected thereby. The prohibition, promulgated on February 22, 1991 (194 SCRA 317)totally invalidating
however, applied to chairmanship and membership in the boards of Executive Order No. 284 dated July 25, 1987 (whose questioned Section 1
sequestered corporations. states: "Even if allowed by law or by the ordinary functions of his position, a

M.R.A.D.C. LUMBRE 81
CONSTITUTIONAL LAW REVIEW

member of the Cabinet, undersecretary or assistant secretary or other authorized by the Constitution itself. In other words, Section 7, Article IX-B is
appointive officials of the Executive Department may, in addition to his primary meant to lay down the general rule applicable to all elective and appointive
position, hold not more than two positions in the government and government public officials and employees, while Section 13, Article VII is meant to be the
corporations and receive the corresponding compensation therefor; Provided, exception applicable only to the President, the Vice-President, Members of the
that this limitation shall not apply to ad hoc bodies or committees, or to boards, Cabinet, their deputies and assistants.
councils or bodies of which the President is the Chairman."). The clarifying
dictum now considered Executive Order No. 284 partly valid to the extent that The general rule contained in Article IX-B of the 1987 Constitution permits an
it included in its coverage "other appointive officials" aside from the members appointive official to hold more than one office only if "allowed by law or by
of the Cabinet, their undersecretaries and assistant secretaries, with the the primary functions of his position." In the case of Quimson v. Ozaeta, this
dispositive part of the clarificatory resolution of August 1, 1991 stating: Court ruled that, "[t]here is no legal objection to a government official
"WHEREFORE, subject to the qualification above-stated, the petitions are occupying two government offices and performing the functions of both as long
GRANTED. Executive Order No. 284 is hereby declared null and void insofar as as there is no incompatibility." The crucial test in determining whether
it allows a member of the Cabinet, undersecretary or assistant secretary to incompatibility exists between two offices was laid out in People v. Green -
hold other positions in the government and government-owned and controlled whether one office is subordinate to the other, in the sense that one office has
corporations." the right to interfere with the other.

vii. Public Interest Center Inc. vs. Magdangal B. Elma, CPLC, GR. No. [I]ncompatibility between two offices, is an inconsistency in the functions of
138965, 30 June 2006 the two; x x x Where one office is not subordinate to the other, nor the relations
of the one to the other such as are inconsistent and repugnant, there is not
FACTS: Petitioners cited the case of Civil Liberties Union v. Executive Secretary that incompatibility from which the law declares that the acceptance of the one
to support their position that respondent Elma’s concurrent appointments as is the vacation of the other. The force of the word, in its application to this
PCGG Chairman and CPLC contravenes Section 13, Article VII and Section 7, matter is, that from the nature and relations to each other, of the two places,
par. 2, Article IX-B of the 1987 Constitution. Petitioners also maintained that they ought not to be held by the same person, from the contrariety and
respondent Elma was holding incompatible offices. antagonism which would result in the attempt by one person to faithfully and
impartially discharge the duties of one, toward the incumbent of the other. x
RULING: The issue in this case is whether the position of the PCGG Chairman x x The offices must subordinate, one [over] the other, and they must, per se,
or that of the CPLC falls under the prohibition against multiple offices imposed have the right to interfere, one with the other, before they are incompatible at
by Section 13, Article VII and Section 7, par. 2, Article IX-B of the 1987 common law. x x x
Constitution, which provide that:
In this case, an incompatibility exists between the positions of the PCGG
Art. VII. x x x x Chairman and the CPLC. The duties of the CPLC include giving independent
Section 13. The President, Vice-President, the Members of the Cabinet, and and impartial legal advice on the actions of the heads of various executive
their deputies or assistants shall not, unless otherwise provided in this departments and agencies and to review investigations involving heads of
Constitution, hold any other office or employment during their tenure. x x x executive departments and agencies, as well as other Presidential appointees.
The PCGG is, without question, an agency under the Executive Department.
Art. IX-B. x x x x Thus, the actions of the PCGG Chairman are subject to the review of the CPLC.
Section 7. No elective official shall be eligible for appointment or designation While Section 7, Article IX-B of the 1987 Constitution applies in general to all
in any capacity to any public office or position during his tenure. elective and appointive officials, Section 13, Article VII, thereof applies in
particular to Cabinet secretaries, undersecretaries and assistant secretaries.
Unless otherwise allowed by law or by the primary functions of his position, no
In the Resolution in Civil Liberties Union v. Executive Secretary, this Court
appointive official shall hold any other office or employment in the Government
already clarified the scope of the prohibition provided in Section 13, Article VII
or any subdivision, agency or instrumentality thereof, including government-
of the 1987 Constitution. Citing the case of US v. Mouat, it specifically identified
owned or controlled corporations or their subsidiaries.
the persons who are affected by this prohibition as secretaries,
To harmonize these two provisions, this Court, in the case of Civil Liberties undersecretaries and assistant secretaries; and categorically excluded public
Union v. Executive Secretary, construed the prohibition against multiple offices officers who merely have the rank of secretary, undersecretary or assistant
contained in Section 7, Article IX-B and Section 13, Article VII in this manner: secretary.

[T]hus, while all other appointive officials in the civil service are allowed to hold Another point of clarification raised by the Solicitor General refers to the
other office or employment in the government during their tenure when such persons affected by the constitutional prohibition. The persons cited in the
is allowed by law or by the primary functions of their positions, members of constitutional provision are the "Members of the Cabinet, their deputies and
the Cabinet, their deputies and assistants may do so only when expressly assistants." These terms must be given their common and general acceptation

M.R.A.D.C. LUMBRE 82
CONSTITUTIONAL LAW REVIEW

as referring to the heads of the executive departments, their undersecretaries In sum, the prohibition in Section 13, Article VII of the 1987 Constitution does
and assistant secretaries. Public officials given the rank equivalent to a not apply to respondent Elma since neither the PCGG Chairman nor the CPLC
Secretary, Undersecretary, or Assistant Secretary are not covered by the is a Cabinet secretary, undersecretary, or assistant secretary. Even if this Court
prohibition, nor is the Solicitor General affected thereby. assumes, arguendo, that Section 13, Article VII is applicable to respondent
Elma, he still could not be appointed concurrently to the offices of the PCGG
It is clear from the foregoing that the strict prohibition under Section 13, Article Chairman and CPLC because neither office was occupied by him in an ex-officio
VII of the 1987 Constitution is not applicable to the PCGG Chairman nor to the capacity, and the primary functions of one office do not require an appointment
CPLC, as neither of them is a secretary, undersecretary, nor an assistant to the other post. Moreover, even if the appointments in question are not
secretary, even if the former may have the same rank as the latter positions. covered by Section 13, Article VII of the 1987 Constitution, said appointments
It must be emphasized, however, that despite the non-applicability of Section are still prohibited under Section 7, Article IX-B, which covers all appointive
13, Article VII of the 1987 Constitution to respondent Elma, he remains and elective officials, due to the incompatibility between the primary functions
covered by the general prohibition under Section 7, Article IX-B and his of the offices of the PCGG Chairman and the CPLC.
appointments must still comply with the standard of compatibility of officers NOTE:
laid down therein; failing which, his appointments are hereby pronounced in
violation of the Constitution.
RULE ON ADDITIONAL POSITIONS
In the aforecited case Civil Liberties Union v. Executive Secretary, the Court
stressed that the language of Section 13, Article VII is a definite and 1. Elective GR: Not allowed as per Sec. 7.
unequivocal negation of the privilege of holding multiple offices or
employment. The Court cautiously allowed only two exceptions to the rule (except President and XPN: In an ex-officio capacity.
against multiple offices: (1) those provided for under the Constitution, such as VP)
Section 3, Article VII, authorizing the Vice-President to become a member of
the Cabinet; or (2) posts occupied by the Executive officials specified in Section 2. Appointive GR: Not allowed as per Sec. 7.
13, Article VII without additional compensation in an ex-officio capacity as
provided by law and as required by the primary functions of said officials’ office. (except Cabinet XPNs:
The Court further qualified that additional duties must not only be closely Secretaries,
Undersecretaries, and 1. Allowed or provided by law.
related to, but must be required by the official’s primary functions. Moreover,
Assistant Secretaries) 2. Allowed by the primary functions of
the additional post must be exercised in an ex-officio capacity, which "denotes
his office.
an act done in an official character, or as a consequence of office, and without
3. Civil Liberties case: In an ex-officio
any other appointment or authority than that conferred by the office."18 Thus,
capacity.
it will not suffice that no additional compensation shall be received by virtue of
3. President
the second appointment, it is mandatory that the second post is required by GR: Not allowed.
Vice President,
the primary functions of the first appointment and is exercised in an ex-officio
Cabinet Secretaries, XPNs:
capacity.
Undersecretaries, and
With its forgoing qualifications, it is evident that even Section 13, Article VII Assistant Secretaries 1. Provided by the Constitution (Art.
does not sanction this dual appointment. Appointment to the position of PCGG VII, Sec. 13)
Chairman is not required by the primary functions of the CPLC, and vice versa. a. VP as Cabinet member;
The primary functions of the PCGG Chairman involve the recovery of ill-gotten b. SOJ as JBC;
wealth accumulated by former President Ferdinand E. Marcos, his family and c. President as NEDA Head;
associates, the investigation of graft and corruption cases assigned to him by d. President as head of all executive
the President, and the adoption of measures to prevent the occurrence of departments (doctrine of qualified
corruption. On the other hand, the primary functions of the CPLC encompass political agency).
a different matter, that is, the review and/or drafting of legal orders referred
If the in-rank cabinet secretary is not governed under #3 (see table
to him by the President. And while respondent Elma did not receive additional
above) he is governed by #2 (also considered as the default rule.
compensation in connection with his position as CPLC, he did not act as either
CPLC or PGCC Chairman in an ex-officio capacity. The fact that a separate (4) Military – Art. XVI, Sec. 5 (4)
appointment had to be made for respondent Elma to qualify as CPLC negates
the premise that he is acting in an ex-officio capacity. Section 5. No member of the armed forces in the active service shall, at any time,
be appointed or designated in any capacity to a civilian position in the Government,
including government-owned or controlled corporations or any of their subsidiaries.

M.R.A.D.C. LUMBRE 83
CONSTITUTIONAL LAW REVIEW

3. Presidential Immunity It has been advanced that whatever power inherent in the government that is
neither legislative nor judicial has to be executive.
i. Soliven vs. Makasiar; Beltran vs. Makasiar, 167 SCRA 393 (1988)
The Constitution declares among the guiding principles that "[t]he prime duty
RULING: Anent the third issue, petitioner Beltran argues that "the reasons which of the Government is to serve and protect the people" and that "[t]he
necessitate presidential immunity from suit impose a correlative disability to file maintenance of peace and order, the protection of life, liberty, and property,
suit." He contends that if criminal proceedings ensue by virtue of the President's and the promotion of the general welfare are essential for the enjoyment by
filing of her complaint-affidavit, she may subsequently have to be a witness for the all the people of the blessings of democracy." [Art. II, Secs. 4 and 5.]
prosecution, bringing her under the trial court's jurisdiction. This, continues Beltran,
would in an indirect way defeat her privilege of immunity from suit, as by testifying Having sworn to defend and uphold the Constitution, the President has the
on the witness stand, she would be exposing herself to possible contempt of court obligation under the Constitution to protect the people, promote their welfare
or perjury. and advance the national interest.

The rationale for the grant to the President of the privilege of immunity from suit The power involved is the President's residual power to protect the general
is to assure the exercise of Presidential duties and functions free from any welfare of the people. It is founded on the duty of the President, as steward of
hindrance or distraction, considering that being the Chief Executive of the the people. It is a power borne by the President's duty to preserve and defend
Government is a job that, aside from requiring all of the office holder's time, also the Constitution.
demands undivided attention. But this privilege of immunity from suit, pertains to
the President by virtue of the office and may be invoked only by the holder of the b. Control power – Art. VII, Sec. 17
office; not by any other person in the President's behalf. Thus, an accused in a Section 17. The President shall have control of all the executive departments,
criminal case in which the President is complainant cannot raise the presidential bureaus, and offices. He shall ensure that the laws be faithfully executed.
privilege as a defense to prevent the case from proceeding against such accused.
i. The Doctrine of Qualified Political Agency
Moreover, there is nothing in our laws that would prevent the President from
waiving the privilege. Thus, if so minded the President may shed the protection Doctrine of qualified political agency or alter ego principle means that the acts
afforded by the privilege and submit to the court's jurisdiction. The choice of of the secretaries of the Executive departments performed and promulgated in
whether to exercise the privilege or to waive it is solely the President's prerogative. the regular course of business are presumptively the acts of the Chief
It is a decision that cannot be assumed and imposed by any other person. Executive. (Villena v. Secretary of the Interior, G.R. No. L46570, April 21,
1939)
4. Powers of the President
Exceptions:
a. Executive power – Art. VII, Secs. 1 and 17
1. In cases wherein the Chief Executive is required by the Constitution or by
Section 1. The executive power shall be vested in the President of the Philippines. the law to act in person, or
Section 17. The President shall have control of all the executive departments, 2. The exigencies of the situation demand that he act personally, the
bureaus, and offices. He shall ensure that the laws be faithfully executed. multifarious executive and administrative functions of the Chief Executive are
i. Residual power - Marcos vs. Manglapus, 177 SCRA 668, 178 SCRA performed by and through the executive departments.
760 All the different executive and administrative organizations are mere adjuncts
ISSUE: Whether in the exercise of the powers granted by the Constitution the of the executive department. This is an adjunct of the Doctrine of Single
President may prohibit the Marcoses from returning to the Philippines? (Or by Executive.
enumerating certain powers of the President did the framers of the Constitution The heads of the various executive departments are assistants and agents of
intend that the President shall exercise those specific powers and no other) the Chief Executive. [Villena v. Sec. of Interior (1939)]
RULING: Yes, The Supreme Court ruled that although the 1987 Constitution In the case of Abakada Guro v. Executive Secretary, G.R. No. 168056, Sept.
imposes limitations on the exercise of specific powers of the President, it 1, 2005, the SC held that the Secretary of Finance can act as an agent of the
maintains intact what is traditionally considered as within the scope of Legislative Dept. to determine and declare the event upon which its expressed
"executive power." Corollarily, the powers of the President cannot be said to will is to take effect. Thus, being the agent of Congress and not of the
be limited only to the specific powers enumerated in the Constitution. In other President, the latter cannot alter, or modify or nullify, or set aside the findings
words, executive power is more than the sum of specific powers so of the Secretary of Finance and to substitute the judgment of the former for
enumerated, that of the latter.

M.R.A.D.C. LUMBRE 84
CONSTITUTIONAL LAW REVIEW

ii. Ocampo vs. Rear Admiral Enriquez, G.R. No. 225973, 8 November Department, the heads of the various executive departments are assistants
2016 and agents of the Chief Executive, and, except in cases where the Chief
Executive is required by the Constitution or law to act in person or the
RULING: The President's decision to bury Marcos at the LNMB is not done exigencies of the situation demand that he act personally, the multifarious
whimsically, capriciously or arbitrarily, out of malice, ill will or personal bias. executive and administrative functions of the Chief Executive are performed
The LNMB was not expressly included in the national shrines enumerated in PD by and through the executive departments, and the acts of the Secretaries of
105. P.D. No. 105 does not apply to the LNMB. Despite the fact that P.D. No. such departments, performed and promulgated in the regular course of
208 predated P.D. No. 105, the LNMB was not expressly included in the business, are, unless disapproved or reprobated by the Chief Executive
national shrines enumerated in the latter. The proposition that the LNMB is presumptively the acts of the Chief Executive.
implicitly covered in the catchall phrase "and others which may be proclaimed It has been held that an administrative regulation adopted pursuant to law has
in the future as National Shrines" is erroneous because: (1) As stated, Marcos the force and effect of law and, until set aside, is binding upon executive and
issued P.D. No. 208 prior to P.D. No. 105; (2) Following the canon of statutory administrative agencies, including the President as the chief executor of laws.
construction known as ejusdem generis, 138 the LNMB is not a site "of the
birth, exile, imprisonment, detention or death of great and eminent leaders of iii. Power Sector Asset and Liabilities Management Corporation
the nation,"; and (3) Since its establishment, the LNMB has been a military (PSALM) vs. Commissioner of Internal Revenue, G.R. No. 198146, 8
shrine under the jurisdiction of the PVAO. August 2017
The LNMB is considered as a national shrine for military memorials. The PVAO, RULING: Since this case is a dispute between PSALM arid NPC, both
which is empowered to administer, develop, and maintain military shrines, is government owned and controlled corporations, and the BIR, a National
under the supervision and control of the DND. The DND, in tum, is under the Government office, PD 242 clearly applies and the Secretary of Justice has
Office of the President. jurisdiction over this case. In fact, the MOA executed by the BIR, NPC, and
PSALM explicitly provides that "[a] ruling from the Department of Justice (DOJ)
The presidential power of control over the Executive Branch of Government is that is favorable to NPC/PSALM shall be tantamount to the filing of an
a self-executing provision of the Constitution and does not require statutory application for refund (in cash)/tax credit certificate (TCC), at the option of
implementation, nor may its exercise be limited, much less withdrawn, by the NPC/PSALM." Such provision indicates that the BIR and petitioner PSALM and
legislature. This is why President Duterte is not bound by the alleged 1992 the NPC acknowledged that the Secretary of Justice indeed has jurisdiction to
Agreement between former President Ramos and the Marcos family to have resolve their dispute.
the remains of Marcos interred in Batac, Ilocos Norte. As the incumbent
President, he is free to amend, revoke or rescind political agreements entered It is only proper that intra-governmental disputes be settled administratively
into by his predecessors, and to determine policies which he considers, based since the opposing government offices, agencies and instrumentalities are all
on informed judgment and presumed wisdom, will be most effective in carrying under the President's executive control and supervision. Section 17, Article VII
out his mandate. of the Constitution states unequivocally that: "The President shall have control
of all the executive departments, bureaus and offices. He shall ensure that the
Moreover, under the Administrative Code, the President has the power to laws be faithfully executed."
reserve for public use and for specific public purposes any of the lands of the
public domain and that the reserved land shall remain subject to the specific In Carpio v. Executive Secretary, the Court expounded on the President's
public purpose indicated until otherwise provided by law or proclamation. At control over all the executive departments, bureaus and offices, thus:
present, there is no law or executive issuance specifically excluding the land in
which the LNMB is located from the use it was originally intended by the past This presidential power of control over the executive branch of government
Presidents. The allotment of a cemetery plot at the LNMB for Marcos as a extends over all executive officers from Cabinet Secretary to the lowliest clerk
former President and Commander-in-Chief, a legislator, a Secretary of National and has been held by us, in the landmark case of Mondano vs. Silvosa, to mean
Defense, a military personnel, a veteran, and a Medal of Valor awardee, "the power of [the President] to alter or modify or nullify or set aside what a
whether recognizing his contributions or simply his status as such, satisfies the subordinate officer had done in the performance of his duties and to substitute
public use requirement. The disbursement of public funds to cover the the judgment of the former with that of the latter." It is said to be at the very
expenses incidental to the burial is granted to compensate him for valuable "heart of the meaning of Chief Executive."
public services rendered. Equally well accepted, as a corollary rule to the control powers of the President,
In Joson v. Torres, we explained the concept of the alter ego principle or the is the "Doctrine of Qualified Political Agency." As the President cannot be
doctrine of qualified political agency and its limit in this wise: expected to exercise his control powers all at the same time and in person, he
will have to delegate some of them to his Cabinet members.
Under this doctrine, which recognizes the establishment of a single executive,
all executive and administrative organizations are adjuncts of the Executive

M.R.A.D.C. LUMBRE 85
CONSTITUTIONAL LAW REVIEW

Under this doctrine, which recognizes the establishment of a single executive, At the expiration of sixty days, the suspended official shall be deemed in office
"all executive and administrative organizations are adjuncts of the Executive without prejudice to the continuation of the proceedings against him until its
Department, the heads of the various executive departments are assistants termination. However, if the delay in the proceedings of the case is due to his
and agents of the Chief Executive, and, except in cases where the Chief fault, neglect or request, the time for the delay shall not be counted in
Executive is required by the Constitution or law to act in person on the computing the time of suspension.
exigencies of the situation demand that he act personally, the multifarious
executive and administrative functions of the Chief Executive are performed The Constitution did nothing more, however, and insofar as existing legislation
by and through the executive departments, and the acts of the Secretaries of authorizes the President (through the Secretary of Local Government) to
such departments, performed and promulgated in the regular course of proceed against local officials administratively, the Constitution contains no
business, are, unless disapproved or reprobated by the Chief Executive prohibition.
presumptively the acts of the Chief Executive." Control has been defined as the power of an officer to alter or modify or nullify
Thus, and in short, "the President's power of control is directly exercised by or set aside what a subordinate officer had done in the performance of his
him over the members of the Cabinet who, in turn, and by his authority, control duties and to substitute the judgment of the former to test of the latter.
the bureaus and other offices under their respective jurisdictions in the Supervision on the other hand means “overseeing or the power or authority
executive department." of an officer to see that subordinate officers perform their duties.”
This power of control vested by the Constitution in the President cannot be As the Constitution itself declares, local autonomy means “a more responsive
diminished by law. and accountable local government structure instituted through a system of
c. Power of general supervision – Art. X, Secs. 4 and 16 decentralization. The Constitution, as we observed, does nothing more than to
break up the monopoly of the national government over the affairs of local
Section 4. The President of the Philippines shall exercise general supervision over governments and as put by political adherents, to “liberate the local
local governments. Provinces with respect to component cities and municipalities, governments from the imperialism of Manila.” Autonomy, however, is not
and cities and municipalities with respect to component barangays, shall ensure meant to end the relation of partnership and interdependence between the
that the acts of their component units are within the scope of their prescribed central administration and local government units, or otherwise, to usher in a
powers and functions. regime of federalism.

Section 16. The President shall exercise general supervision over autonomous d. Power of appointment- Art. VII, Secs. 13 to 16
regions to ensure that laws are faithfully executed.
Section 13. The President, Vice-President, the Members of the Cabinet, and their
i. Ganzon vs. CA, 200 SCRA 271 (1991) deputies or assistants shall not, unless otherwise provided in this Constitution, hold
any other office or employment during their tenure. They shall not, during said
ISSUE: Whether the Secretary of the DILG, as the alter ego of the President, tenure, directly or indirectly, practice any other profession, participate in any
can order the preventive suspension of a City Mayor, finding probable cause. business, or be financially interested in any contract with, or in any franchise, or
RULING: It is to be noted that in meting out the suspensions, the Secretary special privilege granted by the Government or any subdivision, agency, or
of Local Government acted in consonance with the specific legal provision of instrumentality thereof, including government-owned or controlled corporations or
the LGC: their subsidiaries. They shall strictly avoid conflict of interest in the conduct of their
office.
Sec. 63. Preventive Suspension. – Preventive suspension may be imposed by
the Minister of Local Government if the respondent is a provincial or city official, The spouse and relatives by consanguinity or affinity within the fourth civil degree
by the provincial governor if the respondent is an elective municipal, or by the of the President shall not, during his tenure, be appointed as Members of the
city or municipal mayor of the respondent is an elective barangay official. Constitutional Commissions, or the Office of the Ombudsman, or as Secretaries,
Undersecretaries, chairmen or heads of bureaus or offices, including government-
Preventive suspension may be imposed at any time after the issues are joined, owned or controlled corporations and their subsidiaries.
when there is reasonable ground to believe that the respondent has committed
the act or acts complained of, when the evidence of culpability is strong, when Section 14. Appointments extended by an Acting President shall remain effective,
the gravity of the offense so warrants, or when the continuance in office of the unless revoked by the elected President, within ninety days from his assumption or
respondent could influence the witnesses or pose a threat to the safety and reassumption of office.
integrity for the records and other evidence. In all cases, preventive Section 15. Two months immediately before the next presidential elections and
suspension shall not extend beyond sixty days after the start of said up to the end of his term, a President or Acting President shall not make
suspension. appointments, except temporary appointments to executive positions when
continued vacancies therein will prejudice public service or endanger public safety.

M.R.A.D.C. LUMBRE 86
CONSTITUTIONAL LAW REVIEW

Section 16. The President shall nominate and, with the consent of the Commission and binding on both the appointee and the appointing power. But when an ad
on Appointments, appoint the heads of the executive departments, ambassadors, interim appointment is by-passed because of lack of time or failure of the
other public ministers and consuls, or officers of the armed forces from the rank of Commission on Appointments to organize, there is no final decision by the
colonel or naval captain, and other officers whose appointments are vested in him Commission to give or withhold its consent to the appointment. Absent such
in this Constitution. He shall also appoint all other officers of the Government whose decision, the President is free to renew the ad interim appointment.
appointments are not otherwise provided for by law, and those whom he may be
authorized by law to appoint. The Congress may, by law, vest the appointment of Section 16. xxxx The President shall have the power to make appointments
other officers lower in rank in the President alone, in the courts, or in the heads of during the recess of the Congress, whether voluntary or compulsory, but such
departments, agencies, commissions, or boards. appointments shall be effective only until disapproved by the Commission on
Appointments or until the next adjournment of the Congress.
The President shall have the power to make appointments during the recess of the
Congress, whether voluntary or compulsory, but such appointments shall be ii. Ad interim vs. in an acting capacity
effective only until disapproved by the Commission on Appointments or until the An Ad Interim appointment is one made during the recess of Congress and
next adjournment of the Congress. requires the confirmation of the Commission on Appointments. It is permanent
Permanent appointments are those extended to persons possessing the in nature, and the appointee enjoys security of tenure.
qualifications and the requisite eligibility and are thus protected by the An appointment in an acting capacity is made at any time there is a
constitutional guarantee of security of tenure. Temporary appointments are vacancy, whether Congress is in session or not, and does not require
given to persons without such eligibility, revocable at will and without the necessity confirmation of the Commission on Appointments. It is temporary in nature;
of just cause or a valid investigation; made on the understanding that the as such, the appointee does not enjoy security of tenure.
appointing power has not yet decided on a permanent appointee and that the
temporary appointee may be replaced at any time a permanent choice is made. iii. Steps in the appointing process

A temporary appointment and a designation are not subject to confirmation by the 1. Lacson vs. Romero, 84 Phil 740
Commission on Appointments. Such confirmation, if given erroneously, will not
RULING: Before deciding on the case, the Court explained the steps
make the incumbent a permanent appointee [Valencia v. Peralta, 8 SCRA 692].
involved in the appointment to a government post like the provincial fiscal.
i. Regular vs. ad interim - Art. VII, Sec. 16, 2nd par.
First, there must be nomination by the President. Second, to make that
A regular appointment is one made by the President while Congress is in nomination valid and permanent, the Commission on Appointments of the
session, takes effect only after confirmation by the Commission on Legislature has to confirm said nomination. Lastly, the appointee must
Appointments, and once approved, continues until the end of the term of the accept it through his assumption of the office. The first two steps,
appointee. An ad interim appointment is one made by the President while nomination and confirmation, constitute a mere offer of a post. They are
Congress is not in session, takes effect immediately, but ceases to be valid if acts of the Executive and Legislative departments of the Government. But
disapproved by the Commission on Appointments or upon the next the last necessary step to make the appointment complete and effective
adjournment of Congress. In the latter case, the ad interim appointment is rests solely with the appointee himself. He may or he may not accept the
deemed “by-passed” through inaction. The ad interim appointment is intended appointment or nomination.
to prevent interruptions in vital government services that would otherwise
As held in Borromeo v. Mariano, "there is no power in this country which
result from prolonged vacancies in government offices.
can compel a man to accept an office." Consequently, since Lacson has
An ad interim appointment is a permanent appointment because it takes effect declined to accept his appointment as provincial fiscal of Tarlac and no one
immediately and can no longer be withdrawn by the President once the can compel him to do so, then he continues as provincial fiscal of Negros
appointee has qualified into office. The fact that it is subject to confirmation by Oriental and no vacancy in said office was created, unless Lacson had been
the Commission on Appointments does not alter its permanent character. lawfully removed as fiscal of Negros Oriental.

An ad interim appointment can be terminated for two causes specified in the It was held that the intended transfer of Lacson to Tarlac on the basis of
Constitution: disapproval of the appointment by the Commission on his nomination thereto, if carried out, would be equivalent to a removal
Appointments, or adjournment by Congress without the Commission on from his office in Negros Oriental. To appoint and transfer him from one
Appointments acting on the appointment. There is no dispute that when the province to another would mean his removal or separation from the first
Commission on Appointments disapproves an ad interim appointment, the province. The reason is that a fiscal is appointed for each province and
appointee can no longer be extended a new appointment, inasmuch as the Lacson could not well and legally hold and occupy the two posts of fiscal
disapproval is a final decision of the Commission in the exercise of its checking of Tarlac and Negros Oriental simultaneously. To be fiscal for Tarlac must
power on the appointing authority of the President. Such disapproval is final mean his removal from Negros Oriental.

M.R.A.D.C. LUMBRE 87
CONSTITUTIONAL LAW REVIEW

The Court held that the president cannot not remove Lacson at will and and another Commissioner for three years, without reappointment.
without cause. The Constitution, particularly Article XII Section 4, prohibits Appointment to any vacancy shall be only for the unexpired term of the
the removal or suspension of an officer or an employee in the civil service, predecessor. In no case shall any Member be appointed or designated in
except for causes provided by law. This is a safeguard to protect civil a temporary or acting capacity. (Article IX-B)
service officials and employees, secure to them a reasonable tenure of
office and thus give the country the benefit of an efficient civil service Section 1. The Chairman and the Commissioners shall be appointed by
based on the merit system. Since a provincial fiscal is a civil service official, the President with the consent of the Commission on Appointments for a
Lacson enjoys security of tenure and therefore he cannot be removed by term of seven years without reappointment. Of those first appointed, three
the President from his office without any cause, as provided by law. Members shall hold office for seven years, two Members for five years,
and the last Members for three years, without reappointment.
To be fair with the President and the CA, the Court clarified that Lacson’s Appointment to any vacancy shall be only for the unexpired term of the
appointment in Tarlac was not a form of punishment or demotion but predece6ssor. In no case shall any Member be appointed or designated in
rather, a promotion at that time. However, since were changes in the a temporary or acting capacity. (Article IX-C)
classification of Negros Oriental, it was not anymore a promotion in salary.
Section 1. The Chairman and the Commissioners shall be appointed by
In the end, the Court declared Lacson as the provincial fiscal of Negros the President with the consent of the Commission on Appointments for a
Oriental and ordered Romero to surrender the post. term of seven years without reappointment. Of those first appointed, the
Chairman shall hold office for seven years, one Commissioner for five
2. Art. II, Sec. 4 years, and the other Commissioner for three years, without
Section 4. The prime duty of the Government is to serve and protect the reappointment. Appointment to any vacancy shall be only for the
people. The Government may call upon the people to defend the State unexpired portion of the term of the predecessor. In no case shall any
and, in the fulfillment thereof, all citizens may be required, under Member be appointed or designated in a temporary or acting capacity.
conditions provided by law, to render personal, military or civil service. (Article IX-D)

NOTE: This power is also known as “posse commitatus.” Unlike the Section 18. The Congress shall enact an organic act for each autonomous
appointments made by the President, this power does not require region with the assistance and participation of the regional consultative
acceptance by the persons called upon to render service. commission composed of representatives appointed by the President from
a list of nominees from multi-sectoral bodies. The organic act shall define
iv. Presidential appointees: the basic structure of government for the region consisting of the
executive department and legislative assembly, both of which shall be
1. Requires CA confirmation - Art. VII, Sec. 16, 1st sentence;
elective and representative of the constituent political units. The organic
Relate with Art. VIII, Sec. 8(2), Art. IX-B, Sec. 1(2), Art. IX-C, Sec.
acts shall likewise provide for special courts with personal, family, and
1(2), Art. IX-D, Sec. 1(2), and Art. X, Sec. 18 (Ask Sir for a
property law jurisdiction consistent with the provisions of this Constitution
summary of relevant landmark cases)
and national laws. (Article X)
Section 16. The President shall nominate and, with the consent of the
Valencia v. Peralta, 8 SCRA 692 (1963)
Commission on Appointments, appoint the heads of the executive
departments, ambassadors, other public ministers and consuls, or officers A temporary appointment and a designation are not subject to
of the armed forces from the rank of colonel or naval captain, and other confirmation by the Commission on Appointments. Such confirmation, if
officers whose appointments are vested in him in this Constitution. xxx given erroneously, will not make the incumbent a permanent appointee
(Article VII) because confirmation presupposes a valid nomination or recess
appointment (i.e. ad interim appointment is a permanent appointment
Section 8. xxx The regular members of the Council (JBC) shall be
(Pamantasan ng Lungsod ng Maynila v. Intermediate Appellate Court, 140
appointed by the President for a term of four years with the consent of the
SCRA 22).
Commission on Appointments. Of the Members first appointed, the
representative of the Integrated Bar shall serve for four years, the Pacete v. The Secretary of the Commission on Appointments, 40
professor of law for three years, the retired Justice for two years, and the SCRA 58 (1971)
representative of the private sector for one year. (Article VIII)
The Supreme Court held that a mere motion for reconsideration by a
Section 1. The Chairman and the Commissioners shall be appointed by member of the Commission on Appointments to a confirmation (Sec. 18,
the President with the consent of the Commission on Appointments for a Chapter V, CA Rules) duly made (and filed before the same body) which
term of seven years without reappointment. Of those first appointed, the is not approved cannot have the effect of setting aside such confirmation.
Chairman shall hold office for seven years, a Commissioner for five years, A vote of confirmation thus entitles the appointee to invoke constitutional

M.R.A.D.C. LUMBRE 88
CONSTITUTIONAL LAW REVIEW

protection. To argue that the mere filing of a motion for reconsideration Subsequent renewals by the President of ad interim appointment of
did suffice to set it aside, even in the absence of any further action, is to Commissioners of the COMELEC do not violate the constitutional provision
lose sight of what is provided in the Constitution. It is tantamount to one- proscribing their reappointment (Section 1 (2), Article IX-C) because
man rule. Further, it was held that the rules of the Commission on Commission on Appointments (CA) did not act on said appointments after
Appointments concerning its internal business could be reviewed by the submission by the President. An ad interim appointment that has lapsed
Court, that is, it is a justiciable question, when a certain construction of by inaction of the Commission on Appointments does not constitute a term
such rules would defeat the right of the individual to a public office. of office. The phrase "without reappointment" applies only to one who has
been appointed by the President and confirmed by the Commission on
Sarmiento III vs. Mison, 156 SCRA 549 (1987) Appointments, whether or not such person completes his term of office.
The appointment of the Commissioner of the Bureau of Customs (BOC) is There must be a confirmation by the Commission on Appointments of the
not subject to the confirmation power of the Commission on Appointments previous appointment before the prohibition on reappointment can apply.
(CA) because Section 16, Article VII of the Constitution identifies “heads Also in this case, the Court pronounced that an ad interim appointment is
of executive departments” as appointments subject to the confirmation a permanent appointment because it takes effect immediately and can no
power of the CA, excluding “heads of bureaus.” longer be withdrawn by the President once the appointee has qualified into
Bautista v. Salonga, 172 SCRA 160 (1989) office.

Following the reasoning in Sarmiento, the appointment of the Chairman Finally, the court likewise opined that the disapproval by the CA is final
of the Commission on Human Rights does not need confirmation by the and binding on the appointee as well as on the appointing power. In this
Commission on Appointments. instance, the President can no longer renew the appointment not because
of constitutional prohibition on reappointment, but because of a final
Calderon v. Carale, G.R. No. 91636 April 23, 1992 decision by the CA to withhold its consent to the appointment.
Congress may not constitutionally expand by law the list of presidential Fetalino v. COMELEC, 686 SCRA 813 (2012)
appointees subject to confirmation to include officers other than those
expressly mentioned in the first sentence of Section 16, Art. VII (heads of The Supreme Court reiterated that an ad interim appointment that has
the executive departments, ambassadors, other public ministers and lapsed by inaction of the Commission on Appointments (CA) does not
consuls, or officers of the armed forces from the rank of colonel or naval constitute a term of office. Hence, the appointee can never be considered
captain, regular members of the JBC, members of the Regional to have retired from the service to be able to claim for retirement benefits.
Consultative Council, and Chairmen and members of CSC, COMELEC and 2. By the president alone - Art. VII, Sec. 16, 2nd sentence; Art. VII,
COA) because such law would amend the constitutional provision by Sec. 3, 2nd par.
adding thereto appointments which are otherwise entrusted only to the
President. Section 16. xxx He shall also appoint all other officers of the Government
whose appointments are not otherwise provided for by law, and those
Manalo v. Sistoza, 312 SCRA 239 (1999) whom he may be authorized by law to appoint. Xxx
The police force is different from and independent of the armed forces and Section 3. xxx The Vice-President may be appointed as a Member of the
the ranks in the military are not similar to those in the Philippine National Cabinet. Such appointment requires no confirmation.
Police. Thus, Sections 26 and 31 of Republic Act 6975 which empower the
Commission on Appointments (CA) to confirm the appointments of 3. Requires JBC nomination – Art. VIII, Sec. 9 and Art. XI, Sec. 9
Directors and Senior Superintendents of the PNP, although with equivalent
rank of military colonel and higher, are unconstitutional. Section 9. The Members of the Supreme Court and judges of lower courts
shall be appointed by the President from a list of at least three nominees
Soriano v. Lista, 447 Phil. 566 (2003) preferred by the Judicial and Bar Council for every vacancy. Such
appointments need no confirmation.
Following the reasoning in Manalo, appointments among the Philippine
Coast Guard do not need confirmation by the Commission on For the lower courts, the President shall issue the appointment within
Appointments (CA). The paramount effect of the transfer of the Philippine ninety days from the submission of the list. (Article VIII)
Coast Guard (PCG) from the DND to the Office of the President and
eventually to the DOTC is the transformation of the PCG into a non-military Section 9. The Ombudsman and his Deputies shall be appointed by the
agency. Thus, the PCG is already civilian in character. President from a list of at least six nominees prepared by the Judicial and
Bar Council, and from a list of three nominees for every vacancy
Matibag v. Benipayo, G.R. No. 149036, April 2, 2002

M.R.A.D.C. LUMBRE 89
CONSTITUTIONAL LAW REVIEW

thereafter. Such appointments shall require no confirmation. All vacancies can be placed in a cluster with many strong contenders to minimize
shall be filled within three months after they occur. (Article XI) his/her chances of appointment.

a. Aguinaldo vs. President Aquino III, G.R. No. 224302, 29 The Court further points out that its Decision dated November 29,
November 2016 2016 only discussed vacancies in collegiate courts. The constant
referral by the JBC to separate short lists of nominees for vacant
FACTS: Petitioners insist that President Aquino could only choose one judgeship posts in first and second level trial courts as proof of
nominee from each of the six separate shortlists submitted by the JBC previous clustering is inapt. The separate short lists in such situations
for each specific vacancy, and no other; and any appointment made are technically not clustering as the vacancies happened and were
in deviation of this procedure is a violation of the Constitution. Hence, announced at different times and candidates applied for specific
petitioners pray, among other reliefs, that the appointments of vacancies, based on the inherent differences in the location and
respondents Musngi and Econg, who belonged to the same shortlist jurisdiction of the trial courts, as well as the qualifications of nominees
for the position of 21st Associate Justice, be declared null and void for to the same, hence, justifying a separate short list for each vacant
these were made in violation of Article VIII, Section 9 of the 1987 post.
Constitution.
President Aquino validly exercised his discretionary power to appoint
RULING: The clustering of nominees for the six vacancies in the members of the Judiciary when he disregarded the clustering of
Sandiganbayan by the JBC impaired the President's power to appoint nominees into six separate shortlists for the vacancies for the 16th,
members of the Judiciary and to determine the seniority of the newly- 17th, 18th, 19th, 20th, and 21st Sandiganbayan Associate Justices.
appointed Sandiganbayan Associate Justices. President Aquino merely maintained the well-established practice,
In its Decision dated November 29, 2016, the Court ruled that the consistent with the paramount Presidential constitutional prerogative,
clustering impinged upon the President's appointing power in the to appoint the six new Sandiganbayan Associate Justices from the 37
following ways: The President's option for every vacancy was limited qualified nominees, as if embodied in one JBC list. This does not
to the five to seven nominees in each cluster. Once the President had violate Article VIII, Section 9 of the 1987 Constitution which requires
appointed a nominee from one cluster, then he was proscribed from the President to appoint from a list of at least three nominees
considering the other nominees in the same cluster for the other submitted by the JBC for every vacancy. To meet the minimum
vacancies. All the nominees applied for and were found to be qualified requirement under said constitutional provision of three nominees per
for appointment to any of the vacant Associate Justice positions in the vacancy, there should at least be 18 nominees from the JBC for the
Sandiganbayan, but the JBC failed to explain why one nominee should six vacancies for Sandiganbayan Associate Justice; but the minimum
be considered for appointment to the position assigned to one specific requirement was even exceeded herein because the JBC submitted
cluster only. Correspondingly, the nominees' chance for appointment for the President's consideration a total of 37 qualified nominees. All
was restricted to the consideration of the one cluster in which they the six newly appointed Sandiganbayan Associate Justices met the
were included, even though they applied and were found to be requirement of nomination by the JBC under Article VIII, Section 9 of
qualified for all the vacancies. Moreover, by designating the numerical the 1987 Constitution. Hence, the appointments of respondents
order of the vacancies, the JBC established the seniority or order of Musngi and Econg, as well as the other four new Sandiganbayan
preference of the new Sandiganbayan Associate Justices, a power Associate Justices, are valid and do not suffer from any constitutional
which the law (Section 1, paragraph 3 of Presidential Decree No. infirmity.
1606), rules (Rule II, Section 1 (b) of the Revised Internal Rules of NOTE: This kind of appointment applies where:
the Sandiganbayan), and jurisprudence (Re: Seniority Among the
Four Most Recent Appointments to the Position of Associate Justices (1) There are closely-related, successive vacancies within the 90-day
of the Court of Appeals), vest exclusively upon the President. period to fill in vacancy;
Clustering can be used as a device to favor or prejudice a qualified (2) It involves a collegiate court, i.e. there is more than one judge
nominee. The Court does not impose upon the JBC such duty, it only sitting;
requires that the JBC gives all qualified nominees fair and equal
opportunity to be appointed. The clustering by the JBC of nominees (3) The President will issue appointments in one occasion only.
for simultaneous or closely successive vacancies in collegiate courts v. Limitations on the appointing power of the president
can actually be a device to favor or prejudice a particular nominee. A
favored nominee can be included in a cluster with no other strong 1. Relatives – Art. VII, Sec. 13 (2nd par)
contender to ensure his/her appointment; or conversely, a nominee
Section 13. xxx The spouse and relatives by consanguinity or affinity
within the fourth civil degree of the President shall not, during his tenure,

M.R.A.D.C. LUMBRE 90
CONSTITUTIONAL LAW REVIEW

be appointed as Members of the Constitutional Commissions, or the Office Section 15, Article VII is directed against two types of
of the Ombudsman, or as Secretaries, Undersecretaries, chairmen or appointments: (1) those made for buying votes and (2) those
heads of bureaus or offices, including government-owned or controlled made for partisan considerations. The first refers to those
corporations and their subsidiaries. appointments made within the two months preceding a
Presidential election and are similar to those which are declared
2. Midnight appointment – Art. VII, Sec. 15 election offenses in the Omnibus Election Code, viz.:
Section 15. Two months immediately before the next presidential “SEC. 261. Prohibited Acts.— The following shall be guilty of
elections and up to the end of his term, a President or Acting President an election offense:
shall not make appointments, except temporary appointments to
executive positions when continued vacancies therein will prejudice public (a) Vote-buying and vote-selling.—(1) Any person who gives,
service or endanger public safety. offers or promises money or anything of value, gives or
promises any office or employment, franchise or grant, public
NOTE: Contrary to public knowledge, midnight appointments refer to or private, or makes or offers to make an expenditure,
those made in between the period of date of elections until before the directly or indirectly, or cause an expenditure to be made to
inauguration of the President. These are also called appointments made any person, association, corporation, entity, or community in
for partisan considerations. order to induce anyone or the public in general to vote for
Appointments made during the start of the 2-month period before or against any candidate or withhold his vote in the election, or
elections and until the date of elections are called appointments made for to vote for or against any aspirant for the nomination or
buying votes. choice of a candidate in a convention or similar selection
process of a political party.
a. In Re: Hon. Mateo A. Valenzuela and Hon. Placido B.
Vallarta, 298 SCRA 408, 9 November 1998 (g) Appointment of new employees, creation of new position,
promotion, or giving salary increases.—During the period of
FACTS: Notwithstanding the opinion given by the Chief Justice (that forty-five days before a regular election and thirty days
the only exception to the ban on appointments applies only to before a special election, (1) any head, official or appointing
executive positions to which temporary appointments may be made officer of a government office, agency or instrumentality,
within the interdicted period when the vacancies therein will prejudice whether national or local, including government-owned or
the public service or endanger public safety), on May 30, 1998 (within controlled corporations, who appoints or hires any new
period of the election ban), the President signed the appointment of employee, whether provisional, temporary, or casual, or
Hon. Mateo A. Valenzuela and Hon. Placido B. Vallarta as Judges of creates and fills any new position, except upon prior
the Regional Trial Court of Branch 62, Bago City and of Branch authority of the Commission. The Commission shall not grant
24, Cabanatuan City, respectively. The appointments were received the authority sought unless, it is satisfied that the position
at the Chief Justice’s chambers on May 12, 1998. The referral was to be filled is essential to the proper functioning of the
made in view of the aforementioned serious constitutional issue office or agency concerned, and that the position shall not
concerning said appointments. be filled in a manner that may influence the election.”
ISSUE: Whether during the period of the ban on appointments The second type of appointments prohibited by Section 15,
imposed by Section 15, Article VII, the President is nonetheless Article VII consists of the so-called “midnight” appointments. In
required to fill in the vacancies in the Judiciary, in view of Sections Aytona v. Castillo, it was held that after the proclamation of
4(1) and 9 of Article VIII? Diosdado Macapagal as duly elected President, President Carlos P.
Garcia, who was defeated in his bid for reelection, became no
RULING: The Court’s view is that during the period stated in Section
more than a “caretaker” administrator whose duty was to
15, Article VII of the Constitution—“(t)wo months immediately
“prepare for the orderly transfer of authority to the incoming
before the next presidential elections and up to the end of his
President.” Said the Court:
term”—the President is neither required to make appointments
to the courts nor allowed to do so; and that Sections 4(1) and “The filling up of vacancies in important positions, if few, and
9 of Article VIII simply mean that the President is required to so spaced as to afford some assurance of deliberate action and
fill vacancies in the courts within the time frames provided careful consideration of the need for the appointment and the
therein unless prohibited by Section 15 of Article VII. It is appointee’s qualifications may undoubtedly be permitted. But the
noteworthy that the prohibition on appointments comes into effect issuance of 350 appointments in one night and the planned
only once every six years. induction of almost all of them a few hours before the

M.R.A.D.C. LUMBRE 91
CONSTITUTIONAL LAW REVIEW

inauguration of the new President may, with some reason, be Thus, for purposes of the 2010 elections, 10 March 2010 was the
regarded by the latter as an abuse of Presidential prerogatives, cutoff date for valid appointments and the next day, 11 March 2010,
the steps taken being apparently a mere partisan effort to fill was the start of the ban on midnight appointments.
all vacant positions irrespective of fitness and other conditions,
and thereby to deprive the new administration of an opportunity On 30 June 2010, President Benigno S. Aquino III (President Aquino)
to make the corresponding appointments.” took his oath of office as President of the Republic of the Philippines.
On 30 July 2010, President Aquino issued EO 2 recalling, withdrawing,
As indicated, the Court recognized that there may well be and revoking appointments issued by President Macapagal-Arroyo
appointments to important positions which have to be made which violated the constitutional ban on midnight appointments.
even after the proclamation of the new President. Such
appointments, so long as they are “few and so spaced as to On 5 August 2010, Jose Anselmo Cadiz assumed office as Solicitor
afford some assurance of deliberate action and careful consideration General (Sol. Gen. Cadiz). On 6 August 2010, Sol. Gen. Cadiz
of the need for the appointment and the appointee’s instructed a Senior Assistant Solicitor General to inform the officers
qualifications,” can be made by the outgoing President. and employees affected by EO 2 that they were terminated from
Accordingly, several appointments made by President Garcia, which service effective the next day.
were shown to have been well considered, were upheld. ISSUES: Whether Petitioner’s appointments violate Article VII of the
Section 15, Article VII has a broader scope than the Aytona 1987, and whether E.O. No. 2 is constitutional.
ruling. It may not unreasonably be deemed to contemplate not only RULING: The petitions have no merit. All of petitioners' appointments
“midnight” appointments—those made obviously for partisan are midnight appointments and are void for violation of Section 15,
reasons as shown by their Article VII of the 1987 Constitution. The ponencia and the dissent both
number and the time of their making—but also appointments agree that the facts in all these cases show that "none of the
presumed made for the purpose of influencing the outcome of the petitioners have shown that their appointment papers (and
Presidential election. transmittal letters) have been issued (and released) before the ban."
The dates of receipt by the MRO, which in these cases are the only
On the other hand, the exception in the same Section 15 of Article reliable evidence of actual transmittal of the appointment papers by
VII—allowing appointments to be made during the period of the President Macapagal-Arroyo, are dates clearly falling during the
ban therein provided—is much narrower than that recognized in appointment ban. Thus, the ponencia and the dissent both agree that
Aytona. The exception allows only the making of temporary all the appointments in these cases are midnight appointments in
appointments to executive positions when continued vacancies violation of Section 15, Article VII of the 1987 Constitution.
will prejudice public service or endanger public safety. Obviously,
the article greatly restricts the appointing power of the President EO 2 is constitutional. Based on prevailing jurisprudence,
during the period of the ban. appointment to a government post is a process that takes several
steps to complete. Any valid appointment, including one made under
b. Atty. Velicaira-Garafil vs. Office of the President, G.R. No. the exception provided in Section 15, Article VII of the 1987
203372, 16 June 2015 Constitution, must consist of the President signing an appointee's
appointment paper to a vacant office, the official transmittal of the
FACTS: Prior to the conduct of the May 2010 elections, then appointment paper (preferably through the MRO), receipt of the
President Gloria Macapagal-Arroyo (President Macapagal-Arroyo) appointment paper by the appointee, and acceptance of the
issued more than 800 appointments to various positions in several appointment by the appointee evidenced by his or her oath of office
government offices. or his or her assumption to office.
The ban on midnight appointments in Section 15, Article VII of the The 1986 Constitutional Commission put a definite period, or an
1987 Constitution reads: empirical value, on Aytona's intangible "stratagem to beat the
Two months immediately before the next presidential elections and deadline," and also on the act of "preempting the President's
up to the end of his term, a President or Acting President shall not successor," which shows a lack of "good faith, morality and propriety."
make appointments, except temporary appointments to executive Subject to only one exception, appointments made during this period
positions when continued vacancies therein will prejudice public are thus automatically prohibited under the Constitution, regardless
service or endanger public safety. of the appointee's qualifications or even of the President's motives.
The period for prohibited appointments covers two months before the
elections until the end of the President's term. The Constitution, with
a specific exception, ended the President's power to appoint "two

M.R.A.D.C. LUMBRE 92
CONSTITUTIONAL LAW REVIEW

months immediately before the next presidential elections." For an fill the vacancy created by the compulsory retirement of Chief Justice
appointment to be valid, it must be made outside of the prohibited Reynato S. Puno by May 17, 2010, and to prepare the short list of
period or, failing that, fall under the specified exception. nominees and submit it to the incumbent President. Movants argue
that the disputed constitutional provision, Art. VII, Sec. 15 and Art.
[T]he well-settled rule in our jurisprudence, that an appointment is a VIII, Sec. 4(1), clearly intended the ban on midnight appointments to
process that begins with the selection by the appointing power and cover the members of the Judiciary, and they contended that the
ends with acceptance of the appointment by the appointee, stands. principle of stare decisis is controlling, and insisted that the Court
As early as the 1949 case of Lacson v. Romero, this Court laid down erred in disobeying or abandoning the Valenzuela ruling.
the rule that acceptance by the appointee is the last act needed to
make an appointment complete. The Court reiterated this rule in the ISSUE (Section 4): Did the Constitutional Commission extend to the
1989 case of Javier v. Reyes. In the 1996 case of Garces v. Court of Judiciary the ban on presidential appointments during the period
Appeals, this Court emphasized that acceptance by the appointee is stated in Sec. 15, Article VII?
indispensable to complete an appointment. The 1999 case of
Bermudez v. Executive Secretary, cited in the ponencia, affirms this RULING: The Constitutional Commission did not extend to the
standing rule in our jurisdiction, to wit: Judiciary the ban on presidential appointments during the period
stated in Sec. 15, Art. VII. The deliberations that the dissent of Justice
"The appointment is deemed complete once the last act required of Carpio Morales quoted from the records of the Constitutional
the appointing authority has been complied with and its acceptance Commission did not concern either Sec. 15, Art. VII or Sec. 4(1), Art.
thereafter by the appointee in order to render it effective." VIII, but only Sec. 13, Art. VII, a provision on nepotism.
The following elements should always concur in the making of a valid Election ban on appointments does not extend to the Supreme Court.
(which should be understood as both complete and effective) The Court upheld its March 17, 2010 decision ruling that the
appointment: (1) authority to appoint and evidence of the exercise of prohibition under Art. VII, Sec. 15 of the Constitution against
the authority; (2) transmittal of the appointment paper and evidence presidential appointments immediately before the next presidential
of the transmittal; (3) a vacant position at the time of appointment; elections and up to the end of the term of the outgoing president does
and (4) receipt of the appointment paper and acceptance of the not apply to vacancies in the Supreme Court, which shall be filled
appointment by the appointee who possesses all the qualifications and within ninety (90) days from the occurrence thereof.
none of the disqualifications. The concurrence of all these elements
should always apply, regardless of when the appointment is made, The ban on midnight appointments is placed in Article VII, not in
whether outside, just before, or during the appointment ban. These Article VIII, because it limits an executive, not a judicial, power.
steps in the appointment process should always concur and operate d. De Rama vs. CA, 353 SCRA 94, 28 February 2001
as a single process. There is no valid appointment if the process lacks
even one step. And, unlike the dissent's proposal, there is no need to RULING: The CSC correctly ruled that the constitutional prohibition
further distinguish between an effective and an ineffective on so-called "midnight appointments," specifically those made within
appointment when an appointment is valid. two (2) months immediately prior to the next presidential elections,
applies only to the President or Acting President.
Indeed, the power of choice is the heart of the power to appoint.
Appointment involves an exercise of discretion of whom to appoint; it The records reveal that when the petitioner brought the matter of
is not a ministerial act of issuing appointment papers to the appointee. recalling the appointments of the fourteen (14) private respondents
In other words, the choice of the appointee is a fundamental before the CSC, the only reason he cited to justify his action was that
component of the appointing power. these were "midnight appointments" that are forbidden under Article
VII, Section 15 of the Constitution.
An appointment can be made only to a vacant office. An appointment
cannot be made to an occupied office. The incumbent must first be In truth and in fact, there is no law that prohibits local elective officials
legally removed, or his appointment validly terminated, before one from making appointments during the last days of his or her tenure.
could be validly installed to succeed him. Petitioner certainly did not raise the issue of fraud on the part of the
outgoing mayor who made the appointments. Neither did he allege
c. De Castro vs. JBC, GR No. 191002, 20 April 2010; Relate with that the said appointments were tainted by irregularities or anomalies
Art. VIII, Sec. 4 (1), 3rd sentence. that breached laws and regulations governing appointments. His
FACTS: This is a Motion for Reconsideration on the March 17, 2010 solitary reason for recalling these appointments was that they were,
decision of the Court. The said decision directs the Judicial and Bar to his personal belief, "midnight appointments" which the outgoing
Council to resume its proceedings for the nomination of candidates to mayor had no authority to make.

M.R.A.D.C. LUMBRE 93
CONSTITUTIONAL LAW REVIEW

It has been held that upon the issuance of an appointment and the The Chairman and the Commissioners shall be appointed by the President
appointee's assumption of the position in the civil service, "he with the consent of the Commission on Appointments for a term of seven
acquires a legal right which cannot be taken away either by revocation years without reappointment. Of those first appointed, the Chairman shall
of the appointment or by removal except for cause and with previous hold office for seven years, a Commissioner for five years, and another
notice and hearing." Moreover, it is well-settled that the person Commissioner for three years, without reappointment. Appointment to
assuming a position in the civil service under a completed any vacancy shall be only for the unexpired term of the predecessor. In
appointment acquires a legal, not just an equitable, right to the no case shall any Member be appointed or designated in a temporary or
position. This right is protected not only by statute, but by the acting capacity. (Section 1(2), Article IX-B)
Constitution as well, which right cannot be taken away by either
revocation of the appointment, or by removal, unless there is valid The Chairman and the Commissioners shall be appointed by the President
cause to do so, provided that there is previous notice and hearing. with the consent of the Commission on Appointments for a term of seven
years without reappointment. Of those first appointed, three Members
Rule V, Section 9 of the Omnibus Implementing Regulations of the shall hold office for seven years, two Members for five years, and the last
Revised Administrative Code specifically provides that "an Members for three years, without reappointment. Appointment to any
appointment accepted by the appointee cannot be withdrawn or vacancy shall be only for the unexpired term of the predecessor. In no
revoked by the appointing authority and shall remain in force and in
case shall any Member be appointed or designated in a temporary or
effect until disapproved by the Commission."
acting capacity. (Section 1(2), Article IX-C)
Grounds for Recall of Appointments
The Chairman and the Commissioners shall be appointed by the President
Sec. 20. Notwithstanding the initial approval of an appointment, the with the consent of the Commission on Appointments for a term of seven
same may be recalled on any of the following grounds: years without reappointment. Of those first appointed, the Chairman shall
(a) Non-compliance with the procedures/criteria provided in the hold office for seven years, one Commissioner for five years, and the other
agency's Merit Promotion Plan; Commissioner for three years, without reappointment. Appointment to
any vacancy shall be only for the unexpired portion of the term of the
(b) Failure to pass through the agency's Selection/Promotion Board;
predecessor. In no case shall any Member be appointed or designated in
(c) Violation of the existing collective agreement between a temporary or acting capacity. (Section 1(2), Article IX-D)
management and employees relative to promotion; or
a. Funa v. COA Chair, GR No. 192791, 24 April 2012
(d) Violation of other existing civil service law, rules and regulations.
FACTS: On February 7, 2004, President Macapagal-Arroyo appointed
Accordingly, the appointments of the private respondents may only Reynaldo A. Villar (Villar) as the third member of the COA for a term
be recalled on the above-cited grounds. And yet, the only reason of seven (7) years starting February 2, 2004 until February 2, 2011.
advanced by the petitioner to justify the recall was that these were
"midnight appointments." Following the retirement of Carague (COA Chairman) on February 2,
2008 and during the fourth year of Villar as COA Commissioner, Villar
3. Losing candidate – Art. IX-B, Sec. 6 was designated as Acting Chairman of COA from February 4, 2008 to
Section 6. No candidate who has lost in any election, shall within one April 14, 2008. Subsequently, on April 18, 2008, Villar was nominated
and appointed as Chairman of the COA. Shortly thereafter, on June
year after such election, be appointed to any office in the Government or
any Government-owned or controlled corporations or in any of their 11, 2008, the Commission on Appointments confirmed his
appointment. He was to serve as Chairman of COA, as expressly
subsidiaries.
indicated in the appointment papers, until the expiration of the
4. Military – Art. XVI, Sec. 5 (4) original term of his office as COA Commissioner or on February 2,
2011.
Section 5. xxxx No member of the armed forces in the active service
shall, at any time, be appointed or designated in any capacity to a civilian Challenged in this recourse, Villar, in an obvious bid to lend color of
position in the Government, including government-owned or controlled title to his hold on the chairmanship, insists that his appointment as
corporations or any of their subsidiaries. COA Chairman accorded him a fresh term of seven (7) years which is
yet to lapse. He would argue, in fine, that his term of office, as such
5. Members of constitutional commissions – Art. IX (B), (C) and chairman, is up to February 2, 2015, or 7 years reckoned from
(D), Sec. 1(2) February 2, 2008 when he was appointed to that position.

M.R.A.D.C. LUMBRE 94
CONSTITUTIONAL LAW REVIEW

Although deemed moot due to the intervening appointment of appointment to any vacancy shall be only for the unexpired portion of
Chairman Tan and the resignation of Villar, the Court considered the the term of the predecessor.” In addition, such promotional
instant case as falling within the requirements for review of a moot appointment to the position of Chairman must conform to the
and academic case, since it asserts at least four exceptions to the rotational plan or the staggering of terms in the commission
mootness rule discussed in David, namely: there is a grave violation membership such that the aggregate of the service of the
of the Constitution; the case involves a situation of exceptional Commissioner in said position and the term to which he will be
character and is of paramount public interest; the constitutional issue appointed to the position of Chairman must not exceed seven years
raised requires the formulation of controlling principles to guide the so as not to disrupt the rotational system in the commission
bench, the bar and the public; and the case is capable of repetition prescribed by Sec. 1(2), Art. IX(D).
yet evading review.
In conclusion, there is nothing in Sec. 1(2), Article IX(D) that explicitly
ISSUE: Whether Villar’s appointment as COA Chairman, while sitting precludes a promotional appointment from Commissioner to
in that body and after having served for four (4) years of his seven Chairman, provided it is made under the aforestated circumstances
(7) year term as COA commissioner, is valid in light of the term or conditions.
limitations imposed under, and the circumscribing concepts tucked in,
Sec. 1 (2), Art. IX(D) of the Constitution. To sum up, the Court restates its ruling on Sec. 1(2), Art. IX(D) of
the Constitution, viz:
RULING: Sec. 1 (2), Art. IX(D) of the Constitution provides that:
1. The appointment of members of any of the three constitutional
(2) The Chairman and Commissioners [on Audit] shall be commissions, after the expiration of the uneven terms of office
appointed by the President with the consent of the Commission of the first set of commissioners, shall always be for a fixed term
on Appointments for a term of seven years without of seven (7) years; an appointment for a lesser period is void and
reappointment. Of those first appointed, the Chairman shall hold unconstitutional. The appointing authority cannot validly shorten
office for seven years, one commissioner for five years, and the the full term of seven (7) years in case of the expiration of the
other commissioner for three years, without reappointment. term as this will result in the distortion of the rotational system
Appointment to any vacancy shall be only for the unexpired prescribed by the Constitution.
portion of the term of the predecessor. In no case shall any
member be appointed or designated in a temporary or acting 2. Appointments to vacancies resulting from certain causes
capacity. (death, resignation, disability or impeachment) shall only be for
the unexpired portion of the term of the predecessor, but such
The Court finds petitioner’s position bereft of merit. The flaw lies in appointments cannot be less than the unexpired portion as this
regarding the word “reappointment” as, in context, embracing any will likewise disrupt the staggering of terms laid down under Sec.
and all species of appointment. The rule is that if a statute or 1(2), Art. IX(D).
constitutional provision is clear, plain and free from ambiguity, it must
be given its literal meaning and applied without attempted 3. Members of the Commission, e.g. COA, COMELEC or CSC, who
interpretation. were appointed for a full term of seven years and who served the
entire period, are barred from reappointment to any position in
The COA Chairman shall be appointed by the President for a term of the Commission. Corollarily, the first appointees in the
seven years, and if he has served the full term, then he can no longer Commission under the Constitution are also covered by the
be reappointed or extended another appointment. In the same vein, prohibition against reappointment.
a Commissioner who was appointed for a term of seven years who
likewise served the full term is barred from being reappointed. In 4. A commissioner who resigns after serving in the Commission
short, once the Chairman or Commissioner shall have served the full for less than seven years is eligible for an appointment to the
term of seven years, then he can no longer be reappointed to either position of Chairman for the unexpired portion of the term of the
the position of Chairman or Commissioner. The obvious intent of the departing chairman. Such appointment is not covered by the ban
framers is to prevent the president from “dominating” the on reappointment, provided that the aggregate period of the
Commission by allowing him to appoint an additional or two more length of service as commissioner and the unexpired period of
commissioners. the term of the predecessor will not exceed seven (7) years and
provided further that the vacancy in the position of Chairman
On the other hand, the provision, on its face, does not prohibit a resulted from death, resignation, disability or removal by
promotional appointment from commissioner to chairman as long as impeachment. The Court clarifies that “reappointment” found in
the commissioner has not served the full term of seven years, further Sec. 1(2), Art. IX(D) means a movement to one and the same
qualified by the third sentence of Sec. 1(2), Article IX (D) that “the office (Commissioner to Commissioner or Chairman to

M.R.A.D.C. LUMBRE 95
CONSTITUTIONAL LAW REVIEW

Chairman). On the other hand, an appointment involving a In fact, "the actual use to which the President puts the armed forces
movement to a different position or office (Commissioner to is xx x not subject to judicial review." (Lagman v. Executive
Chairman) would constitute a new appointment and, hence, not, Secretary)
in the strict legal sense, a reappointment barred under the
Constitution. a. Requisite?

5. Any member of the Commission cannot be appointed or b. David vs. Macapagal-Arroyo, GR No. 171396, 3 May
designated in a temporary or acting capacity. 2006

NOTE: Appointment is the act of instituting one in a previously The operative portion of PP 1017 may be divided into three
vacated position. Designation is the act of giving additional duties important provisions, thus:
to one who is already appointed and is currently holding a position. First provision:
A question with similar facts was asked in the Bar, only that the "By virtue of the power vested upon me by Section 18, Artilce VII
Commission involved was the COMELEC. However, the case of Funa … do hereby command the Armed Forces of the Philippines, to
still applies because it discusses appointments of members of maintain law and order throughout the Philippines, prevent or
constitutional commissions. suppress all forms of lawless violence as well any act of
vi. Limitation on the appointments extended by an Acting President – insurrection or rebellion."
Art. VII, Sec. 14 Citing Integrated Bar of the Philippines v. Zamora, the Court
Section 14. Appointments extended by an Acting President shall remain ruled that the only criterion for the exercise of the calling-out
effective, unless revoked by the elected President, within ninety days from his power is that "whenever it becomes necessary," the President
assumption or reassumption of office. may call the armed forces "to prevent or suppress lawless
violence, invasion or rebellion." Are these conditions present in
e. Military Powers or the Commander-in-Chief Clause– Art. VII, the instant cases? As stated earlier, considering the
Sec. 18 circumstances then prevailing, President Arroyo found it
necessary to issue PP 1017. Owing to her Office’s vast intelligence
Section 18. The President shall be the Commander-in-Chief of all armed network, she is in the best position to determine the actual
forces of the Philippines and whenever it becomes necessary, he may call condition of the country.
out such armed forces to prevent or suppress lawless violence, invasion
or rebellion. In case of invasion or rebellion, when the public safety Under the calling-out power, the President may summon the
requires it, he may, for a period not exceeding sixty days, suspend the armed forces to aid him in suppressing lawless violence, invasion
privilege of the writ of habeas corpus or place the Philippines or any part and rebellion. This involves ordinary police action. But every act
thereof under martial law. Within forty-eight hours from the proclamation that goes beyond the President’s calling-out power is considered
of martial law or the suspension of the privilege of the writ of habeas illegal or ultra vires.
corpus, the President shall submit a report in person or in writing to the
Congress. The Congress, voting jointly, by a vote of at least a majority of c. Kulayan vs. Tan, GR No. 187298, 3 July 2012
all its Members in regular or special session, may revoke such FACTS: Governor Tan issued Proclamation No. 1, Series of 2009,
proclamation or suspension, which revocation shall not be set aside by the declaring a state of emergency in the province of Sulu. The
President. Upon the initiative of the President, the Congress may, in the Proclamation cited the kidnapping incident as a ground for the
same manner, extend such proclamation or suspension for a period to be said declaration, describing it as a terrorist act pursuant to the
determined by the Congress, if the invasion or rebellion shall persist and Human Security Act (R.A. 9372). It also invoked Section 465 of
public safety requires it. the Local Government Code of 1991 (R.A. 7160), which bestows
1. Calling out powers on the Provincial Governor the power to carry out emergency
measures during man-made and natural disasters and calamities,
Among the three extraordinary powers, the calling out power is the and to call upon the appropriate national law enforcement
most benign and involves ordinary police action. The President may agencies to suppress disorder and lawless violence.
resort to this extraordinary power whenever it becomes necessary to
prevent or suppress lawless violence, invasion, or rebellion. "[T]he In the Proclamation, Tan called upon the PNP and the Civilian
power to call is fully discretionary to the President;" the only Emergency Force (CEF) to set up checkpoints and chokepoints,
limitations being that he acts within permissible constitutional conduct general search and seizures including arrests, and other
boundaries or in a manner not constituting grave abuse of discretion. actions necessary to ensure public safety. Petitioners, Jamar

M.R.A.D.C. LUMBRE 96
CONSTITUTIONAL LAW REVIEW

Kulayan, et al. claimed that Proclamation No. 1-09 was issued national in scope and civilian in character, to be administered and
ultra vires, and thus null and void, for violating Sections 1 and controlled by a national police commission. The authority of local
18, Article VII of the Constitution, which grants the President sole executives over the police units in their jurisdiction shall be
authority to exercise emergency powers and calling-out powers provided by law.” A local chief executive, such as the provincial
as the chief executive of the Republic and commander-in-chief of governor, exercises operational supervision over the police, and
the armed forces. may exercise control only in day-to-day operations. Furthermore
according to the framers, it is still the President who is authorized
ISSUE: Can a governor exercise the calling-out powers of a to exercise supervision and control over the police, through the
President? National Police Commission. In the discussions of the
RULING: NO. It has already been established that there is one Constitutional Commission regarding the above provision it is
repository of executive powers, and that is the President of the clear that the framers never intended for local chief executives to
Republic. This means that when Section 1, Article VII of the exercise unbridled control over the police in emergency
Constitution speaks of executive power, it is granted to the situations. This is without prejudice to their authority over police
President and no one else. Corollarily, it is only the President, as units in their jurisdiction as provided by law, and their prerogative
Executive, who is authorized to exercise emergency powers as to seek assistance from the police in day to day situations, as
provided under Section 23, Article VI, of the Constitution, as well contemplated by the Constitutional Commission. But as a civilian
as what became known as the calling-out powers under Section agency of the government, the police, through the NAPOLCOM,
7, Article VII thereof. Springing from the well-entrenched properly comes within, and is subject to, the exercise by the
constitutional precept of One President is the notion that there President of the power of executive control.
are certain acts which, by their very nature, may only be The calling-out powers contemplated under the Constitution is
performed by the president as the Head of the State. One of these exclusive to the President. An exercise by another official, even
acts or prerogatives is the bundle of Commander-in-Chief powers if he is the local chief executive, is ultra vires, and may not be
to which the “calling-out” powers constitutes a portion. The justified by the invocation of Section 465 of the Local Government
power to declare a state of martial law is subject to the Supreme Code.
Court’s authority to review the factual basis thereof. By
constitutional fiat, the calling-out powers, which is of lesser d. Subject to Judicial Review?
gravity than the power to declare martial law, is bestowed upon
the President alone. Yes, via petition for certiorari under Rule 65, in relation to Article
VIII, Section 1.
While the President is still a civilian, Article II, Section 3 of the
Constitution mandates that civilian authority is, at all times, IBP vs. Zamora, GR No. 141284, 15 August 2000
supreme over the military, making the civilian president the FACTS: In view of the alarming increase in violent crimes in
nation’s supreme military leader. The net effect of Article II, Metro Manila especially in Monumento Circle North Edsa, Araneta
Section 3, when read with Article VII, Section 18, is that a civilian Shopping Center, Greenhills, SM Megamall, and among others,
President is the ceremonial, legal and administrative head of the President Estrada, in a verbal directive, ordered the PNP and
armed forces. The Constitution does not require that the Marines to conduct joint visibility patrols for the purpose of crime
President must be possessed of military training and talents, but prevention and suppression. The Secretary of Defense, the Chief
as Commander-in-Chief, he has the power to direct military of Staff of AFP, the Chief of the PNP and the Secretary of Interior
operations and to determine military strategy. Normally, he and Local Government were tasked to implement and execute
would be expected to delegate the actual command of the armed the said order. In compliance, a Letter of Instruction was
forces to military experts; but the ultimate power is his. In the formulated detailing the manner by which the joint visibility
case of Integrated Bar of the Philippines v. Zamora, the Court patrols called TASK FORCE TULUNGAN would be conducted,
had occasion to rule that the calling-out powers belong solely to under the leadership of Police Chief of Metro Manila.
the President as commander-in-chief: When the President calls
the armed forces to prevent or suppress lawless violence, The President invoking his powers under Section 18, Article VII
invasion or rebellion, he necessarily exercises a discretionary of the Constitution, directed the AFP Chief and PNP Chief to
power solely vested in his wisdom. Regarding the country’s police coordinate with each other for the proper deployment and
force, utilization of the Marines to assist the PNP in crime prevention,
and that the President further expressed that such deployment
Section 6, Article XVI of the Constitution states that: “The State of Marines is merely temporary and for a reasonable period only,
shall establish and maintain one police force, which shall be until such time situation would improve.

M.R.A.D.C. LUMBRE 97
CONSTITUTIONAL LAW REVIEW

The IBP then filed an instant petition to annul LOI 02/2000 and substitute its own. However, the Court explains, this does not
to declare the deployment of Marines null and void and prevent an examination of whether such power to determine the
unconstitutional arguing that there is no emergency situation necessity of calling out the armed forces, it is incumbent upon
obtains in MM as would justify such deployment and a derogation the petitioner to show that the President’s decision is totally
of Article II, Section 3; that such deployment constitutes an bereft of factual basis. The present petition fails to discharge such
insidious incursion by the military in a civilian function of heavy burden as there is no evidence to support the assertion
government in derogation of Article XVI, Section 5 (4); and such that there exists no justification for calling out the armed forces.
deployment creates a dangerous tendency to rely on the military There is, likewise, no evidence to support the proposition that the
to perform civilian functions of the government. President committed grave abuse in such a manner as to violate
the constitutional provision on civilian supremacy over the
ISSUE: Whether the Presidents factual determination of the military.
necessity of calling the armed forces is subject to judicial review?
NOTE: The calling out power of the President is a political
RULING: The Presidents factual determination of the necessity question and is not subject to judicial review. Article VII, Section
of the Marines in crime prevention and suppression together with 18(3) does not give the Supreme Court the power to review the
PNP is not subject to judicial review because the same involves a exercise of calling out power but only the power to review
political question. whether or not there was grave abuse of discretion in exercising
The Court expressed that the power of judicial review is set forth the same.
in Sec. 1 Art. VIII of the Constitution of which vested in one When asked in the Bar if the Supreme Court has the power of
Supreme Court and in such lower courts as may be established judicial review, always answer in the affirmative. This is the
by law. That judicial power includes the duty of the courts of default answer, due to the expanded power of the Supreme Court
justice to settle actual controversies involving rights which are (i.e. to determine whether or not there has been grave abuse of
legally demandable and enforceable, and to determine whether discretion amounting to lack or excess of jurisdiction on the part
or not there has been a grave abuse of discretion amounting to of any branch or instrumentality of the Government).
lack or excess of jurisdiction on the part of any branch or
instrumentality of the Government. The Court further explains Thus, answer like this: “Judicial power not only includes the duty
that when questions of constitutional significance are raised, they of the courts of justice to settle actual controversies involving
can exercise its power of judicial review only if the following rights which are legally demandable and enforceable, but also
requisites are complied with, namely: (1) the existence of an to determine whether or not there has been a grave abuse of
actual and appropriate case; (2) a personal and substantial discretion amounting to lack or excess of jurisdiction on the part
interest of the party raising the constitutional question; (3) the of any branch or instrumentality of the Government.”
exercise of judicial review is pleaded at the earliest opportunity;
and (4) the constitutional question is the lis mota of the case. 2. Power to suspend the privilege of the writ of habeas corpus

A controversy, the Court explains, is justiciable if it refers to a a. Requisites?


matter which is appropriate for court review. It pertains to issues The extraordinary powers of suspending the privilege of the writ
which are inherently susceptible of being decided on ground of habeas corpus and/or declaring martial law may be exercised
recognized by law. As such, the Court does not automatically only when there is actual invasion or rebellion, and public safety
assume jurisdiction over actual constitutional cases brought requires it. The 1987 Constitution imposed the following limits in
before it, that one class of cases wherein the Court hesitates to the exercise of these powers: "(1) a time limit of sixty days; (2)
rule on are political questions. The reason is that political review and possible revocation by Congress; [and] (3) review
questions are concerned with issues dependent upon the wisdom, and possible nullification by the Supreme Court." (Lagman v.
not of legality, of a particular act or measure being assailed. Executive Secretary)
Moreover, the political question being a function of the separation
of powers, the courts will not normally interfere with the workings i. Rebellion as Ground, Quantum of Proof?
of co-equal branch unless the case shows a clear need for the
Lagman vs. Executive Secretary, G.R. No. 231658, 4
courts to step in to uphold the law and the Constitution. When
July 2017
the President calls the armed forces to prevent and suppress
lawless violence, invasion or rebellion, he necessarily exercises a RULING: In determining the existence of rebellion, the
discretionary power solely vested in his wisdom. The Court, thus, President only needs to convince himself that there is
cannot be called upon to overrule the Presidents wisdom or probable cause or evidence showing that more likely than

M.R.A.D.C. LUMBRE 98
CONSTITUTIONAL LAW REVIEW

not a rebellion was committed or is being committed. To iii. Art. 125 of the Revised Penal Code
require him to satisfy a higher standard of proof would
restrict the exercise of his emergency powers. Along this Art. 125. Delay in the delivery of detained persons to the proper
line, Justice Carpio, in his Dissent in Fortun v. President judicial authorities. — The penalties provided in the next
Macapagal-Arroyo, concluded that the President needs only preceding article shall be imposed upon the public officer or
to satisfy probable cause as the standard of proof in employee who shall detain any person for some legal ground and
determining the existence of either invasion or rebellion for shall fail to deliver such person to the proper judicial authorities
purposes of declaring martial law, and that probable cause within the period of; twelve (12) hours, for crimes or offenses
is the most reasonable, most practical and most expedient punishable by light penalties, or their equivalent; eighteen (18)
standard by which the President can fully ascertain the hours, for crimes or offenses punishable by correctional
existence or non-existence of rebellion necessary for a penalties, or their equivalent and thirty-six (36) hours, for
declaration of martial law or suspension of the writ. This is crimes, or offenses punishable by afflictive or capital penalties,
because unlike other standards of proof, which, in order to or their equivalent.
be met, would require much from the President and In every case, the person detained shall be informed of the cause
therefore unduly restrain his exercise of emergency powers, of his detention and shall be allowed upon his request, to
the requirement of probable cause is much simpler. It merely communicate and confer at any time with his attorney or counsel.
necessitates an "average man [to weigh] the facts and
circumstances without resorting to the calibration of the iv. Art. III, Sec. 13
rules of evidence of which he has no technical knowledge.
Section 13. All persons, except those charged with offenses
He [merely] relies on common sense [and] x x x needs only
punishable by reclusion perpetua when evidence of guilt is
to rest on evidence showing that, more likely than not, a
strong, shall, before conviction, be bailable by sufficient sureties,
crime has been committed x x x by the accused."
or be released on recognizance as may be provided by law. The
To summarize, the parameters for determining the right to bail shall not be impaired even when the privilege of the
sufficiency of factual basis are as follows: 1) actual rebellion writ of habeas corpus is suspended. Excessive bail shall not be
or invasion; 2) public safety requires it; the first two required.
requirements must concur; and 3) there is probable cause
c. Role of Congress – Congress’ Power to Revoke vs. Judicial
for the President to believe that there is actual rebellion or
Power to Review –
invasion.
i. Lagman vs. Executive Secretary, G.R. No. 231658, 4 July
b. Effects of the suspension of the privilege
2017
i. Art. VII, Sec. 18, pars. 5
RULING: According to the Supreme Court, the constitutional
Section 18. xxx The suspension of the privilege of the writ shall power of the President to suspend the privilege of the writ of
apply only to persons judicially charged for rebellion or offenses habeas corpus is not subject to judicial inquiry.
inherent in or directly connected with invasion.
The extraordinary powers of suspending the privilege of the writ
ii. Art. III, Sec. 2; Art. VII, Sec. 18, par. 6 of habeas corpus and/or declaring martial law may be exercised
only when there is actual invasion or rebellion, and public safety
Section 2. The right of the people to be secure in their persons, requires it. The 1987 Constitution imposed the following limits in
houses, papers, and effects against unreasonable searches and the exercise of these powers: "(1) a time limit of sixty days; (2)
seizures of whatever nature and for any purpose shall be review and possible revocation by Congress; [and] (3) review
inviolable, and no search warrant or warrant of arrest shall issue and possible nullification by the Supreme Court."
except upon probable cause to be determined personally by the
judge after examination under oath or affirmation of the The Court may strike down the presidential proclamation in an
complainant and the witnesses he may produce, and particularly appropriate proceeding filed by any citizen on the ground of lack
describing the place to be searched and the persons or things to of sufficient factual basis. On the other hand, Congress may
be seized. (Article III) revoke the proclamation or suspension, which revocation shall
not be set aside by the President.
Section 18. During the suspension of the privilege of the writ,
any person thus arrested or detained shall be judicially charged In reviewing the sufficiency of the factual basis of the
within three days, otherwise he shall be released. (Article VII) proclamation or suspension, the Court considers only the

M.R.A.D.C. LUMBRE 99
CONSTITUTIONAL LAW REVIEW

information and data available to the President prior to or at the Congress after his proclamation of martial law and/or suspension
time of the declaration; it is not allowed to "undertake an of the privilege of the writ of habeas corpus and grants the
independent investigation beyond the pleadings." On the other Congress the power to revoke, as well as extend, the
hand, Congress may take into consideration not only data proclamation and/or suspension; and vests upon the Judiciary
available prior to, but likewise events supervening the the power to review the sufficiency of the factual basis for such
declaration. Unlike the Court which does not look into the proclamation and/or suspension.
absolute correctness of the factual basis as will be discussed
below, Congress could probe deeper and further; it can delve into There are four provisions in Article VII, Section 18 of the 1987
the accuracy of the facts presented before it. Constitution specifically pertaining to the role of the Congress
when the President proclaims martial law and/or suspends the
The Court's review power is passive; it is only initiated by the privilege of the writ of habeas corpus, viz.:
filing of a petition "in an appropriate proceeding" by a citizen. On
the other hand, Congress' review mechanism is automatic in the a. Within forty-eight (48) hours from the proclamation of
sense that it may be activated by Congress itself at any time after martial law or the suspension of the privilege of the writ of
the proclamation or suspension was made. habeas corpus, the President shall submit a report in person
or in writing to the Congress;
The power to review by the Court and the power to revoke by
Congress are not only totally different but likewise independent b. The Congress, voting jointly, by a vote of at least a
from each other although concededly, they have the same majority of all its Members in regular or special session, may
trajectory, which is, the nullification of the presidential revoke such proclamation or suspension, which revocation
proclamation. Needless to say, the power of the Court to review shall not be set aside by the President;
can be exercised independently from the power of revocation of c. Upon the initiative of the President, the Congress may, in
Congress. the same manner, extend such proclamation or suspension
ii. Padilla vs. Congress of the Philippines, G.R. No. 231671, for a period to be determined by the Congress, if the invasion
25 July 2017 or rebellion shall persist; and

ISSUE: Whether or not the Congress has the mandatory duty to d. The Congress, if not in session, shall within twenty-four
convene jointly upon the President's proclamation of martial law hours (24) following such proclamation or suspension,
or the suspension of the privilege of the writ of habeas corpus convene in accordance with its rules without need of call.
under Article VII, Section 18 of the 1987 Constitution. It is the second provision that is under judicial scrutiny herein:
RULING: NO. The Congress is not constitutionally mandated to "The Congress, voting jointly, by a vote of at least a majority of
convene in joint session except to vote jointly to revoke the all its Members in regular or special session, may revoke such
President's proclamation of martial law and/or suspension of the proclamation or suspension, which revocation shall not be set
privilege of the writ of habeas corpus. aside by the President."

Outside explicit constitutional limitations, the Commander-in- It is worthy to stress that the provision does not actually refer to
Chief clause in Article VII, Section 18 of the 1987 Constitution a "joint session." While it may be conceded, subject to the
vests on the President, as Commander-in-Chief, absolute discussions below, that the phrase "voting jointly" shall already
authority over the persons and actions of the members of the be understood to mean that the joint voting will be done "in joint
armed forces, in recognition that the President, as Chief session," notwithstanding the absence of clear language in the
Executive, has the general responsibility to promote public peace, Constitution,72 still, the requirement that "[t]he Congress, voting
and as Commander-in-Chief, the more specific duty to prevent jointly, by a vote of at least a majority of all its Members in
and suppress rebellion and lawless violence. However, to regular or special session, x x x" explicitly applies only to the
safeguard against possible abuse by the President of the exercise situation when the Congress revokes the President's
of his power to proclaim martial law and/or suspend the privilege proclamation of martial law and/or suspension of the privilege of
of the writ of habeas corpus, the 1987 Constitution, through the the writ of habeas corpus. Simply put, the provision only requires
same provision, institutionalized checks and balances on the Congress to vote jointly on the revocation of the President's
President's power through the two other co-equal and proclamation and/or suspension.
independent branches of government, i.e., the Congress and the As the Court established in its preceding discussion, the clear
Judiciary. In particular, Article VII, Section 18 of the 1987 meaning of the relevant provision in Article VII, Section 18 of the
Constitution requires the President to submit a report to the

M.R.A.D.C. LUMBRE 100


CONSTITUTIONAL LAW REVIEW

1987 Constitution is that the Congress is only required to vote review on certiorari the decision, order, or ruling of the Commission
jointly on the revocation of the President's proclamation of on Elections and Commission on Audit can be found in Section 7,
martial law and/or suspension of the privilege of the writ of Article IX(A).
habeas corpus.
The Court may strike down the presidential proclamation in an
d. Mode of review –
appropriate proceeding filed by any citizen on the ground of lack of
Any action initiated by a citizen for the purpose of questioning the sufficient factual basis. On the other hand, Congress may revoke the
sufficiency if the factual basis for the exercise. It may be in the form proclamation or suspension, which revocation shall not be set aside
of a petition, complaint, or a matter to be decided by the Court. by the President.
(Lagman v. Executive Secretary)
In reviewing the sufficiency of the factual basis of the proclamation or
Lagman vs. Executive Secretary, G.R. No. 231658, 4 July 2017 suspension, the Court considers only the information and data
available to the President prior to or at the time of the declaration; it
ISSUE: Whether the petitions docketed as G.R. Nos. 231658, is not allowed to "undertake an independent investigation beyond the
231771, and 231774 are the "appropriate proceeding" covered by pleadings." On the other hand, Congress may take into consideration
Paragraph 3, Section 18, Article VII of the Constitution sufficient to not only data available prior to, but likewise events supervening the
invoke the mode of review required of this Court when a declaration declaration. Unlike the Court I which does not look into the absolute
of martial law or the suspension of the privilege of the writ of habeas correctness of the factual basis as will be discussed below, Congress
corpus is promulgated. could probe deeper and further; it can delve into the accuracy of the
facts presented before it.
RULING: It could not have been the intention of the framers of the
Constitution that the phrase "in an appropriate proceeding" would In addition, the Court's review power is passive; it is only initiated by
refer to a Petition for Certiorari pursuant to Section 1 or Section 5 of the filing of a petition "in an appropriate proceeding" by a citizen. On
Article VIII. The standard of review in a petition for certiorari is the other hand, Congress' review mechanism is automatic in the
whether the respondent has committed any grave abuse of discretion sense that it may be activated by Congress itself at any time after the
amounting to lack or excess of jurisdiction in the performance of his proclamation or suspension was made.
or her functions. Thus, it is not the proper tool to review the
sufficiency of the factual basis of the proclamation or suspension. Thus, the power to review by the Court and the power to revoke by
Congress are not only totally different but likewise independent from
It must be emphasized that under Section 18, Article VII, the Court each other although concededly, they have the same trajectory,
is tasked to review the sufficiency of the factual basis of the which is, the nullification of the presidential proclamation. Needless
President's exercise of emergency powers. Put differently, if this Court to say, the power of the Court to review can be exercised
applies the standard of review used in a petition for certiorari, the independently from the power of revocation of Congress.
same would emasculate its constitutional task under Section 18,
Article VII. The Court therefore held that the Court can simultaneously exercise
its power of review with, and independently from, the power to revoke
To conclude that the "appropriate proceeding" refers to a Petition for by Congress. Corollary, any perceived inaction or default on the part
Certiorari filed under the expanded jurisdiction of this Court would, of Congress does not deprive or deny the Court of its power to review.
therefore, contradict the clear intention of the framers of the
Constitution to place additional safeguards against possible martial e. Scope of SC’s Power of Review –
law abuse for, invariably, the third paragraph of Section 18, Article Lagman vs. Executive Secretary, G.R. No. 231658, 4 July 2017
VII would be subsumed under Section 1 of Article VIII. In other words,
the framers of the Constitution added the safeguard under the third RULING: The Court's power to review is limited to the determination
of whether the President in declaring martial law and suspending the
paragraph of Section 18, Article VII on top of the expanded
privilege of the writ of habeas corpus had sufficient factual basis.
jurisdiction of this Court. Thus, our review would be limited to an examination on whether the
President acted within the bounds set by the Constitution, i.e.,
The jurisdiction of this Court is not restricted to those enumerated in
whether the facts in his possession prior to and at the time of the
Sections 1 and 5 of Article VIII. For instance, its jurisdiction to be the
declaration or suspension are sufficient for him to declare martial law
sole judge of all contests relating to the election, returns, and or suspend the privilege of the writ of habeas corpus.
qualifications of the President or Vice-President can be found in the
last paragraph of Section 4, Article VII. The power of the Court to f. Ways to lift the Suspension

M.R.A.D.C. LUMBRE 101


CONSTITUTIONAL LAW REVIEW

1. Revocation by Congress via a Resolution; RULING: A state of martial law is peculiar because the President,
at such a time, exercises police power, which is normally a
2. The President himself; function of the Legislature. In particular, the President exercises
3. Nullification by the Court; police power, with the military's assistance, to ensure public
safety and in place of government agencies which for the time
4. By operation of law, after the lapse of 60 days from being are unable to cope with the condition in a locality, which
proclamation, and the President did not ask for an extension. remains under the control of the State.
3. Power to proclaim martial law In David v. President Macapagal-Arroyo, the Court, quoting
Justice Vicente V. Mendoza's (Justice Mendoza) Statement before
NOTE: The power to proclaim martial law is the broadest of the
the Senate Committee on Justice on March 13, 2006, stated that
military powers of the President, as he or she can suspend the
under a valid declaration of martial law, the President as
privilege of the writ of habeas corpus without proclaiming martial law,
Commander-in-Chief may order the "(a) arrests and seizures
but by proclaiming martial law, the President can likewise suspend
without judicial warrants; (b) ban on public assemblies; (c)
the privilege of the writ.
[takeover] of news media and agencies and press censorship;
a. Requisites and (d) issuance of Presidential Decrees x x x".

The extraordinary powers of suspending the privilege of the writ Worthy to note, however, that the above-cited acts that the
of habeas corpus and/or declaring martial law may be exercised President may perform do not give him unbridled discretion to
only when there is actual invasion or rebellion, and public safety infringe on the rights of civilians during martial law. This is
requires it. because martial law does not suspend the operation of the
Constitution, neither does it supplant the operation of civil courts
Even the recommendation of, or consultation with, the Secretary or legislative assemblies. Moreover, the guarantees under the Bill
of National Defense, or other high-ranking military officials, is not of Rights remain in place during its pendency. And in such
a condition for the President to declare martial law. A plain instance where the privilege of the writ of habeas corpus is also
reading of Section 18, Article VII of the Constitution shows that suspended, the suspension applies only to those judicially
the President's power to declare martial law is not subject to any charged with rebellion or offenses connected with invasion.
condition except for the requirements of actual invasion or
rebellion and that public safety requires it. Besides, it would be Clearly, from the foregoing, while martial law poses the most
contrary to common sense if the decision of the President is made severe threat to civil liberties, the Constitution has safeguards
dependent on the recommendation of his mere alter ego. Rightly against the President's prerogative to declare a state of martial
so, it is only on the President and no other that the exercise of law.
the powers of the Commander-in-Chief under Section 18, Article
xxx
VII of the Constitution is bestowed.
The powers to declare martial law and to suspend the privilege
i. Rebellion as Ground, Quantum of Proof?
of the writ of habeas corpus involve curtailment and suppression
Lagman vs. Executive Secretary, G.R. No. 231658, 4 of civil rights and individual freedom. Thus, the declaration of
July 2017 martial law serves as a warning to citizens that the Executive
Department has called upon the military to assist in the
RULING: To summarize, the parameters for determining the maintenance of law and order, and while the emergency remains,
sufficiency of factual basis are as follows: 1) actual rebellion the citizens must, under pain of arrest and punishment, not act
or invasion; 2) public safety requires it; the first two in a manner that will render it more difficult to restore order and
requirements must concur; and 3) there is probable cause enforce the law. As such, their exercise requires more stringent
for the President to believe that there is actual rebellion or safeguards by the Congress, and review by the Court.
invasion. (supra)
c. Limitations – Art. VII, Sec. 18, 4th par.
b. Effects of the proclamation / Power added to the
President – Section 18. xxx A state of martial law does not suspend the
operation of the Constitution, nor supplant the functioning of the
Lagman vs. Executive Secretary, G.R. No. 231658, 4 July civil courts or legislative assemblies, nor authorize the
2017 conferment of jurisdiction on military courts and agencies over

M.R.A.D.C. LUMBRE 102


CONSTITUTIONAL LAW REVIEW

civilians where civil courts are able to function, nor automatically constitutional organization of the Government of the
suspend the privilege of the writ. Philippines, the power and the duty of interpreting the laws
as when an individual should be considered to have violated
i. “Open Court” Doctrine – the law is primarily a function of the judiciary. It is not, and
Olaguer vs. Military Commission No. 34, 150 SCRA 144 it cannot be the function of the Executive Department,
through the military authorities. And as long as the civil
ISSUE: Whether or not military commissions or tribunals courts in the land remain open and are regularly functioning,
have the jurisdiction to try civilians for offenses allegedly as they do so today and as they did during the period of
committed during martial law when civil courts are open and martial law in the country, military tribunals cannot try and
functioning. exercise jurisdiction over civilians for offenses committed by
them and which are properly cognizable by the civil courts.
RULING: The Court also pronounced that due process of law
To have it otherwise would be a violation of the constitutional
demands that in all criminal prosecutions (where the accused
right to due process of the civilian concerned.
stands to lose either his life or his liberty), the accused shall
be entitled to, among others, a trial. The trial contemplated Finally, the Court deemed it proper to reiterate that as long
by the due process clause of the Constitution, in relation to as the civil courts in the land are open and functioning,
the Charter as a whole, is a trial by judicial process, not by military tribunals cannot try and exercise jurisdiction over
executive or military process. Military commissions or civilians for offenses committed by them. Whether or not
tribunals, by whatever name they are called, are not courts martial law has been proclaimed throughout the country or
within the Philippine judicial system. As explained by Justice over a part thereof is of no moment. The imprimatur for this
Teehankee in his separate dissenting opinion- observation is found in Section 18, Article VII of the 1987
Constitution, to wit —
... Civilians like (the) petitioner placed on trial for civil
offenses under general law are entitled to trial by judicial A state of martial law, does not suspend the operation of the
process, not by executive or military process. Constitution, nor supplant the functioning of the civil courts
or legislative assemblies, nor authorize the conferment of
Judicial power is vested by the Constitution exclusively in the
jurisdiction on military courts and agencies over civilians
Supreme Court and in such inferior courts as are duly
where civil courts are able to function, nor automatically
established by law. Judicial power exists only in the courts,
suspend the privilege of the writ.
which have "exclusive power to hear and determine those
matters which affect the life or liberty or property of a citizen. d. Role of Congress – Congress’ Power to Revoke vs.
Judicial Power to Review
Since we are not enemy-occupied territory nor are we under
a military government and even on the premise that martial i. Lagman vs. Executive Secretary, G.R. No.
law continues in force, the military tribunals cannot try and 231658, 4 July 2017
exercise jurisdiction over civilians for civil offenses
committed by them which are properly cognizable by the civil RULING: Considering the above discussion, the Court
courts that have remained open and have been regularly finds it imperative to re-examine, reconsider, and set
functioning. aside its pronouncement in Fortun v. President
Macapagal-Arroyo to the effect that:
And in Toth v. Quarles, the U.S. Supreme Court further
stressed that the assertion of military authority over civilians Consequently, although the Constitution reserves to the
cannot rest on the President's power as Commander-in-Chief Supreme Court the power to review the sufficiency of
or on any theory of martial law. the factual basis of the proclamation or suspension in a
proper suit, it is implicit that the Court must allow
Moreover, military tribunals pertain to the Executive Congress to exercise its own review powers, which is
Department of the Government and are simply automatic rather than initiated. Only when Congress
instrumentalities of the executive power, provided by the defaults in its express duty to defend the Constitution
legislature for the President as Commander-in-Chief to aid through such review should the Supreme Court step in
him in properly commanding the army and navy and as its final rampart. The constitutional validity of the
enforcing discipline therein, and utilized under his orders or President's proclamation of martial law or suspension of
those of his authorized military representatives. Following the writ of habeas corpus is first a political question in
the principle of separation of powers underlying the existing

M.R.A.D.C. LUMBRE 103


CONSTITUTIONAL LAW REVIEW

the hands of Congress before it becomes a justiciable Constitution, still, the requirement that "[t]he Congress,
one in the hands of the Court. voting jointly, by a vote of at least a majority of all its
Members in regular or special session, x x x" explicitly
xx xx applies only to the situation when the Congress revokes
If the Congress procrastinates or altogether fails to fulfill the President's proclamation of martial law and/or
its duty respecting the proclamation or suspension suspension of the privilege of the writ of habeas corpus.
within the short time expected of it, then the Court can Simply put, the provision only requires Congress to vote
step in, hear the petitions challenging the President's jointly on the revocation of the President's proclamation
action, and ascertain if it has a factual basis. x x x and/or suspension.

By the above pronouncement, the Court willingly but Hence, the plain language of the subject constitutional
unwittingly clipped its own power and surrendered the provision does not support the petitioners' argument
same to Congress as well as: abdicated from its that it is obligatory for the Congress to convene in joint
bounden duty to review. Worse, the Court considered' session following the President's proclamation of martial
itself just on stand-by, waiting and willing to act as a law and/or suspension of the privilege of the writ of
substitute in case Congress "defaults." It is an habeas corpus, under all circumstances.
aberration, a stray declaration, which must be rectified e. Mode of Review –
and set aside in this proceeding.
Lagman vs. Executive Secretary, G.R. No. 231658, 4
We, therefore, hold that the Court can simultaneously July 2017
exercise its power of review with, and independently
from, the power to revoke by Congress. Corollary, any RULING: A proceeding "[i]n its general acceptation, [is] the
perceived inaction or default on the part of Congress form in which actions are to be brought and defended, the
does not deprive or deny the Court of its power to manner of intervening in suits, of conducting them, the mode
review. of deciding them, of opposing judgments, and of executing."
In fine, the phrase "in an appropriate proceeding" appearing
ii. Padilla vs. Congress of the Philippines, G.R. No. on the third paragraph of Section 18, Article VII refers to any
231671, 25 July 2017 action initiated by a citizen for the purpose of questioning
RULING: The provision in question is clear, plain, and the sufficiency of the factual basis of the exercise of the Chief
unambiguous. In its literal and ordinary meaning, the Executive's emergency powers, as in these cases. It could be
provision grants the Congress the power to revoke the denominated as a complaint, a petition, or a matter to be
President's proclamation of martial law or the resolved by the Court.
suspension of the privilege of the writ of habeas corpus f. Scope of SC’s Power of Review –
and prescribes how the Congress may exercise such
power, i.e., by a vote of at least a majority of all its Lagman vs. Executive Secretary, G.R. No. 231658, 4
Members, voting jointly, in a regular or special session. July 2017
The use of the word "may" in the provision - such that
"[t]he Congress x x x may revoke such proclamation or RULING: The calling out power is in a different category
suspension x x x" - is to be construed as permissive and from the power to declare martial law and the power to
operating to confer discretion on the Congress on suspend the privilege of the writ of habeas corpus;
whether or not to revoke, but in order to revoke, the nullification of Proclamation No. 216 will not affect
same provision sets the requirement that at least a Proclamation No. 55.
majority of the Members of the Congress, voting jointly, The Court's ruling in these cases will not, in any way, affect
favor revocation. the President's declaration of a state of national emergency
It is worthy to stress that the provision does not actually on account of lawless violence in Mindanao through
refer to a "joint session." While it may be conceded, Proclamation No. 55 dated September 4, 2016, where he
subject to the discussions below, that the phrase "voting called upon the Armed Forces and the Philippine National
jointly" shall already be understood to mean that the Police (PNP) to undertake such measures to suppress any
joint voting will be done "in joint session," and all forms of lawless violence in the Mindanao region, and
notwithstanding the absence of clear language in the

M.R.A.D.C. LUMBRE 104


CONSTITUTIONAL LAW REVIEW

to prevent such lawless violence from spreading and ISSUE: Whether the State sufficiently established the existence
escalating elsewhere in the Philippines. of conspiracy among GMA, Aguas, and Uriarte.

In Kulayan v. Tan, the Court ruled that the President's calling RULING: No. The Prosecution's effort to show irregularities as
out power is in a different category from the power to badges of bad faith has led it to claim that GMA had known that
suspend the privilege of the writ of habeas corpus and the Uriarte would raid the public treasury, and would misuse the
power to declare martial law: amounts disbursed. This knowledge was imputed to GMA by
virtue of her power of control over PCSO.
x x x Congress may revoke such proclamation or suspension
and the Court may review the sufficiency of the factual basis The Prosecution seems to be relying on the doctrine of command
thereof. However, there is no such equivalent provision responsibility to impute the actions of subordinate officers to GMA
dealing with the revocation or review of the President's as the superior officer. The reliance is misplaced, for incriminating
action to call out the armed forces. The distinction places the GMA under those terms was legally unacceptable and
calling out power in a different category from the power to incomprehensible. The application of the doctrine of command
declare martial law and the power to suspend the privilege responsibility is limited, and cannot be true for all litigations. The
of the writ of habeas corpus, otherwise, the framers of the Court ruled in Rodriguez v. Macapagal-Arroyo that command
Constitution would have simply lumped together the three responsibility pertains to the responsibility of commanders for
powers and provided for their revocation and review without crimes committed by subordinate members of the armed forces
any qualification. or other persons subject to their control in international wars or
domestic conflict. The doctrine has also found application in civil
In other words, the President may exercise the power to call actions for human rights abuses. But this case involves neither a
out the Armed Forces independently of the power to suspend probe of GMA's actions as the Commander-in-Chief of the Armed
the privilege of the writ of habeas corpus and to declare Forces of the Philippines, nor of a human rights issue. As such, it
martial law, although, of course, it may also be a prelude to is legally improper to impute the actions of Uriarte to GMA in the
a possible future exercise of the latter powers, as in this absence of any conspiracy between them.
case.
f. Delegated: Emergency powers – Art. VI, Sec. 23 (2)
Even so, the Court's review of the President's declaration of
martial law and his calling out the Armed Forces necessarily Section 23. xxx In times of war or other national emergency, the Congress
entails separate proceedings instituted for that particular may, by law, authorize the President, for a limited period and subject to such
purpose. restrictions as it may prescribe, to exercise powers necessary and proper to
carry out a declared national policy. Unless sooner withdrawn by resolution of
As explained in Integrated Bar of the Philippines v. Zamora, the Congress, such powers shall cease upon the next adjournment thereof.
the President's exercise of his power to call out the armed
forces to prevent or suppress lawless violence, invasion or i. Conditions of granting emergency powers
rebellion may only be examined by the Court as to whether
such power was exercised within permissible constitutional 1. There must be war or other national emergency;
limits or in a manner constituting grave abuse of discretion. 2. The Congress, by law, authorized the President to exercise emergency
g. Ways to lift the proclamation powers;

1. Revocation by Congress via a Resolution; 3. The exercise is for a limited period and subject to such limitations as it
may prescribe;
2. The President himself;
4. Such exercise is necessary and proper to carry out a declared national
3. Nullification by the Court; policy (theaters of war).

4. By operation of law, after the lapse of 60 days from ii. Relate with Art. XII, Sec. 17 (See also Art. XVI, Sec. 5(7)
proclamation, and the President did not ask for an extension.
Section 17. In times of national emergency, when the public interest so
4. Command Responsibility requires, the State may, during the emergency and under reasonable
terms prescribed by it, temporarily take over or direct the operation of any
a. Macapagal-Arroyo vs. People, G.R. No. 220598, 19 July privately-owned public utility or business affected with public interest.
2016 (Article XII)

M.R.A.D.C. LUMBRE 105


CONSTITUTIONAL LAW REVIEW

Section 5. xxxx The tour of duty of the Chief of Staff of the armed forces Petitioner then requested that she be restored to her former post as
shall not exceed three years. However, in times of war or other national assistant city treasurer since the same was still vacant, she also asked for
emergency declared by the Congress, the President may extend such tour the backpay for the entire period of her suspension.
of duty.
ISSUE: Whether a public officer who was convicted by final judgment and
g. Executive clemencies – Art. VII, Sec. 19 subsequently granted an absolute pardon, may be entitled to
reinstatement to her former position without need of a new appointment.
Section 19. Except in cases of impeachment, or as otherwise provided in this
Constitution, the President may grant reprieves, commutations, and pardons, RULING: No. A pardon reaches both the punishment prescribed for the
and remit fines and forfeitures, after conviction by final judgment. offense and the guilt of the offender; and when the pardon is full, it
releases the punishment and blots out of existence the guilt, so that in the
He shall also have the power to grant amnesty with the concurrence of a eye of the law the offender is as innocent as if he had never committed
majority of all the Members of the Congress. the offense. If granted before conviction, it prevents any of the penalties
i. Limitations: See also Art. IX-C, Sec. 5 and disabilities, consequent upon conviction, from attaching; if granted
after conviction, it removes the penalties and disabilities and restores him
Section 5. No pardon, amnesty, parole, or suspension of sentence for to all his civil rights; it makes him, as it were, a new man, and gives him
violation of election laws, rules, and regulations shall be granted by the a new credit and capacity. But unless expressly grounded on the person’s
President without the favorable recommendation of the Commission. innocence (which is rare), it cannot bring back lost reputation for honesty,
integrity and fair dealing.
ii. Pardon vs. Amnesty
A pardon looks to the future. It is not retrospective. It makes no amends
PARDON AMNESTY for the past. It affords no relief for what has been suffered by the offender.
It is a personal act of the It is a public act which requires It does not impose upon the government any obligation to make
President. As such, no the concurrence of Senate. reparation for what has been suffered.
concurrence by any public officer
To insist on automatic reinstatement because of a mistaken notion that
is needed.
the pardon virtually acquitted one from the offense of estafa would be
It is given after conviction by It may be given before trial. grossly untenable. A pardon, albeit full and plenary, cannot preclude the
final judgment. appointing power from refusing appointment to anyone deemed to be of
bad character, a poor moral risk, or who is unsuitable by reason of the
It is given to a single person. It is usually given to a class of
pardoned conviction.
persons.
The crimes subject of the grant Crimes involved are generally The absolute disqualification or ineligibility from public office forms part of
may be common or political political crimes. the punishment prescribed by the Revised Penal Code for estafa thru
crimes. falsification of public documents.

It is prospective in nature; thus, It is retroactive in nature; thus, The pardon granted to petitioner has resulted in removing her
it affords no relief for what has it is as if the offender never disqualification from holding public employment but it cannot go beyond
been suffered by the offender committed any crime and all that. To regain her former post as assistant city treasurer, she must re-
prior to the grant. effects of the crime are erased. apply and undergo the usual procedure required for a new appointment.
It requires acceptance by the It does not require acceptance. - Garcia vs. Chairman, GR No. 75025, 14 September 1993
grantee.
FACTS: Petitioner Vicente Garcia was employed as a supervising lineman
- Monsanto vs. Factoran, 170 SCRA 190 at the Bureau of Telecommunications. He was accused of stealing some
materials in their company. Thus, public respondents filed a criminal case
FACTS: The Sandiganbayan convicted petitioner Salvacion A. Monsanto against him for qualified theft before a court and on the same ground
of the crime of estafa thru falsification of public documents and sentenced respondents also filed an administrative case in which petitioner was found
her to imprisonment and to indemnify the government in the sum of guilty and was later dismissed from the service. With respect to the
P4,892.50 representing the balance of the amount defrauded and to pay criminal offense, petitioner was acquitted by the court due to insufficiency
the costs proportionately. She was given an absolute pardon by President of evidence. Petitioner was then reinstated from his work and is now
Marcos which she accepted. claiming before the COA for his back salaries from the time of his dismissal
up to present. But COA on the other hand reluctantly denied his pleadings.

M.R.A.D.C. LUMBRE 106


CONSTITUTIONAL LAW REVIEW

Meanwhile, petitioner was extended an executive clemency (absolute conditional pardon upon the judgment of the power that has granted it,
pardon) by the President. Still, respondent COA strongly refused to give cannot invoke the aid of the courts, however erroneous the findings may
due course to petitioners claim. be upon which his recommitment was ordered.
ISSUE: Whether or not respondent is entitled to the payment of back It matters not that in the case of Torres, he has allegedly been acquitted
wages after having been reinstated pursuant to the grant of executive in two of the three criminal cases filed against him subsequent to his
clemency. conditional pardon, and that the third case remains pending for thirteen
(13) years in apparent violation of his right to a speedy trial.
HELD: The Court ruled initially by explaining the mandate of Sec 19 Article
VII of the Constitution and further articulates that the bestowal of Habeas corpus lies only where the restraint of a person's liberty has been
executive clemency on petitioner in effect completely obliterated the judicially adjudged as illegal or unlawful. In the instant petition, the
adverse effects of the administrative decision which found him guilty of incarceration of Torres remains legal considering that, were it not for the
dishonesty and ordered his separation from the service. This can be grant of conditional pardon which had been revoked because of a breach
inferred from the executive clemency itself exculpating petitioner from the thereof, the determination of which is beyond judicial scrutiny, he would
administrative charge and thereby directing his reinstatement, which is have served his final sentence for his first conviction until November 2,
rendered automatic by the grant of the pardon. This signifies that 2000.
petitioner need no longer apply to be reinstated to his former
employment; he is restored to his office ipso facto upon the issuance of Ultimately, solely vested in the Chief Executive, who in the first place was
the clemency. the exclusive author of the conditional pardon and of its revocation, is the
corrollary prerogative to reinstate the pardon if in his own judgment, the
NOTE: The difference between the Monsanto and Garcia cases is that in acquittal of the pardonee from the subsequent charges filed against him,
Monsanto, the latter was found guilty of estafa. The pardon granted by warrants the same. Courts have no authority to interfere with the grant
the President was merely an act of grace or kindness. Thus, Monsanto had by the President of a pardon to a convicted criminal. It has been our
to secure a new appointment. In the Garcia case, the latter, although fortified ruling that a final judicial pronouncement as to the guilt of a
administratively held liable, was subsequently acquitted by the trial court. pardonee is not a requirement for the President to determine whether or
In this case the pardon by the President, as head of the administrative not there has been a breach of the terms of a conditional pardon. There is
department, obliterated the administrative liability of Garcia. Hence, the likewise nil a basis for the courts to effectuate the reinstatement of a
latter may be reinstated without need of a new appointment. conditional pardon revoked by the President in the exercise of powers
undisputedly solely and absolutely lodged in his office.
- In Re: Wilfredo Sumulong Torres, 251 SCRA 709, 29 December
1995 h. The diplomatic / treaty-making power – Art. VII, Sec. 21; Art. XVIII,
Sec. 25 (Will be thoroughly discussed in Public International Law)
ISSUE: Whether the President needs to wait for a judicial pronouncement
of guilt of a subsequent crime or for conviction by final judgment in order No treaty or international agreement shall be valid and effective unless
to effectuate the recommitment of the pardonee to prison. concurred in by at least two-thirds of all the Members of the Senate. (Art. VII,
Sec. 21)
RULING: NO. A conditional pardon is in the nature of a contract between
the sovereign power or the Chief Executive and the convicted criminal to After the expiration in 1991 of the Agreement between the Republic of the
the effect that the former will release the latter subject to the condition Philippines and the United States of America concerning military bases, foreign
that if he does not comply with the terms of the pardon, he will be military bases, troops, or facilities shall not be allowed in the Philippines except
recommitted to prison to serve the unexpired portion of the sentence or under a treaty duly concurred in by the Senate and, when the Congress so
an additional one. By the pardonee's consent to the terms stipulated in requires, ratified by a majority of the votes cast by the people in a national
this contract, the pardonee has thereby placed himself under the referendum held for that purpose, and recognized as a treaty by the other
supervision of the Chief Executive or his delegate who is duty-bound to contracting State. (Art. XVIII, Sec. 25)
see to it that the pardonee complies with the terms and conditions of the
pardon. Under Section 64 (i) of the Revised Administrative Code, the Chief i. Power of impoundment
Executive is authorized to order "the arrest and re-incarceration of any The refusal by the President for whatever reason to spend funds made
such person who, in his judgment, shall fail to comply with the condition, available by Congress. It is the failure to spend or obligate budget authority of
or conditions of his pardon, parole, or suspension of sentence." It is now any type (Philconsa v. Enriquez, supra) This power of the President is
a well-entrenched rule in this jurisdiction that this exercise of presidential derived from Sec. 38 of the Administrative Code of 1987 on suspension.
judgment is beyond judicial scrutiny. The determination of the violation of
the conditional pardon rests exclusively in the sound judgment of the Chief i. PHILCONSA vs. Enriquez, 235 SCRA 506, 9 August 1994
Executive, and the pardonee, having consented to place his liberty on

M.R.A.D.C. LUMBRE 107


CONSTITUTIONAL LAW REVIEW

FACTS: House Bill No. 10900, the General Appropriation Bill of 1994 (GAB for whatever reason, to spend funds made available by Congress. It is the
of 1994), was passed by Congress on December 17, 1993. It imposed failure to spend or obligate budget authority of any type.
conditions and limitations on certain items of appropriations in the
proposed budget submitted by the President. It also authorized members Those who deny to the President the power to impound argue that once
of Congress to propose and identify projects in the "pork barrels" allotted Congress has set aside the fund for a specific purpose in an appropriations
to them and to realign their respective operating budgets. act, it becomes mandatory on the part of the President to implement the
project and to spend the money appropriated therefor. The President has
The President signed the bill into law, and declared the same to have no discretion on the matter, for the Constitution imposes on him the duty
become Republic Act No. 7663. On the same day, the President delivered to faithfully execute the laws.
his Presidential Veto Message, specifying the provisions of the bill he
vetoed and on which he imposed certain conditions. No step was taken in In refusing or deferring the implementation of an appropriation item, the
either House of Congress to override the vetoes. President in effect exercises a veto power that is not expressly granted by
the Constitution. As a matter of fact, the Constitution does not say anything
In G.R. No. 11388, Senators Tañada and Romulo sought the issuance of about impounding. The source of the Executive authority must be found
the writs of prohibition and mandamus and contest the constitutionality elsewhere.
of: (1) the veto on four special provision added to items in the GAA of
1994 for the Armed Forces of the Philippines (AFP) and the Department of Proponents of impoundment have invoked at least three principal sources
Public Works and Highways (DPWH); and (2) the conditions imposed by of the authority of the President. Foremost is the authority to impound
the President in the implementation of certain appropriations for the given to him either expressly or impliedly by Congress. Second is the
CAFGU's, the DPWH, and the National Housing Authority (NHA). executive power drawn from the President's role as Commander-in-Chief.
Third is the Faithful Execution Clause which ironically is the same provision
RULING: Congress appropriated compensation for the CAFGU's, including invoked by petitioners herein.
the payment of separation benefits but it added the following Special
Provision: The proponents insist that a faithful execution of the laws requires that the
President desist from implementing the law if doing so would prejudice
1. CAFGU Compensation and Separation Benefit. The appropriation public interest. An example given is when through efficient and prudent
authorized herein shall be used for the compensation of CAFGU's management of a project, substantial savings are made. In such a case, it
including the payment of their separation benefit not exceeding one is sheer folly to expect the President to spend the entire amount budgeted
(1) year subsistence allowance for the 11,000 members who will be in the law.
deactivated in 1994. The Chief of Staff, AFP, shall, subject to the
approval of the Secretary of National Defense, promulgate policies We do not find anything in the language used in the challenged Special
and procedures for the payment of separation benefit (GAA of 1994, Provision that would imply that Congress intended to deny to the President
p. 740). the right to defer or reduce the spending, much less to deactivate 11,000
CAFGU members all at once in 1994. But even if such is the intention, the
The President declared in his Veto Message that the implementation of this appropriation law is not the proper vehicle for such purpose. Such intention
Special Provision to the item on the CAFGU's shall be subject to prior must be embodied and manifested in another law considering that it
Presidential approval pursuant to P.D. No. 1597 and R.A. No. 6758. He gave abrades the powers of the Commander-in-Chief and there are existing laws
the following reasons for imposing the condition: on the creation of the CAFGU's to be amended. Again we state: a provision
in an appropriations act cannot be used to repeal or amend other laws, in
Petitioners complain that the directive of the President was tantamount to this case, P.D. No. 1597 and R.A. No. 6758.
an administrative embargo of the congressional will to implement the
Constitution's command to dissolve the CAFGU's. They argue that the NOTE: The power of impoundment is a very special principle. When
President cannot impair or withhold expenditures authorized and Congress passes the General Appropriations Act, it is imperative that all
appropriated by Congress when neither the Appropriations Act nor other persons involved follow the provisions of the said Act. All the money or
legislation authorize such impounding (Rollo, G.R. No. 113888, pp. 15-16). amount appropriated for the projects should be spent. In impoundment,
even if the Congress states that the President should spend this much for
The Solicitor General contends that it is the President, as Commander-in- a certain project, the President can disregard the provision of the law
Chief of the Armed Forces of the Philippines, who should determine when enacted by Congress. The Constitution granted the President the power to
the services of the CAFGU's are no longer needed. ensure that laws are faithfully executed. Thus, if the President is of the
This is the first case before this Court where the power of the President to opinion that a provision of law, if ever implemented, will violate
impound is put in issue. Impoundment refers to a refusal by the President, constitutional mandate or rights, then he can disregard that provision.
j. Power of augmentation - Article VI, Section 25(5)

M.R.A.D.C. LUMBRE 108


CONSTITUTIONAL LAW REVIEW

Section 25. No law shall be passed authorizing any transfer of appropriations; package intended to fast-track public spending and to push economic
however, the President, the President of the Senate, the Speaker of the House growth by investing on high-impact budgetary programs, activities and
of Representatives, the Chief Justice of the Supreme Court, and the heads of projects (PAPs) to be funded from the savings generated during the year
Constitutional Commissions may, by law, be authorized to augment any item as well as from unprogrammed funds. In that respect, the DAP was meant
in the general appropriations law for their respective offices from savings in to stimulate the economy by way of accelerated spending. The
other items of their respective appropriations. Administration would thereby accelerate government spending by:
streamlining the implementation process through the clustering of
i. Araullo vs. Aquino, G.R. No. 209287, 3 February 2015 infrastructure projects of the Department Of Public Works and Highways
FACTS: This case is resolution on the Motion for Reconsideration filed by (DPWH) and the Department of Education (DepEd), and (2) frontloading
the respondents, and the Motion for Partial Reconsideration filed by the PPP-related projects due for implementation in the following year.
petitioners, on the decision rendered on the same case on July 1, 2014, Basically, the DAP was to be implemented and funded by declaring savings
declaring Disbursement Acceleration Program (DAP) as unconstitutional, coming from the various departments and agencies derived from pooling
on the following grounds: unobligated allotments and withdrawing unreleased appropriations;
• the creation of savings prior to the end of the fiscal year and the releasing unprogrammed funds; and applying the savings and
withdrawal of these funds for implementing agencies; unprogrammed funds to augment existing PAPs or to support other priority
PAPs. The Government, by spending on public infrastructure, would signify
• the cross-border transfers of the savings from one branch of its commitment of ensuring profitability for prospective investors. The
government to another; PAPs funded under the DAP were chosen for this reason based on their:
(1) multiplier impact on the economy and infrastructure development; (2)
• the allotment of funds for projects, activities, and programs not
beneficial effect on the poor; and (3) translation into disbursements.
outlined in the General Appropriations Act;
In a decision dated July 1, 2014, DAP was declared unconstitutional. On a
On September 25, 2013, Sen. Jinggoy Ejercito Estrada delivered a
Motion for Reconsideration, respondents assail the pronouncement of
privilege speech in the Senate of the Philippines to reveal that some
unconstitutionality of cross-border transfers made by the President. They
Senators, including himself, had been allotted an additional P50 Million
submit that Section 25(5), Article VI of the Constitution prohibits only the
each as incentive for voting in favor of the impeachment of Chief Justice
transfer of appropriation, not savings. They relate that cross-border
Renato C. Corona. Responding to Sen. Estrada's revelation, Secretary
transfers have been the practice in the past, being consistent with the
Abad of the DBM issued a public statement explaining that the funds
President’s role as the Chief Executive.
released to the Senators had been part of the DAP, a program designed
by the DBM to ramp up spending to accelerate economic expansion. Abad ISSUE: Whether or not the president has authority to transfer savings to
clarified that the funds had been released to the Senators based on their other departments pursuant to his constitutional powers.
letters of request for funding; and that it was not the first time that
releases from the DAP had been made because the DAP had already been RULING: No. The president has no authority to transfer savings to other
instituted in 2011 to ramp up spending after sluggish disbursements had departments pursuant to his constitutional powers, as cross-border
caused the growth of the gross domestic product (GDP) to slow down. transfers are constitutionally impermissible.

The revelation also prompted Maria Carolina Araullo, Chairperson of the In view of the clarity of the text of Section 25(5), the Court stands by its
Bagong Alyansang Makabayan, and several other concerned citizens to file pronouncement, and will not brook any strained interpretations. The
various petitions with the Supreme Court questioning the validity of the transfer of appropriated funds, to be valid under Section 25(5), must be
DAP. Among their contentions was: DAP is unconstitutional because it made upon a concurrence of the following requisites, namely:
violates the constitutional rule which provides that “no money shall be paid (1) There is a law authorizing the President, the President of the
out of the Treasury except in pursuance of an appropriation made by law.” Senate, the Speaker of the House of Representatives, the Chief
But Secretary Abad argued that the DAP is based on certain laws Justice of the Supreme Court, and the heads of the Constitutional
particularly the General Appropriations Act (GAA) (savings and Commissions to transfer funds within their respective offices;
augmentation provisions thereof), Sec. 25(5), Art. VI of the Constitution
(power of the President to augment), Secs. 38 and 49 of Executive Order (2) The funds to be transferred are savings generated from the
292 (power of the President to suspend expenditures and authority to use appropriations for their respective offices; and
savings, respectively).
(3) The purpose of the transfer is to augment an item in the general
The economic situation prevailing in the middle of 2011 thus paved the law for their respective offices.
way for the development and implementation of the DAP as a stimulus

M.R.A.D.C. LUMBRE 109


CONSTITUTIONAL LAW REVIEW

As to the first requisite: The General Appropriation Act (GAA) of 2011 programmed appropriation in this Act free from any obligation or
and 2012 lacked valid provisions to authorize transfers of funds under the encumbrance." But the first part of the definition was further qualified by
DAP; hence transfers under the DAP were unconstitutional. And although the three enumerated instances of when savings would be realized. As
the GAA of 2013 had provision for such transfer, it however lacked other such, unobligated allotments could not be indiscriminately declared as
the requisites. savings without first determining whether any of the three instances
existed. The respondents rely on the provision of the Administrative Code
As to the second requisite: There were no savings from which funds of 1987 to justify the withdrawal of unobligated allotments. But the
could be sourced for the DAP. The funds used in the DAP - the unreleased provision authorized only the suspension or stoppage of further
appropriations and withdrawn unobligated allotments -were not actual expenditures, not the withdrawal of unobligated allotments.
savings within the context of Section 25(5), supra, and the relevant
provisions of the GAAs. Savings should be understood to refer to the The power to augment cannot be used to fund non-existent
excess money after the items that needed to be funded have been funded, provisions in the GAAs
or those that needed to be paid have been paid pursuant to the budget.
There could be savings only when the PAPs for which the funds had been The Court finds merit in the respondents’ argument that the Court has
appropriated were actually implemented and completed, or finally erroneously invalidated all the DAP-funded projects by overlooking the
discontinued or abandoned. Savings could not be realized with certainty difference between an item and an allotment class, and by concluding that
in the middle of the fiscal year; the funds for slow-moving PAPs could not they do not have appropriation cover.
be considered as savings because such PAPs had not actually been Section 25(5) of the 1987 Constitution mentions of the term item that may
abandoned or discontinued yet. be the object of augmentation by the President, the Senate President, the
The Supreme Court also discussed that there is no executive impoundment Speaker of the House, the Chief Justice, and the heads of the
in the DAP. Impoundment of funds refers to the President’s power to Constitutional Commissions. An item that is the distinct and several part
refuse to spend appropriations or to retain or deduct appropriations for of the appropriation bill, in line with the item-veto power of the President,
whatever reason. Impoundment is actually prohibited by the GAA unless must contain "specific appropriations of money" and not be only general
there will be an unmanageable national government budget deficit (which provisions. An item of an appropriation bill means an item which, in itself,
did not happen). Nevertheless, there’s no impoundment in the case at bar is a specific appropriation of money, not some general provision of law
because what’s involved in the DAP was the transfer of funds. which happens to be put into an appropriation bill. An item of appropriation
must be an item characterized by singular correspondence – meaning an
As to the third requisite: Cross-border augmentations from savings allocation of a specified singular amount for a specified singular purpose,
were prohibited by the Constitution. The phrase respective offices used in otherwise known as a "line-item." This treatment not only allows the item
Section 25(5), supra refers to the entire Executive, with respect to the to be consistent with its definition as a "specific appropriation of money"
President; the Senate, with respect to the Senate President; the House of but also ensures that the President may discernibly veto the same.
Representatives, with respect to the Speaker; the Judiciary, with respect
to the Chief Justice; the Constitutional Commissions, with respect to their Accordingly, the item referred to by Section 25(5) of the Constitution is
respective Chairpersons. the last and indivisible purpose of a program in the appropriation law,
which is distinct from the expense category or allotment class. There is no
The plain text of Section 25 (5), disallowing cross-border transfers was specificity, indeed, either in the Constitution or in the relevant GAAs that
disobeyed. Cross-border transfers, whether as augmentation, or as aid, the object of augmentation should be the expense category or allotment
were prohibited under Section 25 (5). class. In the same vein, the President cannot exercise his veto power over
an expense category; he may only veto the item to which that expense
The funding of projects, activities and programs that were not covered by category belongs to. There must be an existing item, project or activity,
any appropriation in the General Appropriations Act is unconstitutional. No purpose or object of expenditure with an appropriation to which savings
funds from savings could be transferred under the DAP to augment may be transferred for the purpose of augmentation. Accordingly, so long
deficient items not provided in the GAA. The Supreme Court conclude that as there is an item in the GAA for which Congress had set aside a specified
the "savings" pooled under the DAP were allocated to PAPs that were not amount of public fund, savings may be transferred thereto for
covered by any appropriations in the pertinent GAAs. The Executive augmentation purposes. This interpretation is consistent not only with the
thereby substituted its will to that of Congress. Constitution and the GAAs, but also with the degree of flexibility allowed
The DBM’s withdrawal of unobligated allotments had disregarded the to the Executive during budget execution in responding to unforeseeable
definition of savings under the GAAs. contingencies.

Unobligated allotments were encompassed by the first part of the


definition of "savings" in the GAA, that is, as "portions or balances of any

M.R.A.D.C. LUMBRE 110


CONSTITUTIONAL LAW REVIEW

Nonetheless, this modified interpretation does not take away the caveat
that only DAP projects found in the appropriate GAAs may be the subject
of augmentation by legally accumulated savings.
NOTE: Augmentation is the power of the President to fill-in a shortage.
The only prohibition is “cross-border” transfer or augmentations from
savings. That is, transfer of savings from one department to the other,
such as from the Executive to Congress, which was what happened in DAP.
Specifically, the savings from the Executive was transferred to fund
projects of the Congressmen, and that is illegal.
CFAG was also mentioned in this case. It means “Constitutional Fiscal
Autonomy Group.” This consists of six (6) agencies: the Supreme Court;
the Constitutional Commissions: COMELEC, COA, CSC; the Ombudsman;
and the Commission on Human Rights (CHR). CFAG has fiscal autonomy;
savings from any of these agencies will not be required to be returned to
the Treasury.
k. Power with regard to the utilization of natural resources – Art. XII,
Sec 2, (pars. 4 and 5)
Section 2. xxxx The President may enter into agreements with foreign-owned
corporations involving either technical or financial assistance for large-scale
exploration, development, and utilization of minerals, petroleum, and other
mineral oils according to the general terms and conditions provided by law,
based on real contributions to the economic growth and general welfare of the
country. In such agreements, the State shall promote the development and
use of local scientific and technical resources.
The President shall notify the Congress of every contract entered into in
accordance with this provision, within thirty days from its execution.
i. Congressional power with regard to small-scale utilization Art.
XII, Sec 2, (par. 3)
Section 2. The Congress may, by law, allow small-scale utilization of
natural resources by Filipino citizens, as well as cooperative fish farming,
with priority to subsistence fishermen and fish workers in rivers, lakes,
bays, and lagoons.
ii. Resident Marine Mammals vs. Sec. Angelo Reyes, G.R. No.
180771, 21 April 2015
See discussion in Part IV, 12(a).

M.R.A.D.C. LUMBRE 111


CONSTITUTIONAL LAW REVIEW

VII. Judicial Department As clearly delimited by the aforequoted provision, the BPO issued by the Punong
Barangay or, in his unavailability, by any available Barangay Kagawad, merely
orders the perpetrator to desist from (a) causing physical harm to the woman or
her child; and (2) threatening to cause the woman or her child physical harm. Such
1. Concept
function of the Punong Barangay is, thus, purely executive in nature, in pursuance
a. Judicial power, traditional and expanded meaning – Art. VIII, Sec. 1 of his duty under the Local Government Code to “enforce all laws and ordinances,”
and to “maintain public order in the barangay.”
Section 1. The judicial power shall be vested in one Supreme Court and in such
lower courts, as may be established by law. We have held that “(t)he mere fact that an officer is required by law to inquire into
the existence of certain facts and to apply the law thereto in order to determine
Judicial power includes the duty of the courts of justice to settle actual controversies what his official conduct shall be and the fact that these acts may affect private
involving rights which are legally demandable and enforceable, and to determine rights do not constitute an exercise of judicial powers.”
whether or not there has been a grave abuse of discretion amounting to lack of
jurisdiction on the part of any branch or instrumentality of the Government. In the same manner as the public prosecutor ascertains through a preliminary
inquiry or proceeding “whether there is reasonable ground to believe that an
- Garcia v. Drilon, 699 SCRA 352 (2013) offense has been committed and the accused is probably guilty thereof,” the
ISSUE: WON RA9262 is unconstitutional because it allows an undue delegation of Punong Barangay must determine reasonable ground to believe that an imminent
judicial power to the barangay officials. danger of violence against the woman and her children exists or is about to recur
that would necessitate the issuance of a BPO. The preliminary investigation
RULING: There is no undue delegation of judicial power to barangay officials. conducted by the prosecutor is, concededly, an executive, not a judicial, function.
The same holds true with the issuance of a BPO.
Petitioner contends that protection orders involve the exercise of judicial power,
which, under the Constitution, is placed upon the Supreme Court and such other i. Political Question Doctrine
lower courts as may be established by law, and thus protests the delegation of
power to barangay officials to issue orders. The pertinent provisions reads, as Political questions refer “to those questions which, under the Constitution, are
follows. to be decided by the people in their sovereign capacity, or in regard to which
full discretionary authority has been delegated to the legislative or executive
Sec. 14. Barangay Protection Orders (BPOSs): who may issue: Barangay Protection branch of the government. It is concerned with issues dependent upon the
Orders (BPOs) refer to the protection order issued by the Punong Barangay ordering wisdom, not legality of a particular measure.”
the perpetrator to desist from committing acts under Section 5(1) and (b) of this
Act. A Punong Barangay who received applications for a BPO shall issue the - Vinuya vs. Executive Secretary, 28 April 2010
protection order to the applicant on the date of filing after ex parte determination ISSUE: Whether the Executive Department committed grave abuse of
of the basis of the application. If the Punong Barangay is unavailable to act on the discretion in not espousing petitioners' claims for official apology and other
application for a BPO, the application shall be acted upon by any available forms of reparations against Japan.
Barangay. Kagawad. If the BPO is issued by a Barangay Kagawad, the order must
be accompanied by an attestation by the Barangay Kagawad, that: RULING: The Court held that the petition lacks merit. From a domestic law
perspective, the Executive Department has the exclusive prerogative to
The Punong Barangay was unavailable at the time of the issuance of the BPO. determine whether to espouse petitioners’ claims against Japan.
BPOs shall be effective for fifteen (15) days. Immediately after the issuance of
an ex parte BPO, the Punong Barangay or Barangay Kagawad shall personally To support its ruling, the Court explained the concept of political question.
serve a copy of the same on the respondent, or direct any barangay official to Political questions refer “to those questions which, under the Constitution, are
effect its personal service. to be decided by the people in their sovereign capacity, or in regard to which
full discretionary authority has been delegated to the legislative or executive
The parties may be accompanied by a non-lawyer advocate in any proceeding branch of the government. It is concerned with issues dependent upon the
before the Punong Barangay. wisdom, not legality of a particular measure.”
Judicial power includes the duty of the courts of justice to settle actual controversies One type of case of political questions involves questions of foreign relations.
involving rights which are legally demandable and enforceable, and to determine It is well-established that “the conduct of the foreign relations of our
whether or not there has been a grave abuse of discretion amounting to lack or government is committed by the Constitution to the executive and legislative–
excess of jurisdiction on the part of any branch or instrumentality of the ‘the political’–departments of the government, and the propriety of what may
Government. On the other hand, executive power "is generally defined as the power be done in the exercise of this political power is not subject to judicial inquiry
to enforce and administer the laws. It is the power of carrying the laws into practical or decision.” are delicate, complex, and involve large elements of prophecy.
operation and enforcing their due observance." They are and should be undertaken only by those directly responsible to the

M.R.A.D.C. LUMBRE 112


CONSTITUTIONAL LAW REVIEW

people whose welfare they advance or imperil. But not all cases implicating - Ocampo vs. Rear Admiral Enriquez, G.R. No. 225973, 8 November
foreign relations present political questions, and courts certainly possess the 2016
authority to construe or invalidate treaties and executive agreements.
However, the question whether the Philippine government should espouse RULING: The Court agrees with the OSG that President Duterte's decision to
claims of its nationals against a foreign government is a foreign relations have the remains of Marcos interred at the LNMB involves a political question
matter, the authority for which is demonstrably committed by our Constitution that is not a justiciable controversy. In the excercise of his powers under the
not to the courts but to the political branches. In this case, the Executive Constitution and the Administrative Code of 1987 to allow the interment of
Department has already decided that it is to the best interest of the country to Marcos at the LNMB, which is a land of the public domain devoted for national
waive all claims of its nationals for reparations against Japan in the Treaty of military cemetery and military shrine purposes, President Duterte decided a
Peace of 1951. The wisdom of such decision is not for the courts to question. question of policy based on his wisdom that it shall promote national healing
and forgiveness. There being no taint of grave abuse in the exercise of such
The President, not Congress, has the better opportunity of knowing the discretion, as discussed below, President Duterte's decision on that political
conditions which prevail in foreign countries, and especially is this true in time question is outside the ambit of judicial review.
of war. He has his confidential sources of information. He has his agents in the
form of diplomatic, consular and other officials. The Executive Department has In sum, there is no clear constitutional or legal basis to hold that there was a
determined that taking up petitioners’ cause would be inimical to our country’s grave abuse of discretion amounting to lack or excess of jurisdiction which
foreign policy interests, and could disrupt our relations with Japan, thereby would justify the Court to interpose its authority to check and override an act
creating serious implications for stability in this region. For the to overturn the entrusted to the judgment of another branch. Truly, the President’s discretion
Executive Department’s determination would mean an assessment of the is not totally unfettered. “Discretion is not a free spirited stallion that runs and
foreign policy judgments by a coordinate political branch to which authority to roams wherever it pleases but is reined in to keep it from straying. In its classic
make that judgment has been constitutionally committed. From a municipal formulation, ‘discretion is not unconfined and vagrant’ but ‘canalized within
law perspective, certiorari will not lie. As a general principle, where such an banks that keep it from overflowing.”‘186 At bar, President Duterte, through
extraordinary length of time has lapsed between the treaty’s conclusion and the public respondents, acted within the bounds of the law and jurisprudence.
our consideration – the Executive must be given ample discretion to assess the Notwithstanding the call of human rights advocates, the Court must uphold
foreign policy considerations of espousing a claim against Japan, from the what is legal and just. And that is not to deny Marcos of his rightful place at
standpoint of both the interests of the petitioners and those of the Republic, the LNMB. For even the Framers of our Constitution intend that full respect for
and decide on that basis if apologies are sufficient, and whether further steps human rights is available at any stage of a person’s development, from the
are appropriate or necessary. time he or she becomes a person to the time he or she leaves this earth.

In the international sphere, traditionally, the only means available for There are certain things that are better left for history -not this Court -to
individuals to bring a claim within the international legal system has been when adjudge. The Court could only do so much in accordance with the clearly
the individual is able to persuade a government to bring a claim on the established rules and principles. Beyond that, it is ultimately for the people
individual’s behalf. By taking up the case of one of its subjects and by resorting themselves, as the sovereign, to decide, a task that may require the better
to diplomatic action or international judicial proceedings on his behalf, a State perspective that the passage of time provides. In the meantime, the country
is in reality asserting its own right to ensure, in the person of its subjects, must move on and let this issue rest.”
respect for the rules of international law. Within the limits prescribed by b. Requisites for the proper exercise of the power of judicial review
international law, a State may exercise diplomatic protection by whatever
means and to whatever extent it thinks fit, for it is its own right that the State The power of judicial review, like all powers granted by the Constitution, is
is asserting. Should the natural or legal person on whose behalf it is acting subject to certain limitations. Petitioner must comply with all the requisites for
consider that their rights are not adequately protected, they have no remedy judicial review before this court may take cognizance of the case. The requisites
in international law. All they can do is resort to national law, if means are are:
available, with a view to furthering their cause or obtaining redress. All these
(1) there must be an actual case or controversy calling for the exercise of
questions remain within the province of municipal law and do not affect the
judicial power;
position internationally.
(2) the person challenging the act must have the standing to question the
Even the invocation of jus cogens norms and erga omnes obligations will not
validity of the subject act or issuance; otherwise stated, he must have a
alter this analysis. Petitioners have not shown that the crimes committed by
personal and substantial interest in the case such that he has sustained, or will
the Japanese army violated jus cogens prohibitions at the time the Treaty of
sustain, direct injury as a result of its enforcement;
Peace was signed, or that the duty to prosecute perpetrators of international
crimes is an erga omnes obligation or has attained the status of jus cogens. (3) the question of constitutionality must be raised at the earliest
opportunity; and

M.R.A.D.C. LUMBRE 113


CONSTITUTIONAL LAW REVIEW

(4) the issue of constitutionality must be the very lis mota of the case. 1. interested under a deed, will, contract or other written instrument, or
i. Actual case or controversy 2. whose rights are affected by a statute, executive order or regulation,
ordinance, or any other governmental regulation before breach or violation
1) Cases: thereof,
- In the Matter of: Save the SC Judicial Independence and Fiscal asking the court to determine any question of construction or validity arising,
Autonomy Movement vs. Abolition of Judiciary Development Fund and for a declaration of his rights or duties thereunder (Sec. 1, Rule 63).
(JDF) and Reduction of Fiscal Autonomy, UDK-15143, 21 January 2015
Declaratory judgments are to be distinguished from those which are advisory
FACTS: Petitioner Rolly Mijares prays for the issuance of a writ of in character, since they are res judicata and binding upon the parties and those
mandamus in order to compel the court to exercise its judicial in privity with them, and from decisions of abstract or moot questions since
independence and fiscal autonomy against the perceived hostility of they must involve a real controversy (16 Am. Jur. Declaratory Judgments).
Congress. In his petition, Mijares alleges that he is a Filipino citizen, and a
concerned taxpayer. That he filed this petition as part of his “continuing - International Court of Justice – Article 96, Chapter XIV of the UN
crusade to defend and uphold the Constitution”. That he is concerned Charter
about the threats against the judiciary after this court promulgated
Priority Development Assistance Fund (PDAF) case on November 19, 2013 Article 96. (a) The General Assembly or the Security Council may request the
and Disbursement Acceleration Program (DAP) case on July 1, 2014. The International Court of Justice to give an advisory opinion on any legal question.
complaint implied that certain acts of members of Congress and the President (b) Other organs of the United Nations and specialized agencies, which may
after the promulgation of these cases show a threat to judicial independence at any time be so authorized by the General Assembly, may also request
when in the first week of July 2014, Ilocos Norte Representative Rodolfo advisory opinion of the Court on legal questions arising within the scope of
Fariñas filed House Bill No. 4690, which would require this court to remit its their activities
Judiciary Development Fund collections to the national treasury. A week later,
Iloilo Representative Niel Tupas, Jr., filed House Bill No. 4738 entitled “The 3) “Moot and academic”
Act Creating the Judicial Support Fund (JSF) under the National Treasury,
- International Service for the Acquisition of Agri-Biotech Applications,
repealing for the purpose Presidential Decree No. 1949. On the same day,
Inc. vs. Greenpeace Southeast Asia (Philippines) vs. Court of Appeals,
President Aquino III addressed the nation urging the SC to review its decision
G.R. No. 209271, 26 July 2016
in the PDAF and DAF cases.
FACTS: On April 26, 2012, respondents Greenpeace Southeast Asia
RULING: No. The SC resolved to deny the petition. The power of judicial
(Philippines) (Greenpeace), Magsasaka at Siyentipiko sa Pagpapaunlad ng
review, like all powers granted by the Constitution, is subject to certain
Agrikultura (MASIPAG), and others (respondents) filed before the Court a
limitations. Petitioner must comply with all the requisites for judicial review
Petition for Writ of Continuing Mandamus and Writ of Kalikasan with Prayer for
before this court may take cognizance of the case. The requisites are:
the Issuance of a Temporary Environmental Protection Order (TEPO) (petition
(1) there must be an actual case or controversy calling for the exercise
for Writ of Kalikasan) against herein petitioners the Environmental
of judicial power; (2) the person challenging the act must have the
Management Bureau (EMB) of the Department of Environment and Natural
standing to question the validity of the subject act or issuance; otherwise
Resources (DENR), the BPI and the Fertilizer and Pesticide Authority (FPA) of
stated, he must have a personal and substantial interest in the case such
the DA, UPLBFI, and ISAAA, and UPMFI, alleging that the Bt talong field trials
that he has sustained, or will sustain, direct injury as a result of its
violated their constitutional right to health and a balanced ecology considering,
enforcement; (3) the question of constitutionality must be raised at the
among others, that: (a) the Environmental Compliance Certificate (ECC), as
earliest opportunity; and (4) the issue of constitutionality must be the very
required by Presidential Decree No. (PD) 1151, was not secured prior to the
lis mota of the case.
field trials; (b) the required public consultations under the Local Government
One of the requirements for this court to exercise its power of judicial Code (LGC) were not complied with; and (c) as a regulated article under DAO
review is the existence of an actual controversy. This means that there 08-2002, Bt talong is presumed harmful to human health and the environment,
must be “an existing case or controversy that is appropriate or ripe for and that there is no independent, peer-reviewed study showing its safety for
determination, not conjectural or anticipatory, lest the decision of the court human consumption and the environment.
would amount to an advisory opinion.”
ISSUE: Whether the case should be dismissed on the basis of being moot and
2) Advisory opinion academic.

- Declaratory relief – Sec, 1, Rule 63 of the Rules of Court, as amended RULING: YES, the case should be dismissed. The case does not fall under the
two exceptions of (a) the exceptional character of the situation and the
It is a special civil action brought by a person:

M.R.A.D.C. LUMBRE 114


CONSTITUTIONAL LAW REVIEW

paramount public interest is involved; and (b) the case is capable of repetition and academic, and that the aforesaid exceptions to the said rule should not
yet evading review have been applied.

To resolve respondents' petition for Writ of Kalikasan on its merits, would be ii. Proper party (“locus standi” or legal standing)
tantamount to an unnecessary scholarly exercise for the Court to assess
alleged violations of health and environmental rights that arose from a past The constitutional question must be raised by the proper party. A proper party is
test case whose bearings do not find any - if not minimal -- relevance to cases one who has sustained or is in imminent danger of sustaining an injury as a result
operating under today's regulatory framework. of the act complained of. To be a proper party, one must have “legal standing”, or
locus standi.
The Court discerns that there are two (2) factors to be considered before a
case is deemed one capable of repetition yet evading review: It was clarified that the term “interest” means a material interest, an interest in
issue affected by the challenged official act, as distinguished from mere interest in
(1) the challenged action was in its duration too short to be fully litigated prior the question involved, or a mere incidental interest.
to its cessation or expiration; and
1) General Rule: “direct injury test”
(2) there was a reasonable expectation that the same complaining party would
be subjected to the same action. - Hon. Executive Secretary, et al. vs. Southwing Heavy Industries, Inc.,
et al., GR No. 16417, 20 February 2006
Here, respondents cannot claim that the duration of the subject field tests was
too short to be fully litigated. It must be emphasized that the Biosafety Permits ISSUES: Whether Southwing has legal standing in questioning EO 156 or
for the subject field tests were issued on March 16, 2010 and June 28, 2010, "Providing for a comprehensive industrial policy and directions for the motor
and were valid for two (2) years. However, as aptly pointed out by Justice vehicle development program and its implementing guidelines." The said
Leonen, respondents filed their petition for Writ of Kalikasan only on April 26, provision prohibits the importation of all types of used motor vehicles in the
2012 - just a few months before the Biosafety Permits expired and when the country including the Subic Bay Freeport, or the Freeport Zone, subject to a
field testing activities were already over. Obviously, therefore, the cessation of few exceptions” issued by President GMA.
the subject field tests before the case could be resolved was due to RULING: Yes. The established rule that the constitutionality of a law or
respondents' own inaction. administrative issuance can be challenged by one who will sustain a direct
The Court is not empowered to decide moot questions or abstract propositions, injury as a result of its enforcement has been satisfied in the instant case. The
or to declare principles or rules of law which cannot affect the result as to the broad subject of the prohibited importation is “all types of used motor
thing in issue in the case before it. In other words, when a case is moot, it vehicles.”
becomes non-justiciable. Respondents would definitely suffer a direct injury from the implementation of
An action is considered "moot" when it no longer presents a justiciable EO 156 because their certificate of registration and tax exemption authorize
controversy because the issues involved have become academic or dead or them to trade and/or import new and used motor vehicles and spare parts,
when the matter in dispute has already been resolved and hence, one is not except “used cars.” Other types of motor vehicles imported and/or traded by
entitled to judicial intervention unless the issue is likely to be raised again respondents and not falling within the category of used cars would thus be
between the parties. There is nothing for the court to resolve as the subjected to the ban to the prejudice of their business. Undoubtedly,
determination thereof has been overtaken by subsequent events. respondents have the legal standing to assail the validity of EO 156.

Nevertheless, case law states that the Court will decide cases, otherwise moot, On the issue of the validity of the exercise of the President’s quasi-legislative
if: first, there is a grave violation of the Constitution; second, the exceptional power:
character of the situation and the paramount public interest are involved; Police power is inherent in a government to enact laws, within constitutional
third, when the constitutional issue raised requires formulation of controlling limits, to promote the order, safety, health, morals, and general welfare of
principles to guide the bench, the bar, and the public; and fourth, the case is society. It is lodged primarily with the legislature. By virtue of a valid
capable of repetition yet evading review. Thus, jurisprudence recognizes these delegation of legislative power, it may also be exercised by the President and
four instances as exceptions to the mootness principle. administrative boards, as well as the lawmaking bodies on all municipal levels,
In the December 8, 2015 Decision of the Court, it was held that (a) the present including the barangay.
case is of exceptional character and paramount public interest is involved, and Such delegation confers upon the President quasi-legislative power which may
(b) it is likewise capable of repetition yet evading review. Hence, it was be defined as the authority delegated by the law-making body to the
excepted from the mootness principle. However, upon a closer scrutiny of the administrative body to adopt rules and regulations intended to carry out the
parties' arguments, the Court reconsiders its ruling and now finds merit in
petitioners' assertion that the case should have been dismissed for being moot

M.R.A.D.C. LUMBRE 115


CONSTITUTIONAL LAW REVIEW

provisions of the law and implement legislative policy provided that it must However, being a mere procedural technicality, the requirement of locus standi
comply with the following requisites: may be waived by the Court in the exercise of its discretion.

(1) Its promulgation must be authorized by the legislature; By way of summary, the following rules may be culled from the cases decided
by this Court. Taxpayers, voters, concerned citizens, and legislators may be
(2) It must be promulgated in accordance with the prescribed procedure; accorded standing to sue, provided that the following requirements are met:
(3) It must be within the scope of the authority given by the legislature; (1) the cases involve constitutional issues;
and
(2) for taxpayers, there must be a claim of illegal disbursement of public
(4) It must be reasonable. funds or that the tax measure is unconstitutional;
The Court finds that Article 2, Section 3.1 of EO 156 is VOID insofar as it is (3) for voters, there must be a showing of obvious interest in the validity
made applicable within the secured fenced-in former Subic Naval Base area of the election law in question;
but is declared VALID insofar as it applies to the customs territory or the
Philippine territory outside the presently secured fenced-in former Subic Naval (4) for concerned citizens, there must be a showing that the issues
Base area as stated in Section 1.1 of EO 97-A (an EO executed by Pres. Fidel raised are of transcendental importance which must be settled early; and
V. Ramos in 1993 providing the Tax and Duty Free Privilege within the Subic
Freeport Zone). Hence, used motor vehicles that come into the Philippine (5) for legislators, there must be a claim that the official action
territory via the secured fenced-in former Subic Naval Base area may be complained of infringes upon their prerogatives as legislators.
stored, used or traded therein, or exported out of the Philippine territory, but It must always be borne in mind that the question of locus standi is but
they cannot be imported into the Philippine territory outside of the secured corollary to the bigger question of proper exercise of judicial power. This is the
fenced-in former Subic Naval Base area. underlying legal tenet of the “liberality doctrine” on legal standing. It cannot
Exceptions: be doubted that the validity of PP No. 1017 and G.O. No. 5 is a judicial question
which is of paramount importance to the Filipino people. To paraphrase Justice
a. Oposa v. Factoran, Jr., 224 SCRA 792 [1993] Laurel, the whole of Philippine society now waits with bated breath the ruling
of this Court on this very critical matter. The petitions thus call for the
RULING: We find no difficulty in ruling that they can, for themselves, for application of the “transcendental importance” doctrine, a relaxation of the
others of their generation and for the succeeding generations, file a class suit. standing requirements for the petitioners in the “PP 1017 cases.”
Their personality to sue in behalf of the succeeding generations can only be
based on the concept of intergenerational responsibility insofar as the right to Thus, this Court holds that all the petitioners herein have locus standi.
a balanced and healthful ecology is concerned.
c. Concerned Citizen: Transcendental Importance - Ocampo vs. Rear
Such a right, as hereinafter expounded, considers the "rhythm and harmony Admiral Enriquez, G.R. No. 225973, 8 November 2016
of nature." Nature means the created world in its entirety. Such rhythm and
harmony indispensably include, inter alia, the judicious disposition, utilization, RULING: Petitioners have no locus standi. Defined as a right of appearance in
management, renewal and conservation of the country's forest, mineral, land, a court of justice on a given question, locus standi requires that a party alleges
waters, fisheries, wildlife, off-shore areas and other natural resources to the such personal stake in the outcome of the controversy as to assure that
end that their exploration, development and utilization be equitably accessible concrete adverseness which sharpens the presentation of issues upon which
to the present as well as future generations. 10 Needless to say, every the court depends for illumination of difficult constitutional questions. Unless a
generation has a responsibility to the next to preserve that rhythm and person has sustained or is in imminent danger of sustaining an injury as a
harmony for the full enjoyment of a balanced and healthful ecology. Put a little result of an act complained of, such proper party has no standing. Petitioners,
differently, the minors' assertion of their right to a sound environment who filed their respective petitions for certiorari, prohibition and mandamus,
constitutes, at the same time, the performance of their obligation to ensure in their capacities as citizens, human rights violations victims, legislators,
the protection of that right for the generations to come. members of the Bar and taxpayers, have no legal standing to file such petitions
because they failed to show that they have suffered or will suffer direct and
b. David vs. Macapagal- Arroyo, GR No. 171396, 3 May 2006 personal injury as a result of the interment of Marcos at the LNMB.
RULING: This Court adopted the “direct injury” test in our jurisdiction. In As concerned citizens, petitioners are also required to substantiate
People v. Vera, it held that the person who impugns the validity of a statute that the issues raised are of transcendental importance, of
must have “a personal and substantial interest in the case such that he has overreaching significance to society, or of paramount public interest.
sustained, or will sustain direct injury as a result.” The Vera doctrine was In cases involving such issues, the imminence and clarity of the threat to
upheld in a litany of cases. fundamental constitutional rights outweigh the necessity for prudence. In

M.R.A.D.C. LUMBRE 116


CONSTITUTIONAL LAW REVIEW

Marcos v. Manglapus, the majority opinion observed that the subject member of the House of Representatives has standing to maintain inviolate
controversy was of grave national importance, and that the Court's decision the prerogatives, powers and privileges vested by the Constitution in his office.
would have a profound effect on the political, economic, and other aspects of
national life. The ponencia explained that the case was in a class by itself, It is clear therefrom that each member of Congress has a legal standing to sue
unique and could not create precedent because it involved a dictator forced even without an enabling resolution for that purpose so long as the questioned
out of office and into exile after causing twenty years of political, economic and acts invade the powers, prerogatives and privileges of Congress. Otherwise
social havoc in the country and who, within the short space of three years stated, whenever the acts affect the powers, prerogatives and privileges of
(from 1986), sought to return to the Philippines to die. Congress, anyone of its members may validly bring an action to challenge the
same to safeguard and maintain the sanctity thereof.
d. Legislator’s Suit - Umali vs. JBC, G.R. No. 228628, 25 July 2017
e. Legislator’s Suit - Saguisag vs. Executive Secretary Ochoa, G.R. No.
FACTS: This Petition for Certiorari and Mandamus by Rep. Umali, current Chair 212426, 12 January 2016
of the House of Representatives Committee on Justice, impugns the present-
day practice of six-month rotational representation of Congress in the Judicial ISSUE: Whether petitioners have the legal standing to file the suit against the
and Bar Council (JBC) for it unfairly deprives both Houses of Congress of their executive agreement between the Republic and the US?
full participation in the said body. The aforementioned practice was adopted RULING: No, the present petitions cannot qualify as citizens', taxpayers', or
by the JBC in light of the ruling in Chavez v. JBC. legislators' suits; the Senate as a body has the requisite standing, but
In light of these Decision and Resolution, both Houses of Congress agreed on considering that it has not formally filed a pleading to join the suit, as it merely
a six-month rotational representation in the JBC, wherein the House of conveyed to the Supreme Court its sense that EDCA needs the Senate's
Representatives will represent Congress from January to June and the Senate concurrence to be valid, petitioners continue to suffer from lack of standing.
from July to December. This is now the current practice in the JBC. It is by The question of locus standi or legal standing focuses on the determination of
reason of this arrangement that the votes cast by the petitioner for the whether those assailing the governmental act have the right of appearance to
selection of nominees for the vacancies of then retiring Supreme Court bring the matter to the court for adjudication. They must show that they have
Associate Justices Jose P. Perez (Perez) and Arturo Brion (Brion) were not a personal and substantial interest in the case, such that they have sustained
counted by the JBC during its En Banc deliberations held last December 2 and or are in immediate danger of sustaining, some direct injury as a consequence
9, 2016. Instead, the petitioner's votes were simply placed in an envelope and of the enforcement of the challenged governmental act. Here, "interest" in the
sealed subject to any further disposition as this Court may direct in a proper question involved must be material - an interest that is in issue and will be
proceeding. affected by the official act - as distinguished from being merely incidental or
RULING: Petitioner has legal standing. The petitioner brings this suit in his general. Clearly, the Court states, it would be insufficient to show that the law
capacity as the current Chairman of the House of Representatives Committee or any governmental act is invalid, and that petitioners stand to suffer in some
on Justice and Ex Officio Member of the JBC. His legal standing was challenged indefinite way. They must show that they have a particular interest in bringing
by the JBC for lack of an enabling resolution for that purpose coming from both the suit, and that they have been or are about to be denied some right or
Houses of Congress. privilege to which they are lawfully entitled, or that they are about to be
subjected to some burden or penalty by reason of the act complained of. The
Locus standi or legal standing is defined as a personal and substantial interest reason why those who challenge the validity of a law or an international
in a case such that the party has sustained or will sustain direct injury as a agreement are required to allege the existence of a personal stake in the
result of the challenged governmental act. It requires a personal stake in the outcome of the controversy is "to assure the concrete adverseness which
outcome of the controversy as to assure the concrete adverseness which sharpens the presentation of issues upon which the court so largely depends
sharpens the presentation of issues upon which the court so largely depends for illumination of difficult constitutional questions."
for illumination of difficult constitutional questions. With that definition,
therefore, a party will be allowed to litigate only when he can demonstrate that In assailing the constitutionality of a governmental act, petitioners suing as
(1) he has personally suffered some actual or threatened injury because of the citizens may dodge the requirement of having to establish a direct and personal
allegedly illegal conduct of the government; (2) the injury is fairly traceable to interest if they show that the act affects a public right. In arguing that they
the challenged action; and (3) the injury is likely to be redressed by the remedy have legal standing, they claim that the case they have filed is a concerned
being sought. Otherwise, he/she would not be allowed to litigate. Nonetheless, citizen's suit. But aside from general statements that the petitions involve the
in a long line of cases, concerned citizens, taxpayers and legislators when protection of a public right, and that their constitutional rights as citizens would
specific requirements have been met have been given standing by this Court. be violated, they fail to make any specific assertion of a particular public right
that would be violated by the enforcement of EDCA. For their failure to do so,
As for a legislator, he is allowed to sue to question the validity of any official the present petitions cannot be considered by the Court as citizens' suits that
action which he claims infringes his prerogatives as a legislator. Indeed, a would justify a disregard of the aforementioned requirements. A taxpayer's
suit concerns a case in which the official act complained of directly involves the

M.R.A.D.C. LUMBRE 117


CONSTITUTIONAL LAW REVIEW

illegal disbursement of public funds derived from taxation. Here, those seriously carry their responsibility including ensuring a viable ecology for
challenging the act must specifically show that they have sufficient interest in themselves, which of course includes compassion for all living things.
preventing the illegal expenditure of public money, and that they will sustain
a direct injury as a result of the enforcement of the assailed act. Applying that Our rules on standing are sufficient and need not be further relaxed.
principle to this case, they must establish that EDCA involves the exercise by Creative approaches to fundamental problems should be welcome. However,
Congress of its taxing or spending powers. they should be considered carefully so that no unintended or unwarranted
The Court emphasize that in a legislators' suit, those Members of Congress consequences should follow. I concur with the approach of Madame Justice
who are challenging the official act have standing only to the extent that the Teresita J. Leonardo-De Castro in her brilliant ponencia as it carefully narrows
alleged violation impinges on their right to participate in the exercise of the down the doctrine in terms of standing. Resident Marine Mammals and the
powers of the institution of which they are members. Legislators have the human petitioners have no legal standing to file any kind of petition.
standing "to maintain inviolate the prerogatives, powers, and privileges vested However, I agree that petitioners in G.R. No. 181527, namely, Central
by the Constitution in their office and are allowed to sue to question the validity Visayas Fisherfolk Development Center, Engarcial, Yanong, and Labid,
of any official action, which they claim infringes their prerogatives as have standing both as real parties in interest and as representatives
legislators." As legislators, they must clearly show that there was a direct of subsistence fisherfolks of the Municipalities of Aloguinsan and
injury to their persons or the institution to which they belong. As correctly Pinamungahan, Cebu, and their families, and the present and future
argued by respondent, the power to concur in a treaty or an international generations of Filipinos whose rights are similarly affected. The activities
agreement is an institutional prerogative granted by the Constitution to the undertaken under Service Contract 46 (SC-46) directly affected their source of
Senate, not to the entire Legislature. In Pimentel v. Office of the Executive livelihood, primarily felt through the significant reduction of their fish harvest.
Secretary, the Court did not recognize the standing of one of the petitioners The actual, direct, and material damage they suffered, which has potential
therein who was a member of the House of Representatives. The petition in long-term effects transcending generations, is a proper subject of a legal suit.
that case sought to compel the transmission to the Senate for concurrence of
the signed text of the Statute of the International Criminal Court. Since that g. Steward of Nature: Writ of Kalikasan - West Tower Condominium
petition invoked the power of the Senate to grant or withhold its concurrence Corporation vs. First Philippine Industrial Corporation, G.R. No.
in a treaty entered into by the Executive Department, only then incumbent 194239, 16 June 2015
Senator Pimentel was allowed to assert that authority of the Senate of which
ISSUE: Whether petitioner has the legal capacity to represent the other
he was a member.
petitioners and, whether the other petitioners, apart from the residents of West
f. Steward of Nature: Writ of Kalikasan - Resident Marine Mammals vs. Tower and Barangay Bangkal, are real parties-in-interest.
Sec. Angelo Reyes, G.R. No. 180771, 21 April 2015
RULING: Yes. While a condominium corporation has limited powers under RA
RULING: What may be argued as being parallel to this concept of guardianship 4726, otherwise known as The Condominium Act, it is empowered to pursue
is the principle of human stewardship over the environment in a citizen suit actions in behalf of its members. In this case, the condominium corporation is
under the Rules of Procedure for Environmental Cases. A citizen suit allows any the management body of West Tower and deals with everything that may
Filipino to act as a representative of a party who has enforceable rights under affect some or all of the condominium unit owners or users.
environmental laws before Philippine courts, and is defined in Section 5:
It is of no moment that only five residents of West Tower signed their
SEC. 5. Citizen suit. - Any Filipino citizen in representation of others, acquiescence to the filing of the petition for the issuance of the Writ of
including minors or generations yet unborn, may file an action to enforce Kalikasan, as the merits of such petition is not measured by the number of
rights or obligations under environmental laws. Upon the filing of a citizen persons who signified their assent thereto, but on the existence of a prima
suit, the court shall issue an order which shall contain a brief description facie case of a massive environmental disaster. As to the residents of Barangay
of the cause of action and the reliefs prayed for, requiring all interested Bangkal, they are similarly situated with the unit owners and residents of West
parties to manifest their interest to intervene in the case within fifteen Tower and are real parties-in-interest to the instant case, if they so wish to
(15) days from notice thereof. The plaintiff may publish the order once in join the petitioners.
a newspaper of a general circulation in the Philippines or furnish all
On the other hand, the Court ruled that the fact that no board resolution was
affected barangays copies of said order.
submitted by West Tower Corp. authorizing Manuel Dy Chuaunsu, Jr. to sign
There is no valid reason in law or the practical requirements of this case to the Verification and Certification of Non-forum Shopping is irrelevant. The
implead and feign representation on behalf of animals. To have done so betrays records show that petitioners submitted a notarized Secretary's Certificate
a very anthropocentric view of environmental advocacy. There is no way that attesting that the authority of Chuaunsu to represent the condominium
we, humans, can claim to speak for animals let alone present that they would corporation in filing the petition is from the resolution of the total membership
wish to use our court system, which is designed to ensure that humans of West Tower Corp. issued during their meeting with the requisite quorum. It

M.R.A.D.C. LUMBRE 118


CONSTITUTIONAL LAW REVIEW

is, thus, clear that it was not the Board of West Tower Corp. which granted 3. the actual or threatened violation involves or will lead to an
Chuaunsu the authority but the full membership of the condominium environmental damage of such magnitude as to prejudice the life, health
corporation itself. or property of inhabitants in two or more cities or provinces.
For the intervention of other petitioners, the Court ruled that the filing of a It is well-settled that a party claiming the privilege for the issuance of a writ of
petition for the issuance of a Writ of Kalikasan under Sec. 1, Rule 7 of the Rules kalikasan has to show that a law, rule or regulation was violated or would be
of Procedure for Environmental Cases does not require that a petitioner be violated.
directly affected by an environmental disaster. The rule clearly allows juridical
persons to file the petition on behalf of persons whose constitutional right to a In this case, apart from repeated invocation of the constitutional right to health
balanced and healthful ecology is violated, or threatened with violation. and to a balanced and healthful ecology and bare allegations that their right
was violated, the petitioners failed to show that public respondents are guilty
h. Writ of Kalikasan vs. Writ of Continuing Mandamus - Segovia vs. The of any unlawful act or omission that constitutes a violation of the petitioners'
Climate Change Commission, G.R. No. 211010, 7 March 2017 right to a balanced and healthful ecology.
FACTS: Petitioners, via petition for Writs of Kalikasan and Continuing Similarly, the writ of continuing mandamus cannot issue.
Mandamus, seek to compel: (a) the public respondents to: (1) implement the
Road Sharing Principle in all roads; (2) divide all roads lengthwise, one-half Rule 8, Section 1 of the RPEC lays down the requirements for a petition for
(½) for all-weather sidewalk and bicycling, the other half for Filipino-made continuing mandamus as follows:
transport vehicles; (3) submit a time-bound action plan to implement the Road SECTION 1. Petition for continuing mandamus.-When any agency or
Sharing Principle throughout the country; (b) the Office of the President, instrumentality of the government or officer thereof unlawfully neglects
Cabinet officials and public employees of Cabinet members to reduce their fuel the performance of an act which the law specifically enjoins as a duty
consumption by fifty percent (50%) and to take public transportation fifty resulting from an office, trust or station in connection with the
percent (50%) of the time; (c) Public respondent DPWH to demarcate and enforcement or violation of an environmental law rule or regulation or a
delineate the road right-of-way in all roads and sidewalks; and (d) Public right therein, or unlawfully excludes another from the use or enjoyment
respondent DBM to instantly release funds for Road Users' Tax. of such right and there is no other plain, speedy and adequate remedy in
RULING: The Rules on Procedure for Environmental Cases did liberalize the the ordinary course of law, the person aggrieved thereby may file a
requirements on standing, allowing the filing of citizen's suit for the verified petition in the proper court, alleging the facts with certainty,
enforcement of rights and obligations under environmental laws. This has been attaching thereto supporting evidence, specifying that the petition
confirmed by this Court's rulings in Arigo v. Swift, and International Service concerns an environmental law, rule or regulation, and praying that
for the Acquisition of Agri-BioTech Applications, Inc. v. Greenpeace Southeast judgment be rendered commanding the respondent to do an act or series
Asia (Philippines). of acts until the judgment is fully satisfied, and to pay damages sustained
by the petitioner by reason of the malicious neglect to perform the duties
However, it bears noting that there is a difference between a petition for the of the respondent, under the law, rules or regulations. The petition shall
issuance of a writ of kalikasan, wherein it is sufficient that the person filing also contain a sworn certification of non-forum shopping.
represents the inhabitants prejudiced by the environmental damage subject of
the writ; and a petition for the issuance of a writ of continuing mandamus, First, the petitioners failed to prove direct or personal injury arising from acts
which is only available to one who is personally aggrieved by the unlawful act attributable to the respondents to be entitled to the writ. While the
or omission. requirements of standing had been liberalized in environmental cases, the
general rule of real party-in-interest applies to a petition for continuing
The Court found that the petitioners failed to establish the requisites for the mandamus.
issuance of the writs prayed for, thus a writ of Kalikasan and/or Continuing
Mandamus should not issue. Second, the Road Sharing Principle is precisely as it is denominated - a
principle. It cannot be considered an absolute imposition to encroach upon the
For a writ of kalikasan to issue, the following requisites must concur: province of public respondents to determine the manner by which this principle
is applied or considered in their policy decisions. Mandamus lies to compel the
1. there is an actual or threatened violation of the constitutional right to a performance of duties that are purely ministerial in nature, not those that are
balanced and healthful ecology; discretionary, and the official can only be directed by mandamus to act but not
2. the actual or threatened violation arises from an unlawful act or to act one way or the other. The duty being enjoined in mandamus must be
omission of a public official or employee, or private individual or entity; one according to the terms provided in the law itself. Thus, the recognized rule
and is that, in the performance of an official duty or act involving discretion, the
corresponding official can only be directed by mandamus to act, but not to act
one way or the other.

M.R.A.D.C. LUMBRE 119


CONSTITUTIONAL LAW REVIEW

i. Judicial Review on Martial Law Proclamation and Suspension of the (7) judicial review on martial law proclamation and suspension of the privilege
Privilege of the Writ – Art. VII, Sec. 18 (3rd Par), in view of the ruling of the writ of habeas corpus – a citizen’s suit questioning the validity of
in Lagman vs. Executive Secretary, G.R. No. 231658, 4 July 2017 the proclamation, in Lagman v. Executive Secretary.
RULING: Section 18 of Article VII which provides that any citizen may file the (8) a citizen’s suit invoking a public right that is violated, as laid down
appropriate proceeding to assail the sufficiency of the factual basis of the in the case of Padilla v. Congress (this seems to be a catch-all provision).
declaration of martial law or the suspension of the privilege of the writ of
habeas corpus. "[T]he only requisite for standing to challenge the validity of 2) Prohibition against third-party standing - “As applied” and Facial
the suspension is that the challenger be a citizen. He need not even be a Challenges
taxpayer." a. Southern Hemisphere Engagement Network, Inc. vs. Anti-
In the Lagman Petition, however, petitioners therein did not categorically Terrorism Council, 632 SCRA 146 (2010)
mention that they are suing's citizens but merely referred to themselves as FACTS: Petitioners-organizations assert locus standi on the basis of being
duly elected Representatives. That they are suing in their official capacities as suspected communist fronts by the government, especially the military;
Members of Congress could have elicited a vigorous discussion considering the whereas individual petitioners invariably invoke the transcendental
issuance by the House of Representatives of House Resolution No. 1050 importance doctrine and their status as citizens and taxpayers.
expressing full support to President Duterte and finding no reason to revoke
Proclamation No. 216. By such resolution, the House of Representatives is Also, the petitioners assail the RA 9372 for being intrinsically vague and
declaring that it finds no reason to review the sufficiency of the factual basis impermissibly broad the definition of the crime of terrorism under the said
of the martial law declaration, which is in direct contrast to the views and Act in that terms like widespread and extraordinary fear and panic among
arguments being espoused by the petitioners in the Lagman Petition. the populace and coerce the government to give in to an unlawful demand
are nebulous, leaving law enforcement agencies with no standard to
Considering, however, the trend towards relaxation of the rules on legal measure the prohibited acts.
standing, as well as the transcendental issues involved in the present Petitions,
the Court will exercise judicial self-restraint, and will not venture into this RULING: The petitioners do not have locus standi as none of them faced
matter. After all, "the Court is not entirely without discretion to accept a suit any charge under the law.
which does not satisfy the requirements of a [bona fide] case or of standing.
Petitioners must show an actual, or immediate danger of sustaining, direct
Considerations paramount to [the requirement of legal standing] could compel
injury as a result of the laws enforcement. To rule otherwise would be to
assumption of jurisdiction." In any case, the Court can take judicial cognizance
corrupt the settled doctrine of locus standi, as every worthy cause is an
of the fact that petitioners in the Lagman Petition are all citizens of the
interest shared by the general public.
Philippines since Philippine citizenship is a requirement for them to be elected
as representatives. We will therefore consider them as suing in their own behalf Neither can locus standi be conferred upon individual petitioners as
as citizens of this country. Besides, respondents did not question petitioners' taxpayers and citizens. A taxpayer suit is proper only when there is an
legal standing. exercise of the spending or taxing power of Congress, whereas citizen
standing must rest on direct and personal interest in the proceeding.
NOTE: There are now eight (8) exceptions to the general rule on direct injury
test. The first 5 are enumerated in David v. Macapagal-Arroyo: RA 9372 is a penal statute and does not even provide for any
appropriation from Congress for its implementation, while none of the
(1) the cases involve constitutional issues;
individual petitioner-citizens has alleged any direct and personal interest
(2) for taxpayers, there must be a claim of illegal disbursement of public in the implementation of the law.
funds or that the tax measure is unconstitutional;
It bears to stress that generalized interests, albeit accompanied by the
(3) for voters, there must be a showing of obvious interest in the validity assertion of a public right, do not establish locus standi. Evidence of a
of the election law in question; direct and personal interest is key.

(4) for concerned citizens, there must be a showing that the issues raised The Court stated that the facial challenge on the ground of overbreadth
are of transcendental importance which must be settled early; and and vagueness doctrines have special application only to free-speech
cases and are not appropriate for testing the validity of penal statutes.
(5) for legislators, there must be a claim that the official action
complained of infringes upon their prerogatives as legislators. The Court outlined the schools of thought on whether the void-for-
vagueness and overbreadth doctrines are equally applicable grounds to
(6) as stewards of nature (via Writ of Kalikasan), as discussed in Resident assail a penal statute. It held (by citing the cases of Romualdez vs.
Mammals v. Sec. Angelo Reyes. Sandiganbayan and Estrada vs. Sandiganbayan) that the two doctrines

M.R.A.D.C. LUMBRE 120


CONSTITUTIONAL LAW REVIEW

have special application only to free-speech cases, and are not appropriate inapplicable to plain penal statutes that generally bear an in terrorem
for testing the validity of penal statute. Citing the Separate Opinion of effect in deterring socially harmful conduct. In fact, the legislature may
Justice Mendoza (important): even forbid and penalize acts formerly considered innocent and lawful, so
long as it refrains from diminishing or dissuading the exercise of
A facial challenge is allowed to be made to a vague statute and to one constitutionally protected rights.
which is overbroad because of possible "chilling effect" upon protected
speech. The theory is that "when statutes regulate or proscribe The Court reiterated that there are critical limitations by which a criminal
speech and no readily apparent construction suggests itself as a statute may be challenged and underscored that an on-its-face
vehicle for rehabilitating the statutes in a single prosecution, the invalidation of penal statutes x x x may not be allowed.
transcendent value to all society of constitutionally protected
expression is deemed to justify allowing attacks on overly broad Since a penal statute may only be assailed for being vague as applied to
statutes with no requirement that the person making the attack petitioners, a limited vagueness analysis of the definition of terrorism in
demonstrate that his own conduct could not be regulated by a statute RA 9372 is legally impermissible absent an actual or imminent charge
drawn with narrow specificity." The possible harm to society in against them
permitting some unprotected speech to go unpunished is outweighed While Estrada did not apply the overbreadth doctrine, it did not preclude
by the possibility that the protected speech of others may be deterred the operation of the vagueness test on the Anti-Plunder Law as applied to
and perceived grievances left to fester because of possible inhibitory the therein petitioner, finding, however, that there was no basis to review
effects of overly broad statutes. the law on its face and in its entirety. It stressed that statutes found vague
This rationale does not apply to penal statutes. Criminal statutes have as a matter of due process typically are invalidated only 'as applied' to a
general in terrorem effect resulting from their very existence, and, if particular defendant.
facial challenge is allowed for this reason alone, the State may well In this jurisdiction, the void-for-vagueness doctrine asserted under the
be prevented from enacting laws against socially harmful conduct. In due process clause has been utilized in examining the constitutionality of
the area of criminal law, the law cannot take chances as in the area criminal statutes. In at least three cases, the Court brought the doctrine
of free speech. into play in analyzing an ordinance penalizing the non-payment of
A statute or act suffers from the defect of vagueness when it lacks municipal tax on fishponds, the crime of illegal recruitment punishable
comprehensible standards that men of common intelligence must under Article 132(b) of the Labor Code, and the vagrancy provision under
necessarily guess at its meaning and differ as to its application. It is Article 202 (2) of the Revised Penal Code. Notably, the petitioners in these
repugnant to the Constitution in two respects: (1) it violates due three cases, similar to those in the two Romualdez and Estrada cases,
process for failure to accord persons, especially the parties were actually charged with the therein assailed penal statute, unlike in the
targeted by it, fair notice of the conduct to avoid; and (2) it leaves present case. This means that the petitioners herein are not actually
law enforcers unbridled discretion in carrying out its provisions charged with the violation of RA 9372, hence, void-for-vagueness doctrine
and becomes an arbitrary flexing of the Government muscle. still cannot be applied.

The overbreadth doctrine, meanwhile, decrees that a governmental There is no merit in the claim that RA 9372 regulates speech so as
purpose to control or prevent activities constitutionally subject to state to permit a facial analysis of its validity
regulations may not be achieved by means which sweep unnecessarily In insisting on a facial challenge on the invocation that the law penalizes
broadly and thereby invade the area of protected freedoms. speech, petitioners contend that the element of unlawful demand in the
A facial challenge is likewise different from an as-applied definition of terrorism must necessarily be transmitted through some form
challenge. of expression protected by the free speech clause.

Distinguished from an as-applied challenge which considers only extant The argument does not persuade. What the law seeks to penalize is
facts affecting real litigants, a facial invalidation is an examination of the conduct, not speech.
entire law, pinpointing its flaws and defects, not only on the basis of its Before a charge for terrorism may be filed under RA 9372, there must first
actual operation to the parties, but also on the assumption or prediction be a predicate crime actually committed to trigger the operation of the
that its very existence may cause others not before the court to refrain key qualifying phrases in the other elements of the crime, including the
from constitutionally protected speech or activities. coercion of the government to accede to an unlawful demand. Given the
The allowance of a facial challenge in free speech cases is justified by the presence of the first element, any attempt at singling out or highlighting
aim to avert the chilling effect on protected speech, the exercise of which the communicative component of the prohibition cannot recategorize the
should not at all times be abridged. As reflected earlier, this rationale is unprotected conduct into a protected speech.

M.R.A.D.C. LUMBRE 121


CONSTITUTIONAL LAW REVIEW

b. Disini, Jr. vs. The Secretary of Justice, G.R. No. 203335, The vagueness of the Communications Decency Act of 1996 (CDA), 47
February 18, 2014 U.S.C.S. §223, is a matter of special concern for two reasons. First, the
CDA is a content-based regulation of speech. The vagueness of such a
RULING: The question is: are online postings such as "Liking" an openly regulation raises special U.S. Const. amend. I concerns because of its
defamatory statement, "Commenting" on it, or "Sharing" it with others, to obvious chilling effect on free speech. Second, the CDA is a criminal
be regarded as "aiding or abetting?" In libel in the physical world, if Nestor statute. In addition to the opprobrium and stigma of a criminal conviction,
places on the office bulletin board a small poster that says, "Armand is a the CDA threatens violators with penalties including up to two years in
thief!" he could certainly be charged with libel. If Roger, seeing the poster, prison for each act of violation. The severity of criminal sanctions may well
writes on it, "I like this!" that could not be libel since he did not author the cause speakers to remain silent rather than communicate even arguably
poster. If Arthur, passing by and noticing the poster, writes on it, unlawful words, ideas, and images. As a practical matter, this increased
"Correct!" would that be libel? No, for he merely expresses agreement deterrent effect, coupled with the risk of discriminatory enforcement of
with the statement on the poster. He still is not its author. Besides, it is vague regulations, poses greater U.S. Const. amend. I concerns than
not clear if aiding or abetting libel in the physical world is a crime. those implicated by certain civil regulations.
But suppose Nestor posts the blog, "Armand is a thief!" on a social xxxx
networking site. Would a reader and his Friends or Followers, availing
themselves of any of the "Like," "Comment," and "Share" reactions, be The Communications Decency Act of 1996 (CDA), U.S.C.S. § 223,
guilty of aiding or abetting libel? And, in the complex world of cyberspace presents a great threat of censoring speech that, in fact, falls outside the
expressions of thoughts, when will one be liable for aiding or abetting statute's scope. Given the vague contours of the coverage of the statute,
cybercrimes? Where is the venue of the crime? it unquestionably silences some speakers whose messages would be
entitled to constitutional protection. That danger provides further reason
Except for the original author of the assailed statement, the rest (those for insisting that the statute not be overly broad. The CDA’s burden on
who pressed Like, Comment and Share) are essentially knee-jerk protected speech cannot be justified if it could be avoided by a more
sentiments of readers who may think little or haphazardly of their carefully drafted statute.
response to the original posting. Will they be liable for aiding or abetting?
And, considering the inherent impossibility of joining hundreds or Libel in the cyberspace can of course stain a person’s image with just one
thousands of responding "Friends" or "Followers" in the criminal charge to click of the mouse. Scurrilous statements can spread and travel fast across
be filed in court, who will make a choice as to who should go to jail for the the globe like bad news. Moreover, cyberlibel often goes hand in hand with
outbreak of the challenged posting? cyberbullying that oppresses the victim, his relatives, and friends, evoking
from mild to disastrous reactions. Still, a governmental purpose, which
The old parameters for enforcing the traditional form of libel would be a seeks to regulate the use of this cyberspace communication technology to
square peg in a round hole when applied to cyberspace libel. Unless the protect a person’s reputation and peace of mind, cannot adopt means that
legislature crafts a cyber libel law that takes into account its unique will unnecessarily and broadly sweep, invading the area of protected
circumstances and culture, such law will tend to create a chilling effect on freedoms.
the millions that use this new medium of communication in violation of
their constitutionally-guaranteed right to freedom of expression. If such means are adopted, self-inhibition borne of fear of what sinister
predicaments await internet users will suppress otherwise robust
The United States Supreme Court faced the same issue in Reno v. discussion of public issues. Democracy will be threatened and with it, all
American Civil Liberties Union, a case involving the constitutionality of the liberties. Penal laws should provide reasonably clear guidelines for law
Communications Decency Act of 1996. The law prohibited (1) the knowing enforcement officials and triers of facts to prevent arbitrary and
transmission, by means of a telecommunications device, of "obscene or discriminatory enforcement. The terms "aiding or abetting" constitute
indecent" communications to any recipient under 18 years of age; and (2) broad sweep that generates chilling effect on those who express
the knowing use of an interactive computer service to send to a specific themselves through cyberspace posts, comments, and other messages.
person or persons under 18 years of age or to display in a manner Hence, Section 5 of the cybercrime law that punishes "aiding or abetting"
available to a person under 18 years of age communications that, in libel on the cyberspace is a nullity.
context, depict or describe, in terms "patently offensive" as measured by
contemporary community standards, sexual or excretory activities or When a penal statute encroaches upon the freedom of speech, a facial
organs. challenge grounded on the void-for-vagueness doctrine is acceptable. The
inapplicability of the doctrine must be carefully delineated. As Justice
Those who challenged the Act claim that the law violated the First Antonio T. Carpio explained in his dissent in Romualdez v. Commission on
Amendment’s guarantee of freedom of speech for being overbroad. The Elections, "we must view these statements of the Court on the
U.S. Supreme Court agreed and ruled: inapplicability of the overbreadth and vagueness doctrines to penal

M.R.A.D.C. LUMBRE 122


CONSTITUTIONAL LAW REVIEW

statutes as appropriate only insofar as these doctrines are used to mount in the American law on child pornography that the Cybercrimes law lacks—
‘facial’ challenges to penal statutes not involving free speech." the exemption of a provider or notably a plain user of interactive computer
service from civil liability for child pornography as follows:
In an "as applied" challenge, the petitioner who claims a violation of his
constitutional right can raise any constitutional ground – absence of due No provider or user of an interactive computer service shall be treated as
process, lack of fair notice, lack of ascertainable standards, overbreadth, the publisher or speaker of any information provided by another
or vagueness. Here, one can challenge the constitutionality of a statute information content provider and cannot be held civilly liable for any action
only if he asserts a violation of his own rights. It prohibits one from voluntarily taken in good faith to restrict access to or availability of
assailing the constitutionality of the statute based solely on the violation material that the provider or user considers to be obscene...whether or
of the rights of third persons not before the court. This rule is also known not such material is constitutionally protected.69
as the prohibition against third-party standing.
When a person replies to a Tweet containing child pornography, he
But this rule admits of exceptions. A petitioner may for instance mount a effectively republishes it whether wittingly or unwittingly. Does this make
"facial" challenge to the constitutionality of a statute even if he claims no him a willing accomplice to the distribution of child pornography? When a
violation of his own rights under the assailed statute where it involves free user downloads the Facebook mobile application, the user may give
speech on grounds of overbreadth or vagueness of the statute. consent to Facebook to access his contact details. In this way, certain
information is forwarded to third parties and unsolicited commercial
The rationale for this exception is to counter the "chilling effect" on communication could be disseminated on the basis of this information.70
protected speech that comes from statutes violating free speech. A person As the source of this information, is the user aiding the distribution of this
who does not know whether his speech constitutes a crime under an communication? The legislature needs to address this clearly to relieve
overbroad or vague law may simply restrain himself from speaking in users of annoying fear of possible criminal prosecution.
order to avoid being charged of a crime. The overbroad or vague law thus
chills him into silence. Section 5 with respect to Section 4(c)(4) is unconstitutional. Its vagueness
raises apprehension on the part of internet users because of its obvious
As already stated, the cyberspace is an incomparable, pervasive medium chilling effect on the freedom of expression, especially since the crime of
of communication. It is inevitable that any government threat of aiding or abetting ensnares all the actors in the cyberspace front in a fuzzy
punishment regarding certain uses of the medium creates a chilling effect way. What is more, as the petitioners point out, formal crimes such as
on the constitutionally-protected freedom of expression of the great libel are not punishable unless consummated. In the absence of legislation
masses that use it. In this case, the particularly complex web of interaction tracing the interaction of netizens and their level of responsibility such as
on social media websites would give law enforcers such latitude that they in other countries, Section 5, in relation to Section 4(c)(4) on Libel,
could arbitrarily or selectively enforce the law. Section 4(c)(3) on Unsolicited Commercial Communications, and Section
Who is to decide when to prosecute persons who boost the visibility of a 4(c)(2) on Child Pornography, cannot stand scrutiny.
posting on the internet by liking it? Netizens are not given "fair notice" or Thus, Section 5 that penalizes aiding or abetting and attempt in the
warning as to what is criminal conduct and what is lawful conduct. When commission of cybercrimes as VA L I D and CONSTITUTIONAL only in
a case is filed, how will the court ascertain whether or not one netizen’s relation to Section 4(a)(1) on Illegal Access, Section 4(a)(2) on Illegal
comment aided and abetted a cybercrime while another comment did not? Interception, Section 4(a)(3) on Data Interference, Section 4(a)(4) on
Of course, if the "Comment" does not merely react to the original posting System
but creates an altogether new defamatory story against Armand like "He c. Imbong vs. Ochoa, G.R. NO. 204819, April 8, 2014
beats his wife and children," then that should be considered an original
posting published on the internet. Both the penal code and the cybercrime RULING: The OSG also assails the propriety of the facial challenge lodged
law clearly punish authors of defamatory publications. Make no mistake, by the subject petitions, contending that the RH Law cannot be challenged
libel destroys reputations that society values. Allowed to cascade in the "on its face" as it is not a speech regulating measure.
internet, it will destroy relationships and, under certain circumstances, will
generate enmity and tension between social or economic groups, races, The Court is not persuaded.
or religions, exacerbating existing tension in their relationships. In United States (US) constitutional law, a facial challenge, also known as
In regard to the crime that targets child pornography, when "Google a First Amendment Challenge, is one that is launched to assail the validity
procures, stores, and indexes child pornography and facilitates the of statutes concerning not only protected speech, but also all other rights
completion of transactions involving the dissemination of child in the First Amendment.106 These include religious freedom, freedom of
pornography," does this make Google and its users aiders and abettors in the press, and the right of the people to peaceably assemble, and to
the commission of child pornography crimes?68 Byars highlights a feature petition the Government for a redress of grievances.107 After all, the

M.R.A.D.C. LUMBRE 123


CONSTITUTIONAL LAW REVIEW

fundamental right to religious freedom, freedom of the press and peaceful or detailed standards on how law enforcers should apprehend and properly
assembly are but component rights of the right to one's freedom of determine the age of the alleged curfew violators. They further argue that
expression, as they are modes which one's thoughts are externalized. the law enforcer's apprehension depends only on his physical assessment,
and, thus, subjective and based only on the law enforcer's visual
In this jurisdiction, the application of doctrines originating from the U.S. assessment of the alleged curfew violator, although the lists of exemptions
has been generally maintained, albeit with some modifications. While this do not cover the range and breadth of legitimate activities or reasons as
Court has withheld the application of facial challenges to strictly penal to why minors would be out at night, and, hence, proscribe or impair the
statues, it has expanded its scope to cover statutes not only regulating legitimate activities of minors during curfew hours.
free speech, but also those involving religious freedom, and other
fundamental rights. The underlying reason for this modification is simple. ISSUE: Whether or not the curfew ordinances suffer from overbreadth by
For unlike its counterpart in the U.S., this Court, under its expanded proscribing or impairing legitimate activities of minors during curfew
jurisdiction, is mandated by the Fundamental Law not only to settle actual hours.
controversies involving rights which are legally demandable and
enforceable, but also to determine whether or not there has been a grave RULING: The court in such issue rejects petitioners' invocation of the
abuse of discretion amounting to lack or excess of jurisdiction on the part overbreadth doctrine, considering that petitioners have not claimed any
of any branch or instrumentality of the Government. Verily, the framers transgression of their rights to free speech or any inhibition of speech-
of Our Constitution envisioned a proactive Judiciary, ever vigilant with its related conduct. The application of the overbreadth doctrine is limited to
duty to maintain the supremacy of the Constitution. a facial kind of challenge and, owing to the given rationale of a facial
challenge, applicable only to free speech cases.
Consequently, considering that the foregoing petitions have seriously
alleged that the constitutional human rights to life, speech and religion Ordinarily, a particular litigant claims that a statute is
and other fundamental rights mentioned above have been violated by the unconstitutional as applied to him or her; if the litigant prevails,
assailed legislation, the Court has authority to take cognizance of these the courts carve away the unconstitutional aspects of the law by
kindred petitions and to determine if the RH Law can indeed pass invalidating its improper applications on a case to case basis.
constitutional scrutiny. To dismiss these petitions on the simple expedient Moreover, challengers to a law are not permitted to raise the rights
that there exist no actual case or controversy, would diminish this Court of third parties and can only assert their own interests. In
as a reactive branch of government, acting only when the Fundamental overbreadth analysis, those rules give way; challenges are
Law has been transgressed, to the detriment of the Filipino people. permitted to raise the rights of third parties; and the court
invalidates the entire statute "on its face," not merely "as applied
xxx for" so that the overbroad law becomes unenforceable until a
properly authorized court construes it more narrowly.
In relation to locus standi, the "as applied challenge" embodies the rule
that one can challenge the constitutionality of a statute only if he asserts The factor that motivates courts to depart from the normal adjudicatory
a violation of his own rights. The rule prohibits one from challenging the rules is the concern with the "chilling;" deterrent effect of the overbroad
constitutionality of the statute grounded on a violation of the rights of third statute on third parties not courageous enough to bring suit. The Court
persons not before the court. This rule is also known as the prohibition assumes that an overbroad law's "very existence may cause others not
against third-party standing. before the court to refrain from constitutionally protected speech or
expression. facial challenges are raised on the basis of overbreadth and
d. Samahan ng mga Progresibong Kabataan (SPARK) vs. Quezon limited to the realm of freedom of expression.
City, G.R. No. 225442, 8 August 2017
Thus, the Court finds it improper to undertake an overbreadth analysis in
FACTS: President Duterte implemented a nationwide curfew for minors. this case, there being no claimed curtailment of free speech.
Several LGUs implemented curfew ordinances through police operations
known as Oplan Rody, including Manila, Quezon City and Navotas, which The Court though, finds it proper to examine the assailed regulations
are the subject of this petition. The Ordinances declared curfew on hours under the strict scrutiny test.
between 10 pm - 4 am for children below 18 years old. Petitioners SPARK
filed the petition arguing among others that that the Curfew Ordinances The Court finds that all three Curfew Ordinances have passed the first
are unconstitutional because they: (b) Suffer from overbreadth by prong of the strict scrutiny test - that is, that the State has sufficiently
proscribing or impairing legitimate activities of minors during curfew shown a compelling interest to promote juvenile safety and prevent
hours. juvenile crime in the concerned localities, only the Quezon City Ordinance
has passed the second prong of the strict scrutiny test, as it is the only
More specifically, petitioners posit that the Curfew Ordinances encourage issuance out of the three which provides for the least restrictive means to
arbitrary and discriminatory enforcement as there are no clear provisions achieve this interest.

M.R.A.D.C. LUMBRE 124


CONSTITUTIONAL LAW REVIEW

In fine, the Manila and Navotas Ordinances are declared unconstitutional RULING: This simply means that lumber is a processed log or processed forest
and thus, null and void, while the Quezon City Ordinance is declared as raw material. Clearly, the Code uses the term lumber in its ordinary or common
constitutional and thus, valid in accordance with this Decision usage. In the 1993 copyright edition of Webster's Third New International
Dictionary, lumber is defined, inter alia, as "timber or logs after being prepared for
iii. Earliest opportunity the market." Simply put, lumber is a processed log or timber.
Constitutional question must be raised at the earliest possible opportunity. It must It is settled that in the absence of legislative intent to the contrary, words and
be raised in the pleadings. phrases used in a statute should be given their plain, ordinary, and common usage
- Exceptions meaning. And insofar as possession of timber without the required legal documents
is concerned, Section 68 of P.D. No. 705, as amended, makes no distinction
Criminal case – It may be brought at any stage of the proceedings according to the between raw or processed timber. Neither should we. Ubi lex non distinguit nec nos
discretion of the judge (trial or appeal) because no one shall be brought within the distinguere debemus.
terms of the law who are not clearly within them and the act shall not be punished
when the law does not clearly punish them. To exclude possession of "lumber" from the acts penalized in Sec. 68 would
certainly emasculate the law itself. A law should not be so construed as to allow
Civil case – It may be brought anytime if the resolution of the constitutional issue the doing of an act which is prohibited by law, nor so interpreted as to afford an
is inevitable in resolving the main issue. opportunity to defeat compliance with its terms, create an inconsistency, or
contravene the plain words of the law. After all, the phrase "forest products" is
When the jurisdiction of the lower court is in question except when there is
broad enough to encompass lumber which, to reiterate, is manufactured timber.
estoppel.
Hence, to mention lumber in Sec. 68 would merely result in tautology.
The earliest opportunity to raise a constitutional issue is to raise it in the pleadings
Even should it be conceded that lumber is not timber and is thus not covered by
before a competent court that can resolve the same, such that, if not raised in the
the prohibition, still it cannot be denied that lumber is a forest product and
pleadings, it cannot be considered in trial and, if not considered in trial, it cannot
possession thereof without legal documents is equally and, to the same extent,
be considered on appeal.
prohibited.
The Ombudsman has no jurisdiction to entertain questions regarding
With respect to the constitutionality of Sec. 68 of P.D. No. 705 which petitioner
constitutionality of laws. Thus, when the issue of constitutionality of a law was
would have this Court consider, this Court has always desisted from delving on
raised before the Court of Appeals, which is the competent court, the constitutional
constitutional issues. Thus, even if all the requisites for judicial review of a
question was raised at the earliest opportune time. (Estarija v. Ranada, G.R. No.
constitutional matter are present in a case, this Court will not pass upon a
159314, June 26, 2006)
constitutional question unless it is the lis mota of the case or if the case
iv. The “lis mota” of the case / necessity of deciding constitutional can be disposed of on some other grounds, such as the application of the
question statute or general law.

1) Lalican vs. Vergara, 276 SCRA 518 (1997) 2) Doctrine of Purposeful Hesitation

FACTS: An information for illegal possession of "timber" as defined in Sec. 68 of - Drilon v. Lim, GR 112497, Aug. 4, 1994
Presidential Decree No. 705 was filed by the City Prosecutor of Puerto Princesa City
ISSUE: Whether the lower court has jurisdiction to rule on the constitutionality of
against petitioners for having in their possession, custody and control 1,800 board
Section 187 of the Local Government Code.
feet of assorted species and dimensions of lumber on board two (2) passenger
jeeps, with a value of P14,000.00. RULING: BP 129 vests in the regional trial courts jurisdiction over all civil cases in
which the subject of the litigation is incapable of pecuniary estimation. Moreover,
On August 23, 1991, petitioner Lalican filed a motion to quash the information on
Article X, Section 5(2), of the Constitution vests in the Supreme Court appellate
the ground that the facts charged did not constitute an offense. Contending that
jurisdiction over final judgments and orders of lower courts in all cases in which the
Sec. 68 of P.D. No. 705 refers to "timber and other forest products" and not to
constitutionality or validity of any treaty, international or executive agreement, law,
"lumber," and asserting that "timber" becomes "lumber" only after it is sawed into
presidential decree, proclamation, order, instruction, ordinance, or regulation is in
beams, planks or boards, petitioner alleged that said decree "does not apply to
question.
'lumber.'" He added that the law is "vague and standardless" as it does not specify
the authority or the legal documents required by existing forest laws and In the exercise of this jurisdiction, lower courts are advised to act with the utmost
regulations. Hence, petitioner asserted that the information should be quashed as circumspection, bearing in mind the consequences of a declaration of
it violated his constitutional rights to due process and equal protection of the law. unconstitutionality upon the stability of laws, no less than on the doctrine of
separation of powers. As the questioned act is usually the handiwork of the
legislative or the executive departments, or both, it will be prudent for such courts,

M.R.A.D.C. LUMBRE 125


CONSTITUTIONAL LAW REVIEW

if only out of a becoming modesty, to defer to the higher judgment of the Supreme The strict view considers a legislative enactment which is declared
Court in the consideration of its validity, which is better determined after a thorough unconstitutional as being, for all legal intents and purposes, a total nullity, and it is
deliberation by a collegiate body and with the concurrence of the majority of those deemed as if had never existed. Here, of course, we refer to the law itself being
who participated in its discussion. per se repugnant to the Constitution. It is not always the case, however, that a law
is constitutionally faulty per se. Thus, it may well be valid in its general import, but
It is also emphasized that every court, including the Supreme Court, is charged invalid in its application to certain factual situations. To exemplify, an otherwise
with the duty of a purposeful hesitation before declaring a law unconstitutional, on valid law may be held unconstitutional only insofar as it is allowed to operate
the theory that the measure was first carefully studied by the executive and the retrospectively such as, in pertinent cases, when it vitiates contractually vested
legislative departments and determined by them to be in accordance with the rights. To that extent, its retroactive application may be so declared invalid as
fundamental law before it was finally approved. To doubt is to sustain, The impairing the obligations of contracts.
presumption of the constitutionality can be overcome only by the clearest showing
that there was indeed an infraction of the Constitution, and only when such a A judicial declaration of invalidity, it is also true, may not necessarily obliterate all
conclusion is reached by the required majority may the Court pronounce, in the the effects and consequences of a void act occurring prior to such a declaration.
discharge of the duty it cannot escape, that the challenged act must be struck Thus, in our decisions on the moratorium laws, we have been constrained to
down. recognize the interim effects of said laws prior to their declaration of
unconstitutionality, but there we have likewise been unable to simply ignore strong
3) Doctrine of Operative Fact considerations of equity and fair play. So also, even as a practical matter, a
- Republic v. CA, GR 79732, Nov. 8, 1993 situation that may aptly be described as fait accompli may no longer be open for
further inquiry, let alone to be unsettled by a subsequent declaration of nullity of a
ISSUE: Whether the declaration of nullity of the law should have prospective, not governing statute.
retroactive, application.
The instant controversy, however, is too far distant away from any of the above
RULING: Yes. Instruction is the brief treatise made by Mr. Justice Isagani A. Cruz, exceptional cases. To this day, the controversy between the petitioner and the
whose words we quote — private respondents on the issue of just compensation is still unresolved, partly
attributable to the instant petition that has prevented the finality of the decision
There are two views on the effects of a declaration of the unconstitutionality of a
appealed from. The fact of the matter is that the expropriation cases, involved in
statute.
this instance, were still pending appeal when the EPZA ruling was rendered and
The first is the orthodox view. Under this rule, as announced in Norton v. Shelby, forthwith invoked by said parties.
an unconstitutional act is not a law; it confers no right; it imposes no duties; it
2. Safeguards of Judicial Independence
affords no protection; it creates no office; it is, in legal contemplation, inoperative,
as if it had not been passed. It is therefore stricken from the statute books and a. Constitutional safeguards
considered never to have existed at all. Not only the parties but all persons are
bound by the declaration of unconstitutionality, which means that no one may - Art. VIII, Secs. 3, 4(1), 9(last sentence of 1st par.), 11(1st sentence),
thereafter invoke it nor may the courts be permitted to apply it in subsequent cases. and 12;
It is, in other words, a total nullity. - Art. XI, Sec. 2;
The second or modern view is less stringent. Under this view, the court in passing - Art. VIII, Secs. 10, 11(2nd sentence), 5(5, 6 and 3), and 6;
upon the question of constitutionality does not annul or repeal the statute if it finds
it in conflict with the Constitution. It simply refuses to recognize it and determines - Art. VIII, Sec. 2; Art. VI, Sec. 30
the rights of the parties just as if such statute had no existence. The court may
give its reasons for ignoring or disregarding the law, but the decision affects the The judiciary shall enjoy fiscal autonomy. (Art. VIII, Sec. 3.)
parties only and there is no judgment against the statute. The opinion or reasons The SC is a constitutional body. It cannot be abolished nor may its membership
of the court may operate as a precedent for the determination of other similar or the manner of its meetings be changed by mere legislation. [Art. VIII, Sec. 4
cases, but it does not strike the statute from the statute books; it does not repeal, (1)]
supersede, revoke, or annul the statute. The parties to the suit are concluded by
the judgment, but no one else is bound. Such appointments need no confirmation. [Art. VIII, Sec. 9, last sentence of
first par.]
The orthodox view is expressed in Article 7 of the Civil Code, providing that "when
the courts declare a law to be inconsistent with the Constitution, the former shall The Members of the Supreme Court and judges of the lower court shall hold office
be void and the latter shall govern. during good behavior until they reach the age of seventy years or become
incapacitated to discharge the duties of their office. [Art. VIII, Sec. 11 (1st
sentence)]

M.R.A.D.C. LUMBRE 126


CONSTITUTIONAL LAW REVIEW

The SC has exclusive power to discipline judges of lower courts. (Art. VIII, Sec. The COA avers that the court should have applied the formula found in COA
11., last sentence of 1st par.) Memorandum No. 98-569-A. Instead of applying Constitutional Fiscal
Autonomy Group (CFAG) Joint Resolution No. 35 and its guidelines, in
They shall not be designated to any agency performing quasi-judicial or compliance with the Resolution of the Court En Banc in A.M. No. 03-12-01.
administrative functions. (Art. VIII, Sec. 12.)
In her Memorandum, Atty. Candelaria, from the Office of Administrative
The members of the judiciary are not subject to confirmation by the CA. Services, recommended that the Court advise the COA to respect the in-house
The members of the SC may not be removed except by impeachment. (Art. IX, computation based on the CFAG formula, noting that this was the first time
Sec. 2.) that the COA questioned the authority of the Court in using CFAG Joint
Resolution No. 35 and its guidelines in the appraisal and disposal of
The President, the Vice-President, the Members of the Supreme Court, the government property since these were issued in 1997.
Members of the Constitutional Commissions, and the Ombudsman may be
removed from office on impeachment for, and conviction of, culpable violation of RULING: The opinion of the COA violates the fiscal autonomy and
the Constitution, treason, bribery, graft and corruption, other high crimes, or independence of the Judiciary.
betrayal of public trust. All other public officers and employees may be removed The COA's authority to conduct post-audit examinations on constitutional
from office as provided by law, but not by impeachment. (Art. XI, Sec. 2.) bodies granted fiscal autonomy is provided under Section 2(1), Article IX-D of
The salaries of judges may not be reduced during their continuance in office. (Art. the 1987 Constitution. This authority, however, must be read not only in light
VIII, Sec. 10.) of the Court's fiscal autonomy, but also in relation with the constitutional
provisions on judicial independence and the existing jurisprudence and Court
The Supreme Court en banc shall have the power to discipline judges of lower rulings on these matters.
courts, or order their dismissal by a vote of majority of the Members who actually
took part in the deliberations on the issues in the case and voted in thereon. (Art. Judicial independence can be "broken down into two distinct concepts:
VIII, Sec. 11, 2nd sentence.) Decisional independence "refers to a judge’s ability to render decisions free
from political or popular influence based solely on the individual facts and
The SC alone may initiate rules of court. [Art. VIII, Sec. 5 (5).] applicable law." On the other hand, institutional independence "describes the
separation of the judicial branch from the executive and legislative branches
The SC can appoint all officials and employees of the judiciary. [Art. VIII, Sec.
of government."
5 (6)]
One of the most important aspects of judicial independence is the
Only the SC may order the temporary detail of judges. [Art. VIII, Sec. 5(3)]
constitutional grant of fiscal autonomy. While, as a general proposition, the
The SC now has administrative supervision over all lower courts and their authority of legislatures to control the purse in the first instance is
personnel. (Art. VIII, Sec. 6.) unquestioned, any form of interference by the Legislative or the Executive on
the Judiciary’s fiscal autonomy amounts to an improper check on a co-equal
The SC may not be deprived of its minimum original and appellate jurisdiction as branch of government. If the judicial branch is to perform its primary function
prescribed in Art. X, Sec. 5 of the Constitution. (Art. VIII, Sec. 2.) of adjudication, it must be able to command adequate resources for that
The appellate jurisdiction of the SC may not be increased by law without its advice purpose.
and concurrence. (Art. VI, Sec. 30.) The use of the formula provided in CFAG Joint Resolution No. 35 is a part of
The members of the SC and all lower courts have security of tenure, which cannot the Court’s exercise of its discretionary authority to determine the manner the
be undermined by a law reorganizing the judiciary. (Id.) granted retirement privileges and benefits can be availed of. Any kind of
interference on how these retirement privileges and benefits are exercised and
i. Re: COA Opinion on the computation of the appraised value of the availed of, not only violates the fiscal autonomy and independence of the
properties purchased by the retired chief/associate justices of the Judiciary, but also encroaches upon the constitutional duty and privilege of the
Supreme Court, AM No. 11-7-10-SC, 31 July 2012 Chief Justice and the Supreme Court En Banc to manage the Judiciary’s own
affairs.
FACTS: On June 8, 2010, the Legal Services Sector, Office of the General
Counsel of the COA issued an opinion which found that an underpayment ii. In Re First Indorsement from Honorable Raul M. Gonzales dated 16
amounting to P221,021.50 resulted when five (5) retired Supreme Court March 1988 Requesting Honorable Justice Marcelo B. Fernan to
justices purchased from the Supreme Court the personal properties assigned Comment on an Anonymous Letter-Complaint, A.M. Nos. 88-4-54333,
to them during their incumbency in the Court. The COA attributed this 15 April 1988
underpayment to the use by the Property Division of the Supreme Court of the
wrong formula in computing the appraisal value of the purchased vehicles.

M.R.A.D.C. LUMBRE 127


CONSTITUTIONAL LAW REVIEW

ISSUE: Whether a Supreme Court justice can be disbarred during his term of investigation being conducted by the Ombudsman encroaches into the Court's
office. power of administrative supervision over all courts and its personnel, in
violation of the doctrine of separation of powers.
RULING: A public officer (such as Justice Fernan) who under the Constitution
is required to be a Member of the Philippine Bar as a qualification for the office Article VIII, section 6 of the 1987 Constitution exclusively vests in the Supreme
held by him and who may be removed from office only by impeachment, cannot Court administrative supervision over all courts and court personnel, from the
be charged with disbarment during the incumbency of such public officer. Presiding Justice of the Court of Appeals down to the lowest municipal trial
Further, such public officer, during his incumbency, cannot be charged court clerk. By virtue of this power, it is only the Supreme Court that can
criminally before the Sandiganbayan, or any other court, with any offense oversee the judges' and court personnel's compliance with all laws, and take
which carries with it the penalty of removal from office. the proper administrative action against them if they commit any violation
thereof. No other branch of government may intrude into this power, without
Another reason why the complaint for disbarment should be dismissed is running afoul of the doctrine of separation of powers.
because under the Constitution, members of the SC may be removed only by
impeachment. The above provision proscribes removal from office by any other The Ombudsman cannot justify its investigation of petitioner on the powers
method. Otherwise, to allow such public officer who may be removed solely by granted to it by the Constitution, for such a justification not only runs counter
impeachment to be charged criminally while holding his office with an office to the specific mandate of the Constitution granting supervisory powers to the
that carries the penalty of removal from office, would be violative of the clear Supreme Court over all courts and their personnel, but likewise undermines
mandate of the Constitution. the independence of the judiciary.

The effect of impeachment is limited to the loss of position and disqualification Thus, the Ombudsman should first refer the matter of petitioner's certificates
to hold any office of honor, trust or profit under the Republic. Judgment in of service to this Court for determination of whether said certificates reflected
cases of impeachment shall not extend further than removal from office and the true status of his pending case load, as the Court has the necessary records
disqualification to hold any office. But the party convicted shall nevertheless to make such a determination. The Ombudsman cannot compel this Court, as
be held liable and subject to prosecution, trial and punishment according to one of the three branches of government, to submit its records, or to allow its
law. personnel to testify on this matter, as suggested by public respondent Abiera
in his affidavit-complaint.
The court is not saying that its Members or other constitutional officers are
entitled to immunity from liability for possibly criminal acts or for alleged The rationale for the foregoing pronouncement is evident in this case.
violation of the Canons of Judicial Ethics or other supposed misbehavior. What Administratively, the question before the Court is this: should a judge, having
the court is saying is that there is a fundamental procedural requirement that been granted by this Court an extension of time to decide cases before him,
must be observed before such liability may be determined and enforced. A report these cases in his certificate of service? As this question had not yet
member of the Supreme Court must first be removed from office, via the been raised with, much less resolved by this Court, how could the Ombudsman
constitutional route of impeachment, and then only may he be held liable either resolve the present criminal complaint that requires the resolution of said
criminally or administratively (that is, disbarment), for any wrong or question?
misbehavior that may be proven against him in appropriate proceedings.
In fine, where a criminal complaint against a Judge or other court employee
iii. Maceda vs. Vasquez, 221 SCRA 464 (1993) arises from their administrative duties, the Ombudsman must defer action on
said complaint and refer the same to this Court for determination whether said
ISSUE: Whether the Office of the Ombudsman could entertain a criminal Judge or court employee had acted within the scope of their administrative
complaint for the alleged falsification of a judge’s certification submitted to the duties.
Supreme Court, and assuming it can, whether a referral should be made first
to the Supreme Court. iv. De Vera vs. Pelayo, 335 SCRA 281, 6 July 2000
RULING: No. ISSUE[S]: WON the Ombudsman has jurisdiction to entertain criminal
charges filed against a judge of the regional trial court in connection with his
As to petitioner’s first contention, the Court disagreed, stating that there is handling of cases before the court.
nothing in the Orap case that would restrict it only to offenses committed by a
judge unrelated to his official duties. A judge who falsifies his certificate of RULING: No. The Ombudsman did not exercise his power in an arbitrary or
service is administratively liable to the Supreme Court for serious misconduct despotic manner by reason of passion, prejudice or personal hostility when it
and inefficiency under Section 1, Rule 140 of the Rules of Court, and criminally referred the case to the Supreme Court.
liable to the State under the Revised Penal Code for his felonious act.
The issues have been settled in the case of In Re: Joaquin Borromeo. There,
The Court, however, agreed that in the absence of any administrative action the SC laid down the rule that before a civil or criminal action against a judge
taken against him by this Court with regard to his certificates of service, the for a violation of Art. 204 and 205 (knowingly rendering an unjust judgment

M.R.A.D.C. LUMBRE 128


CONSTITUTIONAL LAW REVIEW

or order) can be entertained, there must first be "a final and authoritative ISSUE: WON the first and second paragraphs of Sec. 14 of R.A. No. 6770 are
judicial declaration" that the decision or order in question is indeed "unjust." unconstitutional and violative of the the principle of judicial independence?
The pronouncement may result from either:
RULING: The first paragraph is declared INEFFECTIVE until the Court adopts
a. An action for certiorari or prohibition in a higher court impugning the the same as part of the rules of procedure through an administrative circular
validity of the judgment or duly issued; the second paragraph is declared UNCONSTITUTIONAL AND
INVALID.
b. An administrative proceeding in the Supreme Court against the judge
precisely for promulgating an unjust judgment or order. The Court ruled that when Congress passed the first paragraph of Section 14,
RA 6770 and, in so doing, took away from the courts their power to issue a
Likewise, the determination of whether a judge has maliciously delayed the TRO and/or WPI to enjoin an investigation conducted by the Ombudsman, it
disposition of the case is also an exclusive judicial function. encroached upon this Court’s constitutional rule-making authority. Through
"To repeat, no other entity or official of the Government, not the prosecution this provision, Congress interfered with a provisional remedy that was created
or investigation service of any other branch, not any functionary thereof, has by this Court under its duly promulgated rules of procedure, which utility is
competence to review a judicial order or decision -- whether final and both integral and inherent to every court’s exercise of judicial power. Without
executory or not -- and pronounce it erroneous so as to lay the basis for a the Court’s consent to the proscription, as may be manifested by an adoption
criminal or administrative complaint for rendering an unjust judgment or order. of the same as part of the rules of procedure through an administrative circular
That prerogative belongs to the courts alone.” issued therefor, there thus, stands to be a violation of the separation of powers
principle.
v. Ampong vs. CSC, GR No. 167916, 26 August 2008
In addition, it should be pointed out that the breach of Congress in prohibiting
ISSUE: Whether the CSC has administrative jurisdiction over an employee of provisional injunctions, such as in the first paragraph of Section 14, RA 6770,
the Judiciary for acts committed while said employee was still with the does not only undermine the constitutional allocation of powers; it also
Executive or Education Department practically dilutes a court’s ability to carry out its functions. This is so since a
particular case can easily be mooted by supervening events if no provisional
RULING: The Court held that CSC has no administrative jurisdiction but ruled
injunctive relief is extended while the court is hearing the same.
against the petition by reason on the ground of estoppel.
Since the second paragraph of Section 14, RA 6770 limits the remedy against
While the CSC has the administrative jurisdiction over civil service, including
“decision or findings” of the Ombudsman to a Rule 45 appeal and thus – similar
irregularity and anomaly with examinations; however, the Constitution
to the fourth paragraph of Section 27, RA 6770- attempts to effectively
provides that the Supreme Court is given exclusive administrative supervision
increase the Supreme Court’s appellate jurisdiction without its advice and
over all courts and judicial personnel. By virtue of this power, it is only the
concurrence, it is therefore concluded that the former provision is also
Supreme Court that can oversee the judges and court personnel’s compliance
unconstitutional and perforce, invalid. Contrary to the Ombudsman’s
with all laws, rules and regulations. It may take the proper administrative
posturing, Fabian should squarely apply since the above-stated Ombudsman
action against them if they commit any violation. No other branch of
Act provisions are in part material in that they “cover the same specific or
government may intrude into this power, without running afoul of the doctrine
particular subject matter,” that is, the manner of judicial review over issuances
of separation of powers.
of the Ombudsman.
However, the Court was constrained to uphold the ruling of the CSC based on
As may be deduced from the various discourses in Gonzales III, the concept
the principle of estoppel. The Court ruled that Ampong was estopped from
of Ombudsman's independence covers three (3) things:
attacking CSC’s jurisdiction since apart from her full participation in the
proceedings before the CSC, petitioner admitted to the offense charged that First: creation by the Constitution, which means that the office cannot be
she impersonated Decir and took the PBET exam in the latter’s place. The Court abolished, nor its constitutionally specified functions and privileges, be
also held that while a party’s right to the assistance of counsel is sacred in removed, altered, or modified by law, unless the Constitution itself allows,
proceedings criminal in nature, there is no such requirement in administrative or an amendment thereto is made;
proceedings. Emphasizing on the non-tolerance of dishonesty in the Judiciary,
the Court pointed out being a judicial employee was not a hindrance for Second: fiscal autonomy, which means that the office "may not be
Ampong to get the penalty she deserved and that had the Court been aware obstructed from [its] freedom to use or dispose of [its] funds for purposes
of her dishonesty, she would not been appointed as a court interpreter. germane to [its] functions; hence, its budget cannot be strategically
decreased by officials of the political branches of government so as to
vi. Conchita Carpio Morales vs. CA and Jejomar Erwin S. Binay, Sr., G.R. impair said functions; and
Nos. 217126-27, 10 November 2015

M.R.A.D.C. LUMBRE 129


CONSTITUTIONAL LAW REVIEW

Third: insulation from executive supervision and control, which means that that popular trust so essential to the maintenance of their vigor as champions
those within the ranks of the office can only be disciplined by an internal of justice." Hence, our Constitutions continuously vested this power to this
authority. Court for it enhances its independence.
Evidently, all three aspects of independence intend to protect the Office of the To reiterate, the Court's authority to promulgate rules on pleading,
Ombudsman from political harassment and pressure, so as to free it from the practice, and procedure is exclusive and one of the safeguards of plea
"insidious tentacles of politics." bargaining, as a rule and a practice, has been existing in our jurisdiction since
July 1, 1940, when the 1940 Rules took effect.
That being the case, the concept of Ombudsman independence cannot be
invoked as basis to insulate the Ombudsman from judicial power NOTE: Plea-bargaining is a procedural matter, within the Constitutional
constitutionally vested unto the courts. Courts are apolitical bodies, which are mandate of the Supreme Court to promulgate rules of procedure. In May 2018,
ordained to act as impartial tribunals and apply even justice to all. Hence, the pursuant to its ruling in Lobrigo, the Supreme Court passed an administrative
Ombudsman's notion that it can be exempt from an incident of judicial power issuance re: plea-bargaining in R.A. No. 9165 cases.
- that is, a provisional writ of injunction against a preventive suspension order
- clearly strays from the concept's rationale of insulating the office from viii. Mamiscal vs. Clerk of Court Macalinog S. Abdullah, A.M. No. SCC-
political harassment or pressure. 13-18-J, 1 July 2015

NOTE: Procedural matters are always within the ambit of the powers of the FACTS: Abdullah denied Mamiscal's motion to revoke the Certificate of
Supreme Court. Registration finalizing the divorce between the latter and his wife, Adelaidah.
In sustaining the divorce between Mamiscal and Abdullah, Abdullah opined that
vii. Estipona vs. Lobrigo, G.R. 226679, 15 August 2017 it was simply his ministerial duty to receive the COD and the attached
kapasadan filed by Adelaidah. Abdullah also noted that when the AAC was
ISSUE: Whether or not Section 23 of RA No. 9165 is unconstitutional as it convened during the February 28, 2010 hearing, only Mamiscal and his
encroached upon the power of the Supreme Court to promulgate rules of representatives appeared. Considering the fact that Adelaidah manifested her
procedure. opposition in writing to any reconciliation with her husband and the fact that
RULING: Yes. Section 23 of Republic Act No. 9165 is declared the 90-day period of 'iddah had already lapsed, Abdullah ruled that any move
UNCONSTITUTIONAL for being contrary to the rule-making authority of to reconstitute the AAC would have been futile because the divorce between
the SC under Section 5(5), Article VIII of the 1987 Constitution which Mamiscal and his wife had already become final and irrevocable. Thus,
explicitly provides: complaint of Baguan M. Mamiscal (Mamiscal) against respondent Macalinog S.
Abdullah (Abdullah), Clerk of Court, Shari'a Circuit Court, Marawi City, for
Sec. 5. The Supreme Court shall have the following powers: partiality, violation of due process, dishonesty, and conduct unbecoming of a
court employee.
(5) Promulgate rules concerning the protection and enforcement of
constitutional rights, pleading, practice, and procedure in all courts, xxx RULING: No, the Court has no jurisdiction over the case. The Court does not
have jurisdiction to impose the proper disciplinary action against civil
The power to promulgate rules of pleading, practice and procedure is now our
registrars. While he is undoubtedly a member of the Judiciary as Clerk of Court
exclusive domain and no longer shared with the Executive and Legislative
of the Shari'a Circuit Court, a review of the subject complaint reveals that
departments. The rule-making power of this Court was expanded. This Court
Mamiscal seeks to hold Abdullah liable for registering the divorce and issuing
for the first time was given the power to promulgate rules concerning the
the CRD pursuant to his duties as Circuit Registrar of Muslim divorces. It has
protection and enforcement of constitutional rights. The Court was also
been said that the test of jurisdiction is the nature of the offense and
granted for the .first time the power to disapprove rules of procedure of
not the personality of the offender. The fact that the complaint charges
special courts and quasi-judicial bodies. But most importantly, the Constitution
Abdullah for "conduct unbecoming of a court employee" is of no moment. Well-
took away the power of Congress to repeal, alter, or supplement rules
settled is the rule that what controls is not the designation of the offense but
concerning pleading, practice and procedure. In fine, the power to
the actual facts recited in the complaint. Verily, unless jurisdiction has been
promulgate rules of pleading, practice and procedure is no longer shared
conferred by some legislative act, no court or tribunal can act on a matter
by this Court with Congress, more so with the Executive.
submitted to it.
In Echegaray v. Secretary of Justice, then Associate Justice (later Chief Justice)
In view of Article 81 of the Muslim Code, it becomes apparent that the Clerk
Reynato S. Puno traced the history of the Court's rule-making power and
of Court of the Shari'a Circuit Court enjoys the privilege of wearing two hats:
highlighted its evolution and development. x x x It should be stressed that the
first, as Clerk of Court of the Shari'a Circuit Court, and second, as Circuit
power to promulgate rules of pleading, practice and procedure was granted by
Registrar within his territorial jurisdiction. Although the Constitution vests the
our Constitutions to this Court to enhance its independence, for in the words
Court with the power of administrative supervision over all courts and its
of Justice Isagani Cruz "without independence and integrity, courts will lose
personnel, this power must be taken with due regard to other prevailing laws.

M.R.A.D.C. LUMBRE 130


CONSTITUTIONAL LAW REVIEW

Section 185 of the Muslim Code provides that neglect of duty by registrars 3. The Supreme Court
shall be penalized under Section 18 of C.A. No. 3753. The same Act also
provides that “all irregularities, negligence or incompetency on the part of the a. Composition, qualifications and vacancy (Art. VIII, Sec. 4[1] and Sec. 7)
officers designated as local civil registrars to the (Chief of the Executive Section 4. The Supreme Court shall be composed of a Chief Justice and fourteen
Bureau or the Director of the Non-Christian Tribes) Secretary of the Associate Justices. It may sit en banc or in its discretion, in division of three, five,
Interior, as the case may be, who shall take the proper disciplinary action or seven Members. Any vacancy shall be filled within ninety days from the
against the offenders.” occurrence thereof.
As to the question of which agency or officer having administrative supervisory Section 7. No person shall be appointed Member of the Supreme Court or any
power to impose disciplinary sanctions against erring civil registrars, the Court lower collegiate court unless he is a natural-born citizen of the Philippines. A
held that the same remains with the National Government. Member of the Supreme Court must be at least forty years of age, and must have
With the advent of the Local Government Code, the power of administrative been for fifteen years or more, a judge of a lower court or engaged in the practice
supervision over civil registrars was devolved to the municipal and city mayors of law in the Philippines.
of the respective local government units. Under the "faithful execution clause" The Congress shall prescribe the qualifications of judges of lower courts, but no
embodied in Section 455(b)(l)(x)35 and Section 444(b)(l)(x)36 of the Local person may be appointed judge thereof unless he is a citizen of the Philippines and
Government Code, in relation to Section 47937 under Article IX, Title V38 of a member of the Philippine Bar.
the same Code, the municipal and city mayors of the respective local
government units, in addition to their power to appoint city or municipal civil A Member of the Judiciary must be a person of proven competence, integrity,
registrars are also given ample authority to exercise administrative probity, and independence.
supervision over civil registrars.
b. Decision-making
Thus, when Administrative Order No. 1, Series of 1993 of the Office of the Civil
i. Sessions of the SC and votes required to render a decision or resolution:
Registrar-General (OCRG) was passed to implement CA No. 3753 it was
declared that the Civil Registrar-General shall have the duty to report any 1) En banc –
violation of the provisions of Act No. 3753 and other laws on civil registration
to the concerned mayor who shall take the proper disciplinary action - Art. VIII, Sec. 4(2) and (3), 2nd and 3rd sentence, and Sec. 11, 2nd
against the offender. sentence;

This authority of the Mayor to exercise administrative jurisdiction over Circuit All cases involving the constitutionality of a treaty, international or executive
Registrars was also recognized generally, under Section 47(2) of the agreement, or law, which shall be heard by the Supreme Court en banc, and
Administrative Code of 1987, and specifically, under Rule 11 of Administrative all other cases which under the Rules of Court are required to be heard en
Order No. 2, Series of 1993 of the OCRG, and the more recent Administrative banc, including those involving the constitutionality, application, or operation
Order No. 5, Series of 2005 of the same office, which applies specially to the of presidential decrees, proclamations, orders, instructions, ordinances, and
registration of acts and events concerning the civil status of Muslim Filipinos. other regulations, shall be decided with the concurrence of a majority of the
Members who actually took part in the deliberations on the issues in the case
At this juncture, it should be remembered that the authority of the Mayor to and voted thereon.
exercise administrative supervision over C/MCRs is not exclusive. The Civil
Service Commission (CSC), as the central personnel agency of the Cases or matters heard by a division shall be decided or resolved with the
government, has the power to appoint and discipline its officials and employees concurrence of a majority of the Members who actually took part in the
and to hear and decide administrative cases instituted by or brought before it deliberations on the issues in the case and voted thereon, and in no case
directly or on appeal. Under Section 9 of the Revised Uniform Rules on without the concurrence of at least three of such Members. When the required
Administrative Cases in the Civil Service, the CSC is granted original concurrent number is not obtained, the case shall be decided en banc: Provided, that no
jurisdiction over administrative cases. doctrine or principle of law laid down by the court in a decision rendered en
banc or in division may be modified or reversed except by the court sitting en
NOTE: Remember that the test of jurisdiction is the nature of the case, not banc.
the personality of the person being sued, especially in cases where he or she
is wearing two hats. Moreover, the CSC always has concurrent jurisdiction over Section 11. xxx The Supreme Court en banc shall have the power to discipline
government employees. judges of lower courts, or order their dismissal by a vote of majority of the
Members who actually took part in the deliberations on the issues in the case
and voted in thereon.

M.R.A.D.C. LUMBRE 131


CONSTITUTIONAL LAW REVIEW

- OR JUST READ Sec. 3, Rule 2 of The Internal Rules of the Supreme Answer: Yes. Under Section 1 of Article VIII, judicial power is vested in ONE
Court (A.M. No. 10-4-20-SC) Supreme Court and in such lower courts as may be established by law. All
other courts were defined by Congress through BP 129. Thus, they also have
Section 3. Court en banc matters and cases. – The Court en banc shall act on the power to abolish the same through legislative enactment.
the following matters and cases:
Question: If a law was passed allowing naturalized citizens to become judges
(a) cases in which the constitutionality or validity of any treaty, international of lower collegiate courts and other lower courts, is the same valid?
or executive agreement, law, executive order, presidential decree,
proclamation, order, instruction, ordinance, or regulation is in question; Answer: The answer must be qualified.
(b) criminal cases in which the appealed decision imposes the death penalty Although Section 7(2) provides that the Congress shall prescribe the
or reclusion perpetua; qualifications of judges of lower courts, the same provision (Section 7(2)(3))
also provides that “no person may be appointed judge thereof unless he is a
(c) cases raising novel questions of law; citizen of the Philippines and a member of the Philippine Bar,” and with proven
(d) cases affecting ambassadors, other public ministers, and consuls; competence, integrity, probity, and independence. Thus, while Congress has
the power to describe qualifications of judges, it cannot change the
(e) cases involving decisions, resolutions, and orders of the Civil Service qualifications laid down in the Constitution. Thus, the law is valid as to judges.
Commission, the Commission on Elections, and the Commission on Audit;
As to qualifications of justices of the lower collegiate courts, Section 7(1)(3)
(f) cases where the penalty recommended or imposed is the dismissal of a provide that “No person shall be appointed Member of… any lower collegiate
judge, the disbarment of a lawyer, the suspension of any of them for a period court unless he is a natural-born citizen of the Philippines,” and with “proven
of more than one year, or a fine exceeding forty thousand pesos; competence, integrity, probity, and independence.” Thus, the law is void
insofar as qualifications for justices are concerned.
(g) cases covered by the preceding paragraph and involving the reinstatement
in the judiciary of a dismissed judge, the reinstatement of a lawyer in the roll a. SM Land vs. BCDA, G.R. No. 203655, 7 September 2015
of attorneys, or the lifting of a judge’s suspension or a lawyer’s suspension
from the practice of law; ISSUES: (1) Whether the 2nd Motion for Reconsideration should be granted,
and (2) whether the Court En Banc should take cognizance of the case.
(h) cases involving the discipline of a Member of the Court, or a Presiding
Justice, or any Associate Justice of the collegial appellate court; RULING: (1) No, a second motion for reconsideration is a prohibited pleading
under Section 2, Rule 56 in relation to Sec. 2, Rule 52 of the Rules of Court.
(i) cases where a doctrine or principle laid down by the Court en banc or by a The rule categorically states: “No second motion for reconsideration of a
Division my be modified or reversed; judgment or final resolution by the same party shall be entertained,” and any
exception to this rule (Section 3, Rule 15 of the Internal Rules of the Supreme
(j) cases involving conflicting decisions of two or more divisions;
Court) can only be granted in the higher interest of justice by the Court en
(k) cases where three votes in a Division cannot be obtained; banc upon a vote of at least two-thirds of its actual membership.

(l) Division cases where the subject matter has a huge financial impact on The following elements are required for a second motion for
businesses or affects the welfare of a community; reconsideration to be granted:

(m) Subject to Section 11 (b) of this rule, other division cases that, in the 1. The motion should satisfactorily explain why granting the same would
opinion of at least three Members of the Division who are voting and present, be in the higher interest of justice;
are appropriate for transfer to the Court en banc;
2. The motion must be made before the ruling sought to be reconsidered
(n) Cases that the Court en banc deems of sufficient importance to merit its attains finality;
attention; and
3. If the ruling sought to be reconsidered was rendered by the Court
(o) All matters involving policy decisions in the administrative supervision of through one of its Divisions, at least three (3) members of the said Division
all courts and their personnel. should vote to elevate the case to the Court En Banc; and

NOTE: 4. The favorable vote of at least two-thirds of the Court En Banc's actual
membership must be mustered for the second motion for reconsideration
Question: If a law was passed amending BP 129, abolishing all appellate to be granted.
courts, paving way for direct appeal to the Supreme Court, is the same valid?

M.R.A.D.C. LUMBRE 132


CONSTITUTIONAL LAW REVIEW

Unfortunately for respondent-movants, the foregoing requirements do not NOTE: The general rule is that when the Supreme Court En Banc is equally
obtain in the case at bench. divided, the decision of the lower court is affirmed.

(2) No. The Supreme Court need not take cognizance of the case. In original cases, petition will be dismissed (status quo will be retained). In
appealed cases, the decision of the lower court will be affirmed (which will
Section 5(2) provides that: “All cases involving the constitutionality of a treaty, have the effect of dismissal). Incidental matters are denied.
international or executive agreement, or law, which shall be heard by the
Supreme Court en banc, and all other cases which under the Rules of Court Exceptions: (1) Where the law will be declared unconstitutional (as laws are
are required to be heard en banc, including those involving the presumed to be constitutional); and (2) where it will amount to conviction of
constitutionality, application, or operation of presidential decrees, the accused (as the accused is presumed to be innocent).
proclamations, orders, instructions, ordinances, and other regulations, shall be
decided with the concurrence of a majority of the Members who actually took - Relate with Sec. 2, Rule 12 of The Internal Rules of the Supreme
part in the deliberations on the issues in the case and voted thereon.” Court (A.M. No. 10-4-20-SC)

Here, it is well to recall that the President did not issue any said executive Section 2. Tie voting in the Court en banc. –
order or presidential issuance in intimating to the BCDA that he wishes for the (a) In civil cases, including special proceedings and special civil actions, where
competitive challenge to be cancelled. There was no document offered that the Court en banc is equally divided in option or the necessary majority vote
was signed by either the Chief Executive or the Executive Secretary, for the cannot be had, the Court shall deliberate on it anew. If after such deliberation
President, to that effect. The situation, therefore, does not involve a still no decision is reached, the Court shall, in an original action filed with it,
presidential order or instruction within the contemplation of Sec. 4(2), Article dismiss the case; in appealed cases, it shall affirm the judgment or order
VIII of the Constitution, and, consequently, does not fall within the jurisdiction appealed from.
of the Court en banc.
(b) In criminal cases, when the Court en banc is equally divided in option of
NOTE: The President made an ORAL presidential order which is not valid. The the necessary majority cannot be had, the Court shall deliberate on it anew. If
1953 case of Ykalina v Oricio, which held that a presidential order may either after such deliberation still no decision is reached, the Court shall reverse the
be oral or in a written memorandum, is not applicable to this case as it is judgment of conviction of the lower court and acquit the accused.
limited specifically to appointments. In the Ykalina case, Oricio’s verbal
appointment was established in evidence by a communication duly signed by (c) When, in an administrative case against any of the Justices of the appealed
the then Acting Executive Secretary “by order of the President,” in hew with courts or any of the Judges of the trial Courts, the impossible penalty is
Section 27(10) of Book III, Title III, Chapter 9-B of E.O. 292, which empowers dismissal and the Court en banc is equally divided in opinion or the majority
the Executive Secretary to attest executive orders and other presidential vote required by the Constitution for dismissal cannot be had, the Court shall
issuances “by authority of the President.” In this case, the oral presidential deliberate on the case anew. If after such deliberation still no decision is
order was not supported by a written communication signed by the Executive reached, the Court shall dismiss the administrative case, unless a majority vote
Secretary, who is also the alter-ego of the President. decides to impose a lesser penalty.

2) In division – Art. VIII, Sec. 4(3), 1st sentence (d) Where the Court en banc is equally divided in opinion of the majority vote
required by the Constitution for annulling any treaty, international or executive
Section 4. xxx Cases or matters heard by a division shall be decided or agreement, law, presidential decree, proclamation, order, instruction,
resolved with the concurrence of a majority of the Members who actually took ordinance, or regulation cannot be had, the Court shall deliberate on the case
part in the deliberations on the issues in the case and voted thereon, and in anew. If such deliberation still no decision is reached, the Court shall deny the
no case without the concurrence of at least three of such Members. xxx challenge to the constitutionally of the act.
3) Procedure if the necessary majority cannot be had – (e) In all matters incidental to the main action where the Court en banc is
- Sec. 7, Rule 56 of the Rules of Court equally divided in opinion, the relief sought shall be denied.

Section 7. Procedure if Opinion is Equally Divided. – Where the Court En Banc ii. Period to decide cases – Art. VIII, Sec. 15(1) and (4)
is equally divided in opinion, or the necessary majority cannot be had, the case Section 15. All cases or matters filed after the effectivity of this Constitution must
shall again be deliberated on, and if after such deliberation no decision is be decided or resolved within twenty-four months from date of submission for the
reached, the original action commenced in the court shall be dismissed; in Supreme Court, and, unless reduced by the Supreme Court, twelve months for all
appealed cases, the judgment or order appealed from shall stand affirmed; lower collegiate courts, and three months for all other lower courts.
and all on incidental matters, the petition or motion shall be denied.
xxx

M.R.A.D.C. LUMBRE 133


CONSTITUTIONAL LAW REVIEW

Despite the expiration of the applicable mandatory period, the court, without prescribed that the Rules of Court promulgated by the Supreme Court shall
prejudice to such responsibility as may have been incurred in consequence thereof, apply to all cases and proceedings filed with the Sandiganbayan.
shall decide or resolve the case or matter submitted thereto for determination,
without further delay. Under Article VIII, Section 5 (5) of the Constitution, Rules of procedure of
special courts and quasi-judicial bodies shall remain effective unless
- Sesbreno vs. CA, GR No. 161390, 16 April 2008 disapproved by the Supreme Court.
ISSUE: Whether or not an appealed case which had been pending beyond the NOTE: Although the Sandiganbayan is a collegiate court, it performs the
time fixed by the Constitution should be "deemed affirmed.” functions of a trial court; hence, the period to decide cases is three (3) months.
The remedy of the party is to file an administrative case against the judge for
RULING: No. Section 11 (2), Article X of the 1973 Constitution, which reads: failure to render decision within three (3) months. Judges, in order to evade
(2) With respect to the Supreme Court and other collegiate appellate liability for failure to render within the allowed period must ask the SC for
courts, when the applicable maximum period shall have lapsed without extension, before the expiration of the 3-month period.
the rendition of the corresponding decision or resolution because the - Re: report on the Judicial Audit Conducted in the Regional Trial Court,
necessary vote cannot be had, the judgment, order, or resolution appealed Br. 56, Mandaue City, Cebu, A.M. No. 09-7-284-RTC, February 16, 2011
from shall be deemed affirmed x x x
ISSUE: Whether Judge Vestil may excuse himself of the untimely disposition
That provision is not found in the present Constitution. The court, under the of the cases by reason of his being understaffed.
1987 Constitution, is now mandated to decide or resolve the case or matter
submitted to it for determination within specified periods. Even when there is RULING: No. SC sustained the findings of OCA.
delay and no decision or resolution is made within the prescribed period, there
is no automatic affirmance of the appealed decision. A review of the records would show the undisputed delay in the disposition of
numerous cases assigned to Branch 56 which was then presided by Judge
- Re: Problem of Delays in Cases Before the Sandiganbayan, AM No. Vestil. There were at least 80 civil cases, some were filed as early as 1997,
00-8-05-SC, 28 November 2001 which are still pending as of March 2007. Furthermore, at least 100 criminal
cases are still pending beyond the 90-day reglementary period.
ISSUE: What is the reglementary period within which the Sandiganbayan must
decide/resolve cases falling within its jurisdiction? In his defense, Judge Vestil sought refuge from the fact that Branch 56 was
saddled with a heavy caseload. We are, however, unconvinced. The Court knew
RULING: Period To Decide/Resolve Cases.-- There are two views. The first the heavy caseloads heaped on the shoulders of every trial judge. But such
view is that from the time a case is submitted for decision or resolution, the cannot excuse him from doing his mandated duty to resolve cases with
Sandiganbayan has twelve (12) months to decide or resolve it. The second diligence and dispatch. Judges burdened with heavy caseloads should request
view is that as a court with trial function, the Sandiganbayan has three (3) the Court for an extension of the reglementary period within which to decide
months to decide the case from the date of submission for decision. their cases if they think they cannot comply with their judicial duty. This, Judge
Article VIII, Section 15 (1) and (2), of the 1987 Constitution provides: Vestil failed to do. Corollarily, a heavy caseload may excuse a judge’s failure
to decide cases within the reglementary period but not their failure to request
"Sec. 15. (1) All cases or matters filed after the effectivity of this an extension of time within which to decide the case on time. Hence, all that
Constitution must be decided or resolved within twenty-four months from respondent judge needs to do is request for an extension of time over which
date of submission to the Supreme Court, and, unless reduced by the the Court has, almost customarily, been considerate.
Supreme Court, twelve months for all lower collegiate courts, and three
months for all other lower courts. Moreover, as correctly pointed out by the OCA, it is not enough that he pens
his decision; it is imperative to promulgate the same within the mandated
(2) A case or matter shall be deemed submitted for decision or resolution period. The lack of staff that will prepare and type the decision is equally
upon the filing of the last pleading, brief or memorandum required by the inexcusable to justify the delay in the promulgation of the cases.
Rules of Court or by the court itself.
We cannot overemphasize the Courts policy on prompt resolution of disputes.
This Constitutional provision does not apply to the Sandiganbayan. The Justice delayed is justice denied. Failure to resolve cases submitted for decision
provision refers to regular courts of lower collegiate level. within the period fixed by law constitutes a serious violation of Section 16,
Article III of the Constitution.
The Sandiganbayan is not a regular court but a special one. The
Sandiganbayan was originally empowered to promulgate its own rules of The honor and integrity of the judicial system is measured not only by the
procedure. However, on March 30, 1995, Congress repealed the fairness and correctness of decisions rendered, but also by the efficiency with
Sandiganbayan’s power to promulgate its own rules of procedure and instead which disputes are resolved. Thus, judges must perform their official duties

M.R.A.D.C. LUMBRE 134


CONSTITUTIONAL LAW REVIEW

with utmost diligence if public confidence in the judiciary is to be preserved. Article VIII of the 1987 Constitution that in the phrase, “a representative of
There is no excuse for mediocrity in the performance of judicial functions. The Congress,” the use of the singular letter “a” preceding “representative of Congress”
position of judge exacts nothing less than faithful observance of the law and is unequivocal and leaves no room for any other construction. It is indicative of
the Constitution in the discharge of official duties. what the members of the Constitutional Commission had in mind, that is, Congress
may designate only one (1) representative to the JBC. Had it been the intention
that more than one (1) representative from the legislature would sit in the JBC, the
4. The Judicial and Bar Council – Art. VIII, Sec. 8 and 9 Framers could have, in no uncertain terms, so provided.

Section 8. A Judicial and Bar Council is hereby created under the supervision of the The framers of Constitution, in creating JBC, hoped that the private sector and the
Supreme Court composed of the Chief Justice as ex officio Chairman, the Secretary of three branches of government would have an active role and equal voice in the
Justice, and a representative of the Congress as ex officio Members, a representative selection of the members of the Judiciary. Therefore, to allow the Legislature to
of the Integrated Bar, a professor of law, a retired Member of the Supreme Court, and have more quantitative influence in the JBC by having more than one voice speak,
a representative of the private sector. whether with one full vote or one-half (1/2) a vote each, would “negate the principle
of equality among the three branches of government which is enshrined in the
The regular members of the Council shall be appointed by the President for a term of Constitution.”
four years with the consent of the Commission on Appointments. Of the Members first
appointed, the representative of the Integrated Bar shall serve for four years, the It is clear, therefore, that the Constitution mandates that the JBC be composed of
professor of law for three years, the retired Justice for two years, and the representative seven (7) members only. Thus, any inclusion of another member, whether with one
of the private sector for one year. whole vote or half (1/2) of it, goes against that mandate. Section 8(1), Article VIII
of the Constitution, providing Congress with an equal voice with other members of
The Clerk of the Supreme Court shall be the Secretary ex officio of the Council and shall the JBC in recommending appointees to the Judiciary is explicit. Any circumvention
keep a record of its proceedings. of the constitutional mandate should not be countenanced for the Constitution is
the supreme law of the land. The Constitution is the basic and paramount law to
The regular Members of the Council shall receive such emoluments as may be
which all other laws must conform and to which all persons, including the highest
determined by the Supreme Court. The Supreme Court shall provide in its annual budget
officials of the land, must defer. Constitutional doctrines must remain steadfast no
the appropriations for the Council.
matter what may be the tides of time. It cannot be simply made to sway and
The Council shall have the principal function of recommending appointees to the accommodate the call of situations and much more tailor itself to the whims and
judiciary. It may exercise such other functions and duties as the Supreme Court may caprices of the government and the people who run it.
assign to it.
Notwithstanding its finding of unconstitutionality in the current composition of the
Section 9. The Members of the Supreme Court and judges of lower courts shall be JBC, all its prior official actions are nonetheless valid. In the interest of fair play
appointed by the President from a list of at least three nominees preferred by the under the doctrine of operative facts, actions previous to the declaration of
Judicial and Bar Council for every vacancy. Such appointments need no confirmation. unconstitutionality are legally recognized. They are not nullified.

For the lower courts, the President shall issue the appointment within ninety days from b. Jardeleza vs. Chief Justice Sereno, G.R. No. 213181, 19 August 2014
the submission of the list.
FACTS: Jardeleza filed the present petition for certiorari and mandamus under Rule
a. Chavez vs. JBC, G.R. 202242, April 16, 2013 65 of the Rules of Court with prayer for the issuance of a Temporary Restraining
Order (TRO), seeking to compel the JBC to include him in the list of nominees for
ISSUE: Whether the practice of the JBC to perform its functions with eight (8) Supreme Court Associate Justice vice Associate Justice Abad, on the grounds that
members, two (2) of whom are members of Congress, defeats the letter and spirit the JBC and Chief Justice Sereno acted in grave abuse of discretion amounting to
of the 1987 Constitution. lack or excess of jurisdiction in excluding him, despite having garnered a sufficient
RULING: No. The current practice of JBC in admitting two members of the number of votes to qualify for the position.
Congress to perform the functions of the JBC is violative of the 1987 Constitution. Notably, Jardeleza’s petition decries that despite the obvious urgency of his earlier
As such, it is unconstitutional. letter-petition and its concomitant filing on June 25, 2014, the same was raffled
One of the primary and basic rules in statutory construction is that where the words only on July 1, 2014 or a day after the controversial JBC meeting. By the time that
of a statute are clear, plain, and free from ambiguity, it must be given its literal his letter-petition was scheduled for deliberation by the Court en banc on July 8,
meaning and applied without attempted interpretation. It is a well-settled principle 2014, the disputed shortlist had already been transmitted to the Office of the
of constitutional construction that the language employed in the Constitution must President. He attributed this belated action on his letter-petition to Chief Justice
be given their ordinary meaning except where technical terms are employed. As Sereno, whose action on such matters, especially those impressed with urgency,
such, it can be clearly and unambiguously discerned from Paragraph 1, Section 8, was discretionary.

M.R.A.D.C. LUMBRE 135


CONSTITUTIONAL LAW REVIEW

An in-depth perusal of Jardeleza’s petition would reveal that his resort to judicial disqualified for nomination is raised or challenged, the affirmative vote of all
intervention hinges on the alleged illegality of his exclusion from the shortlist due the Members of the Council must be obtained for the favorable consideration
to: 1) the deprivation of his constitutional right to due process; and 2) the JBC’s of his nomination.
erroneous application, if not direct violation, of its own rules. Suffice it to say,
Jardeleza directly ascribes the supposed violation of his constitutional rights to the A simple reading of the above provision undoubtedly elicits the rule that a higher
acts of Chief Justice Sereno in raising objections against his integrity and the voting requirement is absolute in cases where the integrity of an applicant is
manner by which the JBC addressed this challenge to his application, resulting in questioned. Simply put, when an integrity question arises, the voting requirement
his arbitrary exclusion from the list of nominees. for his or her inclusion as a nominee to a judicial post becomes "unanimous" instead
of the "majority vote" required in the preceding section. Considering that JBC-009
RULING: The purpose of the JBC’s existence is indubitably rooted in the categorical employs the term "integrity" as an essential qualification for appointment, and its
constitutional declaration that "[a] member of the judiciary must be a person of doubtful existence in a person merits a higher hurdle to surpass, that is, the
proven competence, integrity, probity, and independence." To ensure the unanimous vote of all the members of the JBC, the Court is of the safe conclusion
fulfillment of these standards in every member of the Judiciary, the JBC has been that "integrity" as used in the rules must be interpreted uniformly. Hence, Section
tasked to screen aspiring judges and justices, among others, making certain that 2, Rule 10 of JBC-009 envisions only a situation where an applicant’s moral fitness
the nominees submitted to the President are all qualified and suitably best for is challenged. It follows then that the "unanimity rule" only comes into operation
appointment. In this way, the appointing process itself is shielded from the when the moral character of a person is put in issue. It finds no application where
possibility of extending judicial appointment to the undeserving and mediocre and, the question is essentially unrelated to an applicant’s moral uprightness.
more importantly, to the ineligible or disqualified.
Does the original invocation of Section 2, Rule 10 of JBC-009 involve a question on
In the performance of this sacred duty, the JBC itself admits, as stated in the Jardeleza’s integrity? Does his adoption of a specific legal strategy in the handling
"whereas clauses" of JBC-009, that qualifications such as "competence, integrity, of a case bring forth a relevant and logical challenge against his moral character?
probity and independence are not easily determinable as they are developed and Does the "unanimity rule" apply in cases where the main point of contention is the
nurtured through the years." Additionally, "it is not possible or advisable to lay professional judgment sans charges or implications of immoral or corrupt behavior?
down iron-clad rules to determine the fitness of those who aspire to become a
Justice, Judge, Ombudsman or Deputy Ombudsman." Given this realistic situation, The Court answers these questions in the negative.
there is a need "to promote stability and uniformity in JBC’s guiding precepts and While Chief Justice Sereno claims that the invocation of Section 2, Rule 10 of JBC-
principles." A set of uniform criteria had to be established in the ascertainment of 009 was not borne out of a mere variance of legal opinion but by an "act of
"whether one meets the minimum constitutional qualifications and possesses disloyalty" committed by Jardeleza in the handling of a case, the fact remains that
qualities of mind and heart expected of him" and his office. Likewise for the sake the basis for her invocation of the rule was the "disagreement" in legal strategy as
of transparency of its proceedings, the JBC had put these criteria in writing, now in expressed by a group of international lawyers. The approach taken by Jardeleza in
the form of JBC-009. True enough, guidelines have been set in the determination that case was opposed to that preferred by the legal team. For said reason, criticism
of competence," "probity and independence," "soundness of physical and mental was hurled against his "integrity." The invocation of the "unanimity rule" on
condition, and "integrity." integrity traces its roots to the exercise of his discretion as a lawyer and nothing
As disclosed by the guidelines and lists of recognized evidence of qualification laid else. No connection was established linking his choice of a legal strategy to a
down in JBC-009, "integrity" is closely related to, or if not, approximately equated treacherous intent to trounce upon the country’s interests or to betray the
to an applicant’s good reputation for honesty, incorruptibility, irreproachable Constitution.
conduct, and fidelity to sound moral and ethical standards. That is why proof of an Verily, disagreement in legal opinion is but a normal, if not an essential form of,
applicant’s reputation may be shown in certifications or testimonials from reputable interaction among members of the legal community. A lawyer has complete
government officials and non-governmental organizations and clearances from the discretion on what legal strategy to employ in a case entrusted to him provided
courts, National Bureau of Investigation, and the police, among others. In fact, the that he lives up to his duty to serve his client with competence and diligence, and
JBC may even conduct a discreet background check and receive feedback from the that he exert his best efforts to protect the interests of his client within the bounds
public on the integrity, reputation and character of the applicant, the merits of of the law. Consonantly, a lawyer is not an insurer of victory for clients he
which shall be verified and checked. As a qualification, the term is taken to refer to represents. An infallible grasp of legal principles and technique by a lawyer is a
a virtue, such that, "integrity is the quality of person’s character." utopian ideal. Stripped of a clear showing of gross neglect, iniquity, or immoral
The foregoing premise then begets the question: Does Rule 2, Section 10 of purpose, a strategy of a legal mind remains a legal tactic acceptable to some and
JBC-009, in imposing the "unanimity rule," contemplate a doubt on the deplorable to others. It has no direct bearing on his moral choices.
moral character of an applicant? Section 2, Rule 10 of JBC-009 provides: As to the availability of due process in the proceedings of the JBC, the Court held,
SEC. 2. Votes required when integrity of a qualified applicant is challenged. - after a tedious review of the parties’ respective arguments, the Court concludes
In every case where the integrity of an applicant who is not otherwise that the right to due process is available and thereby demandable as a matter of
right.

M.R.A.D.C. LUMBRE 136


CONSTITUTIONAL LAW REVIEW

The Court does not brush aside the unique and special nature of JBC proceedings. no room for interpretation, only application.42 The wordings of Section 8(1), Article
Indeed, they are distinct from criminal proceedings where the finding of guilt or VIII of the 1987 Constitution are to be considered as indicative of the final intent
innocence of the accused is sine qua non. The JBC’s constitutional duty to of its Framers, that is, for Congress as a whole to only have one representative to
recommend qualified nominees to the President cannot be compared to the duty of sit in the JBC. This Court, therefore, cannot simply make an assumption that the
the courts of law to determine the commission of an offense and ascribe the same Framers merely by oversight failed to take into account the bicameral nature of
to an accused, consistent with established rules on evidence. Even the quantum of Congress in drafting the same. As further laid down in Chavez, the Framers were
evidence required in criminal cases is far from the discretion accorded to the JBC. not keen on adjusting the provision on congressional representation in the JBC as
it was not in the exercise of its primary function, which is to legislate. Notably, the
In JBC proceedings, an aspiring judge or justice justifies his qualifications for the JBC was created to support the executive power to appoint, and Congress, as one
office when he presents proof of his scholastic records, work experience and whole body, was merely assigned a contributory non-legislative function. No
laudable citations. His goal is to establish that he is qualified for the office applied parallelism can be drawn between the representative of Congress in the JBC and
for. The JBC then takes every possible step to verify an applicant's track record for the exercise by Congress of its legislative powers under Article VI and constituent
the purpose of determining whether or not he is qualified for nomination. It powers under Article XVII of the Constitution. Congress, in relation to the executive
ascertains the factors which entitle an applicant to become a part of the roster from and judicial branches of government, is constitutionally treated as another co-equal
which the President appoints. branch in the matter of its JBC representation.
The fact that a proceeding is sui generis and is impressed with discretion, however, This Court cannot succumb to the argument that Congress, being composed of two
does not automatically denigrate an applicant’s entitlement to due process. It is distinct and separate chambers, cannot represent each other in the JBC. Again, as
well-established in jurisprudence that disciplinary proceedings against lawyers are this Court explained in Chavez, such an argument is misplaced because in the JBC,
sui generis in that they are neither purely civil nor purely criminal; they involve any member of Congress, whether from the Senate or the House of
investigations by the Court into the conduct of one of its officers, not the trial of an Representatives, is constitutionally empowered to represent the entire Congress.
action or a suit. It may be a constricted constitutional authority, but it is not an absurdity. To
Notwithstanding being "a class of its own," the right to be heard and to explain broaden the scope of congressional representation in the JBC is tantamount to the
one’s self is availing. The Court subscribes to the view that in cases where an inclusion of a subject matter which was not included in the provision as enacted.
objection to an applicant’s qualifications is raised, the observance of due process True to its constitutional mandate, the Court cannot craft and tailor constitutional
provisions in order to accommodate all situations no matter how ideal or reasonable
neither negates nor renders illusory the fulfillment of the duty of JBC to
the proposed solution may sound. To the exercise of this intrusion, the Court
recommend. This holding is not an encroachment on its discretion in the nomination
declines.
process. Actually, its adherence to the precepts of due process supports and
enriches the exercise of its discretion. When an applicant, who vehemently denies While it is true that Section 8(1), Article VIII of the 1987 Constitution did not
the truth of the objections, is afforded the chance to protest, the JBC is presented explicitly state that the JBC shall be composed of seven members, however, the
with a clearer understanding of the situation it faces, thereby guarding the body same is implied in the enumeration of who will be the members thereof. And though
from making an unsound and capricious assessment of information brought before it is unnecessary for the JBC composition to be an odd number as no tie-breaker is
it. The JBC is not expected to strictly apply the rules of evidence in its assessment needed in the preparation of a shortlist since judicial nominees are not decided by
a "yes" or "no" vote, still, JBC's membership cannot be increased from seven to
of an objection against an applicant. Just the same, to hear the side of the person
eight for it will be a clear violation of the aforesaid constitutional provision. To add
challenged complies with the dictates of fairness for the only test that an exercise
another member in the JBC or to increase the representative of Congress to the
of discretion must surmount is that of soundness. JBC, the remedy is not judicial but constitutional amendment.
c. Umali vs. JBC, G.R. No. 228628, 25 July 2017 In sum, this Court will not overthrow Chavez for it is in accord with the
constitutional mandate of giving Congress "a representative" in the JBC. In the
FACTS: This Petition for Certiorari and Mandamus by Rep. Umali, current Chair of
same manner, the adoption of the rotational scheme will not in any way deprive
the House of Representatives Committee on Justice, impugns the present-day
Congress of its full participation in the JBC for such an arrangement is also in line
practice of six-month rotational representation of Congress in the Judicial and Bar
with that constitutional
Council (JBC) for it unfairly deprives both Houses of Congress of their full
participation in the said body. The aforementioned practice was adopted by the JBC d. Judge Ferdinand R. Villanueva vs. JBC, G.R. No. 211833, 7 April
in light of the ruling in Chavez v. JBC. 2015
RULING: As this Court declared in Chavez, Section 8(1), Article VIII of the 1987 FACTS: In his petition, Judge Villanueva argued that: (1) the Constitution already
Constitution is clear, categorical and unambiguous. Thus, it needs no further prescribed the qualifications of an RTC judge, and the JBC could add no more; (2)
construction or interpretation. Time and time again, it has been repeatedly declared the JBC’s five-year requirement violates the equal protection and due process
by this Court that where the law speaks in clear and categorical language, there is clauses of the Constitution; and (3) the JBC’s five-year requirement violates the

M.R.A.D.C. LUMBRE 137


CONSTITUTIONAL LAW REVIEW

constitutional provision on Social Justice and Human Rights for Equal Opportunity adoption of the five-year requirement policy applied by JBC to the petitioner’s case
of Employment. The petitioner also asserted that the requirement of the is necessary and incidental to the function conferred by the Constitution to the JBC.
Prejudicature Program mandated by Section 104 of Republic Act (R.A.) No. 85575
should not be merely directory and should be fully implemented. He further alleged
that he has all the qualifications for the position prescribed by the Constitution and
by Congress, since he has already complied with the requirement of 10 years of
practice of law.
RULING: As to the propriety of the remedy of certiorari, the Court ruled in the
affirmative. In the process of selecting and screening applicants, the JBC neither
acted in any judicial or quasi-judicial capacity nor assumed unto itself any
performance of judicial or quasi-judicial prerogative. However, since the
formulation of guidelines and criteria, including the policy that the petitioner now
assails, is necessary and incidental to the exercise of the JBC's constitutional
mandate, a determination must be made on whether the JBC has acted with grave
abuse of discretion amounting to lack or excess of jurisdiction in issuing and
enforcing the said policy.

Moroever, the Court can appropriately take cognizance of this case by virtue of the
Court's power of supervision over the JBC. Jurisprudence provides that the power
of supervision is the power of oversight, or the authority to see that subordinate
officers perform their duties. Following this definition, the supervisory authority of
the Court over the JBC is to see to it that the JBC complies with its own rules and
procedures. Thus, when the policies of the JBC are being attacked, then the Court,
through its supervisory authority over the JBC, has the duty to inquire about the
matter and ensure that the JBC complies with its own rules.
Again, the Court reiterates that no person possesses a legal right under the
Constitution to be included in the list of nominees for vacant judicial positions. The
opportunity of appointment to judicial office is a mere privilege, and not a judicially
enforceable right that may be properly claimed by any person.
Under the Constitution, the JBC is mandated to recommend appointees to the
judiciary. While the 1987 Constitution has provided the qualifications of members
of the judiciary, this does not preclude the JBC from having its own set of rules and
procedures and providing policies to effectively ensure its mandate.
The functions of searching, screening, and selecting are necessary and
incidental to the JBC’s principal function of choosing and recommending
nominees for vacancies in the judiciary for appointment by the President.
However, the Constitution did not lay down in precise terms the process that the
JBC shall follow in determining applicants’ qualifications. In carrying out its main
function, the JBC has the authority to set the standards/criteria in
choosing its nominees for every vacancy in the judiciary, subject only to
the minimum qualifications required by the Constitution and law for every
position.

JBC’s ultimate goal is to recommend nominees and not simply to fill up judicial
vacancies in order to promote an effective and efficient administration of justice.
Given this pragmatic situation, the JBC had to establish a set of uniform criteria in
order to ascertain whether an applicant meets the minimum constitutional
qualifications and possesses the qualities expected of him and his office. Thus, the

M.R.A.D.C. LUMBRE 138


CONSTITUTIONAL LAW REVIEW

VIII. The Bill of Rights RULING: YES. To regard the demonstration against police officers, not against
the employer, as evidence of bad faith in collective bargaining and hence a
violation of the collective bargaining agreement and a cause for the dismissal
from employment of the demonstrating employees, stretches unduly the
1. Due process – Right to life, liberty and property
compass of the collective bargaining agreement, is “a potent means of
“No person shall be deprived of life, liberty, or property without due process of law, inhibiting speech” and therefore inflicts a moral as well as mortal wound on
xxxx.” (Sec. 1) the constitutional guarantees of free expression, of peaceful assembly and of
petition.
a. Doctrine of Relative Constitutionality
The collective bargaining agreement which fixes the working shifts of the
The constitutionality of a statute cannot, in every instance, be determined by a employees, according to the respondent Court Industrial Relations, in effect
mere comparison of its provisions with applicable provisions of the Constitution, imposes on the workers the “duty … to observe regular working hours.” The
since the statute may be constitutionally valid as applied to one set of facts and strain construction of the Court of Industrial Relations that a stipulated working
invalid in its application to another. shifts deny the workers the right to stage mass demonstration against police
A statute valid at one time may become void at another time because of altered abuses during working hours, constitutes a virtual tyranny over the mind and
circumstances. Thus, if a statute in its practical operation becomes arbitrary or life the workers and deserves severe condemnation. Renunciation of the
confiscatory, its validity, even though affirmed by a former adjudication, is open to freedom should not be predicated on such a slender ground.
inquiry and investigation in the light of changed conditions. xxx The respondent company is the one guilty of unfair labor practice. Because the
In the Philippine setting, this Court declared the continued enforcement of a valid refusal on the part of the respondent firm to permit all its employees and
law as unconstitutional as a consequence of significant changes in circumstances. workers to join the mass demonstration against alleged police abuses and the
Rutter v. Esteban upheld the constitutionality of the moratorium law - its enactment subsequent separation of the eight (8) petitioners from the service constituted
and operation being a valid exercise by the State of its police power - but also ruled an unconstitutional restraint on the freedom of expression, freedom of
that the continued enforcement of the otherwise valid law would be unreasonable assembly and freedom petition for redress of grievances, the respondent firm
and oppressive. It noted the subsequent changes in the country's business, committed an unfair labor practice defined in Section 4(a-1) in relation to
industry and agriculture. Thus, the law was set aside because its continued Section 3 of Republic Act No. 875, otherwise known as the Industrial Peace
operation would be grossly discriminatory and lead to the oppression of the Act. Section 3 of Republic Act No. 8 guarantees to the employees the right “to
creditors. (Central Bank Employees Association v. Bangko Sentral ng engage in concert activities for … mutual aid or protection”; while Section 4(a-
Pilipinas and the Executive Secretary) 1) regards as an unfair labor practice for an employer interfere with, restrain
or coerce employees in the exercise their rights guaranteed in Section Three.
b. Hierarchy of rights
Property and property rights can be lost thru prescription; but human rights
Life includes the right of an individual to his body and its completeness, free from are imprescriptible. If human rights are extinguished by the passage of time,
dismemberment, and extends to the use of God-given faculties which make life then the Bill of Rights is a useless attempt to limit the power of government
enjoyable. and ceases to be an efficacious shield against the tyranny of officials, of
majorities, of the influential and powerful, and of oligarchs — political,
Liberty includes the right to exist and the right to be free from arbitrary personal
economic or otherwise.
restraint or servitude. It includes the right of the citizen to be free to use his
faculties in all lawful ways. In the hierarchy of civil liberties, the rights of free expression and of
assembly occupy a preferred position as they are essential to the
Property is anything that can come under the right of ownership and be the subject
preservation and vitality of our civil and political institutions; and such priority
of contract. It represents more than the things a person owns; it includes the right
"gives these liberties the sanctity and the sanction not permitting dubious
to secure, use, and dispose of them.
intrusions."
i. Philippine Blooming Mills Employees Organization vs. Philippine
The superiority of these freedoms over property rights is underscored by the
Blooming Mills Co., Inc., 51 SCRA 189 (1973)
fact that a mere reasonable or rational relation between the means employed
ISSUE: Whether the “no strike and no lockout” provision in the collective by the law and its object or purpose — that the law is neither arbitrary nor
bargaining agreement (CBA) is an inhibition of the rights of free expression, discriminatory nor oppressive — would suffice to validate a law which restricts
free assembly and petition of the employers or impairs property rights. On the other hand, a constitutional or valid
infringement of human rights requires a more stringent criterion, namely
existence of a grave and immediate danger of a substantive evil which the
State has the right to prevent.

M.R.A.D.C. LUMBRE 139


CONSTITUTIONAL LAW REVIEW

c. Property right outside those three, the State can take away those other rights without due
process. Thus, one must always ask if the right to be taken away by the State
i. Chavez vs. Romulo, 431 SCRA 534 (2004) is life, liberty, or property.
ISSUE: Whether petitioner is correct in contending that the issuance of e. Aspects of due process
guidelines suspending the issuance of Permits to Carry Firearms Outside of
Residence (PTCFOR) infringes his vested right to bear arms. i. Substantive due process
RULING: NO. Petitioner invokes Article III Section 1 of the 1987 Constitution, Serves as a restriction on government’s law- and rule-making powers.
asserting that the revocation of his PTCFOR pursuant to the assailed Guidelines
deprived him of his "vested property right" without due process of law and in 1. Requisites
violation of the equal protection of law. a. The interests of the public, in general, as distinguished from those of a
In evaluating a due process claim, the first and foremost consideration must particular class, require the intervention of the State; and
be whether life, liberty or property interest exists. The bulk of jurisprudence is b. The means employed are reasonably necessary for the accomplishment
that a license authorizing a person to enjoy a certain privilege is neither a of the purpose, and not unduly oppressive on individuals.
property nor property right. In Tan vs. The Director of Forestry, we ruled that
"a license is merely a permit or privilege to do what otherwise would be ii. Procedural due process
unlawful, and is not a contract between the authority granting it and the person
Serves as a restriction on the actions of judicial or quasi-judicial
to whom it is granted; neither is it property or a property right, nor does it
agencies of the government.
create a vested right."
1. Requisites
d. Constitutional vs. Statutory Due Process
a. An impartial court or tribunal clothed with judicial power to hear and
i. Agabon vs. NLRC, G.R. No. 158693, November 17, 2004
determine the matter before it;
ISSUE: Whether the violation of the procedural requirements of notice and
b. Jurisdiction must be lawfully acquired over the person of the defendant
hearing for termination of employees is a violation of Constitutional due
and over the property which is the subject matter of the proceedings;
process.
c. The defendant must be given the opportunity to be heard;
RULING: No. Constitutional due process is that provided under the
Constitution, which involves the protection of the individual against d. Judgment must be rendered upon lawful hearing.
governmental oppression and the assurance of his rights In civil, criminal and
administrative proceedings; statutory due process is that found in the Labor
Code and its Implementing Rules and protects the individual from being 2. Equal Protection
unjustly terminated without just or authorized cause after notice and hearing.
“xxx nor shall any person be denied the equal protection of the laws.” (Sec. 1)
The two are similar in that they both have two aspects: substantive due
process and procedural due process. However, they differ in that under the All persons or things similarly situated should be treated alike, both as to rights
Labor Code, the first one refers to the valid and authorized causes of conferred and responsibilities imposed. Natural and juridical persons are entitled to this
employment termination, while the second one refers to the manner of guarantee; but with respect to artificial persons, they enjoy the protection only insofar
dismissal. A denial of statutory due process is not the same as a denial of as their property is concerned.
Constitutional due process for reasons enunciated in Serrano v. NLRC.
a. Requisites for valid classification (ReGLA)
The dismissal is valid, but Riviera should pay nominal damages to the Agabons
Such classification, however, to be valid must pass the test of reasonableness. The
in vindication of the latter for violating their right to notice and hearing. The
test has four requisites:
penalty is in the nature of a penalty or indemnification, the amount dependent
on the facts of each case, including the nature of gravity of offense of the (1) the classification rests on substantial distinctions;
employer. Moreover, the payment of backwages is unjustified as only illegal
termination gives the employee the right to be paid full backwages. When the (2) it is germane to the purpose of the law;
dismissal is valid or upheld, the employee has no right to backwages.
(3) it is not limited to existing conditions only; and
NOTE: For purposes of the Bar, always remember that the State cannot take (4) it applies equally to all members of the same class.
all three (3) rights – life, liberty, and property – without due process, but

M.R.A.D.C. LUMBRE 140


CONSTITUTIONAL LAW REVIEW

i. Garcia v. Executive Secretary, GR No. 198554, July 30, 2012 legislation which is limited either in the object to which it is directed or by the
territory within which it is to operate.
ISSUE: Whether Article 29 of the Revised Penal Code can be made applicable
to Garcia’s case, having served at least 6 years of preventive imprisonment. The equal protection of the laws clause of the Constitution allows classification.
Classification in law, as in the other departments of knowledge or practice, is
RULING: Yes. In Marcos v. Chief of Staff, Armed Forces of the Philippines, this the grouping of things in speculation or practice because they agree with one
Court ruled that a court-martial case is a criminal case and the General Court another in certain particulars. A law is not invalid because of simple inequality.
Martial is a "court" akin to any other courts. In the same case, this Court The very idea of classification is that of inequality, so that it goes without
clarified as to what constitutes the words "any court" used in Section 17 of the saying that the mere fact of inequality in no manner determines the matter of
1935 Constitution prohibiting members of Congress to appear as counsel in constitutionality. All that is required of a valid classification is that it be
any criminal case in which an officer or employee of the Government is accused reasonable, which means that the classification should be based on substantial
of an offense committed in relation to his office. distinctions which make for real differences; that it must be germane to the
Hence, the General Court Martial is a court within the strictest sense of the purpose of the law; that it must not be limited to existing conditions only; and
word and acts as a criminal court. On that premise, certain provisions of the that it must apply equally to each member of the class. This Court has held
Revised Penal Code, insofar as those that are not provided in the Articles of that the standard is satisfied if the classification or distinction is based on a
War and the Manual for Courts-Martial, can be supplementary. reasonable foundation or rational basis and is not palpably arbitrary.

Nevertheless, the application of Article 29 of the Revised Penal Code in the In this case, first, R.A. 9262 rests on substantial distinctions. The unequal
Articles of War is in accordance with the Equal Protection Clause of the 1987 power relationship between women and men; the fact that women are more
Constitution. According to a long line of decisions, equal protection simply likely than men to be victims of violence; and the widespread gender bias and
requires that all persons or things similarly situated should be treated alike, prejudice against women all make for real differences justifying the
both as to rights conferred and responsibilities imposed. It requires public classification. Second, the classification is germane to the purpose of the law,
bodies and institutions to treat similarly situated individuals in a similar which is to address violence committed against women and children, spelled
manner. The purpose of the equal protection clause is to secure every person out in its Declaration of Policy. Third, the classification is not limited to existing
within a state's jurisdiction against intentional and arbitrary discrimination, conditions only, and apply equally to all members. The application of R.A. 9262
whether occasioned by the express terms of a statute or by its improper is not limited to the existing conditions when it was promulgated, but to future
execution through the state's duly-constituted authorities. In other words, the conditions as well, for as long as the safety and security of women and their
concept of equal justice under the law requires the state to govern children are threatened by violence and abuse.
impartially, and it may not draw distinctions between individuals NOTE: This was asked in the 2016 Bar Exams. Women can be charged with
solely on differences that are irrelevant to a legitimate governmental violation of R.A. 9262 as the language of the law is gender-neutral. Even
objective. It, however, does not require the universal application of parents-in-law can be charged and punished as principals, accomplices, or
the laws to all persons or things without distinction. What it simply accessories through conspiracy.
requires is equality among equals as determined according to a valid
classification. b. Judicial standards of equal protection (Rational basis, strict scrutiny and
intermediate scrutiny tests)
ii. Garcia v. Drilon, 699 SCRA 352 (2013)
i. Central Bank Employees Association vs. BSP, GR No. 148208,
ISSUE: Whether R.A. No. 9262 violates the equal protection clause. December 15, 2004
RULING: No. Equal protection simply requires that all persons or things ISSUE: Whether the proviso in Section 15(c), Article II of R.A. No. 7653 is
similarly situated should be treated alike, both as to rights conferred and violative of the equal protection clause because after the same was enacted,
responsibilities imposed. In the case of Victoriano v. Elizalde Rope Workers' the charters of GSIS, LBP, DBP, and SSS were also amended, but the personnel
Union, The guaranty of equal protection of the laws is not a guaranty of equality of the latter GFIs were all exempted from the coverage of the Salary
in the application of the laws upon all citizens of the state. It is not, therefore, Standardization Law (SSL). Thus, within the class of rank-and-file personnel of
a requirement, in order to avoid the constitutional prohibition against GFIs, the BSP rank-and-file are also discriminated upon.
inequality, that every man, woman and child should be affected alike by a
statute. Equality of operation of statutes does not mean indiscriminate RULING: Yes. In the case at bar, it is clear in the legislative deliberations that
operation on persons merely as such, but on persons according to the the exemption of officers (SG 20 and above) from the SSL was intended to
circumstances surrounding them. It guarantees equality, not identity of rights. address the BSPs lack of competitiveness in terms of attracting competent
The Constitution does not require that things which are different in fact be officers and executives. It was not intended to discriminate against the rank-
treated in law as though they were the same. The equal protection clause does and-file. If the end-result did in fact lead to a disparity of treatment between
not forbid discrimination as to things that are different. It does not prohibit the officers and the rank-and-file in terms of salaries and benefits, the

M.R.A.D.C. LUMBRE 141


CONSTITUTIONAL LAW REVIEW

discrimination or distinction has a rational basis and is not palpably, purely, justices sought formulations that would blur the sharp distinctions of the two-
and entirely arbitrary in the legislative sense. tiered approach or that would narrow the gap between strict scrutiny and
deferential review. The most elaborate attack came from Justice Marshall,
However, it was further held that enactment of subsequent laws exempting all whose frequently stated position was developed most elaborately in his dissent
other rank and file employees of GFIs from SSL renders the continued in the Rodriguez case:
application of provision in violation of equal protection clause.
The Court apparently seeks to establish [that] equal protection cases fall into
A statute valid at one time may become void at another time because one of two neat categories which dictate the appropriate standard of review -
of altered circumstances. Thus, if a statute in its practical operation strict scrutiny or mere rationality. But this (sic) Court's [decisions] defy
becomes arbitrary or confiscatory, its validity, even though affirmed by a such easy categorization. A principled reading of what this Court has done
former adjudication, is open to inquiry and investigation in the light of changed reveals that it has applied a spectrum of standards in reviewing discrimination
conditions. In the Philippine setting, this Court declared the continued allegedly violative of the equal protection clause. This spectrum clearly
enforcement of a valid law as unconstitutional as a consequence of significant comprehends variations in the degree of care with which Court will scrutinize
changes in circumstances. particular classification, depending, I believe, on the constitutional and societal
In our jurisdiction, the standard and analysis of equal protection challenges in importance of the interests adversely affected and the recognized
the main have followed the "rational basis" test, coupled with a deferential invidiousness of the basis upon which the particular classification is drawn.
attitude to legislative classifications and a reluctance to invalidate a law unless Justice Marshall's "sliding scale" approach describes many of the modern
there is a showing of a clear and unequivocal breach of the Constitution. decisions, although it is a formulation that the majority refused to embrace.
In contrast, jurisprudence in the U.S. has gone beyond the static "rational But the Burger Court's results indicate at least two significant changes in equal
basis" test. Professor Gunther highlights the development in equal protection protection law: First, invocation of the "old" equal protection formula no longer
jurisprudential analysis, to wit: signals, as it did with the Warren Court, an extreme deference to legislative
classifications and a virtually automatic validation of challenged statutes.
Traditionally, equal protection supported only minimal judicial intervention in Instead, several cases, even while voicing the minimal "rationality" "hands-off"
most contexts. Ordinarily, the command of equal protection was only that standards of the old equal protection, proceed to find the statute
government must not impose differences in treatment "except upon some unconstitutional. Second, in some areas the modern Court has put forth
reasonable differentiation fairly related to the object of regulation." The old standards for equal protection review that, while clearly more intensive than
variety of equal protection scrutiny focused solely on the means used by the the deference of the "old" equal protection, are less demanding than the
legislature: it insisted merely that the classification in the statute reasonably strictness of the "new" equal protection. Sex discrimination is the best
relates to the legislative purpose. Unlike substantive due process, equal established example of an "intermediate" level of review. Thus, in one case,
protection scrutiny was not typically concerned with identifying "fundamental the Court said that "classifications by gender must serve important
values" and restraining legislative ends. And usually the rational classification governmental objectives and must be substantially related to achievement of
requirement was readily satisfied: the courts did not demand a tight fit those objectives." That standard is "intermediate" with respect to both ends
between classification and purpose; perfect congruence between means and and means: where ends must be "compelling" to survive strict scrutiny and
ends was not required. xxx xxx xxx merely "legitimate" under the "old" mode, "important" objectives are required
here; and where means must be "necessary" under the "new" equal protection,
A sharply differentiated two-tier approach evolved by the late 1960s: in
and merely "rationally related" under the "old" equal protection, they must be
addition to the deferential "old" equal protection, a "new" equal protection,
"substantially related" to survive the "intermediate" level of review. (Central
connoting strict scrutiny, arose…. The intensive review associated with the new
Bank Employees Association v. Bangko Sentral ng Pilipinas, supra)
equal protection imposed two demands - a demand not only as to means but
also one as to ends. Legislation qualifying for strict scrutiny required a far
closer fit between classification and statutory purpose than the rough and
ready flexibility traditionally tolerated by the old equal protection: means had 3. Search and Seizure
to be shown "necessary" to achieve statutory ends, not merely "reasonably Section 2. The right of the people to be secure in their persons, houses, papers, and
related" ones. Moreover, equal protection became a source of ends scrutiny as effects against unreasonable searches and seizures of whatever nature and for any
well: legislation in the areas of the new equal protection had to be justified by purpose shall be inviolable, and no search warrant or warrant of arrest shall issue except
"compelling" state interests, not merely the wide spectrum of "legitimate" state upon probable cause to be determined personally by the judge after examination under
ends. oath or affirmation of the complainant and the witnesses he may produce, and
Even while the two-tier scheme has often been adhered to in form, there has particularly describing the place to be searched and the persons or things to be seized.
also been an increasingly noticeable resistance to the sharp difference between a. Availability of right to alien -
deferential "old" and interventionist "new" equal protection. A number of

M.R.A.D.C. LUMBRE 142


CONSTITUTIONAL LAW REVIEW

People vs. Chua Ho San, 307 SCRA 432 (1999) apprehended." In short, there is no probable cause. Persistent reports of rampant
smuggling of firearm and other contraband articles, Chua's watercraft differing in
ISSUE: Whether persistent reports of rampant smuggling of firearm and other appearance from the usual fishing boats that commonly cruise over the Bacnotan
contraband articles, Chua's watercraft differing in appearance from the usual seas, Chua’s illegal entry into the Philippines, Chua’s suspicious behavior, i.e. he
fishing boats that commonly cruise over the Bacnotan seas, Chua’s illegal entry attempted to flee when he saw the police authorities, and the apparent ease by
into the Philippines, Chua’s suspicious behavior, i.e. he attempted to flee when he which Chua can return to and navigate his speedboat with immediate dispatch
saw the police authorities, and the apparent ease by which Chua can return to and towards the high seas, do not constitute "probable cause." None of the telltale
navigate his speedboat with immediate dispatch towards the high seas, constitute clues, e.g., bag or package emanating the pungent odor of marijuana or other
"probable cause." prohibited drug, 20 confidential report and/or positive identification by informers
RULING: No. It is fundamental, however, that to constitute a waiver, it must first of courier(s) of prohibited drug and/or the time and place where they will
appear that the right exists; secondly, that the person involved had knowledge, transport/deliver the same, suspicious demeanor or behavior and suspicious bulge
actual or constructive, of the existence of such a right; and lastly, that said person in the waist — accepted by the Court as sufficient to justify a warrantless arrest
had an actual intention to relinquish the right. CHUA (an alien) never exhibited that exists in the case. There was no classified information that a foreigner would
he knew, actually or constructively of his right against unreasonable searches or disembark at Tammocalao beach bearing prohibited drug on the date in question.
that he intentionally conceded the same. Chua was not identified as a drug courier by a police informer or agent. The fact
that the vessel that ferried him to shore bore no resemblance to the fishing boats
CHUA obviously failed to understand the events that overran and overwhelmed of the area did not automatically mark him as in the process of perpetrating an
him. The police officers already introduced themselves to CHUA in three languages, offense. The search cannot therefore be denominated as incidental to an arrest. To
but he remained completely deadpan. The police hence concluded that CHUA failed reiterate, the search was not incidental to an arrest. There was no warrant of arrest
to comprehend the three languages. When CHUA failed to respond again to the and the warrantless arrest did not fall under the exemptions allowed by the Rules
police’s request to open the bag, they resorted to what they called sign language. of Court as already shown. From all indications, the search was nothing but a fishing
They claimed that CHUA finally understood their hand motions and gestures. This expedition. Casting aside the regulated substance as evidence, the same being the
Court disagrees. If CHUA could not understand what was orally articulated to him, fruit of a poisonous tree, the remaining evidence on record are insufficient, feeble
how could he understand the police’s sign language. More importantly, it cannot and ineffectual to sustain Chua’s conviction.
logically be inferred from his alleged cognizance of the sign language that he
deliberately, intelligently, and consciously waived his right against such an intrusive NOTE: The Bill of Rights may be invoked only by citizens of the Philippines
search. This Court is not unmindful of cases upholding the validity of consented (against the State, and not private persons), unless the Supreme Court provides
warrantless searches and seizure. But in these cases, the police officers' request to otherwise in particular cases. Ratio: some are basic human rights under the
search personnel effects was orally articulated to the accused and in such language Universal Declaration of Human Rights.
that left no room for doubt that the latter fully understood what was requested. In b. Requisites of valid warrant
some instances, the accused even verbally replied to the request demonstrating
that he also understood the nature and consequences of such request. i. Probable-Cause Standard
The Constitutional proscription against unreasonable searches and seizures does 1. The difference between the determination of probable cause of
not, of course, forestall reasonable searches and seizure. This interdiction against a prosecutor and the determination of probable cause of a judge
warrantless searches and seizures, however, is not absolute and such warrantless
searches and seizures have long been deemed permissible by jurisprudence. a. Reyes vs. Ombudsman, G.R. Nos. 212593-94, 15 March 2016

The Rules of Court recognize permissible warrantless arrests, to wit: RULING: In assessing if the Ombudsman had committed grave abuse
of discretion, attention must be drawn to the context of its ruling -
(1) arrests in flagrante delicto, that, is: preliminary investigation is merely an inquisitorial mode of
discovering whether or not there is reasonable basis to believe that a
(2) arrests effected in hot pursuit, and crime has been committed and that the person charged should be
(3) arrests of escaped prisoners. held responsible for it. Being merely based on opinion and belief, "a
finding of probable cause does not require an inquiry as to whether
The prosecution and the defense painted extremely divergent versions of the there is sufficient evidence to secure a conviction."
incident, but the Court is certain that Chua was arrested and his bag searched
without the benefit of a warrant. There are no facts on record reasonably suggestive In Fenequito v. Vergara, Jr., "[p]robable cause, for the purpose of
or demonstrative of Chua’s participation in an ongoing criminal enterprise that filing a criminal information, has been defined as such facts as are
could have spurred police officers from conducting the obtrusive search. The RTC sufficient to engender a well-founded belief that a crime has been
never took the pains of pointing to such facts, but predicated mainly its decision on committed and that respondent is probably guilty thereof. The term
the finding that "accused was caught red-handed carrying the bagful of shabu when does not mean 'actual or positive cause nor does it import absolute

M.R.A.D.C. LUMBRE 143


CONSTITUTIONAL LAW REVIEW

certainty. It is merely based on opinion and reasonable belief. a. SJS vs. Dangerous Drugs Board, GR No. 157870, November
Probable cause does riot require an inquiry x x x whether there is 3, 2008
sufficient evidence to procure a conviction. It is enough that it is
believed that the act or omission complained of constitutes the ISSUES: Whether certain provisions under Section 36 of R.A. No.
offense charged." 9165, insofar as it requires mandatory drug testing of: (1) candidates
for public office; (2) students of secondary and tertiary schools; (3)
Once the public prosecutor (or the Ombudsman) determines probable officers and employees of public and private offices; and (4) persons
cause and thus, elevates the case to the trial court (or the charged before the prosecutor’s office with certain offenses
Sandiganbayan), a judicial determination of probable cause is made (imprisonment of 6 years and up), is unconstitutional for violating the
in order to determine if a warrant of arrest should be issued ordering right against unreasonable searches and seizures.
the detention of the accused. The Court, in People v. Castillo,
delineated the functions and purposes of a determination of probable RULING:
cause made by the public prosecutor, on the one hand, and the trial (1) As to Pimentel Petition: Unconstitutional. Pimentel claims that
court, on the other: Sec. 36(g) of RA 9165 and COMELEC Resolution No. 6486 illegally
There are two kinds of determination of probable case: executive and impose an additional qualification on candidates for senator. He points
judicial. The executive determination of probable cause is one made out that, subject to the provisions on nuisance candidates, a
during preliminary investigation. It is a function that properly pertains candidate for senator needs only to meet the qualifications laid down
to the public prosecutor who is given a broad discretion to determine in Sec. 3, Art. VI of the Constitution, to wit: (1) citizenship, (2) voter
whether probable cause exists and to charge those whom he believes registration, (3) literacy, (4) age, and (5) residency. Beyond these
to have committed the crime as defined by law and thus should be stated qualification requirements, candidates for senator need not
held for trial. Otherwise stated, such official has the quasi-judicial possess any other qualification to run for senator and be voted upon
authority to determine whether or not a criminal case must be filed in and elected as member of the Senate. The Congress cannot validly
court. Whether or not that function has been correctly discharged by amend or otherwise modify these qualification standards, as it cannot
the public prosecutor, i.e., whether or not he has made a correct disregard, evade, or weaken the force of a constitutional mandate, or
ascertainment of the existence of probable cause in a case, is a matter alter or enlarge the Constitution.
that the trial court itself does not and may not be compelled to pass Pimentel's contention is well - taken. Accordingly, Sec. 36(g) of RA
upon. 9165 should be, as it is hereby declared as, unconstitutional. It is
The judicial determination of probable cause, on the other hand, is basic that if a law or an administrative rule violates any norm of the
one made by the judge to ascertain whether a warrant of arrest Constitution, that issuance is null and void and has no effect. The
should be issued against the accused. The judge must satisfy himself Constitution is the basic law to which all laws must conform; no act
that based on the evidence submitted, there is necessity for placing shall be valid if it conflicts with the Constitution. In the discharge of
the accused under custody in order not to frustrate the ends of justice. their defined functions, the three departments of government have
If the judge finds no probable cause, the judge cannot be forced to no choice but to yield obedience to the commands of the Constitution.
issue the arrest warrant. (Emphasis and underscoring supplied) Whatever limits it imposes must be observed.

As above-articulated, the executive determination of probable cause In the same vein, the COMELEC cannot, in the guise of enforcing and
concerns itself with whether there is enough evidence to support an administering election laws or promulgating rules and regulations to
Information being filed. The judicial determination of probable cause, implement Sec. 36(g), validly impose qualifications on candidates for
on the other hand, determines whether a warrant of arrest should be senator in addition to what the Constitution prescribes. If Congress
issued. cannot require a candidate for senator to meet such additional
qualification, the COMELEC, to be sure, is also without such power.
NOTE: The quantum of proof for purposes of determining probable The right of a citizen in the democratic process of election should not
cause on the part of the prosecution is: more than suspicion but less be defeated by unwarranted impositions of requirement not otherwise
than proof to sustain conviction. specified in the Constitution.

There are three (3) options given to the judge upon filing by the Sec. 36(g) of RA 9165, as sought to be implemented by the assailed
prosecutor of the information in court: (1) dismiss information; (2) COMELEC resolution, effectively enlarges the qualification
issue warrant of arrest; or (3) order the prosecutor to present requirements enumerated in the Sec. 3, Art. VI of the Constitution.
evidence. As couched, said Sec. 36(g) unmistakably requires a candidate for
senator to be certified illegal - drug clean, obviously as a pre -
2. Drug, alcohol and blood tests condition to the validity of a certificate of candidacy for senator or,

M.R.A.D.C. LUMBRE 144


CONSTITUTIONAL LAW REVIEW

with like effect, a condition sine qua non to be voted upon and, if -under RA 9165 is in the nature of administrative search needing what
proper, be proclaimed as senator - elect. The COMELEC resolution was referred to in Vernonia as "swift and informal disciplinary
completes the chain with the proviso that "[n]o person elected to any procedures," the probable - cause standard is not required or even
public office shall enter upon the duties of his office until he has practicable. Be that as it may, the review should focus on the
undergone mandatory drug test." Viewed, therefore, in its proper reasonableness of the challenged administrative search in question.
context, Sec. 36(g) of RA 9165 and the implementing COMELEC
Resolution add another qualification layer to what the 1987 The first factor to consider in the matter of reasonableness is the
Constitution, at the minimum, requires for membership in the Senate. nature of the privacy interest upon which the drug testing, which
Whether or not the drug - free bar set up under the challenged effects a search within the meaning of Sec. 2, Art. III of the
provision is to be hurdled before or after election is really of no Constitution, intrudes. In this case, the office or workplace serves as
moment, as getting elected would be of little value if one cannot the backdrop for the analysis of the privacy expectation of the
assume office for non - compliance with the drug - testing employees and the reasonableness of drug testing requirement. The
requirement. employees' privacy interest in an office is to a large extent
circumscribed by the company's work policies, the collective
(2) As to SJS Petition: Constitutional. The drug test prescribed bargaining agreement, if any, entered into by management and the
under Sec. 36(c), (d), and (f) of RA 9165 for secondary and tertiary bargaining unit, and the inherent right of the employer to maintain
level students and public and private employees, while mandatory, is discipline and efficiency in the workplace. Their privacy expectation in
a random and suspicionless arrangement. The objective is to stamp a regulated office environment is, in fine, reduced; and a degree of
out illegal drug and safeguard in the process "the well being of [the] impingement upon such privacy has been upheld.
citizenry, particularly the youth, from the harmful effects of
dangerous drugs." This statutory purpose, per the policy - declaration Just as defining as the first factor is the character of the intrusion
portion of the law, can be achieved via the pursuit by the state of "an authorized by the challenged law (second factor). Reduced to a
intensive and unrelenting campaign against the trafficking and use of question form, is the scope of the search or intrusion clearly set forth,
dangerous drugs x x x through an integrated system of planning, or, as formulated in Ople v. Torres, is the enabling law authorizing a
implementation and enforcement of anti - drug abuse policies, search "narrowly drawn" or "narrowly focused"?
programs and projects." The primary legislative intent is not criminal The poser should be answered in the affirmative. For one, Sec. 36 of
prosecution, as those found positive for illegal drug use as a result of RA 9165 and its implementing rules and regulations (IRR), as
this random testing are not necessarily treated as criminals. They may couched, contain provisions specifically directed towards preventing a
even be exempt from criminal liability should the illegal drug user situation that would unduly embarrass the employees or place them
consent to undergo rehabilitation. under a humiliating experience. While every officer and employee in
In sum, what can reasonably be deduced from the above two cases a private establishment is under the law deemed forewarned that he
and applied to this jurisdiction are: (1) schools and their or she may be a possible subject of a drug test, nobody is really
administrators stand in loco parentis with respect to their students; singled out in advance for drug testing. The goal is to discourage drug
(2) minor students have contextually fewer rights than an adult, and use by not telling in advance anyone when and who is to be tested.
are subject to the custody and supervision of their parents, guardians, And as may be observed, Sec. 36(d) of RA 9165 itself prescribes what,
and schools; (3) schools, acting in loco parentis, have a duty to in Ople, is a narrowing ingredient by providing that the employees
safeguard the health and well - being of their students and may adopt concerned shall be subjected to "random drug test as contained in the
such measures as may reasonably be necessary to discharge such company's work rules and regulations x x x for purposes of reducing
duty; and (4) schools have the right to impose conditions on the risk in the work place."
applicants for admission that are fair, just, and non-discriminatory. For another, the random drug testing shall be undertaken under
As the warrantless clause of Sec. 2, Art III of the Constitution is conditions calculated to protect as much as possible the employee's
couched and as has been held, "reasonableness" is the touchstone of privacy and dignity. As to the mechanics of the test, the law specifies
the validity of a government search or intrusion. And whether a search that the procedure shall employ two testing methods, i.e., the
at issue hews to the reasonableness standard is judged by the screening test and the confirmatory test, doubtless to ensure as much
balancing of the government - mandated intrusion on the individual's as possible the trustworthiness of the results. But the more important
privacy interest against the promotion of some compelling state consideration lies in the fact that the test shall be conducted by
interest. In the criminal context, reasonableness requires showing of trained professionals in access - controlled laboratories monitored by
probable cause to be personally determined by a judge. Given that the Department of Health (DOH) to safeguard against results
the drug - testing policy for employees--and students for that matter- tampering and to ensure an accurate chain of custody. In addition,
the IRR issued by the DOH provides that access to the drug results

M.R.A.D.C. LUMBRE 145


CONSTITUTIONAL LAW REVIEW

shall be on the "need to know" basis; that the "drug test result and opportunity for the presentation of the respondent’s side with regard to the
the records shall be [kept] confidential subject to the usual accepted accusation. Afterwards, the investigating officer shall decide whether the
practices to protect the confidentiality of the test results." Notably, allegations and defenses lead to a reasonable belief that a crime has been
RA 9165 does not oblige the employer concerned to report to the committed, and that it was the respondent who committed it. Otherwise, the
prosecuting agencies any information or evidence relating to the investigating officer is bound to dismiss the complaint. As long as efforts to
violation of the Comprehensive Dangerous Drugs Act received as a reach a respondent were made, and he was given an opportunity to present
result of the operation of the drug testing. All told, therefore, the countervailing evidence, the preliminary investigation remains valid.
intrusion into the employees' privacy, under RA 9165, is accompanied
by proper safeguards, particularly against embarrassing leakages of In this case, the Resolution stated that efforts were undertaken to serve
test results, and is relatively minimal. subpoenas on the named respondents at their last known addresses. This is
sufficient for due process. It was only because a majority of them could no
(3) As to Laserna Petition: Unconstitutional. Unlike the situation longer be found at their last known addresses that they were not served copies
covered by Sec. 36(c) and (d) of RA 9165, the Court finds no valid of the complaint and the attached documents or evidence.
justification for mandatory drug testing for persons accused of crimes.
In the case of students, the constitutional viability of the mandatory, As to the Warrant of Arrest, although the Constitution provides that probable
random, and suspicionless drug testing for students emanates cause shall be determined by the judge after an examination under oath or an
primarily from the waiver by the students of their right to privacy affirmation of the complainant and the witnesses, we have ruled that a hearing
when they seek entry to the school, and from their voluntarily is not necessary for the determination thereof. In fact, the judge’s personal
submitting their persons to the parental authority of school examination of the complainant and the witnesses is not mandatory and
authorities. In the case of private and public employees, the indispensable for determining the aptness of issuing a warrant of arrest.
constitutional soundness of the mandatory, random, and It is enough that the judge personally evaluates the prosecutor’s report and
suspicionless drug testing proceeds from the reasonableness of the supporting documents showing the existence of probable cause for the
drug test policy and requirement. indictment and, on the basis thereof, issue a warrant of arrest; or if, on the
We find the situation entirely different in the case of persons charged basis of his evaluation, he finds no probable cause, to disregard the
before the public prosecutor's office with criminal offenses punishable prosecutor's resolution and require the submission of additional affidavits of
with six (6) years and one (1) day imprisonment. The operative witnesses to aid him in determining its existence.
concepts in the mandatory drug testing are "randomness" and The determination of probable cause for the issuance of warrants of arrest
"suspicionless." In the case of persons charged with a crime before against petitioners is addressed to the sound discretion of Judge Abando as
the prosecutor's office, a mandatory drug testing can never be the trial judge.
random or suspicionless. The ideas of randomness and being
suspicionless are antithetical to their being made defendants in a Under the political offense doctrine, "common crimes, perpetrated in
criminal complaint. They are not randomly picked; neither are they furtherance of a political offense, are divested of their character as "common"
beyond suspicion. When persons suspected of committing a crime are offenses and assume the political complexion of the main crime of which they
charged, they are singled out and are impleaded against their will. are mere ingredients, and, consequently, cannot be punished separately from
The persons thus charged, by the bare fact of being hailed before the the principal offense, or complexed with the same, to justify the imposition of
prosecutor's office and peaceably submitting themselves to drug a graver penalty."
testing, if that be the case, do not necessarily consent to the
Any ordinary act assumes a different nature by being absorbed in the crime of
procedure, let alone waive their right to privacy. To impose
rebellion. Thus, when a killing is committed in furtherance of rebellion, the
mandatory drug testing on the accused is a blatant attempt to harness
killing is not homicide or murder. Rather, the killing assumes the political
a medical test as a tool for criminal prosecution, contrary to the stated
complexion of rebellion as its mere ingredient and must be prosecuted and
objectives of RA 9165. Drug testing in this case would violate a
punished as rebellion alone.
persons' right to privacy guaranteed under Sec. 2, Art. III of the
Constitution. Worse still, the accused persons are veritably forced to But when the political offense doctrine is asserted as a defense in the trial
incriminate themselves. court, it becomes crucial for the court to determine whether the act of killing
was done in furtherance of a political end, and for the political motive of the
ii. Saturnino C. Ocampo vs. Hon. Ephrem S. Abando, G.R. No. 176830,
act to be conclusively demonstrated.
February 11, 2014
iii. Stonehill vs. Diokno, No. L-19550, June 19, 1967
RULING: The petitioners were not denied due process during preliminary
investigation. In the context of a preliminary investigation, the right to due RULING: The search warrants were not validly issued. The warrant was issued
process of law entails the opportunity to be heard. It serves to accord an from mere allegation that petitioners committed a “violation of Central Bank

M.R.A.D.C. LUMBRE 146


CONSTITUTIONAL LAW REVIEW

Laws, Tariff and Customs Laws, Internal Revenue (Code) and Revised Penal 2. By the Commissioner of Immigration, upon recommendation by the
Code.” As no specific violation has been alleged, it was impossible for Board of Commissioners, under Section 37 of Commonwealth Act No. 613.
the judges who issued said warrants to have found the existence of
probable cause, for the same presupposes the introduction of competent It is thus doubtful whether the arrest of an individual may be ordered by any
proof that the party against whom it is sought has performed or committed authority other than the judge if the purpose is merely to determine the existence
violations of the law. In other words, it would be a legal heresy, of the highest of a probable cause, leading to an administrative investigation. The Constitution
order, to convict anybody of a “violation of Central Bank Laws, Tariff and does not distinguish between warrants in a criminal case and administrative
Customs Laws, Internal Revenue (Code) and Revised Penal Code,” — as warrants in administrative proceedings. And, if one suspected of having committed
alleged in the aforementioned applications — without reference to any a crime is entitled to a determination of the probable cause against him, by a judge,
determinate provision of said laws or codes. General warrants are also to be why should one suspected of a violation of an administrative nature deserve less
eliminated, as the legality or illegality of petitioners’ transactions is immaterial guarantee? Of course it is different if the order of arrest is issued to carry out a
to the invalidity of the general warrant that sought these effects to be searched final finding of a violation, either by an executive or legislative officer or agency
and seized: “Books of accounts, financial records, vouchers, journals, duly authorized for the purpose, as then the warrant is not that mentioned in the
correspondence, receipts, ledgers, portfolios, credit journals, typewriters, and Constitution which is issuable only on probable cause. Such, for example, would be
other documents and/or papers showing all business transactions including a warrant of arrest to carry out a final order of deportation, or to effect compliance
disbursement receipts, balance sheets and related profit and loss statements.” of an order of contempt.

The Court also holds that the only practical means of enforcing the The contention of the Solicitor General that the arrest of a foreigner is necessary
constitutional injunction against unreasonable searches and seizures is, in the to carry into effect the power of deportation is valid only when, as already stated,
language of the Federal Supreme Court: x x x If letters and private there is already an order of deportation. To carry out the order of deportation, the
documents can thus be seized and held and used in evidence against President obviously has the power to order the arrest of the deportee. But,
a citizen accused of an offense, the protection of the 4th Amendment, certainly, during the investigation, it is not indispensable that the alien be arrested.
declaring his rights to be secure against such searches and seizures, It is enough, as was true before the executive order of President Quirino, that a
is of no value, and, so far as those thus placed are concerned, might as well bond be required to insure the appearance of the alien during the investigation, as
be stricken from the Constitution. The efforts of the courts and their officials was authorized in the executive order of President Roxas.
to bring the guilty to punishment, praiseworthy as they are, are not to be aided d. Knock and Announce Principle
by the sacrifice of those great principles established by years of endeavor and
suffering which have resulted in their embodiment in the fundamental law of People vs. Huang Zhen Hua, GR No. 139301, September 29, 2004
the land.
ISSUE: Whether there were irregularities attending the issuance and
The Constitution also requires the things to be seized described with implementation of the search warrant, as follows: 1. The policemen who
particularity. This is to eliminate general warrants. implemented the search warrant failed in their duty to show to her the said warrant,
inform her of their authority and explain their presence in the condominium unit;
c. Administrative warrants of arrest 2. The policemen gained entry into the condominium unit by force while she was
Qua Chee Gan vs. The Deportation Board, 9 SCRA 27 (1963) sleeping; and 3. Articles and personal affects owned by her and Law were taken
and confiscated by the policemen, although not specified in the search warrant.
ISSUE: Whether the President has the power to deport aliens, and consequently,
the delegation to the Deportation Board of the ancillary power to investigate, as RULING: No. The constitutional proscription against unreasonable search
there must be a legislation authorizing the same. and seizure applies to Filipino citizens, as well as to aliens temporarily
residing in the country. The rules against search and seizure forbids every search
RULING: Yes. Decision was affirmed, but with modifications. that is unreasonable; it protects all those suspected or known to be offenders, as
well as the innocent. The guarantee is as important and imperative as the
(RULING RELATED TO THE TOPIC: EO 398, insofar as it empowers the guarantee of the other fundamental rights of the citizens. Section 7, Rule 126 of
Deportation Board to issue warrant of arrest upon the filing of formal the Revised Rules of Criminal Procedure provides:
charges against an alien and to fix bond and prescribe the conditions for
the temporary release of said aliens, is declared illegal.) SEC. 7. Right to break door or window to effect search. The officer, if refused
admittance to the place of directed search after giving notice of his purpose
Under the present and existing laws, therefore, deportation of an undesirable alien and authority, may break open any outer or inner door or window of a house
may be effected in two ways: or any part of a house or anything therein to execute the warrant or liberate
1. By order of the President, after due investigation, pursuant to Section 69 himself or any person lawfully aiding him when unlawfully detained therein.
of the Revised Administrative Code, and

M.R.A.D.C. LUMBRE 147


CONSTITUTIONAL LAW REVIEW

The police officers were obliged to give the appellant notice, show to her case did not constitute as probable cause and thus, do not justify
their authority, and demand that they be allowed entry. They may only warrantless arrest. The search cannot therefore be denominated as
break open any outer or inner door or window of a house to execute the incidental to an arrest. In a search incidental to a lawful arrest, as
search warrant if, after such notice and demand, such officers are refused the precedent arrest determines the validity of the incidental
entry to the place of directed search. This is known as the knock and search, the legality of the arrest is questioned in a large majority
announce principle which is embodied in Anglo-American Law. of these cases, e.g., whether an arrest was merely used as a
pretext for conducting a search. In this instance, the law requires
Generally, officers implementing a search warrant must announce their presence, that there be first a lawful arrest before a search can be made -
identify themselves to the accused and to the persons who rightfully have the process cannot be reversed.
possession of the premises to be searched, and show to them the search warrant
to be implemented by them and explain to them said warrant in a language or The Court reiterated that the search was not incidental to an arrest. There
dialect known to and understood by them. The requirement is not a mere was no warrant of arrest and the warrantless arrest did not fall under the
procedural formality but is of the essence of the substantial provision which exemptions allowed by the Rules of Court as already shown. From all
safeguards individual liberty. indications, the search was nothing but a fishing expedition. The Court
thought it was worth mentioning that after introducing themselves, the
Unless the person to whom the warrant is addressed and whose property is to be police officers immediately inquired about the contents of the bag. Hence,
searched is notified of the search warrant and apprised of the authority of the the Court said, “What else could have impelled the officers from displaying
person serving the warrant, he may consider the unannounced intrusion into the such inordinate interest in the bag but to ferret out evidence and discover
premises as an unlawful aggression on his property which he will be justified in if a felony had indeed been committed by Chua -- in effect to "retroactively
resisting, and in the process, may cause injury even to the life of the officer establish probable cause and validate an illegal search and seizure."
implementing the warrant for which he would not be criminally liable.
The Court also emphasized that it is fundamental, however, that to
Unannounced intrusion into the premises is permissible when: constitute a waiver, it must first appear that the right exists; secondly,
(a) a party whose premises or is entitled to the possession thereof refuses, that the person involved had knowledge, actual or constructive, of the
upon demand, to open it; existence of such a right; and lastly, that said person had an actual
intention to relinquish the right. CHUA never exhibited that he knew,
(b) when such person in the premises already knew of the identity of the actually or constructively of his right against unreasonable searches or
officers and of their authority and persons; that he intentionally conceded the same.
(c) when the officers are justified in the honest belief that there is an imminent 2. People vs. Tudtud, 412 SCRA 142 (2003)
peril to life or limb; and
RULING: There was no valid search incidental to a lawful arrest.
(d) when those in the premises, aware of the presence of someone outside
(because, for example, there has been a knock at the door), are then engaged The rule is that a search and seizure must be carried out through or with
in activity which justifies the officers to believe that an escape or the a judicial warrant; otherwise, such search and seizure becomes
destruction of evidence is being attempted. "unreasonable" within the meaning of the above-quoted constitutional
provision, and any evidence secured thereby, will be inadmissible in
In order to justify a no-knock entry, the police must have a reasonable suspicion evidence "for any purpose in any proceeding." Section 3 (2), Article III of
that knocking and announcing their presence, under the particular circumstances, the Constitution explicitly provides:
would be dangerous or futile, or that it would inhibit the effective investigation of
the crime by, for example, allowing the destruction of evidence. This standard as (2) Any evidence obtained in violation of . . . the preceding section shall
opposed to a probable-cause requirement strikes the appropriate balance between be inadmissible for any purpose in any proceeding.
the legitimate law enforcement concerns at issue in the execution of search The proscription in Section 2, Article III, however, covers only
warrants and the individual privacy interest affected by no-knock entries. "unreasonable" searches and seizures. The following instances are not
e. Warrantless searches deemed "unreasonable" even in the absence of a warrant:

i. Search incidental to a lawful arrest 1. Warrantless search incidental to a lawful arrest. (Sec. 12,
Rule 126 of the Rules of Court and prevailing jurisprudence);
1. People vs. Chua Ho San, 307 SCRA 432 (1999)
2. Search of evidence in "plain view."
RULING: Guided by these principles, the Court then held that the facts,
such as report received by the police, entry of Chua in the Philippines, The elements are:
actuation of Chua before his arrest, the bag containing shabu, etc., in the

M.R.A.D.C. LUMBRE 148


CONSTITUTIONAL LAW REVIEW

(a) a prior valid intrusion based on the valid warrantless ii. Consented search –
arrest in which the police are legally present in the pursuit
of their official duties; People vs. Leila Johnson, GR No. 138881, December 18, 2000

(b) the evidence was inadvertently discovered by the police RULING: The (consented) search was valid. (By entering such airport, a
who have the right to be where they are; person is deemed to have consented to the search.)

(c) the evidence must be immediately apparent; Persons may lose the protection of the search and seizure clause
by exposure of their persons or property to the public in a manner
(d) "plain view" justified mere seizure of evidence without reflecting a lack of subjective expectation of privacy, which
further search; expectation society is prepared to recognize as reasonable. Such
recognition is implicit in airport security procedures. With increased
3. Search of a moving vehicle. concern over airplane hijacking and terrorism has come increased security at
Highly regulated by the government, the vehicle’s inherent mobility the nation’s airports. Passengers attempting to board an aircraft routinely pass
reduces expectation of privacy especially when its transit in public through metal detectors; their carry-on baggage as well as checked luggage
thoroughfares furnishes a highly reasonable suspicion amounting to are routinely subjected to x-ray scans. Should these procedures suggest the
probable cause that the occupant committed a criminal activity; presence of suspicious objects, physical searches are conducted to determine
what the objects are. There is little question that such searches are
4. Consented warrantless search; reasonable, given their minimal intrusiveness, the gravity of the safety
interests involved, and the reduced privacy expectations associated with
5. Customs search;
airline travel. Indeed, travelers are often notified through airport public
6. Stop and Frisk; and address systems, signs, and notices in their airline tickets that they are subject
to search and, if any prohibited materials or substances are found, such
7. Exigent and emergency circumstances. would be subject to seizure. These announcements place passengers on
Section 5 (a), Rule 113 of the Rules, in turn, allows warrantless arrests: notice that ordinary constitutional protections against warrantless searches
and seizures do not apply to routine airport procedures.
SEC. 5. Arrest without warrant; when lawful. — A peace officer or a
person may, without a warrant, arrest a person: (a) When, in his The packs of methamphetamine hydrochloride having thus been obtained
presence, the person to be arrested has committed, is actually through a valid warrantless search, they are admissible in evidence.
committing, or is attempting to commit an offense; iii. Plain View Doctrine –
It is significant to note that the search in question preceded the arrest. People vs. Doria, 301 SCRA 668, January 22, 1999
Recent jurisprudence holds that the arrest must precede the search; the
process cannot be reversed. Nevertheless, a search substantially RULING: The buy-bust operation conducted by the officers was valid.
contemporaneous with an arrest can precede the arrest if the police have However, the prosecution failed to prove that accused-appellant Gaddao
probable cause to make the arrest at the outset of the search. In this case, conspired with Doria in the sale of said drug.
this is absent.
We also hold that the warrantless arrest of accused-appellant Doria is not
The long-standing rule in this jurisdiction, applied with a great degree of unlawful. Warrantless arrests are allowed in three instances as provided by
consistency, is that "reliable information" alone is not sufficient to justify Section 5 of Rule 113 of the 1985 Rules on Criminal Procedure, to wit:
a warrantless arrest under Section 5 (a), Rule 113. The rule requires, in
"Sec. 5. Arrest without warrant; when lawful. -- A peace officer or a private
addition, that the accused perform some overt act that would indicate that
person may, without a warrant, arrest a person:
he "has committed, is actually committing, or is attempting to commit an
offense.” (a) When, in his presence, the person to be arrested has committed, is
actually committing, or is attempting to commit an offense;
Appellants implied acquiescence, if at all, could not have been more than
mere passive conformity given under coercive or intimidating (b) When an offense has in fact just been committed, and he has personal
circumstances and is, thus, considered no consent at all within the purview knowledge of facts indicating that the person to be arrested has committed
of the constitutional guarantee. Consequently, appellants lack of objection it; and
to the search and seizure is not tantamount to a waiver of his
constitutional right or a voluntary submission to the warrantless search (c) When the person to be arrested is a prisoner who escaped from a penal
and seizure. establishment or place where he is serving final judgment or temporarily

M.R.A.D.C. LUMBRE 149


CONSTITUTIONAL LAW REVIEW

confined while his case is pending, or has escaped while being transferred reasonable grounds to believe that she was engaged in drug pushing. If there
from one confinement to another. is no showing that the person who effected the warrantless arrest had, in his
own right, knowledge of facts implicating the person arrested to the
Under Section 5 (a), as above-quoted, a person may be arrested without a perpetration of a criminal offense, the arrest is legally objectionable.
warrant if he "has committed, is actually committing, or is attempting to
commit an offense." Appellant Doria was caught in the act of committing an Since the warrantless arrest of accused-appellant Gaddao was illegal, it follows
offense. When an accused is apprehended in flagrante delicto as a result of a that the search of her person and home and the subsequent seizure of the
buy-bust operation, the police are not only authorized but duty-bound to arrest marked bills and marijuana cannot be deemed legal as an incident to her
him even without a warrant. arrest. This brings us to the question of whether the trial court correctly found
that the box of marijuana was in plain view, making its warrantless seizure
The warrantless arrest of appellant Gaddao, the search of her person and valid.
residence, and the seizure of the box of marijuana and marked bills are
different matters. Objects falling in plain view of an officer who has a right to be in the position
to have that view are subject to seizure even without a search warrant and
The prosecution admits that appellant Gaddao was arrested without a warrant may be introduced in evidence. The "plain view" doctrine applies when the
of arrest and the search and seizure of the box of marijuana and the marked following requisites concur: (a) the law enforcement officer in search of the
bills were likewise made without a search warrant. It is claimed, however, that evidence has a prior justification for an intrusion or is in a position from which
the warrants were not necessary because the arrest was made in "hot pursuit" he can view a particular area; (b) the discovery of the evidence in plain view
and the search was an incident to her lawful arrest. is inadvertent; (c) it is immediately apparent to the officer that the item he
To be lawful, the warrantless arrest of appellant Gaddao must fall under any observes may be evidence of a crime, contraband or otherwise subject to
of the three (3) instances enumerated in Section 5 of Rule 113 of the 1985 seizure. The law enforcement officer must lawfully make an initial intrusion or
Rules on Criminal Procedure as aforequoted. properly be in a position from which he can particularly view the area In the
course of such lawful intrusion, he came inadvertently across a piece of
Accused-appellant Gaddao was not caught red-handed during the buy-bust evidence incriminating the accused. The object must be open to eye and hand
operation to give ground for her arrest under Section 5 (a) of Rule 113. She and its discovery inadvertent.
was not committing any crime. Contrary to the finding of the trial court, there
was no occasion at all for appellant Gaddao to flee from the policemen to justify It is clear that an object is in plain view if the object itself is plainly exposed to
her arrest in "hot pursuit." In fact, she was going about her daily chores when sight. The difficulty arises when the object is inside a closed container. Where
the policemen pounced on her. the object seized was inside a closed package, the object itself is not in plain
view and therefore cannot be seized without a warrant. However, if the
Neither could the arrest of appellant Gaddao be justified under the second package proclaims its contents, whether by its distinctive configuration, its
instance of Rule 113. "Personal knowledge" of facts in arrests without warrant transparency, or if its contents are obvious to an observer, then the contents
under Section 5 (b) of Rule 113 must be based upon "probable cause" which are in plain view and may be seized. In other words, if the package is such
means an "actual belief or reasonable grounds of suspicion." The grounds of that an experienced observer could infer from its appearance that it contains
suspicion are reasonable when, in the absence of actual belief of the arresting the prohibited article, then the article is deemed in plain view. It must be
officers, the suspicion that the person to be arrested is probably guilty of immediately apparent to the police that the items that they observe may be
committing the offense, is based on actual facts, i.e., supported by evidence of a crime, contraband or otherwise subject to seizure.
circumstances sufficiently strong in themselves to create the probable cause
of guilt of the person to be arrested. A reasonable suspicion therefore must be NOTE: In determining whether a search was validly made, one must ask 2
founded on probable cause, coupled with good faith on the part of the peace questions. First, was there a valid warrant of arrest? If not, was the search
officers making the arrest. incidental to a lawful warrantless arrest falling under Sec. 5 of Rule 113?

Accused-appellant Gaddao was arrested solely on the basis of the alleged iv. Stop-and-frisk Search –
identification made by her co-accused. PO3 Manlangit, however, declared in Malacat vs. CA, 283 SCRA 159, December 12, 1997
his direct examination that appellant Doria named his co-accused in response
to his (PO3 Manlangit's) query as to where the marked money was. Appellant RULING: The search made upon Malacat was invalid.
Doria did not point to appellant Gaddao as his associate in the drug business,
but as the person with whom he left the marked bills. This identification does The concepts of a "stop-and-frisk" and of a search incidental to a lawful arrest
not necessarily lead to the conclusion that appellant Gaddao conspired with must not be confused. These two types of warrantless searches differ in terms
her co-accused in pushing drugs. Appellant Doria may have left the money in of the requisite quantum of proof before they may be validly effected and in
her house, with or without her knowledge, with or without any conspiracy. their allowable scope. In a search incidental to a lawful arrest, as the precedent
Save for accused-appellant Doria's word, the Narcom agents had no arrest determines the validity of the incidental search. Here, there could have

M.R.A.D.C. LUMBRE 150


CONSTITUTIONAL LAW REVIEW

been no valid in flagrante delicto or hot pursuit arrest preceding the search in RULING: No. That a search and seizure must be supported by a valid warrant
light of the lack of personal knowledge on the part of Yu, the arresting officer, is not an absolute rule. There are at least three (3) well-recognized exceptions
or an overt physical act, on the part of Malacat, indicating that a crime had thereto. As set forth in the case of Manipon, Jr. vs. Sandiganbayan, these are:
just been committed, was being committed or was going to be committed. [1] a search incidental to an arrest, [2] a search of a moving vehicle, and [3]
seizure of evidence in plain view. The circumstances of the case clearly show
Plainly, the search conducted on Malacat could not have been one incidental that the search in question was made as regards a moving vehicle. Therefore,
to a lawful arrest. On the other hand, while probable cause is not required to a valid warrant was not necessary to effect the search on appellant
conduct a "stop and frisk," it nevertheless holds that mere suspicion or a hunch and his co-accused.
will not validate a "stop and frisk." A genuine reason must exist, in light of the
police officer's experience and surrounding conditions, to warrant the belief The rules governing search and seizure have over the years been steadily
that the person detained has weapons concealed about him. Finally, a "stop- liberalized whenever a moving vehicle is the object of the search on the basis
and-frisk" serves a two-fold interest: (1) the general interest of effective crime of practicality. This is so considering that before a warrant could be obtained,
prevention and detection, which underlies the recognition that a police officer the place, things and persons to be searched must be described to the
may, under appropriate circumstances and in an appropriate manner, satisfaction of the issuing judge — a requirement which boarders on the
approach a person for purposes of investigating possible criminal behavior impossible in the case of smuggling effected by the use of a moving vehicle
even without probable cause; and (2) the more pressing interest of safety and that can transport contraband from one place to another with impunity. We
self-preservation which permit the police officer to take steps to assure himself might add that a warrantless search of a moving vehicle is justified on the
that the person with whom he deals is not armed with a deadly weapon that ground that "it is not practicable to secure a warrant because the vehicle can
could unexpectedly and fatally be used against the police officer. be quickly moved out of the locality or jurisdiction in which the warrant must
be sought.
Here, there are at least three (3) reasons why the "stop-and-frisk" was invalid:
The warrantless search in the case at bench is not bereft of a probable cause.
First, there is grave doubts as to Yu's claim that Malacat was a member of the The Baguio INP Narcotics Intelligence Division received an information that
group which attempted to bomb Plaza Miranda 2 days earlier. This claim is appellant was going to transport marijuana in a bag to Manila. Their
neither supported by any police report or record nor corroborated by any other surveillance operations revealed that appellant, whose movements had been
police officer who allegedly chased that group. previously monitored by the Narcotics Division boarded a Dangwa bus bound
Second, there was nothing in Malacat's behavior or conduct which could have for Manila carrying a suspicious looking gray luggage bag. When the moving,
reasonably elicited even mere suspicion other than that his eyes were "moving public bus was stopped, her bag, upon inspection, yielded marijuana. Under
very fast" — an observation which leaves us incredulous since Yu and his those circumstances, the warrantless search of appellant's bag was not illegal.
teammates were nowhere near Malacat and it was already 6:30 p.m., thus Thus, the conviction of appellant was affirmed in toto.
presumably dusk. Malacat and his companions were merely standing at the vi. Customs search –
corner and were not creating any commotion or trouble.
1. Papa vs. Mago, No. L-27360, February 28, 1968
Third, there was at all no ground, probable or otherwise, to believe that Malacat
was armed with a deadly weapon. None was visible to Yu, for as he admitted, RULING: The seizure was valid.
the alleged grenade was "discovered" "inside the front waistline" of Malacat,
and from all indications as to the distance between Yu and Malacat, any telltale It is the settled rule, therefore, that the Bureau of Customs acquires
bulge, assuming that Malacat was indeed hiding a grenade, could not have exclusive jurisdiction over imported goods, for the purposes of
been visible to Yu. What is unequivocal then are blatant violations of Malacat's enforcement of the customs laws, from the moment the goods are actually
rights solemnly guaranteed in Sections 2 and 12(1) of Article III of the in its possession or control, even if no warrant of seizure or detention had
Constitution. previously been issued by the Collector of Customs in connection with
seizure and forfeiture proceedings.
NOTE: In stop-and-frisk incidental to a lawful warrantless arrest, there must
only be a protected search of the outer clothing of the person arrested, so as In the present case, the Bureau of Customs actually seized the goods in
to ensure that there are no weapons hidden in his person. question on November 4, 1966, and so from that date the Bureau of
Customs acquired jurisdiction over the goods for the purposes of the
v. Search of a moving vehicle – enforcement of the tariff and customs laws, to the exclusion of the regular
courts. Much less then would the Court of First Instance of Manila have
People vs. Balingan, 58 SCAD 861, 241 SCRA 277 jurisdiction over the goods in question after the Collector of Customs had
ISSUE: Whether the search done inside the Dangwa bus and the consequent issued the warrant of seizure and detention on January 12, 1967. And so,
seizure of the marijuana flowering tops were done in violation of the it cannot be said, as respondents contend, that the issuance of the said
Constitution. warrant was only an attempt to divest the respondent Judge of jurisdiction

M.R.A.D.C. LUMBRE 151


CONSTITUTIONAL LAW REVIEW

over the subject matter of the case. The court presided by respondent RULING: Search and seizure without search warrant of vessels and air
Judge did not acquire jurisdiction over the goods in question when the crafts for violations of the customs laws have been the traditional
petition for mandamus was filed before it, and so there was no need of exception to the constitutional requirement of a search warrant, because
divesting it of jurisdiction. Not having acquired jurisdiction over the goods, the vessel can be quickly moved out of the locality or jurisdiction in which
it follows that the Court of First Instance of Manila had no jurisdiction to the search warrant must be sought before such warrant could be secured;
issue the questioned order of March 7, 1967 releasing said goods. hence it is not practicable to require a search warrant before such search
or seizure can be constitutionally effected.
The Chief of the Manila Police Department, Ricardo G. Papa, having been
deputized in writing by the Commissioner of Customs, could, for the The same exception should apply to seizures of fishing vessels breaching
purposes of the enforcement of the customs and tariff laws, effect our fishery laws. They are usually equipped with powerful motors that
searches, seizures, and arrests, and it was his duty to make seizure, enable them to elude pursuing ships of the Philippine Navy or Coast Guard.
among others, of any cargo, articles or other movable property when the
same may be subject to forfeiture or liable for any fine imposed under In the case at bar, the members of the crew of the two vessels were caught
customs and tariff laws. in flagrante illegally fishing with dynamite and without the requisite
license. Thus, their apprehension without a warrant of arrest while
He could lawfully open and examine any box, trunk, envelope or other committing a crime is lawful. Consequently, the seizure of the vessel, its
containers wherever found when he had reasonable cause to suspect the equipment and dynamites therein were equally valid as an incident to a
presence therein of dutiable articles introduced into the Philippines lawful arrest.
contrary to law; and likewise to stop, search and examine any vehicle,
beast or person reasonably suspected of holding or conveying such article vii. Searches at checkpoints –
as aforesaid. People vs. Usana, 323 SCRA 754, January 28, 2000
Martin Alagao and his companion policemen had authority to effect the RULING: This Court has ruled that not all checkpoints are illegal.
seizure without any search warrant issued by a competent court. The Tariff Those which are warranted by the exigencies of public order and are
and Customs Code does not require said warrant in the instant case. The conducted in a way least intrusive to motorists are allowed.47 For,
Code authorizes persons having police authority under Section 2203 of the admittedly, routine checkpoints do intrude, to a certain extent, on
Tariff and Customs Code to enter, pass through or search any land, motorists' right to "free passage without interruption," but it cannot
inclosure, warehouse, store or building, not being a dwelling house; and be denied that, as a rule, it involves only a brief detention of travelers
also to inspect, search and examine any vessel or aircraft and any trunk, during which the vehicle's occupants are required to answer a brief
package, or envelope or any person on board, or to stop and search and question or two. For as long as the vehicle is neither searched nor its
examine any vehicle, beast or person suspected of holding or conveying occupants subjected to a body search, and the inspection of the
any dutiable or prohibited article introduced into the Philippines contrary vehicle is limited to a visual search, said routine checks cannot be
to law, without mentioning the need of a search warrant in said cases. regarded as violative of an individual's right against unreasonable
But in the search of a dwelling house, the Code provides that said: search. In fact, these routine checks, when conducted in a fixed area,
"dwelling house may be entered and searched only upon a warrant issued are even less intrusive.
by a judge or justice of the peace. . . ." It is our considered view,
therefore, that except in the case of the search of a dwelling house, The checkpoint herein conducted was in pursuance of the gun ban
persons exercising police authority under the customs law may effect enforced by the COMELEC. The COMELEC would be hard put to
search and seizure without a search warrant in the enforcement of implement the ban if its deputized agents were limited to a visual
customs laws. search of pedestrians. It would also defeat the purpose for which such
ban was instituted. Those who intend to bring a gun during said period
An automobile is a swift and powerful vehicle of recent development, would know that they only need a car to be able to easily perpetrate
which has multiplied by quantity production and taken possession of our their malicious designs.
highways in battalions until the slower, animal-drawn vehicles, with their
easily noted individuality, are rare. Constructed as covered vehicles to The facts adduced do not constitute a ground for a violation of the
standard form in immense quantities, and with a capacity for speed constitutional rights of the accused against illegal search and seizure.
rivaling express trains, they furnish for a successful commission of a crime PO3 Suba admitted that they were merely stopping cars they deemed
a disguising means of silent approach and swift escape unknown in the suspicious, such as those whose windows are heavily tinted just to
history of the world before their advent. see if the passengers thereof were carrying guns. At best they would
merely direct their flashlights inside the cars they would stop, without
2. Roldan vs. Arca, No. L-25434, July 25, 1975 opening the car's doors or subjecting its passengers to a body search.

M.R.A.D.C. LUMBRE 152


CONSTITUTIONAL LAW REVIEW

There is nothing discriminatory in this as this is what the situation believe that a crime was being committed. There was consequently
demands. more than sufficient probable cause to warrant their action.

We see no need for checkpoints to be announced, as the accused have Prior to the raid, there was a surveillance conducted on the premises
invoked. Not only would it be impractical, it would also forewarn those wherein the surveillance team was fired at by a group of men coming
who intend to violate the ban. Even so, badges of legitimacy of from the Eurocar building. When the military operatives raided the
checkpoints may still be inferred from their fixed location and the place, the occupants thereof refused to open the door despite
regularized manner in which they are operated. requests for them to do so, thereby compelling the former to break
into the office. The Eurocar Sales Office is obviously not a gun store
Even though there was ample opportunity to obtain a search warrant, and it is definitely not an armory or arsenals which are the usual
we cannot invalidate the search of the vehicle, for there are depositories for explosives and ammunition. It is primarily and solely
indications that the search done on the car of Escaño was consented engaged in the sale of automobiles. The presence of an unusual
to by him. Both Lopez and Usana testified that Escaño was with the quantity of high-powered firearms and explosives could not be
police officers when they searched the car. justifiably or even colorably explained. In addition, there was general
There was no apparent objection made by Escaño as he seemed to chaos and disorder at that time because of simultaneous and intense
have freely accompanied the police officers to the car. PO3 Suba, on firing within the vicinity of the office and in the nearby Camp
the other hand, testified that "Escaño readily agreed to open the Aguinaldo which was under attack by rebel forces. The courts in the
trunk," upon request of SPO4 de los Santos. But according to Escaño, surrounding areas were obviously closed and, for that matter, the
he refused the request of the police officers to search his car. We must building and houses therein were deserted.
give credence to the testimony of PO3 Suba. Not only is it buttressed In the case of Umil v. Ramos, the Court, in analogy applied the ruling
by the testimony of Usana and Lopez that Escaño freely accompanied to the present case that: “The arrest of persons involved in the
the police officers to the car, it is also deemed admitted by Escaño in rebellion whether as its fighting armed elements, or for committing
failing to appeal the decision. The findings of fact of the trial court are non-violent acts but in furtherance of the rebellion, is more an act of
thus deemed final as against him. capturing them in the course of an armed conflict, to quell the
Despite the validity of the search, we cannot affirm the conviction of rebellion, than for the purpose of immediately prosecuting them in
Usana and Lopez for violation of R.A. No. 6425, as amended. The court for a statutory offense. The arrest, therefore, need not follow
following facts militate against a finding of conviction: (1) the car the usual procedure in the prosecution of offenses which requires the
belonged to Escaño; (2) the trunk of the car was not opened soon determination by a judge of the existence of probable cause before
after it was stopped and after the accused were searched for firearms; the issuance of a judicial warrant of arrest and the granting of bail if
(3) the car was driven by a policeman from the place where it was the offense is bailable. Obviously the absence of a judicial warrant is
stopped until the police station; (4) the car's trunk was opened, with no legal impediment to arresting or capturing persons committing
the permission of Escaño, without the presence of Usana and Lopez; overt acts of violence against government forces, or any other milder
and (5) after arrival at the police station and until the opening of the acts but really in pursuance of the rebellious movement. The arrest
car's trunk, the car was in the possession and control of the police or capture is thus impelled by the exigencies of the situation that
authorities. No fact was adduced to link Usana and Lopez to the involves the very survival of society and its government and duly
hashish found in the trunk of the car. Their having been with Escaño constituted authorities. If killing and other acts of violence against the
in the latter's car before the "finding" of the hashish sometime after rebels find justification in the exigencies of armed hostilities which
the lapse of an appreciable time and without their presence left much (are) of the essence of waging a rebellion or insurrection, most
to be desired to implicate them to the offense of selling, distributing, assuredly so in case of invasion, merely seizing their persons and
or transporting the prohibited drug. In fact, there was no showing detaining them while any of these contingencies continues cannot be
that Usana and Lopez knew of the presence of hashish in the trunk of less justified.”
the car or that they saw the same before it was seized. ix. Search by private persons –
viii. Exigent and emergency circumstances – People vs. Marti, 193 SCRA 57 (1991)
People vs. De Gracia, 233 SCRA 716 (1994) RULING: The search made by a private individual is valid. The
RULING: The warrantless search and seizure made is valid, and the constitutional right against unreasonable search and seizure refers to
instant case falls under one of the exceptions to the prohibition the immunity of one's person, whether citizen or alien, from
against a warrantless search, that is, under exigent and emergency interference by government. Its protection is directed only to
circumstances. The military operatives had reasonable ground to governmental action.

M.R.A.D.C. LUMBRE 153


CONSTITUTIONAL LAW REVIEW

This right do not require exclusion of evidence obtained through a As applied to in flagrante delicto arrests, it is settled that reliable
search by a private citizen. In this case, the evidence was primarily information alone, absent any overt act indicative of a felonious
discovered and obtained by a private person, acting in a private enterprise in the presence and within the view of the arresting
capacity and without the intervention of State authorities. Therefore, officers, are not sufficient to constitute probable cause that would
there is no reason why it should not be admitted to prosecute him. justify an in flagrante delicto arrest. To constitute a valid in flagrante
delicto arrest, two requisites must concur:
Marti, however, alleged that the NBI agents made an illegal search
and seizure of the evidence. (1) the person to be arrested must execute an overt act
indicating that he has just committed, is actually
The Court pointed out that: a) It was the proprietor who made a committing, or is attempting to commit a crime; and
reasonable search of the packages in compliance with SOP, and b)
the mere presence of the NBI agents did not convert the reasonable (2) such overt act is done in the presence or within the
search effected into a warrantless search and seizure. Merely to view of the arresting officer.
observe and look at that which is in plain sight is not a search.
In this case, accused manifested no outward indication that would
f. Warrantless arrests justify their arrest. In holding a bag on board a trisikad, accused could
not be said to be committing, attempting to commit or have
i. Section 5, Rule 113, Revised Rules of Criminal Procedure committed a crime. It matters not that accused Molina responded
Section 5, Rule 113, Revised Rules of Criminal Procedure. Arrest without Boss, if possible we will settle this to the request of SPO1 Pamplona
warrant; when lawful. — A peace officer or a private person may, without to open the bag. Such response which allegedly reinforced the
a warrant, arrest a person: suspicion of the arresting officers that accused were committing a
crime, is an equivocal statement which standing alone will not
(a) When, in his presence, the person to be arrested has committed, is constitute probable cause to effect an in flagrante delicto arrest. Note
actually committing, or is attempting to commit an offense; that were it not for SPO1 Paguidopon (who did not participate in the
arrest but merely pointed accused to the arresting officers), accused
(b) When an offense has just been committed, and he has probable cause
could not be the subject of any suspicion, reasonable or otherwise.
to believe based on personal knowledge of facts or circumstances that the
person to be arrested has committed it; and While SPO1 Paguidopon claimed that he and his informer conducted
a surveillance of accused Mula, SPO1 Paguidopon, however, admitted
(c) When the person to be arrested is a prisoner who has escaped from a
that he only learned Mula’s name and address after the arrest. What
penal establishment or place where he is serving final judgment or is
is more, it is doubtful if SPO1 Paguidopon indeed recognized accused
temporarily confined while his case is pending, or has escaped while being
Mula. It is worthy to note that, before the arrest, he was able to see
transferred from one confinement to another.
Mula in person only once, pinpointed to him by his informer while they
In cases falling under paragraph (a) and (b) above, the person arrested were on the side of the road. These circumstances could not have
without a warrant shall be forthwith delivered to the nearest police station afforded SPO1 Paguidopon a closer look at accused Mula, considering
or jail and shall be proceeded against in accordance with section 7 of Rule that the latter was then driving a motorcycle when SPO1 Paguidopon
112. caught a glimpse of him. With respect to accused Molina, SPO1
Paguidopon admitted that he had never seen him before the arrest.
1. In flagrante delicto – Evidently, SPO1 Paguidopon, who acted as informer of the arresting
People vs. Molina, 352 SCRA 174, February 19, 2001 officers, more so the arresting officers themselves, could not have
been certain of accused-appellants identity, and were, from all
RULING: The search made upon the accused inside a trisikad was indications, merely fishing for evidence at the time of the arrest.
not valid. Probable cause means an actual belief or reasonable
grounds of suspicion. The grounds of suspicion are reasonable when, 2. Arrest in hot pursuit –
in the absence of actual belief of the arresting officers, the suspicion Pestilos vs. Generoso, G.R. No. 182601, 10 November 2014;
that the person to be arrested is probably guilty of committing the
offense, is based on actual facts, i.e., supported by circumstances RULING: The petitioners were validly arrested. In light of the
sufficiently strong in themselves to create the probable cause of guilt discussion on the developments of Section 5(b), Rule 113 of the
of the person to be arrested. A reasonable suspicion therefore must Revised Rules of Criminal Procedure and our jurisprudence on the
be founded on probable cause, coupled with good faith on the part of matter, we hold that the following must be present for a valid
the peace officers making the arrest. warrantless arrest: 1) the crime should have been just committed;
and 2) the arresting officer's exercise of discretion is limited by the

M.R.A.D.C. LUMBRE 154


CONSTITUTIONAL LAW REVIEW

standard of probable cause to be determined from the facts and There is no hard and fast rule on the lapse of time before the
circumstances within his personal knowledge. The requirement of the warrantless arrest under hot pursuit may be deemed valid. However,
existence of probable cause objectifies the reasonableness of the the case of Pestilos v. Generoso is the most recent.
warrantless arrest for purposes of compliance with the Constitutional
mandate against unreasonable arrests. 3. Continuing offense –

To summarize, the arresting officers went to the scene of the crime Umil vs. Ramos, GR No. 79731, July 9, 1990
upon the complaint of Atty. Generoso of his alleged mauling; the RULING: Rolando Dural was arrested for being a member of the New
police officers responded to the scene of the crime less than one (1) Peoples Army (NPA), an outlawed subversive organization.
hour after the alleged mauling; the alleged crime transpired in a Subversion being a continuing offense, the arrest of Rolando Dural
community where Atty. Generoso and the petitioners reside; Atty. without warrant is justified as it can be said that he was committing
Generoso positively identified the petitioners as those responsible for an offense when arrested. The crimes of rebellion, subversion,
his mauling and, notably, the petitioners and Atty. Generoso lived conspiracy or proposal to commit such crimes, and crimes or offenses
almost in the same neighborhood; more importantly, when the committed in furtherance thereof or in connection therewith
petitioners were confronted by the arresting officers, they did not constitute direct assaults against the State and are in the nature of
deny their participation in the incident with Atty. Generoso, although continuing crimes.
they narrated a different version of what transpired.
It is to be noted that, in all the petitions here considered, criminal
With these facts and circumstances that the police officers gathered charges have been filed in the proper courts against the petitioners.
and which they have personally observed less than one hour from the The rule is, that if a person alleged to be restrained of his liberty is in
time that they have arrived at the scene of the crime until the time of the custody of an officer under process issued by a court judge, and
the arrest of the petitioners, we deem it reasonable to conclude that that the court or judge had jurisdiction to issue the process or make
the police officers had personal knowledge of facts or circumstances the order, of if such person is charged before any court, the writ of
justifying the petitioners' warrantless arrests. These circumstances habeas corpus will not be allowed.
were well within the police officers' observation, perception and
evaluation at the time of the arrest. These circumstances qualify as A writ of habeas corpus is no longer available after an information is
the police officers' personal observation, which are within their filed against the person detained and a warrant of arrest or an order
personal knowledge, prompting them to make the warrantless of commitment, is issued by the court where said information has
arrests. been filed in all petitions for habeas corpus the court must inquire into
every phase and aspect of petitioner's detention-from the moment
In determining the reasonableness of the warrantless arrests, it is petition was taken into custody up to the moment the court passes
incumbent upon the courts to consider if the police officers have upon the merits of the petition;" and "only after such a scrutiny can
complied with the requirements set under Section 5(b), Rule 113 of the court satisfy itself that the due process clause of our Constitution
the Revised Rules of Criminal Procedure, specifically, the requirement has in fact been satisfied." This is exactly what the Court has done in
of immediacy; the police officer's personal knowledge of facts or the petitions at bar. This is what should henceforth be done in all
circumstances; and lastly, the propriety of the determination of future cases of habeas corpus. In short, all cases involving deprivation
probable cause that the person sought to be arrested committed the of individual liberty should be promptly brought to the courts for their
crime. immediate scrutiny and disposition.
The records show that soon after the report of the incident occurred, NOTE: Question: Whether a law amending R.A. No. 9372 allowing
SPOl Monsalve immediately dispatched the arresting officer, SP02 the Secretary of Justice to issue an authority to conduct wiretapping
Javier, to render personal assistance to the victim. This fact alone is valid?
negates the petitioners' argument that the police officers did not have
personal knowledge that a crime had been committed - the police Answer: Yes. Section 3 of Article III provides, “The privacy of
immediately responded and had personal knowledge that a crime had communication and correspondence shall be inviolable except upon
been committed. lawful order of the court, or when public safety or order requires
otherwise, as prescribed by law.” Thus, there are two (2) exceptions
NOTE: It is not proper to say that the arresting police officer had to the privacy of communication and correspondence: (1) lawful order
probable cause; there are only two (2) types of probable cause – of the court; and (2) when public safety or order requires otherwise,
executive (by the prosecutor) and judicial (by the judge). as prescribed by law.
At present, there is no law allowing the same.

M.R.A.D.C. LUMBRE 155


CONSTITUTIONAL LAW REVIEW

4. Privacy of Communication and Correspondence identities. All other data to be collected or seized or disclosed will require a
court warrant. Service providers are required to cooperate and assist law
Section 3. The privacy of communication and correspondence shall be inviolable except enforcement authorities in the collection or recording of the above-stated
upon lawful order of the court, or when public safety or order requires otherwise, as information.
prescribed by law
The Court recognizes certain constitutional guarantees work together to create
Any evidence obtained in violation of this or the preceding section shall be inadmissible zones of privacy wherein governmental powers may not intrude, and that there
for any purpose in any proceeding. exists an independent constitutional right of privacy. Such right to be left alone
a. Right to Privacy has been regarded as the beginning of all freedoms. But that right is not
unqualified. In Whalen v. Roe, the United States Supreme Court classified
i. Categories - Disini, Jr. vs. The Secretary of Justice, G.R. No. 203335, privacy into two categories: decisional privacy and informational privacy.
February 18, 2014 Decisional privacy involves the right to independence in making
certain important decisions, while informational privacy refers to the
RULING: Section 4(b)(3) punishing computer-related identity theft is
interest in avoiding disclosure of personal matters. It is the latter
constitutional.
right—the right to informational privacy—that those who oppose
Relevant to any discussion of the right to privacy is the concept known as the government collection or recording of traffic data in real-time seek to
"Zones of Privacy." The Court explained in "In the Matter of the Petition for protect.
Issuance of Writ of Habeas Corpus of Sabio v. Senator Gordon" the relevance
Informational privacy has two aspects: the right not to have private
of these zones to the right to privacy:
information disclosed, and the right to live freely without surveillance
Zones of privacy are recognized and protected in our laws. Within these zones, and intrusion. In determining whether or not a matter is entitled to
any form of intrusion is impermissible unless excused by law and in accordance the right to privacy, this Court has laid down a two-fold test. The first
with customary legal process. The meticulous regard we accord to these zones is a subjective test, where one claiming the right must have an actual
arises not only from our conviction that the right to privacy is a "constitutional or legitimate expectation of privacy over a certain matter. The second
right" and "the right most valued by civilized men," but also from our is an objective test, where his or her expectation of privacy must be
adherence to the Universal Declaration of Human Rights which mandates that, one society is prepared to accept as objectively reasonable.
"no one shall be subjected to arbitrary interference with his privacy" and
Since the validity of the cybercrime law is being challenged, not in relation to
"everyone has the right to the protection of the law against such interference
its application to a particular person or group, petitioners’ challenge to Section
or attacks."
12 applies to all information and communications technology (ICT) users,
Two constitutional guarantees create these zones of privacy: (a) the meaning the large segment of the population who use all sorts of electronic
right against unreasonable searches and seizures, which is the basis of the devices to communicate with one another. Consequently, the expectation of
right to be let alone, and (b) the right to privacy of communication and privacy is to be measured from the general public’s point of view. Without
correspondence. In assessing the challenge that the State has impermissibly reasonable expectation of privacy, the right to it would have no basis in fact.
intruded into these zones of privacy, a court must determine whether a
Mainly, the Court struck down this provision as vague because of the word
person has exhibited a reasonable expectation of privacy and, if so,
“with due case” it held:
whether that expectation has been violated by unreasonable
government intrusion. Section 12 empowers law enforcement authorities, "with due cause," to collect
or record by technical or electronic means traffic data in real-time. Petitioners
The usual identifying information regarding a person includes his name, his
point out that the phrase "due cause" has no precedent in law or jurisprudence
citizenship, his residence address, his contact number, his place and date of
and that whether there is due cause or not is left to the discretion of the police.
birth, the name of his spouse if any, his occupation, and similar data. The law
Replying to this, the Solicitor General asserts that Congress is not required to
punishes those who acquire or use such identifying information without right,
define the meaning of every word it uses in drafting the law.
implicitly to cause damage. Petitioners simply fail to show how government
effort to curb computer-related identity theft violates the right to privacy and Indeed, courts are able to save vague provisions of law through statutory
correspondence as well as the right to due process of law. construction. But the cybercrime law, dealing with a novel situation, fails to
hint at the meaning it intends for the phrase "due cause." The Solicitor General
Section 12, allowing the collection of real-time data, was struck down
suggests that "due cause" should mean "just reason or motive" and "adherence
as invalid.
to a lawful procedure." But the Court cannot draw this meaning since Section
Traffic data refer only to the communication’s origin, destination, route, time, 12 does not even bother to relate the collection of data to the probable
date, size, duration, or type of underlying service, but not content, nor commission of a particular crime. It just says, "with due cause," thus justifying

M.R.A.D.C. LUMBRE 156


CONSTITUTIONAL LAW REVIEW

a general gathering of data. It is akin to the use of a general search warrant knows with the other. And this has nothing to do with the duty of fidelity that
that the Constitution prohibits. each owes to the other.

Due cause is also not descriptive of the purpose for which data collection will ii. Anti-wiretapping Act (RA 4200)
be used. Will the law enforcement agencies use the traffic data to identify the
perpetrator of a cyber attack? Or will it be used to build up a case against an 1. Salcedo-Ortanez vs. CA, 235 SCRA 111, August 4, 1994
identified suspect? Can the data be used to prevent cybercrimes from RULING: The tape recordings of telephone conversations of private
happening? respondent’s wife with unidentified persons are inadmissible in evidence (in
The authority that Section 12 gives law enforcement agencies is too sweeping the complaint for annulment of marriage).
and lacks restraint. While it says that traffic data collection should not disclose Rep. Act No. 4200 entitled "An Act to Prohibit and Penalize Wire Tapping and
identities or content data, such restraint is but an illusion. Admittedly, nothing Other Related Violations of the Privacy of Communication, and for other
can prevent law enforcement agencies holding these data in their hands from purposes" expressly makes such tape recordings inadmissible in evidence. The
looking into the identity of their sender or receiver and what the data contains. relevant provisions of Rep. Act No. 4200 are as follows:
This will unnecessarily expose the citizenry to leaked information or, worse, to
extortion from certain bad elements in these agencies. Sec. 1. It shall be unlawful for any person, not being authorized by all the
parties to any private communication or spoken word, to tap any wire or
Section 12, of course, limits the collection of traffic data to those "associated cable, or by using any other device or arrangement, to secretly overhear,
with specified communications." But this supposed limitation is no limitation at intercept, or record such communication or spoken word by using a device
all since, evidently, it is the law enforcement agencies that would specify the commonly known as a dictaphone or dictagraph or detectaphone or
target communications. The power is virtually limitless, enabling law walkie-talkie or tape-recorder, or however otherwise described.
enforcement authorities to engage in "fishing expedition," choosing whatever
specified communication they want. This evidently threatens the right of Sec. 4. Any communication or spoken word, or the existence, contents,
individuals to privacy. substance, purport, or meaning of the same or any part thereof, or any
information therein contained, obtained or secured by any person in
b. Concept of Informational Privacy violation of the preceding sections of this Act shall not be admissible in
i. Zulueta vs. CA, 253 SCRA 699 evidence in any judicial, quasi-judicial, legislative or administrative
hearing or investigation.
RULING: The documents and papers gathered by the wife of private
respondent are inadmissible in evidence. Clearly, public respondents trial court and Court of Appeals failed to consider
the afore-quoted provisions of the law in admitting in evidence the cassette
The constitutional injunction declaring "the privacy of communication and tapes in question. Absent a clear showing that both parties to the telephone
correspondence [to be] inviolable" is no less applicable simply because it is the conversations allowed the recording of the same, the inadmissibility of the
wife (who thinks herself aggrieved by her husband's infidelity) who is the party subject tapes is mandatory under Rep. Act No. 4200.
against whom the constitutional provision is to be enforced. The only exception
to the prohibition in the Constitution is if there is a "lawful order [from a] court 2. Navarro vs. CA, GR No. 121087, August 26, 1999
or when public safety or order requires otherwise, as prescribed by law." Any RULING: The exchange between Navarro and Lingan secretly recorded by
violation of this provision renders the evidence obtained inadmissible "for any Jalbuena and submitted to the court is admissible.
purpose in any proceeding."
R.A. No. 4200 provides:
The intimacies between husband and wife do not justify any one of them in
breaking the drawers and cabinets of the other and in ransacking them for any SECTION 1. It shall be unlawful for any person, not being authorized by
telltale evidence of marital infidelity. A person, by contracting marriage, does all the parties to any private communication or spoken word, to tap any
not shed his/her integrity or his right to privacy as an individual and the wire or cable, or by using any other device or arrangement, to secretly
constitutional protection is ever available to him or to her. overhear, intercept, or record such communication or spoken word by
using a device commonly known as a dictaphone or dictagraph or
The law insures absolute freedom of communication between the spouses by detectaphone or walkie-talkie or tape-recorder, or however otherwise
making it privileged. Neither husband nor wife may testify for or against the described:
other without the consent of the affected spouse while the marriage subsists.
Neither may be examined without the consent of the other as to any It shall also be unlawful for any person, be he a participant or not in the
communication received in confidence by one from the other during the act or acts penalized in the next preceding sentence, to knowingly possess
marriage, save for specified exceptions. But one thing is freedom of any tape record, wire record, disc record, or any other such record, or
communication; quite another is a compulsion for each one to share what one copies thereof, of any communication or spoken word secured either

M.R.A.D.C. LUMBRE 157


CONSTITUTIONAL LAW REVIEW

before or after the effective date of this Act in the manner prohibited by search engine, which serves to make the service offered by that engine
this law; or to replay the same for any other person or persons; or to profitable.” [para. 55]
communicate the contents thereof, either verbally or in writing, or to
furnish transcriptions thereof, whether complete or partial, to any other Lastly, the Court addressed the extent of Google’s responsibility as an
person: Provided, That the use of such record or any copies thereof as Internet search engine with respect to personal information published by
evidence in any civil, criminal investigation or trial of offenses mentioned third party websites and subsequently sought to be removed or altered by
in section 3 hereof, shall not be covered by this prohibition. the data subject. Applicable provisions here are Article 12(b) and 14(a) of
the Directive. Under Article 12(b), every personal data subject has the
SEC. 4. Any communication or spoken word, or the existence, contents, right to obtain from controller “as appropriate the rectification, erasure or
substance, purport, effect, or meaning of the same or any part thereof, or blocking of data the processing of which does not comply with the
any information therein contained obtained or secured by any person in provisions of this Directive, in particular because of the incomplete or
violation of the preceding sections of this Act shall not be admissible in inaccurate nature of the data.” Article 14(a) also grants the data subject
evidence in any judicial, quasi-judicial, legislative or administrative the right to “object at any time on compelling legitimate grounds relating
hearing or investigation. to his particular situation to the processing of data relating to him, save
where otherwise provided by national legislation. Where there is a justified
Thus, the law prohibits the overhearing, intercepting, or recording of private objection, the processing instigated by the controller may no longer
communications. Since the exchange between petitioner Navarro and Lingan involve those data.”
was not private, its tape recording is not prohibited.
Google Spain and Google Inc. argued that based on the principle of
iii. Data Privacy Act of 2012 (RA 10173) proportionally, the removal of personal information must be addressed to
1. “Right to be Forgotten” - Google Spain vs. AEPD (Spanish Data the website that published the data and made it publicly available. And
Protection Agency) and Mario Costeja Gonzalez, C-131/12 that the publisher is in the best position to assess the lawfulness of that
information.
RULING: As to whether Google search engine must be regarded as a
“controller” of processing personal data, the Court was of the opinion that In addressing this question, the Court first underscored the fundamental
the concept of “controller” within the Directive must be interpreted broadly rights to privacy and the protection of personal data. Article 8 of the EU
in order to ensure “effective and complete protection of data subjects.” Charter of Fundamental Rights states that “[e]veryone has the right to the
[para. 34] And that it would be contrary to the objectives envisioned in protection of personal data concerning him or her. [And] [s]uch data must
the Directive to exclude the operators of Internet search engines as “they be processed fairly for specified purposes and on the basis of the consent
play a decisive role in the overall dissemination of [personal] data.” [para. of the person concerned or some other legitimate basis laid down by law.
36] Everyone has the right of access to data, which has been collected
concerning him or her, and the right to have it rectified.” As implemented,
Regarding whether the Directive as implemented by the national laws of inter alia, under Articles 6, 7, 12, 14, and 28 of the Directive 95/46, the
Spain can be applied to Google as a “controller” of processing personal Court also stressed the importance of balancing the right to privacy
data, the Court records indicated that Google Spain was established in against the right to information access.
2003 by Google Inc. to primarily act as its commercial agent in Spain “to
promote, facilitate and effect the sale of on-line advertising products and In light to the above principles, the Court held that Internet search engines
services to third parties and the marketing of that advertising.” Under are subject to “affect [] the fundamental rights to privacy and to the
Article 4(1)(a), the provisions of the Directive are applicable where: protection of personal data when the search by means of that engine is
carried out on the basis of an individual’s name.” [para. 80] However,
“the processing is carried out in the context of the activities of an Internet users’ right to access personal information through search
establishment of the controller on the territory of the Member State; when engines must also be respected, depending “on the nature of the
the same controller is established on the territory of several Member information in question and its sensitivity for the data subject’s private life
States, he must take the necessary measures to ensure that each of these and on the interest of the public in having that information, an interest
establishments complies with the obligations laid down by the national law which may vary, in particular, according to the role played by the data
applicable.” subject in public life.” [para. 81]
By taking into account the objectives of the Directive and the wording of In sum, the Court ruled that “the operator of a search engine is obliged to
Article 4(1)(a), the Court held that Google is subject to the provisions remove from the list of results displayed following a search made on the
because its subsidiary Google Spain is an establishment in Spain “intended basis of a person’s name links to web pages, published by third parties
to promote and sell, in that [country], advertising space offered by the and containing information relating to that person, also in a case where
that name or information is not erased beforehand or simultaneously from

M.R.A.D.C. LUMBRE 158


CONSTITUTIONAL LAW REVIEW

those web pages, and even, as the case may be, when its publication in directed the MISO to retrieve the messages for purposes of these cases. Based
itself on those pages is lawful.” [para. 88] The Court also held that on the certification issued by the authorized MISO personnel, the messages
individuals whose personal data are publicly available through Internet were extracted from the Yahoo and Facebook accounts of Judge San Gaspar-
search engines may “request that the information in question no longer Gito with the use of her official workstation. Hence, the exclusionary rule did
be made available to the general public on account of its inclusion in such not apply.
a list of results” as their rights to privacy and protection of personal data
override “not only the economic interest of the operator of the search e. Two-part test to determine reasonableness of person’s expectation of
engine but also the interest of the general public in having access to that privacy
information upon a search relating to the data subject’s name.” [para. 81] i. Disini, Jr. vs. The Secretary of Justice, G.R. No. 203335, February 18,
The Court, however, emphasized that the right to initiate such request 2014
may cease to exist when access to personal information “is justified by the
preponderant interest of the general public in having, on account of its RULING: In Whalen v. Roe, the United States Supreme Court classified privacy
inclusion in the list of results, access to the information in question.” into two categories: decisional privacy and informational privacy. Decisional
privacy involves the right to independence in making certain important
c. Limitations decisions, while informational privacy refers to the interest in avoiding
Section 3. xxxx except upon lawful order of the court, or when public safety or disclosure of personal matters. It is the latter right—the right to informational
order requires otherwise, as prescribed by law. privacy—that those who oppose government collection or recording of traffic
data in real-time seek to protect.
d. Exclusionary Rule
Informational privacy has two aspects: the right not to have private
Section 3. xxxx Any evidence obtained in violation of this or the preceding section information disclosed, and the right to live freely without surveillance and
shall be inadmissible for any purpose in any proceeding. intrusion. In determining whether or not a matter is entitled to the right to
privacy, this Court has laid down a two-fold test. The first is a subjective test,
i. Relate with Article 723 of the Civil Code: where one claiming the right must have an actual or legitimate expectation of
Office of the Court Administrator vs. Judge Yu, A.M. No. MTJ-12-1813, privacy over a certain matter. The second is an objective test, where his or her
22 November 2016 expectation of privacy must be one society is prepared to accept as objectively
reasonable.
RULING: The Facebook and Yahoo messages of Judge Yu to fellow Judge San
Gaspar-Gito is admissible in evidence. Since the validity of the cybercrime law is being challenged, not in relation to
its application to a particular person or group, petitioners’ challenge to Section
The exclusionary rule, or the fruit of the poisonous tree doctrine, presupposes 12 applies to all information and communications technology (ICT) users,
a violation of law on the part of the agents of the Govemment, and bars the meaning the large segment of the population who use all sorts of electronic
admission of evidence obtained in violation of the right against unreasonable devices to communicate with one another. Consequently, the expectation of
searches and seizures expressly defined under Section 2, Article III of the privacy is to be measured from the general public’s point of view. Without
Constitution. The exclusionary rule under Section 3(2), Article III of the reasonable expectation of privacy, the right to it would have no basis in fact.
Constitution refers to the prohibition against the issuance of general warrants
that encourage law enforcers to go on fishing expeditions. The Court, however, agrees with Justices Carpio and Brion that when
seemingly random bits of traffic data are gathered in bulk, pooled together,
Further, Art. 723 of the Civil Code provides: “Letters and other private and analyzed, they reveal patterns of activities which can then be used to
communications in writing are owned by the person to whom they are create profiles of the persons under surveillance. With enough traffic data,
addressed and delivered, but they cannot be published or disseminated without analysts may be able to determine a person’s close associations, religious
the consent of the writer or his heirs. However, the court may authorize views, political affiliations, even sexual preferences. Such information is likely
their publication or dissemination if the public good or the interest of beyond what the public may expect to be disclosed, and clearly falls within
justice so requires.” matters protected by the right to privacy. But has the procedure that Section
12 of the law provides been drawn narrowly enough to protect individual
Judge Yu did not specify that the State had unlawfully intruded into her privacy.
rights?
The subjects of the present inquiry were the messages sent by her to Judge
San Gaspar-Gito. Regardless of the mode of their transmission, the ownership Section 12 empowers law enforcement authorities, "with due cause," to collect
of the messages pertained to the latter as the recipient. Considering that it or record by technical or electronic means traffic data in real-time. Petitioners
was the latter who granted access to such messages, there was no violation of point out that the phrase "due cause" has no precedent in law or jurisprudence
Judge Yu's right to privacy. As such, the grant of access by Judge San Gaspar- and that whether there is due cause or not is left to the discretion of the police.
Gito did not require the consent of Judge Yu as the writer. To recall, the Court

M.R.A.D.C. LUMBRE 159


CONSTITUTIONAL LAW REVIEW

Replying to this, the Solicitor General asserts that Congress is not required to 2. Right to data inheritance – right to inherit information about a deceased
define the meaning of every word it uses in drafting the law. relative or a person unable to give consent.

Indeed, courts are able to save vague provisions of law through statutory Data Privacy Act in an EXCEPTION TO THE TERRITORIALITY PRINCIPLE
construction. But the cybercrime law, dealing with a novel situation, fails to IN CRIMINAL LAW. Thus, a violation of the provisions of the said law
hint at the meaning it intends for the phrase "due cause." The Solicitor General committed against a resident of the Philippines, regardless of the place where
suggests that "due cause" should mean "just reason or motive" and "adherence the violation was committed, may be tried in the Philippines.
to a lawful procedure." But the Court cannot draw this meaning since Section
12 does not even bother to relate the collection of data to the probable ii. In the Matter of the Petition for Habeas Corpus of Capt. Garry
commission of a particular crime. It just says, "with due cause," thus justifying Alejano, et al. vs. Gen. Pedro Cabuay, et al., GR No. 160792, August
a general gathering of data. It is akin to the use of a general search warrant 25, 2005
that the Constitution prohibits. RULING: The court ruled that the right to privacy of those detained is subject
Due cause is also not descriptive of the purpose for which data collection will to Section 4 of RA 7438, as well as to the limitations inherent in lawful
be used. Will the law enforcement agencies use the traffic data to identify the detention or imprisonment. By the very fact of their detention, pre-trial
perpetrator of a cyber attack? Or will it be used to build up a case against an detainees and convicted prisoners have a diminished expectation of privacy
identified suspect? Can the data be used to prevent cybercrimes from rights.
happening? The letters were not in a sealed envelope. The inspection of the folded letters
The authority that Section 12 gives law enforcement agencies is too sweeping is a valid measure as it serves the same purpose as the opening of sealed
and lacks restraint. While it says that traffic data collection should not disclose letters for the inspection of contraband.
identities or content data, such restraint is but an illusion. Admittedly, nothing The letters alleged to have been read by the ISAFP authorities were not
can prevent law enforcement agencies holding these data in their hands from confidential letters between the detainees and their lawyers. The petitioner
looking into the identity of their sender or receiver and what the data contains. who received the letters from detainees Trillanes and Maestrecampo was
This will unnecessarily expose the citizenry to leaked information or, worse, to merely acting as the detainees’ personal courier and not as their counsel when
extortion from certain bad elements in these agencies. he received the letters for mailing. In the present case, since the letters were
Section 12, of course, limits the collection of traffic data to those "associated not confidential communication between the detainees and their lawyers, the
with specified communications." But this supposed limitation is no limitation at officials of the ISAFP Detention Center could read the letters. If the letters are
all since, evidently, it is the law enforcement agencies that would specify the marked confidential communication between the detainees and their lawyers,
target communications. The power is virtually limitless, enabling law the detention officials should not read the letters but only open the envelopes
enforcement authorities to engage in "fishing expedition," choosing whatever for inspection in the presence of the detainees.
specified communication they want. This evidently threatens the right of NOTE: To invoke right to privacy in this case, the letter must be: (1) marked
individuals to privacy. as confidential; or (2) must be a letter to or from counsel of the detainee
NOTE: The right to privacy is covered not only in Section 3 but also in Sections (privileged communication between attorney and client).
2 and 8. The right to privacy under Section 3 is also known as informational iii. Ayer Productions Pty. Ltd. Vs. Capulong, 160 SCRA 861 (1988)
privacy. The purpose of the Data Privacy Act is to supervise private
corporations who sell personal information of persons to other private RULING: The projected motion picture “The Four Day Revolution” does not
corporations. constitute an unlawful intrusion upon private respondent’s right of privacy.

Personal information pertains not only to basic personal information, i.e. name In the case at bar, the interests observable are the right to privacy asserted
and address, but also other information that, taken together, will direct to by respondent and the right of freedom of expression invoked by petitioner
the identity of a person. taking into account the interplay of those interests, we hold that under the
particular circumstances presented, and considering the obligations assumed
Personal information includes two (2) types: (1) normal personal information; in the Licensing Agreement entered into by petitioner, the validity of such
and (2) sensitive personal information. Violation of the latter has a higher agreement will have to be upheld particularly because the limits of freedom of
penalty. expression are reached when expression touches upon matters of essentially
The right to data portability has two (2) aspects: private concern." Whether the “balancing of interest test” or the “clear and
present danger test” be applied in respect of the instant Petitions, the Court
1. Right to access data subjects – right to request personal information which believes that a different conclusion must here be reached.
the data process/controller has about a person, which must be organized and
handed out in a portable manner, i.e. in flash disks, CDs, etc.

M.R.A.D.C. LUMBRE 160


CONSTITUTIONAL LAW REVIEW

Neither private respondent nor the respondent trial Judge knew what the expectation of privacy. This reasonableness test implicates neither probable
completed film would precisely look like. There was, in other words, no “clear cause nor the warrant requirement, which are related to law enforcement.
and present danger” of any violation of any right to privacy that private
respondent could lawfully assert. O’Connor was applied in subsequent cases raising issues on employees’ privacy
rights in the workplace. One of these cases involved a government employer’s
The subject matter of “The Four Day Revolution” relates to the non-bloody search of an office computer, United States v. Mark L. Simons where the
change of government which took place at EDSA. Clearly such subject matter defendant Simons, an employee of a division of the CIA, was convicted of
is one of public interest and concern or even international interest. The subject receiving and possessing materials containing child pornography. In this case,
matter relates to a highly critical state in the history of this country and thus the US Supreme Court held that the search remains valid under the O’Connor
passed into the public domain and as an appropriate subject for speech and exception to the warrant requirement because evidence of the crime was
expression and coverage by any form of mass media. The synopsis provided discovered in the course of an otherwise proper administrative inspection.
by petitioner does not relate to the individual life and certainly not the private Simons’ violation of the agency’s Internet policy happened also to be a violation
life of respondent Ponce Enrile. The “Four Day Revolution” is not principally of criminal law; this does not mean that said employer lost the capacity and
about, nor is it focused upon, the man Juan Ponce Enrile. interests of an employer. The warrantless entry into Simons’ office was
reasonable under the Fourth Amendment standard announced in O’Connor
Moreso, Private respondent Enrile is a public figure (which gives the public a because at the inception of the search, the employer had “reasonable grounds
legitimate interest of his doings, his affairs, his character and has become a for suspecting” that the hard drive would yield evidence of misconduct, as the
public “personage”), in other words he is a celebrity. To be included in this employer was already aware that Simons had misused his Internet access to
category are those who have achieved some degree of reputation by appearing download over a thousand pornographic images. The retrieval of the hard
before the public. This includes public officers, famous inventors and explorers, drive was reasonably related to the objective of the search, and the search
war heroes and even ordinary soldiers, an infant prodigy, in short anyone who was not excessively intrusive. Thus, while Simons had a reasonable
has arrived at a position where public is focused upon him as a person. Private expectation of privacy in his office, he did not have such legitimate expectation
respondent Enrile is a public figure because of his participation as principal of privacy with regard to the files in his computer.
action in the culminating events of the change of government. The right of
privacy of a public figure is necessarily narrower than that of an ordinary Applying the analysis and principles announced in O’Connor and Simons to the
citizen. case at bar, we now address the following questions: (1) Did petitioner have
a reasonable expectation of privacy in his office and computer files?; and (2)
But it must be noted that the proposed motion picture is required to be fairly Was the search authorized by the CSC Chair, [which involved] the copying of
truthful and historical in its presentation of events. This serves as a line of the contents of the hard drive on petitioner’s computer, reasonable in its
equilibrium in this case between the constitutional freedom of speech and of inception and scope?
expression and the right of privacy. There must be no presentation of the
private life of the unwilling private respondent and certainly no revelation of (1) NO, the petitioner had no reasonable expectation of privacy in his
intimate or embarrassing personal facts. Portrayal of the participation of office and computer files.
private respondent in the EDSA Revolution should be related to the public facts
of the EDSA Revolution. Petitioner failed to prove that he had an actual (subjective) expectation of
privacy either in his office or government-issued computer which contained his
iv. Government-issued Computer? Pollo v. David, GR No. 181881, personal files. Petitioner did not allege that he had a separate enclosed office
October 18, 2011 which he did not share with anyone, or that his office was always locked and
not open to other employees or visitors. Neither did he allege that he used
RULING: The right to privacy has been accorded recognition in this jurisdiction passwords or adopted any means to prevent other employees from accessing
as a facet of the right protected by the guarantee against unreasonable search his computer files. On the contrary, he submits that being in the public
and seizure under Section 2, Article III of the 1987 Constitution (which was assistance office of the CSC-ROIV, he normally would have visitors in his office
derived almost verbatim from the Fourth Amendment to the US Constitution). like friends, associates and even unknown people, whom he even allowed to
The constitutional guarantee, however, is not a prohibition of all searches and use his computer which to him seemed a trivial request. He described his
seizures but only of “unreasonable” searches and seizures. office as “full of people, his friends, unknown people” and that in the past 22
(The Supreme Court cited various US jurisprudence to support its ruling). x x years he had been discharging his functions at the PALD, he is “personally
x x That the Fourth Amendment [of the U.S. Constitution] equally applies to a assisting incoming clients, receiving documents, drafting cases on appeals, in
government workplace was addressed in the 1987 case of O’Connor v. Ortega. charge of accomplishment report, Mamamayan Muna Program, Public Sector
In O’Connor the [U.S. Supreme] Court recognized that “special needs” Unionism, Correction of name, accreditation of service, and hardly had any
authorize warrantless searches involving public employees for work-related time for himself alone, that in fact he stays in the office as a paying customer.”
reasons. The [U.S. Supreme] Court thus laid down a balancing test under Under this scenario, it can hardly be deduced that petitioner had such
which government interests are weighed against the employee’s reasonable expectation of privacy that society would recognize as reasonable.

M.R.A.D.C. LUMBRE 161


CONSTITUTIONAL LAW REVIEW

(2) YES, the search authorized by the respondent CSC Chair, which similar acts, though they may not constitute a criminal offense, shall
involved the copying of the contents of the hard drive on petitioner’s produce a cause of action for damages, prevention and other relief:
computer, was reasonable in its inception and scope.
(1) Prying into the privacy of another’s residence.
The search of petitioner’s computer files was conducted in connection with
investigation of work-related misconduct prompted by an anonymous letter- This provisions recognizes that a man’s house is his castle, where his right to
complaint addressed to Chairperson David regarding anomalies in the CSC-RO privacy cannot be denied or even restricted by others. It includes any act of
IV where the head of the Mamamayan Muna Hindi Mamaya Na division is intrusion into, peeping or inquisitively into the residence of another without
supposedly “lawyering” for individuals with pending cases in the CSC. A search the consent of the latter. The phrase “prying into the privacy of another’s
by a government employer of an employee’s office is justified at inception when residence”, however, does not mean only the residence is entitled to privacy
there are reasonable grounds for suspecting that it will turn up evidence that as the law covers also similar acts – A public business office is entitled to the
the employee is guilty of work-related misconduct. same privacy when the public is excluded therefrom and only such individuals
as are allowed to enter may come in.
Thus, petitioner’s claim of violation of his constitutional right to privacy must
necessarily fail. His other argument invoking the privacy of communication The two reasonable expectation of privacy test is used to determine whether
and correspondence under Section 3(1), Article III of the 1987 Constitution is there is a violation of the right to privacy.
also untenable considering the recognition accorded to certain legitimate The SC in one case held that the reasonableness of a person’s expectation of
intrusions into the privacy of employees in the government workplace under privacy depends on a two-part test:
the aforecited authorities. We likewise find no merit in his contention that
O’Connor and Simons are not relevant because the present case does not 1. Whether, by his conduct, the individual has exhibited an expectation of
involve a criminal offense like child pornography. As already mentioned, the privacy (subjective test); and
search of petitioner’s computer was justified there being reasonable ground for
2. This expectation is one that society recognized as reasonable
suspecting that the files stored therein would yield incriminating evidence
(objective test). [Customs, community norms, and practices, may,
relevant to the investigation being conducted by CSC as government employer
therefore, limit or extend an individual’s reasonable expectation of
of such misconduct subject of the anonymous complaint. This situation clearly
privacy.
falls under the exception to the warrantless requirement in administrative
searches defined in O’Connor. Hence, the reasonableness of a person’s expectation of privacy must be
determined on a case-to-case basis since it depends on the factual surrounding
v. CCTVs? Spouses Hing vs. Choachuy, G.R. No. 179736, 26 June 2013
the case.
RULING: The right to privacy is the right to be alone.
In this day age, video surveillance cameras are installed practically everywhere
The right to privacy is enshrined in our Constitution and in our laws. It is for the protection and safety of everyone. The installation of these cameras,
defined as “the right to be free from unwarranted exploitation of one’s person however, should not cover places where there is reasonable expectation of
or from intrusion into one’s private activities in such a way as to cause privacy, unless the consent of the individual, whose right to privacy would be
humiliation to a person’s ordinary sensibilities.” It is the right of an individual affected, was obtained. Nor should this cameras be used to pry in to the privacy
“to be free from unwarranted publicity, or to live without unwarranted of another’s residence or business office as it would be no different from
interference by the public in matters in which the public is not necessarily eavesdropping, which is a crime under RA4200 or the Anti-Wiretapping Law.
concerned.” Simply put, the right to privacy is “the right to be let alone.”
In the case, the petitioners have a reasonable expectation of privacy in their
The Bill of Rights guarantees the people’s right to privacy and protects them property, whether they used it as a business office or as a residence and that
against the State’s abuse of power. In this regard, the State recognizes the the installation of video surveillance cameras directly facing the petitioners’
right of the people to be secure in their houses. No one, not even the State, property or covering a significant portion thereof, without their consent, I sa
except in case of overriding social need and then only under the stringent clear violation of their right to privacy.
procedural safeguards, can disturb them in the privacy of their homes.
vi. Online Social Networks? Vivares vs. St. Theresa’s College, G.R. No.
The right to privacy under Article 26(1) for the Civil Code covers business 202666, September 29, 2014
offices where the public are excluded therefrom and only certain individuals
RULING: It was held that the before one can have an expectation of privacy
are allowed. Said article provides:
in his or her OSN (online social network) activity, it is first necessary that said
Art. 26. Every person shall respect the dignity, personality, privacy and user, in this case the children of petitioners, manifest the intention to keep
peace of mind of his neighbors and other persons. The following and certain posts private, through the employment of measures to prevent access
thereto or to limit its visibility. And this intention can materialize in cyberspace

M.R.A.D.C. LUMBRE 162


CONSTITUTIONAL LAW REVIEW

through the utilization of the OSN’s privacy tools. In other words, utilization of 1. Freedom of speech
these privacy tools is the manifestation, in cyber world, of the user’s invocation
of his or her right to informational privacy. 2. Freedom of the press

Therefore, a Facebook user who opts to make use of a privacy tool to grant or 3. Right of assembly and to petition the government to redress grievances
deny access to his or her post or profile detail should not be denied the 4. Right to form associations or societies not contrary to law
informational privacy right which necessarily accompanies said choice.
Otherwise, using these privacy tools would be a feckless exercise, such that if, 5. Freedom of religion
for instance, a user uploads a photo or any personal information to his or her
6. Right to access to information on matters of public concern
Facebook page and sets its privacy level at "Only Me" or a custom list so
that only the user or a chosen few can view it, said photo would still be deemed i. Freedom from censorship or prior restraint
public by the courts as if the user never chose to limit the photo’s visibility and
accessibility. Such position, if adopted, will not only strip these privacy tools of It refers to the official government restriction on the press or other forms of
their function but it would also disregard the very intention of the user to keep expression in the advance of actual publication or dissemination.
said photo or information within the confines of his or her private space. It is the curtailment of the freedom of expression and of the press made
The Court did not give weight to the contention of the petitioners due to one through restrictions or conditions in advance of actual publication or
key reason: they failed to question the act of the students who showed the dissemination. This means that the exercise of the said freedom of expression
photos to the respondent. The fact that not one of petitioners disputed or of the press depends upon the prior approval of the government.
Escudero’s sworn account that her students, who are the girls’ Facebook 1. Content-based and Content-neutral regulation or restraint
friends, showed her the photos using their own Facebook accounts. This only
goes to show that no special means to be able to view the allegedly private Content-Based Content-Neutral
posts were ever resorted to by Escudero’s students, and that it is reasonable The restriction is based on the Merely concerned with the
to assume, therefore, that the photos were, in reality, viewable either by (1) subject matter of the utterance incidents of the speech, or one
their Facebook friends, or (2) by the public at large. or speech. The cast of the that merely controls the time,
restriction determines the test by place or manner, and under well-
Considering that the default setting for Facebook posts is public, it can be
which the challenged act is defined standards.
surmised that the photos in question were viewable to everyone on Facebook,
assailed with.
absent any proof that petitioners’ children positively limited the disclosure of
There is presumption of No presumption of
the photograph. If such were the case, they cannot invoke the protection
unconstitutionality unconstitutionality.
attached to the right to informational privacy.
Thus, the Court dismissed the petition and declared that the respondent did NOTE: The burden of proof to
not violate the right to informational privacy of the petitioner’s child. overcome the presumption of
unconstitutionality is with the
government.
5. Freedom of Expression, Press and Right to Peaceably Assemble Test to be used: Intermediate Test to be used: Clear and Present
Approach Danger
Section 4. No law shall be passed abridging the freedom of speech, of expression, or
of the press, or the right of the people peaceably to assemble and petition the This test is used when the speech The government must also show
government for redress of grievances. restraints take the form of a the type of harm the speech
content-neutral regulation, only sought to be restrained would
a. Concept and scope
a substantial governmental bring about—especially the
Freedom of speech includes: interest is required for its gravity and the imminence of the
validity. Because regulations of threatened harm – otherwise the
1. Freedom from censorship or prior restraint; and this type are not designed to prior restraint will be invalid. Prior
2. Freedom from subsequent punishment. suppress any particular message, restraint on speech based on its
they are not subject to the content cannot be justified by
Generally, any and all modes of expression are embraced in the guaranty which is strictest form of judicial scrutiny hypothetical fears, “but only by
reinforced by Article III, Section 18. The scope of protected freedom of expression but an intermediate approach showing a substantive and
under the Constitution includes: somewhere between the mere imminent evil that has taken the
rationality that is required of any life of a reality already on ground.

M.R.A.D.C. LUMBRE 163


CONSTITUTIONAL LAW REVIEW

other law and the compelling If we apply the test for content-neutral regulation, the questioned acts of
interest standard applied to (Please see the separate COMELEC will not pass the three requirements for evaluating such
content-based restrictions. The discussion of Clear and Present restraints on freedom of speech. "When the speech restraints take the
test is called intermediate Danger on Item form of a content-neutral regulation, only a substantial governmental
because the Court will not merely 5.B.I.) interest is required for its validity," and it is subject only to the
rubberstamp the validity of a law intermediate approach.
but also require that the
This intermediate approach is based on the test that we have prescribed
restrictions be narrowly-tailored
in several cases. A content-neutral government regulation is sufficiently
to promote an important or
justified.
significant governmental interest
that is unrelated to the [1] if it is within the constitutional power of the Government; [2] if it
suppression of expression. furthers an important or substantial governmental interest; [3] if the
governmental interest is unrelated to the suppression of free expression;
and [4] if the incident restriction on alleged [freedom of speech &
2. The Diocese of Bacolod vs. COMELEC, G.R. No. 205728, 21 expression] is no greater than is essential to the furtherance of that
January 2015 interest.

RULING: The Order issued by the COMELEC to petitioner ordering the On the first requisite, it is not within the constitutional powers of the
latter to remove the tarpaulin for violating COMELEC Resolution No. 9615, COMELEC to regulate the tarpaulin. As discussed earlier, this is protected
which provides for the size requirement of two feet (2’) by three feet (3’), speech by petitioners who are non-candidates. On the second
constitute censorship. requirement, not only must the governmental interest be important or
substantial, it must also be compelling as to justify the restrictions made.
The regulation may reasonably be considered as either content-neutral or
content-based. Regardless, the disposition of this case will be the same. Compelling governmental interest would include constitutionally declared
Generally, compared with other forms of speech, the proposed speech is principles. We have held, for example, that "the welfare of children and
content-based. the State’s mandate to protect and care for them, as parens patriae,
constitute a substantial and compelling government interest in regulating
Size limitations during elections hit at a core part of expression. The . . . utterances in TV broadcast."
content of the tarpaulin is not easily divorced from the size of its medium.
In this case, the size regulation is not unrelated to the suppression of
A content-based regulation, however, bears a heavy presumption of speech. Limiting the maximum size of the tarpaulin would render
invalidity and is measured against the clear and present danger rule. The ineffective petitioners’ message and violate their right to exercise freedom
latter will pass constitutional muster only if justified by a compelling of expression.
reason, and the restrictions imposed are neither overbroad nor vague.
The COMELEC’s act of requiring the removal of the tarpaulin has the effect
Under this rule, "the evil consequences sought to be prevented must be of dissuading expressions with political consequences. These should be
substantive, ‘extremely serious and the degree of imminence extremely encouraged, more so when exercised to make more meaningful the
high.’ Only when the challenged act has overcome the clear and present equally important right to suffrage.
danger rule will it pass constitutional muster, with the government having
the burden of overcoming the presumed unconstitutionality. The restriction in the present case does not pass even the lower test of
intermediate scrutiny for content-neutral regulations.
Even with the clear and present danger test, respondents failed to justify
the regulation. There is no compelling and substantial state interest The action of the COMELEC in this case is a strong deterrent to further
endangered by the posting of the tarpaulines to justify curtailment of the speech by the electorate. Given the stature of petitioners and their
right of freedom of expression. There is no reason for the state to minimize message, there are indicators that this will cause a "chilling effect" on
the right of non-candidate petitioners to post the tarpaulin in their private robust discussion during elections.
property. The size of the tarpaulin does not affect anyone else’s
The form of expression is just as important as the message itself. In the
constitutional rights.
words of Marshall McLuhan, "the medium is the message." McLuhan’s
Content-based restraint or censorship refers to restrictions "based on the colleague and mentor Harold Innis has earlier asserted that "the materials
subject matter of the utterance or speech." In contrast, content-neutral on which words were written down have often counted for more than the
regulation includes controls merely on the incidents of the speech such as words themselves."
time, place, or manner of the speech.

M.R.A.D.C. LUMBRE 164


CONSTITUTIONAL LAW REVIEW

3. The O’ Brien Test If the words uttered create a dangerous tendency of an evil which the State
has the right to prevent, the words are punishable. It is sufficient if the natural
This rule provides that in situations when “speech” and “non-speech” tendency and the probable effect of the utterance were to bring about the
elements are combined in the same course of conduct, whether there is a substantive evil that the legislative body seeks to prevent.
sufficiently important governmental interest that warrants regulating the
non-speech element, incidentally limiting the “speech” element. iii. Balancing of Interest Rule
A government regulation is valid if: When particular conduct is regulated in the interest of public order, and the
regulation results to indirect, conditional, or partial abridgment of speech, the
a. It is within the constitutional power of the government; duty of the courts is to determine which of the conflicting interests demands
b. In furtherance of an important or substantial governmental greater protection under the particular circumstances presented. This rule
interest; requires the court to take conscious and detailed consideration of the interplay
of interests observable in a given situation.
c. Governmental interest is unrelated to the suppression of free
expression; and c. Commercial speech –

d. The incidental restriction on the freedom is essential to the Commercial speech is a separate category of speech which is not accorded the
furtherance of that interest. same level of protection as that given to other constitutionally guaranteed forms of
expression but is nonetheless entitled to protection. The State cannot rob him of
ii. Freedom from subsequent punishment this right without violating the constitutionally guaranteed freedom of expression.
Unsolicited advertisements are legitimate forms of expression.
This refers to freedom from any punishment as a consequence of or in
connection with a speech, utterance, or writing. The phrase “subsequent This is a communication which “no more than proposes a commercial transaction”.
punishment” means that in order to make the freedom of expression clause of Advertisement of goods or of services is an example of this.
the Constitution more meaningful, there must be an assurance that after
making any utterance or publication, the author is not subject to any form of To enjoy protection, commercial speech:
punishment. If the State would impose punishment as a condition for such 1. Must not be false or misleading; and
exercise, it would negate the right and make the freedom of expression a
mockery and a delusion. 2. Should not propose an illegal transaction.
It refers to a limitation on the power of the State from imposing a punishment i. Disini, Jr. vs. The Secretary of Justice, G.R. No. 203335, February 18,
after publication or dissemination. Without this assurance, the individual would 2014
hesitate to speak for fear that he might be held to account for his speech, or
that he might be provoking the vengeance of the officials he may have RULING: To prohibit the transmission of unsolicited ads would deny a person
criticized. the right to read his emails, even unsolicited commercial ads addressed to him.
Commercial speech is a separate category of speech which is not
b. Tests for valid government interference to freedom of expression accorded the same level of protection as that given to other
constitutionally guaranteed forms of expression but is nonetheless
i. Clear and Present Danger Rule (BP 880, Sec. 6(a)) entitled to protection. The State cannot rob him of this right without
In Schenck v US, the US Supreme Court said that this rule is used to determine violating the constitutionally guaranteed freedom of expression. Unsolicited
whether the words are used in such circumstances and of such circumstances advertisements are legitimate forms of expression.
and of such a nature as to create a clear and present danger that they will The above penalizes the transmission of unsolicited commercial
bring about the substantive evils that the State has the right to prevent. The communications, also known as “spam.” The term “spam” surfaced in early
substantive evil must be extremely serious and the degree of imminence internet chat rooms and interactive fantasy games. One who repeats the same
extremely high before utterances can be punished. sentence or comment was said to be making a “spam.” The term referred to a
The rule is that the danger created must not only be clear and present but also Monty Python’s Flying Circus scene in which actors would keep saying “Spam,
traceable to the ideas expressed. In Gonzales v. COMELEC, the Court said that Spam, Spam, and Spam” when reading options from a menu.
the term “clear” seems to point to a causal connection with the danger of the The Government, represented by the Solicitor General, points out that
substantive evil arising from the utterance questioned; while “present refers unsolicited commercial communications or spams are a nuisance that wastes
to the time element, identified with imminent and immediate danger. The the storage and network capacities of internet service providers, reduces the
danger must not only be probable but very likely inevitable efficiency of commerce and technology, and interferes with the owner’s
ii. Dangerous Tendency Rule peaceful enjoyment of his property. Transmitting spams amounts to trespass

M.R.A.D.C. LUMBRE 165


CONSTITUTIONAL LAW REVIEW

to one’s privacy since the person sending out spams enters the recipient’s To justify a restriction, the promotion of a substantial government interest
domain without prior permission. The OSG contends that commercial speech must be clearly shown. Thus:
enjoys less protection in law.
A government regulation is sufficiently justified if it is within the constitutional
But, firstly, the government presents no basis for holding that power of the government, if it furthers an important or substantial government
unsolicited electronic ads reduce the “efficiency of computers.” interest; if the governmental interest is unrelated to the suppression of free
Secondly, people, before the arrival of the age of computers, have expression; and if the incidental restriction on alleged First Amendment
already been receiving such unsolicited ads by mail. These have never freedoms is no greater than is essential to the furtherance of that interest.
been outlawed as nuisance since people might have interest in such ads. What
matters is that the recipient has the option of not opening or reading Hence, even though the government's purposes are legitimate and substantial,
these mail ads. That is true with spams. Their recipients always have they cannot be pursued by means that broadly stifle fundamental personal
the option to delete or not to read them. liberties, when the end can be more narrowly achieved.

d. State regulation of mass media The freedoms of speech and of the press should all the more be upheld when
what is sought to be curtailed is the dissemination of information meant. to
The propriety of granting or denying permission to the media to broadcast, record, add meaning to the equally vital right of suffrage. We cannot support any ruling
or photograph court proceedings involves weighing the constitutional guarantees or order "the effect of which would be to nullify so vital a constitutional right
of freedom of the press, the right of the public to information and the right to public as free speech." When faced with borderline situations in which the freedom of
trial, on the one hand, and on the other hand, the due process rights of the a candidate or a party to speak or the freedom of the electorate to know is
defendant and the inherent and constitutional power of the courts to control their invoked against actions allegedly made to assure clean and free elections, this
proceedings in order to permit the fair and impartial administration of justice. Court shall lean in favor of freedom. For in the ultimate analysis, the freedom
of the citizen and the State's power to regulate should not be antagonistic.
i. ABS-CBN Broadcasting Corporation vs. COMELEC, GR No. 133486, There can be no free and honest elections if, in the efforts to maintain them,
January 28, 2000 the freedom to speak and the right to know are unduly curtailed.
RULING: The assailed COMELEC Resolution enjoining petitioner or any True, the government has a stake in protecting the fundamental right to vote
other group from conducting exit polls during the May 11 elections is by providing voting places that are safe and accessible. It has the duty to
invalid. It is too broad, since its application is without qualification as secure the secrecy of the ballot and to preserve the sanctity and the integrity
to whether the polling is disruptive or not. The Omnibus Election Code of the electoral process. However, in order to justify a restriction of the people's
prohibits disruptive behavior around the voting centers. There is no showing, freedoms of speech and of the press, the state's responsibility of ensuring
however, that exit polls or the means to interview voters cause chaos in voting orderly voting must far outweigh them.
centers. Neither has any evidence been presented proving that the presence
of exit poll reporters near an election precinct tends to create disorder or These freedoms have additional importance, because exit polls generate
confuse the voters. important research data which may be used to study influencing factors and
trends in voting behavior. An absolute prohibition would thus be unreasonably
The holding of exit polls and the dissemination of their results through restrictive, because it effectively prevents the use of exit poll data not only for
mass media constitute an essential part of the freedoms of speech and election-day projections, but also for long-term research.
of the press. Hence, the Comelec cannot ban them totally in the guise
of promoting clean, honest, orderly and credible elections. Quite the The Comelec's concern with the possible non-communicative effect of exit polls
contrary, exit polls — properly conducted and publicized — can be vital — disorder and confusion in the voting centers — does not justify a total ban
tools in eliminating the evils of election-fixing and fraud. Narrowly on them. Undoubtedly, the assailed Comelec Resolution is too broad,
tailored countermeasures may be prescribed by the Comelec so as to since its application is without qualification as to whether the polling
minimize or suppress the incidental problems in the conduct of exit is disruptive or not. Concededly, the Omnibus Election Code prohibits
polls, without transgressing in any manner the fundamental rights of disruptive behavior around the voting centers. There is no showing, however,
our people. that exit polls or the means to interview voters cause chaos in voting centers.
Neither has any evidence been presented proving that the presence of exit poll
Doctrinally, the Court has always ruled in favor of the freedom of expression, reporters near an election precinct tends to create disorder or confuse the
and any restriction is treated an exemption. The power to exercise prior voters.
restraint is not to be presumed; rather the presumption is against its
validity. And it is respondent's burden to overthrow such presumption. Any act Moreover, the prohibition incidentally prevents the collection of exit poll data
that restrains speech should be greeted with furrowed brows, so it has been and their use for any purpose. The valuable information and ideas that could
said. be derived from them, based on the voters' answer to the survey questions
will forever remain unknown and unexplored. Unless the ban is restrained,

M.R.A.D.C. LUMBRE 166


CONSTITUTIONAL LAW REVIEW

candidates, researchers, social scientists and the electorate in general would Considering the prejudice it poses to the defendant's right to due process as
be deprived of studies on the impact of current events and of election-day and well as to the fair and orderly administration of justice and considering further
other factors on voters' choices. that the freedom of the press and the right of the people to information may
be served and satisfied by less distracting, degrading and prejudicial means,
The absolute ban imposed by the Comelec cannot, therefore, be justified. It live radio and television coverage of court proceedings shall not be allowed.
does not leave open any alternative channel of communication to gather the Video footages of court hearings for news purposes shall be restricted and
type of information obtained through exit polling. On the other hand, there are limited to shots of the courtroom, the judicial officers, the parties and their
other valid and reasonable ways and means to achieve the Comelec end of counsel taken prior to the commencement of official proceedings. No
avoiding or minimizing disorder and confusion that may be brought about by video shots or photographs shall be permitted during the trial proper.
exit surveys.
iii. Re: Request Radio-TV Coverage of the Trial in the Sandiganbayan
ii. Re: Live TV and Radio Coverage of the Hearing of President Corazon of the Plunder Case Against Former President Joseph E. Estrada, AM
C. Aquino’s Libel Case, October 22, 1991 No. 01-4-03-SC, June 29, 2001
RULING: The live telecasting of the case constitutes a violation of the right of RULING: The live media coverage should not be allowed.
the accused to due process.
The Court is not all that unmindful of recent technological and scientific
The propriety of granting or denying permission to the media to broadcast, advances but to chance forthwith the life or liberty of any person in a hasty bid
record, or photograph court proceedings involves weighing the constitutional to use and apply them, even before ample safety nets are provided and the
guarantees of freedom of the press, the right of the public to information and concerns heretofore expressed are aptly addressed, is a price too high to pay.
the right to public trial, on the one hand, and on the other hand, the due
process rights of the defendant and the inherent and constitutional power of The propriety of granting or denying the instant petition involve the weighing
the courts to control their proceedings in order to permit the fair and impartial out of the constitutional guarantees of freedom of the press and the right to
administration of justice. public information, on the one hand, and the fundamental rights of the
accused, on the other hand, along with the constitutional power of a court to
In Estes vs. Texas, the United States Supreme Court held that television control its proceedings in ensuring a fair and impartial trial.
coverage of judicial proceedings involves an inherent denial of the due process
rights of a criminal defendant. The Court through Mr. Justice Clark, identified When these rights race against one another, jurisprudence tells us
four (4) areas of potential prejudice which might arise from the impact of the that the right of the accused must be preferred to win.
cameras on the jury, witnesses, the trial judge and the defendant. The decision
in part pertinently stated: A public trial is not synonymous with publicized trial; it only implies that
the court doors must be open to those who wish to come, sit in the available
Experience likewise has established the prejudicial effect of telecasting on seats, conduct themselves with decorum and observe the trial process. In the
witnesses. Witnesses might be frightened, play to the camera, or become constitutional sense, a courtroom should have enough facilities for a
nervous. They are subject to extraordinary out-of-court influences which might reasonable number of the public to observe the proceedings, not too small as
affect their testimony. Also, telecasting not only increases the trial judge's to render the openness negligible and not too large as to distract the trial
responsibility to avoid actual prejudice to the defendant, it may as well affect participants from their proper functions, who shall then be totally free to report
his own performance. Judges are human beings also and are subject to the what they have observed during the proceedings.
same psychological reactions as laymen. For the defendant, telecasting is a
form of mental harassment and subjects him to excessive public exposure and The courts recognize the constitutionally embodied freedom of the
distracts him from the effective presentation of his defense. press and the right to public information. It also approves of media's
exalted power to provide the most accurate and comprehensive means of
The television camera is a powerful weapon which intentionally or inadvertently conveying the proceedings to the public and in acquainting the public with the
can destroy an accused and his case in the eyes of the public. judicial process in action; nevertheless, within the courthouse, the
overriding consideration is still the paramount right of the accused to
Representatives of the press have no special standing to apply for a writ of due process which must never be allowed to suffer diminution in its
mandate to compel a court to permit them to attend a trial, since within the constitutional proportions. Justice Clark thusly pronounced, "while a
courtroom a reporter's constitutional rights are no greater than those of any maximum freedom must be allowed the press in carrying out the important
other member of the public. Massive intrusion of representatives of the news function of informing the public in a democratic society, its exercise must
media into the trial itself can also alter or destroy the constitutionally necessary necessarily be subject to the maintenance of absolute fairness in the judicial
judicial atmosphere and decorum that the requirements impartiality imposed process."
by due process of law are denied the defendant.

M.R.A.D.C. LUMBRE 167


CONSTITUTIONAL LAW REVIEW

iv. Re: Live Media Broadcast of Ampatuan Trial, Res., AM Nos. 10-11- case to generate undue publicity with its concomitant undesirable effects
5-SC, 10-11-6-SC, and 10-11-7-SC, October 23, 2012 weighs heavily against broadcasting the trial.

RULING: Upon reconsideration, and after weighing once more the rights To address the physical impossibility of accommodating large number of
guaranteed by the Constitution that are involved in this case, this Court interested parties inside the courtroom in Camp Bagong Diwa, it is not
partially grants reconsideration of June 14, 2011 Resolution. For reasons to be necessary to allow the press to broadcast the proceedings here and abroad,
discussed below, this Court is now disallowing the live media broadcast of the but the Court may allow the opening of closed-circuit viewing areas outside
trial of the “Maguindanao Massacre” cases but is still allowing the filming of the the courtroom where those who may be so minded can come and watch the
proceedings for (1) the real-time transmission to specified viewing areas, and proceedings. This out-of-court, real-time viewing grants to a larger audience
(2) documentation. the opportunity to monitor the proceedings as if they were inside the trial court
but at the same time obviates the massive publicity entailed in media
Upon review of the matter, the Court has sought a way to provide a public trial broadcasting.
as required by the Constitution and the Rules, which is a right granted to the
accused, without inviting media frenzy that affect the due process rights of the NOTE: This is the most recent case, wherein the Court allowed real-time
accused in this high profile case. transmission to specified viewing areas and documentation.
While this Court recognizes the freedom of the press and the right to public v. GMA Network, Inc. vs. COMELEC, G.R. No. 205357, 2 September
information, which, by the way are rights that belong to non-direct parties to 2014
the case, the rights of the direct parties should not be forgotten. In a clash
among these competing interests and in terms of values the Constitution RULING: The Court held that the assailed rule on “aggregate-based” airtime
recognizes, jurisprudence makes it clear that the balance should always be limits is unreasonable and arbitrary as it unduly restricts and constrains the
weighed in favor of the accused. ability of candidates and political parties to reach out and communicate with
the people. Here, the adverted reason for imposing the “aggregate-based”
The Constitutional rights specific to the accused under Section 14, Article III airtime limits – leveling the playing field – does not constitute a compelling
of the Constitution such as the right to due process of law, to be presumed state interest which would justify such a substantial restriction on the freedom
innocent until the contrary is proved, and to an impartial and public trial and of candidates and political parties to communicate their ideas, philosophies,
the requirement of the highest quantum of proof to justify deprivation of his platforms and programs of government. And, this is specially so in the absence
liberty (or even of his life) provide more than ample justification to take a of a clear-cut basis for the imposition of such a prohibitive measure.
second look at the view that a camera that broadcasts the proceedings live on
television has no place in a criminal trial because of its prejudicial effects on It is also particularly unreasonable and whimsical to adopt the aggregate-
the rights of accused individuals. based time limits on broadcast time when we consider that the Philippines is
not only composed of so many islands. There are also a lot of languages and
In this case that has achieved notoriety and sensational status, a greater dialects spoken among the citizens across the country. Accordingly, for a
degree of care is required to safeguard the constitutional rights of the accused. national candidate to really reach out to as many of the electorates as possible,
To be in the best position to weigh the conflicting testimonies of the witnesses, then it might also be necessary that he conveys his message through his
the judge must not be affected by any outside force or influence. advertisements in languages and dialects that the people may more readily
understand and relate to. To add all of these airtimes in different dialects would
So must the witnesses be shielded from the pressure of being aware that their greatly hamper the ability of such candidate to express himself – a form of
testimony is broadcasted live over television or radio, to be scrutinized and suppression of his political speech.
judged by the court of public opinion. A witness’ behavior and self-
consciousness before the camera in a high profile case such as this case might vi. Section 5 of the Data Privacy Act of 2012 (RA 10173) and RA 53, as
compromise the reliability of the fact-finding process, which in turn could skew amended by RA 1477
the judge’s assessment of his or her credibility, necessarily affecting the
resolution of the case. Section 5. Protection Afforded to Journalists and Their Sources. – Nothing in
this Act shall be construed as to have amended or repealed the provisions of
In a constitutional sense, public trial is not synonymous with publicized trial. Republic Act No. 53, which affords the publishers, editors or duly accredited
The right to a public trial belongs to the accused. The requirement of a public reporters of any newspaper, magazine or periodical of general circulation
trial is satisfied by the opportunity of the members of the public and the press protection from being compelled to reveal the source of any news report or
to attend the trial and to report what they have observed. information appearing in said publication which was related in any confidence
to such publisher, editor, or reporter. (R.A. No. 10173, Section 5)
The accused’s right to a public trial should not be confused with the freedom
of the press and the public’s right to know as a justification for allowing the Section 1. Section one of Republic Act Numbered Fifty-three is amended to
live broadcast of the trial. The tendency of a high profile case like the subject read as follows:

M.R.A.D.C. LUMBRE 168


CONSTITUTIONAL LAW REVIEW

“Sec. 1. Without prejudice to his liability under the civil and criminal laws, The processions, rallies, parades, demonstrations, public meetings and
the publisher, editor, columnist or duly accredited reporter of any assemblages for religious purposes shall be governed by local ordinances:
newspaper, magazine or periodical of general circulation cannot be Provided, however, That the declaration of policy as provided in Section 2 of
compelled to reveal the source of any news-report or information this Act shall be faithfully observed.
appearing in said publication which was related in confidence to such
publisher, editor or reporter unless the court or a House or committee of The definition herein contained shall not include picketing and other concerted
Congress finds that such revelation is demanded by the security of the action in strike areas by workers and employees resulting from a labor dispute
State.” (R.A. No. 53, as amended by R.A. No. 1477) as defined by the Labor Code, its implementing rules and regulations, and by
the Batas Pambansa Bilang 227.
e. Academic freedom and the right of the school to discipline its students
– (b) "Public place" shall include any highway, boulevard, avenue, road, street,
bridge or other thoroughfare, park, plaza, square, and/or any open space of
Miriam College Foundation, Inc. vs. Court of Appeals, 348 SCRA 265 (2000) public ownership where the people are allowed access.
RULING: The Court resolved the issue before it by looking through the power of (c) "Maximum tolerance" means the highest degree of restraint that the
DECS and the Disciplinary Committee in imposing sanctions upon the defendants. military, police and other peace keeping authorities shall observe during a
public assembly or in the dispersal of the same.
Section 5 (2), Article XIV of the Constitution guarantees all institutions of higher
learning academic freedom. This institutional academic freedom includes the right (d) "Modification of permit" shall include the change of the place and time of
of the school or college to decide for itself, its aims and objectives, and how best the public assembly, rerouting of the parade or street march, the volume of
to attain them free from outside coercion or interference save possibly when the loud-speakers or sound system and similar changes.
overriding public welfare calls for some restraint. Such duty gives the institution
the right to discipline its students and inculcate upon them good values, ideals and Section 4. Permit when required and when not required - A written permit
attitude. shall be required for any person or persons to organize and hold a public
assembly in a public place. However, no permit shall be required if the public
The right of students to free speech in school is not always absolute. The Court assembly shall be done or made in a freedom park duly established by law or
upheld the right of students for the freedom of expression but it does not rule out ordinance or in private property, in which case only the consent of the owner
disciplinary actions of the school on the conduct of their students. or the one entitled to its legal possession is required, or in the campus of a
government-owned and operated educational institution which shall be subject
Further, Sec. 7 of the of the Campus Journalism Act provides that the school cannot to the rules and regulations of said educational institution. Political meetings
suspend or expel a student solely on the basis of the articles they write EXCEPT or rallies held during any election campaign period as provided for by law are
when such article materially disrupts class work of involve substantial disorder or not covered by this Act.
invasion of the rights of others.
Section 6. Action to be taken on the application -
Therefore the Court ruled that the power of the school to investigate is an adjunct
of its power to suspend or expel. It is a necessary corollary to the enforcement of (a) It shall be the duty of the mayor or any official acting in his behalf to issue
rules and regulations and the maintenance of a safe and orderly educational or grant a permit unless there is clear and convincing evidence that the public
environment conducive to learning. That power, like the power to suspend or expel, assembly will create a clear and present danger to public order, public safety,
is an inherent part of the academic freedom of institutions of higher learning public convenience, public morals or public health.
guaranteed by the Constitution. The court held that Miriam College has the
authority to hear and decide the cases filed against respondent students. (b) The mayor or any official acting in his behalf shall act on the application
within two (2) working days from the date the application was filed, failing
f. Assembly and petition which, the permit shall be deemed granted. Should for any reason the mayor
or any official acting in his behalf refuse to accept the application for a permit,
i. BP 880 (The Public Assembly Act of 1985) said application shall be posted by the applicant on the premises of the office
Section 3. Definition of terms - For purposes of this Act: of the mayor and shall be deemed to have been filed.

(a) "Public assembly" means any rally, demonstration, march, parade, (c) If the mayor is of the view that there is imminent and grave danger of a
procession or any other form of mass or concerted action held in a public place substantive evil warranting the denial or modification of the permit, he shall
for the purpose of presenting a lawful cause; or expressing an opinion to the immediately inform the applicant who must be heard on the matter.
general public on any particular issue; or protesting or influencing any state of (d) The action on the permit shall be in writing and served on the application
affairs whether political, economic or social; or petitioning the government for within twenty-four hours.
redress of grievances.

M.R.A.D.C. LUMBRE 169


CONSTITUTIONAL LAW REVIEW

(e) If the mayor or any official acting in his behalf denies the application or (g) Acts described hereunder if committed within one hundred (100) meters
modifies the terms thereof in his permit, the applicant may contest the decision from the area of activity of the public assembly or on the occasion thereof;
in an appropriate court of law.
1. the carrying of a deadly or offensive weapon or device such as firearm,
(f) In case suit is brought before the Metropolitan Trial Court, the Municipal pillbox, bomb, and the like;
Trial Court, the Municipal Circuit Trial Court, the Regional Trial Court, or the
Intermediate Appellate Court, its decisions may be appealed to the appropriate 2. the carrying of a bladed weapon and the like;
court within forty-eight (48) hours after receipt of the same. No appeal bond 3 the malicious burning of any object in the streets or thoroughfares;
and record on appeal shall be required. A decision granting such permit or
modifying it in terms satisfactory to the applicant shall, be immediately 4. the carrying of firearms by members of the law enforcement unit;
executory.
5. the interfering with or intentionally disturbing the holding of a public
(g) All cases filed in court under this Section shall be decided within twenty- assembly by the use of a motor vehicle, its horns and loud sound systems.
four (24) hours from date of filing. Cases filed hereunder shall be immediately
Section 15. Freedom parks - Every city and municipality in the country shall
endorsed to the executive judge for disposition or, in his absence, to the next
within six months after the effectivity of this Act establish or designate at least
in rank.
one suitable "freedom park" or mall in their respective jurisdictions which, as
(h) In all cases, any decision may be appealed to the Supreme Court. far as practicable, shall be centrally located within the poblacion where
demonstrations and meetings may be held at any time without the need of any
(i) Telegraphic appeals to be followed by formal appeals are hereby allowed. prior permit.
Section 9. Non-interference by law enforcement authorities - Law In the cities and municipalities of Metropolitan Manila, the respective mayors
enforcement agencies shall not interfere with the holding of a public assembly. shall establish the freedom parks within the period of six months from the
However, to adequately ensure public safety, a law enforcement contingent effectivity of this Act.
under the command of a responsible police officer may be detailed and
stationed in a place at least one hundred (100) meter away from the area of 1. IBP vs. Atienza, GR No. 175241, February 24, 2010
activity ready to maintain peace and order at all times.
RULING: The Court in Bayan stated that the provisions of the Public
Section 12. Dispersal of public assembly without permit - When the public Assembly Act of 1985 practically codified the 1983 ruling in Reyes v.
assembly is held without a permit where a permit is required, the said public Bagatsing. In juxtaposing Sections 4 to 6 of the Public Assembly Act with
assembly may be peacefully dispersed. the pertinent portion of the Reyes case, the Court elucidated as follows:

Section 13. Prohibited acts - The following shall constitute violations of this x x x [The public official concerned shall] appraise whether there may be
Act: valid objections to the grant of the permit or to its grant but at another
public place. It is an indispensable condition to such refusal or modification
(a) The holding of any public assembly as defined in this Act by any leader or that the clear and present danger test be the standard for the decision
organizer without having first secured that written permit where a permit is reached. If he is of the view that there is such an imminent and grave
required from the office concerned, or the use of such permit for such purposes danger of a substantive evil, the applicants must be heard on the matter.
in any place other than those set out in said permit: Provided, however, That Thereafter, his decision, whether favorable or adverse, must be
no person can be punished or held criminally liable for participating in or transmitted to them at the earliest opportunity. Thus if so minded, they
attending an otherwise peaceful assembly; can have recourse to the proper judicial authority.
(b) Arbitrary and unjustified denial or modification of a permit in violation of In modifying the permit outright, respondent gravely abused his discretion
the provisions of this Act by the mayor or any other official acting in his behalf. when he did not immediately inform the IBP who should have been heard
(c) The unjustified and arbitrary refusal to accept or acknowledge receipt of first on the matter of his perceived imminent and grave danger of a
the application for a permit by the mayor or any official acting in his behalf; substantive evil that may warrant the changing of the venue. The
opportunity to be heard precedes the action on the permit, since the
(d) Obstructing, impeding, disrupting or otherwise denying the exercise of the applicant may directly go to court after an unfavorable action on the
right to peaceful assembly; permit.
(e) The unnecessary firing of firearms by a member of any law enforcement Respondent failed to indicate how he had arrived at modifying the terms
agency or any person to disperse the public assembly; of the permit against the standard of a clear and present danger test
which, it bears repeating, is an indispensable condition to such
(f) Acts in violation of Section 10 hereof; modification. Nothing in the issued permit adverts to an imminent and

M.R.A.D.C. LUMBRE 170


CONSTITUTIONAL LAW REVIEW

grave danger of a substantive evil, which "blank" denial or modification within the context, in other words, of viable independent institutions for
would, when granted imprimatur as the appellate court would have it, delivery of justice which are accepted by the general community.
render illusory any judicial scrutiny thereof.
The US Supreme Court held that “The Court has recognized that the
It is true that the licensing official, here respondent Mayor, is not devoid unhindered and untrammeled functioning of our courts is part of the very
of discretion in determining whether or not a permit would be granted. It foundation of our constitutional democracy. X x x The state may adopt
is not, however, unfettered discretion. While prudence requires that there safeguards necessary and appropriate to assure that the administration of
be a realistic appraisal not of what may possibly occur but of what may justice at all stages is free from outside control and influence. It is obviously a
probably occur, given all the relevant circumstances, still the assumption safeguard both necessary and appropriate to vindicate the State’s interest in
– especially so where the assembly is scheduled for a specific public place assuring justice under the law.
– is that the permit must be for the assembly being held there. The
exercise of such a right, in the language of Justice Roberts, The conduct of picketing and parading as mentioned in the statute (Louisiana
speaking for the American Supreme Court, is not to be "abridged Statute) is subject to regulation even though intertwined with expression and
on the plea that it may be exercised in some other place." association. It has never been deemed an abridgment of freedom of speech or
press to make a course of conduct illegal merely because the conduct was in
Notably, respondent failed to indicate in his Comment any basis or part initiated, evidenced, or carried out by means of language, either spoken,
explanation for his action. It smacks of whim and caprice for respondent written, or printed.”
to just impose a change of venue for an assembly that was slated for a
specific public place. It is thus reversible error for the appellate court not It is undisputed that a major purpose of the demonstration was to protest what
to have found such grave abuse of discretion and, under specific statutory the demonstrators considered an illegal arrest of 23 students the previous day.
provision, not to have modified the permit "in terms satisfactory to the While the students had not been arraigned or their trial set for any day certain,
applicant.” they were charged with violation of the law, and the judges responsible for
trying them and passing upon the legality of their arrest were then in the
ii. In Re: Petition to Annul En Bank Resolution AM No. 98-7-02-SC – building.
Ricardo C. Valmonte and Union of Lawyers and Advocates for
Transparency in Government (ULAT), GR No. 134621, September 29, It is, of course, true that most judges will be influenced only by what they see
1998 and hear in court. However judges are human; and the legislature has the right
to recognize the danger that some judges, jurors, and other court officials, will
RULING: It is true that safeguarding of the people’s freedom of expression to be consciously or unconsciously influenced by demonstrations in or near their
the end that individuals may speak as they think on matters vital to them and courtrooms both prior to and at the same time of the trial. A State may also
that falsehoods may be expressed through the processes of education and properly protect the judicial process from being misjudged in the minds of the
discussion, is essential to free government. But freedom of speech and public. A State may protect against the possibility of a conclusion by the public
expression despite its indispensability has its limitations. It has never been under these circumstances that the judge’s action was in part a product of
understood as the absolute right to speak whenever, however, and wherever intimidation and did not flow only from the fair and orderly working of the
one pleases for the manner, place, and time of public discussion can be judicial process.
constitutionally controlled. As well as put by our Justice Isagani Cruz, the better
policy is not liberty untamed but liberty regulated by law where every freedom Re: Public Assembly Act
is exercised in accordance with the law and with due regard for the rights of Petitioners claim that this Court committed an act of judicial legislation
others. violating the principle of separation of powers by converting sidewalks and
The Court reiterates that judicial independence and the fair and orderly streets within a radius of 200 meters from every courthouse from a public
administration of justice constitute paramount governmental interests that can forum place into a “no rally zone.” The Court rejects this argument. Public
justify the regulation of the public’s right of free speech and peaceful assembly places are historically associated with the free exercise of expressive activities
in the vicinity of courthouses. BUT even in such public fore, it is settled jurisprudence that the government
may restrict speech plus activities and enforce reasonable time, place, and
Freedom of expression needs on occasion to be adjusted to and accommodated manner regulations as long as the restrictions are content-neutral, are
with the requirements of equally important public interests. One of these narrowly tailored to serve a significant governmental interest, and leave open
fundamental public interests is the maintenance of the integrity and orderly ample alternative channels of communication.
functioning of the administration of justice. There is no antimony between free
expression and the integrity of the system of administering justice. For the BP. Bldg. 880 did not establish streets and sidewalks, among other places, as
protection and maintenance of freedom of expression itself can be secured only public for a. A close look at the law will reveal that in fact prescribes reasonable
within the context of a functioning and orderly system of dispensing justice, time, place, and manner regulations. Thus, it requires a written permit for the
holding of public assemblies in public places subject, even, to the right of the

M.R.A.D.C. LUMBRE 171


CONSTITUTIONAL LAW REVIEW

mayor to modify the place and time of the public assembly, to impose a In the same breath that the establishment clause restricts what the
rerouting of the parade or street march, to limit the volume of loud speakers government can do with religion, it also limits what religious sects can or
or sound system and to prescribe other appropriate restrictions on the conduct cannot do with the government. They can neither cause the government
of the public assembly. to adopt their particular doctrines as policy for everyone, nor can they not
cause the government to restrict other groups. To do so, in simple terms,
would cause the State to adhere to a particular religion and, thus,
6. Freedom of Religion establishing a state religion. Consequently, the petitioners are misguided
in their supposition that the State cannot enhance its population control
a. Non-establishment clause program through the RH Law simply because the promotion of
contraceptive use is contrary to their religious beliefs. Indeed, the State
No law shall be made respecting an establishment of religion or prohibiting the
is not precluded to pursue its legitimate secular objectives without being
free exercise thereof. (Article III, Section 3)
dictated upon by the policies of any one religion.
i. Concept and basis
The Free Exercise Clause and the Duty to Refer
CONCEPT: The non- establishment clause means that the state should adopt
While the RH Law, in espousing state policy to promote reproductive
a “position of neutrality” when it comes to religious matters. The non-
health manifestly respects diverse religious beliefs in line with the Non-
establishment clause bars the State from establishing, through laws and rules,
Establishment Clause, the same conclusion cannot be reached with respect
moral standards according to a specific religion. Prohibitions against immorality
to Sections 7, 23 and 24 thereof. The said provisions commonly mandate
should be based on a purpose that is independent of religious beliefs. When it
that a hospital or a medical practitioner to immediately refer a person
forms part of our laws, rules, and policies, morality must be secular. Laws and
seeking health care and services under the law to another accessible
rules of conduct must be based on a secular purpose.
healthcare provider despite their conscientious objections based on
BASIS: Separation of Church and State. (Section 6, Article II) religious or ethical beliefs.

1. Imbong vs. Ochoa, G.R. No. 204819, April 8, 2014 In a situation where the free exercise of religion is allegedly burdened by
government legislation or practice, the compelling state interest test in
ISSUE: W/N (R.A.) No. 10354, otherwise known as the Responsible line with the Court's espousal of the Doctrine of Benevolent Neutrality finds
Parenthood and Reproductive Health Act of 2012 (RH Law), is application. In this case, the conscientious objector's claim to religious
unconstitutional. freedom would warrant an exemption from obligations under the RH Law,
RULING: R.A. No. 10354 is NOT UNCONSTITUTIONAL except with unless the government succeeds in demonstrating a more compelling
respect to the following provisions, among others, which are declared state interest in the accomplishment of an important secular objective.
UNCONSTITUTIONAL: The obligation to refer imposed by the RH Law violates the religious belief
1) Section 7 insofar as they: a) require private health facilities and non- and conviction of a conscientious objector. Once the medical practitioner,
maternity specialty hospitals and hospitals owned and operated by a against his will, refers a patient seeking information on modem
religious group to refer patients, not in an emergency or life-threatening reproductive health products, services, procedures and methods, his
case, as defined under Republic Act No. 8344, to another health facility conscience is immediately burdened as he has been compelled to perform
which is conveniently accessible; and b) allow minor-parents or minors an act against his beliefs.
who have suffered a miscarriage access to modem methods of family In case of conflict between the religious beliefs and moral convictions of
planning without written consent from their parents or guardian/s; individuals, on one hand, and the interest of the State, on the other, to
2) Section 23(a)(3) as they punish any healthcare service provider who provide access and information on reproductive health products, services,
fails and/or refuses to refer a patient not in an emergency or life- procedures and methods to enable the people to determine the timing,
threatening case, to another health care service provider within the same number and spacing of the birth of their children, the Court is of the strong
facility or one which is conveniently accessible regardless of his or her view that the religious freedom of health providers, whether public or
religious beliefs; private, should be accorded primacy. Accordingly, a conscientious objector
should be exempt from compliance with the mandates of the RH Law. If
3) Section 23(b) insofar as they punish any public officer who refuses to he would be compelled to act contrary to his religious belief and conviction,
support reproductive health programs or shall do any act that hinders the it would be violative of "the principle of non-coercion" enshrined in the
full implementation of a reproductive health program, regardless of his or constitutional right to free exercise of religion.
her religious beliefs;
Freedom of religion means more than just the freedom to believe. It also
The Establishment Clause and Contraceptives means the freedom to act or not to act according to what one believes.

M.R.A.D.C. LUMBRE 172


CONSTITUTIONAL LAW REVIEW

And this freedom is violated when one is compelled to act against one's government. A cursory reading of the assailed provision bares that the
belief or is prevented from acting according to one's belief. Apparently, in religious freedom of the petitioners is not at all violated. All the law
these cases, there is no immediate danger to the life or health of an requires is for would-be spouses to attend a seminar on parenthood,
individual in the perceived scenario of the subject provisions. After all, a family planning breastfeeding and infant nutrition. It does not even
couple who plans the timing, number and spacing of the birth of their mandate the type of family planning methods to be included in the
children refers to a future event that is contingent on whether or not the seminar, whether they be natural or artificial. Those who receive any
mother decides to adopt or use the information, product, method or supply information during their attendance in the required seminars are not
given to her or whether she even decides to become pregnant at all. On compelled to accept the information given to them, are completely free to
the other hand, the burden placed upon those who object to contraceptive reject the information they find unacceptable, and retain the freedom to
use is immediate and occurs the moment a patient seeks consultation on decide on matters of family life without the intervention of the State.
reproductive health matters.
ii. Exceptions under the Constitution (Art. VI, Secs. 28(3) and 29(2),
The Court finds no compelling state interest which would limit the free Art. XIV, Secs. 3(3) and 4(2))
exercise clause of the conscientious objectors. Only the prevention of an
immediate and grave danger to the security and welfare of the community Charitable institutions, churches, parsonages or convents appurtenant thereto,
can justify the infringement of religious freedom. If the government fails mosques, non-profit cemeteries, and all lands, buildings, and improvements,
to show the seriousness and immediacy of the threat, State intrusion is actually, directly, and exclusively used for religious, charitable or educational
constitutionally unacceptable. Granting that a compelling interest exists to purposes shall be exempt from taxation. (Article VI, Section 28 (3))
justify the infringement of the conscientious objector's religious freedom, No public money or property shall be appropriated, applied, paid, or employed,
the respondents have failed to demonstrate "the gravest abuses, directly or indirectly, for the use, benefit, or support of any sect, church,
endangering paramount interests" which could limit or override a person's denomination, sectarian institution, or system of religion, or of any priest,
fundamental right to religious freedom. Also, the respondents have not preacher, minister, other religious teacher, or dignitary as such, except, when
presented any government effort exerted to show that the means it takes such priest, preacher, minister or dignitary is assigned to the armed forces, or
to achieve its legitimate state objective is the least intrusive means. Other to any penal institution, or government orphanage or leprosarium. (Article
than the assertion that the act of referring would only be momentary, VI, Section 29 (2))
considering that the act of referral by a conscientious objector is the very
action being contested as violative of religious freedom, it behooves the At the option expressed in writing by the parents or guardians, religion shall
respondents to demonstrate that no other means can be undertaken by be allowed to be taught to their children or wards in public elementary and
the State to achieve its objective without violating the rights of the high schools within the regular class hours by instructors designated or
conscientious objector. The health concerns of women may still be approved by the religious authorities of the religion to which the children or
addressed by other practitioners who may perform reproductive health- wards belong, without additional cost to the Government. (Article XIV,
related procedures with open willingness and motivation. Suffice it to say, Section 3 (3))
a person who is forced to perform an act in utter reluctance deserves the
Educational institutions, other than those established by religious groups and
protection of the Court as the last vanguard of constitutional freedoms.
mission boards, shall be owned solely by citizens of the Philippines or
Exception: Life Threatening Cases corporations or associations at least sixty per centum of the capital of which is
owned by such citizens. The Congress may, however, require increased Filipino
The Court properly recognizes an exception set forth in the law. While equity participation in all educational institutions.
generally healthcare service providers cannot be forced to render
reproductive health care procedures if doing it would contravene their The control and administration of educational institutions shall be vested in
religious beliefs, an exception must be made in life-threatening cases that citizens of the Philippines.
require the performance of emergency procedures. In these situations,
No educational institution shall be established exclusively for aliens and no
the right to life of the mother should be given preference, considering that
group of aliens shall comprise more than one-third of the enrollment in any
a referral by a medical practitioner would amount to a denial of service,
school. The provisions of this subsection shall not apply to schools established
resulting to unnecessarily placing the life of a mother in grave danger.
for foreign diplomatic personnel and their dependents and, unless otherwise
Thus, the forced referral clause that is being objected on grounds of
provided by law, for other foreign temporary residents. (Article XIV, Section
violation of freedom of religion does not contemplate an emergency.
4 (2))
Family Planning Seminars
1. Re: Letter of Tony Q. Valenciano, Holding of Religious Rituals at
The requirement imposed under Section 15 as a condition for the issuance the Hall of Justice Building in Quezon City, A.M. No. 10-4-19-SC, 7
of a marriage license is reasonable exercise of police power by the March 2017

M.R.A.D.C. LUMBRE 173


CONSTITUTIONAL LAW REVIEW

ISSUES: Whether the holding of masses during lunch break at the or support" to describe the same. Even the exception to the same
basement of the Quezon City Hall of Justice is a case of establishment of provision bolsters this interpretation. The exception contemplates a
religion and whether there is an appropriation of public money for the situation wherein public funds are paid to a priest, preacher, minister, or
benefit of the Catholic Church. other religious teacher, or dignitary because they rendered service in the
armed forces, or to any penal institution, or government orphanage or
RULING: No. (1) Establishment entails a positive action on the part of leprosarium. That a priest belongs to a particular church and the latter
the State. Accommodation, on the other hand, is passive. In the former, may have benefited from the money he received is of no moment, for the
the State becomes involved through the use of government resources with purpose of the payment of public funds is merely to compensate the priest
the primary intention of setting up a state religion. In the latter, the State, for services rendered and for which other persons, who will perform the
without being entangled, merely gives consideration to its citizens who same services will also be compensated in the same manner.
want to freely exercise their religion.
The Constitution even grants tax exemption to properties actually, directly
Guided by the foregoing, it is our considered view that the holding of and exclusively devoted to religious purposes. Certainly, this benefits the
Catholic masses at the basement of the QC Hall of Justice is not a case of religious sects for a portion of what could have been collected for the
establishment, but merely accommodation. First, there is no law, benefit of the public is surrendered in their favor.
ordinance or circular issued by any duly constitutive authorities expressly
mandating that judiciary employees attend the Catholic masses at the Here, the basement of the QC Hall of Justice is not appropriated, applied
basement. Second, when judiciary employees attend the masses to or employed for the sole purpose of supporting the Roman Catholics.
profess their faith, it is at their own initiative as they are there on their
own free will and volition, without any coercion from the judges or b. Free exercise clause
administrative officers. Third, no government funds are being spent xxx The free exercise and enjoyment of religious profession and worship, without
because the lightings and airconditioning continue to be operational even discrimination or preference, shall forever be allowed. Xxx (Section 6, Article III)
if there are no religious rituals there. Fourth, the basement has neither
been converted into a Roman Catholic chapel nor has it been permanently i. Dual aspect
appropriated for the exclusive use of its faithful. Fifth, the allowance of the
The aspects of freedom of religious profession and worship are the following:
masses has not prejudiced other religions.
a. Right to believe, which is absolute; and
(2) Section 29 (2), Article VI of the 1987 Constitution provides, "No public
money or property shall be appropriated, applied, paid, or employed, b. Right to act according to one’s beliefs, which is subject to regulation.
directly or indirectly, for the use, benefit, or support of any sect, church,
denomination, sectarian institution, or system of religion, or of any priest, 1. Dominador L. Taruc, et al. vs. Bishop Porfirio dela Cruz. GR No.
preacher, minister, or other religious teacher, or dignitary as such, except 144801, March 10, 2005
when such priest, preacher, minister, or dignitary is assigned to the armed ISSUE: Whether the courts of law may validly interfere with disputes
forces, or to any penal institution, or government orphanage or involving excommunication/expulsion of members of a church.
leprosarium."
RULING: No. We agree with the Court of Appeals that the
The word "apply" means "to use or employ for a particular expulsion/excommunication of members of a religious
purpose." "Appropriate" means "to prescribe a particular use for particular institution/organization is a matter best left to the discretion of the
moneys or to designate or destine a fund or property for a distinct use, or officials, and the laws and canons, of said institution/organization. It is not
for the payment of a particular demand." for the courts to exercise control over church authorities in the
It has also been held that the aforecited constitutional provision "does not performance of their discretionary and official functions. Rather, it is for
inhibit the use of public property for religious purposes when the religious the members of religious institutions/organizations to conform to just
character of such use is merely incidental to a temporary use which is church regulations.
available indiscriminately to the public in general." Hence, a public street In the leading case of Fonacier v. Court of Appeals, we enunciated the
may be used for a religious procession even as it is available for a civic doctrine that in disputes involving religious institutions or organizations,
parade, in the same way that a public plaza is not barred to a religious there is one area which the Court should not touch: doctrinal and
rally if it may also be used for a political assemblage. disciplinary differences.
In relation thereto, the phrase "directly or indirectly" refers to the manner The amendments of the constitution, restatement of articles of religion
of appropriation of public money or property, not as to whether a particular and abandonment of faith or abjuration alleged by appellant, having to do
act involves a direct or a mere incidental benefit to any church. Otherwise, with faith, practice, doctrine, form of worship, ecclesiastical law, custom
the framers of the Constitution would have placed it before "use, benefit

M.R.A.D.C. LUMBRE 174


CONSTITUTIONAL LAW REVIEW

and rule of a church and having reference to the power of excluding from to take cognizance of the case and to determine whether the SDA, as
the church those allegedly unworthy of membership, are unquestionably employer, rightfully exercised its management prerogative to dismiss an
ecclesiastical matters which are outside the province of the civil courts. employee.
2. Austria vs. NLRC, GR No. 124382, August 16, 1999 a. Definition of Ecclesiastical affair
ISSUE: Whether the termination of the services of the petitioner is an An ecclesiastical affair involves the relationship between the church
ecclesiastical affair, and, as such, involves the separation of church and and its members and relates to matters of faith, religious doctrines,
state. worship and governance of the congregation. Examples of so-called
ecclesiastical affairs to which the State cannot meddle are
RULING: No. The Court held that the case at bar does not concern an proceedings for excommunication, ordinations of religious ministers,
ecclesiastical or purely religious affair as to bar the State from taking and administration of sacraments. While the matter at hand relates
cognizance of the same. An ecclesiastical affair is "one that concerns to the church and its religious minister, it does not give the case a
doctrine, creed, or form of worship of the church, or the adoption and religious significance. What is involved is the relationship of the
enforcement within a religious association of needful laws and regulations church as an employer and the minister as an employee. It is purely
for the government of the membership, and the power of excluding from secular and has no relation whatsoever with the practice of faith,
such associations those deemed unworthy of membership. Based on this worship or doctrines of the church.
definition, an ecclesiastical affair involves the relationship between the
church and its members and relate to matters of faith, religious doctrines, ii. Case – Estrada vs. Escritor, 492 SCRA 1, AM No. P-02-1651, June 22,
worship and governance of the congregation. To be concrete, examples of 2006 and August 4, 2003
this so-called ecclesiastical affairs to which the State cannot meddle are
proceedings for excommunication, ordinations of religious ministers, ISSUE: Whether the State could penalize respondent for conjugal
administration of sacraments and other activities with attached religious arrangements considered illicit, but in conformity with her religious beliefs.
significance. The case at bar does not even remotely concern any of the RULING: No. The State could not penalize respondent for she is exercising
above cited examples. While the matter at hand relates to the church and her right to freedom of religion. The free exercise of religion is specifically
its religious minister it does not ipso facto give the case a religious articulated as one of the fundamental rights in our Constitution. As Jefferson
significance. Simply stated, what is involved here is the relationship of the put it, it is the most inalienable and sacred of human rights. The State’s interest
church as an employer and the minister as an employee. It is purely in enforcing its prohibition cannot be merely abstract or symbolic in order to
secular and has no relation whatsoever with the practice of faith, worship be sufficiently compelling to outweigh a free exercise claim. In the case at bar,
or doctrines of the church. In this case, petitioner was not ex- the State has not evinced any concrete interest in enforcing the concubinage
communicated or expelled from the membership of the SDA but was or bigamy charges against respondent or her partner. Thus the State’s interest
terminated from employment. Indeed, the matter of terminating an only amounts to the symbolic preservation of an unenforced prohibition.
employee, which is purely secular in nature, is different from the Furthermore, a distinction between public and secular morality and religious
ecclesiastical act of expelling a member from the religious congregation. morality should be kept in mind. The jurisdiction of the Court extends only to
The grounds invoked for petitioner's dismissal, namely: misappropriation public and secular morality.
of denominational funds, willful breach of trust, serious misconduct, gross The Court further states that our Constitution adheres the benevolent
and habitual neglect of duties and commission of an offense against the neutrality approach that gives room for accommodation of religious exercises
person of his employer's duly authorized representative, are all based on as required by the Free Exercise Clause. This benevolent neutrality could allow
Article 282 of the Labor Code which enumerates the just causes for for accommodation of morality based on religion, provided it does not offend
termination of employment. By this alone, it is palpable that the reason compelling state interests. Assuming arguendo that the OSG has proved a
for petitioner's dismissal from the service is not religious in nature. compelling state interest, it has to further demonstrate that the state has used
Coupled with this is the act of the SDA in furnishing NLRC with a copy of the least intrusive means possible so that the free exercise is not infringed any
petitioner's letter of termination. Aside from these, SDA admitted in a more than necessary to achieve the legitimate goal of the state. Thus the
certification issued by its officer, that petitioner has been its employee for conjugal arrangement cannot be penalized for it constitutes an exemption to
twenty-eight (28) years. SDA even registered petitioner with the Social the law based on her fundamental right to freedom of religion.
Security System (SSS) as its employee. As a matter of fact, the worker's
records of petitioner have been submitted by private respondents as part 1. Standards
of their exhibits. From all of these it is clear that when the SDA terminated
a. Strict Neutrality Theory
the services of petitioner, it was merely exercising its management
prerogative to fire an employee which it believes to be unfit for the job. Separation - strict or tame - protects the principle of church-state
As such, the State, through the Labor Arbiter and the NLRC, has the right separation with a rigid reading of the principle.

M.R.A.D.C. LUMBRE 175


CONSTITUTIONAL LAW REVIEW

b. Benevolent Neutrality Theory Clause. In this case, the court finds that establishment concerns
prevail over potential accommodation interests.
Accommodation - protects religious realities, tradition and
established practice with a flexible reading of the principle. 2. Tests
With religion looked upon with benevolence and not hostility, a. Clear and Present Danger Test
benevolent neutrality allows accommodation of religion under certain
circumstances. Accommodations are government policies that take The state can regulate the exercise of religious freedom, that is, when
religion specifically into account not to promote the governments the exercise will bring about the clear and present danger of some
favored form of religion, but to allow individuals and groups to substantive evil which the State is duty bound to prevent, i.e., serious
exercise their religion without hindrance. Their purpose or effect detriment to the more overriding interest of public health, public
therefore is to remove a burden on, or facilitate the exercise of, a morals, or public welfare.
persons or institutions religion. As Justice Brennan explained, the b. Compelling State Interest Test
government [may] take religion into account to exempt, when
possible, from generally applicable governmental regulation The compelling state interest test is proper where conduct is involved
individuals whose religious beliefs and practices would otherwise for the whole gamut of human conduct has different effects on the
thereby be infringed, or to create without state involvement an states interests: some effects may be immediate and short-term while
atmosphere in which voluntary religious exercise may flourish.[ others delayed and far-reaching. A test that would protect the
Accommodation is forbearance and not alliance. it does not reflect interests of the state in preventing a substantive evil, whether
agreement with the minority, but respect for the conflict between the immediate or delayed, is therefore necessary. The compelling state
temporal and spiritual authority in which the minority finds itself. interest serves the purpose of revering religious liberty while at the
same time affording protection to the paramount interests of the
Accommodation is distinguished from strict neutrality in that the state.
latter holds that government should base public policy solely on
secular considerations, without regard to the religious consequences Three-step process. First, has the statute created a burden on the
of its actions. The debate between accommodation and strict free exercise of religion? Second, is there a sufficiently compelling
neutrality is at base a question of means: Is the freedom of religion state interest to justify this infringement of religious liberty? Third,
best achieved when the government is conscious of the effects of its has the State, in achieving its legitimate purposes, used the least
action on the various religious practices of its people, and seeks to intrusive means possible, so that free exercise has not been infringed
minimize interferences with those practices? Or is it best advanced any more than necessary to achieve its legitimate goal?
through a policy of religious blindness - keeping government aloof
c. Conscientious Objector Test
from religious practices and issues? An accommodationist holds
that it is good public policy, and sometimes constitutionally required, A conscientious objector is an "individual who has claimed the right
for the state to make conscious and deliberate efforts to avoid to refuse to perform military service" on the grounds of freedom of
interference with religious freedom. On the other hand, the strict thought, conscience, or religion.
neutrality adherent believes that it is good public policy, and also
constitutionally required, for the government to avoid religion-specific c. Non-religious test clause
policy even at the cost of inhibiting religious exercise. No religious test shall be required for the exercise of civil or political rights.
Types of Accommodation under the Free Exercise Clause: (Section 5, Article III)

1. Mandatory accommodation - results when the court finds that


accommodation is required by the Free Exercise Clause; that is, when 7. Liberty of Abode and the Right to Travel
the court itself carves out an exemption. This accommodation occurs
when all three conditions of the compelling interest test are met. The liberty of abode and of changing the same within the limits prescribed by law shall
not be impaired except upon lawful order of the court. Neither shall the right to travel
2. Permissive accommodation - the court finds that the State may, be impaired except in the interest of national security, public safety, or public health,
but is not required to, accommodate religious interests. as may be provided by law. (Section 6, Article III)
3. Prohibited accommodation - when the court finds no basis for a a. Limitations of Liberty of abode
mandatory accommodation, or it determines that legislative
accommodation runs afoul of the Establishment or the Free Exercise Lawful order of the court.

M.R.A.D.C. LUMBRE 176


CONSTITUTIONAL LAW REVIEW

i. Lorenzo vs. Director of Health, 50 Phil 595 (1950) liberty and security, for which there exists no readily available legal recourse
or remedy.
ISSUE: Whether the confinement of petitioner, a leper, in the San Lazaro
Hospital by the health authorities, is unconstitutional. c. Limitations of the right to travel
RULING: Section 1058 of the Administrative Code was enacted by the i. Public Safety - Samahan ng mga Progresibong Kabataan (SPARK) vs.
legislative body in the legitimate exercise of the police power which extends to Quezon City, G.R. No. 225442, 8 August 2017
the preservation of the public health. It was place on the statute books in
recognition of leprosy as a grave health problem. The methods provided for ISSUE: Whether the Curfew Ordinances are unconstitutional for violating the
the control of leprosy plainly constitute due process of law. The assumption right to travel.
must be that if evidence was required to establish the necessity for the law RULING: The Manila and Navotas Ordinances are declared unconstitutional
that it was before the legislature when the act was passed. and thus, null and void, while the Quezon City Ordinance is declared as
Health officers may restrict access to contaminated areas and also quarantine constitutional and thus, valid.
those already exposed to the disease sought to be contained. Right to Travel
b. Concept of the right to travel The right to travel is recognized and guaranteed as a fundamental right under
i. Coverage - Marcos vs. Manglapus, GR No. 88211, October 27, 1989 Section 6, Article III of the 1987 Constitution.

RULING: The right to return to one's country is not among the rights Jurisprudence provides that this right refers to the right to move freely from
specifically guaranteed in the Bill of Rights, which treats only of the liberty of the Philippines to other countries or within the Philippines. It is a right
abode and the right to travel, but it is the Court’s well-considered view that embraced within the general concept of liberty. Liberty - a birthright of every
the right to return may be considered, as a generally accepted principle of person - includes the power of locomotion and the right of citizens to be free
international law and, under our Constitution, is part of the law of the land. to use their faculties in lawful ways and to live and work where they desire or
However, it is distinct and separate from the right to travel and enjoys a where they can best pursue the ends of life.
different protection under the International Covenant on Civil and Political Nevertheless, grave and overriding considerations of public interest justify
Rights. restrictions even if made against fundamental rights. Specifically on the
ii. Aliens - Nishimura Ekiu vs. United States, 142 US 651, 659 (1892) freedom to move from one place to another, jurisprudence provides that this
right is not absolute. As the 1987 Constitution itself reads, the State may
RULING: The right to travel is not afforded to aliens seeking to enter a foreign impose limitations on the exercise of this right, provided that they: (1) serve
country. the interest of national security, public safety, or public health; and (2) are
provided by law.
iii. HDO - Reyes vs. CA, GR No. 182161, December 3, 2009
The stated purposes of the Curfew Ordinances, specifically the promotion of
ISSUE: Whether the right to travel is covered by the Rule on the Writ of juvenile safety and prevention of juvenile crime, inarguably serve the interest
Amparo. of public safety. The restriction on the minor's movement and activities within
RULING: No. The rights that fall within the protective mantle of the Writ of the confines of their residences and their immediate vicinity during the curfew
Amparo under Section 1 of the Rules thereon are the following: (1) right to period is perceived to reduce the probability of the minor becoming victims of
life; (2) right to liberty; and (3) right to security. or getting involved in crimes and criminal activities. As to the second
requirement, i.e., that the limitation "be provided by law," our legal system is
The right to travel refers to the right to move from one place to another. As replete with laws emphasizing the State's duty to afford special protection to
stated in Marcos v. Sandiganbayan, “xxx a person’s right to travel is subject children.
to the usual constraints imposed by the very necessity of safeguarding the
system of justice. In such cases, whether the accused should be permitted to Particularly relevant to this case is Article 139 of PD 603, which explicitly
leave the jurisdiction for humanitarian reasons is a matter of the court’s sound authorizes local government units, through their city or municipal councils, to
discretion.” set curfew hours for children. It reads:

Here, the restriction on petitioner’s right to travel as a consequence of the Article 139. Curfew Hours for Children. - City or municipal councils may
pendency of the criminal case filed against him was not unlawful. Petitioner prescribe such curfew hours for children as may be warranted by local
has also failed to establish that his right to travel was impaired in the manner conditions. The duty to enforce curfew ordinances shall devolve upon the
and to the extent that it amounted to a serious violation of his right to life, parents or guardians and the local authorities.

M.R.A.D.C. LUMBRE 177


CONSTITUTIONAL LAW REVIEW

As explicitly worded, city councils are authorized to enact curfew ordinances RULING: No. True, the right to travel is guaranteed by the Constitution.
and enforce the same through their local officials. In other words, PD 603 However, the exercise of such right is not absolute. Section 6, Article III of the
provides sufficient statutory basis - as required by the Constitution - to restrict 1987 Constitution allows restrictions on one’s right to travel provided that such
the minors' exercise of the right to travel. restriction is in the interest of national security, public safety or public health
as may be provided by law. This, however, should by no means be construed
ii. Commander in Chief Clause - Gudani vs. Senga. 498 SCRA 671, as limiting the Court’s inherent power of administrative supervision over lower
August 15, 2006 courts. OCA Circular No. 49-2003 does not restrict but merely regulates, by
RULING: The petitioners may be subjected to military discipline on account of providing guidelines to be complied by judges and court personnel, before they
their defiance of a direct order of the AFP Chief of Staff restricting the former’s can go on leave to travel abroad. To "restrict" is to restrain or prohibit a person
right to travel. from doing something; to "regulate" is to govern or direct according to rule.

The ability of the President to require a military official to secure prior consent To ensure management of court dockets and to avoid disruption in the
before appearing before Congress pertains to a wholly different and administration of justice, OCA Circular No. 49-2003 requires a judge who
independent specie of presidential authority, the commander-in-chief powers wishes to travel abroad to submit, together with his application for leave of
of the President. By tradition and jurisprudence, the commander-in-chief absence duly recommended for approval by his Executive Judge.
powers of the President are not encumbered by the same degree of restriction For traveling abroad without having been officially allowed by the Court, the
as that which may attach to executive privilege or executive control. respondent is guilty of violation of OCA Circular No. 49-2003. Under Section
The commander-in-chief provision in the Constitution is denominated as 9(4), Rule 140 of the Revised Rules of Court, violation of Supreme Court
Section 18, Article VII, which begins with the simple declaration that [t]he directives and circular is considered a less serious charge and, therefore,
President shall be the Commander-in-Chief of all armed forces of the punishable by suspension from office without salary and other benefits for not
Philippines x x x Outside explicit constitutional limitations, such as those found less than one (1) month nor more than three (3) months; or a fine of more
in Section 5, Article XVI, the commander-in-chief clause vests on the President, than P10,000.00 but not exceeding P20,000.00.
as commander-in-chief, absolute authority over the persons and actions of the Section 53, Rule IV of the Revised Rules on Administrative Cases in the Civil
members of the armed forces. Such authority includes the ability of the Service grants the disciplining authority the discretion to consider mitigating
President to restrict the travel, movement and speech of military officers, circumstances in the imposition of the proper penalty. The Court had in several
activities which may otherwise be sanctioned under civilian law. instances refrained from imposing the actual penalties in the presence of
Indeed, the military practice is to require a soldier to obtain permission from mitigating facts, such as the employee’s length of service, acknowledgement
the commanding officer before he/she may leave his destination. A soldier who of his or her infractions and feelings of remorse for the same, advanced age,
goes from the properly appointed place of duty or absents from his/her family circumstances, and other humanitarian and equitable considerations.
command, guard, quarters, station, or camp without proper leave is subject to iv. Other statutory and inherent limitations - Leave Division, OCA-OAS
punishment by court-martial. It is even clear from the record that petitioners vs. Heusdens, etc., A.M. No. P-11-2927. December 13, 2011
had actually requested for travel authority from the PMA in Baguio City to
Manila, to attend the Senate Hearing. Even petitioners are well aware that it RULING: The exercise of one’s right to travel or the freedom to move from
was necessary for them to obtain permission from their superiors before they one place to another, as assured by the Constitution, is not absolute. There
could travel to Manila to attend the Senate Hearing. are constitutional, statutory and inherent limitations regulating the right to
travel. Section 6 itself provides that "neither shall the right to travel be
If the President or the Chief of Staff refuses to allow a member of the AFP to impaired except in the interest of national security, public safety or public
appear before Congress, the legislative body seeking such testimony may seek health, as may be provided by law."
judicial relief to compel the attendance.
Inherent limitations on the right to travel are those that naturally emanate
The duty falls on the shoulders of the President, as commander-in-chief, to from the source. These are very basic and are built-in with the power. An
authorize the appearance of the military officers before Congress. Even if the example of such inherent limitation is the power of the trial courts to prohibit
President has earlier disagreed with the notion of officers appearing before the persons charged with a crime to leave the country. In such a case, permission
legislature to testify, the Chief Executive is nonetheless obliged to comply with of the court is necessary. Another is the inherent power of the legislative
the final orders of the courts. department to conduct a congressional inquiry in aid of legislation. In the
iii. SC’s administrative supervision over lower courts - OAS-OCA vs. exercise of legislative inquiry, Congress has the power to issue a subpoena and
Judge Ignacio B. Macarine, A.M. No. MTJ-10-1770, July 18, 2012 subpoena duces tecum to a witness in any part of the country, signed by the
chairperson or acting chairperson and the Speaker or acting Speaker of the
ISSUE: Whether OCA Circular No. 49-2003 restricts the right to travel. House; or in the case of the Senate, signed by its Chairman or in his absence
by the Acting Chairman, and approved by the Senate President.

M.R.A.D.C. LUMBRE 178


CONSTITUTIONAL LAW REVIEW

Nonetheless, granting that it is an issue, the exercise of one’s right to travel requirement for the exercise of the right to information. Otherwise, the people can
or the freedom to move from one place to another, as assured by the never exercise the right if no contract is consummated, and if one is consummated,
Constitution, is not absolute. There are constitutional, statutory and inherent it may be too late for the public to expose its defects.
limitations regulating the right to travel. Section 6, Article III itself provides
that "neither shall the right to travel be impaired except in the interest of The right covers three categories of information which are matters of public
national security, public safety or public health, as may be provided by law." concern, namely: (1) official records; (2) documents and papers pertaining to
Some of these statutory limitations are the following: official acts, transactions and decisions; and (3) government research data used in
formulating policies.
1] The Human Security Act of 2010 or R.A. No. 9372. The law restricts the
right to travel of an individual charged with the crime of terrorism even though i. EO 2, series of 2016
such person is out on bail. OPERATIONALIZING IN THE EXECUTIVE BRANCH THE PEOPLE’S
2] The Philippine Passport Act of 1996 or R.A. No. 8239. Pursuant to said CONSTITUTIONAL RIGHT TO INFORMATION AND THE STATE POLICIES TO
law, the Secretary of Foreign Affairs or his authorized consular officer may FULL PUBLIC DISCLOSURE AND TRANSPARENCY IN THE PUBLIC SERVICE AND
refuse the issuance of, restrict the use of, or withdraw, a passport of a Filipino PROVIDING GUIDELINES THEREFOR.
citizen. SECTION 2. Coverage. This order shall cover all government offices under
3] The "Anti- Trafficking in Persons Act of 2003" or R.A. No. 9208. the Executive Branch, including but not limited to the national government and
Pursuant to the provisions thereof, the Bureau of Immigration, in order to all its offices, departments, bureaus, offices, and instrumentalities, including
manage migration and curb trafficking in persons, issued Memorandum Order government-owned or -controlled corporations, and state universities and
Radjr No. 2011-011,12 allowing its Travel Control and Enforcement Unit to colleges. Local government units (LGUs) are encouraged to observe and be
"offload passengers with fraudulent travel documents, doubtful purpose of guided by this Order.
travel, including possible victims of human trafficking" from our ports. b. Limitations
4] The Migrant Workers and Overseas Filipinos Act of 1995 or R. A. No. i. AKBAYAN vs. Aquino, GR No. 170516, July 16, 2008
8042, as amended by R.A. No. 10022. In enforcement of said law, the
Philippine Overseas Employment Administration (POEA) may refuse to issue RULING: To be covered by the right to information, the information sought
deployment permit to a specific country that effectively prevents our migrant must meet the threshold requirement that it be a matter of public concern.
workers to enter such country. From the nature of the JPEPA, as in international trade agreement, it is evident
that the Japanese and Philippine offers submitted during the negotiations
5] The Act on Violence against Women and Children or R.A. No. 9262. towards its executions are matters of public concern.
The law restricts movement of an individual against whom the protection order
is intended. It is established, however, that neither the right to information nor the policy
of full public disclosure is absolute, there being matters which, albeit of public
6] Inter-Country Adoption Act of 1995 or R.A. No. 8043. Pursuant concern or interest, are recognized as privileged. The categories of information
thereto, the Inter-Country Adoption Board may issue rules restrictive of an that may be considered privileged includes matters of diplomatic character and
adoptee’s right to travel "to protect the Filipino child from abuse, exploitation, under negotiation and review.
trafficking and/or sale or any other practice in connection with adoption which
is harmful, detrimental, or prejudicial to the child." The documents on the proposed JPEPA as well as the text which is subject to
negotiations and legal review by the parties fall under the exceptions to the
right of access to information on matters of public concern and policy of public
8. Right to Information disclosure. They come within the coverage of executive privilege. At the time
when the Committee was requesting for copies of such documents, the
The right of the people to information on matters of public concern shall be recognized. negotiations were ongoing as they are still now and the text of the proposed
Access to official records, and to documents, and papers pertaining to official acts, JPEPA is still uncertain and subject to change. Considering the status and
transactions, or decisions, as well as to government research data used as basis for nature of such documents then and now, these are evidently covered by
policy development, shall be afforded the citizen, subject to such limitations as may be executive privilege consistent with existing legal provisions and settled
provided by law. (Section 7, Article III) jurisprudence.
a. Scope Also, the privileged character of diplomatic negotiations has been recognized
in this jurisdiction. While the final text of the JPEPA may not be kept perpetually
The right to information contemplates inclusion of negotiations leading to the confidential - since there should be "ample opportunity for discussion before
consummation of the transaction. Certainly, a consummated contract is not a

M.R.A.D.C. LUMBRE 179


CONSTITUTIONAL LAW REVIEW

[a treaty] is approved" - the offers exchanged by the parties during the 4. Information deemed confidential for the protection of the privacy and certain
negotiations continue to be privileged even after the JPEPA is published. individuals such as minors, victims of crimes or the accused;

A ruling that Philippine offers in treaty negotiations should now be open to 5. Information, documents, or records known by reason of official capacity and
public scrutiny would discourage future Philippine representatives from frankly are deemed as confidential, including those submitted or disclosed by entities
expressing their views during negotiations. Diplomatic negotiations, therefore, to government agencies, tribunals and boards or officers, in relation to the
are recognized as privileged in this jurisdiction, the JPEPA negotiations performance of their functions or to inquires or investigation conducted by
constituting no exception. them in the exercise of their administrative, regulatory or quasi-judicial
powers;
ii. Neri vs. Senate, GR No. 180643, September 4, 2008
6. Prejudicial, premature disclosure;
RULING: With respect to respondents’ invocation of constitutional
prescriptions regarding the right of the people to information and public 7. Records of proceedings or information from proceedings which pursuant to
accountability and transparency, the Court found nothing in these arguments law or relevant rules and regulations are treated as confidential or privileged;
to support respondents’ case. The Court held that it did not rule that the Senate
has no power to investigate the NBN Project in aid of legislation but merely 8. Matters considered confidential under banking and finance laws and their
excludes from the scope of respondents’ investigation the three (3) questions amendatory laws, and;
that elicit answers covered by executive privilege and rules that petitioner 9. Other exceptions to the right to information under laws, jurisprudence, and
cannot be compelled to appear before respondents to answer the said rules and regulations.
questions. Furthermore, the Court said that the constitutional provisions cited
by respondents did not espouse an absolute right to information. By their
wording, the intention of the Framers to subject such right to the regulation of
9. Right of Association
the law is unmistakable. The Constitutional provision shows the obvious
limitations on the right to information. The right of the people, including those employed in the public and private sectors, to
form unions, associations, or societies for purposes not contrary to law shall not be
In Chavez v. Presidential Commission on Good Government, it was stated that
abridged. (Section 8, Article III)
there are no specific laws prescribing the exact limitations within which the
right may be exercised or the correlative state duty may be obliged. a. Membership in subversive organizations
Nonetheless, it enumerated the recognized restrictions to such rights, among
them: (1) national security matters, (2) trade secrets and banking i. People vs. Ferrer, 48 SCRA 382 (1972)
transactions, (3) criminal matters, and (4) other confidential information. ISSUE: Whether the Anti-Subversive Act of 1957 outlawing subversive
National security matters include state secrets regarding military and associations is violative of the constitutional right of association.
diplomatic matters, as well as information on inter-government exchanges
prior to the conclusion of treaties and executive agreements. It was further RULING: No. The Act is aimed against conspiracies to overthrow the
held that even where there is no need to protect such state secrets, they must Government by force, violence or other illegal means. Whatever interest in
be "examined in strict confidence and given scrupulous protection." freedom of speech and freedom of association is infringed by the prohibition
against knowing membership in the Communist Party of the Philippines, is so
Incidentally, the right primarily involved here is the right of respondent indirect and so insubstantial as to be clearly and heavily outweighed by
Committees to obtain information allegedly in aid of legislation, not the the overriding considerations of national security and the preservation
people’s right to public information. This is the reason why the Court stressed of democratic institutions in this country.
in the assailed Decision the distinction between these two rights.
NOTE: The Act is not a bill of attainder as the government has to prove and
iii. Memorandum from the Executive Secretary dated 24 Nov. 2016 re establish the elements of the crime.
Inventory of Exceptions to EO No. 2 (S. 2016)
b. Right not to join
1. Information covered by executive privilege;
i. Sta. Clara Homeowners Association vs. Gaston, GR No. 141961,
2. Privileged information relating to national security, defense, or international January 23, 2002
relations;
ISSUE: Whether private respondents may automatically become members of
3. Information concerning law enforcement and protection of public and the petitioner association by virtue of thei. Being lot owners of the Sta. Clara
personal safety; subdivision.

M.R.A.D.C. LUMBRE 180


CONSTITUTIONAL LAW REVIEW

RULING: No. The constitutionally guaranteed freedom of association includes organization for the well-defined but unorganized and incohesive group of
the freedom not to associate. The right to choose with whom one will which every lawyer is a ready a member.
associate oneself is the very foundation and essence of that partnership. It
should be noted that the provision guarantees the right to form an association. Bar integration does not compel the lawyer to associate with anyone. He is
It does not include the right to compel others to form or join one. free to attend or not attend the meetings of his Integrated Bar Chapter or vote
or refuse to vote in its elections as he chooses. The only compulsion to which
More to the point, private respondents cannot be compelled to become he is subjected is the payment of annual dues. The Supreme Court, in order to
members of the SCHA by the simple expedient of including them in its Articles further the State's legitimate interest in elevating the quality of professional
of Incorporation and By-laws without their express or implied consent. True, it legal services, may require that the cost of improving the profession in this
may be to the mutual advantage of lot owners in a subdivision to band fashion be shared by the subjects and beneficiaries of the regulatory program
themselves together to promote their common welfare. But that is possible — the lawyers.
only if the owners voluntarily agree, directly or indirectly, to become members
of the association. True also, memberships in homeowners’ associations may Assuming that the questioned provision does in a sense compel a lawyer to be
be acquired in various ways -- often through deeds of sale, Torrens certificates a member of the Integrated Bar, such compulsion is justified as an exercise of
or other forms of evidence of property ownership. In the present case, the police power of the State.
however, other than the said Articles of Incorporation and By-laws, there is no We must here emphasize that the practice of law is not a property right but a
showing that private respondents have agreed to be SCHA members. mere privilege, and as such must bow to the inherent regulatory power of the
ii. Bel Air Village Association, Inc. vs. Dionisio, 174 SCRA 589 Court to exact compliance with the lawyer's public responsibilities.

ISSUE: Whether the act of petitioner in compelling respondent to be a member An "Integrated Bar" is a State-organized Bar, to which every lawyer must
and pay the corresponding association dues is unconstitutional and outside the belong, as distinguished from bar associations organized by individual lawyers
scope of its corporate power. themselves, membership in which is voluntary. Integration of the Bar is
essentially a process by which every member of the Bar is afforded an
RULING: No. When the petitioner voluntarily bought the subject parcel of land opportunity to do his share in carrying out the objectives of the Bar as well as
it was understood that he took the same free of all encumbrances except obliged to bear his portion of its responsibilities. Organized by or under the
notations at the back of the certificate of title, among them, that he direction of the State, an integrated Bar is an official national body of which all
automatically becomes a member of the respondent association. lawyers are required to be members. They are, therefore, subject to all the
rules prescribed for the governance of the Bar, including the requirement of
The mode of payment as well as the purposes for which the dues are intended payment of a reasonable annual fee for the effective discharge of the purposes
clearly indicate that the dues are not in the concept of a property tax as claimed of the Bar, and adherence to a code of professional ethics or professional
by the petitioner. They are shares in the common expenses for necessary responsibility breach of which constitutes sufficient reason for investigation by
services. the Bar and, upon proper cause appearing, a recommendation for discipline or
The contention that this lien collides with the constitutional guarantee of disbarment of the offending member
freedom of association is not tenable. The transaction between the defendants The most compelling argument sustaining the constitutionality and validity of
and the original seller (defendant's immediate predecessor) of the land is a Bar integration in the Philippines is the explicit unequivocal grant of precise
sale and the conditions have been validly imposed by the said vendor/the same power to the Supreme Court by Section 5 (5) of Article X of the 1973
not being contrary to law, morals and good customs and public policy. The fact Constitution of the Philippines, which reads:
that it has been approved by the Land Registration Commission did not make
it a governmental act subject to the constitutional restriction against Sec. 5. The Supreme Court shall have the following powers:
infringement of the right of association. The constitutional proscription that no
person can be compelled to be a member of an association against his will xxx xxxxxx
applies only to government acts and not to private transactions like the one in (5) Promulgate rules concerning pleading, practice, and procedure in all
question. courts, and the admission.
iii. In re: Marcial Edillion, 84 SCRA 554 (1978)
RULING: To compel a lawyer to be a member of the Integrated Bar is not 10. Taking of Private Property for Public Use (Note: Refer to discussion under
violative of his constitutional freedom to associate. Eminent Domain)
Integration does not make a lawyer a member of any group of which he is not
already a member. He became a member of the Bar when he passed the Bar
examinations. All that integration actually does is to provide an official national 11. Non-impairment of contracts

M.R.A.D.C. LUMBRE 181


CONSTITUTIONAL LAW REVIEW

No law impairing the obligation of contracts shall be passed. (Section 10, Article III) impairment of the right to contract. Dacion en pago is a special mode of
payment where the debtor offers another thing to the creditor who accepts it
a. When there is impairment
as equivalent of payment of an outstanding debt. The undertaking really
Impairment is anything that diminishes the efficacy of the contract. There is partakes in a sense of the nature of sale, that is, the creditor is really buying
substantial impairment when the law changes the terms of a legal contract between the thing or property of the debtor, the payment for which is to be charged
parties, either in the time or mode of performance, or imposes new conditions, or against the debtor’s debt. As such, the essential elements of a contract of sale,
dispenses with those expressed, or authorizes for its satisfaction something namely; consent, object certain, and cause or consideration must be
different from that provided in its terms. present. Being a form of contract, the dacion en pago agreement cannot be
i. China Banking Corp vs. ASB Holdings, GR No. 172192, December 23, perfected without the consent of the parties involved.
2008
We find no element of compulsion in the dacion en pago provision of the
ISSUE: Whether respondent’s Rehabilitation Plan violates the mutuality of Rehabilitation Plan. It was not the only solution presented by the ASB to pay
contracts and curtails a party’s freedom to contract. its creditors.
RULING: No. In a related case, Bank of the Philippine Islands v. Securities b. Exceptions
and Exchange Commission, the Court En Banc would be more emphatic in
i. Ortigas and Co. vs. Feati Bank and Trust Co., GR No. L-24670,
holding that:
December 14, 1979
The very same issues confronted the Court in the case of Metropolitan Bank & ISSUE: Whether Resolution No. 27, S. 1960 issued by the Municipal Council
Trust Company v. ASB Holdings, et al. In this case, Metropolitan Bank & Trust of Mandaluyong can nullify or supersede contractual obligations assumed by
Company (MBTC) refused to enter into a dacion en pago arrangement the defendant.
contained in ASB’s proposed Rehabilitation Plan. MBTC argued, among others,
RULING: Yes. While non-impairment of contracts is constitutionally
that the forced transfer of properties and the diminution of its right to enforce
guaranteed, the rule is not absolute, since it has to be reconciled with the
its lien on the mortgaged properties violate its constitutional right against
legitimate exercise of police power, i.e. “the power to prescribe regulations to
impairment of contracts and right to due process. The Court ruled that there promote the health, morals, peace, education, good order or safety of the
is no impairment of contracts because the approval of the Rehabilitation Plan general welfare of the people.” This general welfare clause shall be liberally
and the appointment of a rehabilitation receiver merely suspend the action for interpreted in case of doubt, so as to give more power to local governments in
claims against the ASB Group, and MBTC may still enforce its preference when promoting the economic conditions, social welfare and material progress of the
the assets of the ASB Group will be liquidated. But if the rehabilitation is found people in the community. The only exceptions under Section 12 of the Local
to be no longer feasible, then the claims against the distressed corporation Autonomy Act (R.A. 2264) are existing vested rights arising out of a contract
would have to be settled eventually and the secured creditors shall between a province, city or municipality on one hand and a third party on the
enjoy preference over the unsecured ones. Moreover, the Court stated that other hand. Said case is not present in this petition.
there is no compulsion to enter into a dacion en pago agreement, nor to waive Resolution No. 27 s-1960 declaring the western part of EDSA as an industrial
the interests, penalties and related charges, since these are merely proposals and commercial zone was passed in the exercise of police power to safeguard
to creditors such as MBTC, such that in the event the secured creditors refuse or promote the health, safety, peace, good order and general welfare of the
the dacion, the Rehabilitation Plan proposes to settle the obligations to secured people in the locality.
creditors with mortgaged properties at selling prices.
c. Franchises, privileges and licenses
The Court reiterates that the SEC’s approval of the Rehabilitation Plan did not i. Ysmael vs. Deputy Executive Secretary, 190 SCRA 673
impair BPI’s right to contract. As correctly contended by private respondents,
the non-impairment clause is a limit on the exercise of legislative power and ISSUE: Whether the Timber Licensing Agreement is a contract.
not of judicial or quasi-judicial power. The SEC, through the hearing panel that RULING: Timber licenses, permits and license agreements are the principal
heard the petition for approval of the Rehabilitation Plan, was acting as a quasi- instruments by which the State regulates the utilization and disposition of
judicial body and, thus, its order approving the plan cannot constitute an forest resources to the end that public welfare is promoted. And it can hardly
impairment of the right and the freedom to contract. be gainsaid that they merely evidence a privilege granted by the State to
qualified entities, and do not vest in the latter a permanent or irrevocable right
Besides, the mere fact that the Rehabilitation Plan proposes
to the particular concession area and the forest products therein. They may be
a dacion en pago approach does not render it defective on the ground of
validly amended, modified, replaced or rescinded by the Chief Executive when

M.R.A.D.C. LUMBRE 182


CONSTITUTIONAL LAW REVIEW

national interests so require. Thus, they are not deemed contracts within the RULING: Yes. There is no merit in Gomez's claim that Edward's
purview of the due process of law clause. identification of her during trial might have been preconditioned by the
"suggestive identification" made during the police lineup.
The Court explained the procedure for out-of-court identification and the
12. Free Access to Courts and Quasi-Judicial Bodies and Adequate Legal test to determine the admissibility of such identifications in this manner:
Assistance
Out-of-court identification is conducted by the police in various ways. It is
a. Re: Query of Mr. Roger C. Prioreschi Re Exemption from Legal and Filing done thru show-ups where the suspect alone is brought face to face with
Fees of the Good Shepherd Foundation, Inc., 596 SCRA 40, AM No. 09-6-9- the witness for identification. It is done thru mug shots where photographs
SC, August 19, 2009 are shown to the witness to identify the suspect. It is also done
thru lineups where a witness identifies the suspect from a group of
RULING: The Court answers in the negative. The Courts cannot grant to persons lined up for the purpose x xx In resolving the admissibility of and
foundations like the Good Shepherd Foundation, Inc. the same exemption from relying on out-of-court identification of suspects, courts have adopted
payment of legal fees granted to indigent litigants even if the foundations are the totality of circumstances test where they consider the following
working for indigent and underprivileged people. The basis for the exemption from factors, viz: (1) the witness' opportunity to view the criminal at the time
legal and filing fees is the free access clause, embodied in Sec. 11, Art. III of the of the crime; (2) the witness' degree of attention at that time; (3) the
1987 Constitution, thus: accuracy of any prior description given by the witness; (4) the level of
Sec. 11. Free access to the courts and quasi-judicial bodies and adequate legal certainty demonstrated by the witness at the identification; (5) the length
assistance shall not be denied to any person by reason of poverty. of time between the crime and the identification; and (6) the
suggestiveness of the identification procedure.
The importance of the right to free access to the courts and quasi-judicial bodies
and to adequate legal assistance cannot be denied. A move to remove the provision Applying the totality-of-circumstances test, we find Edward's out-of-court
on free access from the Constitution on the ground that it was already covered by identification to be reliable and thus admissible.
the equal protection clause was defeated by the desire to give constitutional stature
ii. Luz vs. People, 667 SCRA 421 (2012)
to such specific protection of the poor. In implementation of the right of free access
under the Constitution, the Supreme Court promulgated rules, specifically, Sec. 21, RULING: There was no valid arrest, search and seizure.
Rule 3, Rules of Court, and Sec. 19, Rule 141, Rules of Court. Only a natural party
litigant may be regarded as an indigent litigant. Under R.A. 4136, or the Land Transportation and Traffic Code, the general
procedure for dealing with a traffic violation is not the arrest of the offender,
but the confiscation of the driver’s license of the latter.
13. Rights of Suspects Under Custodial Investigation At the time that he was waiting for PO3 Alteza to write his citation ticket, Luz
could not be said to have been under arrest. There was no intention on the
a. When rights available part of PO3 Alteza to arrest him, deprive him of his liberty, or take him into
The guarantees of Sec. 12(1), Art. III of the 1987 Constitution, or the so-called custody. Prior to the issuance of the ticket, the period during which Luz was at
Miranda rights, may be invoked only by a person while he is under custodial the police station may be characterized merely as waiting time. Hence, it was
investigation. Custodial investigation starts when the police investigation is no only for the sake of convenience that they were waiting there. There was no
longer a general inquiry into an unsolved crime but has begun to focus on a intention to take Luz into custody.
particular suspect taken into custody by the police who starts the interrogation and
Citing Berkemer v. McCarty,the Court ruled that roadside questioning of a
propounds questions to the person to elicit incriminating statements. Police line-up motorist detained pursuant to a routine traffic stop should not be considered
is not part of the custodial investigation; hence, the right to counsel guaranteed by
custodial interrogation. The Court held that, such questioning does not fall
the Constitution cannot yet be invoked at this stage. under custodial interrogation, nor can it be considered a formal arrest, by
i. Police line-up virtue of the nature of the questioning, the expectations of the motorist and
the officer, and the length of time the procedure is conducted.
1. People vs. Pepino, G.R. No. 174471, 12 January 2016
The U.S. Court in Berkemer thus ruled that, since the motorist therein was
ISSUE: Whether the identification of the accused in the police lineup is only subjected to modest questions while still at the scene of the traffic stop,
admissible. he was not at that moment placed under custody (such that he should have
been apprised of his Miranda rights), and neither can treatment of this sort be
fairly characterized as the functional equivalent of a formal arrest. Similarly,
neither can petitioner here be considered under arrest at the time that his

M.R.A.D.C. LUMBRE 183


CONSTITUTIONAL LAW REVIEW

traffic citation was being made. Even if one were to work under the assumption (1) Those which are the product of third degree methods such as torture,
that petitioner was deemed arrested upon being flagged down for a traffic force, violence, threat, intimidation, which are dealt with in paragraph 2
violation and while awaiting the issuance of his ticket, then the requirements of Section 12, and
for a valid arrest were not complied with.
(2) Those which are given without the benefit of Miranda warnings, which
The Court held that at the time a person is arrested, it shall be the duty of the are the subject of paragraph 1 of the same Section 12.
arresting officer to inform the latter of the reason for the arrest and must show
that person the warrant of arrest, if any. Persons shall be informed of their a. People vs. Obrero, 332 SCRA 190, May 17, 2000
constitutional rights to remain silent and to counsel, and that any statement RULING: Under the Constitution, an uncounseled statement, such as
they might make could be used against them. In this case, it may be noted it is called in the United States from which Article III, Section 12(1)
that these constitutional requirements were complied with by the police officers was derived, is presumed to be psychologically coerced. Swept into
only after petitioner had been arrested for illegal possession of dangerous an unfamiliar environment and surrounded by intimidating figures
drugs. typical of the atmosphere of police interrogation, the suspect really
In Berkemer, the U.S. Court also noted that the Miranda warnings must also needs the guiding hand of counsel.
be given to a person apprehended due to a traffic violation: Now, under the first paragraph of this provision, it is required that the
The purposes of the safeguards prescribed by Miranda are to ensure that the suspect in custodial interrogation must be given the following
police do not coerce or trick captive suspects into confessing, to relieve the warnings: (1) he must be informed of his right to remain silent; (2)
inherently compelling pressures generated by the custodial setting itself, which he must be warned that anything he says can and will be used against
work to undermine the individuals will to resist, and as much as possible to him; and (3) he must be told that he has a right to counsel, and that
free courts from the task of scrutinizing individual cases to try to determine, if he is indigent, a lawyer will be appointed to represent him…x xx
after the fact, whether particular confessions were voluntary. Those purposes There was thus only a perfunctory reading of the Miranda rights to
are implicated as much by in-custody questioning of persons suspected of accused-appellant without any effort to find out from him whether he
misdemeanors as they are by questioning of persons suspected of felonies. wanted to have counsel and, if so, whether he had his own counsel or
There being no valid arrest, the warrantless search that resulted from it was he wanted the police to appoint one for him. This kind of giving of
likewise illegal. warnings, in several decisions of this Court, has been found to be
merely ceremonial and inadequate to transmit meaningful information
NOTE: This is one of Sir’s favorite cases as it highlights two (2) important to the suspect. Especially in this case, care should have been
points. scrupulously observed by the police investigator that accused-
appellant was specifically asked these questions considering that he
First, flagging down of a motorist for violation of traffic regulation is NOT an only finished the fourth grade of the elementary school. X xx
arrest. Hence, there is no need to recite the Miranda warnings (Miranda rights
are read to the accused in a custodial investigation). Moreover, Article III, Section 12(1) requires that counsel assisting
suspects in custodial interrogations be competent and independent.
Second, a search incidental to a valid warrantless arrest is applicable ONLY Here, accused-appellant was assisted by Atty. De los Reyes, who,
where there was a valid warrantless arrest. The absence of the latter makes though presumably competent, cannot be considered an
the subsequent search void, and anything that was seized in said search is “independent counsel” as contemplated by the law for the reason that
considered inadmissible, being a “fruit of the poisonous tree.” he was station commander of the WPD at the time he assisted
b. Rights included accused-appellant.

i. Miranda Doctrine This is error. As observed in People v. Bandula (232 SCRA 566
[1994]), the independent counsel required by Article III, Section
xxx Any person under investigation for the commission of an offense shall have 12(1) cannot be special counsel, public or private prosecutor,
the right to be informed of his right to remain silent and to have competent municipal attorney, or counsel of the police whose interest is
and independent counsel, preferably of his own choice. If the person cannot admittedly adverse to the accused.
afford the services of counsel, he must be provided with one. These rights
cannot be waived except in writing and in the presence of counsel. (Section In this case, Atty. De los Reyes, as PC Captain and Station
12, Article III) Commander of the WPD, was part of the police force who could not
be expected to have effectively and scrupulously assisted accused-
1. Kinds of involuntary or coerced confessions appellant in the investigation. To allow such a happenstance would

M.R.A.D.C. LUMBRE 184


CONSTITUTIONAL LAW REVIEW

render illusory the protection given to the suspect during custodial b. Confession given to Municipal Mayor – People vs. Andan,
investigation. 269 SCRA 95, GR No. 116437, March 3, 1997

2. Right to counsel RULING: The appellant’s confession of guilt to a municipal mayor,


although not assisted by counsel, is admissible in evidence against
a. People vs. Viduya, 189 SCRA 403 (1990) him.
RULING: The accused cannot be made convicted of parricide by It is true that a municipal mayor has "operational supervision and
relying on her extrajudicial confession. control" over the local police and may arguably be deemed a law
The confession was made without the presence of the counsel, the enforcement officer for purposes of applying Section 12 (1) and (3)
reliance of the accused that a fiscal may act as her counsel is a of Article III of the Constitution. However, appellant's confession to
palpable mistake and a misunderstanding of her right to counsel. It is the mayor was not made in response to any interrogation by the
also elementary that a Fiscal or Prosecutor cannot be a lawyer for the latter. It was appellant himself who spontaneously, freely and
accused at the same time. voluntarily sought the mayor for a private meeting. When appellant
talked with the mayor as a confidant and not as a law enforcement
An assistant fiscal, or a fiscal for that matter, cannot exercise the officer, his uncounseled confession to him did not violate his
function of defense counsel even during custodial investigation. To constitutional rights.
allow such a happenstance would render illusory the protection given
to the accused during custodial investigation. What the Constitution Thus, it has been held that the constitutional procedures on custodial
requires in Article III Section 12 (1) is the presence of competent and investigation do not apply to a spontaneous statement, not elicited
independent counsel, one who will effectively undertake his client's through questioning by the authorities, but given in an ordinary
defense without any intervening conflict of interest. manner whereby appellant orally admitted having committed the
crime. What the Constitution bars is the compulsory disclosure of
Moreover, the answer of the accused ("Nandiyan naman po si Fiscal incriminating facts or confessions. The rights under Section 12 are
kaya hindi ko na kailangan ng abogado") is a palpable indication that guaranteed to preclude the slightest use of coercion by the state as
she did not fully understand her in-custody rights, hence it cannot be would lead the accused to admit something false, not to prevent him
said that she knowingly and intelligently waived those rights. from freely and voluntarily telling the truth.
3. Admissibility - “Fruit of the Poisonous Tree” Hence, appellant's confession to the mayor was correctly admitted by
the trial court. Appellant's confessions to the media were likewise
According to this rule, once the primary source (“the tree”) is shown to
properly admitted. The confessions were made in response to
have been unlawfully obtained, any secondary or derivative evidence (“the
questions by news reporters, not by the police or any other
fruit”) derived from it is also inadmissible. The rule is based on the
investigating officer. It was held that statements spontaneously made
principle that evidence illegally obtained by the State should not be used
by a suspect to news reporters on a televised interview are deemed
to gain other evidence, because the originally illegally obtained evidence
voluntary and are admissible in evidence. The Court therefore held
taints al evidence subsequently obtained.
accused-appellant Pablito Andan guilty of the special complex crime
a. Confession given to media – People vs. Endino, 353 SCRA of rape with homicide.
307, February 20, 2001
c. Giving urine samples – Gutang vs. People, 335 SCRA 479
RULING: The extrajudicial confession made by Galgarin in front of (2000)
the media is admissible.
RULING: The urine samples taken are admissible in evidence.
Apropos the court a quos admission of accused-appellants videotaped
The right to counsel begins from the time a person is taken into
confession, we find such admission proper. The interview was
custody and placed under investigation for the commission of crime.
recorded on video and it showed accused-appellant unburdening his
Such right is guaranteed by the Constitution and cannot be waived
guilt willingly, openly and publicly in the presence of newsmen. Such
except in writing and in the presence of counsel.
confession does not form part of custodial investigation as it
was not given to police officers but to media men in an attempt However, what the Constitution prohibits is the use of physical or
to elicit sympathy and forgiveness from the public. Besides, if moral compulsion to extort communication from the accused, but not
he had indeed been forced into confessing, he could have easily an inclusion of his body in evidence, when it may be material to
sought succor from the newsmen who, in all likelihood, would have ascertain physical attributes determinable by simple observation and
been sympathetic with him. not to unearth undisclosed facts.

M.R.A.D.C. LUMBRE 185


CONSTITUTIONAL LAW REVIEW

An accused may validly be compelled to be photographed or 1. Government of the USA vs. Purganan, GR No. 148571,
measured, or his garments or shoes removed or replaced, or to move September 24, 2002
his body to enable the foregoing things to be done without going
against the proscription against testimonial compulsion. RULING: Marc Jimenez is not entitled to post bail. Extradition case is
different from ordinary criminal proceedings. Article III, Section 13 of the
NOTE: The right against self-incrimination covers testimonial Constitution, as well as Section 4 of Rule 114 of the Rules of Court, applies
compulsion and acts which require (1) intelligence, and (2) attention. only when a person has been arrested and detained for violation of
Purely mechanical acts, i.e. taking of pregnancy test, or STD test, or Philippine criminal laws. It does not apply to extradition proceedings,
wearing of shorts are not included. because extradition courts do not render judgments of conviction or
acquittal. Jimenez should apply for bail before the courts trying the
d. Admissibility of other evidence obtained - Ho Wai Pang vs. criminal cases against him, not before the extradition court.
People, 659 SCRA 624 (2011)
The provision in the Constitution stating that the "right to bail shall not be
RULING: Constitutional provision of right under custodial impaired even when the privilege of the writ of habeas corpus is
investigation only prohibits as evidence, confessions and admissions suspended" does not detract from the rule that the constitutional right to
of the accused as against himself. The infractions of the so-called bail is available only in criminal proceedings. It must be noted that the
Miranda rights render inadmissible only the extrajudicial confession suspension of the privilege of the writ of habeas corpus finds application
or admission made during custodial investigation. The admissibility of "only to persons judicially charged for rebellion or offenses inherent in or
other evidence, provided they are relevant to the issue are not directly connected with invasion." 61 Hence, the second sentence in the
otherwise excluded by law or rules, and are not affected even if constitutional provision on bail merely emphasizes the right to bail in
obtained or taken in the course of custodial investigation. criminal proceedings for the aforementioned offenses. It cannot be taken
In the case, petitioner did not make any confession or admission to mean that the right is available even in extradition proceedings that are
during his custodial investigation. The prosecution did not present any not criminal in nature.
extrajudicial confession extracted from him as evidence of his guilt. 2. Government of Hongkong Special Administrative Region vs.
Verily, in determining the guilt of the petitioner and his co-accused, Olalia, Jr., GR No. 153675, April 19, 2007
the trial court based its Decision on the testimonies of the prosecution
witnesses and on the existence of the confiscated shabu. Any RULING: As a rule prospective extradites are entitled to notice and
allegation of violation of rights during custodial investigation is hearing only when the case is filed in court and not during the process of
relevant and material only to cases in which an extrajudicial admission evaluation.
or confession extracted from the accused becomes the basis of their
conviction. Hence, petitioner’s claim that the trial court erred in not Bail may be granted to a possible extraditee only upon a clear and
excluding evidence taken during the custodial investigation deserves convincing showing (1) that he will not be a flight risk or a danger to the
scant consideration. community; and (2) that there exist special, humanitarian and compelling
circumstances. (Rodriguez, et al. vs. The Hon. Presiding Judge, RTC,
Manila Branch 17, et al., supra.).
14. Right to Bail Extradition has been characterized as the right of a foreign power, created
by treaty, to demand the surrender of one accused or convicted of a crime
a. What is Bail - Section 1, Rule 114, Revised Rules of Criminal Procedure within its territorial jurisdiction, and the correlative duty of the other state
Bail is the security given for the release of a person in custody of the law, furnished to surrender him to the demanding state. It is not a criminal proceeding.
by him or a bondsman, to guarantee his appearance before any court as Even if the potential extraditee is a criminal, an extradition proceeding is
required under the conditions hereinafter specified. Bail may be given in the not by its nature criminal, for it is not punishment for a crime, even though
form of corporate surety, property bond, cash deposit, or recognizance. such punishment may follow extradition. It is sui generis, tracing its
existence wholly to treaty obligations between different nations. It is not
i. Extradition a trial to determine the guilt or innocence of the potential extraditee. Nor
is it a full-blown civil action, but one that is merely administrative in
Section 2(a) of Presidential Decree (P.D.) No. 1069 (The Philippine Extradition
character. Its object is to prevent the escape of a person accused or
Law) defines "extradition" as "the removal of an accused from the Philippines
convicted of a crime and to secure his return to the state from which he
with the object of placing him at the disposal of foreign authorities to enable
fled, for the purpose of trial or punishment.
the requesting state or government to hold him in connection with any criminal
investigation directed against him or the execution of a penalty imposed on [W]hile extradition is not a criminal proceeding, it is characterized by the
him under the penal or criminal law of the requesting state or government." following: (a) it entails a deprivation of liberty on the part of the potential

M.R.A.D.C. LUMBRE 186


CONSTITUTIONAL LAW REVIEW

extraditee and (b) the means employed to attain the purpose of As a rule prospective extradites are entitled to notice and hearing only
extradition is also “the machinery of criminal law.” This is shown by when the case is filed in court and not during the process of
Section 6 of P.D. No. 1069 (The Philippine Extradition Law) which evaluation.
mandates the “immediate arrest and temporary detention of the accused”
if such “will best serve the interest of justice.” We further note that Section Bail may be granted to a possible extraditee only upon a clear and
20 allows the requesting state “in case of urgency” to ask for the convincing showing (1) that he will not be a flight risk or a danger to
“provisional arrest of the accused, pending receipt of the request for the community; and (2) that there exist special, humanitarian and
extradition;” and that release from provisional arrest “shall not prejudice compelling circumstances.
re-arrest and extradition of the accused if a request for extradition is ii. Deportation
received subsequently.”
1. Go vs. Ramos, GR No. 167569, September 4, 2009
Obviously, an extradition proceeding, while ostensibly administrative,
bears all earmarks of a criminal process. A potential extraditee may be RULING: Once a person detained is duly charged in court, he may no
subjected to arrest, to a prolonged restraint of liberty, and forced to longer question his detention through a petition for issuance of a writ of
transfer to the demanding state following the proceedings. “Temporary habeas corpus. His remedy would be to quash the information and/or the
detention” may be a necessary step in the process of extradition, but the warrant of arrest duly issued. The writ of habeas corpus should not be
length of time of the detention should be reasonable. allowed after the party sought to be released had been charged before
any court. The term "court" in this context includes quasi-judicial bodies
If bail can be granted in deportation cases, we see no justification why it of governmental agencies authorized to order the person’s confinement,
should not also be allowed in extradition cases. Likewise, considering that like the Deportation Board of the Bureau of Immigration. Likewise, the
the Universal Declaration of Human Rights applies to deportation cases, cancellation of his bail cannot be assailed via a petition for habeas corpus.
there is no reason why it cannot be invoked in extradition cases. After all, When an alien is detained by the Bureau of Immigration for
both are administrative proceedings where the innocence or guilt of the deportation pursuant to an order of deportation by the Deportation
person detained is not in issue. Clearly, the right of a prospective Board, the Regional Trial Courts have no power to release such
extraditee to apply for bail in this jurisdiction must be viewed in the light alien on bail even in habeas corpus proceedings because there is
of the various treaty obligations of the Philippines concerning respect for no law authorizing it.
the promotion and protection of human rights. Under these treaties, the
presumption lies in favor of human liberty. Thus, the Philippines should iii. Military
see to it that the right to liberty of every individual is not impaired.
1. Comendador vs. De Villa, GR No. 93177, August 2, 1991
While our extradition law does not provide for the grant of bail to an
RULING: The right to bail invoked by the private respondents in G.R.
extraditee, however, there is no provision prohibiting him or her from filing
Nos. 95020 has traditionally not been recognized and is not available
a motion for bail, a right to due process under the Constitution
in the military, as an exception to the general rule embodied in the
In extradition proceeding being sui generis, the standard of proof required Bill of Rights.
in granting or denying bail can neither be the proof beyond reasonable
The right to a speedy trial is given more emphasis in the military
doubt in criminal cases nor the standard of proof of preponderance of
where the right to bail does not exist. The unique structure of the
evidence in civil cases. While administrative in character, the standard of
military should be enough reason to exempt military men from the
substantial evidence used in administrative cases cannot likewise apply
constitutional coverage on the right to bail. Aside from structural
given the object of extradition law which is to prevent the prospective
peculiarity, it is vital to note that mutinous soldiers operate within the
extraditee from fleeing our jurisdiction. In his Separate Opinion in
framework of democratic system, are allowed the fiduciary use of
Purganan, then Associate Justice, now Chief Justice Reynato S. Puno,
firearms by the government for the discharge of their duties and
proposed that a new standard which he termed “clear and convincing
responsibilities and are paid out of revenues collected from the
evidence” should be used in granting bail in extradition cases.
people. All other insurgent elements carry out their activities outside
According to him, this standard should be lower than proof beyond
of and against the existing political system. (See Arula v. Espino)
reasonable doubt but higher than preponderance of evidence. The
potential extraditee must prove by “clear and convincing The argument that denial from the military of the right to bail would
evidence” that he is not a flight risk and will abide with all the violate the equal protection clause is not acceptable. This guaranty
orders and processes of the extradition court. requires equal treatment only of persons or things similarly situated
and does not apply where the subject of the treatment is substantially
a. Quantum of proof in granting bail in extradition cases
different from others. The accused officers can complain if they are
denied bail and other members of the military are not. But they

M.R.A.D.C. LUMBRE 187


CONSTITUTIONAL LAW REVIEW

cannot say they have been discriminated against because they are For having ruled that an accused charged with the complex crime of
not allowed the same right that is extended to civilians. Malversation of Public Funds thru Falsification of Official/Public Documents
that involves an amount in excess of P22,000.00 is entitled to bail as a
b. When bail a matter of right – Section 4, Rule 114, Rules on Criminal matter of right, a summary hearing on bail application is, therefore,
Procedure unnecessary.
a. Before or after conviction by the metropolitan and municipal trial courts, c. When bail discretionary – Section 5, Rule 114, Rules on Criminal
and Procedure
b. Before conviction by the RTC of an offense not punishable by death, a. Upon conviction by the RTC of an offense not punishable by death, reclusion
reclusion perpetua or life imprisonment. (Sec. 4, Rule 114) perpetua or life imprisonment;
Under SC Issuance: b. Regardless of the stage of the criminal prosecution, a person charged with
c. Before final conviction by all children in conflict with the law for an offense a capital offense, or an offense punishable by reclusion perpetua or life
not punishable by reclusion perpetua or life imprisonment. imprisonment, when evidence of guilt is not strong. (Section 5, Rule 114)

i. People vs. Valdez, G.R. Nos. 216007-09, December 2015 Under SC Issuance:

RULING: Valdez is entitled to bail because the evidence of guilt is not c. A child in conflict with the law charged with an offense punishable by death,
strong. reclusion perpetua or life imprisonment when evidence of guilt is strong. (Sec.
28, A.M. No. 02-1-18-SC)
The term "punishable" should refer to prescribed, not imposable, penalty.
i. Juan Ponce Enrile vs. Sandiganbayan, G.R. No. 213847, 18
Following Temporada, for the complex crime of Malversation of Public August 2015
Funds thru Falsification of Official/Public Documents involving an amount
that exceeds P22,000.00, the "prescribed penalty" is reclusion temporal RULING: Enrile is entitled to bail as a matter of right based on
in its maximum period to reclusion perpetua. humanitarian grounds.

After trial, should the commission of such crime be proven by the The decision whether to detain or release an accused before and during
prosecution beyond reasonable doubt, the "imposable penalty" is reclusion trial is ultimately an incident of the judicial power to hear and determine
perpetua in view of the RPC mandate that the prescribed penalty of his criminal case. The strength of the Prosecution’s case, albeit a good
reclusion temporal maximum to reclusion perpetua shall be applied in its measure of the accused’s propensity for flight or for causing harm to the
maximum. The falsification, which is the means used to commit the crime public, is subsidiary to the primary objective of bail, which is to ensure
of malversation, is in the nature of a generic aggravating circumstance that the accused appears at trial.
that effectively directs the imposition of the prescribed penalty in its The Court is guided by the earlier mentioned principal purpose of bail,
maximum period. Finally, the "penalty actually imposed" is still reclusion which is to guarantee the appearance of the accused at the trial, or
perpetua, considering that the ISL finds no application as the penalty is whenever so required by the court. The Court is further mindful of the
indivisible. Philippines’ responsibility in the international community arising from the
At this point, there is no certainty that Valdez would be found guilty of national commitment under the Universal Declaration of Human Rights to:
Malversation of Public Funds thru Falsification of Official/Public Documents x x x uphold the fundamental human rights as well as value the worth and
involving an amount that exceeds P22,000.00. Falsification, like an dignity of every person. This commitment is enshrined in Section II, Article
aggravating circumstance, must be alleged and proved during the trial. II of our Constitution which provides: “The State values the dignity of
For purposes of bail proceedings, it would be premature to rule that the every human person and guarantees full respect for human rights.” The
supposed crime committed is a complex crime since it is only when the Philippines, therefore, has the responsibility of protecting and promoting
trial has terminated that falsification could be appreciated as a means of the right of every person to liberty and due process, ensuring that those
committing malversation. Further, it is possible that only the elements of detained or arrested can participate in the proceedings before a court, to
one of the constituent offenses, i.e., either malversation or falsification, or enable it to decide without delay on the legality of the detention and order
worse, none of them, would be proven after-full-blown trial. their release if justified. In other words, the Philippine authorities are
It would be the height of absurdity to deny Valdez the right to bail and under obligation to make available to every person under detention such
grant her the same only after trial if it turns out that there is no complex remedies which safeguard their fundamental right to liberty. These
crime committed. remedies include the right to be admitted to bail.

M.R.A.D.C. LUMBRE 188


CONSTITUTIONAL LAW REVIEW

This national commitment to uphold the fundamental human rights as well


as value the worth and dignity of every person has authorized the grant RECLUSION PERPETUA LIFE IMPRISONMENT
of bail not only to those charged in criminal proceedings but also to Pertains to the penalty Pertains to the penalty imposed
extraditees upon a clear and convincing showing: (1) that the detainee imposed for violation of the for violation of special laws.
will not be a flight risk or a danger to the community; and (2) that there RPC.
exist special, humanitarian and compelling circumstances. It has fixed duration. It has no fixed duration.
It carries with it accessory GR: It does not carry with it
In our view, his social and political standing and his having immediately penalties. accessory penalty.
surrendered to the authorities upon his being charged in court indicate XPN: Unless the law so
that the risk of his flight or escape from this jurisdiction is highly unlikely. provides.
His personal disposition from the onset of his indictment for plunder,
formal or otherwise, has demonstrated his utter respect for the legal
processes of this country. We also do not ignore that at an earlier time 15. Rights of an Accused During Trial
many years ago when he had been charged with rebellion with murder (1) No person shall be held to answer for a criminal offense without due process of law.
and multiple frustrated murder, he already evinced a similar personal
disposition of respect for the legal processes, and was granted bail during (2) In all criminal prosecutions, the accused shall be presumed innocent until the
the pendency of his trial because he was not seen as a flight risk. With his contrary is proved, and shall enjoy the right to be heard by himself and counsel, to be
solid reputation in both his public and his private lives, his long years of informed of the nature and cause of the accusation against him, to have a speedy,
public service, and history’s judgment of him being at stake, he should be impartial, and public trial, to meet the witnesses face to face, and to have compulsory
granted bail. process to secure the attendance of witnesses and the production of evidence in his
behalf. However, after arraignment, trial may proceed notwithstanding the absence of
NOTE: The phrases used by the Supreme Court, in granting Enrile’s the accused: Provided, that he has been duly notified and his failure to appear is
motion to fix bail, were: (1) regardless of the offense charged, whether unjustifiable. (Section 14, Article III)
punishable by death, life imprisonment, or reclusion perpetua; and (2)
regardless of the merits of the case, or whether the evidence of guilt is a. Summary of rights
strong or not.
i. Criminal due process
Because of this case, there are now THREE (3) instances wherein bail is a
ii. Presumption of innocence
matter of right, in addition to the two stated in Rule 114, the third one
being: (3) the continued incarceration of the accused poses danger to his 1. Equipoise Rule – Corpus vs. People, GR No. 74259, February 14,
life. 1991
This is considered judicial legislation used to accommodate the immortal RULING: The equipoise rule invoked by the petitioner is applicable only
senator. where the evidence of the parties is evenly balanced, in which case the
constitutional presumption of innocence should tilt the scales in favor of
One must also remember that after conviction in the RTC of offenses not
the accused. There is no such equipoise here. The evidence of the
punishable by D/RP/LI:
prosecution is overwhelming and has not been overcome by the petitioner
General Rule: Bail is discretionary. with his nebulous claims of persecution and conspiracy. The presumed
innocence of the accused must yield to the positive finding that he
Exception: If punishment is imprisonment of more than six (6) years AND malversed the sum of P50,310.87 to the prejudice of the public whose
attended by the ten (10) circumstances, bail is NOT discretionary. Bail confidence he has breached. His conviction must be affirmed.
shall be denied or cancelled.
2. People vs. Pepino, G.R. No. 174471, 12 January 2016
d. Distinction between Reclusion Perpetua and Life Imprisonment
RULING: The constitutional rights to presumption of innocence and
counsel were not violated.
As to admissibility of identification: Out of court identification is conducted
by the police in various ways. It is done thru show-ups where the suspect
alone is brought face to face with the witness for identification; thru mug
shots where photographs are shown to the witness to identify the suspect;
thru line-ups where a witness identified the suspect form a group of
persons lined up for the purpose. In resolving the admissibility of and

M.R.A.D.C. LUMBRE 189


CONSTITUTIONAL LAW REVIEW

relying on out-of-court identification of suspects, courts have adopted the NOTE: Thus, even if the other co-accused withdrew the appeal before the
totality of circumstances test where they consider the following factors. 1) passage of R.A. No. 9346, the reduced penalty brought about by the law
the witness’ opportunity to view the criminal at the time for the crime; 2) was applied as to him, because the effect of said law was favorable to the
the witness’ degree of attention at that time; 3) the accuracy of any prior accused.
description given by the witness; 4) the level of certainty demonstrated
by the witness at the identification; 5) the length of time between the iii. Right to be heard
crime and the identification; and 6) the suggestiveness if the identification iv. Right to be informed
procedure.
1. Juan Ponce Enrile vs. People, G.R. No. 213455, 11 August 2015
Applying the totality of circumstances test, the SC find Edward’s out-of-
court identification to be reliable and thus admissible. When the three RULING: In this case, the Court ruled that the requested detail by Enrile
individuals entered Edward’s office, they initially pretended to be on who among the accused acquired the ill-gotten wealth was not a proper
customers, and even asked about the products that were for sale. The subject in bill of particulars since the crime of plunder in the information
three had told Edward that they were going to pay, but Pepino pulled out was alleged to be committed through conspiracy and thus, it is
a gun instead. After Pepino’s companion had taken the money from the unnecessary to identify who among them acquired the ill-gotten wealth.
cashier’s box, the malefactors handcuffed Edward and forced him to go On the detail which project from 2004-2010 was ghost or spurious, among
down to the parked car. From this sequence of events, there were thus others, it was held that these are evidentiary matters that do not need to
ample opportunity for Edward.- before and after the gun had been pointed be reflected with particularity in the Information, and may be passed upon
at him – to view the faces of the three persons who entered his office. at the full-blown trial on the merits of the case. The Court also believed
that the exact amounts of Enrile’s yearly PDAF allocations, if any, from
Edward also saw Gomez seated at the front seat of the getaway metallic 2004 to 2010 need not be pleaded with specific particularity to enable him
green Toyota Corolla vehicle. In addition, the abductors removed the tape to properly plead and prepare for his defense since Enrile may be in a
from Edward’s eyes when they arrive at the apartment, and among those better position to know these details than the prosecution.
whom he saw there was Gomez.
However, the Court also found that Enrile was entitled for bill of particulars
Also, the lineup had not been attended by any suggestiveness on the part on certain details. One of which is what particular overt acts constitute the
of the police or NBI agents; there was no evidence that they had supplied crime of plunder. The Court held that the heart of the Plunder Law lies in
or even suggested to either Edward or Jocelyn that the appellants were the phrase combination or series of overt or criminal acts. Hence, even if
the kidnappers. the accumulated ill-gotten wealth amounts to at least P50 million, a person
As to the right to counsel: The right to counsel is a fundamental right and cannot be prosecuted for the crime of plunder if this resulted from a single
is intended to preclude the slightest coercion that would lead the accused criminal act. It is insufficient, too, to merely allege that a set of acts had
admit to something false. The right to counsel attaches upon the start of been repeatedly done (although this may constitute a series if averred
the investigation (when the investigating officers starts to ask questions with sufficient definiteness), and aver that these acts resulted in the
to elicit information and/or confession from the accused. accumulation or acquisition of ill-gotten wealth as in this case. The
Information should reflect with particularities the predicate acts that
Custodial investigation commences when a person is taken into custody underlie the crime of plunder. A reading of the Information filed against
and is singled out as a suspect in the commission of the crime under Enrile shows that the prosecution made little or no effort to particularize
investigation. As a rule, a police line-up is not part of the custodial the transactions that would constitute the required series or combination
investigation; hence the right to counsel guaranteed by the Constitution of overt acts. In addition, Enrile is also entitled for bill of particulars on the
cannot yet be invoked at this stage. The right to be assisted by counsel approximate dates of commission and kickbacks, and as the projects
attaches only during custodial investigation and cannot be claimed by the funded and the Napoles NGOs involved because he should not be left
accused during identification in a police lineup. guessing and speculating as to the periods, project and NGO involved.
The guarantees of Section 12(1), Article III, of the 1987 Constitution, or The Court further added that the Sandiganbayan also acted with grave
the so-called Miranda rights, may be invoked only by a person while he is abuse of discretion when it missed the legal points on the issue of the
under custodial investigation. Custodial investigation starts when the reiteration of the grounds in Enrile’s motion for bill of particulars and
police investigation is no longer a general inquiry into an unsolved crime opposition on the warrant of arrest. It was held that the question of
but has begun to focus on a particular suspect taken into custody by the whether there is probable cause to issue a warrant of arrest against an
police who starts the interrogation and propounds questions to the person accused, is separate and distinct from the issue of whether the allegations
to elicit incriminating statements. Police line-up is not a part of the in the Information have been worded with sufficient definiteness to enable
custodial investigation, hence, the right to counsel guaranteed by the the accused to properly plead and prepare his defense. While the grounds
Constitution cannot yet be invoked at this stage. cited for each may seemingly be the same, they are submitted for different

M.R.A.D.C. LUMBRE 190


CONSTITUTIONAL LAW REVIEW

purposes and should be appreciated from different perspectives, so that Speedy disposition of cases is embodied in Section 16, Article III. This
the insufficiency of these grounds for one does not necessarily translate right is not limited to the accused in criminal proceedings, but extends to
to insufficiency for the other. Thus, the resolution of the issue of probable all parties in all cases, including civil and administrative cases, and in all
cause should not bar Enrile from seeking a more detailed averment of the proceedings, including judicial and quasi-judicial hearings. Thus, any party
allegations in the Information. to a case may demand expeditious action on all his officials who are tasked
with the administration of justice.
2. Quimvel vs. People, G.R. No. 214497, 18 April 2017
Unlike the right to speedy trial, this constitutional privilege applies not
RULING: Petitioner’s contention that he was denied of his constitutional only during the trial stage but also when the case has already been
right to be informed of the nature and cause of the accusation against him submitted for decision.
was not violated is without merit.
vi. Right to meet the witnesses face to face (Confrontation Clause)
In criminal prosecutions, every element constituting the offense must be
alleged in the Information before an accused can be convicted of the crime The right to cross-examine complainant and witnesses. The Testimony of a
charged. This is to apprise the accused of the nature of the accusation witness who has not submitted himself to cross-examination is not admissible
against him. The Information must allege clearly and accurately the in evidence. The affidavits of the witnesses who are not present during the trial
elements of the crime charged. An investigating prosecutor is not required – thus are not subject to cross-examination are inadmissible because they are
to be absolutely accurate in designating the offense by its formal name in hearsay.
the law.
1. Rule on Examination of a Child Witness (AM No. 004-07-SC)
What determines the real nature and cause of the accusation against an
accused is the actual recital of facts stated in the Information or Section 13. Courtroom environment. - To create a more comfortable
Complaint, not the caption or preamble thereof nor the specification of the environment for the child, the court may, in its discretion, direct and
provision of law alleged to have been violated, being conclusions of law. supervise the location, movement and deportment of all persons in the
Under Article 336 of the RPC, the accused performs the acts of courtroom including the parties, their counsel, child, witnesses, support
lasciviousness on a child who is neither exploited in prostitution nor persons, guardian ad litem, facilitator, and court personnel. The child may
subjected to "other sexual abuse." be allowed to testify from a place other than the witness chair. The witness
chair or other place from which the child testifies may be turned to
In contrast, under Section 5 of RA 7610, the accused performs the acts of facilitate his testimony but the opposing party and his counsel must have
lasciviousness on a child who is either exploited in prostitution or a frontal or profile view of the child during the testimony of the child. The
subjected to "other sexual abuse." The very definition of "child abuse" witness chair or other place from which the child testifies may also be
under Sec. 3(b) of RA 7610 does not require that the victim suffer a rearranged to allow the child to see the opposing party and his counsel, if
separate and distinct act of sexual abuse aside from the act complained he chooses to look at them, without turning his body or leaving the witness
of. It refers to the maltreatment, whether habitual or not, of the child. stand. The judge need not wear his judicial robe.

Thus, a violation of Sec. 5(b) of RA 7610 occurs even though the accused
committed sexual abuse against the child victim only once, even without Nothing in this section or any other provision of law, except official
a prior sexual affront. in-court identification provisions, shall be construed to require a
child to look at the accused.
v. Right to speedy, impartial and public trial
Section 25. Live-link television testimony in criminal cases where the
1. Speedy trial vs. Speedy disposition of cases; application
child is a victim or a witness. – xxxx
A speedy trial is one free from vexatious, capricious, and oppressive
delays or when unjustified postponements of the trial are asked for and (d) The judge may exclude any person, including the accused,
secured, or when without cause or justifiable motive, a long period of time whose presence or conduct causes fear to the child.
is allowed to elapse without the party having his case tried. Accused is
entitled to dismissal, equivalent to an acquittal, if trial is unreasonably Section 26. Screens, one-way mirrors, and other devices to shield
delayed. child from accused. –

R.A. No. 8493 is a means of enforcing the right of the accused to a speedy (a) The prosecutor or the guardian ad litem may apply for an order
trial. The spirit of the law is that the accused must go on record in the that the chair of the child or that a screen or other device be placed
attitude of demanding a trial or resisting delay. If he does not do this, he in the courtroom in such a manner that the child cannot see the
must be held, in law, to have waived the privilege. accused while testifying. Before the guardian ad litem applies for

M.R.A.D.C. LUMBRE 191


CONSTITUTIONAL LAW REVIEW

an order under this section, he shall consult with the prosecutor 18. Right Against Self-Incrimination
or counsel subject to the second and third paragraphs of section
25(a) of this Rule. The court shall issue an order stating the No person shall be compelled to be a witness against himself. (Section 17, Article III)
reasons and describing the approved courtroom arrangement. a. Scope and coverage

(b) If the court grants an application to shield the child from the Scope: The right against self-incrimination includes the right to refuse to testify to
accused while testifying in the courtroom, the courtroom shall be a fact which would be a necessary link in a chain of evidence to prove the
arranged to enable the accused to view the child. commission of a crime by a witness.

Coverage: The self-incrimination clause only applies to compulsory testimonial. It


does not apply to material objects. It refers therefore to the use of the mental
16. Writ of Habeas Corpus process and the communicative faculties, and not to a merely physical activity. If
The writ of habeas corpus is a writ directed to the person detaining another, the act is physical or mechanical, the accused can be compelled to allow or perform
commanding him to produce the body of the detainee at a designated time and place, the act, and the result can be used in evidence against him.
and to show the cause of his detention. i. Alih vs. Castro, 151 SCRA 279
Rule 102, Sec 1 states that it extends to all cases of illegal confinement or detention ISSUE: Whether the seizing of the items and taking of the fingerprints and
by which any person is deprived of his liberty, or by which the rightful custody of any photographs and subjecting them to paraffin testing are violative of the
person is withheld from the person entitled thereto. constitutional right against self-incrimination.
a. Go vs. Ramos, GR No. 167569, September 4, 2009 RULING: The court held that superior orders nor the suspicion that the
RULING: A petition for the issuance of a writ of habeas corpus is a special respondents had against petitioners did not excuse the former from observing
proceeding governed by Rule 102 of the Revised Rules of Court. The objective of the guaranty provided for by the constitution against unreasonable searches
the writ is to determine whether the confinement or detention is valid or lawful. If and seizure. The petitioners were entitled to due process and should be
it is, the writ cannot be issued. What is to be inquired into is the legality of a protected from the arbitrary actions of those tasked to execute the law.
person’s detention as of, at the earliest, the filing of the application for the writ of Furthermore, there was no showing that the operation was urgent nor was
habeas corpus, for even if the detention is at its inception illegal, it may, by reason there any showing of the petitioners as criminals or fugitives of justice to merit
of some supervening events, such as the instances mentioned in Section 4 of Rule approval by virtue of Rule 113, Section 5 of the Rules of Court.
102, be no longer illegal at the time of the filing of the application. Once a person The items seized, having been the “fruits of the poisonous tree” were held
detained is duly charged in court, he may no longer question his detention through inadmissible as evidence in any proceedings against the petitioners. The
a petition for issuance of a writ of habeas corpus. His remedy would be to quash operation by the respondents was done without a warrant and so the items
the information and/or the warrant of arrest duly issued. seized during said operation should not be acknowledged in court as evidence.
The writ of habeas corpus should not be allowed after the party sought to be But said evidence should remain in the custody of the law (custodia legis).
released had been charged before any court. The term “court” in this context However, as to the issue on finger-printing, photographing and paraffin-testing
includes quasi-judicial bodies of governmental agencies authorized to order the as violative of the provision against self-incrimination, the court held that the
person’s confinement, like the Deportation Board of the Bureau of Immigration. prohibition against self-incrimination applies to testimonial compulsion only.
Likewise, the cancellation of his bail cannot be assailed via a petition for habeas
corpus. When an alien is detained by the Bureau of Immigration for deportation As Justice Holmes put it in Holt v. United States, “The prohibition of compelling
pursuant to an order of deportation by the Deportation Board, the Regional Trial a man in a criminal court to be a witness against himself is a prohibition of the
Courts have no power to release such alien on bail even in habeas corpus use of physical or moral compulsion to extort communications from him, not
proceedings because there is no law authorizing it. an exclusion of his body as evidence when it may be material.”
ii. Bataan Shipyard and Engineering Corporation vs. PCGG, 150 SCRA
181
17. Speedy Disposition of Cases
RULING: It is elementary that the right against self-incrimination has no
All persons shall have the right to a speedy disposition of their cases before all judicial, application to juridical persons. While an individual may lawfully refuse to
quasi-judicial, or administrative bodies. (Section 16, Article III) answer incriminating questions unless protected by an immunity statute, it
does not follow that a corporation, vested with special privileges and
franchises, may refuse to show its hand when charged with an abuse of such
privileges.

M.R.A.D.C. LUMBRE 192


CONSTITUTIONAL LAW REVIEW

Corporations are not entitled to all of the constitutional protections which determine whether or not he is the falsifier, as the petition of the respondent
private individuals have. They are not at all within the privilege against self- fiscal clearly states. Except that it is more serious, we believe the present case
incrimination, although this court more than once has said that the privilege is similar to that of producing documents or chattels in one's possession. And
runs very closely with the 4th Amendment's Search and Seizure provisions. It as to such production of documents or chattels, which to our mind is not so
is also settled that an officer of the company cannot refuse to produce its serious as the case now before us, the same eminent Professor Wigmore, in
records in its possession upon the plea that they will either incriminate him or his work cited, says:
may incriminate it.
. . . 2264. Production or Inspection of Documents and Chattels. — 1. It
At any rate, Executive Order No. 14-A, amending Section 4 of Executive Order follows that the production of documents or chattels by a person (whether
No. 14 assures protection to individuals required to produce evidence before ordinary witness or party-witness) in response to a subpoena, or to a
the PCGG against any possible violation of his right against self-incrimination. motion to order production, or to other form of process treating him as a
It gives them immunity from prosecution on the basis of testimony or witness (i.e. as a person appearing before a tribunal to furnish testimony
information he is compelled to present. on his moral responsibility for truthtelling), may be refused under the
protection of the privilege; and this is universally conceded.
iii. Beltran vs. Samson and Jose, 53 Phil 57
We say that, for the purposes of the constitutional privilege, there is a similarity
RULING: As to its scope, this privilege is not limited precisely to testimony, between one who is compelled to produce a document, and one who is
but extends to all giving or furnishing of evidence. compelled to furnish a specimen of his handwriting, for in both cases, the
The rights intended to be protected by the constitutional provision that no man witness is required to furnish evidence against himself.
accused of crime shall be compelled to be a witness against himself is so It cannot be contended in the present case that if permission to obtain a
sacred, and the pressure toward their relaxation so great when the suspicion specimen of the petitioner's handwriting is not granted, the crime would go
of guilt is strong and the evidence obscure, that is the duty of courts liberally unpunished. Considering the circumstance that the petitioner is a municipal
to construe the prohibition in favor of personal rights, and to refuse to permit treasurer, according to Exhibit A, it should not be a difficult matter for the fiscal
any steps tending toward their invasion. Hence, there is the well-established to obtained genuine specimens of his handwriting. But even supposing it is
doctrine that the constitutional inhibition is directed not merely to giving of impossible to obtain specimen or specimens without resorting to the means
oral testimony, but embraces as well the furnishing of evidence by other means complained herein, that is no reason for trampling upon a personal right
than by word of mouth, the divulging, in short, of any fact which the accused guaranteed by the constitution. It might be true that in some cases criminals
has a right to hold secret. may succeed in evading the hand of justice, but such cases are accidental and
Whenever the defendant, at the trial of his case, testifying in his own behalf, do not constitute the raison d' etre of the privilege. This constitutional privilege
denies that a certain writing or signature is in his own hand, he may on cross- exists for the protection of innocent persons.
examination be compelled to write in open court in order that the jury may be With respect to the judgments rendered by this court and cited on behalf of
able to compare his handwriting with the one in question. the respondents, it should be remembered that in the case of People vs.
It was so held in the case of Bradford vs. People inasmuch as the defendant, Badilla, it does not appear that the defendants and other witnesses were
in offering himself as witness in his own behalf, waived his personal privileges. questioned by the fiscal against their will, and if they did not refuse to answer,
they must be understood to have waived their constitutional privilege, as they
Of like character is the case of Sprouse vs. Com., where the judge asked the could certainly do.
defendant to write his name during the hearing, and the latter did so
voluntarily. The privilege not to give self-incriminating evidence, while absolute when
claimed, maybe waived by any one entitled to invoke it.
But the cases so resolved cannot be compared to the one now before us. We
are not concerned here with the defendant, for it does not appear that any The same holds good in the case of United States vs. Tan Teng, were the
information was filed against the petitioner for the supposed falsification, and defendant did not oppose the extraction from his body of the substance later
still less as it a question of the defendant on trial testifying and under cross- used as evidence against him.
examination. This is only an investigation prior to the information and with a In the case of Villaflor vs. Summers, it was plainly stated that the court
view to filing it. And let it further be noted that in the case of Sprouse vs. Com., preferred to rest its decision on the reason of the case rather than on blind
the defendant performed the act voluntarily. adherence to tradition. The said reason of the case there consisted in that it
Furthermore, in the case before us, writing is something more than moving was the case of the examination of the body by physicians, which could be and
the body, or the hands, or the fingers; writing is not a purely mechanical act, doubtless was interpreted by this court, as being no compulsion of the
because it requires the application of intelligence and attention; and in the case petitioner therein to furnish evidence by means of testimonial act. In reality
at bar writing means that the petitioner herein is to furnish a means to she was not compelled to execute any positive act, much less a testimonial

M.R.A.D.C. LUMBRE 193


CONSTITUTIONAL LAW REVIEW

act; she was only enjoined from something preventing the examination; all of We also rule that there was nothing irregular when PCGG granted a section 5
which is very different from what is required of the petitioner of the present immunity to petitioners while they were already undergoing trial in Criminal
case, where it is sought to compel him to perform a positive, testimonial act, Case No. 11960. Section 5 of E.O. 14, as amended, does not prohibit the PCGG
to write and give a specimen of his handwriting for the purpose of comparison. from granting immunity to persons already charged in court and undergoing
Besides, in the case of Villamor vs. Summers, it was sought to exhibit trial. As long as the privilege of immunity so given will in the judgment of the
something already in existence, while in the case at bar, the question deals PCGG assist it in attaining its greater objectives, the PCGG is well within legal
with something not yet in existence, and it is precisely sought to compel the grounds to exercise this power at any stage of the proceedings. This section 5
petitioner to make, prepare, or produce by this means, evidence not yet in immunity frees and releases one from liability, and as it inures to the benefit
existence; in short, to create this evidence which may seriously incriminate of an accused, it can be invoked at any time after its acquisition and before his
him. final conviction. Our regard for the rights of an accused dictates this result.

Wherefore, we find the present action well taken, and it is ordered that the Thus, we have consistently held that laws that decriminalize an act or a grant
respondents and those under their orders desist and abstain absolutely and of amnesty may be given retroactive effect. They constitute a bar against the
forever from compelling the petitioner to take down dictation in his handwriting further prosecution of their beneficiaries' regardless of the appearance of their
for the purpose of submitting the latter for comparison. guilt.

b. Statutory immunity 1. Power of the Commission on Human Rights – Sec. 18(8), Art.
XIII
There are two types of statutory immunity granted to a witness. They are the
transactional immunity and the used-and-derivative-use immunity. Transactional xxx “Grant immunity from prosecution to any person whose testimony or
immunity is broader in the scope of its protection. By its grant, a witness can no whose possession of documents or other evidence is necessary or
longer be prosecuted for any offense whatsoever arising out of the act or convenient to determine the truth in any investigation conducted by it or
transaction. 4 In contrast, by the grant of used-and-derivative-use immunity, under its authority.” xxx
a witness is only assured that his or her particular testimony and evidence derived
from it will not be used against him or her in a subsequent prosecution.
i. Two types – Mapa, Jr. vs. Sandiganbayan, 231 SCRA 783, GR No. 19. Freedom of Political Beliefs (political prisoners)
100295, April 26, 1994 No person shall be detained solely by reason of his political beliefs and aspirations.
RULING: Contrary to the ruling of the respondent court, the failure of (Section 18(1), Article III)
petitioners to testify in the RICO cases against the Marcoses in New York
cannot nullify their immunity. They have satisfied the requirements both of the
law and the parties' implementing agreements. Under section 5 of E.O. No. 14, 20. Freedom Against Involuntary Servitude – Relate with Art. II, Sec. 4
as amended, their duty was to give information to the prosecution, and they
No involuntary servitude in any form shall exist except as a punishment for a crime
did. Under their Memorandum of Agreement, they promised to make
whereof the party shall have been duly convicted. (Section 18(2), Article III)
themselves available as witnesses in the said RICO cases, and they did.
Petitioners were ready to testify but they were not called to testify by the US The prime duty of the Government is to serve and protect the people. The Government
prosecutors of the RICO case. Their failure to testify was not of their own may call upon the people to defend the State and, in the fulfillment thereof, all citizens
making. It was brought about by the decision of the US prosecutors who may may be required, under conditions provided by law, to render personal, military or civil
have thought that their evidence was enough to convict the Marcoses. Since service. (Section 4, Article II)
petitioners' failure to testify was not of their own choosing nor was it due to
any fault of their own, justice and equity forbid that they be penalized by the NOTE: The latter provision serves as an exception to Section 18, Article III.
withdrawal of their immunity.
Indeed, initially, the PCGG itself adopted the posture that the immunity of 21. Right Against Cruel, Degrading and Inhuman Punishment
petitioners stayed and should not be disturbed. It joined the motion to dismiss
filed by petitioners in the respondent court. When the respondent court denied Excessive fines shall not be imposed, nor cruel, degrading or inhuman punishment
the motion, PCGG stuck to its previous position as it again joined the inflicted. Neither shall death penalty be imposed, unless, for compelling reasons
petitioners in their motion for reconsideration. It is only in this petition for involving heinous crimes, the Congress hereafter provides for it. Any death penalty
review on certiorari that PCGG, after a change of Chairman, flip-flopped in its already imposed shall be reduced to reclusion perpetua.
position.

M.R.A.D.C. LUMBRE 194


CONSTITUTIONAL LAW REVIEW

The employment of physical, psychological, or degrading punishment against any jeopardy is available only where an identity is shown to exist between the
prisoner or detainee or the use of substandard or inadequate penal facilities under earlier and the subsequent offenses charged.
subhuman conditions shall be dealt with by law. (Section 19, Article III)
ii. Same “Act”
1. People vs. Relova, 148 SCRA 292 [1987]
22. Right Against Imprisonment for Debt or Non-Payment of a Poll Tax
RULING: The first sentence of Article IV (22) sets forth the general rule: the
No person shall be imprisoned for debt or non-payment of a poll tax. (Section 20, constitutional protection against double jeopardy is not available where the
Article III) second prosecution is for an offense that is different from the offense charged
in the first or prior prosecution, although both the first and second offenses
may be based upon the same act or set of acts. The second sentence of Article
23. Double-Jeopardy IV (22) embodies an exception to the general proposition: the constitutional
protection, against double jeopardy is available although the prior offense
No person shall be twice put in jeopardy of punishment for the same offense. If an act
charged under an ordinance be different from the offense charged
is punished by a law and an ordinance, conviction or acquittal under either shall
subsequently under a national statute such as the Revised Penal Code,
constitute a bar to another prosecution for the same act. (Section 21, Article III)
provided that both offenses spring from the same act or set of acts.
a. Two kinds
That is, where the offenses charged are penalized either by different sections
i. Same “Offense” of the same statute or by different statutes, the important inquiry relates to
the identity of offenses charge: the constitutional protection against double
1. Lamera vs. Court of Appeals, 198 SCRA 186 (1991) jeopardy is available only where an Identity is shown to exist between the
RULING: The SC affirmed that the Articles 275 and 365 penalize different and earlier and the subsequent offenses charged. In contrast, where one offense
distinct offenses. The rule on double jeopardy, which petitioner has, in effect, is charged under a municipal ordinance while the other is penalized by a
invoked, does not, therefore, apply pursuant to existing jurisprudence. Hence, statute, the critical inquiry is to the identity of the acts which the accused is
the petition should be dismissed for lack of merit. said to have committed and which are alleged to have given rise to the two
offenses: the constitutional protection against double jeopardy is available so
Legal jeopardy attaches only (a) upon a valid indictment, (b) before a long as the acts which constitute or have given rise to the first offense under
competent court, (c) after arraignment, (d) a valid plea having been entered, a municipal ordinance are the same acts which constitute or have given rise to
and (e) the case was dismissed or otherwise terminated without the express the offense charged under a statute.
consent of the accused.
The question may be raised why one rule should exist where two offenses
He is charged for two separate offenses under the Revised Penal Code. In under two different sections of the same statute or under different statutes are
People vs. Doriquez, the SC held that it is a cardinal rule that the protection charged, and another rule for the situation where one offense is charged under
against double jeopardy may be invoked only for the same offense or identical a municipal ordinance and another offense under a national statute. If the
offenses. Where two different laws (or articles of the same code) defines two second sentence of the double jeopardy provision had not been written into
crimes, prior jeopardy as to one of them is no obstacle to a prosecution of the the Constitution, conviction or acquittal under a municipal ordinance would
other, although both offenses arise from the same facts, if each crime involves never constitute a bar to another prosecution for the same act under a national
some important act which is not an essential element of the other. statute. An offense penalized by municipal ordinance is, by definition, different
from an offense under a statute. The two offenses would never constitute the
The two informations filed against petitioner are clearly for separate offenses. same offense having been promulgated by different rule-making authorities —
The first, for reckless imprudence (Article 365), falls under the sole chapter though one be subordinate to the other — and the plea of double jeopardy
(Criminal Negligence) of Title Fourteen (Quasi Offenses) of Book Two of the would never lie.
Revised Penal Code. The second, for Abandonment of one's victim (par. 2, Art.
275), falls under Chapter Two (Crimes Against Security) of Title Nine (Crimes In the instant case, the relevant acts took place within the same time frame:
Against Personal Liberty and Security) of Book Two of the same Code. from November 1974 to February 1975. During this period, the accused Manuel
Opulencia installed or permitted the installation of electrical wiring and devices
Quasi offenses under Article 365 are committed by means of culpa. Crimes in his ice plant without obtaining the necessary permit or authorization from
against Security are committed by means of dolo. the municipal authorities. The accused conceded that he effected or permitted
Where the offenses charged are penalized either by different sections of the such unauthorized installation for the very purpose of reducing electric power
same statute or by different statutes, the important inquiry relates to the bill. This corrupt intent was thus present from the very moment that such
identity of the offenses charged. The constitutional protection against double unauthorized installation began. The immediate physical effect of the
unauthorized installation was the inward flow of electric current into

M.R.A.D.C. LUMBRE 195


CONSTITUTIONAL LAW REVIEW

Opulencia's ice plant without the corresponding recording thereof in his electric 3. Attempted;
meter. In other words, the "taking" of electric current was integral with the
unauthorized installation of electric wiring and devices. 4. Necessarily included; or

It is perhaps important to note that the rule limiting the constitutional 5. Necessarily includes
protection against double jeopardy to a subsequent prosecution for the same the first offense, there is double jeopardy.
offense is not to be understood with absolute literalness. The Identity of
offenses that must be shown need not be absolute Identity: the first and Double jeopardy for the same act, on the other hand, only requires that so
second offenses may be regarded as the "same offense" where the second long as the same act is punishable by law and an ordinance, the conviction or
offense necessarily includes the first offense or is necessarily included in such acquittal of one, shall constitute as bar to another prosecution for the same
first offense or where the second offense is an attempt to commit the first or act.
a frustration thereof. Thus, for the constitutional plea of double jeopardy to be
b. Requisites
available, not all the technical elements constituting the first offense need be
present in the technical definition of the second offense. The law here seeks to i. Ivler vs. Modesto-San Pedro, 635 SCRA 191, G.R. No. 172716
prevent harrassment of an accused person by multiple prosecutions for November 17, 2010
offenses which though different from one another are nonetheless each
constituted by a common set or overlapping sets of technical elements. RULING: The accused’s negative constitutional right not to be "twice put in
Further, the dismissal by the Batangas City Court of the information for jeopardy of punishment for the same offense" protects him from, among
violation of the Batangas City Ordinance upon the ground that such offense others, post-conviction prosecution for the same offense, with the prior verdict
had already prescribed, amounts to an acquittal of the accused of that offense. rendered by a court of competent jurisdiction upon a valid information.
Under Article 89 of the Revised Penal Code, "prescription of the crime" is one Petitioner adopts the affirmative view, submitting that the two cases concern
of the grounds for "total extinction of criminal liability." Under the Rules of the same offense of reckless imprudence. The MTC ruled otherwise, finding
Court, an order sustaining a motion to quash based on prescription is a bar to that Reckless Imprudence Resulting in Slight Physical Injuries is an entirely
another prosecution for the same offense. separate offense from Reckless Imprudence Resulting in Homicide and Damage
NOTE: Under the first kind of double jeopardy (for the same offense), the to Property "as the [latter] requires proof of an additional fact which the other
following must be considered: does not."

First, the first jeopardy must have attached. The requisites are the following: The two charges against petitioner, arising from the same facts, were
prosecuted under the same provision of the Revised Penal Code, as amended,
1. There is a valid complaint/information charging the accused with an namely, Article 365 defining and penalizing quasi-offenses.
offense;
The provisions contained in this article shall not be applicable. Indeed, the
2. The same is filed before a court of competent jurisdiction; notion that quasi-offenses, whether reckless or simple, are distinct species of
crime, separately defined and penalized under the framework of our penal
3. The accused has been arraigned and has pleaded; and
laws, is nothing new.
4. Accused was previously acquitted, convicted, or the case was dismissed
The doctrine that reckless imprudence under Article 365 is a single quasi-
or otherwise terminated without his express consent.
offense by itself and not merely a means to commit other crimes such that
As to the fourth (4th) requisite, dismissal, even with the express consent of the conviction or acquittal of such quasi-offense bars subsequent prosecution for
accused, will have the effect of first jeopardy, if the: the same quasi-offense, regardless of its various resulting acts, undergirded
this Court’s unbroken chain of jurisprudence on double jeopardy as applied to
1. Dismissal was due to the insufficiency of evidence (i.e. accused filed a Article 365.
demurrer to evidence, and the same was granted by the court); and
These cases uniformly barred the second prosecutions as constitutionally
2. Accused’s right to speedy trial was violated (the dismissal of the case impermissible under the Double Jeopardy Clause.
will have the effect of acquittal).
Our ruling today secures for the accused facing an Article 365 charge a
Second, once the first jeopardy has validly attached, the subsequent filing of stronger and simpler protection of their constitutional right under the Double
an information for an offense: Jeopardy Clause. True, they are thereby denied the beneficent effect of the
1. Identical; favorable sentencing formula under Article 48, but any disadvantage thus
caused is more than compensated by the certainty of non-prosecution for
2. Frustrated; quasi-crime effects qualifying as "light offenses" (or, as here, for the more

M.R.A.D.C. LUMBRE 196


CONSTITUTIONAL LAW REVIEW

serious consequence prosecuted belatedly). If it is so minded, Congress can was caused by the very superficial examination then made. As we have stated,
re-craft Article 365 by extending to quasi-crimes the sentencing formula of we find therefore that no supervening fact had occurred which justifies the
Article 48 so that only the most severe penalty shall be imposed under a single application of the rule in the case of Melo vs. People and People vs. Manolong,
prosecution of all resulting acts, whether penalized as grave, less grave or light for which reason we are constrained to apply the general rule of double
offenses. This will still keep intact the distinct concept of quasi-offenses. jeopardy.
Meanwhile, the lenient schedule of penalties under Article 365, befitting crimes
occupying a lower rung of culpability, should cushion the effect of this ruling. d. MRs and appeals

ii. People vs. Sandiganbayan, GR No. 164185, July 23, 2008 i. People vs. Tria-Tirona, GR No. 130106, July 15, 2005

RULING: Yes, the granting of demurrer to evidence amounts to acquittal. ISSUE: Whether or not the court will consider the appeal considering that the
decision acquitting the accused is final.
Once a court grants the demurrer to evidence, such order amounts to an
acquittal and any further prosecution of the accused would violate the RULING: No. It is clear in this jurisdiction that after trial on the merits, an
constitutional proscription on double jeopardy, hence such ruling shall not be acquittal is immediately final and cannot be appealed on the ground of double
disturbed in the absence of a grave abuse of discretion. jeopardy. The only exception where double jeopardy cannot be invoked is
where there is a finding of mistrial resulting in a denial of due process.
Notably, a judgment rendered with grave abuse of discretion or without due
process is void, does not exist in legal contemplation and, thus, cannot be the We find petitioner’s argument that, despite our ruling in People v. Velasco,
source of an acquittal. since we gave due course to the petition, the issue on the sufficiency of the
evidence may be reviewed, to be untenable. The fact that the petition was
c. Doctrine of Supervening Event / Supervening Fact Doctrine given due course does not necessarily mean we have to look into the
sufficiency of the evidence since the issue to be resolved is the appealability of
The accused may still be prosecuted for another offense if a subsequent an acquittal. We have categorically ruled in People v. Velasco20 that, except
development changes the character of the first indictment under which he may when there is a finding of mistrial, no appeal will lie in case of an acquittal.
have already been charged or convicted. There being no mistrial in the case before us, we find no need to reexamine
i. People vs. Buling, 107 Phil 712 [1960] the evidence, because if we do so, we will be allowing an appeal to be made
on an acquittal which would clearly be in violation of the accused’s right against
RULING: NO. We are inclined to agree with the contention made on behalf of double jeopardy.
appellant that no new supervening fact has existed or occurred, which has
transformed the offense from less serious physical injuries to serious physical ii. Lejano vs. People, GR No. 176389, January 18, 2011
injuries. ISSUE: Whether or not the judgment by the Court can be reconsidered and
Two conflicting doctrines on double jeopardy have been enunciated by this the Court should re-examine the case.
Court: RULING: NO. Because a reconsideration of a judgment of acquittal places the
. . . Stating it in another form, the rule is that "where after the first prosecution accused under double jeopardy.
a new fact supervenes for which the defendant is responsible, which changes On occasions, a motion for reconsideration after an acquittal is possible. But
the character of the offense and, together with the facts existing at the time, the grounds are exceptional and narrow as when the court that absolved the
constitutes a new and distinct offense," the accused cannot be said to be in accused gravely abused its discretion, resulting in loss of jurisdiction, or when
second jeopardy if indicted for the new offense.. a mistrial has occurred. In any of such cases, the State may assail the decision
If the X-ray examination discloses the existence of a fracture on January 17, by special civil action of certiorari under Rule 65.
1957, that fracture must have existed when the first examination was made Here, although complainant Vizconde invoked the exceptions, he has been
on December 10, 1956. There is, therefore, no now or supervening fact that unable to bring his pleas for reconsideration under such exceptions. For
could be said to have developed or arisen since the filing of the original action, instance, he avers that the Court "must ensure that due process is afforded to
which would justify the application of the ruling enunciated by us in the cases all parties and there is no grave abuse of discretion in the treatment of
of Melo vs. People and People vs. Manolong, supra. We attribute the new witnesses and the evidence." But he has not specified the violations of due
finding of fracture, which evidently lengthened the period of healing of the process or acts constituting grave abuse of discretion that the Court
wound, to the very superficial and inconclusive examination made on supposedly committed. His claim that "the highly questionable and suspicious
December 10, 1956. Had an X-ray examination taken at the time, the fracture evidence for the defense taints with serious doubts the validity of the decision"
would have certainly been disclosed. The wound causing the delay in healing is, without more, a mere conclusion drawn from personal perception.
was already in existence at the time of the first examination, but said delay

M.R.A.D.C. LUMBRE 197


CONSTITUTIONAL LAW REVIEW

Ultimately, what the complainant actually questions is the Court’s appreciation A legislative act which inflicts punishment without judicial trial. If the punishment
of the evidence and assessment of the prosecution witnesses’ credibility. He is less than death, the act is termed a bill of pains and penalties. (Cummings v.
ascribes grave error on the Court’s finding that Alfaro was not a credible Missouri)
witness and assails the value assigned by the Court to the evidence of the
defense. In other words, private complainant wants the Court to review the Elements of Bill of Attainder:
evidence anew and render another judgment based on such a re-evaluation. 1. There must be a law.
This is not constitutionally allowed as it is merely a repeated attempt to secure
Webb, et al’s conviction. The judgment acquitting Webb, et al is final and can 2. The law imposes a penal burden on a named individual or easily ascertainable
no longer be disturbed. members of a group.
3. There is a direct imposition of penal burden without judicial trial.

24. Prohibition Against Ex Post Facto Law or Bill of Attainder i. People vs. Ferrer, 48 SCRA 382 (1972)

No ex post facto law or bill of attainder shall be enacted. (Section 22, Article III) RULING: No. The Supreme Court held that the Anti-Subversion Act is not a
bill of attainder, because it does not specify the Communist Party of the
a. Kinds of ex post facto law Philippines or the members thereof for the purpose of punishment; what it
(1) One which makes an action done before the passing of the law, and which was does is simply declare the Party to be an organized conspiracy to overthrow
innocent when done, criminal, and punishes such action. the Government; and the term “Communist Party of the Philippines” is used
solely for definitional purposes.
(2) One which aggravates the crime or makes it greater than when it was
committed. Only when a statute applies either to named individuals or to easily
ascertainable members of a group in such a way as to inflict punishment on
(3) One which changes the punishment and inflicts a greater punishment than them without a judicial trial does it become a bill of attainder.
that which the law annexed to the crime when it was committed.
IX. Citizenship (will be discussed in Election Law)
(4) One which alters the legal rules of evidence and receives less testimony than
the law required at the time of the commission of the offense in order to convict CHAPTER 2: ELECTION LAW
the accused.
(5) One which assumes to regulate civil rights and remedies only BUT, in effect,
imposes a penalty or deprivation of a right, which, when done, was lawful. I. COMELEC
(6) One which deprives a person accused of a crime of some lawful protection to
which he has become entitled such as the protection of a former conviction or
acquittal, or a proclamation of amnesty. Section 1. (1) There shall be a Commission on Elections composed of a Chairman and
six Commissioners who shall be natural-born citizens of the Philippines and, at the time
i. In Re Kay Villegas Kami, Inc., 35 SCRA 429, GR No. L-32485, October of their appointment, at least thirty-five years of age, holders of a college degree, and
22, 1970 must not have been candidates for any elective positions in the immediately preceding
elections. However, a majority thereof, including the Chairman, shall be members of
RULING: From the aforesaid definition as well as classification of ex post facto
the Philippine Bar who have been engaged in the practice of law for at least ten years.
laws, the constitutional inhibition refers only to criminal laws which are given
retroactive effect. (2) The Chairman and the Commissioners shall be appointed by the President with the
consent of the Commission on Appointments for a term of seven years without
While it is true that Sec. 18 penalizes a violation of any provision of R.A. No.
reappointment. Of those first appointed, three Members shall hold office for seven
6132 including Sec. 8(a) thereof, the penalty is imposed only for acts
years, two Members for five years, and the last Members for three years, without
committed after the approval of the law and not those perpetrated prior
reappointment. Appointment to any vacancy shall be only for the unexpired term of the
thereto. There is nothing in the law that remotely insinuates that Secs. 8(a)
predecessor. In no case shall any Member be appointed or designated in a temporary
and 18, or any other provision thereof, shall apply to acts carried out prior to
or acting capacity.
its approval. On the contrary, See. 23 directs that the entire law shall be
effective upon its approval. It was approved on August 24, 1970. Sec. 2. The Commission on Elections shall exercise the following powers and functions:
b. Bill of attainder (1) Enforce and administer all laws and regulations relative to the conduct of an
election, plebiscite, initiative, referendum, and recall.

M.R.A.D.C. LUMBRE 198


CONSTITUTIONAL LAW REVIEW

(2) Exercise exclusive original jurisdiction over all contests relating to the elections, Section 4. The Commission may, during the election period, supervise or regulate the
returns, and qualifications of all elective regional, provincial, and city officials, and enjoyment or utilization of all franchises or permits for the operation of transportation
appellate jurisdiction over all contests involving elective municipal officials decided and other public utilities, media of communication or information, all grants, special
by trial courts of general jurisdiction, or involving elective barangay officials decided by privileges, or concessions granted by the Government or any subdivision, agency, or
trial courts of limited jurisdiction. instrumentality thereof, including any government-owned or controlled corporation or
its subsidiary. Such supervision or regulation shall aim to ensure equal opportunity,
Decisions, final orders, or rulings of the Commission on election contests involving time, and space, and the right to reply, including reasonable, equal rates therefor, for
elective municipal and barangay offices shall be final, executory, and not public information campaigns and forums among candidates in connection with the
appealable. (NOTE: Can only be assailed by way of certiorari under Rule 64 in relation objective of holding free, orderly, honest, peaceful, and credible elections.
to Rule 65)
Section 5. No pardon, amnesty, parole, or suspension of sentence for violation of
(3) Decide, except those involving the right to vote, all questions affecting elections, election laws, rules, and regulations shall be granted by the President without the
including determination of the number and location of polling places, appointment of favorable recommendation of the Commission.
election officials and inspectors, and registration of voters.
Section 6. A free and open party system shall be allowed to evolve according to the
(4) Deputize, with the concurrence of the President, law enforcement agencies and free choice of the people, subject to the provisions of this Article.
instrumentalities of the Government, including the Armed Forces of the Philippines, for
the exclusive purpose of ensuring free, orderly, honest, peaceful, and credible elections. Section 7. No votes cast in favor of a political party, organization, or coalition shall be
valid, except for those registered under the party-list system as provided in this
(5) Register, after sufficient publication, political parties, organizations, or coalitions Constitution.
which, in addition to other requirements, must present their platform or program of
government; and accredit citizens’ arms of the Commission on Elections. Religious Section 8. Political parties, or organizations or coalitions registered under the party-
denominations and sects shall not be registered. Those which seek to achieve their goals list system, shall not be represented in the voters’ registration boards, boards of election
through violence or unlawful means, or refuse to uphold and adhere to this Constitution, inspectors, boards of canvassers, or other similar bodies. However, they shall be entitled
or which are supported by any foreign government shall likewise be refused registration. to appoint poll watchers in accordance with law.

Financial contributions from foreign governments and their agencies to political parties, Section 9. Unless otherwise fixed by the Commission in special cases, the election
organizations, coalitions, or candidates related to elections, constitute interference in period shall commence ninety days before the day of election and shall end thirty days
national affairs, and, when accepted, shall be an additional ground for the cancellation thereafter.
of their registration with the Commission, in addition to other penalties that may be
prescribed by law. Section 10. Bona fide candidates for any public office shall be free from any form of
harassment and discrimination.
(6) File, upon a verified complaint, or on its own initiative, petitions in court for inclusion
or exclusion of voters; investigate and, where appropriate, prosecute cases of violations Section 11. Funds certified by the Commission as necessary to defray the expenses
of election laws, including acts or omissions constituting election frauds, offenses, and for holding regular and special elections, plebiscites, initiatives, referenda, and recalls,
malpractices. shall be provided in the regular or special appropriations and, once approved, shall be
released automatically upon certification by the Chairman of the Commission.
(7) Recommend to the Congress effective measures to minimize election spending,
including limitation of places where propaganda materials shall be posted, and to Definitions:
prevent and penalize all forms of election frauds, offenses, malpractices, and nuisance Election. The means by which the people choose their officials for a definite and fixed
candidates. period, and to whom they entrust for the time being the exercise of the powers of the
(8) Recommend to the President the removal of any officer or employee it has government.
deputized, or the imposition of any other disciplinary action, for violation or disregard Plebiscite. The submission of constitutional amendments of important legislative
of, or disobedience to, its directive, order, or decision. measures to the people for ratification.
(9) Submit to the President and the Congress, a comprehensive report on the conduct Initiative. The power of the people to propose amendments to the Constitution, or to
of each election, plebiscite, initiative, referendum, or recall. propose and enact legislation through an election called for the purpose. (Sec. 2(a),
Section 3. The Commission on Elections may sit en banc or in two divisions, and shall R.A. 6735)
promulgate its rules of procedure in order to expedite disposition of election cases, Referendum. The power of the electorate to approve or reject national or local legislation
including pre-proclamation controversies. All such election cases shall be heard and through an election called for the purpose.
decided in division, provided that motions for reconsideration of decisions shall be
decided by the Commission en banc.

M.R.A.D.C. LUMBRE 199


CONSTITUTIONAL LAW REVIEW

Recall. The termination of official relationship of a local elective official for loss of Original and Exclusive:
confidence prior to the expiration of his term through the will of the electorate.

Power: President/Vice Supreme Court/Presidential


President Electoral Tribunal
1. Ascertainment of the identity of a political party and its legitimate officers. (LDP v.
COMELEC) Senator Senate Electoral Tribunal
2. Authority to annul the results of a plebiscite, to fix other dates for a plebiscite. (Cagas
v. COMELEC) Representative House of the Representatives
Electoral Tribunal
3. Regulatory power over media of transportation, communication, and information
during election period, i.e. enjoyment or utilization of all franchises and permits. (Sec.
4, Art. IX-C, National Press Club v. COMELEC) Regional/Provincial/City COMELEC

4. No pardon, amnesty, parole, or suspension of sentence for violation of election laws, Municipal Regional Trial Court
rules and regulations shall be granted by the President without the favorable
recommendation of COMELEC. (Sec. 5, Art. IX-C)
Barangay Municipal/Metropolitan Trial Court
5. Power to declare failure of election. (Sison v. COMELEC)

6. No power of apportionment. Appellate:

7. COMELEC cannot make an official quick count of presidential election results. From decisions of the RTC and MTC, appeal shall be made exclusively to the COMELEC,
(Brillantes v. COMELEC) within 5 days from promulgation or receipt of a copy thereof, whose decision shall be
final and executory, and not appealable. A motion for reconsideration is a prohibited
8. COMELEC cannot deny due course or cancel certificate of candidacy in due form pleading and does not interrupt the running of the 5-day period to appeal.
without proper proceedings. (Cipriano v. COMELEC)
The COMELEC cannot deprive the RTC of its competence to order the execution of its
9. May deny request for special registration of voters in the youth sector who failed decision pending appeal.
within the prescribed period. (Akbayan Youth v. COMELEC)
In the exercise of its appellate jurisdiction the COMELEC has the power to issue writs of
prohibition, mandamus, or certiorari.
1. Reyes vs. COMELEC, G.R. No. 207264, June 25, 2013 From the decisions of the COMELEC, appeal shall be made through a Petition for Review
on Certiorari under Rule 64, in relation to Rule 65, to be filed with the Supreme Court
The jurisdiction of the HRET begins only after the candidate is considered a Member of
within 30 days from receipt of a copy of the decision, on the ground of grave abuse of
the House of Representatives (NOTE: 30th of June next following their elections, as
discretion, amounting to lack or excess of jurisdiction or violation of due process.
provided by Sec. 7, Art. VI), prior to this, the COMELEC retains jurisdiction.
From a decision of an Electoral Tribunal appeal shall be made through a Petition for
To be considered a Member of the House of Representatives, there must be a
Review on Certiorari under Rule 65, filed with the Supreme Court within 30 days from
concurrence of the following requisites:
receipt of a copy of the decision, on the ground of grave abuse of discretion, amounting
(1) a valid proclamation, to lack or excess of jurisdiction or violation of due process.

(2) a proper oath, and 2. Velasco vs. Speaker Belmonte, G.R. No. 211140, 12 January 2016

(3) assumption of office. DOCTRINE: To be considered a Member of the House of Representatives, there must
be a concurrence of the following requisites: (1) a valid proclamation, (2) a proper oath,
NOTE: Jurisdiction of courts in electoral contests. and (3) assumption of office.
3. Legaspi vs. COMELEC, G.R. No. 216572, 19 April 2016
This voting threshold, however, is easily rendered illusory by the application of the
Mendoza ruling, which virtually allows the grant of a motion for reconsideration even
though the movant fails to secure four votes in his or her favor, in blatant violation of
Sec. 7, Art. IX-A of the Constitution. In this case, in spite of securing only two (2) votes

M.R.A.D.C. LUMBRE 200


CONSTITUTIONAL LAW REVIEW

to grant their motion for reconsideration, private respondents were nevertheless “[T]he effects of the COMELEC en banc's failure to decide vary depending on the type
declared the victors in the January 28, 2015 COMELEC en banc Resolution. of case or matter that is before the commission. Thus, under the provision, the first
effect (i.e., the dismissal of the action or proceeding) only applies when the type of case
To exacerbate the situation, the circumvention of the four-vote requirement, in turn, before the COMELEC is an action or proceeding "originally commenced in the
trivializes the proceedings before the COMELEC divisions and presents rather commission"; the second effect (i.e., the affirmance of a judgment or order) only applies
paradoxical scenarios, to wit: when the type of case before the COMELEC is an "appealed case"; and the third effect
(i.e., the denial of the petition or motion) only applies when the case or matter before
1. The failure of the COMELEC en banc to muster the required majority vote only the COMELEC is an "incidental matter."
means that it could not have validly decided the case. Yet curiously, it managed to
reverse the ruling of a body that has properly exercised its adjudicatory powers; and The motion for reconsideration before the COMELEC en banc is an incidental matter, a.
The pending issue at the time was not directly private respondents' qualification or
2. A motion for reconsideration may be filed on the ground that the evidence is disqualification to run for or hold office, but, more precisely, whether or not the
insufficient to justify the decision, order or ruling; or that the said decision, order or COMELEC division committed reversible error in its ruling.
ruling is contrary to law. If the COMELEC en banc does not find that either ground In the case of League Cities v. COMELEC, the Court applied Sec. 7, Rule 56 of the Rules
exists, there would be no cogent reason to disturb the ruling of the COMELEC division. of Court, which bears striking similarity to Sec. 6, Rule 18 of the COMELEC Rules of
Otherwise stated, failure to muster four votes to sustain the motion for Procedure. In that ruling Justice Carpio explained that a motion for reconsideration is
reconsideration should be understood as tantamount to the COMELEC en banc finding an incidental matter, and that application of Section 7, Rule 56 has been clarified in
no reversible error attributable to its division's ruling. Said decision, therefore, ought A.M. No. 99-1-09-SC wherein the Court resolved as follows:
to be affirmed, not reversed nor vacated.
“A motion for reconsideration of a decision or resolution of the Court En Banc or of a
These resultant paradoxes have to be avoided. Under the prevailing interpretation of Division may be granted upon a vote of a majority of the members of the En Banc or of
Sec. 6, Rule 18 of the COMELEC Rules of Procedure, a movant, in situations such as a Division, as the case may be, who actually took part in the deliberation of the motion.
this, need not even rely on the strength of his or her arguments and evidence to win a If the voting results in a tie, the motion for reconsideration is deemed denied.”
case, and may, instead, choose to rest on inhibitions and abstentions of COMELEC
members to produce the same result. To demonstrate herein, it is as though the two
(2) abstention votes were counted in favor of the private respondents to reach the
majority vote of four (4). This impedes and undermines the adjudicatory powers of the
COMELEC divisions by allowing their rulings to be overruled by the en banc without the
latter securing the necessary number to decide the case.

As discussed in the September 1, 2015 ponencia:

“[T]he effects of the COMELEC en banc's failure to decide vary depending on the
type of case or matter that is before the commission. Thus, under the provision,
the first effect (i.e., the dismissal of the action or proceeding) only applies when the
type of case before the COMELEC is an action or proceeding "originally commenced in
the commission"; the second effect (i.e., the affirmance of a judgment or order) only
applies when the type of case before the COMELEC is an "appealed case"; and the third
effect (i.e., the denial of the petition or motion) only applies when the case or
matter before the COMELEC is an "incidental matter."
Under Sec. 3, Article IX-C of the 1987 Constitution, the COMELEC Divisions are granted
adjudicatory powers to decide election cases, provided that the COMELEC en banc shall
resolve motions for reconsideration of the division rulings. Further, under Sec. 7, Article
IX-A of the Constitution, four (4) votes are necessary for the COMELEC en banc to
decide a case. Naturally, the party moving for reconsideration, as the party seeking
affirmative relief, carries the burden of proving that the division committed reversible
error. The movant then shoulders the obligation of convincing four (4) Commissioners
to grant his or her plea.

M.R.A.D.C. LUMBRE 201


CONSTITUTIONAL LAW REVIEW

II. Registration of Voters SEC. 4. Katipunan ng Kabataan. – There shall be in every barangay a Katipunan
ng Kabataan to be composed of all citizens of the Philippines residing in the
barangay for at least six (6) months, who are at least fifteen (15) but not more
1. Qualification and Disqualification of Voters than thirty (30) years of age, and who are duly registered in the list of the
Commission on Elections (COMELEC) and/or the records of the Sangguniang
Registration is the act of accomplishing and filing a sworn application for the registration Kabataan secretary.
by a qualified voter before the election officer of the city or municipality wherein he
resides and including the same in the book of registered voters upon approval by the
Election registration board. (Voters Registration Act 1996, RA. 8189). It does not confer 2. Inclusion and Exclusion Proceedings
the right to vote; it is but a conditional precedent to the exercise of the right.
Registration is a regulation, not a qualification. Inclusion Proceedings: Any person whose application for registration has been
disapproved by the board or whose name has been stricken out from the list may file
Qualifications: with the court (MTC) a petition to include his name in the permanent list of voters in
1. Filipino Citizenship his precinct.

2. At least 18 years of age Exclusion Proceedings: Any registered voter, representative of a political party or the
Election Officer, may file with the court a sworn petition for the exclusion of a voter
3. Resident of the Philippines for at least one year from the permanent list of voters giving name, address and the precinct of the
challenged voter.
4. Resident of the place where he proposes to vote for at least 6 months
immediately preceding the election. a. Inclusion and Exclusion Proceedings vs. Denial of Due Course to or
Cancellation of COC Proceedings - Velasco v. Comelec, G.R. No. 180051,
5. Not otherwise disqualified by law. December 24, 2008
Disqualifications:
In terms of purpose, voter’s inclusion/exclusion and COC denial/cancellation are
1. Sentenced by a final judgment to suffer imprisonment for not less than 1 year, different proceedings; one refers to the application to be registered as a voter to
unless pardoned or granted amnesty. be eligible to vote, while the other refers to the application to be a candidate.
Because of their differing purposes, they also involve different issues and entail
2. Conviction by final judgment of any of the following different reliefs although the facts on which they rest may have commonalities
where they may be said to converge or interface.
a. Crime involving disloyalty to the government
The remedies available in the two proceedings likewise differ. xxx From the MTC,
b. Violation against national security
the recourse is to the RTC whose decision is final and executory, correctible by the
c. Firearms law Court of Appeals only by a writ of certiorari based on grave abuse of discretion
amounting to lack of jurisdiction. On the other hand, the approval of a certificate
The right to vote is reacquired upon expiration of 5 years after service of sentence of candidacy or its denial is a matter directly cognizable by the COMELEC, with the
referred to in the two preceding items. decision of its Division reviewable by the COMELEC en banc whose decision is in
a. Kabataan Party List vs. COMELEC, G.R. No. 221318, 16 December 2015 turn reviewable by this Court under Rule 64 of the Rules of Court and Section 7, of
Article IX-A of the 1987 Constitution.
It has long been recognized that the right to vote is not a natural right but is a right
created by law. Suffrage is a privilege granted by the State to such persons or
classes as are most likely to exercise it for the public good.
As to the procedural limitation, the right of a citizen to vote is necessarily
conditioned upon certain procedural requirements he must undergo: among others,
the process of registration. Specifically, a citizen in order to be qualified to exercise
his right to vote, in addition to the minimum requirements set by the fundamental
charter, is obliged by law to register, at present, under the provisions of Republic
Act No. 8189, otherwise known as the Voters Registration Act of 1996.
b. RA 10742, Sec. 4

M.R.A.D.C. LUMBRE 202


CONSTITUTIONAL LAW REVIEW

III. Political Parties and Party-lists


INCLUSION EXCLUSION

1. Any private person whose 1. Any registered voter in the city or


1. Political Parties to Candidates
application was disapproved by the municipality (Sec. 142, OEC);
Election Registration Board; or a. Omnibus Election Code (OEC), Secs. 70-71
2. Representative of political party;
2. Those whose names were stricken Section 70. Guest Candidacy. – A political party may nominate and/or support
out from the list of voters (Sec. 139, 3. Election officer;
candidates not belonging to it.
OEC); or 4. COMELEC (OEC).
Section 71. Changing political party affiliation. – An elective official may change
3. COMELEC. his party affiliation for purposes of the election next following his change of party
within 1 year prior to such election.
Any time except 105 days before the Any time except 100 days before a
NOTE: Disregard Section 71 as it was already repealed.
regular election; or regular election; or
2. The Party-list System
75 days before a special election. 65 days before a special election
(COMELEC Reso. 8820) (COMELEC Reso. 9021) It is a mechanism of proportional representation in the election of representatives to
the HoR from national, regional and sectoral parties or organizations or coalitions
1. Application for registration has 1. Not qualified for possession registered with the COMELEC. The party-list representatives shall constitute 20% of the
been disapproved by the Board disqualification; total number of representatives in the HoR including those under the party-list.
(ERB);
2. Flying voter; The purpose of party-list system is to make the marginalized and the underrepresented
2. Name has been stricken out in the not merely passive recipient, and to democratize political power by giving political
list of voters. 3. Ghost voter. parties that cannot win in legislative district elections a chance to win seats in the HoR.
a. The four parameters in the Party-List election

b. Cases:
Inclusion/Exclusion Proceedings v. Sec. 78
i. Veterans Federation Party vs. COMELEC, GR No. 136781, 6 October
INCLUSION/ DENIAL/ 2000
EXCLUSION CANCELLATION
The twenty-percent (20%) allocation for party-list representatives mentioned
Application with the Election Application to deny due course or in the Constitution is not mandatory, it merely provides a ceiling for the party-
Registration Board (ERB) to be cancel a Certificate of Candidacy list seats in the HoR. In the exercise of its constitutional prerogative, Congress
registered as a voter. (COC). deemed it necessary to require parties participating in the system to obtain at
least 2% of the total votes cast for the party list system to be entitled to a
The issue involves whether False representation must refer to a party-list seat. Congress wanted to ensure that only those parties having a
petitioner shall be included/excluded material fact (qualification), which sufficient number of constituents deserving of representation are actually
from the list of voters. includes status as a registered voter. represented in Congress.
FORMULA FOR:
Remedy: MTC to RTC (decision is Remedy: COMELEC to COMELEC en
final and executory). CA only on the banc via MR. SC only via Rule 65 in 1. Determination of total number of party-list representatives = # of district
ground of grave abuse of discretion. relation to Rule 65. representatives/.80 x .20 additional representatives of first party = # of votes
of first party/ # of votes of party list system
Commonality: includes status as a registered voter. 2. Additional seats for concerned party = # of votes of concerned party/ #
votes of first party x additional seats for concerned party
ii. BANAT vs. COMELEC, GR No. 179271, 21 April 2009

The Four Parameters:

M.R.A.D.C. LUMBRE 203


CONSTITUTIONAL LAW REVIEW

(1) The twenty percent allocation pertains to the combined number of all party- In declaring the two percent threshold unconstitutional, we do not limit our
list congressmen shall not exceed twenty percent of the total membership of allocation of additional seats in Table 3 below to the two-percenters. The
the House of Representatives, including those elected under the party list; percentage of votes garnered by each party-list candidate is arrived at by
dividing the number of votes garnered by each party by 15,950,900, the total
(2) Under the two percent threshold parties garnering a minimum of two number of votes cast for party-list candidates. There are two steps in the
percent of the total valid votes cast for the party-list system are qualified to second round of seat allocation. First, the percentage is multiplied by the
have a seat in the House of Representatives; remaining available seats, 38, which is the difference between the 55
(3) The three-seat limit states that each qualified party, regardless of the maximum seats reserved under the Party-List System and the 17 guaranteed
number of votes it actually obtained, is entitled to a maximum of three seats; seats of the two-percenters. The whole integer of the product of the percentage
that is, one qualifying and two additional seats; and of the remaining available seats corresponds to a partys share in the
remaining available seats. Second, we assign one party-list seat to each of the
(4) Under proportional representation the additional seats which a qualified parties next in rank until all available seats are completely distributed. We
party is entitled to shall be computed in proportion to their total number of distributed all of the remaining 38 seats in the second round of seat allocation.
votes. Finally, we apply the three-seat cap to determine the number of seats each
qualified party-list candidate is entitled.
Neither the Constitution nor R.A. No. 7941 mandates the filling-up of the entire
20% allocation of party-list representatives found in the Constitution. FORMULA:
However, we cannot allow the continued existence of a provision in the law
which will systematically prevent the constitutionally allocated 20% party-list 1. F1 = [(No. of Seats in Congress)/(0.8)] * 0.2 = Seats for Party List
representatives from being filled. Paragraph 1, Section 5 of Article VI, 1987 Members;
Constitution provides: 2. F2 = No. of Votes Garnered for ALL Political Parties / No. of Votes Garnered
“The parties, organizations, and coalitions receiving at least two for Each Party = To determine ranking and percentage of votes;
percent (2%) of the total votes cast for the party-list system shall be 3. F3 = F1 – Seats Given to Qualified Parties = To determine how many
entitled to one seat each: Provided, That those garnering more than additional seats may be given.
two percent (2%) of the votes shall be entitled to additional seats in
proportion to their total number of votes.” 4. F4 = F2 * f3 = To determine how many additional seats may be given to
party list groups. Maximum of 3 allowed.
In determining the allocation of seats for party-list representatives under
Section 11 of R.A. No. 7941, the following procedure shall be observed: NOTE: Difference between Veterans and BANAT is that in Veterans, the Court
ruled that the 20% allocation rule is merely a ceiling and filling up is not
1. The parties, organizations, and coalitions shall be ranked from the mandatory, as opposed to its later ruling in BANAT. Moreover, the Court gave
highest to the lowest based on the number of votes they garnered during a simple formula in determining the number of allowed seats per party list by
the elections. giving 1 seat for those who got 2% but below 4%, 2 seats for 4% and above
2. The parties, organizations, and coalitions receiving at least two percent but below 6%, and 3 seats for those who got 6% and above. In BANAT, since
(2%) of the total votes cast for the party-list system shall be entitled to the Court allowed the filling up of all seats for party list groups, the formula
one guaranteed seat each. abovementioned was used.

3. Those garnering sufficient number of votes, according to the ranking in iii. Atong Paglaum vs. COMELEC, GR No. 203766, 02 April 2013
paragraph 1, shall be entitled to additional seats in proportion to their total 1. Three different groups may participate in the party-list system:
number of votes until all the additional seats are allocated.
(1) national parties or organizations,
4. Each party, organization, or coalition shall be entitled to not more than
three (3) seats. (2) regional parties or organizations, and
In computing the additional seats, the guaranteed seats shall no longer be (3) sectoral parties or organizations.
included because they have already been allocated, at one seat each, to every
two-percenter. Thus, the remaining available seats for allocation as additional 2. National parties or organizations and regional parties or organizations do
seats are the maximum seats reserved under the Party List System less the not need to organize along sectoral lines and do not need to represent any
guaranteed seats. Fractional seats are disregarded in the absence of a "marginalized and underrepresented" sector.
provision in R.A. No. 7941 allowing for a rounding off of fractional seats.

M.R.A.D.C. LUMBRE 204


CONSTITUTIONAL LAW REVIEW

3. Political parties can participate in party-list elections provided they and the Party-List System Act. As an administrative agency, it cannot amend
register under the party-list system and do not field candidates in legislative an act of Congress nor issue IRRs that may enlarge, alter or restrict the
district elections. provisions of the law it administers and enforces. Section 8 of R.A. No. 7941
provides that: Each registered party, organization or coalition shall submit to
A political party, whether major or not, that fields candidates in legislative the COMELEC not later than forty-five (45) days before the election a list of
district elections can participate in party-list elections only through its names, not less than five (5), from which party-list representatives shall be
sectoral wing that can separately register under the party-list system. chosen in case it obtains the required number of votes.
The sectoral wing is by itself an independent sectoral party, and is linked to a A person may be nominated in one (1) list only. Only persons who have given
political party through a coalition. their consent in writing may be named in the list. The list shall not include any
4. Sectoral parties or organizations may either be: candidate of any elective office or a person who has lost his bid for an elective
office in the immediately preceding election. No change of names or
(1)"marginalized and underrepresented" or alteration of the order of nominees shall be allowed after the same
shall have been submitted to the COMELEC except in cases where the
(2) lacking in "well-defined political constituencies."
nominee dies, or withdraws in writing his nomination, becomes
It is enough that their principal advocacy pertains to the special interest and incapacitated in which case the name of the substitute nominee shall
concerns of their sector. The sectors that are "marginalized and be placed last in the list. Incumbent sectoral representatives in the House
underrepresented" include labor, peasant, fisherfolk, urban poor, indigenous of Representatives who are nominated in the party-list system shall not be
cultural communities, handicapped, veterans, and overseas workers. The considered resigned.
sectors that lack "well-defined political constituencies" include
The above provision is clear and unambiguous and expresses a single and
professionals, the elderly, women, and the youth.
definite meaning, there is no room for interpretation or construction but only
5. A majority of the members of sectoral parties or organizations that for application. Section 8 clearly prohibits the change of nominees and
represent the "marginalized and underrepresented" must belong to the alteration of the order in the list of nominees’ names after submission of the
"marginalized and underrepresented" sector they represent. list to the COMELEC.

Similarly, a majority of the members of sectoral parties or organizations that COMELEC Section 13 of Resolution No. 7804 (IRR), on the other hand, provides:
lack "well-defined political constituencies" must belong to the sector they
Section 13. Substitution of nominees. A party-list nominee may be
represent.
substituted only when he dies, or his nomination is withdrawn by the
The nominees of sectoral parties or organizations that represent the party, or he becomes incapacitated to continue as such, or he
"marginalized and underrepresented," or that represent those who lack "well- withdraws his acceptance to a nomination. In any of these cases, the
defined political constituencies," either must belong to their respective name of the substitute nominee shall be placed last in the list of nominees.
sectors, or must have a track record of advocacy for their respective sectors.
The nominees of national and regional parties or organizations must be bona-
fide members of such parties or organizations. IV. Candidacy
6. National, regional, and sectoral parties or organizations shall not be
disqualified if some of their nominees are disqualified, provided that they have
at least one nominee who remains qualified. 1. Qualifications/Eligibilities of Candidates
NOTE: In BANAT, the Court disallowed political parties from participating in a. General Guiding Principles in Candidates’ Qualifications
the party list elections. In Atong Paglaum, the Court allowed participation by
political parties provided that they register under the party list system and do i. Qualifications for Public Office are Continuing Requirements -
not field their candidates in the legislative district elections. Frivaldo v. Comelec, 174 SCRA 245 (1989)

iv. Lokin vs. COMELEC, G.R. Nos. 179431-32, 22 June 2010 Qualifications for public office are continuing requirements and must be
possessed not only at the time of appointment or election or assumption of
The Court held that Section 13 of Resolution No. 7804 was invalid and that office but during the officer's entire tenure. Once any of the required
the COMELEC committed grave abuse of discretion in including the same in qualifications is lost, his title may be seasonably challenged.
said Resolution.
The will of the people as expressed through the ballot cannot cure the vice of ineligibility,
The COMELEC issued Resolution No. 7804 as an implementing rules and especially if they mistakenly believed, as in this case, that the candidate was qualified.
regulations in accordance with the provisions of the Omnibus Election Code

M.R.A.D.C. LUMBRE 205


CONSTITUTIONAL LAW REVIEW

Obviously, this rule requires strict application when the deficiency is lack of citizenship. process of election should not be defeated by unwarranted impositions of
If a person seeks to serve in the Republic of the Philippines, he must owe his requirement not otherwise specified in the Constitution.
total loyalty to this country only, abjuring and renouncing all fealty and fidelity
to any other state. XXX XXX XXX This country of ours, for all its difficulties and b. Constitutional and Statutory Qualifications of Candidates
limitations, is like a jealous and possessive mother. Once rejected, it is not quick i. Update: RA 10742, Sec. 10
to welcome back with eager arms its prodigal if repentant children. The returning
renegade must show, by an express and unequivocal act, the renewal of his loyalty and SEC. 10. Qualifications. – An official of the Sangguniang Kabataan, either
love. elective or appointee, must be a citizen of the Philippines, a qualified voter of
the Katipunan ng Kabataan, a resident of the barangay for not less than one
ii. Vice of Ineligibility Cannot be Cured by Election - Hayudini v. (1) year immediately preceding the day of the elections, at least eighteen (18)
Comelec, G.R. No. 207900, April 22, 2014 years but not more than twenty-four (24) years of age on the day of the
The false representation mentioned in these provisions must pertain to a elections, able to read and write Filipino, English, or the local dialect, must not
material fact, not to a mere innocuous mistake. A candidate who falsifies a be related within the second civil degree of consanguinity or affinity to any
material fact cannot run; if he runs and is elected, cannot serve; in both cases, incumbent elected national official or to any incumbent elected regional,
he or she can be prosecuted for violation of the election laws. These facts provincial, city, municipal, or barangay official, in the locality where he or she
pertain to a candidate's qualification for elective office, such as his or her seeks to be elected, and must not have been convicted by final judgment of
citizenship and residence. Similarly, the candidate's status as a registered any crime involving moral turpitude. (R.A. No. 10742 also known as the
voter falls under this classification as it is a legal requirement which must be “Sangguniang Kabataan Reform Act of 2015”)
reflected in the CoC. The reason for this is obvious: the candidate, if he or she c. Citizenship
wins, will work for and represent the local government under which he or she
is running. Even the will of the people, as expressed through the ballot, cannot i. Who are Filipino Citizens - Art. IV, Secs. 1-5
cure the vice of ineligibility, especially if they mistakenly believed, as in the
Sec. 1. The following are citizens of the Philippines:
instant case, that the candidate was qualified.
(1) Those who are citizens of the Philippines at the time of the adoption of this
NOTE: The misrepresentation, however, by a person who falsely states in his
Constitution;
COC that he exercises a profession is cured by his subsequent election to
office. It is not the kind of false misrepresentation that may be ground for the (2) Those whose fathers or mothers are citizens of the Philippines;
denial of due course or cancellation of the COC, as profession is not a
requirement in running for public office. (3) Those born before January 17, 1973, of Filipino mothers, who elect
Philippine citizenship upon reaching the age of majority; and
iii. Qualifications of Candidates Specified in the Constitution Cannot be
Expanded by a Statute - Social Justice Society vs. Dangerous Drug (4) Those who are naturalized in accordance with law.
Board, et al., GR No. 157870, 3 November 2008 Sec. 2. Natural-born citizens are those who are citizens of the Philippines from
Pimentel’s contention is valid. Accordingly, Sec. 36 of RA 9165 is birth without having to perform any act to acquire or perfect their Philippine
unconstitutional. It is basic that if a law or an administrative rule violates any citizenship.
norm of the Constitution, that issuance is null and void and has no effect. The Sec. 3. Philippine citizenship may be lost or reacquired in the manner provided
Constitution is the basic law to which all laws must conform; no act shall be by law.
valid if it conflicts with the Constitution. In the discharge of their defined
functions, the three departments of government have no choice but to yield Sec. 4. Citizens of the Philippines who marry aliens shall retain their citizenship
obedience to the commands of the Constitution. Whatever limits it imposes unless by their act or omission, they are deemed, under the law, to have
must be observed. renounced it.

The provision “[n]o person elected to any public office shall enter upon the Sec. 5. Dual allegiance of citizens is inimical to the national interest and shall
duties of his office until he has undergone mandatory drug test” is not tenable be dealt with by law.
as it enlarges the qualifications. COMELEC cannot, in the guise of enforcing
1. Tecson v. Comelec, G.R. No. 161434. March 3, 2004
and administering election laws or promulgating rules and regulations to
implement Sec. 36, validly impose qualifications on candidates for senator in Natural-born citizens are those who are citizens of the Philippines from
addition to what the Constitution prescribes. If Congress cannot require a birth without having to perform any act to acquire or perfect their
candidate for senator to meet such additional qualification, the COMELEC, to Philippine citizenship. Based on the evidence presented which the
be sure, is also without such power. The right of a citizen in the democratic Supreme Court considers as viable is the fact that the death certificate of

M.R.A.D.C. LUMBRE 206


CONSTITUTIONAL LAW REVIEW

Lorenzo Poe, father of Allan Poe, who in turn was the father of private Certain government offices are exclusive to natural-born citizens of the
respondent Fernando Poe, Jr. indicates that he died on September 11, Philippines:
1954 at the age of 84 years, in San Carlos, Pangasinan. Evidently, in such
death certificate, the residence of Lorenzo Poe was stated to be San 1. President;
Carlos, Pangansinan. 2. Vice President;
In the absence of any evidence to the contrary, it should be sound to 3. Senator;
conclude, or at least to presume, that the place of residence of a person 4. Member of the House of Representatives;
at the time of his death was also his residence before death. Considering
that the allegations of petitioners are not substantiated with proof and 5. Member of the Supreme Court or any lower collegiate court (SC);
since Lorenzo Poe may have been benefited from the “en masse 6. Chairperson and Commissioners of the Civil Service Commission
Filipinization” that the Philippine Bill had effected in 1902, there is no (CSC);
doubt that Allan Poe father of private respondent Fernando Poe, Jr. was a
7. Chairperson and Commissioners of the Commission on Elections
Filipino citizen. And, since the latter was born on August 20, 1939,
(COMELEC);
governed under 1935 Constitution, which constitution considers as citizens
of the Philippines those whose fathers are citizens of the Philippines, 8. Chairperson and Commissioners of the Commission on Audit
Fernando Poe, Jr. was in fact a natural-born citizen of the Philippines (COA);
regardless of whether or not he is legitimate or illegitimate. 9. Ombudsman and his or her deputies;
2. Poe-Llamanzares v. Comelec, G.R. Nos. 221697, March 8, 2016 10. Board of Governors of the Bangko Sentral ng Pilipinas (BSP); and
Yes, herein petitioner is a natural-born citizen. The factual issue is not who 11. Chairperson and Members of the Commission on Human Rights
the parents of petitioner are, as their identities are unknown, but whether (CHR).
such parents are Filipinos. The Court states that there is no provision in
NOTE: If one were to assail the natural-born citizen status of Senator
the Constitution with intent or language permitting discrimination against
Poe, the person must prove that both parents are not citizens of the
foundlings as the 1935, 1973, and 1987 Constitution guarantee the basic
Philippines.
right to equal protection of laws. Domestic laws on adoption also support
the principle that foundlings are Filipinos. These laws do not provide that ii. Renunciation of Foreign Citizenship to Run for Public Office
adoption confers citizenship upon the adoptee. Rather, the adoptee must
be a Filipino in the first place to be adopted. 1. “Twin requirements” now Three Requirements –

It has been argued that the process to determine that the child is a David v. SET, G.R. No. 221538, September 20, 2016
foundling leading to the issuance of a foundling certificate under these Section 3's implications are clear. Natural-born Philippine citizens who,
laws and the issuance of said certificate are acts to acquire or perfect after Republic Act 9225 took effect, are naturalized in foreign countries
Philippine citizenship which make the foundling a naturalized Filipino at "retain," that is, keep, their Philippine citizenship, although the effectivity
best. This is erroneous. Under Article IV, Section 2 "Natural-born citizens of this retention and the ability to exercise the rights and capacities
are those who are citizens of the Philippines from birth without having to attendant to this status are subject to certain solemnities (i.e., oath of
perform any act to acquire or perfect their Philippine citizenship." In the allegiance and other requirements for specific rights and/or acts, as
first place, "having to perform an act" means that the act must be enumerated in Section 5). On the other hand, those who became citizens
personally done by the citizen. In this instance, the determination of of another country before the effectivity of Republic Act No. 9225
foundling status is done not by the child but by the authorities. Secondly, "reacquire" their Philippine citizenship and may exercise attendant rights
the object of the process is the determination of the whereabouts of the and capacities, also upon compliance with certain solemnities. Read in
parents, not the citizenship of the child. Lastly, the process is certainly not conjunction with Section 2's declaration of a policy of immutability, this
analogous to naturalization proceedings to acquire Philippine citizenship, reacquisition is not a mere restoration that leaves a vacuum in the
or the election of such citizenship by one born of an alien father and a intervening period. Rather, this reacquisition works to restore natural-born
Filipino mother under the 1935 Constitution, which is an act to perfect it. status as though it was never lost at all.
3. David v. SET, G.R. No. 221538, September 20, 2016 Taking the Oath of Allegiance effects the retention or reacquisition of
The presumption that all foundlings found in the Philippines are born to at natural-born citizenship. It also facilitates the enjoyment of civil and
least either a Filipino father or a Filipino mother (and are thus natural- political rights, "subject to all attendant liabilities and responsibilities."248
born, unless there is substantial proof otherwise) arises when one reads However, other conditions must be met for the exercise of other faculties:
the Constitution as a whole, so as to "effectuate [its] whole purpose."

M.R.A.D.C. LUMBRE 207


CONSTITUTIONAL LAW REVIEW

Sec. 5. Civil and Political Rights and Liabilities. - Those who retain Third, "mak[ing] a personal and sworn renunciation of any and all foreign
or re-acquire Philippine citizenship under this Act shall enjoy full citizenship before any public officer authorized to administer an oath."
civil and political rights and be subject to all attendant liabilities This, along with satisfying the other qualification requirements under
and responsibilities under existing laws of the Philippines and the relevant laws, makes one eligible for elective public office.
following conditions:
2. Dual Citizen from Birth –
(1) Those intending to exercise their right of suffrage must meet
the requirements under Section 1, Article V of the Constitution, Cordora v. Comelec, G.R. No. 176947, February 19, 2009;
Republic Act No. 9189, otherwise known as "the Overseas We agree with Commissioner Sarmiento’s observation that Tambunting
Absentee Voting Act of 2003" and other existing laws; possesses dual citizenship. Because of the circumstances of his birth, it
(2) Those seeking elective public office in the Philippines shall was no longer necessary for Tambunting to undergo the naturalization
meet the qualifications for holding such public office as required process to acquire American citizenship. The process involved in INS Form
by the Constitution and existing laws and, at the time of the filing I-130 only served to confirm the American citizenship which Tambunting
of the certificate of candidacy, make a personal and sworn acquired at birth. The certification from the Bureau of Immigration which
renunciation of any and all foreign citizenship before any public Cordora presented contained two trips where Tambunting claimed that he
officer authorized to administer an oath; is an American. However, the same certification showed nine other trips
where Tambunting claimed that he is Filipino. Clearly, Tambunting
(3) Those appointed to any public office shall subscribe and possessed dual citizenship prior to the filing of his certificate of candidacy
swear to an oath of allegiance to the Republic of the Philippines before the 2001 elections. The fact that Tambunting had dual citizenship
and its duly constituted authorities prior to their assumption of did not disqualify him from running for public office.
office; Provided, That they renounce their oath of allegiance to
the country where they took that oath; The twin requirements of swearing to an Oath of Allegiance and executing
a Renunciation of Foreign Citizenship in R.A. No. 9225 do not apply to a
(4) Those intending to practice their profession in the Philippines natural-born Filipino who did not subsequently become a naturalized
shall apply with the proper authority for a license or permit to citizen of another country. In Sections 2 and 3 of R.A. No. 9225, the
engage in such practice; and framers were not concerned with dual citizenship per se, but with the
status of naturalized citizens who maintain their allegiance to their
(5) That the right to vote or be elected or appointed to any public countries of origin even after their naturalization. Section 5(3) of R.A. No.
office in the Philippines cannot be exercised by, or extended to, 9225 states that naturalized citizens who reacquire Filipino citizenship and
those who: desire to run for elective public office in the Philippines shall “meet the
a. are candidates for or are occupying any public office in the qualifications for holding such public office as required by the Constitution
country of which they are naturalized citizens; and/or and existing laws and, at the time of filing the certificate of candidacy,
make a personal and sworn renunciation of any and all foreign citizenship
b. are in active service as commissioned or non- before any public officer authorized to administer an oath” aside from the
commissioned officers in the armed forces of the country oath of allegiance prescribed in Section 3 of R.A. No. 9225.
which they are naturalized citizens.
The twin requirements of swearing to an Oath of Allegiance and executing
Thus, natural-born Filipinos who have been naturalized elsewhere and a Renunciation of Foreign Citizenship served as the bases for our recent
wish to run for elective public office must comply with all of the following rulings in Jacot v. Dal and COMELEC, 572 SCRA 295 (2008); Velasco v.
requirements: COMELEC, 575 SCRA 590 (2008) and Japzon v. COMELEC, 576 SCRA 331
(2009), all of which involve natural-born Filipinos who later became
First, taking the oath of allegiance to the Republic. This effects the
naturalized citizens of another country and thereafter ran for elective
retention or reacquisition of one's status as a natural-born Filipino. This
office in the Philippines. In the present case, Tambunting, a natural-born
also enables the enjoyment of full civil and political rights, subject to all
Filipino, did not subsequently become a naturalized citizen of another
attendant liabilities and responsibilities under existing laws, provided the
country. Hence, the twin requirements in R.A. No. 9225 do not apply to
solemnities recited in Section 5 of Republic Act No. 9225 are satisfied.
him.
Second, compliance with Article V, Section 1 of the 1987 Constitution,
Valles v. Comelec, G.R. No. 137000, August 9, 2000
Republic Act No. 9189, otherwise known as the Overseas Absentee Voting
Act of 2003, and other existing laws. This is to facilitate the exercise of In the aforecited case of Mercado vs. Manzano, the Court clarified “dual
the right of suffrage; that is, to allow for voting in elections. citizenship” as used in the Local Government Code and reconciled the
same with Article IV, Section 5 of the 1987 Constitution on dual allegiance.

M.R.A.D.C. LUMBRE 208


CONSTITUTIONAL LAW REVIEW

Recognizing situations in which a Filipino citizen may, without performing authorized public officer prior to or simultaneous to the filing of their
any act, and as an involuntary consequence of the conflicting laws of certificates of candidacy, to qualify as candidates in Philippine
different countries, be also a citizen of another state, the Court explained elections. 36 The rule applies to all those who have re-acquired their Filipino
that dual citizenship as a disqualification must refer to citizens with dual citizenship, like petitioner, without regard as to whether they are still dual
allegiance. The Court succinctly pronounced: “x x x the phrase ‘dual citizens or not. It is a pre-requisite imposed for the exercise of the right
citizenship’ in R.A. No. 7160, x x x 40 (d) and in R.A. No. 7854, x x x 20
to run for public office.
must be understood as referring to ‘dual allegiance.’ Consequently,
persons with mere dual citizenship do not fall under this disqualification.” Stated differently, it is an additional qualification for elective office specific
The fact that the private respondent had dual citizenship did not only to Filipino citizens who re-acquire their citizenship under Section 3 of
automatically disqualify her from running for a public office. Furthermore, R.A. No. 9225. It is the operative act that restores their right to run for
it was ruled that for candidates with dual citizenship, it is enough that they public office. The petitioner's failure to comply therewith in accordance
elect Philippine citizenship upon the filing of their certificate of candidacy, with the exact tenor of the law, rendered ineffectual the Declaration of
to terminate their status as persons with dual citizenship. The filing of a Renunciation of Australian Citizenship she executed on September 18,
certificate of candidacy sufficed to renounce foreign citizenship, effectively 2006. As such, she is yet to regain her political right to seek elective office.
removing any disqualification as a dual citizen. This is so because in the Unless she executes a sworn renunciation of her Australian citizenship,
certificate of candidacy, one declares that he/she is a Filipino citizen and she is ineligible to run for and hold any elective office in the Philippines.
that he/she will support and defend the Constitution of the Philippines and
will maintain true faith and allegiance thereto. Such declaration, which is 4. Recantation of Oath of Renunciation - Maquiling vs. Comelec,
under oath, operates as an effective renunciation of foreign citizenship. G.R. No. 195649, April 16, 2013
Therefore, when the herein private respondent filed her certificate of
candidacy in 1992, such fact alone terminated her Australian citizenship. The Court held that the use of foreign passport after renouncing one’s
foreign citizenship is a positive and voluntary act of representation as to
3. Dual Citizenship Obtained through Naturalization – one’s nationality and citizenship; it does not divest Filipino citizenship
Sobejana-Condon v. Comelec, G.R. No. 198742, August 10, 2012 regained by repatriation but it recants the Oath of Renunciation required
to qualify one to run for an elective position.
An oath is a solemn declaration, accompanied by a swearing to God or a
revered person or thing, that one’s statement is true or that one will be By renouncing his foreign citizenship, he was deemed to be solely a Filipino
citizen, regardless of the effect of such renunciation under the laws of the
bound to a promise. The person making the oath implicitly invites
punishment if the statement is untrue or the promise is broken. The legal foreign country. However, this legal presumption does not operate
permanently and is open to attack when, after renouncing the foreign
effect of an oath is to subject the person to penalties for perjury if the
testimony is false.28 citizenship, the citizen performs positive acts showing his continued
possession of a foreign citizenship.
Indeed, the solemn promise, and the risk of punishment attached to an
oath ensures truthfulness to the prospective public officer’s abandonment Arnado himself subjected the issue of his citizenship to attack when, after
renouncing his foreign citizenship, he continued to use his US passport to
of his adopted state and promise of absolute allegiance and loyalty to the
Republic of the Philippines. travel in and out of the country before filing his certificate of candidacy on
30 November 2009. The pivotal question to determine is whether he was
To hold the oath to be a mere pro forma requirement is to say that it is solely and exclusively a Filipino citizen at the time he filed his certificate
only for ceremonial purposes; it would also accommodate a mere qualified of candidacy, thereby rendering him eligible to run for public office.
or temporary allegiance from government officers when the Constitution
and the legislature clearly demand otherwise. d. Residency
i. Change of Domicile
The fact that petitioner won the elections cannot cure the defect of her
candidacy. Garnering the most number of votes does not validate the 1. Sabili vs. COMELEC, 670 SCRA 664 (2012)
election of a disqualified candidate because the application of the
constitutional and statutory provisions on disqualification is not a matter As in all administrative cases, the quantum of proof necessary in election
of popularity. cases is substantial evidence, or such relevant evidence as a reasonable
mind will accept as adequate to support a conclusion.
In fine, R.A. No. 9225 categorically demands natural-born Filipinos who
re-acquire their citizenship and seek elective office, to execute a personal The COMELEC correctly ruled that the Voter Certification issued by the
and sworn renunciation of any and all foreign citizenships before an COMELEC Election Officer, Atty. Juan B. Aguila, Jr., was not conclusive

M.R.A.D.C. LUMBRE 209


CONSTITUTIONAL LAW REVIEW

proof that petitioner had been a resident of Lipa City since April 2007. It Considering all of the foregoing discussion, it is clear that while separately,
noted that Aguila is not the competent public officer to certify the veracity each evidence presented by petitioner might fail to convincingly show the
of this claim, particularly because petitioners COMELEC registration was fact of his residence at Pinagtong-ulan since 2007, collectively, these
approved only in October 2009. pieces of evidence tend to sufficiently establish the said fact.
It is true that property ownership is not among the qualifications ii. The Law Treats Citizenship Independently of Domicile; However:
required of candidates for local election. Rather, it is a candidate’s
residence in a locality through actual residence in whatever capacity. 1. Caballero vs. Comelec, G.R. No. 209835, September 22, 2015

We have gone so far as to rule that there is nothing wrong in an individual RA No. 9225 treats citizenship independently of residence. This is only
changing residences so he could run for an elective post, for as long as he logical and consistent with the general intent of the law to allow for dual
is able to prove with reasonable certainty that he has effected a change citizenship. Since a natural-born Filipino may hold, at the same time, both
of residence for election law purposes for the period required by law. Philippine and foreign citizenships, he may establish residence either in
the Philippines or in the foreign country of which he is also a citizen.
The law does not require a person to be in his home twenty-four (24) However, when a natural-born Filipino with dual citizenship seeks for an
hours a day, seven (7) days a week, to fulfill the residency requirement. elective public office, residency in the Philippines becomes material.
Income Tax Returns and Revenue Official Receipts for 2007 and 2008 Hence, petitioner's retention of his Philippine citizenship under RA No.
shows that petitioner invariably declares his residence to be Pinagtong- 9225 did not automatically make him regain his residence in Uyugan,
ulan, Lipa City, rather than San Juan, Batangas.Hence, while petitioner Batanes. He must still prove that after becoming a Philippine citizen on
may be submitting his income tax return in the same RDO, the declaration September 13, 2012, he had reestablished Uyugan, Batanes as his new
therein is unmistakable. Petitioner considers Lipa City to be his domicile. domicile of choice which is reckoned from the time he made it as such.
Assuming that the barangay captain’s certification only pertains to The COMELEC found that petitioner failed to present competent evidence
petitioner’s bodily presence in Pinagtong-ulan, still, the COMELEC cannot to prove that he was able to reestablish his residence in Uyugan within a
deny the strength of this evidence in establishing petitioner’s bodily period of one year immediately preceding the May 13, 2013 elections. It
presence in Pinagtong-ulan since 2007. found that it was only after reacquiring his Filipino citizenship by virtue of
RA No. 9225 on September 13, 2012 that petitioner can rightfully claim
The COMELEC ruled that the Affidavit was self-serving for having been that he re-established his domicile in Uyugan, Batanes, if such was
executed by petitioner’s common-law wife. accompanied by physical presence thereat, coupled with an actual intent
The COMELEC brushed this Affidavit aside as one that merely narrates the to reestablish his domicile there. However, the period from September 13,
circumstances surrounding the sale of the property and mentions in 2012 to May 12, 2013 was even less than the one year residency required
passing that Sabili and Palomares lived in Pinagtong-ulan since April 2007 by law.
up to the present. NOTE:
Macasaet also swore that the couple had actually resided in the house Q: What are the kinds of domicile?
located in Pinagtong-ulan since April 2007, and that she knew this because
her own house was very near the couples own. Macasaets Affidavit is a A: 1. Domicile of Origin or By Birth – the domicile of a person’s parents at
positive assertion of petitioner’s actual physical presence in Brgy. the time of his birth, which is not easily lost, and it continues until, upon
Pinagtong-ulan, Lipa City. reaching majority age, he abandons it and acquires a new domicile.

Nevertheless, coupled with the fact that petitioner had twice been elected 2. Domicile of Choice – that which a person chooses upon reaching age of
as Provincial Board Member representing the Fourth District of Batangas, majority, abandoning the domicile of origin;
which encompasses Lipa City, petitioners involvement in the religious life
of the community, as attested to by the certificate of appreciation issued 3. Domicile by operation of law – that which the law attributes to a person
to him by the Pinagtong-ulan parish for his material and financial support because of his disability to make a choice, such as when he is a minor or
as President of the Barangay Fiesta Committee in 2009, as well as his suffers from mental or physical disability, in which case he follows, as a
assumption of a leadership role in the socio-civic sphere of the locality as rule, the domicile of his father.
a member of the advisory body of the Pinagtong-ulan, San Jose/Lipa City Change of Domicile:
Chapter of the Guardians Brotherhood Inc. , manifests a significant level
of knowledge of and sensitivity to the needs of the said community. 1. An actual removal or an actual change of domicile;

M.R.A.D.C. LUMBRE 210


CONSTITUTIONAL LAW REVIEW

2. A bona fide intention of abandoning the former place of residence and no such official shall serve for more than three consecutive terms.
establishing a new one; and Voluntary renunciation of the office for any length of time shall not be
considered as an interruption in the continuity of his service for the
3. Acts which correspond with such purpose. full term for which he was elected.
e. Age which is restated in Section 43 of the Local Government Code, thus:
i. Garvida v. Sales, G.R. No. 124893, April 18, 1997 Section 43. Term of Office. - (a) x x x
A closer look at the Local Government Code will reveal a distinction between (b) No local elective official shall serve for more than three (3)
the maximum age of a member in the Katipunan ng Kabataan and the consecutive terms in the same position. Voluntary renunciation of the
maximum age of an elective SK official. Section 424 of the Code sets a office for any length of time shall not be considered as an interruption
member's maximum age at 21 years only. There is no further provision as to in the continuity of service for the full term for which the elective
when the member shall have turned 21 years of age. On the other hand, official concerned was elected.
Section 428 provides that the maximum age of an elective SK official is 21
years old "on the day of his election." The addition of the phrase "on the day Section 74 of the OEC provides that the certificate of candidacy shall state
of his election" is an additional qualification. The member may be more than that the person filing it is announcing his candidacy for the office stated
21 years of age on Election Day or on the day he registers as member of the therein and that he is eligible for said office. The word "eligible" in Section
Katipunan ng Kabataan. The elective official, however, must not be more than 74 means having the right to run for elective public office, that is, having
21 years old on the day of election. all the qualifications and none of the ineligibilities to run for the public
office. 20 And We had held 21 that a violation of the three-term limit rule
The provision that an elective official of the SK should not be more than 21 is an ineligibility which is a proper ground for a petition to deny due course
years of age on the day of his election is very clear. The Local Government to or to cancel a COC under Section 78 of the Omnibus Election Code.
Code speaks of years, not months nor days. When the law speaks of years, it
is understood that years are of 365 days each. One born on the first day of the To reiterate, a violation of the three-term limit rule is not included among
year is consequently deemed to be one year old on the 365th day after his the grounds for disqualification, but a ground for a petition to deny due
birth -- the last day of the year. In computing years, the first year is reached course to or cancel certificate of candidacy; thus, it is Rule 23 of COMELEC
after completing the first 365 days. After the first 365th day, the first day of Resolution No. 9523 which is applicable, and We quote:
the second 365-day cycle begins. On the 365th day of the second cycle, the
person turns two years old. This cycle goes on and on in a lifetime. A person Rule 23 - Petition to Deny Due Course to or Cancel Certificates of
turns 21 years old on the 365th day of his 21st 365-day cycle. This means on Candidacy
his 21st birthday, he has completed the entire span of 21 365-day cycles. After Section 1. Ground for Denial or Cancellation of Certificate of
this birthday, the 365-day cycle for his 22nd year begins. The day after the Candidacy. - A verified Petition to Deny Due Course to or Cancel a
365th day is the first day of the next 365-day cycle and he turns 22 years old Certificate of Candidacy for any elective office may be filed by any
on the 365th day. registered voter or a duly registered political party, organization, or
The phrase "not more than 21 years of age" means not over 21 years, not coalition of political parties on the exclusive ground that any material
beyond 21 years. It means 21 365-day cycles. It does not mean 21 years and representation contained therein as required by law is false.
one or some days or a fraction of a year because that would be more than 21 Section 2. Period to File Petition. - The Petition must be filed within
365-day cycles. "Not more than 21 years old" is not equivalent to "less than five (5) days from the last day for filing of certificate of candidacy;
22 years old," contrary to petitioner's claims. The law does not state that the but not later than twenty five (25) days from the time of filing of the
candidate be less than 22 years on Election Day. certificate of candidacy subject of the Petition. In case of a substitute
f. Other Eligibilities of Candidates candidate, the Petition must be filed within five (5) days from the time
the substitute candidate filed his certificate of candidacy.
i. Non-suffering from Any Term Limitation
We, likewise, find no grave abuse of discretion committed by the COMELEC
1. Albania v. Comelec, G.R. No. 226792, June 7, 2017 En Banc when it found that the petition to deny due course to or cancel a
COC will not also prosper as there was no violation of the three-term limit
The three-term limit rule is embodied in Section 8 of Article X of the rule.
Constitution, to wit:
We held that two conditions must concur for the application of the
Section 8. The term of office of elective local officials, except barangay disqualification of a candidate based on violation of the three-term limit
officials, which shall be determined by law, shall be three years and rule, which are: (1) that the official concerned has been elected for three

M.R.A.D.C. LUMBRE 211


CONSTITUTIONAL LAW REVIEW

consecutive terms in the same local government post, and (2) that he has Re: Recall election
fully served three consecutive terms.
2. An elective official, who has served for three consecutive terms and
In this case, while respondent ran as Governor of Camarines Norte in the who did not seek the elective position for what could be his fourth
2007 elections, he did not win as such. It was only after he filed la petition term, but later won in a recall election, had an interruption in the
for correction of manifest error that he was proclaimed as the duly-elected continuity of the official’s service. For, he had become in the interim,
Governor. He assumed the post and served the unexpired term of his i.e., from the end of the 3rd term up to the recall election, a private
opponent from March 22, 2010 until June 30, 2010. Consequently, he did citizen (Adormeo vs Comelec, 2002 and Socrates vs Comelec,
not hold the office for the full term of three years to which he was 2002).
supposedly entitled to. Thus, such period of time that respondent served
as Governor did not constitute a complete and full service of his term. The Re: Conversion of a Municipality into a City
period when he was out of office involuntarily interrupted the continuity of
his service as Governor.26 As he had not fully served the 2007-2010 term, 3. The abolition of an elective local office due to the conversion of a
and had not been elected for three consecutive terms as Governor, there municipality to a city does not, by itself, work to interrupt the
was no violation of the three-term limit rule when he ran again in the 2016 incumbent official’s continuity of service (Latasa vs Comelec,
elections. 2003).
The word "term" in a legal sense means a fixed and definite period of time
which the law describes that an officer may hold an office. According to Re: Period of Preventive Suspension
Mechem, the term of office is the period during which an office may be
held. Upon expiration of the officer's term, unless he is authorized by law 4. Preventive suspension is not a term-interrupting event as the elective
to holdover, his rights, duties and authority as a public officer must ipso officer’s continued stay and entitlement to the office remain
facto cease. In the law of public officers, the most and natural frequent unaffected during the period of suspension, although he is barred
method by which a public officer ceases to be such is by the expiration of from exercising the functions of his office during this period
the terms for which he was elected or appointed. (Aldovino, Jr.vs Comelec, 2009)

2. Summary of prevailing jurisprudence on issues affecting Re: Election Protest


consecutiveness of terms and involuntary interruption - Abundo v.
Commission on Elections, G.R. No. 201716, 8 January 2013 5. When a candidate is proclaimed as winner for an elective position and
assumes office, his term is interrupted when he loses in an election
To constitute a disqualification to run for an elective local office pursuant
to the constitutional and statutory provisions, the following requisites protest and is ousted from office, thus disenabling him from serving
must concur: (1) that the official concerned has been elected for three what would otherwise be the unexpired portion of his term of office
consecutive terms; and (2) that he has fully served three had the protest been dismissed (Lonzanida vs Comelec, 1999 and
consecutive terms. Dizon vs Comelec 2009). The break or interruption need not be for
a full term of three years or for the major part of the 3-year term; an
The following are the prevailing jurisprudence on issues affecting interruption for any length of time, provided the cause is involuntary,
consecutiveness of terms and/or involuntary interruption: is sufficient to break the continuity of service (Socrates, citing
Re: Assumption of Office by Operation of Law Lonzanida).

1. When a permanent vacancy occurs in an elective position and the 6. When an official is defeated in an election protest and said decision
official merely assumed the position pursuant to the rules on becomes final after said official had served the full term for said office,
succession under the LGC, then his service for the unexpired portion then his loss in the election contest does not constitute an interruption
of the term of the replaced official cannot be treated as one full term since he has managed to serve the term from start to finish. His full
as contemplated under the subject constitutional and statutory service, despite the defeat, should be counted in the application of
provision that service cannot be counted in the application of any term term limits because the nullification of his proclamation came after
limit (Borja, Jr. vs Comelec, 1998). If the official runs again for the the expiration of the term (Ong vs Alegre, 2006 and Rivera III vs
same position he held prior to his assumption of the higher office, Comelec, 2007).
then his succession to said position is by operation of law and is
considered an involuntary severance or interruption (Montebon vs
Comelec, 2008).

M.R.A.D.C. LUMBRE 212


CONSTITUTIONAL LAW REVIEW

The two-year period during which his opponent, Torres, was serving as office. The Court emphasized that pending the favorable resolution of
mayor should be considered as an interruption, which effectively removed Abundo’s election protest, he was relegated to being an ordinary
Abundo’s case from the ambit of the three-term limit rule. constituent and private citizen since his opponent, as presumptive victor
in the 2004 elections, was occupying the mayoralty seat. While awaiting
The consecutiveness of what otherwise would have been Abundo’s three the pendency of the election protest, Abundo ceased from exercising
successive, continuous mayorship was effectively broken during the 2004- power or authority over the constituents of Viga and cannot be said to
2007 term when he was initially deprived of title to, and was veritably have retained title to the mayoralty office as he was at that time not the
disallowed to serve and occupy, an office to which he, after due duly proclaimed winner. It stressed that Abundo’s case differs from other
proceedings, was eventually declared to have been the rightful choice of cases involving the “effects of an election protest because while Abundo
the electorate. was the winning candidate, he was the one deprived of his right and
opportunity to serve his constituents.”
To constitute a disqualification to run for an elective local office pursuant
to the aforequoted constitutional and statutory provisions, the following ii. Non-Suffering from Any Accessory Penalty of Disqualification
requisites must concur: (1) that the official concerned has been elected
for three consecutive terms in the same local government post; and (2) 1. Jalosjos vs. Comelec, G.R. No. 205033, June 18, 2013
that he has fully served three consecutive terms. It is petitioner’s submission that Article 30 of the RPC was partially amended
by Section 40(a) of the LGC and thus, claims that his perpetual absolute
As is clearly provided in Sec. 8, Art. X of the Constitution as well as in Sec.
disqualification had already been removed. The argument is untenable.
43(b) of the LGC, voluntary renunciation of the office by the incumbent
elective local official for any length of time shall NOT, in determining The Court observes that the conflict between these provisions of law may be
service for three consecutive terms, be considered an interruption in the properly reconciled. In particular, while Section 40(a) of the LGC allows a prior
continuity of service for the full term for which the elective official convict to run for local elective office after the lapse of two (2) years from the
concerned was elected. In Aldovino, Jr., however, the Court stated the time he serves his sentence, the said provision should not be deemed to cover
observation that the law “does not textually state that voluntary cases wherein the law imposes a penalty, either as principal or accessory,
renunciation is the only actual interruption of service that does not affect which has the effect of disqualifying the convict to run for elective office. An
example of this would be Article 41 of the RPC, which imposes the penalty of
‘continuity of service for a full term’ for purposes of the three-term limit
perpetual absolute disqualification as an accessory to the principal penalties of
rule.”
reclusion perpetua and reclusion temporal:
As stressed in Socrates v. Commission on Elections, the principle behind ART. 41. Reclusion perpetua and reclusion temporal – Their accessory
the three term limit rule covers only consecutive terms and that what the penalties. - The penalties of reclusion perpetua and reclusion temporal
Constitution prohibits is a consecutive fourth term. There has, in fine, to shall carry with them that of civil interdiction for life or during the period
be a break or interruption in the successive terms of the official after his of the sentence as the case may be, and that of perpetual absolute
or her third term. Of course, the basic law is unequivocal that a “voluntary disqualification which the offender shall suffer even though pardoned as
renunciation of the office for any length of time shall NOT be considered to the principal penalty, unless the same shall have been expressly
an interruption in the continuity of service for the full term for which the remitted in the pardon.
elective official concerned was elected.” This qualification was made as a
In this relation, Article 30 of the RPC, as earlier cited, provides that the penalty
deterrent against an elective local official intending to skirt the three-term of perpetual absolute disqualification has the effect of depriving the convicted
limit rule by merely resigning before his or her third term ends. This is a felon of the privilege to run for elective office. To note, this penalty, as well as
voluntary interruption as distinguished from involuntary interruption which other penalties of similar import, is based on the presumptive rule that one
may be brought about by certain events or causes. who is rendered infamous by conviction of a felony, or other base offense
indicative of moral turpitude, is unfit to hold public office, as the same partakes
The Court further ruled that the COMELEC erred in applying Aldovino, Jr. of a privilege which the State grants only to such classes of persons which are
v. Commission on Elections, which held that “service of the unexpired most likely to exercise it for the common good.
portion of a term by a protestant who is declared winner in an election
protest is considered as service for one full term within the contemplation Pertinently, it is observed that the import of Article 41 in relation to Article 30
of the three-term limit rule” as the doctrine refers to a situation where the of the RPC is more direct and specific in nature – insofar as it deprives the
elected official is under preventive suspension and is only temporarily candidate to run for elective office due to his conviction – as compared to
unable to discharge his functions yet is still entitled to the office as Section 40(a) of the LGC which broadly speaks of offenses involving moral
turpitude and those punishable by one (1) year or more of imprisonment
compared to the situation of Abundo where he did not have title to the

M.R.A.D.C. LUMBRE 213


CONSTITUTIONAL LAW REVIEW

without any consideration of certain disqualifying effects to one’s right to language or dialect. (RA 7160 Local Government Code of the Philippines,
suffrage. Accordingly, Section 40(a) of the LGC should be considered as a law Sec. 39)
of general application and therefore, must yield to the more definitive RPC
provisions in line with the principle of lex specialis derogat generali – general Congress may not add to qualifications for elective officials provided in the
legislation must give way to special legislation on the same subject, and Constitution.
generally is so interpreted as to embrace only cases in which the special a. Grounds for Disqualification (1994, 1999, 2010 Bar)
provisions are not applicable. In other words, where two statutes are of equal
theoretical application to a particular case, the one specially designed therefor 1. Declared as incompetent or insane by competent authority
should prevail.
2. Convicted by final judgment for subversion, insurrection, rebellion, or
Further, it is well to note that the use of the word "perpetual" in the any offense for which he has been sentenced to a penalty of 18 months
aforementioned accessory penalty connotes a lifetime restriction and in this imprisonment
respect, does not depend on the length of the prison term which is imposed as
3. Convicted by final judgment for a crime involving moral turpitude
its principal penalty.
4. Election offenses under Sec. 261 of the OEC
The accessory penalty of perpetual special disqualification takes effect
immediately once the judgment of conviction becomes final. The effectivity of 5. Committing acts of terrorism to enhance candidacy
this accessory penalty does not depend on the duration of the principal penalty,
or on whether the convict serves his jail sentence or not. The last sentence of 6. Spending in his election campaign an amount in excess of that allowed
Article 32 states that "the offender shall not be permitted to hold any public 7. Soliciting, receiving, making prohibited contributions
office during the period of his [perpetual special] disqualification." Once the
judgment of conviction becomes final, it is immediately executory. Any public 8. Not possessing qualifications and possessing disqualifications under the
office that the convict may be holding at the time of his conviction becomes Local Government Code
vacant upon finality of the judgment, and the convict becomes ineligible to run
9. Sentenced by final judgment for an offense involving moral turpitude
for any elective public office perpetually.
or for an offense punishable by one year or more of imprisonment within
2. Disqualification of Candidates (included Qualifications) – Pres, VP, two years after serving sentence
and Legislative Qualification already discussed during midterms.
10. Removed from office as a result of an administrative case
Local Level
11. Convicted by final judgment for violating the oath of allegiance to the
A. For District Representatives Republic

1. Natual-born citizen; 12. Dual citizenship (more specifically, dual allegiance)

2. Registered voter in the district in which he shall be elected; 13. Fugitives from justice in criminal or non-political cases here or abroad

3. Resident of the same district for a period not less than one (1) year 14. Permanent residents in a foreign country or those who have acquired
immediately preceding the day of the election; the right to reside abroad and continue to avail of the same right

4. Able to read and write; and 5. At least 25 years old on the day of the 15. Insane or feeble-minded
election. (1987 Constitution, Art. VI, Sec. 6)
16. Nuisance candidate
B. For Governor, Vice Governor, Mayor, Vice-Mayor, Punong Barangay
17. Violation of Sec. 73 OEC with regard to CoC 18. Violation of Sec. 78:
and Sangguniang Members (1994, 2005 BAR)
material misrepresentation in the COC.
1. Citizen of the Philippines;
a. Grounds for Disqualification
2. Registered voter in the barangay, municipality, city, or province or, in
i. Candidate Subject to Disqualification is Eligible but
the case of a member of the sangguniang panlalawigan, sangguniang
Committed Infraction or Placed in a Disqualifying Situation –
panlungsod, or sangguniang bayan, the district where he intends to be
Tagolino v. HRET, G.R. No. 202202, March 19, 2013
elected;
The Omnibus Election Code (OEC) provides for certain remedies to
3. Resident therein for at least one (1) year immediately preceding the
assail a candidate’s bid for public office. Among these which obtain
day of the election; 4. Able to read and write Filipino or any other local
particular significance to this case are: (1) a petition for

M.R.A.D.C. LUMBRE 214


CONSTITUTIONAL LAW REVIEW

disqualification under Section 68; and (2) a petition to deny due conviction by final judgment of an offense involving moral turpitude,
course to and/or cancel a certificate of candidacy under Section 78. inter alia, to run for and hold any public office, whether local or
The distinctions between the two are well-perceived. national position.
Primarily, a disqualification case under Section 68 of the OEC is hinged The pardoning power of the President cannot be limited by legislative
on either: (a) a candidate’s possession of a permanent resident status action. Articles 36 and 41 of the Revised Penal Code cannot serve to
in a foreign country; or (b) his or her commission of certain acts of abridge or diminish the exclusive power and prerogative of the
disqualification. Anent the latter, the prohibited acts under Section 68 President to pardon persons convicted of violating penal statutes.
refer to election offenses under the OEC, and not to violations of other Where the words of a statute are clear, plain, and free from
penal laws. In particular, these are: (1) giving money or other ambiguity, it must be given its literal meaning and applied without
material consideration to influence, induce or corrupt the voters or attempted interpretation.
public officials performing electoral functions; (2) committing acts of
terrorism to enhance one’s candidacy; (3) spending in one’s election A close scrutiny of the text of the pardon extended to former President
campaign an amount in excess of that allowed by the OEC; (4) Estrada shows that both the principal penalty of reclusion perpetua
soliciting, receiving or making any contribution prohibited under and its accessory penalties are included in the pardon. The first
Sections 89, 95, 96, 97 and 104 of the OEC; and (5) violating Sections sentence refers to the executive clemency extended to former
80, 83,85,86 and 261, paragraphs d,e,k, v, and cc, sub-paragraph 6 President Estrada who was convicted by the Sandiganbayan of
of the OEC. Accordingly, the same provision (Section 68) states that plunder and imposed a penalty of reclusion perpetua. The latter is the
any candidate who, in an action or protest in which he or she is a principal penalty pardoned which relieved him of imprisonment. The
party, is declared by final decision of a competent court guilty of, or sentence that followed, which states that “[h]e is hereby restored to
found by the COMELEC to have committed any of the foregoing acts his civil and political rights,” expressly remitted the accessory
shall be disqualified from continuing as a candidate for public office, penalties that attached to the principal penalty of reclusion perpetua.
or disallowed from holding the same, if he or she had already been Hence, even if we apply Articles 36 and 41 of the Revised Penal Code,
elected. it is indubitable from the text of the pardon that the accessory
penalties of civil interdiction and perpetual absolute disqualification
It must be stressed that one who is disqualified under Section 68 is were expressly remitted together with the principal penalty of
still technically considered to have been a candidate, albeit proscribed reclusion perpetua.
to continue as such only because of supervening infractions which do
not, however, deny his or her statutory eligibility. In other words, The right to seek public elective office is unequivocally considered as
while the candidate’s compliance with the eligibility requirements as a political right. Hence, the pardon granted to former President
prescribed by law, such as age, residency, and citizenship, is not in Estrada admits no other interpretation other than to mean that, upon
question, he or she is, however, ordered to discontinue such acceptance of the pardon granted to him, he regained his FULL civil
candidacy as a form of penal sanction brought by the commission of and political rights – including the right to seek elective office.
the above-mentioned election offenses. iii. Effect of Penal Provision to the LGC Provision Which Allows
While a person who is disqualified under Section 68 is merely Prior Convict to Run After Two (2) Years – Jalosjos vs.
prohibited to continue as a candidate, a person who certificate is Comelec, G.R. No. 205033, June 18, 2013
cancelled or denied due course under Section 78 is not treated as a Well-established is the rule that every new statute should be
candidate at all, as if he/she never filed a CoC. construed in connection with those already existing in relation to the
ii. Restoration of Pardonee’s Right to Run for Public Office – same subject matter and all should be made to harmonize and stand
Risos-Vidal vs. Comelec, G. R. No. 206666, 21 January 2015 together, if they can be done by any fair and reasonable
interpretation.
Yes. Former President Estrada was granted an absolute pardon that
fully restored all his civil and political rights. The disqualification of SEC. 40. Disqualifications. The following persons are disqualified
former President Estrada under Section 40 of the LGC in relation to from running for any elective local position:
Section 12 of the OEC was removed by his acceptance of the absolute (a) Those sentenced by final judgment for an offense involving
pardon granted to him. While it may be apparent that the proscription moral turpitude or for an offense punishable by one (1) year or
in Section 40(a) of the LGC is worded in absolute terms, Section 12 more of imprisonment, within two (2) years after serving
of the OEC provides a legal escape from the prohibition – a plenary sentence
pardon or amnesty. In other words, the latter provision allows any
person who has been granted plenary pardon or amnesty after

M.R.A.D.C. LUMBRE 215


CONSTITUTIONAL LAW REVIEW

ART. 30. Effects of the penalties of perpetual or temporary (a) given money or other material consideration to influence,
absolute disqualification. - The penalties of perpetual or induce or corrupt the voters or public officials performing
temporary absolute disqualification for public office shall produce electoral functions; (b) committed acts of terrorism to enhance
the following effects: his candidacy; (c) spent in his election campaign an amount in
excess of that allowed by this Code; (d) solicited, received or
The deprivation of the right to vote in any election for any popular made any contribution prohibited under Sections 89, 95, 96, 97
office or to be elected to such office. and 104; or (e) violated any of Sections 80, 83, 85, 86 and 261,
In particular, while Section 40(a) of the LGC allows a prior convict to paragraphs d, e, k, v, and cc, subparagraph 6, shall be
run for local elective office after the... lapse of two (2) years from the disqualified from continuing as a candidate, or if he has been
time he serves his sentence, the said provision should not be deemed elected, from holding the office. Any person who is a permanent
to cover cases wherein the law imposes a penalty, either as principal resident of or an immigrant to a foreign country shall not be
or accessory, which has the effect of disqualifying the convict to run qualified to run for any, elective office under this Code, unless
for elective office. In this relation, Article 30 of the RPC, as earlier said person has waived his status as a permanent resident or
cited, provides that the penalty of perpetual absolute disqualification immigrant of a foreign country in accordance with the residence
has the effect of depriving the convicted felon of the privilege to run requirement provided for in the election laws.
for elective office. xxxx
This penalty, as well as other penalties of similar import, is based on SECTION 40. Disqualifications - The following persons are
the presumptive rule that one who is rendered infamous by conviction disqualified from running for any elective local position:
of a felony, or other base offense indicative of moral turpitude, is unfit
to hold public office, as the same partakes of a privilege which the (a) Those sentence by final judgment for an offense involving moral
State grants only to such classes of persons which are most likely to turpitude or for an offense punishable by one (1) year or more of
exercise it for the common good. imprisonment, within two (2) years after serving sentence;

iv. Administratively ‘Removed from Office’ Defined - Albania (b) Those removed from office as a result of an administrative
vs. Comelec, G.R. No. 226792, June 7, 2017 case;

The grounds for disqualification of a candidate are found under (c) Those convicted by final judgment for violating the oath of
Sections 12 and 68 of Batas Pambansa Blg. 881, as amended, allegiance to the Republic;
otherwise known as the Omnibus Election Code of the Philippines, as
well as Section 40 of the Local Government Code, which respectively (d) Those with dual citizenship;
provide: (e) Fugitive from justice in criminal or nonpolitical cases here or
SEC. 12. Disqualifications. Any person who has been declared by abroad;
competent authority insane or incompetent, or has been (f) Permanent residents in a foreign country or those who have
sentenced by final judgment for subversion, insurrection, acquired the right to reside abroad and continue to avail of the same
rebellion, or for any offense for which he has been sentenced to right after the effectivity of this Code; and
a penalty of more than eighteen months or for a crime involving
moral turpitude, shall be disqualified to be a candidate and to (g) The insane or feeble-minded.
hold any office, unless he has been given plenary pardon or
v. Doctrine of Condonation - Morales vs. CA, G.R. Nos. 217126-
granted amnesty.
27, November 10, 2015
The disqualifications to be a candidate herein provided shall be
Generally speaking, condonation has been defined as "[a] victim's
deemed removed upon the declaration by competent authority that
express or implied forgiveness of an offense, [especially] by treating
said insanity or incompetence had been removed or after the
the offender as if there had been no offense."
expiration of a period of five years from his service or sentence, unless
within the same period he again becomes disqualified. The condonation doctrine - which connotes this same sense of
complete extinguishment of liability is not based on statutory law. It
xxxx
is a jurisprudential creation that originated from the 1959 case of
SEC. 68. Disqualifications. Any candidate who, in an action or Pascual v. Hon. Provincial Board of Nueva Ecija, (Pascual), which was
protest in which he is a party is declared by final decision of a therefore decided under the 1935 Constitution.
competent court guilty of, or found by the Commission of having

M.R.A.D.C. LUMBRE 216


CONSTITUTIONAL LAW REVIEW

As there was no legal precedent on the issue at that time, the Court, Hence, owing to either their variance or inapplicability, none of these
in Pascual, resorted to American authorities and "found that cases on cases can be used as basis for the continued adoption of the
the matter are conflicting due in part, probably, to differences in condonation doctrine under our existing laws.
statutes and constitutional provisions, and also, in part, to a
divergence of views with respect to the question of whether the At best, Section 66 (b) of the LGC prohibits the enforcement of the
subsequent election or appointment condones the prior misconduct." penalty of suspension beyond the unexpired portion of the elective
local official's prior term, and likewise allows said official to still run
Article XIII. Section 1 of the 1987 Constitution provides that: for re-election This treatment is similar to People ex rel Bagshaw v.
Public office is a public trust. Public officers and employees must at Thompson and Montgomery v. Novell both cited in Pascual, wherein
all times be accountable to the people, serve them it was ruled that an officer cannot be suspended for a misconduct
with utmost responsibility, integrity, loyalty, and efficiency committed during a prior term. However, as previously stated,
and act with patriotism and justice, and lead modest lives. nothing in Section 66 (b) states that the elective local official's
administrative liability is extinguished by the fact of re-election. Thus,
Related to this provision is Section 40 (b) of the LGC which states at all events, no legal provision actually supports the theory
that those removed from office as a result of an administrative that the liability is condoned.
case shall be disqualified from running for any elective local
position. b. Petition for Disqualification (Sec. 68) vs. Petition to Deny Due
Course to or Cancel COC For False Material Representation in the
Reading the 1987 Constitution together with the above-cited legal COC (Sec. 78)
provisions, the Court concluded that the doctrine of
condonation is actually bereft of legal bases. i. Tagolino v. HRET, G.R. No. 202202, March 19, 2013

The Omnibus Election Code (OEC) provides for certain remedies to


To begin with, the concept of public office is a public trust and the assail a candidate’s bid for public office. Among these which obtain
corollary requirement of accountability to the people at all particular significance to this case are: (1) a petition for
times, as mandated under the 1987 Constitution, is plainly disqualification under Section 68; and (2) a petition to deny due
inconsistent with the idea that an elective local official's course to and/or cancel a certificate of candidacy under Section 78.
administrative liability for a misconduct committed during a prior term The distinctions between the two are well-perceived.
can be wiped off by the fact that he was elected to a second term of
office, or even another elective post. Election is not a mode of Primarily, a disqualification case under Section 68 of the OEC is hinged
condoning an administrative offense, and there is simply no on either: (a) a candidate’s possession of a permanent resident status
constitutional or statutory basis in our jurisdiction to support the in a foreign country; or (b) his or her commission of certain acts of
notion that an official elected for a different term is fully absolved of disqualification. Anent the latter, the prohibited acts under Section 68
any administrative liability arising from an offense done during a prior refer to election offenses under the OEC, and not to violations of other
term. In this jurisdiction, liability arising from administrative penal laws. In particular, these are: (1) giving money or other
offenses may be condoned bv the President in light of Section material consideration to influence, induce or corrupt the voters or
19, Article VII of the 1987 Constitution which was interpreted public officials performing electoral functions; (2) committing acts of
in Llamas v. Orbos to apply to administrative offenses. terrorism to enhance one’s candidacy; (3) spending in one’s election
campaign an amount in excess of that allowed by the OEC; (4)
Also, it cannot be inferred from Section 60 of the LGC that the grounds soliciting, receiving or making any contribution prohibited under
for discipline enumerated therein cannot anymore be invoked against Sections 89, 95, 96, 97 and 104 of the OEC; and (5) violating Sections
an elective local official to hold him administratively liable once he is 80, 83, 85, 86 and 261, paragraphs d, e, k, v, and cc, sub-paragraph
re-elected to office. In fact, Section 40 (b) of the LGC precludes 6 of the OEC. Accordingly, the same provision (Section 68) states that
condonation since in the first place, an elective local official who is any candidate who, in an action or protest in which he or she is a
meted with the penalty of removal could not be re-elected to an party, is declared by final decision of a competent court guilty of, or
elective local position due to a direct disqualification from running for found by the COMELEC to have committed any of the foregoing acts
such post. In similar regard, Section 52 (a) of the RRACCS imposes a shall be disqualified from continuing as a candidate for public office,
penalty of perpetual disqualification from holding public office as an or disallowed from holding the same, if he or she had already been
accessory to the penalty of dismissal from service. elected.
It must be stressed that one who is disqualified under Section 68 is
still technically considered to have been a candidate, albeit proscribed

M.R.A.D.C. LUMBRE 217


CONSTITUTIONAL LAW REVIEW

to continue as such only because of supervening infractions which do candidate at all. The reason being is that a cancelled CoC is considered
not, however, deny his or her statutory eligibility. In other words, void ab initio and thus, cannot give rise to a valid candidacy and
while the candidate’s compliance with the eligibility requirements as necessarily, to valid votes.41 In Talaga v. COMELEC42 (Talaga), the
prescribed by law, such as age, residency, and citizenship, is not in Court ruled that:
question, he or she is, however, ordered to discontinue such
candidacy as a form of penal sanction brought by the commission of x x x x While a person who is disqualified under Section 68 is merely
the above-mentioned election offenses. prohibited to continue as a candidate, a person who certificate is
cancelled or denied due course under Section 78 is not treated as a
On the other hand, a denial of due course to and/or cancellation of a candidate at all, as if he/she never filed a CoC.
CoC proceeding under Section 78 of the OEC is premised on a person’s
misrepresentation of any of the material qualifications required for The foregoing variance gains utmost importance to the present case
the elective office aspired for. It is not enough that a person lacks the considering its implications on candidate substitution.
relevant qualification; he or she must have also made a false As explained in the case of Miranda v. Abaya, a candidate who is
representation of the same in the CoC. The nature of a Section 78 disqualified under Section 68 can be validly substituted pursuant to
petition was discussed in the case of Fermin v. COMELEC, where the Section 77 because he remains a candidate until disqualified; but a
Court illumined: person whose CoC has been denied due course to and/or cancelled
Let it be misunderstood, the denial of due course to or the under Section 78 cannot be substituted because he is not considered
cancellation of the CoC is not based on the lack of qualifications a candidate. Stated differently, since there would be no candidate to
but on a finding that the candidate made a material speak of under a denial of due course to and/or cancellation of a CoC
representation that is false, which may relate to the qualifications case, then there would be no candidate to be substituted; the same
required of the public office he/she is running for. It is noted that does not obtain, however, in a disqualification case since there
the candidates states in his/her CoC that he/she is eligible for the remains to be a candidate to be substituted, although his or her
office he/she seeks. Section 78 of the OEC, therefore, is to be candidacy is discontinued.
read in relation to the constitutional and statutory provisions on On this note, it is equally revelatory that Section 77 expressly
qualifications or eligibility for public office. If the candidate enumerates the instances where substitution is permissible, that is
subsequently states a material representation in the CoC that is when an official candidate of a registered or accredited political party
false, the COMELEC, following the law, is empowered to deny due "dies, withdraws or is disqualified for any cause." Noticeably, material
course to or cancel such certificate. Indeed, the Court has already misrepresentation cases are not included in the said section and
likened a proceeding under Section 78 to a quo warranto therefore, cannot be a valid basis to proceed with candidate
proceeding under Section 253 of the OEC since they both deal substitution.
with the eligibility or qualification of a candidate, with the
distinction mainly in the fact that a "Section 78" petition is filed c. Effects of Disqualification Case
before proclamation, while a petition for quo warranto is filed
i. If the judgement for disqualification has attained its finality
after proclamation of the winning candidate.
before election
Corollary thereto, it must be noted that the deliberateness of the
1. RA 6646 (The Electoral Reforms Law of 1987), Sec. 6;
misrepresentation, much less one’s intent to defraud, is of bare
significance in a Section 78 petition as it is enough that the person’s Sec. 6 - Effect of Disqualification Case.—Any candidate who has
declaration of a material qualification in the CoC be false. In this been declared by final judgment to be disqualified shall not be
relation, jurisprudence holds that an express finding that the person voted for, and the votes cast for him shall not be counted. If for
committed any deliberate misrepresentation is of little consequence any reason a candidate is not declared by final judgment before
in the determination of whether one’s CoC should be deemed an election to be disqualified and he is voted for and receives the
cancelled or not. What remains material is that the petition essentially winning number of votes in such election, the Court or
seeks to deny due course to and/or cancel the CoC on the basis of Commission shall continue with the trial and hearing of the
one’s ineligibility and that the same be granted without any action, inquiry, or protest and, upon motion of the complainant
qualification. or any intervenor, may during the pendency thereof order the
suspension of the proclamation of such candidate whenever the
Pertinently, while a disqualified candidate under Section 68 is still
evidence of his guilt is strong.
considered to have been a candidate for all intents and purposes, on
the other hand, a person whose CoC had been denied due course to Grego v. Comelec, G.R. No. 125955, June 19, 1997
and/or cancelled under Section 78 is deemed to have not been a

M.R.A.D.C. LUMBRE 218


CONSTITUTIONAL LAW REVIEW

Under Section 20, paragraph (i) of Rep. Act 7166 reads: respected and even given finality. The COMELEC has not found
any ground to suspend the proclamation and the records likewise
SEC. 20. Procedure in Disposition of Contested Election fail to show any so as to warrant a different conclusion from this
Returns.- Court. Hence, there is no ample justification to hold that the
xxxxxxxxx COMELEC gravely abused its discretion.

(i) The board of canvassers shall not proclaim any candidate The simple purpose and duty of the canvassing board is to
as winner unless authorized by the Commission after the ascertain and declare the apparent result of the voting. All other
latter has ruled on the objections brought to it on appeal by questions are to be tried before the court or other tribunal for
the losing party. Any proclamation made in violation hereof contesting elections or in quo warranto proceedings.
shall be void ab initio, unless the contested returns will not ii. If the judgment for disqualification is still unresolved or has
adversely affect the results of the election. not yet attained its finality before election
The inapplicability of the abovementioned provision to the 1. Sunga v. Comelec, 288 SCRA 76 (1998)
present case is very much patent on its face considering that
the same refers only to a void proclamation in relation to The Court discerns nothing in COMELEC Resolution No. 2050
contested returns and NOT to contested qualifications of a declaring, ordering or directing the dismissal of a disqualification
candidate. case filed before the election but which remained unresolved after
the election. What the Resolution mandates in such a case is for
Next, petitioner cites Section 6 of Rep. Act 6646 which states: the Commission to refer the complaint to its Law Department for
SEC. 6. Effect of Disqualification Case. - Any candidate who investigation to determine whether the acts complained of have
has been declared by final judgment to be disqualified shall in fact been committed by the candidate sought to be
not be voted for, and the votes cast for him shall not be disqualified. The findings of the Law Department then become
counted. If for any reason, a candidate is not declared by the basis for disqualifying the erring candidate. This is totally
final judgment before an election to be disqualified and he is different from the other two situations contemplated by
voted for and receives the winning number of votes in such Resolution No. 2050, i.e., a disqualification case filed after the
election, the Court or Commission shall continue with the election but before the proclamation of winners and that filed
trial and hearing of the action, inquiry or protest and, upon after the election and the proclamation of winners, wherein it was
motion of the complainant or any intervenor, may during the specifically directed by the same Resolution to be dismissed as a
pendency thereof order the suspension of the proclamation disqualification case. Resolution No. 2050 as interpreted in
of such candidate whenever the evidence of his guilt is Silvestre v. Duavit infringes on Sec. 6 of RA No. 6646:
strong. Sec. 6. Effects of Disqualification Case. — Any candidate who
This provision, however, does not support petitioners contention has been declared by final judgment to be disqualified shall
that the COMELEC, or more properly speaking, the Manila City not be voted for, and the votes cast for him shall not be
BOC, should have suspended the proclamation. The use of the counted. If for any reason a candidate is not declared by final
word may indicates that the suspension of a proclamation is judgment before an election to be disqualified and he is
merely directory and permissive in nature and operates to confer voted for and receives the winning number of votes in such
discretion. What is merely made mandatory, according to the election, the Court or Commission shall continue with the
provision itself, is the continuation of the trial and hearing of the trial and hearing of the action, inquiry or protest and, upon
action, inquiry or protest. Thus, in view of this discretion granted motion of the complainant or any intervenor, may during the
to the COMELEC, the question of whether or not evidence of guilt pendency thereof order the suspension of the proclamation
is so strong as to warrant suspension of proclamation must be of such candidate whenever the evidence of his guilt is
left for its own determination and the Court cannot interfere strong.
therewith and substitute its own judgment unless such discretion Clearly, the legislative intent is that the COMELEC should
has been exercised whimsically and capriciously. The COMELEC, continue the trial and hearing of the disqualification case to its
as an administrative agency and a specialized constitutional body conclusion, i.e., until judgment is rendered thereon. The word
charged with the enforcement and administration of all laws and "shall" signifies that this requirement of the law is mandatory,
regulations relative to the conduct of an election, plebiscite, operating to impose a positive duty which must be enforced. The
initiative, referendum, and recall, has more than enough implication is that the COMELEC is left with no discretion but to
expertise in its field that its findings or conclusions are generally proceed with the disqualification case even after the election.

M.R.A.D.C. LUMBRE 219


CONSTITUTIONAL LAW REVIEW

Thus, in providing for the outright dismissal of the disqualification wise to delegate its authority to its Law Department as partial
case which remains unresolved after the election, Silvestre v. solution to the problem. The May 8, 1995 elections, however, did
Duavit in effect disallows what RA No. 6646 imperatively not result in a surfeit of disqualification cases which the COMELEC
requires. This amounts to a quasi-judicial legislation by the cannot handle. Hence, its decision to resolve the disqualification
COMELEC which cannot be countenanced and is invalid for having case of Blanco directly and without referring it to its Law
been issued beyond the scope of its authority. Interpretative Department is within its authority, a sound exercise of its
rulings of quasi-judicial bodies or administrative agencies must discretion. The action of the COMELEC is in accord with Section
always be in perfect harmony with statutes and should be for the 28 of R.A. No. 6646, viz:
sole purpose of carrying their general provisions into effect. By
such interpretative or administrative rulings, of course, the scope "x x x.
of the law itself cannot be limited. Indeed, a quasi-judicial body "SEC. 28. Prosecution of Vote-Buying and Vote-selling. - The
or an administrative agency for that matter cannot amend an act presentation of a complaint for violations of paragraph (a) or
of Congress. Hence, in case of a discrepancy between the basic (b) of Section 261 of Batas Pambansa Blg. 881 supported by
law and an interpretative or administrative ruling, the basic law affidavits of complaining witness attesting to the offer or
prevails. promise by or of the voter's acceptance of money or other
Time and again this Court has given its imprimatur on the consideration from the relatives, leaders or sympathizers of
principle that COMELEC is with authority to annul any canvass a candidate, shall be sufficient basis for an investigation to
and proclamation which was illegally made. The fact that a be immediately conducted by the Commission, directly or
candidate proclaimed has assumed office, we have said, is no bar through its duly authorized legal officers under Section 68 or
to the exercise of such power. It of course may not be availed of Section 265 of said Batas Pambansa Blg. 881.
where there has been a valid proclamation. Since private Blanco also urges that COMELEC erred in using summary
respondent's petition before the COMELEC is precisely directed at proceedings to resolve his disqualification case. Again, the
the annulment of the canvass and proclamation, we perceive that COMELEC action is safely anchored on section 4 of its Rules of
inquiry into this issue is within the area allocated by the Procedure which expressly provides that petitions for
Constitution and law to COMELEC. disqualification "shall be heard summarily after due notice." Vote-
2. Nolasco v. Comelec, 275 SCRA 762 (1997) buying has its criminal and electoral aspects. Its criminal aspect
to determine the guilt or innocence of the accused cannot be the
Blanco was not denied due process and equal protection of the subject of summary hearing. However, its electoral aspect to
laws. He was given all the opportunity to prove that the evidence ascertain whether the offender should be disqualified from office
on his disqualification was not strong. can be determined in an administrative proceeding that is
summary in character.
"Sec. 6. Effect of Disqualification Case - Any candidate who
has been declared by final judgment to be disqualified shall iii. If the judgment for disqualification has attained its finality
not be voted for, and the votes cast for him shall not be after election
counted. If for any reason a candidate is not declared by final
judgment before an election to be disqualified and he is 1. Aquino v. Comelec, G.R. No. 120265, September 18,
voted for and receives the winning number of votes in such 1995
election, the Court or Commission shall continue with the Petitioner conveniently confuses the distinction between an
trial and hearing of the action, inquiry or protest and, upon unproclaimed candidate to the House of Representatives and a
motion of the complainant or any intervenor, may during the member of the same. Obtaining the highest number of votes in
pendency thereof order the suspension of the proclamation an election does not automatically vest the position in the winning
of such candidate whenever the evidence of his guilt is candidate. Section 17 of Article VI of the 1987 Constitution
strong." reads:
Despite these laws and existing jurisprudence, Blanco contends The Senate and the House of Representatives shall have an
that COMELEC must follow the procedure in Resolution No. 2050 Electoral Tribunal which shall be the sole judge of all contests
as amended. We hold that COMELEC cannot always be relating to the election, returns and qualifications of their
straitjacketed by this procedural rule. The COMELEC has respective Members.
explained that the resolution was passed to take care of the
proliferation of disqualification cases at that time. It deemed it

M.R.A.D.C. LUMBRE 220


CONSTITUTIONAL LAW REVIEW

Under the above-stated provision, the electoral tribunal clearly As to If disqualified before If disqualified
assumes jurisdiction over all contests relative to the election, Replacement election: Rule on before and after
returns and qualifications of candidates for either the Senate or succession is election: bona fide
the House only when the latter become members of either the applicable. candidate with 2nd
Senate or the House of Representatives. A candidate who has highest number of
not been proclaimed[ and who has not taken his oath of office If disqualified after votes will be
cannot be said to be a member of the House of Representatives election: proclaimed winner.
subject to Section 17 of Article VI of the Constitution. Thus,
petitioner's contention that "after the conduct of the election and Period to File After filing of COC Within 5 days from
(petitioner) has been established the winner of the electoral and before the last day of
exercise from the moment of election, the COMELEC is proclamation. filing of the COC
automatically divested of authority to pass upon the question of but within 25 days
qualification" finds no basis in law, because even after the from the time the
elections the COMELEC is empowered by Section 6 (in relation to COC was filed.
Section 7) of R.A. 6646 to continue to hear and decide questions (NOTE: In practice
relating to qualifications of candidates. nowadays, the 25
day rule does not
Under the above provision, not only is a disqualification case
apply.)
against a candidate allowed to continue after the election (and
Commonality Both included status as a registered voter.
does not oust the COMELEC of its jurisdiction), but his obtaining
the highest number of votes will not result in the suspension or
termination of the proceedings against him when the evidence of
guilt is strong. As petitioner clearly lacks one of the essential 3. Certificates of Candidacy (COC)
qualifications for running for membership in the House of
A CoC evidences candidate’s statutory eligibility to be elected for an elective post. It is
Representatives, not even the will of a majority or plurality of the
the document which formally accords upon a person the status of a candidate.
voters of the Second District of Makati City would substitute for
a requirement mandated by the fundamental law itself. A CoC may be amended before the elections, even after the date of its filing.
NOTE: Provisions of the election law on certificates of candidacy are mandatory in terms.
However, after the elections, they are regarded as directory so as to give effect to the
DISQUALIFICATI DENIAL/ will of the electorate.
ON (Sec. 68) CANCELLATION
(Sec. 78) a. Ministerial duty of the Comelec to receive a COC
As to Ground The candidate Pertains to material
i. Cerafica v. Comelec, G.R. No. 205136, December 2, 2014
committed acts misrepresentation
prohibited under the of eligibility. In Cipriano v. COMELEC, we ruled that the COMELEC has no discretion to give
OEC and other or not to give due couse to COCs. We emphasized that the duty of the
special laws. (Sec. COMELEC to give due course to COCs filed in due form is ministerial in
12 of OEC, Sec. 40 character, and that whilethe COMELEC may look into patent defects in the
of LGC) COCs, it may not go into matters not appearing on their face. The question of
As to Effect Person who filed the Considered to eligibility or ineligibility of a candidate is thus beyond the usual and proper
COC is considered a never have been a cognizance of the COMELEC.
candidate until candidate.
COMELEC Section 77 of the Omnibus Election Code (B.P. Blg. 881) provides for the
disqualifies him. procedure of substitution of candidates, to wit:
As to May be substituted No substitution Sec. 77. Candidates in case of death, disqualification or withdrawal of
Substitution if candidate dies, allowed. another. – If after the last day for the filing of certificates of candidacy, an
withdraws his COC, official candidate of a registered or accredited political party dies,
or is disqualified. withdraws or is disqualified for any cause, only a person belonging to, and
(Sec. 77) certified by, the same political party may file a certificate of candidacy to
replace the candidate who died, withdrew or was disqualified. The

M.R.A.D.C. LUMBRE 221


CONSTITUTIONAL LAW REVIEW

substitute candidate nominated by the political party concerned may file Election Day, the substitute candidate may file a CoC with any Board of Election
his certificate of candidacy for the office affected in accordance with the Inspectors, Election Officers, Provincial Election Supervisor, or Regional Election
preceding sections not later than mid-day of election day of the election. Director, as the case may be, in the political subdivision where such person is a
candidate, or in the case of a candidate for President, Vice-President or Senator,
b. Effect of filing a COC with the Law Department; Provided that, the substitute and the substituted
i. Quinto v. Comelec, G.R. No. 189698, February 22, 2010 candidate have the same surnames. (COMELEC Resolution 9984, August 18,
2015)
The new decision upheld Section 4(a) of Resolution 8678, Section 13 of RA
9369 and Section 66 of the Omnibus Election Code. Requisites for valid substitution.

Section 4. Effects of Filing of Certificates of Candidacy. (a) Any 1. The substitute must belong to the same party or coalition; and
person holding a public appointive office or position, including active 2. The deceased, disqualified or withdrawn candidate must have duly filed a valid
members of the Armed Forces of the Philippines, and other officers and CoC.
employees in government-owned or controlled corporations, shall be
considered ipso facto resigned from his office upon the filing of his Note: The second requisite is a condition sine qua non.
certificate of candidacy. (b) Any person holding an elective office or
position shall not be considered resigned upon the filing of his i. Federico vs. COMELEC, G.R. No. 199612, January 22, 2013
certificate of candidacy for the same or any other elective office or The Court held that the COMELEC is empowered by law to prescribe rules so
position. as to make efficacious and successful the conduct of the first national
The new majority who deliberated for the new ruling held that these provisions automated election. Since the 2010 election was automated, there was a need
satisfy the requisites of the equal protection test, especially the second for early printing of ballots and set deadline for the filing of COC, as well as for
requirement that it must be germane to the purpose of the law. It was substitution of candidates.
emphasized that the purpose of the law is to defer to the sovereign will of the The Court did not agree with the contention of Federico that he has until before
people by letting elective officials serve until the end of the terms for which mid-day of election to file his COC as substitute candidate to Edna. Regarding
they were elected notwithstanding the filing of their certificates of candidacy. the 2010 automated elections, COMELEC came out with Resolution No. 8678
On the contrary, the automatic resignation rule was imposed upon appointive which provides different deadline to file COC, depending on the ground of
officials because unlike elected politicians, “appointive officials, as officers and substitution. For withdrawal, the substitute for a candidate who withdrew may
employees in the civil service, are strictly prohibited from engaging in any file his certificate of candidacy not later than December 14, 2009. On the other
partisan political activity or from taking part in any election, except to vote. hand, the substitute for a candidate who died or suffered permanent incapacity
The Chief underscored the fact that the American Jurisprudence heavily relied or disqualified by final judgment, may file his certificate of candidacy up to
upon on the original decision had already been overturned by prevailing mid-day of election day.
jurisprudence in the United States. The Court cited several decisions of the US The Court said that as correctly pointed out by the OSG, it is clear that different
Supreme Court stating that the right to express one’s views through candidacy deadlines were set to govern the specific circumstances that would necessitate
is not a fundamental right and is neither covered by the freedom of expression the substitution of a candidate due to death, disqualification or withdrawal. The
nor the right to association. More importantly, it was ruled that the resign-to- reason for the distinction can easily be divined. Unlike death or disqualification,
run rule on appointive officials does not violate a person’s right to run for withdrawal is voluntary. Generally, a candidate has sufficient time to ponder
public office because such right must give way to the substantial public interest on his candidacy and to withdraw while the printing has not yet started. If a
being protected by the rule – to maintain a civil service that is impartial and candidate withdraws after the printing, the name of the substitute candidate
free from the evils of partisan politicians. can no longer be accommodated in the ballot and a vote for the substitute will
c. Substitution of Candidates just be wasted.

An official candidate of a duly registered political party or coalition who dies, As Federico's substitution was not valid, there was only one qualified candidate
withdraws, or is disqualified for any cause after the last day for the filing of CoCs in the mayoralty race in Sto. Tomas, Batangas Maligaya. Being the only
may be substituted by a candidate belonging to, and nominated by, the same candidate, he received the highest number of votes. Accordingly, he should be
political party or coalition. No substitute shall be allowed for any independent proclaimed as the duly elected mayor in the May 10, 2010 elections.
candidate. The substitute for a candidate who died or is disqualified by final Considering that Maligaya was the winner, the position of Intervenor Silva that
judgment, may file a CoC up to mid-day of Election Day; Provided that, the he be considered the legal successor of Federico, whom he claims failed to
substitute and the substituted have the same surnames. If the death or qualify, has no legal basis. There is simply no vacancy. When there is no
disqualification should occur between the day before the election and mid-day of vacancy, the rule on succession under Section 44 of the LGC cannot be
invoked.

M.R.A.D.C. LUMBRE 222


CONSTITUTIONAL LAW REVIEW

d. Petition to Deny Due Course to or Cancel COC Jurisdiction over a petition to cancel a certificate of candidacy lies with the
COMELEC in division, not with the COMELEC en banc. (Gravida v. Sales, G.R.
i. Denial of Due Course to or Cancelling COC of a Nuisance Candidate No. 122872)
(OEC, Sec. 69)
NOTE: Material misrepresentation in a CoC refers to the qualification for
Any registered candidate for the same office may file a petition to declare a elective office, which includes false statement as to age, residency, citizenship,
duly registered candidate as a nuisance candidate, personally or through duly being a registered voter and any other legal qualifications necessary to run for
authorized representative with COMELEC, within 5 days from the last day of an elective office. A misrepresentation which does not affect one’s qualification
filing of CoC. [RA 6646 (The Electoral Reforms Law of 1987), Sec. 5] to run or hold public office will not suffice for the cancellation of a CoC, and the
Grounds. The COMELEC may motu proprio or upon verified petition refuse to subsequent election of the candidate into office will cure the said
give due course to or cancel a certificate of candidacy if shown that it was filed misrepresentation.
to: 1. Hayudini v. Comelec, G.R. No. 207900, April 22, 2014
1. Put the election process in mockery or disrepute; Section 74 requires the candidate to state under oath in his CoC "that he
2. Cause confusion among the voters by the similarity of the names of is eligible for said office." A candidate is eligible if he has a right to run
the registered candidates for the public office. If a candidate is not actually eligible because he is
not a registered voter in the municipality where he intends to be elected,
3. Clearly demonstrate that the candidate has no bona fide but still he states under oath in his certificate of candidacy that he is
intention to run for the office for which the CoC has been filed and thus eligible to run for public office, then the candidate clearly makes a false
prevent a faithful determination of the true will of the electorate. (OEC, material representation, a ground to support a petition under Section 78.
Sec. 69)
In this case, when petitioner stated in his CoC that he is a resident of
Power of COMELEC Barangay Bintawlan, South Ubian, Tawi Tawi and eligible for a public
office, but it turned out that he was declared to be a non-resident thereof
General Rule: The COMELEC may, motu proprio or upon verified petition of
in a petition for his inclusion in the list of registered voters, he therefore
an interested party, refuse to give due course to or cancel a CoC upon showing
committed a false representation in his CoC which pertained to a material
of the above-stated circumstances. (OEC, Sec. 69)
fact which is a ground for the cancellation of his CoC under Section 78 of
Exception: The COMELEC cannot motu proprio deny due course to or cancel the Omnibus Election Code. Petitioner's ineligibility for not being a resident
an alleged nuisance candidate’s certificate of candidacy without providing the of the place he sought election is not a ground for a petition for
candidate his opportunity to be heard. disqualification, since the grounds enumerated under Section 68[39] of
the Omnibus Election Code specifically refer to the commission of
ii. Denial of Due Course to or Cancellation of COC for False Material prohibited acts, and possession of a permanent resident status in a foreign
Representation in the COC (OEC, Sec. 78) country.
A verified petition seeking to deny due course or to cancel a CoC may be filed 2. Petition to Deny Due Course to or Cancel COC For False Material
by the person exclusively on the ground that any material representation Representation in the COC (OEC, Sec. 78) vs. Petition for Quo
contained therein as required under Sec. 74 of the OEC is false (B.P. 881, Sec. Warranto (OEC, Sec. 253) –
78), provided that:
The denial of due course to or the cancellation of the CoC is not based on
1. The false representation pertains to material matter affecting the lack of qualifications but on a finding that the candidate made a
substantive rights of a candidate; and material representation that is false, which may relate to the qualifications
2. The false representation must consist of deliberate attempt to mislead, required of the public office he/she is running for. It is noted that the
misinform, or hide a fact which would otherwise render a candidate candidate states in his/her CoC that he/she is eligible for the office he/she
ineligible. (Salcedo II v. COMELEC, G.R. No. 135886, Aug. 16, 1999) seeks. Section 78 of the OEC, therefore, is to be read in relation to the
constitutional and statutor provisions on qualifications or eligibility for
These two requirements must concur to warrant the cancellation of the CoC. public office. If the candidate subsequently states a material
Period to file a petition to deny due course to or cancel a CoC The petition may representation in the CoC that is false, the COMELEC, following the law, is
be filed at any time not later than 25 days from the time of the filing of the empowered to deny due course to or cancel such certificate. Indeed, the
CoC and shall be decided, after due notice and hearing, not later than 15 days Court has already likened a proceeding under Section 78 to a quo warranto
before the election. proceeding under Section 253 of the OEC since they both deal with the
eligibility or qualification of a candidate, with the distinction mainly in the
fact that a Section 78 petition is filed before proclamation, while a petition

M.R.A.D.C. LUMBRE 223


CONSTITUTIONAL LAW REVIEW

for quo warranto is filed after proclamation of the winning candidate. hearing of the action, inquiry, or protest and, upon motion of the
(Fermin vs COMELEC, December 18, 2008) complainant or any intervenor, may during the pendency thereof order the
suspension of the proclamation of such candidate whenever the evidence
Denial/Cancellation Quo Warranto of his guilt is strong.
(Section 78) (Section 253)
Section 7. Petition to Deny Due Course To or Cancel a Certificate of
As to False Material 1. Ineligibility;
Candidacy. - The procedure hereinabove provided shall apply to
Grounds Representation/ 2. Disloyalty to the
petitions to deny due course to or cancel a certificate of candidacy
Material Republic of the
as provided in Section 78 of Batas Pambansa Blg. 881.
Misrepresentation Philippines.
As to By any person Any registered voter in 2. If the denial of due course to or cancellation of a COC is ordered
Who may the constituency where because of the declaration of a nuisance candidate - Dela Cruz v.
File the candidate sought to Comelec, G.R. No. 192221, November 13, 2012
be disqualified ran for
office. In the more recent case of Martinez III v. HRET, this Court likewise applied
Period to Within 5 days from Within 10 days from the rule in COMELEC Resolution No. 4116 not to consider the votes cast
File the last day of filing proclamation. for a nuisance candidate stray but to count them in favor of the
of the COC but bona fide candidate notwithstanding that the decision to declare him as
within 25 days from such was issued only after the elections.
the time the COC Here, Aurelio was declared a nuisance candidate long before the
was filed. (NOTE:
May 10, 2010 elections. On the basis of Resolution No. 4116, the
In practice
votes cast for him should not have been considered stray but
nowadays, the 25
counted in favor of petitioner. COMELEC's changing of the rule on
day rule does not
apply.) votes cast for nuisance candidates resulted in the invalidation of
significant number of votes and the loss of petitioner to private
Quo Warranto. The issue is whether respondent possesses all the qualifications and respondent by a slim margin.
none of the disqualifications prescribed by law (A.M. No. 07-4-15-SC, May 15, 2007).
Bautista upheld the basic rule that the primordial objective of election laws
Quo warranto proceedings against a Congressman-elect, Senator-elect, President- is to give effect to, rather than frustrate, the will of the voter. The inclusion
elect and VP-elect are brought before the appropriate electoral tribunals created of nuisance candidates turns the electoral exercise into an uneven playing
by the Constitution. field where the bona fide candidate is faced with the prospect of having a
significant number of votes cast for him invalidated as stray votes by the
Quo warranto proceedings against any regional, provincial or city officials are
brought before the COMELEC. mere presence of another candidate with a similar surname.

Quo warranto proceedings against municipal officials and barangay officials are 3. Rule applicable to both grounds - Ara Tea v. Comelec, G.R. No.
brought before the RTCs and MTCs respectively. 195229, October 9, 2012

e. Effects of a Case for Denial of Due Course to or Cancellation of COC In a certificate of candidacy, the candidate is asked to certify under oath
his eligibility, and thus qualification, to the office he seeks election. Even
i. If the judgement for a petition to deny due course to or cancel a COC though the certificate of candidacy does not specifically ask the candidate
has attained its finality before election for the number of terms elected and served in an elective position, such
fact is material in determining a candidate’s eligibility, and thus
1. If the ground for cancellation or denial is false material qualification for the office. Election to and service of the same local elective
representation in the COC - RA 6646 (The Electoral Reforms Law
position for three consecutive terms renders a candidate ineligible from
of 1987), Sec. 6 in relation to Sec. 7 thereof running for the same position in the succeeding elections.
Section 6. Effect of Disqualification Case. - Any candidate who has been
A candidate for mayor in the 2010 local elections was thus required to
declared by final judgment to be disqualified shall not be voted for, and provide 12 items of information in the certificate of candidacy: name;
the votes cast for him shall not be counted. If for any reason a
nickname or stage name; gender; age; place of birth; political party that
candidate is not declared by final judgment before an election to be nominated the candidate; civil status; residence/address; profession or
disqualified and he is voted for and receives the winning number of votes
occupation; post office address for election purposes; locality of which the
in such election, the Court or Commission shall continue with the trial and candidate is a registered voter; and period of residence in the Philippines

M.R.A.D.C. LUMBRE 224


CONSTITUTIONAL LAW REVIEW

before 10 May 2010. The candidate also certifies four statements: a Loong for the office of Regional Vice-Governor of the Autonomous
statement that the candidate is a natural born or naturalized Filipino Government of Muslim Mindanao for false representation as to his age.
citizen; a statement that the candidate is not a permanent resident of, or The petition was filed 16 days after the election, and clearly beyond the
immigrant to, a foreign country; a statement that the candidate is eligible prescribed 25 day period from the last day of filing certificates of
for the office he seeks election; and a statement of the candidate’s candidacy. This Court ruled that Ututalum’s petition was one based on
allegiance to the Constitution of the Republic of the Philippines. The false representation under Section 78, and not for disqualification under
certificate of candidacy should also be under oath, and filed within the Section 68. Hence, the 25-day prescriptive period provided in Section 78
period prescribed by law. should be strictly applied. We recognized the possible gap in the law:

The conviction of Lonzanida by final judgment, with the penalty of prisión It is true that the discovery of false representation as to material facts
mayor, disqualifies him perpetually from holding any public office, or from required to be stated in a certificate of candidacy, under Section 74 of the
being elected to any public office. This perpetual disqualification took Code, may be made only after the lapse of the 25-day period prescribed
effect upon the finality of the judgment of conviction, before Lonzanida by Section 78 of the Code, through no fault of the person who discovers
filed his certificate of candidacy. such misrepresentations and who would want the disqualification of the
candidate committing the misrepresentations. It would seem, therefore,
Lest it be misunderstood, the denial of due course to or the cancellation that there could indeed be a gap between the time of the discovery of the
of the CoC is not based on the lack of qualifications but on a finding that misrepresentation, (when the discovery is made after the 25-day period
the candidate made a material representation that is false, which may under Sec. 78 of the Code has lapsed) and the time when the proclamation
relate to the qualifications required of the public office he/she is running of the results of the election is made. During this so-called "gap" the
for. It is noted that the candidate states in his/her CoC that he/she is would-be petitioner (who would seek the disqualification of the candidate)
eligible for the office he/she seeks. Section 78 of the OEC, therefore, is to is left with nothing to do except to wait for the proclamation of the results,
be read in relation to the constitutional and statutory provisions so that he could avail of a remedy against the misrepresenting candidate,
on qualifications or eligibility for public office. If the candidate that is, by filing a petition for quo warranto against him. Respondent
subsequently states a material representation in the CoC that is false, the Commission sees this "gap" in what it calls a procedural gap which,
COMELEC, following the law, is empowered to deny due course to or cancel according to it, is unnecessary and should be remedied.
such certificate.
At the same time, it cannot be denied that it is the purpose and intent of
Indeed, the Court has already likened a proceeding under Section 78 to a the legislative branch of the government to fix a definite time within which
quo warranto proceeding under Section 253 of the OEC since they both petitions of protests related to eligibility of candidates for elective offices
deal with the eligibility or qualification of a candidate, with the distinction must be filed, as seen in Sections 78 and 253 of the Code. Respondent
mainly in the fact that a "Section 78" petition is filed before proclamation, Commission may have seen the need to remedy this so-called “procedural
while a petition for quo warranto is filed after proclamation of the winning gap", but it is not for it to prescribe what the law does not provide, its
candidate. function not being legislative. The question of whether the time to file
It is obvious from a reading of the laws and jurisprudence that there is an these petitions or protests is too short or ineffective is one for the
overlap in the grounds for eligibility and ineligibility vis-à-vis qualifications Legislature to decide and remedy.
and disqualifications. For example, a candidate may represent that he is a NOTE: Effects of disqualification under Section 68 in relation to Sections
resident of a particular Philippine locality when he is actually a permanent 6 and 7 of R.A. No. 6646 is the same in Section 78; i.e. the proclamation
resident of another country. In cases of such overlap, the petitioner should may be suspended whenever evidence of guilt is strong.
not be constrained in his choice of remedy when the Omnibus Election
Code explicitly makes available multiple remedies. Section 78 allows the ii. If the judgment for a petition to deny due course to or cancel a COC
filing of a petition to deny due course or to cancel a certificate of candidacy is still unresolved or has not yet attained its finality before election
before the election, while Section 253 allows the filing of a petition for quo
warranto after the election. Despite the overlap of the grounds, one should 1. Sunga v. Comelec, 288 SCRA 76 (1998)
not confuse a petition for disqualification using grounds enumerated in
Section 68 with a petition to deny due course or to cancel a certificate of The purpose of a disqualification proceeding is to prevent the candidate
candidacy under Section 78. from running or, if elected, from serving, or to prosecute him for violation
of election laws. The fact that a candidate has been proclaimed and had
The distinction between a petition under Section 68 and a petition under assumed the position to which he was elected does not divest the
Section 78 was discussed in Loong v. Commission on Elections with COMELEC of authority and jurisdiction to continue the hearing and
respect to the applicable prescriptive period. Respondent Nur Hussein eventually decide the disqualification. The COMELEC should not dismiss
Ututalum filed a petition under Section 78 to disqualify petitioner Benjamin the case simply because the respondent has been proclaimed.

M.R.A.D.C. LUMBRE 225


CONSTITUTIONAL LAW REVIEW

Also, the fact that no docket fee was initially paid is not fatal. The to substitute our judgment for the mind of the voter. The second placer is just
Procedural defect as cured by the subsequent payment of the docket fee. that, a second placer.

As provided in Sec. 44, RA No. 7160 and echoed in Art. 83 of the iii. If the judgment for disqualification has attained its finality after
Implementing Rules and Regulations of the Local Government Code of election
1991, the language of the law is clear, explicit and unequivocal, The Court or Commission shall continue with the trial and hearing of the action,
accordingly, in the event that Trinidad is adjudged to be disqualified, a inquiry or protest and upon motion of the complainant or any intervenor, may,
permanent vacancy will be created for failure of the elected mayor to during the pendency thereof, order the suspension of the proclamation of such
qualify for the said office. In such eventuality, the duly elected vice-mayor candidate whenever the evidence of his guilt is strong.
shall succeed as provided by law.
1. If the ground for cancellation or denial is false material
2. Nolasco v. Comelec, 275 SCRA 762 (1997) representation in the COC - RA 6646, Sec. 6 in relation to Sec. 7
thereof
It cannot be denied that the COMELEC has jurisdiction over proclamation
Section 6. Effect of Disqualification Case. - Any candidate who has been
and disqualification cases. Article IX-C, section 2 of the Constitution
declared by final judgment to be disqualified shall not be voted for, and
endows the COMELEC the all encompassing power to "enforce and
the votes cast for him shall not be counted. If for any reason a
administer all laws and regulations relative to the conduct of an election .
candidate is not declared by final judgment before an election to be
. . ." We have long ruled that this broad power includes the power to
disqualified and he is voted for and receives the winning number of votes
cancel proclamations.
in such election, the Court or Commission shall continue with the trial and
hearing of the action, inquiry, or protest and, upon motion of the
Vote-buying has its criminal and electoral aspects. Its criminal aspect to
complainant or any intervenor, may during the pendency thereof order the
determine the guilt or innocence of the accused cannot be the subject of
suspension of the proclamation of such candidate whenever the evidence
summary hearing. However, its electoral aspect to ascertain whether the
of his guilt is strong.
offender should be disqualified from office can be determined in an
administrative proceeding that is summary in character. Section 7. Petition to Deny Due Course To or Cancel a Certificate of
Candidacy. - The procedure hereinabove provided shall apply to
Section 44, Chapter 2 of the Local Government Code of 1991 (R.A. petitions to deny due course to or cancel a certificate of candidacy
No. 7160) is unequivocal, thus: as provided in Section 78 of Batas Pambansa Blg. 881.

xxx xxx xxx 2. If the denial of due course to or cancellation of a COC is ordered
because of the declaration of a nuisance candidate - Dela Cruz v.
Sec. 44. Permanent Vacancies in the Offices of the Comelec, G.R. No. 192221, November 13, 2012
Governor, Vice Governor, Mayor, and Vice Mayor. — (a) If a It bears to stress that Sections 211 (24) and 72 applies to all
permanent vacancy occurs in the office of the governor or
disqualification cases and not to petitions to cancel or deny due course to
mayor, the vice governor or vice mayor concerned shall
a certificate of candidacy such as Sections 69 (nuisance candidates) and
become the governor or mayor.
78 (material representation shown to be false). Notably, such facts
xxx xxx xxx indicating that a certificate of candidacy has been filed “to put the election
process in mockery or disrepute, or to cause confusion among the voters
For purposes of this Chapter, a permanent vacancy arises by the similarity of the names of the registered candidates, or other
when an elective local official fills a higher vacant office, circumstances or acts which clearly demonstrate that the candidate has
refuses to assume office, fails to qualify, dies, is removed from no bona fide intention to run for the office for which the certificate of
office, voluntarily resigns, or is otherwise permanently candidacy has been filed and thus prevent a faithful determination of the
incapacitated to discharge the functions of his office. true will of the electorate” are not among those grounds enumerated in
Section 68 (giving money or material consideration to influence or corrupt
In a mayoralty election, the candidate who obtained the second voters or public officials performing electoral functions, election campaign
highest number of votes, in this case Alarilla, cannot be proclaimed
overspending and soliciting, receiving or making prohibited contributions)
winner in case the winning candidate is disqualified. To simplistically
of the OEC or Section 4022 of Republic Act No. 7160 (Local Government
assume that the second placer would have received the other votes would be
Code of 1991).

M.R.A.D.C. LUMBRE 226


CONSTITUTIONAL LAW REVIEW

In Fermin v. COMELEC, this Court distinguished a petition for (a) Petition to deny due course to a certificate of candidacy; (b) Petition
disqualification under Section 68 and a petition to cancel or deny due to declare a candidate as a nuisance candidate; (c) Petition to disqualify a
course to a certificate of candidacy (COC) under Section 78. Said candidate; and (d) Petition to postpone or suspend an election.
proceedings are governed by different rules and have distinct outcomes.
At this point, we must stress that a “Section 78” petition ought not to be Considering the foregoing and in order to guide field officials on the finality
interchanged or confused with a “Section 68” petition. They are different of decisions or resolutions on special action cases (disqualification cases)
remedies, based on different grounds, and resulting in different the Commission, RESOLVES, as it is hereby RESOLVED, as follows:
eventualities. Private respondent’s insistence, therefore, that the petition
(1) the decision or resolution of the En Banc of the Commission on
it filed before the COMELEC is in the nature of a disqualification case under
disqualification cases shall become final and executory after five (5) days
Section 68, as it is in fact captioned a “Petition for Disqualification,” does
from its promulgation unless restrained by the Supreme Court; x x x
not persuade the Court. x x x x To emphasize, a petition for
disqualification, on the one hand, can be premised on Section 12 or 68 of (4) the decision or resolution of the En Banc on nuisance appeared on the
the OEC, or Section 40 of the LGC. On the other hand, a petition to deny ballots on election day because while the COMELEC rendered its decision
due course to or cancel a CoC can only be grounded on a statement of a to cancel Edwin Bautista’s COC on April 30, 1998, it denied his motion for
material representation in the said certificate that is false. The petitions reconsideration only on May 13, 1998 or three days after the election. We
also have different effects. While a person who is disqualified under said that the votes for candidates for mayor separately tallied on orders
Section 68 is merely prohibited to continue as a candidate, the person of the COMELEC Chairman was for the purpose of later counting the votes
whose certificate is cancelled or denied due course under Section 78 is not and hence are not really stray votes. These separate tallies actually made
treated as a candidate at all, as if he/she never filed a CoC. the will of the electorate determinable despite the apparent confusion
caused by a potential nuisance candidate. Candidates, particularly
Thus, in Miranda v. Abaya, this Court made the distinction that a candidate
whether the nuisance candidate has the same name as the bona fide
who is disqualified under Section 68 can validly be substituted under
candidate shall be immediately executory;
Section 77 of the OEC because he/she remains a candidate until
disqualified; but a person whose CoC has been denied due course or (5) the decision or resolution of a DIVISION on nuisance
cancelled under Section 78 cannot be substituted because he/she is never candidate, particularly where the nuisance candidate has the same
considered a candidate. Clearly, a petition to cancel or deny due course to name as the bona fide candidate shall be immediately executory
a COC under Section 69 as in Section 78 cannot be treated in the same after the lapse of five (5) days unless a motion for reconsideration
manner as a petition to disqualify under Section 68 as what COMELEC did is seasonably filed. In which case, the votes cast shall not be
when it applied the rule provided in Section 72 that the votes cast for a considered stray but shall be counted and tallied for the bona fide
disqualified candidate be considered stray, to those registered candidates candidate.
whose COC’s had been cancelled or denied due course.
The votes cast for a nuisance candidate declared as such in a final
Strictly speaking, a cancelled certificate cannot give rise to a valid judgment, particularly where such nuisance candidate has the same
candidacy, and much less to valid votes. Said votes cannot be counted in surname as that of the legitimate candidate, not stray but counted in favor
favor of the candidate whose COC was cancelled as he/she is not treated of the latter.
as a candidate at all, as if he/she never filed a COC. But should these votes
cast for the candidate whose COC was cancelled or denied due course be 3. Rule applicable to both grounds - Hayudini v. Comelec, G.R. No.
considered stray? 207900, April 22, 2014

COMELEC Resolution No. 4116 issued in relation to the finality of Section 74 of the Omnibus Election Code requires the candidate to state
resolutions or decisions in special action cases, provides: under oath in his CoC "that he is eligible for said office." A candidate is
eligible if he has a right to run for the public office. If a candidate is not
This pertains to the finality of decisions or resolutions of the Commission actually eligible because he is not a registered voter in the municipality
en banc or division, particularly on Special Actions (Disqualification where he intends to be elected, but still he states under oath in his
certificate of candidacy that he is eligible to run for public office, then the
Cases). Special Action cases refer to the following:
candidate clearly makes a false material representation, a ground to
support a petition to deny due course or cancel a CoC.

M.R.A.D.C. LUMBRE 227


CONSTITUTIONAL LAW REVIEW

V. Campaign necessarily mean it is election propaganda. The tarpaulin was not paid for or posted
"in return for consideration" by any candidate, political party, or party-list group.

NOTE: Campaign includes crediting and discrediting witnesses as well.


1. Premature Election Campaigning
b. Ejercito v. Comelec, G.R. No. 212398, November 25, 2014
a. Penera v. Comelec, G.R. No. 181613, November 25, 2009
x x x [T]he jurisdiction of the COMELEC to disqualify candidates is limited to those
A candidate is liable for an election offense only for acts done during the campaign
enumerated in Section 68 of the [OEC]. All other election offenses are beyond the
period, not before. The law is clear as daylight — any election offense that may be
ambit of COMELEC jurisdiction. They are criminal and not administrative in nature.
committed by a candidate under any election law cannot be committed before the Pursuant to Sections 265 and 268 of the [OEC], the power of the COMELEC is
start of the campaign period. In ruling that Penera is liable for premature confined to the conduct of preliminary investigation on the alleged election offenses
campaigning for partisan political acts before the start of the campaigning, the for the purpose of prosecuting the alleged offenders before the regular courts of
assailed Decision ignores the clear and express provision of the law. justice, viz:

NOTE: Q: Can a candidate, after filing a COC, campaign without being penalized? Section 265. Prosecution. The Commission shall, through its duly authorized
A: Yes, as stated in Penera. legal officers, have the exclusive power to conduct preliminary investigation of
all election offenses punishable under this Code, and to prosecute the same.
Q: The Supreme Court in Penera seemed to legalize premature campaigning. What The Commission may avail of the assistance of other prosecuting arms of the
about the prohibition on premature campaigning under Section 80? A: There is still government: Provided, however, That in the event that the Commission fails
premature campaigning if done during these days (considered outside the to act on any complaint within four months from its filing, the complainant may
file the complaint with the office of the fiscal or with the Ministry of Justice for
campaign period):
proper investigation and prosecution, if warranted.
1. Maundy Thursday; xxxxxxxxx
2. Good Friday;
3. Eve of Election Day. Section 268. Jurisdiction. The regional trial court shall have the exclusive
original jurisdiction to try and decide any criminal action or proceeding for
2. Lawful Election Propaganda violation of this Code, except those relating to the offense of failure to register
or failure to vote which shall be under the jurisdictions of metropolitan or
a. The Diocese of Bacolod v. Comelec, G.R. No. 205728, January 21, 2015 municipal trial courts. From the decision of the courts, appeal will lie as in other
No, the tarpaulins are not election propaganda. It is within the right of the petitioner criminal cases.
guaranteed by the Constitution to free expression given to private citizens. In the case at bar, the COMELEC First Division and COMELEC En Banc correctly
ruled that the petition filed by San Luis against Ejercito is not just for prosecution
The term "political advertisement" or "election propaganda" refers to any of election offense but for disqualification as well.
matter broadcasted, published, printed, displayed or exhibited, in any
medium, which contain the name, image, logo, brand, insignia, color motif, Section 5. Effect of Petition if Unresolved Before Completion of Canvass. If a
initials, and other symbol or graphic representation that is capable of Petition for Disqualification is unresolved by final judgment on the day of
being associated with a candidate or party, and is intended to draw the elections, the petitioner may file a motion with the Division or Commission En
attention of the public or a segment thereof to promote or oppose, directly Banc where the case is pending, to suspend the proclamation of the candidate
or indirectly, the election of the said candidate or candidates to a public concerned, provided that the evidence for the grounds to disqualify is strong.
office. In broadcast media, political advertisements may take the form of For this purpose, at least three (3) days prior to any election, the Clerk of the
spots, appearances on TV shows and radio programs, live or taped Commission shall prepare a list of pending cases and furnish all Commissioners
announcements, teasers, and other forms of advertising messages or copies of said the list.
announcements used by commercial advertisers. Political advertising includes
In the event that a candidate with an existing and pending Petition to disqualify is
matters, not falling within the scope of personal opinion, that appear on any
proclaimed winner, the Commission shall continue to resolve the said Petition.
Internet website, including, but not limited to, social networks, blogging sites, and
micro-blogging sites, in return for consideration, or otherwise capable of pecuniary In Bagatsing v. COMELEC, the Court stated that the above-quoted resolution covers
estimation. It is clear that this paragraph suggests that personal opinions are not two (2) different scenarios:
included, while sponsored messages are covered. While the tarpaulin may influence
the success or failure of the named candidates and political parties, this does not First, as contemplated in paragraph 1, a complaint for disqualification filed before
the election which must be inquired into by the COMELEC for the purpose of

M.R.A.D.C. LUMBRE 228


CONSTITUTIONAL LAW REVIEW

determining whether the acts complained of have in fact been committed. Where Notably, R.A. No. 9006 explicitly directs that broadcast advertisements donated to
the inquiry results in a finding before the election, the COMELEC shall order the the candidate shall not be broadcasted without the written acceptance of the
candidate's disqualification. In case the complaint was not resolved before the candidate, which shall be attached to the advertising contract and shall be
election, the COMELEC may motu propio or on motion of any of the parties, refer submitted to the COMELEC, and that, in every case, advertising contracts shall be
the said complaint to the Law Department of the COMELEC for preliminary signed by the donor, the candidate concerned or by the duly-authorized
investigation. representative of the political party.88 Conformably with the mandate of the law,
COMELEC Resolution No. 9476 requires that election propaganda materials donated
Second, as laid down in paragraph 2, a complaint for disqualification filed after the to a candidate shall not be broadcasted unless it is accompanied by the written
election against a candidate (a) who has not yet been proclaimed as winner, or (b) acceptance of said candidate, which shall be in the form of an official receipt in the
who has already been proclaimed as winner. In both cases, the complaint shall be name of the candidate and must specify the description of the items donated, their
dismissed as a disqualification case but shall be referred to the Law Department of quantity and value, and that, in every case, the advertising contracts, media
the COMELEC for preliminary investigation. purchase orders or booking orders shall be signed by the candidate concerned or
However, if before proclamation, the Law Department makes a prima facie finding by the duly authorized representative of the party and, in case of a donation, should
of guilt and the corresponding information has been filed with the appropriate trial be accompanied by a written acceptance of the candidate, party or their authorized
court, the complainant may file a petition for suspension of the proclamation of the representatives. COMELEC Resolution No. 9615 also unambiguously states that it
respondent with the court before which the criminal case is pending and the said shall be unlawful to broadcast any election propaganda donated or given free of
court may order the suspension of the proclamation if the evidence of guilt is charge by any person or broadcast entity to a candidate without the written
strong. acceptance of the said candidate and unless they bear and be identified by the
words airtime for this broadcast was provided free of charge by followed by the
The exclusive power [of the COMELEC] to conduct a preliminary investigation of true and correct name and address of the donor.
all cases involving criminal infractions of the election laws stated in Par. 1 of
COMELEC Resolution No. 2050 pertains to the criminal aspect of a disqualification In tracing the legislative history of Sections 100, 101, and 103 of the OEC, it can
case. It has been repeatedly underscored that an election offense has its criminal be said, therefore, that the intent of our lawmakers has been consistent through
and electoral aspects. While its criminal aspect to determine the guilt or innocence the years: to regulate not just the election expenses of the candidate but also of
of the accused cannot be the subject of summary hearing, its electoral aspect to his or her contributor/supporter/donor as well as by including in the aggregate limit
ascertain whether the offender should be disqualified from office can be determined of the former’s election expenses those incurred by the latter. The phrase those
in an administrative proceeding that is summary in character. This Court said in incurred or caused to be incurred by the candidate is sufficiently adequate to cover
Sunga: those expenses which are contributed or donated in the candidate s behalf. By
virtue of the legal requirement that a contribution or donation should bear the
It is worth to note that an election offense has criminal as well as electoral aspects. written conformity of the candidate, a contributor/supporter/donor certainly
Its criminal aspect involves the ascertainment of the guilt or innocence of the qualifies as any person authorized by such candidate or treasurer. Ubi lex non
accused candidate. Like in any other criminal case, it usually entails a full-blown distinguit, nec nos distinguere debemus.126 (Where the law does not distinguish,
hearing and the quantum of proof required to secure a conviction is beyond neither should We.) There should be no distinction in the application of a law where
reasonable doubt. Its electoral aspect, on the other hand, is a determination of none is indicated.
whether the offender should be disqualified from office. This is done through an
administrative proceeding which is summary in character and requires only a clear The inclusion of the amount contributed by a donor to the candidate’s allowable
preponderance of evidence. Thus, under Sec. 4 of the COMELEC Rules of Procedure, limit of election expenses does not trample upon the free exercise of the voters’
petitions for disqualification "shall be heard summarily after due notice." It is the rights of speech and of expression under Section 4, Artticle III of the Constitution.
electoral aspect that we are more concerned with, under which an erring candidate As a content-neutral regulation,127 the law s concern is not to curtail the message
may be disqualified even without prior criminal conviction. or content of the advertisement promoting a particular candidate but to ensure
equality between and among aspirants with deep pockets and those with less
The criminal aspect of a disqualification case determines whether there is probable financial resources. Any restriction on speech or expression is only incidental and
cause to charge a candidate for an election offense. The prosecutor is the COMELEC, is no more than necessary to achieve the substantial governmental interest of
through its Law Department, which determines whether probable cause exists. If promoting equality of opportunity in political advertising. It bears a clear and
there is probable cause, the COMELEC, through its Law Department, files the reasonable connection with the constitutional objectives set out in Section 26,
criminal information before the proper court. Proceedings before the proper court Article II, Section 4, Article IX-C, and Section 1, Art. XIII of the Constitution.128
demand a full-blown hearing and require proof beyond reasonable doubt to convict. Indeed, to rule otherwise would practically result in an unlimited expenditure for
A criminal conviction shall result in the disqualification of the offender, which may political advertising, which skews the political process and subverts the essence of
even include disqualification from holding a future public office. a truly democratic form of government.
xxxxxxx

M.R.A.D.C. LUMBRE 229


CONSTITUTIONAL LAW REVIEW

NOTE: and to reach out would not be meaningful if it is just a token ability to be heard by
a few. It must be coupled with substantially reasonable means by which the
Section 13 of RA 7166 provides: Authorized Expenses of Candidates and Political communicator and the audience could effectively interact. Section 9 (a) of
Parties. The aggregate amount that a candidate or registered political party may COMELEC Resolution No. 9615, with its adoption of the "aggregate-based" airtime
spend for election campaign shall be as follows: limits unreasonably restricts the guaranteed freedom of speech and of the press.
(a) For candidates Ten pesos (P10.00) for President and Vice President; and for 4. Electoral Expenditure
other candidates, Three pesos (P3.00) for every voter currently registered in the
constituency where he filed his certificate of candidacy: Provided, That, a candidate a. Garcia v. Comelec, 611 SCRA 55 (2010)
without any political party and without support from any political party may be
allowed to spend Five pesos (P5.00) for every such voter; and In this case, the COMELEC did not question petitioner’s averment that the
advertisement in question was paid for by the organization named Friends of Alvin
(b) For political parties - Five pesos (P5.00) for every voter currently registered in Garcia. The advertisement may be considered as a donation to petitioner under
the constituency or constituencies where it has official candidates. Section 4 of R.A. No. 9006 and its IRR. Paragraph 4.3, Section 4 of R.A. No. 9006
explicitly requires that “print x x x advertisements donated to the candidate or
Any provision of law to the contrary notwithstanding, any contribution in cash or in political party shall not be printed, published x x x without the written acceptance
kind to any candidate or political party or coalition of parties for campaign purposes, by the said candidate.” Since the advertisement in question was published by the
duly reported to the Commission, shall not be subject to the payment of any gift Sun Star, there arises a presumption that there was written acceptance by
tax. petitioner of the advertisement paid for or donated by his friends in the absence of
Sections 100, 101, and 103 of the OEC are not repealed by R.A. No. 7166. evidence to the contrary. Under the Rules on Evidence, it is presumed that the law
has been obeyed, and that private transactions have been fair and regular.
Non-filing of the Statement of Contributions and Expenses (SOCE) within 30 days
after the elections is also an election offense. Section 4 of R.A. No. 9006 provides for the requirements for published or printed
election propaganda, thus:
3. Equal Access to Media
Sec. 4. Requirements for Published or Printed and Broadcast Election Propaganda?
a. GMA Network v. Comelec, G.R. No. 205357, September 2, 2014 4.1. Any newspaper x x x or any published or printed political matter and any
broadcast of election propaganda by television or radio for or against a candidate
All registered parties and bona fide candidates shall have equal access to media
or group of candidates to any public office shall bear and be identified by the
time and space. (Sec. 6, R.A. No. 9006)
reasonably legible or audible words “political advertisement paid for,” followed by
the true and correct name and address of the candidate or party for whose benefit
The COMELEC went beyond the authority granted it by the law in adopting
the election propaganda was printed or aired.
"aggregate" basis in the determination of allowable airtime. The law, on its face,
does not justify a conclusion that the maximum allowable airtime should be based xxxx
on the totality of possible broadcast in all television or radio stations. This is further
buttressed by the fact that the Fair Election Act (R.A. No. 9006) actually repealed 4.3. Print, broadcast or outdoor advertisements donated to the candidate or
the previous provision of Republic Act No. 6646, which prohibited direct political political party shall not be printed, published, broadcast or exhibited without the
advertisements -the so-called "political ad ban." If under the previous law, no written acceptance by the said candidate or political party. Such written acceptance
candidate was allowed to directly buy or procure on his own his broadcast or print shall be attached to the advertising contract and shall be submitted to the COMELEC
campaign advertisements, and that he must get it through the COMELEC Time or as provided in Subsection 6.3 hereof.
COMELEC Space, R.A. No. 9006 relieved him or her from that restriction and
Paragraphs 4.1 and 4.3, Section 4 of R.A. No. 9006 are reflected in Section 13 (3) and
allowed him or her to broadcast time or print space subject to the limitations set Section 14 of COMELEC Resolution No. 6520.
out in the law. Congress, in enacting R.A. No. 9006, felt that the previous law was
not an effective and efficient way of giving voice to the people. Noting the To emphasize, Section 4 of R.A. No. 9006 requires that print advertisements donated
debilitating effects of the previous law on the right of suffrage and Philippine to a candidate shall not be published without the written acceptance of the said
democracy, Congress decided to repeal such rule by enacting the Fair Election Act. candidate, which written acceptance shall be attached to the advertising contract and
submitted to the COMELEC.
COMELEC Resolution No. 9615 on airtime limits also goes against the constitutional
guaranty of freedom of expression, of speech and of the press. The guaranty of The requirement for a written acceptance by a candidate of donated advertisements is
freedom to speak is useless without the ability to communicate and disseminate a safeguard provided by law against the danger of publishing or broadcasting election
what is said. And where there is a need to reach a large audience, the need to propaganda beyond the required frequency, size and other limitations imposed by law
access the means and media for such dissemination becomes critical. This is where without the candidate’s express agreement, since the violation of such requirements
the press and broadcast media come along. At the same time, the right to speak results in the prosecution of the candidate for an election offense punishable under the

M.R.A.D.C. LUMBRE 230


CONSTITUTIONAL LAW REVIEW

first and second paragraphs of Section 264 of the Omnibus Election Code.[23] Under (b) National government employees:
Section 264 of the Omnibus Election Code, a person found guilty of an election offense
“shall be punished with imprisonment of not less than one year but not more than six (1) DepED nonteaching personnel;
years and shall not be subject to probation.” In addition, “the guilty party shall be
sentenced to suffer disqualification to hold public office and deprivation of the right of (2) Other national government officials and employees holding regular or
suffrage.” permanent positions, excluding uniformed personnel of the Department of
National Defense and all its attached agencies;
VI. Casting and Counting of Votes
1. Automated Election System (AES) (c) Members of the Commission-accredited citizen arms or other civil society
organizations and nongovernmental organizations duly accredited by the
a. Two Types - RA 8436, as amended by RA 9369, Sec. 2 (7) and (8) Commission; and
Paper-based election system - a type of automated election system that uses paper
(d) Any registered voter of the city or municipality of known integrity and
ballots, records and counts votes, tabulates, consolidates/ canvasses and transmits
competence who is not connected with any candidate or political party.
electronically the results of the vote count;” [RA 9369, Sec. 2 (7)]
In cases where the peace and order situation so requires as determined by the
Direct recording electronic election system - a type of automated election system
Commission and where there are no qualified voters willing to serve, uniformed
that uses electronic ballots, records votes by means of a ballot display provided
personnel of the Philippine National Police shall be deputized to render election
with mechanical or electrooptical components that can be activated by the voter,
service as a last resort.
processes data by means of a computer program, records voting data and ballot
images, and transmits voting results electronically; [RA 9369, Sec. 2 (8)]

b. Some Requirements for the Use of AES - RA 8436, as amended by RA


9369, Secs. 11 (4) and (5) and 14,
Sec. 11 (4). A certification that the source code is kept, in escrow with the Bangko
Sentral ng Pilipinas.

Sec. 11 (5). A certification that the source code reviewed is one and the same as
that used by the equipment;

SEC. 14. Section 13 of Republic Act No. 8436 is hereby amended to read as follows:

‘‘SEC. 17. Ballot box. - Where applicable, there shall be in each precinct on election
day a ballot box with such safety features that the Commission may prescribe and
of such size as to accommodate the official ballots.

2. Electoral Boards
a. Can Teachers Still be Compelled to Serve in Elections? Republic Act No.
10756 or the Election Service Reform Act (April 8, 2016), Sec. 3
SECTION 3. Rendering of Election Service. – The Electoral Boards to be constituted
by the Commission shall be composed of a Chairperson and two (2) members, all
of whom shall be public school teachers who are willing and available to render
election service

Should there be a lack of public school teachers willing, available or qualified to


serve, the Commission may instead appoint the following persons in this order of
preference:

(a) Private school teachers;

M.R.A.D.C. LUMBRE 231


CONSTITUTIONAL LAW REVIEW

VII. Proclamation that (1) no voting has been held in any precint or precincts because of force
majeure, violence or terrorism and (2) that the votes not cast therein suffice to
affect the results of the elections. The language of the provision clearly requires
the concurrence of the two circumstances to justify the calling of a special election.
1. Pre-proclamation Remedies
a. Suspension of Proclamation (Pending Case for Disqualification or The Comelec concedes that what transpired in Karomatan constitutes "not merely
Cancellation of COC) – RA 6646, Sec. 6, last sentence, in relation to Sec. 7 a simple case of irregularity in the voting but a case of no voting or no
thereof election at all. However, the Comelec attributes this to "massive fraud
Section 6. Effect of Disqualification Case. - Any candidate who has been declared rather than to force majeure, violence or terrorism the — three causes
by final judgment to be disqualified shall not be voted for, and the votes cast for explicitly enumerated by section 17 (e). Unlike section 17 (d) which empowers
the Comelec to postpone the election in any political division or subdivision
him shall not be counted. If for any reason a candidate is not declared by final
whenever it finds that the holding of a free, orderly and honest election therein is
judgment before an election to be disqualified and he is voted for and receives the
rendered impossible by reason of fraud, violence, coercion, terrorism, or any other
winning number of votes in such election, the Court or Commission shall continue serious cause or causes, section 17 (e) excludes the situation where no voting has
with the trial and hearing of the action, inquiry, or protest and, upon motion of the been held because of fraud. Furthermore, doubt exists whether or not the
complainant or any intervenor, may during the pendency thereof order the irregularities committed in Karomatan properly partake of violence or terrorism.
suspension of the proclamation of such candidate whenever the evidence of his This being the case, we find that the first circumstance is not attendant.
guilt is strong.

Section 7. Petition to Deny Due Course To or Cancel a Certificate of Candidacy. - c. Pre-proclamation Controversies – OEC, Sec. 243
The procedure hereinabove provided shall apply to petitions to deny due course to Section 243. Issues that may be raised in pre-proclamation controversy. - The
or cancel a certificate of candidacy as provided in Section 78 of Batas Pambansa following shall be proper issues that may be raised in a pre-proclamation
Blg. 881. controversy:

b. Failure of Election - Usman v. Comelec, 42 SCRA 667 (1971) (a) Illegal composition or proceedings of the board of canvassers;
The broad power of the Comelec, conferred upon it the Constitution, to enforce and
(b) The canvassed election returns are incomplete, contain material defects,
administer "all laws relative to the conduct of elections" and to decide all
appear to be tampered with or falsified, or contain discrepancies in the same
administrative questions affecting elections "for the purpose of insuring free,
returns or in other authentic copies thereof as mentioned in Sections 233, 234,
orderly and honest elections," has been the key in the resolution of many pre-
235 and 236 of this Code;
proclamation controversies involving the integrity and authenticity of election
returns. Invoking the aforestated power of the Comelec, justified the action and (c) The election returns were prepared under duress, threats, coercion, or
upheld the authority of the Comelec to order the exclusion of "obviously intimidation, or they are obviously manufactured or not authentic; and
manufactured returns, or tampered returns, or returns prepared under threats and
coercion or under circumstances affecting returns' integrity and (d) When substitute or fraudulent returns in controverted polling places were
authenticity,7 emphasizing the duty of the Comelec to see to the use and inclusion canvassed, the results of which materially affected the standing of the
in the canvass of only genuine elections. aggrieved candidate or candidates.

d. Correction of Manifest Error - Chavez v. Comelec, 211 SCRA 315 (1992)


One commissioner believed that the canvass should be completed on the basis of
the valid returns from the other precincts of Lanao del Norte and that the The Court held that the petitioner’s prayer does not call for the correction of
proclamation of the third winning candidate on the basis of the said canvass should "manifest errors in the certificates of canvass or election returns" before the
logically follow; the other commissioner maintained his original view that there is Comelec but for the re-opening of the ballot boxes and appreciation of the ballots
need of a special election in Karomatan. contained therein. Indeed, petitioner has not even pointed to any "manifest error"
in the certificates of canvass or election returns he desires to be rectified. There
being none, petitioner's proper recourse is to file a regular election protest which,
A reading of section 17 (e) of Republic Act 6132 makes it apparent that Congress
under the Constitution and the Omnibus Election Code, exclusively pertains to the
has delegated to the Comelec the power to call for a special election — a power
Senate Electoral Tribunal.
essentially legislative in nature, being merely an incident to or an extension or
modality of the power to fix the date of the elections. 10 However, in the proper
exercise of the delegated power, Congress saw fit to require the Comelec ascertain

M.R.A.D.C. LUMBRE 232


CONSTITUTIONAL LAW REVIEW

While the Commission has exclusive jurisdiction over pre-proclamation protract and delay the trial of an ordinary action. (Vialogo v. COMELEC, G.R.
controversies involving local elective officials (Sec. 242, Omnibus Election Code), No. 194143, Oct. 4, 2011)
nevertheless, pre-proclamation cases are not allowed in elections for President,
Vice-President, Senator and Member of the House of Representatives. 1. COMELEC – sole judge of all contests relating to elections, returns, and
qualifications of all elective regional, provincial and city officials
Sec. 15 of Republic Act 7166 provides: (reviewable by SC under Rule 64 using Rule 65).

Sec. 15. Pre-proclamation Cases Not Allowed in Elections for President, Vice- 2. Presidential Electoral Tribunal – President and Vice President
President, Senator, and Member of the House of Representatives. — For 3. SET – Senator
purposes of the elections for President, Vice-President, Senator and Member 4. HRET – representative
of the House of Representatives, no pre-proclamation cases shall be allowed 5. RTC – over contests for municipal officials which may be appealed to
on matters relating to the preparation, transmission, receipt, custody and COMELEC
appreciation of the election returns or the certificate of canvass, as the case 6. MeTC or MTC – for barangay officials which may be appealed to RTC.
may be. However, this does not preclude the authority of the appropriate
canvassing body motu proprio or upon written complaint of an interested Grounds for the filing of election protests:
person to correct manifest errors in the certificate of canvass or election
returns before it. 1. Fraud;
2. Vote-buying;
xxx xxx xxx 3. Terrorism;
4. Presence of flying voters;
Any objection on the election returns before the city or municipal board of 5. Misreading or misappreciation of ballots;
canvassers, or on the municipal certificates of canvass before the provincial 6. Disenfranchisement of voters;
boards of canvassers or district board of canvassers in Metro Manila Area, shall 7. Unqualified members of board of election inspector; and
be specifically noted in the minutes of their respective proceedings. 8. Other election irregularities.

It is clear from the above-quoted provision of the law that "pre-proclamation cases Pendency of election protest is not sufficient basis to enjoin the protestee from
(are) not allowed in elections for President, Vice-President, Senator and Member of assuming office.
the House of Representatives.'' What is allowed is the correction of "manifest errors
in the certificate of canvass or election returns." To be manifest, the errors must Content of an election protest
appear on the face of the certificates of canvass or election returns sought to be
corrected and/or objections thereto must have been made before the board of It must be initiated by filing a protest that must contain the following
canvassers and specifically noted in the minutes of their respective proceedings. allegations:
a. The protestant is a candidate who duly filed a COC and was voted for
2. Post-proclamation Remedies in the election;
b. The protestee has been proclaimed; and
Disputes which arise or are instituted after proclamation of winning candidates and c. The petition was filed within ten (10) days after the proclamation. (Miro
which issues pertain to the casting and counting of votes (election protests), or to the v. COMELEC, G.R. No. L-57574, April 20, 1983)
eligibility or disloyalty of the winning candidates (quo warranto).
a. Election Protest Effect if the protestant accepts a permanent appointment

It is a special summary proceeding the object of which is to expedite the Acceptance of a permanent appointment to a regular office during the
settlement of controversies between candidates as to who received the pendency of his protest is an abandonment of the electoral protest. The same
majority of legal votes. is true if a protestant voluntarily sought election to an office whose term would
extend beyond the expiry date of the term of the contested office, and after
Statutes providing for election contests are to be liberally construed to the end winning the said election, took her oath and assumed office and there after
that the will of the people in the choice of public officers may not be defeated continuously serves it. The reason for this is that the dismissal of the protest
by mere technical objections. It is imperative that his claim be immediately would serve public interest as it would dissipate the aura of uncertainty as to
cleared not only for the benefit of the winner but for the sake of public interest, the results of the presidential election, thereby enhancing the all-to crucial
which can only be achieved by brushing aside technicalities of procedure which political stability of the nation during this period of national recovery. (Santiago
v. Ramos, P.E.T. Case No. 001, Feb. 13, 1996)

M.R.A.D.C. LUMBRE 233


CONSTITUTIONAL LAW REVIEW

In assuming the office of Senator, one has effectively abandoned or withdrawn the Constitution. Quo warranto proceedings against any regional, provincial or city
this protest. Such abandonment or withdrawal operates to render moot the officials are brought before the COMELEC. Quo warranto proceedings against
instant protest. Moreover, the dismissal of this protest would serve public municipal officials and barangay officials are brought before the RTCs and MTCs
interest as it would dissipate the aura of uncertainty as to the results of the respectively.
election. (Legarda v. De Castro, PET case no. 003, January 18, 2008)
i. Election Protest vs. Quo Warranto – Luison v. Garcia, 103 Phil 453
Requisites for an execution pending appeal in election protest cases (1958)
1. It must be upon motion by the prevailing party with notice to the As this Court has held, "The general rule is that the fact a plurality or a majority
adverse party; of the votes are cast for an ineligible candidate at a popular election does not
2. There must be good reasons for the said execution; and entitle the candidate receiving the next highest number of votes to be declared
3. The order granting the said execution must state the good reasons elected. In such case the electors have failed to make a choice and the election
(Navarosa v. COMELEC, G.R. No. 157957, Sept. 18, 2003) is a nullity" (Llamoso vs. Ferrer, et al., 84 Phil., 490). In a subsequent case,
this Court also said that where the winning candidate has been declared
“Good reasons” ineligible, the person who obtained second place in the election cannot be
A combination of two or more of the following: declared elected since our law not only does not contain an express provision
1. That public interest is involved or the will of the electorate; authorizing such declaration but apparently seems to prohibit it (Villar vs.
2. The shortness of the remaining portion of the term of the contested Paraiso, 96 Phil., 659: See also Nuval vs. Guray, 52 Phil., 654 and Topacio vs.
office; Paredes, 23 Phil., 238).
3. The length of time that the election contest has been pending (Ramas
v. COMELEC, G.R. No. 130831. Feb. 10, 1998). Moreover, a protest to disqualify a protestee on the ground of ineligibility is
different from that a protest based on frauds and irregularities where it may
If instead of issuing a preliminary injunction in place of a TRO, a court opts to be shown that protestant was the one really elected for having obtained a
decide the case on its merits with the result that it also enjoins the same acts plurality of the legal votes. In the first case, while the protestee may be ousted
covered by its TRO, it stands to reason that the decision amounts to a grant of the protestant will not be seated; in the second case, the protestant may
preliminary injunction. Such injunction should be deemed in force pending any assume office after protestee is unseated. The first case is brought to court by
appeal from the decision. The view that execution pending appeal should still a petition of quo warranto, while the second by instituting an election protest.
continue notwithstanding a decision of the higher court enjoining such Thus, the Supreme Court, in defining these two remedies, said:
execution—does not make sense. It will render quite inutile the proceedings
All election disputes may be divided into two distinct classes: (1) those which
before such court. (Panlilio v. COMELEC, G.R. No. 184286, Feb. 26, 2010)
pertain to the casting and counting of the ballots; and (2) those which pertain
to the eligibility of the candidates. If there be cases incapable of being so
Best pieces of evidence in an election contest
classified, they have not been suggested.
1. Ballots are the best and most conclusive evidence in an election contest
where the correctness of the number of votes of each candidate is involved Considering the fundamental difference existing between the nature of a
(Delos Reyes, G.R. No. 170070, Feb. 28, 2007); and petition for quo warranto and that of an election protest, it may be said that a
2. Election returns are the best evidence when the ballots are lost, candidate who files a protest against one who has been proclaimed as having
destroyed, tampered or fake. received the highest number of votes basing his protest cannot disguise his
action so as to make his protest a justification to be seated in office. In other
Right to withdraw words, he cannot convert an action for quo warranto into an election protest.
A protestant has the right to withdraw his protest or drop polling places from This is because these two cases are fundamentally different in nature and in
his protest. The protestee, in such cases, has no cause to complain because purpose. In quo warranto, "there is not, strictly speaking, a contest, and the
the withdrawal is the exclusive prerogative of the protestant. wreath of victory cannot be transferred from an ineligible candidate to any
other candidate", while in a protest, "the question is as to who received a
b. Quo Warranto
plurality of the legally cast (Topacio vs. Paredes, supra). The present action
Refers to an election contest relating to the qualifications of an elective official on therefore, partakes of the nature of quo warranto and as such has no reason
the ground of (1) ineligibility or (2) disloyalty to the Republic of the Philippines. The to exist. This question is already involved in the other case (G.R. No. L-10916).
issue is whether respondent possesses all the qualifications and none of the
disqualifications prescribed by law. (A.M. No. 07-4-15-SC, May 15, 2007)
Quo warranto proceedings against a Congressman-elect, Senator-elect, President-
elect and VP-elect are brought before the appropriate electoral tribunals created by

M.R.A.D.C. LUMBRE 234


CONSTITUTIONAL LAW REVIEW

Election Protest Quo Warranto NOTE: Coercion of subordinates as an election offense [OEC, Sec. 261(d)]
(Section 253) has been expressly repealed by Sec. 2, R.A. 7890 and the express repeal
has been affirmed by SC in Javier v. COMELEC, G.R. No. 215847, January
As to By a losing candidate By any registered
12, 2016.
Who May for the same office voter in the
File for which the winner constituency where 5. Threats, intimidation, terrorism, use of fraudulent device or other
filed his COC. the winning forms of coercion;
candidate sought to
be disqualified ran for 6. Coercion of election officials and employees;
office. 7. Appointment of new employees, creation of new position, promotion,
As to Who received the Whether the giving of salary increases;
Dispute/ majority or plurality candidate who was
Issue of the votes which proclaimed and 8. Intervention of public officers and employees;
were legally cast? elected should be
9. Undue influence;
Whether there were disqualified because
irregularities in the of ineligibility or 10. Unlawful electioneering;
conduct of the disloyalty to the
election which Republic. 11. Carrying firearms outside the residence or place of business; and
affected its results.
12. Use of armored land, water or aircraft (OEC. Sec. 261)
Effect of filing an election protest or a petition for quo warranto Prescriptive period of election offenses
Generally, it bars the subsequent filing of a pre-proclamation controversy or a 5 years from the date of their commission (OEC, Sec. 267)
petition to annul proclamation. It also amounts to the abandonment of one
filed earlier, thus, depriving the COMELEC of the authority to inquire into and Jurisdiction to investigate and prosecute election offenses
pass upon the title of the protestee or the validity of his proclamation. Once
the competent tribunal has acquired jurisdiction over an election protest or a According to Sec. 2 (6), Article IX-C of the 1987 Constitution, the COMELEC
petition for quo warranto, all questions relative thereto will have to be decided has jurisdiction to investigate and prosecute cases involving violations of
in the case itself and not in another proceeding. (Villamor v. COMELEC, G.R. election laws, but it may delegate the power to the Provincial prosecutor
No. 169865, July 21, 2006) (People v. Judge Basilia, G.R. Nos. 83938-40, November 6, 1989). The
COMELEC shall, through its duly authorized legal officers, have the power,
ii. Summary of Jurisdiction of Courts concurrent with the other prosecuting arms of the government, to conduct
preliminary investigation of all election offenses punishable under this Code,
Authority to prosecute election offenses and prosecute the same. (R.A. 9369, Sec. 43)
DOJ and COMELEC exercise concurrent jurisdiction in conducting preliminary Jurisdiction to try and decide violation of election laws
investigation of election offenses. The grant of exclusive power to investigate
and prosecute cases of election offenses to the COMELEC was not by virtue of General Rule: The RTC has the exclusive and original jurisdiction to hear and
the Constitution but by the OEC which was eventually amended by Sec. 43 of decide any criminal action or proceedings for violation of the OEC.
RA 9369. Thus, the DOJ now conducts preliminary investigation of election
offenses concurrently with the COMELEC and no longer as mere deputies (Jose Exception: The MTC has jurisdiction over offenses relating to failure to
Miguel T. Arroyo v. DOJ, et al., G.R. No. 199082, Sept. 18, 2012). register or failure to vote. (OEC, Sec 267)

Prosecution of election offenses c. Annulment of Election and Proclamation - Frivaldo v. Commission on


Elections, 174 SCRA 245 [1989]
Election offenses are prohibited acts such as:
The argument that the petition filed with the Commission on Elections should be
1. Vote buying and vote selling (1991 Bar); dismissed for tardiness is not well-taken. The herein private respondents are
seeking to prevent Frivaldo from continuing to discharge his office of governor
2. Conspiracy to bribe voters; because he is disqualified from doing so as a foreigner. Qualifications for public
3. Wagering upon result of election; office are continuing requirements and must be possessed not only at the time of
appointment or election or assumption of office but during the officer's entire
4. Coercion of subordinates; tenure. Once any of the required qualifications is lost, his title may be seasonably
challenged. If, say, a female legislator were to marry a foreigner during her term

M.R.A.D.C. LUMBRE 235


CONSTITUTIONAL LAW REVIEW

and by her act or omission acquires his nationality, would she have a right to remain
in office simply because the challenge to her title may no longer be made within
ten days from her proclamation? It has been established, and not even denied, that
the evidence of Frivaldo's naturalization was discovered only eight months after his
proclamation and his title was challenged shortly thereafter.
This Court will not permit the anomaly of a person sitting as provincial governor in
this country while owing exclusive allegiance to another country. The fact that he
was elected by the people of Sorsogon does not excuse this patent violation of the
salutary rule limiting public office and employment only to the citizens of this
country. The qualifications prescribed for elective office cannot be erased by the
electorate alone. The will of the people as expressed through the ballot cannot cure
the vice of ineligibility, especially if they mistakenly believed, as in this case, that
the candidate was qualified. Obviously, this rule requires strict application when
the deficiency is lack of citizenship. If a person seeks to serve in the Republic of
the Philippines, he must owe his total loyalty to this country only, abjuring and
renouncing all fealty and fidelity to any other state.

M.R.A.D.C. LUMBRE 236


CONSTITUTIONAL LAW REVIEW

CHAPTER 3: ADMINISTRATIVE LAW Administrative agencies have powers and functions which may be administrative,
investigatory, regulatory, quasi-legislative, or quasi-judicial or mix of the five, as may
be conferred by the constitution or by the statute. They have in fine only such powers
or authority as are granted or delegated, expressly or impliedly, by law. And in
I. General Principles determining whether an agency has certain powers, the inquiry should be from the law
itself. But once ascertained as existing, the authority given should be liberally
construed. (Soriano v. MTRCB, G.R. No. 165785, April 29, 2009)
It is a branch of public law fixing the organization and determines the competence of
Instrumentality
administrative authorities, and indicates the individual remedies for the violation of the
rights. [Administrative Code, Sec. 2(3)] It refers “to any agency of the National Government, not integrated within the
Scope: department framework, vested with special functions or jurisdiction by law, endowed
with some if not all corporate powers, administering special funds and enjoying
1. Fixes the administrative operation and structure of the government operational autonomy, usually through a charter. It includes regulatory agencies,
2. Executes or enforces that which is entrusted to administrative authorities (all those chartered institutions and government-owned or controlled corporations.” (United
public officers and organs of the government charged with the amplification, application Residents of Dominican Hills v. Commission on the Settlement of Land Problems, G.R.
and execution of the law) No. 135945, March 7, 2001)
3. Governs public officers and creates administrative officers Agency
4. Provides remedies to those aggrieved by these agencies Any of the various units of the government, including a department, bureau, office,
5. Governs Judicial Review instrumentality, or government-owned or controlled corporations, or a local government
or a distinct unit therein. (Administrative Code, Sec. 2)
6. Includes rules, regulation, orders and decisions made by administrative authorities
7. Includes the body of judicial doctrines on any of the above Department
An executive department created by law. [Administrative Code of 1987, Sec. 2(7)]

Kinds: Bureau

1. Statutes setting up administrative authorities. Any principal subdivision or unit of any department. [Administrative Code, Sec. 2(8)]

2. Body of doctrines and decisions dealing with the creation, operation, and effect of Office
determinations and regulations of such administrative authorities.
It refers to any major functional unit of a department or bureau including regional
3. Rules, regulations, or orders of such administrative authorities in pursuance of the offices. It may also refer to any position held or occupied by individual persons, whose
purposes, for which administrative authorities were created or endowed. functions are defined by law or regulation [Administrative Code, Sec. 2(9)].
Example: Omnibus Rules Implementing the Labor Code, circulars of Central
Monetary Authority
1. Manner of creation
4. Determinations, decisions, and orders of such administrative authorities in the
settlement of controversies arising in their particular field. a. Constitutional provision
Example: Awards of NLRC with respect to money claims of employees b. Authority of law
c. Legislative enactment
II. Administrative Agencies Creation and Abolition of Office

The creation and abolition of public offices is primarily a legislative function (Eugenio
v. CSC, G.R. No. 15863, March 31, 1995). However, the President may abolish an
Definition: It is an organ of government, other than a court and the legislature, which office either from a valid delegation from Congress, or his inherent duty to faithfully
affects the rights of private parties either through adjudication or rule making. execute the laws. (Biraogo v. Philippine Truth Commission of 2010, G.R. No. 192935,
Interpretation of the powers of the administrative agencies: December 7, 2010)

M.R.A.D.C. LUMBRE 237


CONSTITUTIONAL LAW REVIEW

Elements of a valid abolition of office: 6. Those agencies set up to function in situations where the government seeks to
adjust individual controversies because of strong social policy involved.
1. In good faith; (good faith is presumed) Example: NLRC, ECC, SEC
2. Not for political or personal reasons; and 7. Those with investigative power. Example: DOJ, NBI
3. Not in violation of law.
NOTE: The Congress has the right to abolish an office even during the term for which III. Powers of Administrative Agencies/Bodies
an existing incumbent may have been elected EXCEPT when restrained by the
Constitution. (Read: Alliance for the Family Foundation vs. Garin, G.R. No. 217872, 26 April
2017)
Reorganization
The powers of an administrative body are classified into two fundamental powers: quasi-
Reorganization involves the reduction of personnel, consolidation of offices, or abolition legislative and quasi-judicial. Quasi-legislative power, otherwise known as the power of
thereof by reason of economy or redundancy of functions. It alters the existing structure subordinate legislation, has been defined as the authority delegated by the lawmaking
of government offices or the units therein, including the lines of control, authority and body to the administrative body to adopt rules and regulations intended to carry out
responsibility between them to make the bureaucracy more responsive to the needs of the provisions of law and implement legislative policy. 18 "[A] legislative rule is in the
the public clientele as authorized by law. (Pan v. Pena G.R. No. 174244, Feb. 13, 2009) nature of subordinate legislation, designed to implement a primary legislation by
Circumstances that may be considered as evidence of bad faith in a removal pursuant providing the details thereof." The exercise by the administrative body of its quasi-
to reorganization, thus warranting reinstatement or reappointment: legislative power through the promulgation of regulations of general application does
not, as a rule, require notice and hearing. The only exception being where the
1. Where there is a significant increase in the number of positions in the new Legislature itself requires it and mandates that the regulation shall be based on certain
staffing pattern of the department or agency concerned; facts as determined at an appropriate investigation.
2. Where an office is abolished and other performing substantially the same Quasi-judicial power, on the other hand, is known as the power of the administrative
functions is created; agency to determine questions of fact to which the legislative policy is to apply, in
3. Where incumbents are replaced by those less qualified in terms of status of accordance with the standards laid down by the law itself. As it involves the exercise of
appointment, performance and merit; discretion in determining the rights and liabilities of the parties, the proper exercise of
quasi-judicial power requires the concurrence of two elements: one, jurisdiction which
4. Where there is reclassification of offices in the department or agency concerned
must be acquired by the administrative body and two, the observance of the
and the classified offices perform substantially the same function as the original
requirements of due process, that is, the right to notice and hearing.
offices;
5. Where the removal violates the order of separation provided in Sec. 3 of R.A. On the argument that the certification proceedings were conducted by the FDA in the
6656 (Cotiangco v. Province of Biliran, G.R. No. 157139, Oct. 19, 2011) exercise of its "regulatory powers" and, therefore, beyond judicial review, the Court
holds that it has the power to review all acts and decisions where there is a commission
2. Kinds of grave abuse of discretion. No less than the Constitution decrees that the Court must
exercise its duty to ensure that no grave abuse of discretion amounting to lack or excess
1. Those created to function in situations where the government offers gratuity,
of jurisdiction is committed by any branch or instrumentality of the Government. Such
grant, or special privilege. Example: GSIS, SSS, PAO
is committed when there is a violation of the constitutional mandate that "no person is
2. Those set up to function in situations where the government seeks to carry on deprived of life, liberty, and property without due process of law." The Court's power
certain functions of government. Example: BIR, BOC, BOI cannot be curtailed by the FDA's invocation of its regulatory power.
3. Those set up in situations where the government performs business service for In so arguing, the respondents cited Atty. Carlo L. Cruz in his book, Philippine
the public. Example: PNR, MWSS, NFA, NHA Administrative Law.
4. Those set up to function in situations where the government seeks to regulate
Lest there be any inaccuracy, the relevant portions of the book cited by the respondents
businesses imbued with public interest. Example: Insurance Commission, LTFRB,
are hereby quoted as follows:
NTC
5. Those set up to function in situations where the government seeks under the xxx.
police power to regulate private businesses and individuals. Example: SEC, B. The Quasi-Judicial Power
MTRCB
xxx

M.R.A.D.C. LUMBRE 238


CONSTITUTIONAL LAW REVIEW

2. Determinative Powers are not strictly applied. Ledesma v. Court of Appeals elaborates on the well-
established meaning of due process in administrative proceedings in this wise:
To better enable the administrative body to exercise its quasi-judicial authority,
it is also vested with what is known as determinative powers and functions. x x x Due process, as a constitutional precept, does not always and in all situations
require a trial-type proceeding. Due process is satisfied when a person is notified
Professor Freund classifies them generally into the enabling powers and the of the charge against him and given an opportunity to explain or defend himself.
directing powers. The latter includes the dispensing, the examining, and the In administrative proceedings, the filing of charges and giving reasonable
summary powers. opportunity for the person so charged to answer the accusations against him
The enabling vowers are those that permit the doing of an act which the constitute the minimum requirements of due process. The essence of due process
law undertakes to regulate and which would be unlawful with government is simply to be heard, or as applied to administrative proceedings, an opportunity
approval. The most common example is the issuance of licenses to engage in a to explain one's side, or an opportunity to seek a reconsideration of the action or
particular business or occupation, like the operation of a liquor store or restaurant. ruling complained of.
x x x. xxxxxxxxxxxxxxxx
From the above, two things are apparent: one, the "enabling powers" cover "regulatory Incidentally, Section 32 of R.A. No. 3720 and Section 9 of Executive Order (E.O.)
powers" as defined by the respondents; and two, they refer to a subcategory of a quasi- No. 247 provide that any decision by the FDA would then be appealable to the
judicial power which, as explained in the Decision, requires the compliance with the twin Secretary of Health, whose decision, in tum, may be appealed to the Office of the
requirements of notice and hearing. Nowhere from the above-quoted texts can it be President (OP). Thus:
inferred that the exercise of "regulatory power" places an administrative agency beyond
the reach of judicial review. When there is grave abuse of discretion, such as denying a Sec. 32. The orders, rulings or decisions of the FDA shall be appealable
party of his constitutional right to due process, the Court can come in and exercise its to the Secretary of Health. - An appeal shall be deemed perfected upon
power of judicial review. It can review the challenged acts, whether exercised by the filing of the notice of appeal and posting of the corresponding appeal bond.
FDA in its ministerial, quasi-judicial or regulatory power. In the past, the Court exercised
its power of judicial review over acts and decisions of agencies exercising their An appeal shall not stay the decision appealed from unless an order from the
regulatory powers, such as DPWH, TRB, NEA, and the SEC, among others. In Diocese Secretary of Health is issued to stay the execution thereof.
of Bacolod v. Commission on Elections, the Court properly exercised its power of judicial Sec. 9. Appeals. - Decisions of the Secretary (DENR, DA, DOH or DOST)
review over a Comelec resolution issued in the exercise of its regulatory power. may be appealed to the Office of the President. Recourse to the courts
xxxxxxxxxxxx shall be allowed after exhaustion of all administrative remedies.

The Court is of the view that the FDA need not conduct a trial-type hearing. Indeed, In view thereof, the Court should modify that part of the Decision which allows
due process does not require the conduct of a trial-type hearing to satisfy its direct appeal of the FDA decision to the Court of Appeals. As stated in the said
requirements. All that the Constitution requires is that the FDA afford the people their decision, the FDA decision need not be appealed to the Secretary of Health because
right to due process of law and decide on the applications submitted by MAHs after she herself is a party herein. Considering that the Executive
affording the oppositors like the petitioners a genuine opportunity to present their NOTE: In this case, the Supreme Court held that licensing is considered an exercise of
science-based evidence. As earlier pointed out, this the FDA failed to do. It simply quasi-judicial power. As such, notice and hearing is required.
ignored the opposition of the petitioners. In the case of Perez, et al. v. Philippine
Telegraph and Telephone Company, et al., it was stated that: 1. Discretionary – the law imposes a duty upon a public officer, and gives him the
right to decide how orwhen the duty shall be performed.
A formal trial-type hearing is not even essential to due process. It is enough that the
parties are given a fair and reasonable opportunity to explain their respective sides of 2. Ministerial – one which is as clear and specific as to leave no room for the exercise
the controversy and to present supporting evidence on which a fair decision can be of discretion in its performance.
based.
1. Quasi-legislative (rule-making) power or subordinate legislation.
In the fairly recent case of Vivo v. Pagcor, the Court explained:
The exercise of delegated legislative power, involving no discretion as to what the law
The observance of fairness in the conduct of any investigation is at the very heart shall be, but merely the authority to fix the details in the execution or enforcement of
of procedural due process. The essence of due process is to be heard, and, as a policy set out in the law itself.
applied to administrative proceedings, this means a fair and reasonable opportunity
Limitations to the exercise of quasi-legislative power
to explain one's side, or an opportunity to seek a reconsideration of the action or
ruling complained of. Administrative due process cannot be fully equated 1. Within the limits of the powers granted to administrative agencies.
with due process in its strict judicial sense, for in the former a formal or
trial-type hearing is not always necessary, and technical rules of procedure

M.R.A.D.C. LUMBRE 239


CONSTITUTIONAL LAW REVIEW

2. Cannot make rules or regulations which are inconsistent with the provision of the The administrative rule goes beyond merely providing for the means that can
Constitution or statute. facilitate or render least cumbersome the implementation of the law but
substantially adds to or increases the burden of those governed. (CIR v. CA, G.R.
3. Cannot defeat the purpose of the statute. No. 11976, August 26, 1996)
4. May not amend, alter, modify, supplant, enlarge, or limit the terms of the statute. Publication Requirement
5. A rule or regulation must be uniform in operation, reasonable and not unfair or Required as a condition precedent to the effectivity of a law to inform the public of
discriminatory. the contents of the law or rules and regulations before their rights and interests
Administrative rule are affected by the same. (Philippine International Trading Corporation v. COA,
G.R. No. 132593, June 25, 1999)
Any agency statement of general applicability, which implements or interprets a law
fixes and describes procedures in, or practice requirements of, an agency, including its NOTE: If not otherwise required by law, an agency shall, as far as practicable,
regulations. The term includes memoranda or statements concerning the internal publish or circulate notices of proposed rules and afford interested parties the
administration or management of an agency not affecting the rights of, or procedure opportunity to submit their views prior to the adoption of any rule. [1987
available to the public. [Administrative Code of 1987, Sec. 2 (2)] Administrative Code, Administrative Procedure, Sec. 9(1)]

Source of the power to promulgate administrative rules and regulations Requisites for a valid delegation of quasi-legislative or rule-making power

Derived from the legislature, by virtue of a valid delegation, either express or implied. 1. Completeness Test - The statute is complete in itself, setting forth the policy to
be executed by the agency
Doctrine of Subordinate Legislation
2. Sufficient Standard Test - Statute fixes a standard, mapping out the boundaries
Power of administrative agency to promulgate rules and regulations on matters within of the agency’s authority to which it must conform. It lays down a sufficient
their own specialization. standard when it provides adequate guidelines or limitations in the law to map out
the boundaries of the delegate’s authority and prevent the delegation from running
Reason behind the delegation
riot. To be sufficient, the standard must specify the limits of the delegate’s
It is well established in this jurisdiction that, while the making of laws is a non-delegable authority, announce the legislative policy and identify the conditions under which
activity that corresponds exclusively to Congress, nevertheless the latter may it is to be implemented. (ABAKADA Guro Party List v. Purisima, G.R. No. 166715,
constitutionally delegate authority to promulgate rules and regulations to implement a August 14, 2008)
given legislation and effectuate its policies, for the reason that the legislature often finds
2. Quasi-judicial power (Determinative powers)
it impracticable (if not impossible) to anticipate and provide for the multifarious and
complex situations that may be met in carrying the law into effect. All that is required Power of administrative authorities to make determinations of facts in the performance
is that: of their official duties and to apply the law as they construe it to the facts so found. It
partakes the nature of judicial power, but exercised by a person other than a judge.
(1) the regulation should be germane to the objects and purposes of the law;
Limited jurisdiction of quasi-judicial agencies:
(2) that the regulation be not in contradiction with it, but conforms to the standards
that the law prescribes (People of the Philippines v. Exconde, G.R. No. L-9820, An administrative body could wield only such powers as are specifically granted to it by
August 30, 1957) its enabling statute. Its jurisdiction is interpreted strictissimi juris.
i. Notice and hearing required? Conditions for the Proper Exercise of Quasi-Judicial Power:
General Rule: An administrative body need not comply with the requirements of 1. Jurisdiction must be properly acquired by the administrative body;
notice and hearing, in the performance of its executive or legislative functions, such
as issuing rules and regulations. (Corona v. United Harbor Pilots Association of the 2. Due process must be observed in the conduct of the proceedings.
Philippines, G.R. No. 111963, December 12, 1997) a. Dispensing powers – Notice and hearing required?
Exceptions: Exemplified by the authority to exempt from or relax a general prohibition, or
The legislature itself requires it and mandates that the regulation shall be based on authority to relieve from an affirmative duty. Its difference from licensing power is
certain facts as determined at an appropriate investigation. (Hon. Executive that dispensing power sanctions a deviation from a standard.
Secretary v. Southwing Heavy Industries, Inc., G.R. No. 164171, August 22, 2006) As it is an exercise of a quasi-judicial power, notice and hearing is required.

b. Enabling powers (Regulatory powers) – Notice and hearing required?

M.R.A.D.C. LUMBRE 240


CONSTITUTIONAL LAW REVIEW

Permits the doing of an act which the law undertakes to regulate and which would due process is the denial of the opportunity to be heard. (Flores v. Montemayor,
be unlawful without governmental orders. It is characterized by the grant or denial G.R. No. 170146, June 6, 2011)
of permit or authorization. Example: Issuance of licenses to engage in a particular
business. Notice and Hearing, Effect of Non-observance –

As mentioned earlier, notice and hearing is required. As a rule, it will invalidate the administrative proceedings. A failure to comply with
the requirements may result in a failure to acquire jurisdiction.
Other Classifications of Adjudicatory Powers:
NOTE: Right to notice may be waived.
3. Directing powers – Orders the doing or performing of particular acts to
ensure the compliance with the law and are often exercised for corrective purposes. Necessity of Notice and Hearing
Examples: public utility commissions, powers of assessment under the revenue A hearing may take place after the deprivation occurs. What the law prohibits is
laws, reparations under public utility laws, and awards under workmen’s not the absence of previous notice but the absolute absence thereof and the lack
compensation laws, and powers of abstract determination such as definition- of opportunity to be heard.
valuation, classification and fact finding
NOTE: There has been no denial of due process if any irregularity in the premature
4. Summary powers – Apply compulsion or force against person or property to issuance of the assailed decision has been remedied by an order giving the petitions
effectuate a legal purpose without a judicial warrant to authorize such action. the right to participate in the hearing of the MR. The opportunity granted by,
Examples: Abatement of nuisance, summary restraint, levy of property of technically, allowing petitioners to finally be able to file their comment in the case,
delinquent taxpayers resolves the procedural irregularity previously inflicted upon petitioners. (Nasecore
5. Equitable powers – The power to determine the law upon a particular state v. ERC, G.R. No. 190795, July 6, 2011)
of facts that has the right to, and must, consider and make proper application of Exceptions to the Requirement of Notice and Hearing
the rules of equity. Examples: Power to appoint a receiver, power to issue
injunctions 1. Urgency of immediate action;

6. Examining powers – This is also called as investigatory power. Requires 2. Tentativeness of administrative action;
production of books, papers, etc., and the attendance of witnesses and compelling 3. Grant or revocation of licenses or permits to operate certain businesses
the testimony. affecting public order or morals;
Cardinal Requirements of Due Process in Administrative Proceedings 4. Summary abatement of nuisance per se which affects safety of persons or
(HEDS-RIK) property;

1. Right to a hearing which includes the right to present one’s case and submit 5. Preventive suspension of public officer or employee facing administrative
evidence in support thereof. charges;
6. Cancellation of a passport of a person sought for criminal prosecution;
2. The tribunal must consider the evidence presented.
7. Summary proceedings of distraint and levy upon property of a delinquent
3. The decision must be supported by evidence. taxpayer;
4. Such evidence must be substantial. 8. Replacement of a temporary or acting appointee;
5. The decision must be rendered on the evidence presented at the hearing or at 9. Right was previously offered but not claimed.
least contained in the record, and disclosed to the parties affected.
3. Fact-finding, investigative, licensing and rate-fixing powers
6. The tribunal or body or any of its judges must act on its own independent
consideration of the law and facts of the controversy in arriving at a decision. Fact-finding Power:

7. The board or body should render decision in such a manner that parties can a) Power to declare the existence of facts which call into operation the provisions
know the various issues involved and the reasons for the decision rendered. (Ang of a statute;
Tibay v. CIR, G.R. No. L-46496, February 27, 1940). b) Power to ascertain and determine appropriate facts as a basis for procedure in
NOTE: The essence of due process in administrative proceedings is the opportunity the enforcement of particular laws
to explain one’s side or seek a reconsideration of the action or ruling complained NOTE: The mere fact that an officer is required by law to inquire the existence of certain
of. As long as the parties are given the opportunity to be heard before judgment is facts and to apply the law thereto in order to determine what his official conduct shall
rendered, the demands of due process are sufficiently met. What is offensive to

M.R.A.D.C. LUMBRE 241


CONSTITUTIONAL LAW REVIEW

be does not affect private rights do not constitute an exercise of judicial powers. (Lovina License
v. Moreno, G.R. No. L-17821, November 21, 1963)
Includes the whole or any part of any agency’s permit, certificate, passport, clearance,
Exceptions to the Rule that Findings of Facts of Administrative Agencies are approval, registration, charter, membership, statutory exemption or other form of
Binding on the Courts: permission, or regulation of the exercise of a right or privilege. [1987 Administrative
Code, Sec. 2(10), Administrative Procedure]
1. Findings are vitiated by fraud, imposition, or collusion;
Licensing
2. Procedure which led to factual findings is irregular;
Includes agency process involving the grant, renewal, denial, revocation, suspension,
3. Palpable errors are committed; annulment, withdrawal, limitation, amendment, modification or conditioning of a
4. Factual findings not supported by evidence; license. [1987 Administrative Code, Sec. 2(11), Administrative Procedure]

5. Grave abuse of discretion, arbitrariness, or capriciousness is manifest; NOTE: Except in cases of willful violation of pertinent laws, rules and regulations or
when public security, health, or safety requires otherwise, no license may be withdrawn,
6. When expressly allowed by statute; suspended, revoked or annulled without notice and hearing. [1987 Administrative Code,
Sec. 17(2), Administrative Procedure]
7. Error in appreciation of the pleadings and in the interpretation of the
documentary evidence presented by the parties. Nature of an administrative agency’s act if it is empowered by a statute to
revoke a license for noncompliance or violation of agency regulations
Fact-finding Quasi-judicial Body
Where a statute empowers an agency to revoke a license for non-compliance with or
A fact-finding quasi-judicial body (e.g., Land Transportation Franchising and Regulatory
violation of agency regulations, the administrative act is of a judicial nature, since it
Board) whose decisions (on questions regarding certificate of public convenience) are
depends upon the ascertainment of the existence of certain past or present facts upon
influenced not only by the facts as disclosed by the evidence in the case before it but
which a decision is to be made and rights and liabilities determined.
also by the reports of its field agents and inspectors that are periodically submitted to
it, has the power to take into consideration the result of its own observation and Rate
investigation of the matter submitted to it for decision, in connection with other
evidence presented at the hearing of the case. (Pantranco South Express, Inc. v Board It means any charge to the public for a service open to all and upon the same terms,
of Transportation, G.R. No. L-49664, November 22, 1990) including individual or joint rates, tolls, classification or schedules thereof, as well as
communication, mileage, kilometrage and other special rates which shall be imposed
Investigatory Power by law or regulation to be observed and followed by a person. [1987 Administrative
Code, Administrative Procedure, Sec. 2(3)]
Power to inspect, secure, or require the disclosure of information by means of accounts,
records, reports, statements and testimony of witnesses. It is implied and not inherent Rate-fixing power
in administrative agencies.
Power usually delegated by the legislature to administrative agencies for the latter to
Power to issue subpoena not inherent in administrative bodies fix the rates which public utility companies may charge the public.
It is settled that these bodies may summon witnesses and require the production of NOTE: The power to fix rates is essentially legislative but may be delegated. (Philippine
evidence only when duly allowed by law, and always only in connection with the matter Inter-Island v. CA, G.R. No. 100481, Jan. 22, 1997)
they are authorized to investigate.
The legislature may directly provide for these rates, wages, or prices. But while the
Power to cite a person in contempt not inherent in administrative bodies legislature may deal directly with these subjects, it has been found more advantageous
to place the performance of these functions in some administrative agency. The need
It must be expressly conferred upon the body, and additionally, must be used only in
for dispatch, for flexibility and technical know-how is better met by entrusting the rate-
connection with its quasi-judicial as distinguished from its purely administrative or
fixing to an agency other than the legislature itself. (Cortes, 1963)
routinary functions.
Rate-fixing Procedure:
NOTE: If there is no express grant, the agency must invoke the aid of the RTC under
Rule 71 of the Rules of Court. The administrative agencies perform this function either by issuing rules and regulations
in the exercise of their quasi-legislative power or by issuing orders affecting a specified
Licensing power
person in the exercise of its quasi-judicial power.
The action of an administrative agency in granting or denying, or in suspending or
NOTE: In the fixing of rates, no rule or final order shall be valid unless the proposed
revoking, a license, permit, franchise, or certificate of public convenience and necessity.
rates shall have been published in a newspaper of general circulation at least 2 weeks

M.R.A.D.C. LUMBRE 242


CONSTITUTIONAL LAW REVIEW

before the first hearing thereon. [1987 Administrative Code, Administrative Procedure, IV. Judicial Recourse and Review
Sec. 9(2)]

Requirements for the delegation of the power to ascertain facts to be valid:


Judicial Review
The law delegating the power to determine some facts or state of things upon which
the law may take effect or its operation suspended must provide the standard, fix the Re-examination or determination by the courts in the exercise of their judicial power in
limits within which the discretion may be exercised, and define the conditions therefor. an appropriate case instituted by a party aggrieved thereby as to whether the
Absent these requirements, the law and the rules issued thereunder are void, the former questioned act, rule, or decision has been validly or invalidly issued or whether the
being an undue delegation of legislative power and the latter being the exercise of rule- same should be nullified, affirmed or modified.
making without legal basis. (U.S. v. Ang Tang Ho, G.R. No. L-17122, February 27, 1992)
It is not trial de novo, but merely an ascertainment of whether the findings of the
Standard required on delegated power to fix rates administrative agency are consistent with law, free from fraud or imposition, and
supported by evidence.
That the rate be reasonable and just. (American Tobacco Co. v. Director of Patents,
G.R. No. L-26803, October 14, 1975) In any case, the rates must both be non- NOTE: The mere silence of the law does not necessarily imply that judicial review is
confiscatory and must have been established in the manner prescribed by the unavailable.
legislature. Even in the absence of an express requirement as to reasonableness, this
Jurisdiction
standard may be implied. A rate-fixing order, though temporary or provisional it may
be, is not exempt from the procedural requirements of notice and hearing when Rule 43 of the 1997 Rules of Civil Procedure provides that the Court of Appeals shall
prescribed by statute, as well as the requirement of reasonableness. (Philippine have appellate jurisdiction over judgments or final orders of the Court of Tax Appeals
Communications Satellite Corporation v. NTC, G.R. No. 84818, December 18, 1989) and from awards, judgments, final orders or resolutions of or authorized by any quasi-
judicial agency in the exercise of its quasi-judicial functions.
Re-delegating power to fix rates is prohibited
Requisites of Judicial Review of Administrative Action
The power delegated to an administrative agency to fix rates cannot, in the absence of
a law authorizing it, be delegated to another. This is expressed in the maxim, potestas 1. Principle of finality of administrative action - Administrative action must have been
delagata non delegari potest. (Kilusang Mayo Uno Labor Center v. Garcia, Jr., G.R. No. completed.
115381, December 23, 1994)
2. Principle of exhaustion of administrative remedies - Administrative remedies must
Power to Fix Rates Power to Fix Rates have been exhausted.
Exercised as Exercised as Quasi-
Legislative Function Judicial Function Limitations on Judicial Review
1. Final and executory decisions cannot be made the subject of judicial review.
Rules and/or rates laid Rules and the rate
down are meant to apply imposed apply exclusively 2. Administrative acts involving a political question are beyond judicial review, except
to all enterprises to a particular party when there is an allegation that there has been grave abuse of discretion.
3. Courts are generally bound by the findings of fact of an administrative agency.
Prior notice and hearing Prior notice and hearing
to the affected parties is are essential to the NOTE: Courts will not render a decree in advance of administrative action. Such action
not a requirement, except validity of such rates. But would be rendered nugatory. It is not for the court to stop an administrative officer from
where the legislature an administrative agency performing his statutory duty for fear that he will perform it wrongly.
itself requires it. may be empowered by aw
to approve provisionally, 1. Doctrine of primary administrative jurisdiction (aka doctrine of prior resort)
when demanded by Under the principle of primary jurisdiction, courts cannot or will not determine a
urgent public need, rates controversy involving question within the jurisdiction of an administrative body prior to
of public utilities without a the decision of that question by the administrative tribunal where the:
hearing.
1. Question demands administrative determination requiring special knowledge,
experience and services of the administrative tribunal;
2. Question requires determination of technical and intricate issues of a fact;

M.R.A.D.C. LUMBRE 243


CONSTITUTIONAL LAW REVIEW

3. Uniformity of ruling is essential to comply with purposes of the regulatory statute It calls for resorting first to the appropriate administrative authorities in the resolution
administered of a controversy falling under their jurisdiction and must first be appealed to the
administrative superiors up to the highest level before the same may be elevated to the
NOTE: In such instances, relief must first be obtained in administrative proceeding courts of justice for review. Premature invocation of court intervention is fatal to one’s
before a remedy will be supplied by the courts even though the matter is within the cause of action. Exhaustion of administrative remedies is a prerequisite for judicial
proper jurisdiction of a court. The judicial process is accordingly suspended pending review; it is a condition precedent which must be complied with.
referral of the claim to the administrative agency for its view.
Rationale
Rationale
1. To enable the administrative superiors to correct the errors committed by their
1. To take full advantage of administrative expertness subordinates.
2. To attain uniformity of application of regulatory laws which can be secured only 2. Courts should refrain from disturbing the findings of administrative bodies in
if determination of the issue is left to the administrative body deference to the doctrine of separation of powers.
Instances Where the Doctrine Finds No Application 3. Courts should not be saddled with the review of administrative cases.
1. By the court's determination, the legislature did not intend that the issues be 4. Judicial review of administrative cases is usually effected through special civil
left solely to the initial determination of the administrative body. actions which are available only if there
2. The issues involve purely questions of law. 5. To avail of administrative remedy entails lesser expenses and provides for a
3. Courts and administrative bodies have concurrent jurisdiction. speedier disposition of controversies.

Exceptions to the Doctrine of Primary Jurisdiction Exceptions to the Application of the Doctrine (1991, 2000, 2004 Bar)

1. Where there is estoppel on the part of the party invoking the doctrine; 1. Violation of due process;

2. Where the challenged administrative act is patently illegal, amounting to lack of 2. When there is estoppel on the part of the administrative agency concerned;
jurisdiction; 3. When the issue involved is a purely legal question;
3. Where there is unreasonable delay or official inaction that will irretrievably 4. When there is irreparable injury;
prejudice the complainant;
5. When the administrative action is patently illegal amounting to lack or excess of
4. Where the amount involved is relatively small so as to make the rule impractical jurisdiction;
and oppressive;
6. When the respondent is a Department Secretary whose acts as an alter ego of
5. Where the question involved is purely legal and will ultimately have to be decided the President bears the implied and assumed approval of the latter;
by the courts of justice;
7. When the subject matter is a private land case proceedings;
6. Where judicial intervention is urgent;
8. When it would be unreasonable;
7. When its application may cause great and irreparable damage;
9. When no administrative review is provided by law;
8. Where the controverted acts violate due process;
10. When the rule does not provide a plain, speedy, and adequate remedy;
9. When the issue of non-exhaustion of administrative remedies has been rendered
moot; 11. When the issue of non-exhaustion of administrative remedies has been
rendered moot;
10. When there is no other plain, speedy and adequate remedy;
12. When there are circumstances indicating the urgency of judicial intervention;
11. When strong public interest is involved;
13. When it would amount to a nullification of a claim; and
12. In quo warranto proceedings (The Province of Aklan v. Jody King Construction
and Development Corp., G.R. Nos. 197592 & 202623, November 27, 2013) 14. Where the rule on qualified political agency applies (Laguna CATV Network v.
Maraan, G.R. No. 139492, November 19, 2002)
2. Doctrine of exhaustion of administrative remedies

M.R.A.D.C. LUMBRE 244


CONSTITUTIONAL LAW REVIEW

Effect of Non-exhaustion of Administrative Remedies Considering that EDUPLAN had already paid the full purchase price of the subject
unit, the latter is entitled to the transfer of ownership of the subject property in its
Failure to observe the doctrine of exhaustion of administrative remedies does not affect favor.
the jurisdiction of the Court. The only effect of non-compliance with this rule is that it
will deprive the complainant of a cause of action, which is a ground for a motion to
dismiss. If not invoked at the proper time, this ground is deemed waived and the court
can take cognizance of the case and try it. (Republic v. Sandiganbayan, G.R. Nos. 3. Doctrine of finality of administrative action
112708-09, March 29, 1996) No resort to the courts will be allowed unless the administrative action has been
a. Department of Finance vs. Hon. Mariano M. Dela Cruz, G.R. No. 209331, completed and there is nothing left to be done in the administrative structure. (Sta.
24 April 2015 Rosa Mining v. Leido, 156 SCRA 1) A party aggrieved must not merely initiate the
prescribed administrative procedure to obtain relief, but must also pursue it to its
Under the doctrine of exhaustion of administrative remedies, before a party may appropriate conclusion before seeking judicial intervention in order to give that
seek intervention from the court, he or she should have already exhausted all the administrative agency an opportunity to decide the matter by itself correctly and
remedies in the administrative level. If there is still a remedy available within the prevent unnecessary and premature resort to the courts. (Zabat v. Court of Appeals,
administrative machinery, "then such remedy should be exhausted first before 338 SCRA 551)
[the] court's judicial power can be sought." The doctrine of exhaustion of
administrative remedies presupposes that both the courts and the administrative
agency have concurrent jurisdiction. This is because non-observance of the doctrine
of exhaustion of administrative remedies does not affect the court's jurisdiction.
xxx xxx xxx
Hence, when jurisdiction is exclusively granted to an administrative agency, the
doctrine of exhaustion of administrative remedies does not apply. Here, considering
that the Civil Service Commission is granted exclusive jurisdiction over cases
involving personnel actions, the doctrine of primary administrative jurisdiction, not
the doctrine of exhaustion of administrative remedies, applies.

b. United Overseas Bank of the Philippines vs. The Board of


Commissioners-HLURB, G.R. No. 182133, 23 June 2015

The issue on whether non-compliance with the clearance requirement with


the HLURB would result to the nullification of the entire mortgage contract
or only a part of it is purely legal which will have to be decided ultimately by a
regular court of law. It does not involve an examination of the probative value of
the evidence presented by the parties. There is a question of law when the doubt
or difference arises as to what the law is on a certain state of facts, and not as to
the truth or the falsehood of alleged facts. Said question at best could be resolved
only tentatively by the administrative authorities. The final decision on the matter
rests not with them but with the courts of justice. Exhaustion of administrative
remedies does not apply, because nothing of an administrative nature is to be or
can be done. The issue does not require technical knowledge and experience, but
one that would involve the interpretation and application of law. There is, thus, no
need to exhaust administrative remedies, under the premises.
The HLURB erred in declaring null and void the entire mortgage executed between
JOS Managing Builders and United Overseas Bank.
Since EDUPLAN has an actionable interest only over Unit E, 10th Floor, Aurora
Milestone Tower, it is but logical to conclude that it has no standing to seek for the
complete nullification of the subject mortgage and the HLURB was incorrect when
it voided the whole mortgage between JOS Managing Builders and United Overseas
Bank.

M.R.A.D.C. LUMBRE 245


CONSTITUTIONAL LAW REVIEW

CHAPTER 4: LAW ON PUBLIC OFFICERS 2. Invested with Authority to exercise some portion of the sovereign power of the
State;

3. The powers conferred and the duties to be discharged must be defined directly
I. General principles or impliedly by the Legislature or through legislative authority;
4. Duties are performed Independently without control unless those of a
subordinate;
Public Office
5. Continuing and permanent (Fernandez v. Sto. Tomas, G.R. No. 116418, March
It is the right, authority, and duty created and conferred by law, by which for a given 7, 1995; Tejada v. Domingo, G.R. No. 91860, Jan. 13, 1992)
period, either fixed by law or enduring at the pleasure of the creating power, an
individual is invested with some portion of the sovereign functions of the government, Creation
to be exercised by him for the benefit of the public. (Fernandez v. Sto. Tomas, G.R. No. 1. By the Constitution, e.g. Office of the President;
116418, March 7, 1995)
2. By valid statutory enactments, e.g. Office of the Insurance Commissioner; and
Purpose of a Public Office
3. By authority of law, e.g. the Davide Commission.
A public office is created to effect the end for which government has been instituted
which is the common good; not profit, honor, or private interest of any person, family Public office vs. Public contract
or class of persons (63C Am. Jur. 2d Public Officers and Employees 667 [1997]).
Basis Public Office Public Contract
Characteristics of Public Office (P3VN) As to Creation Incident of sovereignty Originates from the will of the
1. It is a Public trust – The principle of “public office is a public trust” means that contracting parties, subject to
the officer holds the public office in trust for the benefit of the people—to whom the limitations imposed by law.
such officers are required to be accountable at all times, and to serve with utmost As to persons Has for its object the Imposes obligations only upon
responsibility, loyalty, and efficiency, act with patriotism and justice, and lead affected carrying out of sovereign persons who entered the same.
modest lives (1987 Constitution, Art. XI, Sec. 1). as well as governmental
functions affecting even
2. It is not a Property – The concept "public office is not a property” means that it persons not bound by
is outside the commerce of man; hence, it cannot be the subject of a contract. contract
(Santos v. Secretary of Labor, G.R. No.L-21624, Feb. 27, 1968) As to subject Embraces the idea of Is almost always limited in its
3. It is personal to the Public officer – It is not a property transmissible to the heirs matter and tenure, duration, and duration and specific in its
of the public officer upon the latter’s death (Santos v. Secretary of Labor, G.R. scope continuity. The duties objects. Its terms define and
No.L-21624, Feb. 27, 1968). connected therewith are limit the rights and obligations of
generally continuing and the parties, and neither may
4. It is not a Vested right. permanent. depart therefrom without the
consent of the other.
NOTE: However, right to a public office is nevertheless a protected right. With the
exception of constitutional offices that provide for some immunity as regards salary
and tenure, right to a public office is protected by the constitutional provision on Public officer
security of tenure. It cannot be taken from its incumbent without due process.
(Morfe v. Mutuc, G.R. No. L-20387, Jan. 31, 1968; Aparri v. CA, G.R. No. L-30057, Any person who, by direct provision of law, popular election or appointment by
Jan. 31, 1984) competent authority, shall take part in the performance of public functions in the
5. It is not a Natural right – Under our political system, the right to hold public government of the Philippine Islands, or shall perform in said Government or in any of
office exists only because and by virtue of some law expressly or impliedly creating its branches, public duties as an employee, agent, or subordinate official, of any rank
and conferring it. or class (Revised Penal Code, Art. 203).

Elements of a public office (CALIC) NOTE: Under Sec. 2. RA 3019, the term public officer includes elective and appointive
1. Created by Constitution or by law or by some body or agency to which the power officials and employees, permanent or temporary, whether in the classified, unclassified
to create the office has been delegated; or exempt service, receiving compensation, even nominal, from the government.

M.R.A.D.C. LUMBRE 246


CONSTITUTIONAL LAW REVIEW

Kinds of a public officer Relative to public offices created by statute, Congress has virtually plenary powers to
prescribe qualifications, provided that:
1. Constitutional or statutory;
2. National or local; 1. the qualifications are germane to the objective/s for which the public office was
3. Legislative, executive, or judicial; created; and
4. Lucrative or honorary; 2. the qualifications are not too specific as to fit a particular, identifiable person,
5. Discretionary or ministerial; because that would deprive the appointing authority of discretion in the selection
6. Appointive or elective; of the appointee.
7. Civil or military;
8. De jure or de facto. Disqualifications

Kinds of Government Employment


The legislature has the right to prescribe disqualifications in the same manner that it
CAREER SERVICE NON-CAREER SERVICE can prescribe qualifications, provided that the prescribed disqualifications do not violate
Entrance is based on merits and fitness, Entrance is based on qualifications other the Constitution.
which is determined by competitive than merit and fitness.
examination (except for non- competitive General Disqualifications under the Constitution
positions) or based on highly technical
qualifications. 1. No candidate who lost in an election shall, within one (1) year after such election, be
appointed to any office in Government. (Section 6, Article IX-B)
Impeachable Officers:
1. President; 2. No elective official shall be eligible for appointment or designation in any capacity to
any public office or position during his tenure. (Section 7(1), Article IX-B)
2. Vice President;
3. Chief Justice and Associate Justices of the Supreme Court; 3. Unless otherwise allowed by law or by primary functions of his position, no appointive
4. Chairmen and Members of the Constitutional Commissions; and official shall hold any other position in Government. (Section 7(2), Article IX-B)
5. the Ombudsman.
It was held that when another office is held by a public officer in an ex officio
The foregoing enumeration is exclusive. capacity, as provided by law and as required by the primary functions of his office,
there is no violation, because such other office does not comprise “any other
Qualifications position.” The ex officio position is actually and, in legal contemplation, part of the
principal office. But the official concerned is not entitled to receive additional
It is to be understood in 2 different senses: compensation for his services in the said position because his services are already
paid for and covered by the compensation attached to his principal office. (National
1. May refer to endowments, qualities, or attributes which make an Amnesty Commission v. COA, G.R. No. 156982)
individual eligible for public office, e.g. citizenship; or
Specific Disqualifications Under the Constitution
2. May refer to the act of entering into the performance of the functions of a
public office, e.g. taking the oath of office. a) The President, Vice President, the Members of the Cabinet, and their deputies or
assistants shall not, unless otherwise provided in the Constitution, hold any other office
When used in the sense of endowments, qualities, or attributes, the individual must or employment during their tenure [Sec. 13, Art. VII].
possess the qualifications at the time of appointment or election and continuously for
as long as the official relationship continues. b) No Senator or Member of the House of Representatives may hold any other office or
employment in the Government, or any subdivision, agency, or instrumentality thereof,
When the qualifications are prescribed by the Constitution, they are generally exclusive, including government-owned or controlled corporations or their subsidiaries, during his
except where the Constitution itself provides otherwise. term without forfeiting his seat. Neither shall he be appointed to any office which may
have been created or the emoluments thereof increased during the term for which he
was elected [Sec. 13, Art. VI]. See Adaza v. Pacana, 135 SCRA 431.

M.R.A.D.C. LUMBRE 247


CONSTITUTIONAL LAW REVIEW

c) The Members of the Supreme Court and of other courts established by law shall not a) By reputation or acquiescence, the public, without inquiry, relies on the
be designated to any agency performing quasi-judicial or administrative functions [Sec. supposition that he is the public officer that he purports to be. This is acquired
12, Art. VIII]. See In Re: Manzano, 166 SCRA usually when the individual has acted as an officer for such a length of time that
the public believes that he is the public officer that he assumes to be.
d) No Member of a Constitutional Commission shall, during his tenure, hold any other
office or employment [Sec. 2, Art. IX-A]. The same disqualification applies to the b) Under a known and valid appointment or election, but the officer failed to
Ombudsman and his Deputies [Sec. 8, Art. XI]. conform to a requirement imposed bv law, e.g., taking the oath of office.

c) Under a known appointment or election, void because of the ineligibility of the


e) The Ombudsman and his Deputies shall not be qualified to run for any office in the
officer, or want of authority of the appointing or electing authority, or because of
election immediately succeeding their cessation from office [Sec. 11, Art. XI],
an irregularity in his appointment or election, such ineligibility, want of authority or
f) . Members of Constitutional Commissions, the Ombudsman and his Deputies must irregularity being unknown to the public.
not have been candidates for any elective position in the elections immediately
d) Under a known appointment or election pursuant to an unconstitutional law,
preceding their appointment. [Sec. 1, Art. IX-B' Sec. 1 Art. IX-C; Sec. 1, Art. IX-D;
before the law is declared unconstitutional.
Sec. 8, Art. XI]. ’
As to Entitlement to Salaries: The general rule is that the rightful incumbent of a
g) Members of Constitutional Commissions, the Ombudsman and his Deputies are public office may recover from an officer de facto the salary received by the latter during
appointed to a term of seven (7) years, without reappointment [Sec. 1(2), Art. IX-B; the time of his wrongful tenure, even though he entered into the office in good faith
Sec. 1 (2), Art. IX-C; Sec. 1 (2), Art. IX-D;.] and under color of title.

h) The spouse and relatives by consanguinity or affinity within the fourth civil degree of However, where there is no de jure public officer, the officer de facto who in good faith
the President shall not during his tenure be appointed as Members of the Constitutional has had possession of the office and has discharged the duties pertaining thereto, is
Commissions, or the Office of the Ombudsman, or as Secretaries, Undersecretaries, legally entitled to the emoluments of the office, and may, in an appropriate action,
chairmen or heads of bureaus or offices, including government-owned or controlled recover the salary, fees and other compensations attached to the office.
corporations [Sec. 13, Art. VII],

De Facto Officers
II. Modes and Kinds of Appointment
One who has the reputation of being the officer that he assumes to be, and yet is not a
good officer in point of law [Torres v. Ribo, 81 Phil 44]. He must have acted as an officer
for such length of time, under color of title and under such circumstances of reputation Public offices are filled up either by:
or acquiescence by the public and public authorities, as to afford a presumption of 1. Appointment;
election or appointment, and induce people, without inquiry, and relying on the
supposition that he is the officer he assumes to be, to submit to or invoke his action. 2. Election;
3. Designation – The mere imposition of new or additional duties upon an officer to
The acts of the de facto public officer, insofar as they affect the public, are valid, binding
be performed by him in a special manner.
and with full legal effect. The doctrine is intended not for the protection of the public
officer, but for the protection of the public and individuals who get involved in the official 4. In some instances by contract or by some other modes authorized by law.
acts of persons discharging the duties of a public office [Monroy v. Court of Appeals, 20 (Preclaro v. Sandiganbayan, G.R. No. 111091, Aug. 21, 1995)
SCRA 620]. Appointment
Elements of De Facto Officer The act of designation by the executive officer, board or body to whom that power has
been delegated, the individual who is to exercise the powers and functions of a given
1. A validly existing public office. office. In this sense, it is to be distinguished from the selection or designation by a
popular vote. (Borromeo v. Mariano, G.R. No. L-16808, Jan. 3, 1921)
2. Actual physical possession of said office.
It refers to the nomination or designation of an individual to an office (Borromeo v.
3. Color of title to the office.
Mariano, ibid.). It is, in law, equivalent to “filling a vacancy”. (Conde v. National Tobacco
There is color of title to the office in any of the following cases: Corp., G.R. No. L-11985, Jan. 28, 1961)

M.R.A.D.C. LUMBRE 248


CONSTITUTIONAL LAW REVIEW

Classifications of Appointment should have been preferred. This is a political question involving considerations of
wisdom which only the appointing authority can decide. (Luego v. CSC, G.R. No. L-
Permanent or temporary. Permanent appointments are those extended to persons 69137, Aug. 5, 1986)
possessing the qualifications and the requisite eligibility and are thus protected by the
constitutional guarantee of security of tenure. Appointment vs. Designation
Temporary appointments are given to persons without such eligibility, revocable at will CAREER SERVICE NON-CAREER SERVICE
and without the necessity of just cause or a valid investigation; made on the Entrance is based on merits and fitness, Entrance is based on qualifications other
understanding that the appointing power has not yet decided on a permanent appointee which is determined by competitive than merit and fitness.
and that the temporary appointee may be replaced at any time a permanent choice is examination (except for non- competitive
made. positions) or based on highly technical
qualifications.
A temporary appointment and a designation are not subject to confirmation by the
Commission on Appointments. Such confirmation, if given erroneously, will not make
the incumbent a permanent appointee.
Appointing authority
Regular or Ad Interim. A regular appointment is one made by the President while
Congress is in session, takes effect only after confirmation by the Commission on Inherently belongs to the people. It belongs to where the people have chosen to place
Appointments, and once approved, continues until the end of the term of the appointee. it by their Constitution or laws. (63C Am. Jur. 2d Public Officers and Employees 738,
1997)
An ad interim appointment is one made by the President while Congress is not in
session, takes effect immediately, but ceases to be valid if disapproved by the Entrusted to designated elected and appointed public officials. The appointment of
Commission on Appointments or upon the next adjournment of Congress. In the latter public officials is generally looked upon as properly belonging to the executive
case, the ad interim appointment is deemed “by-passed” through inaction. The ad department. Appointments may also be made by Congress or the courts, but when so
interim appointment is intended to prevent interruptions in vital government services made should be taken as an incident to the discharge of functions within their respective
that would otherwise result from prolonged vacancies in government offices. spheres. [(Government v. Springer, 50 Phil. 259, affirmed in Springer v. Government,
277 U.S. 189, 72 Ed. 845, 48 S.CT. 480 (1928)]
An ad interim appointment is a permanent appointment [Pamantasan ng Lungsod ng
Maynila v. Intermediate Appellate Court, 140 SCRA 22]. It is a permanent appointment NOTE: The general rule is that the appointing power is the exclusive prerogative of the
because it takes effect immediately and can no longer be withdrawn by the President President, upon which no limitations may be imposed by Congress, except those
once the appointee has qualified into office. resulting from the need of securing the concurrence of the Commission of Appointments
and from the exercise of the limited power to prescribe the qualifications or
The fact that it is subject to confirmation by the Commission on Appointments does not disqualifications to a given appointive office. (Rafael v. Embroidery and Apparel Control
alter its permanent character. and Inspections Board, G.R. No. L-19978, Sept. 29, 1967)
An ad interim appointment can be terminated for two causes specified in the Where the law is silent as to who is the appointing authority, it is understood to be the
Constitution: disapproval of the appointment by the Commission on Appointments, or President of the Philippines. (Rufino v. Endriga, G.R. No. 139554, July 21, 2006)
adjournment by Congress without the Commission on Appointments acting on the
appointment. There is no dispute that when the Commission on Appointments Absent any contrary statutory provision, the power to appoint carries with it the power
disapproves an ad interim appointment, the appointee can no longer be extended a new to remove or discipline. (Aguirre, Jr. v. De Castro, G.R. No. 127631, Dec. 17, 1999)
appointment, inasmuch as the disapproval is a final decision of the Commission in the Appointee’s Acceptance of Office
exercise of its checking power on the appointing authority of the President. Such
disapproval is final and binding on both the appointee and the appointing power. But GR: An appointee’s acceptance of office is not necessary to complete or to make the
when an ad interim appointment is by-passed because of lack of time or failure of the appointment valid where there is no provision of law to the contrary.
Commission on Appointments to organize, there is no final decision by the Commission
to give or withhold its consent to the appointment. Absent such decision, the President XPN: Acceptance, however, is necessary to enable the appointee to have full
is free to renew the ad interim appointment. possession, enjoyment, and responsibility of an office.

Nature of appointment Procedure for the appointment of those that require confirmation by the
Commission on Appointments
Appointment is an essentially discretionary power and must be performed by the officer
in which it is vested according to his best lights, the only condition being that the 1. Nomination by the President;
appointee should possess the qualifications required by law. If he does, then the 2. Confirmation by the Commission on Appointments;
appointment cannot be faulted on the ground that there are others better qualified who

M.R.A.D.C. LUMBRE 249


CONSTITUTIONAL LAW REVIEW

3. Issuance of commission; and the first offense. It is relevant to point out, too, that Judge Yu had no personality to
object to or oppose Ms. Lagman's appointment, considering that only a qualified next-
4. Acceptance by the appointee. in-rank employee has been recognized as a party-in-interest to file the protest in
NOTE: Appointment is deemed complete upon acceptance. Pending such acceptance, accordance with paragraph 1.6.1, Article IX of the 2002 Revised Manual of Clerks of
which is optional on the part of the appointee, the appointment may still be validly Court.
withdrawn.
GR: Appointment to a public office cannot be forced upon any citizen.
III. Rights of Public Officers
XPN: For purposes of defense of the State under Sec. 4, Art. 2 (also an XPN to the rule
against involuntary servitude: POSSE COMMITATUS). (Lacson v. Romero, No. L-3081,
Oct. 14, 1949) A. Right to Office.
NOTE:
The just and legal claim to exercise the powers and the responsibilities of the public
• For ad interim appointments, steps 1, 3 and 4 precede step 2. office.

• For appointments which do not confirmation, step 2 is skipped. 1. Term v. Tenure. Term is the period during which the officer may claim to hold the
1. Office of the Court Administrator vs. Judge Yu, A.M. No. MTJ-12-1813, 22 office as of right; while tenure is the period during which the officer actually holds office.
November 2016 In Nunez v. Averia, 57 SCRA 726, it was held that the extension of the tenure of elective
local officials beyond their term is predicated on having been duly elected in the
Although Judge Yu insisted on the irregularity of the appointment of Ms. Tejero-Lopez November 8, 1971 elections.
for lack of personal endorsement from her as the Presiding Judge, and of the
appointment of Ms. Lagman due to a pending administrative complaint, the B. Right to Salary.
appointments of Ms. Tejero-Lopez and Ms. Lagman were valid and regular. As such,
Judge Yu had no good reason to reject the appointments. 1. Salary is the personal compensation to be paid to the public officer for his services,
and it is generally a fixed annual or periodical payment depending on the time and not
To start with, Ms. Tejero-Lopez and other applicants had undergone scrutiny and
on the amount of the services he may render. It is distinguished from wages, in that
processing by the duly constituted committee, and the OCA had then signed and
executed the appointment. Nonetheless, the authority to appoint still emanated from salary is given to officers of higher degree of employment than those to whom wages
the Court itself. Judge Yu's objection to Ms. Tejero-Lopez's appointment for lack of her are given; salary is regarded as compensation per annum, while wages are paid day by
personal endorsement was not enough to negate the appointment. Judge Yu had no day or week by week.
right to reject the appointment, making her rejection another instance of gross
insubordination by her. 2. Basis: The legal title to the office and the fact that the law attaches compensation to
the office.
Judge Yu could only recommend an applicant for a vacant position in her court for the
consideration of the SPBLC, which then accorded priority to the recommendee if the a) In Dimaandal v. Commission on Audit, 291 SCRA 322, the petitioner, a Supply
latter possessed superior qualifications than or was at least of equal qualifications as Officer II who was designated by the Provincial Governor as Assistant Provincial
the other applicants she did not recommend. The SPBLC explained to Judge Yu the Treasurer for Administration, was declared not entitled to claim the difference
selection process that had resulted in the appointment of Ms. Tejero-Lopez. She could between the salary and representation allowance of Assistant Provincial Treasurer
not impose her recommendee on the SPBLC which was legally mandated to maintain and Supply Officer II, because the Provincial Governor was without authority to
fairness and impartiality in its assessment of the applicants based on performance,
designate petitioner, the power being vested in the Secretary of Finance under the
eligibility, education and training, experience and outstanding accomplishments,
Local Government Code. Because petitioner’s designation was without color of
psycho-social attributes and personality traits, and potentials.
authority, the right to salary or to an allowance due from the said office never
Secondly, Judge Yu's rejection of the appointment of Ms. Lagman was just as existed.
unwarranted.
b) Right of a de facto officer to salary: Where there is no de jure officer, a de facto
Under Section 34, Rule II of the Uniform Rules on Administrative Cases in the Civil
officer who, in good faith, has possession of the office and has discharged the duties
Service (URACCS), a pending administrative complaint shall not disqualify an employee
thereof, is entitled to salary. See Menzon v. Petilla, supra; Civil Liberties Union v.
from promotion. Accordingly, Judge Yu's administrative complaint had no bearing on
Ms. Lagman's appointment, more so because Ms. Lagman was held liable only for simple Executive Secretary, supra; Rodriguez v. Tan, supra.; Monroyv. Court of Appeals,
misconduct, a less grave offense that did not merit termination from public service for supra.

M.R.A.D.C. LUMBRE 250


CONSTITUTIONAL LAW REVIEW

c) Salary cannot be garnished. The salary of a public officer cannot, by e) Standardization of compensation [Sec. 5, Art. IX-B]. R.A. 6758 (Salary
garnishment, attachment, or order of execution be seized before being paid to him, Standardization Law) was passed in compliance with the constitutional provision.
and appropriated to the payment of his debts. Public policy also prohibits the
assignment of unearned salaries or fees. Agreements affecting compensation are i) In Intia v. Commission on Audit, G.R. No. 131529, April 30, 1999, it was
void as contrary to public policy. held that the discretion of the Philippine Postal Corporation Board of Directors
on the matter of personnel compensation is not absolute, as the same must
d) Where, on account of reorganization, the position is abolished, and the strictly conform with R.A. 6758 in relation to the General Appropriations Act.
incumbent thereof requests retention and even accepts an appointment to a lower
position, she cannot demand that she be paid the salary equivalent to that of her ii) In Central Bank Employees Association v. Bangko Sentral ng Pilipinas, G.R.
former position, because she is now barred by estoppel from claiming the desired No. 148208, December 15, 2004, the Supreme Court said that while the “policy
relief [Manalo v. Gloria, G.R. No. 106692, September 1, 1994]. determination” argument may support the inequality of treatment of the rank-
and-file employees and the officers of BSP, it cannot justify the inequality of
e) But compensation, allowances and other benefits received by government treatment between BSP rank-and-file employees and the employees of other
officials and employees without the requisite approval or authority of the Government Financing Institutions (who are exempted from the Salary
Department of Budget and Management (DBM) are unauthorized and irregular. It Standardization Act by their respective charters).
is within the turf of the DBM Secretary to disallow the upgrading, reclassification
and creation of additional plantilla positions in the Commission on Human Rights, iii) In De Jesus v. Commission on Audit, G.R. No. 127515, May 10, 2005, the
based on its finding that such scheme lacks legal justification. The Commission on Supreme Court upheld the entitlement of LWUA officials and employees to the
Human Rights is not a constitutional commission; it does not enjoy fiscal autonomy rice subsidy, since it was shown that the benefit has been existing prior to the
[Commission on Human Rights Employees Association v. Commission on Human effectiyity of RA 6758, that it has not been included in the standardized salary
Rights, G.R. No. 155336, November 25, 2004]. rates, and that the grant thereof is limited to incumbents as of July 1, 1989 (in
order not to upset the policy of non-diminution of pay). In this case, it was
f) The Commission on Audit has the authority to order the withholding of an officer’s also reiterated that DBM Corporate Compensation Circular No. 10, issued
salary and other emoluments up to the amount of his alleged shortage, but not to October 2, 1989, was ineffective, because it was not published either in the
apply the withheld amount to the alleged shortage for which her liability is still Official Gazette or in a newspaper of general circulation in the country.
being litigated [Santiago v. Commission on Audit, G.R. No. 146824, November 21,
2007]. f) Separation pay to be given to career Civil Service employees who are separated
from the service not for cause but by reason of reorganization [Sec. 16, Art. XVIII].
3. Some constitutional provisions affecting salaries:
4. Preventive suspension and the right to salary. In Gloria v. Court of Appeals,
a) No increase in the salaries of members of Congress shall take effect until after G.R. No. 131012, April 21, 1999, the Supreme Court clarified that there are two kinds
the expiration of the full term of the Members of the Senate and House of of preventive suspension of civil service employees who are charged with offenses
Representatives who approved the increase [Sec. 10, Art. VI]. See Ligot v. Mathay, punishable by removal or suspension, viz: (a) preventive suspension pending
supra. investigation under Sec. 51, Book V, Title I, Subtitle A of the Administrative Code of
1987; and (b) preventive suspension pending appeal if the penalty imposed by the
b) Salaries of the President and Vice President shall be fixed by law and shall not disciplining authority is suspension or dismissal and, after review, the respondent is
be decreased during their tenure. No increase shall take effect until after the exonerated under Sec. 47 of the same Code. It was then held that the employee has
expiration of the term of the incumbent during which such increase was approved no right to compensation during preventive suspension pending investigation, even if
[Sec. 6, Art. VII]. he is exonerated, because in order to be entitled to payment of back salaries, it is not
enough that an employee be exonerated of the charges against him. In addition, it must
c) The salary of members of the Judiciary shall not be decreased during their
be shown that his suspension is unjustified. The preventive suspension of civil service
continuance in office [Sec. 10, Art. VIII]. See Nitafan v. Tan, 152 SCRA 284, which
employees charged with dishonesty, oppression, grave misconduct or neglect of duty,
is authority for the rule that the imposition of income taxes on salaries of judges
is authorized by the Civil Service Law. It cannot, therefore, be considered “unjustified”
does not constitute unconstitutional diminution of salaries.
even if later the charges are dismissed. It is one of the sacrifices which holding a public
d) Additional, double or indirect compensation are prohibited, unless specifically office requires for the public good.
authorized by law [Sec. 8, Art. IX-B].
a) However, if the penalty imposed by the disciplining authority is suspension or
dismissal and, after review, the respondent is exonerated, the civil service officer

M.R.A.D.C. LUMBRE 251


CONSTITUTIONAL LAW REVIEW

or employee is entitled not only to reinstatement but also to back salaries for the C. Right to Preference in Promotion. See Meram v. Edralin, 154 SCRA 238. But the
period of preventive suspension pending appeal [Gloria v. Court of Appeals, right does not prevail over the discretion of the appointing authority [Luego v. Civil
supra.]. Service Commission, supra.].

5. Right to back salaries of illegal dismissed employee. The Court has, time and D. Right to vacation and sick leave.
again, held that an illegally dismissed government employee who is later ordered
reinstated is entitled to back wages and other monetary benefits from the time of his 1. In Maleniza v. Commission on Audit, 179 SCRA 408, it was held that elective officials,
illegal dismissal up to his reinstatement. The policy of “no work, no pay” cannot be e.g., municipal mayor, are not entitled to accrued vacation and sick leave credits,
applied, for such distressing state of affairs was not of her own making. To withhold her because they have no official hours of work. Note: This ruling may now be deemed
back salaries and benefits during her illegal dismissal would put to naught the abandoned in view of the specific provision of Sec. 81, R.A. 7160 [Local Government
constitutional guarantee of security of tenure for those in the civil service [Constantino- Code] that elective local officials shall be entitled to the same leave privileges as those
David v. Pangandaman-Gania, G.R. No. 156039, August 14, 2003]. enjoyed by appointive local officials, including the cumulation and commutation thereof.

a) Thus, in Civil Service Commission v. Gentallan, G.R. No. 152833, May 9, 2005, 2. In Request of CTA Presiding Judge Alex Reyes, 216 SCRA 728, it was held that under
(and Municipality of Jasaan, Misamis Oriental v. Gentallan, G.R. No. 154961, May Office of the President Memorandum Circular No. 54, dated March 24, 19,88,
9, 2005), the Supreme Court, in agreement with the Court of Appeals that the government officers or employees are now entitled to commutation of all leave credits
respondent was qualified and eligible for the position of local civil registrar, and without limitation and regardless of the period when the credits were earned, provided
finding that there was no factual or legal basis for her removal from the position, the claimant was in the service as of January 9, 1986.
ruled that as an illegally dismissed government employee who is later ordered
3. In Peralta v. Civil Service Commission, 212 SCRA 425, the Supreme Court ruled that
reinstated, the respondent is entitled to back wages and other monetary benefits
government employees, whether or not they have accumulated leave credits, are not
from the time of her illegal dismissal up to her reinstatement.
required by law to work on Saturdays, Sundays and holidays, and thus cannot be
b) However, in Balitaosan v. Secretary, DECS, G.R. No. 138238, September 2, declared absent on such non-working days. Accordingly, they cannot and should not be
2003, it was held that where the reinstatement was not the result of exoneration deprived of their salary corresponding to said non- working days just because they were
but an act of liberality of the Court of Appeals, the claim for back wages for the absent without pay on the day immediately prior to, or after said non-working days. A
period during which the employee was not allowed to work must be denied. In this different rule would constitute deprivation of property without due process of law.
case, the Court noted that the petitioner participated in the mass action which
E. Right to Maternity Leave.
resulted in the filing of charges against him and his subsequent dismissal from the
service. He was ordered reinstated by the Court of Appeals only as an act of F. Right to Retirement Pay.
liberality. The general rule, then, is that a public official is not entitled to
compensation if he has not rendered any service. 1. Retirement laws are liberally construed in favor of the retiree [Profeta v. Drilon, 216
SCRA 777]. Thus, in GS/S v. Civil Service Commission, 245 SCRA 179, the period when
c) Likewise, in Brugada v. Secretary of Education, G.R. No. 14233243, January 31, respondent was paid on a per diem basis was held creditable for purposes of retirement,
2005, the Supreme Court held that the petitioners have no right to back wages it being clear that the per diem received was paid for performance of services and not
because they were neither exonerated nor unjustifiably suspended. an allowance for expenses incurred while the respondent was away from home base.
See also Conte v. Commission on Audit, 264 SCRA 19, where it was held that the
6. Right to additional allowances and benefits. Under the Local Government Code
petitioners should be allowed to avail of retirement benefits under R.A. 1616, after the
(R. A. 7160), local government units may provide for additional allowances and other
Commission on Audit disallowed their claim for entitlement to additional benefits
benefits to national government officials stationed or assigned to their municipality or
granted by SSS Resolution No. 56- 71 (which was adopted in order to induce employees
city. This authority, however, is not without limitations. Where, as in this case, it runs
to retire under R.A. 660).
counter to R.A. 6758, then the grant of financial assistance given by Marikina City to its
Auditing Office is in excess of its powers. The equal protection clause is not trenched, 2. The well-settled ruled is that the money value of the terminal leave of a retiring
because COA officials may be treated differently from other national government government official shall be computed at the retiree’s highest monthly salary. In
officials. For one, they should be “insulated for unwarranted influences so they can act Belicena v. Secretary of Finance, G.R. No. 143190, October 17, 2001, it was held that
with independence and integrity”. There has been no repeal by R.A. 7160 of R.A. 6758. petitioner’s highest monthly salary, for purposes of computing his terminal leave pay,
They can be harmonized and applied together. should be that corresponding to the salary of the Secretary of Finance which he received
as Acting Secretary of Finance. When the President designated the petitioner as Acting
Secretary of Finance on May 22, 1997, he did so under a well-considered opinion that

M.R.A.D.C. LUMBRE 252


CONSTITUTIONAL LAW REVIEW

the absence of Secretary Ocampo was of such an extent that the latter would be unable of service credits is only resorted to when the retiree does not qualify for benefits in
to perform his duties and, by reason of such opinion, the President extended a either or both of the Systems.
temporary designation to the petitioner consistent with Sec. 17, Administrative Code of
1987. Even the Commission on Audit has opined that a government official appointed G. Others.
or designated in an acting capacity pursuant to the Administrative Code is entitled to
1. Right to reimbursement for expenses incurred in the due performance of his duty.
salary differential, and that his highest monthly salary for purposes of computing his
But a public officer who uses a government vehicle is not entitled to, nor can he charge,
terminal leave pay shall include such salary differential.
a transportation allowance [Domingo v. Commission on Audit, G.R. No. 112371, October
3. In Cena v. Civil Service Commission, 211 SCRA 179, it was held that CSC 7, 1998].
Memorandum Circular No. 27, allowing extension of service only for one year (instead
2. Right to be indemnified against any liability which they may incur in the bona fide
of what is needed to complete the 15-year service requirement for retirement), cannot
discharge of their duties.
prevail over Sec. 11 (b), PD 1146, which allows extension in order to complete the 15-
year service requirement. This ruling was re-examined and modified in Raborv. Civil 3. Right to longevity pay (e.g. loyalty award)
Service Commission, G.R. No. 111812, May 31, 1995, where the Supreme Court said
that when it enunciated the Cena ruling, it took the narrow view on what subordinate a. The Provincial Government of Camarines Norte vs. Gonzalez, G.R. No.
rule-making by an administrative agency is permissible and valid, and it likewise laid 185740, July 23, 2013
heavy stress on the interest of retirees by allowing extension of services without
considering the significance of the general principle of compulsory retirement at the age As the CSC correctly noted in Resolution No. 0001158, the administrator position
of 65. Henceforth, CSC MC No. 27, series of 1990, is deemed valid and effective, and demands a close intimate relationship with the office of the governor (its
Sec. 11, P.D. 1146, is to be read together with CSC MC 27. However, the head of the appointing authority) to effectively develop, implement and administer the
different programs of the province. The administrator’s functions are to
agency is vested with discretionary authority to allow or disallow extension of service
recommend to the Sanggunian and to advise the governor on all matters
of an official or employee who has reached 65 without completing 15 years of
regarding the management and administration of the province, thus requiring
government service, although this discretion is to be exercised conformably with CSC
that its occupant enjoy the governor’s full trust and confidence.
MC 27. To emphasize the close relations that the provincial administrators’ functions
have with the office of the governor, RA 7160 even made the provincial
4. In the judiciary, however, the Court allows such extension if satisfied that the career
administrator position coterminous with its appointing authority. This provision,
of the retiree was marked by competence, integrity and dedication to public service [In along with the interrelations between the provincial administrator and governor
Re: Gregorio Pineda, 187 SCRA 469]. See also Cruz v. Tantuico, 166 SCRA 670. under Section 480, renders clear the intent of Congress to make the provincial
administrator position primarily confidential under the non-career service
5. Pursuant to E.O. 79-86, a reserved officer who satisfactorily rendered a total of ten category of the civil service.
years continuous active commissioned military service shall not be reverted to inactive
status except for cause or upon his own request. Accordingly, they are covered by In the current case, Congress, through RA 7160, did not abolish the provincial
compulsory membership in the GSIS [GSIS v. Commission on Audit, G.R. No. 125982, administrator position but significantly modified many of its aspects. It is now a
January 22, 1999]. primarily confidential position under the non-career service tranche of the civil
service. This change could not have been aimed at prejudicing Gonzales, as she
6. Liberally interpreting the provisions of R.A. 910, along the lines of Profeta v. Drilon, was not the only provincial administrator incumbent at the time RA 7160 was
supra., the Supreme Court approved the recommendation that the retirement benefits enacted. Rather, this change was part of the reform measures that RA 7160
of Justice Jorge Imperial shall be computed on the basis of the highest salary, introduced to further empower local governments and decentralize the delivery
emoluments and allowances he received as Acting Presiding Justice of the Court of of public service. Section 3(b) of RA 7160 provides as one of its operative
Appeals [Request of Clerk of Court Tessie L. Gatmaitan For Payment of Retirement principles that:
Benefits of CA Associate Justice Jorge S. Imperial, A.M. No. 97-77-RET, August 26,
1999]. (b) There shall be established in every local government unit an
accountable, efficient, and dynamic organizational structure and operating
7. But in Gamogamo v. PNOC Shipping & Transport Corp., G.R. No. 141707. May 07, mechanism that will meet the priority needs and service requirements of its
2002, the Supreme Court rejected the petitioner’s contention that for the purpose of communities.
computing his retirement pay, his 14 years of service with the Department of Health
Thus, Gonzales’ permanent appointment as provincial administrator prior to the
should be tacked in and added to the creditabje service later rendered in two
enactment of RA 7160 is immaterial to her removal as provincial administrator.
government-owned and controlled corporations without an original charter. Totalization

M.R.A.D.C. LUMBRE 253


CONSTITUTIONAL LAW REVIEW

For purposes of determining whether Gonzales’ termination violated her right to IV. ACCOUNTABILITY OF PUBLIC OFFICERS
security of tenure, the nature of the position she occupied at the time of her
removal should be considered, and not merely the nature of her appointment at Modes of Terminating Official Relationship
the time she entered government service.
1. Expiration of term or tenure;
Congress’ reclassification of the provincial administrator position in RA 7160 is a 2. Reaching the age limit;
valid exercise of legislative power that does not violate Gonzales’ security of
tenure. 3. Resignation;
4. Recall;
To be sure, both career and non-career service employees have a right to
5. Removal;
security of tenure. All permanent officers and employees in the civil service,
regardless of whether they belong to the career or non-career service category, 6. Abandonment;
are entitled to this guaranty; they cannot be removed from office except for 7. Acceptance of an incompatible office;
cause provided by law and after procedural due process. The concept of
security of tenure, however, labors under a variation for primarily 8. Abolition of office;
confidential employees due to the basic concept of a "primarily 9. Prescription of the right to office;
confidential" position. Serving at the confidence of the appointing
authority, the primarily confidential employee’s term of office expires 10. Impeachment;
when the appointing authority loses trust in the employee. When this 11. Death;
happens, the confidential employee is not "removed" or "dismissed"
12. Failure to assume elective office within six (6) months from proclamation;
from office; his term merely "expires" and the loss of trust and
confidence is the "just cause" provided by law that results in the 13. Conviction of a crime;
termination of employment. In the present case where the trust and 14. Filing of a certificate of candidacy.
confidence has been irretrievably eroded, we cannot fault Governor Pimentel’s
exercise of discretion when he decided that he could no longer entrust his
confidence in Gonzales.
Security of tenure in public office simply means that a public officer or employee 1. The “Threefold Liability Rule” - Office of the Ombudsman vs. Andutan, Jr.,
654 SCRA 539 (2011)
shall not be suspended or dismissed except for cause, as provided by law and
after due process. It cannot be expanded to grant a right to public office despite The issue of whether Section 20(5) of R.A. 6770 is mandatory or discretionary has been
a change in the nature of the office held. In other words, the CSC might have settled by jurisprudence. [W]ell-entrenched is the rule that administrative
been legally correct when it ruled that the petitioner violated Gonzales’ right to offenses do not prescribe. Administrative offenses by their very nature pertain to the
security of tenure when she was removed without sufficient just cause from her character of public officers and employees. In disciplining public officers and employees,
position, but the situation had since then been changed. In fact, Gonzales was the object sought is not the punishment of the officer or employee but the improvement
reinstated as ordered, but her services were subsequently terminated under the of the public service and the preservation of the public’s faith and confidence in our
law prevailing at the time of the termination of her service; i.e., she was then government. The Court held that the period stated in Section 20(5) of R.A. No. 6770
does not refer to the prescription of the offense but to the discretion given to the
already occupying a position that was primarily confidential and had to
Ombudsman on whether it would investigate a particular administrative offense. The
be dismissed because she no longer enjoyed the trust and confidence of
use of the word "may" in the provision is construed as permissive and operating to
the appointing authority. Thus, Gonzales’ termination for lack of confer discretion. The use of the word "may" clearly shows that it is directory in nature
confidence was lawful. She could no longer be reinstated as provincial and not mandatory as petitioner contends. When used in a statute, it is permissive only
administrator of Camarines Norte or to any other comparable position. This and operates to confer discretion; while the word "shall" is imperative, operating to
conclusion, however, is without prejudice to Gonzales’ entitlement to retirement impose a duty which may be enforced. Applying Section 20(5), therefore, it is
benefits, leave credits, and future employment in government service. discretionary upon the Ombudsman whether or not to conduct an investigation on a
complaint even if it was filed after one year from the occurrence of the act or omission
complained of. In fine, the complaint is not barred by prescription. (Emphasis supplied)
Where the words of a statute are clear, plain and free from ambiguity, they must be
given their literal meaning and applied without attempted interpretation

Clearly, Section 20 of R.A. 6770 does not prohibit the Ombudsman from conducting an
administrative investigation after the lapse of one year, reckoned from the time the

M.R.A.D.C. LUMBRE 254


CONSTITUTIONAL LAW REVIEW

alleged act was committed. Without doubt, even if the administrative case was filed service. This explains why court employees have been enjoined to strictly observe
beyond the one (1) year period stated in Section 20(5), the Ombudsman was well within official time and to devote every second or moment of such time to serving the public.
its discretion to conduct the administrative investigation.
Although many "moonlighting" activities were themselves legal acts that
xxxxxxx would be permitted or tolerated had the actors not been employed in the public
sector, moonlighting, albeit not usually treated as a serious misconduct, can
The court held that ombudsman can no longer file the administrative case because it is amount to a malfeasance in office by the very nature of the position held. In
moot and academic First, Andutans resignation was neither his choice nor of his own the case of Lopez, her being the Chief of the Checks Disbursement Division of the FMBO,
doing; he was forced to resign. Second, Andutan resigned from his DOF post on July 1, a major office of the Court itself, surely put the integrity of the Checks Disbursement
1998, while the administrative case was filed on September 1, 1999, exactly one (1) Division and the entire FMBO under so much undeserved suspicion. She ought to have
year and two (2) months after his resignation. refrained from engaging in money lending, particularly to the employees of the Court.
However, the State is not without remedy against Andutan or any public official who We do not need to stress that she was expected to be circumspect about her acts and
committed violations while in office, but had already resigned or retired therefrom. actuations, knowing that the impression of her having taken advantage of her position
Under the threefold liability rule, the wrongful acts or omissions of a public and her having abused the Only confidence reposed in her office and functions as such
officer may give rise to civil, criminal and administrative liability. Even if the would thereby become unavoidable. There is no doubt about her onerous lending
Ombudsman may no longer file an administrative case against a public official activities greatly diminishing the reputation of her office and of the Court itself in the
who has already resigned or retired, the Ombudsman may still file criminal and esteem of the public.
civil cases to vindicate Andutans alleged transgressions. In fact, here, the Considering that the official and personal conduct and deportment of all the people who
Ombudsman through the FFIB filed a criminal case for Estafa and violations of Section work for the Judiciary mirrored the image of the Court itself, they should strive to
3(a), (e) and (j) of the Anti-Graft and Corrupt Practices Act against Andutan. If found comport themselves with propriety and decorum at all times, and to be above suspicion
guilty, Andutan will not only be meted out the penalty of imprisonment, but also the of any misdeed and misconduct. As a public servant, therefore, Lopez knew only too
penalties of perpetual disqualification from office, and confiscation or forfeiture of any well that she was expected at all times to exhibit the highest sense of honesty and
prohibited interest. integrity. No less that the Constitution itself impresses this expectation in Section 1 of
2. “Moonlighting” - Re: Anonymous Letter-Complaint on the Alleged its Article XI that “Public office is a public trust. Public officers and employees must at
Involvement and for Engaging in the Business of Lending Money at Usurious all times, be accountable to the people, serve them with utmost responsibility, integrity,
rates of Interest of Ms. Dolores T. Lopez, SC Chief Judicial Staff Officer, and loyalty, and efficiency, act with patriotism and justice, and lead modest lives."
Mr. Fernando M. Montalvo, SC Supervising Judicial Staff Officer, Checks 3. Initiation of Complaint - IA1 Magcamit vs. Internal Affairs Service -
Disbursement Division, Fiscal Management and Budget Office, A.M. No. 2010- Philippine Drug Enforcement Agency (IAS-PDEA), G.R. No. 198140, 25 January
21-SC, 30 September 2014 2016
Courts are considered temples of justice and should never be utilized for any other The administrative complaint was initiated when Jaen and Delfm executed sworn
purpose. The fact remains that it is done within the premises of the Court and statements and filed them with the IAS-PDEA. As the CA correctly pointed out, the
presumably inside their office where official resources are utilized. This alone is highly letter-complaint did not, by itself, commence the administrative proceedings
reprehensible. By allowing anybody to enter their office solely for the purpose of against Magcamit; it merely triggered a fact-finding investigation by the IAS-
borrowing money, she has compromised the safety of the Checks Disbursement PDEA. Accordingly, these sworn statements - together with the letter-
Division. The Code of Conduct for Court Employees specifically Canon I, Section 5 complaint -were used as pieces of evidence to build a prima facie case for
provides that "Court personnel shall use the resources, property and funds under their extortion warranting a formal charge for grave misconduct.
official custody in judicious manner and solely in accordance with the prescribed
statutory and regulatory guidelines or procedures." Considering that Ms. Lopez is Administrative determinations of contested cases are by their nature quasi-judicial;
engaged in lending business, her integrity as a public servant and the reputation of her there is no requirement for strict adherence to technical rules that are observed in truly
office and of this Court have been seriously tarnished. judicial proceedings. As a rule, technical rules of procedure and evidence are relaxed in
administrative proceedings in order "to assist the parties in obtaining just, speedy and
Administrative Circular No. 5 (Re: Prohibition for All Officials and Employees of the inexpensive determination of their respective claims and defenses." By relaxing
Judiciary to Work as Insurance Agents), dated October 4, 1988, has prohibited all technical rules, administrative agencies are, thus, given leeway in coming up with a
officials and employees of the Judiciary from engaging directly in any private business, decision.
vocation or profession, even outside their office hours. The prohibition has been at
ensuring that full-time officers and employees of the courts render full-time service, for Nonetheless, in deciding disciplinary cases pursuant to their quasi-judicial powers,
only thereby could any undue delays in the administration of justice and in the administrative agencies must still comply with the fundamental principle of due process.
disposition of court cases be avoided. The nature of the work of court employees and Administrative tribunals exercising quasi-judicial powers are unfettered by the rigidity
officials demanded their highest degree of efficiency and responsibility, but they would of certain procedural requirements, subject to the observance of fundamental and
not ably meet the demand except by devoting their undivided time to the government essential requirements of due process in justiciable cases presented before them.

M.R.A.D.C. LUMBRE 255


CONSTITUTIONAL LAW REVIEW

Due process in administrative cases, in essence, is simply an opportunity to explain In dropping from the rolls, the due process requirement is not applicable. The position
one's side or to seek a reconsideration of the action or ruling. For as long as the parties is merely vacated. There is no forfeiture of benefits and the person dropped from the
were given fair and reasonable opportunity to be heard before judgment was rendered, rolls may be re-employed.
the demands of due process were sufficiently met.
5. Quantum of Proof in Administrative Case Against Judges - Datoon vs. Kapili,
4. Dropping from the Rolls - Re: Dropping from the Rolls of Cornelio Reniette 644 SCRA 384 (2011)
Cabrera, Utility Worker I, Municipal Trial Court in Cities, Branch 1, Lipa City,
653 SCRA 695 (2011) Administrative charges against judges have been viewed by this Court with utmost care,
as the respondent stands to face the penalty of dismissal or disbarment. Thus,
Pursuant to Section 63, Rule XVI of the Omnibus Rules on Leave, as amended by Civil proceedings of this character are in their nature highly penal in character and are
Service Resolution No. 070631, an employees absence without official leave for at least to be governed by the rules of law applicable to criminal cases. The charges in
30 working days warrants his separation from the service. The Rule specifically such case must, therefore, be proven beyond reasonable doubt.
provides:
NOTE: This is similar to administrative cases of revocation of doctor’s license. The
Sec. 63. Effect of absences without approved leave.-An official or employee who is quantum of proof required is also proof beyond reasonable doubt as the proceeding is
continuously absent without approved leave for at least thirty (30) working days akin to a criminal case.
shall be considered on absence without official leave (AWOL) and shall be separated
from the service or dropped from the rolls without prior notice. However, when it 6. Jurisdiction of the Civil Service Commission (CSC)
is clear under the obtaining circumstances that the official or employee concerned, a. Civil Service Commission vs. Court of Appeals, 682 SCRA 353 (2012)
has established a scheme to circumvent the rule by incurring substantial absences
though less than thirty working (30) days 3x in a semester, such that a pattern is The Civil Service Commission (CSC) has jurisdiction over the cases filed directly
already apparent, dropping from the rolls without notice may likewise be justified. with it, regardless who initiated the complaint.

If the number of unauthorized absences incurred is less than thirty (30) working days, CSC as the central personnel agency of the government has the power to appoint
a written Return-to-Work-Order shall be served to him at his last known address on and discipline its officials and employees and to hear and decide administrative
record. Failure on his part to report for work within the period stated in the order shall cases instituted by or brought before it directly or on appeal. Section 2(1), Article
be valid ground to drop him from the rolls. XI(B) of the 1987 Constitution defines the scope of the Civil Service:

In this connection, Section 63, Rule XVI, of the Omnibus Civil Service Rules and The Civil Service embraces all branches, subdivisions, instrumentalities, and
Regulations, as amended by Circular No. 14, s. 1999, provides: agencies of the Government, including government-owned and controlled
corporations with original charters.
Section 63. Effect of absences without approved leave. An official or
employee who is continuously absent without approved leave for at least By virtue of PD 1341, PUP became a chartered stated university, thereby making it
thirty (30) calendar days shall be considered on absence without official a government-owned or controlled corporation with an original charter whose
leave (AWOL) and shall be separated from the service or dropped from the employees are part of the Civil Service and are subject to the provisions of EO 292.
rolls without prior notice. He shall, however, be informed, at his address
However, the CSC has concurrent original jurisdiction with the Board of
appearing on his 201 files, of his separation from the service, not later than five
Regents over administrative cases.
(5) days from its effectivity.
The uniform Rules on Administrative Cases in Civil Service explicitly allows the CSC
While there is jurisprudence to the effect that a court employees AWOL for a
to hear and decided administrative cases directly brought before it:
prolonged period of time warrants the penalty of dismissal from the service
and the forfeiture of his benefits, the Court, given the circumstances of the Section 4. Jurisdiction of the Civil Service Commission. – The Civil Service
case, is inclined to adhere to the evaluation and recommendation of the OCA, Commission shall hear and decide administrative cases instituted by, brought
and refrain from imposing the administrative penalties of forfeiture of benefits before it, directly or on appeal, including contested appointments, and shall
and disqualification from re-employment. review decisions and actions of its offices and of the agencies attached to it.
Thus, Cornelio Reniette Cabrera was DROPPED from the rolls of service and his position Except as otherwise provided by the Constitution or by law, the Civil Service
was hereby declared VACANT. Commission shall have the final authority to pass upon the removal, separation
and suspension of all officers and employees in the civil service and upon all
NOTE: In dismissal, an employee is separated for just or authorized causes. Here, the
matters relating to the conduct, discipline and efficiency of such officers and
due process requirement under the law should be followed. Once dismissed, he can no
employees.
longer receive benefits under the law. He also cannot re-apply.
The CA regrettably failed to take into consideration the succeeding section of the
same rules which undeniably granted original concurrent jurisdiction to the CSC

M.R.A.D.C. LUMBRE 256


CONSTITUTIONAL LAW REVIEW

and belied its suggestion that the CSC could only take cognizance of cases on We agree with the CA that the Ombudsman has jurisdiction over respondent’s
appeal: complaint against petitioner although the act complained of involves a private deal
between them.
Section 7. Jurisdiction of Heads of Agencies. – Heads of Departments,
agencies, provinces, cities, municipalities and other instrumentalities Section 13(1), Article XI of the 1987 Constitution states that the Ombudsman can
shall have original concurrent jurisdiction, with the Commission, over investigate on its own or on complaint by any person any act or omission of any
their respective officers and employees. public official or employee when such act or omission appears to be illegal, unjust,
or improper.
b. Department of Finance vs. Hon. Mariano M. Dela Cruz, G.R. No. 209331,
24 August 2015 Under Section 1614 of R.A. No. 6770, otherwise known as the Ombudsman Act of
1989, the jurisdiction of the Ombudsman encompasses all kinds of
The CSC has jurisdiction over all employees of government branches, malfeasance, misfeasance, and nonfeasance committed by any public
subdivisions, instrumentalities, and agencies, including government- officer or employee during his/her tenure.
owned or controlled corporations with original charters. The CSC is the
sole arbiter of controversies relating to the civil service. The rule is that Section 1915 of R.A. No. 6770 also states that the Ombudsman shall act on all
disciplinary cases and cases involving personnel actions, including complaints relating, but not limited, to acts or omissions which are unfair
"appointment through certification, promotion, transfer, reinstatement, or irregular. Thus, even if the complaint concerns an act of the public
reemployment, detail, reassignment, demotion, and separation," are official or employee which is not service-connected, the case is within the
within the exclusive jurisdiction of the CSC. This rule is embodied in Section jurisdiction of the Ombudsman. The law does not qualify the nature of the
1, Rule V of the Omnibus Rules Implementing Book V of Executive Order No. 292 illegal act or omission of the public official or employee that the
and Other Pertinent Civil Service Laws (Omnibus Rules) which states: Ombudsman may investigate. It does not require that the act or omission
be related to or be connected with or arise from the performance of official
SECTION 1. x x x. As used in these Rules, any action denoting movement or duty. Since the law does not distinguish, neither should we.
progress of personnel in the civil service shall be known as personnel action.
Such action shall include promotion, transfer, reinstatement, reemployment,
detail, secondment, reassignment, demotion and separation, x x x.
A reading of the petition filed before the RTC shows that respondents were
questioning their mass detail and reassignment to CPRO. According to respondents,
their detail was carried out in bad faith and was meant to remove them from their
permanent positions in the BOC. The action appears to be a personnel action under
the jurisdiction of the CSC.
However, the petition went beyond questioning the detail of respondents.
Respondents further assailed the validity and constitutionality of CPO 189-2013.
Respondents alleged that CPO 189-2013 was issued even before EC) 140, pursuant
to which CPO 189-2013 was issued, became effective. Respondents alleged that
CPO 189-2013 was issued to beat the deadline of the Commission on Elections' ban
on personnel movement from 28 September 2013 to 20 October 2013 due to the
scheduled barangay elections. When respondents raised the issue of validity and
constitutionality of CPO 189-2013, the issue took the case beyond the scope of the
CSC's jurisdiction because the matter is no longer limited to personnel action. Thus,
the RTC did not abuse its discretion in taking cognizance of the action.

7. Jurisdiction of the Ombudsman


a. Samson vs. Restrivera, 646 SCRA 481 (2011)
We affirm the CA and Ombudsman that petitioner is administratively liable. We
hasten to add, however, that petitioner is guilty of conduct unbecoming a public
officer.

M.R.A.D.C. LUMBRE 257


CONSTITUTIONAL LAW REVIEW

CHAPTER 5: LAW ON LOCAL GOVERNMENTS municipal corporation exercises, by delegation, a part of the sovereignty of the
state.

2. Private or proprietary - It acts as an agent of the community in the administration


Constitutional Provisions of local affairs which is wholly beyond the sphere of public purposes, for which its
governmental powers are conferred. It acts as separate entity for its own purposes,
a. Article II, Section 25. The State shall ensure the autonomy of local
and not as a subdivision of the State.
governments.
NOTE: Not all corporations, which are not government owned or controlled, are ipso
b. Article X, Section 2. The territorial and political subdivisions shall enjoy local
facto private corporations. These corporations are treated by law as agencies or
autonomy.
instrumentalities of the government which are not subject to the tests of ownership or
The principle of local autonomy under the 1987 Constitution simply means control and economic viability but to different criteria relating to their public
“decentralization;” it does not make the local governments sovereign within the State purposes/interests or constitutional policies and objectives and their administrative
or an “imperium in imperio.” relationship to the government or any of its Departments or Offices.

Even as we recognize that the Constitution guarantees autonomy to local government The economic viability test would only apply in cases wherein the corporation is engaged
units, the exercise of local autonomy is subject to the power of control by Congress, in some economic activity or business function for the government. (Boy Scouts of the
and the power of general supervision by the President. (Judge Dadole v. COA) Philippines v. COA, G.R. No. 177131, June 7, 2011)
Municipal Corporations in the Philippines

I. Public Corporations 1. Province. The province, composed of a cluster of municipalities, or municipalities


and component cities, and as a political and corporate unit of government, serves
as a dynamic mechanism for developmental processes and effective governance of
local government units within its territorial jurisdiction [Sec. 459, R.A. 7160]
It is one created by the State, either by general or special act for purposes of
administration of local government, or rendering service for the public interest. 2. City. The city, composed of more urbanized and developed barangays, serves as
a general-purpose government for the coordination and delivery of basic, regular
Classes of Public Corporations
and direct services and effective governance of the inhabitants within its territorial
1. Quasi-Corporation – created by the State for a narrow or limited purpose. jurisdiction [Sec. 448, R.A. 7160].

2. Municipal Corporation – a body politic and corporate constituted by the 3. Municipality. The municipality, consisting of a group of barangays, serves
incorporation of the inhabitants for the purpose of the local government. primarily as a general purpose government for the coordination and delivery of
basic, regular and direct services and effective governance of the inhabitants within
Elements of Municipal Corporations its jurisdiction [Sec. 440, R.A. 7160]
1. Legal Creation or Incorporation – the law creating or authorizing the creation or 4. Barangay. As the basic political unit, the barangay serves as the primary
incorporation of a municipal corporation. planning and implementing unit of government policies, plans, programs, projects
and activities in the community, and as a forum wherein the collective views of the
2. Corporate Name – the name by which the corporation shall be known.
people may be expressed, crystallized and considered, and where disputes may be
3. Inhabitants – the people residing in the territory of the corporation. amicably settled [Sec. 384, R.A. 7160].

4. Territory – the land mass where the inhabitants reside, together with the internal 5. Autonomous Regions in Muslim Mindanao and the Cordilleras. (Sec. 1, Art. X,
and external waters, and the air space above the land and waters. Constitution), In Limbonas v. Mangelin, supra, relative to the establishment of the
autonomous regional governments in Regions IX and XII under the 1973
Criterion to determine whether a corporation is a public corporation Constitution, it was held that autonomy is either decentralization of administration
It is the relationship of the corporation to the state. If it was created by the State as its or decentralization of power. The second is abdication by the national government
own agency to help it in carrying out its governmental functions, it is public. Otherwise, of political power in favor of the local government; the first consists merely in the
it is private. delegation of administrative powers to broaden the base of governmental power.
The regional governments in Regions IX and XII are of the first variety. In Datu
Dual characteristic of public corporation Firdausi Abbas v. Comelec, 179 SCRA 287, RA 6734, the organic act establishing
the Autonomous Regional Government of Muslim Mindanao was held valid. It was
1. Public or governmental - It acts as an agent of the State as the government of
passed pursuant to the mandate in Art. X, Constitution. In Cordillera Broad Coalition
the territory it occupies and its inhabitants within the municipal limits. The
v. Commission on Audit, 181 SCRA 495, Executive Order No. 220, issued by

M.R.A.D.C. LUMBRE 258


CONSTITUTIONAL LAW REVIEW

President Aquino in the exercise of legislative powers, creating the Cordillera Created special laws; or
Administrative Region [CAR] was held valid. It prepared the groundwork for 2) General corporation
autonomy and the adoption of the organic law. In Ordillo v. Comelec, 192 SCRA law, as a stock or non-
100, the sole province of Ifugao which, in the plebiscite, alone voted in favor of RA stock
6766, cannot validly constitute the Autonomous Region of the Cordilleras.
NOTE: There is no CAR as only one province voted in favor thereof.
2. Classifications
6. Special Metropolitan Political Subdivisions. Pursuant to Sec. 11, Art. X,
Constitution, Congress may, by law, create special metropolitan political Kinds of Corporations
subdivisions subject to a plebiscite set forth in Sec. 10, (but) the component cities
and municipalities shall retain their basic autonomy and shall be entitled to their 1. Quasi-public corporations - Private corporations that render public service,
own local executives and legislative assemblies. The jurisdiction of the metropolitan supply public wants, or pursue other eleemosynary objectives. While purposely
authority that will thereby be created shall be limited to basic services requiring organized for the gain or benefit of its members, they are required by law to
coordination. discharge functions for the public benefit. It must be stressed that a quasi-public
corporation is a specie of private corporation, but the qualifying factor is the type
1. Distinguished from government-owned or controlled corporations of service the former renders to the public: if it performs a public service, then it
Government owned and controlled corporations (GOCC) becomes a quasi-public corporation. (Philippine Society for the Prevention of
Any agency organized as a stock or non-stock corporation, vested with functions Cruelty to Animals v. Commission on Audit, G.R. No. 169752, September 25, 2007)
relating to public needs, whether governmental or proprietary in nature, and owned
by the Government of the Republic of the Philippines directly or through its 2. Municipal corporations - A political and corporate body constituted by the
instrumentalities either wholly or, where applicable as in the case of stock incorporation of inhabitants for the purpose of local government. It is established
corporations, to the extent of at least a majority of its outstanding capital stock. by law, partly as an agency of the State to assist in the civil government of the
[R.A. 10149, GOCC Governance Act of 2011, Chapter 1, Sec. 3 (o)] country, but chiefly to regulate and administer the local or internal affairs of the
city, town or district which is incorporated.
NOTE: Provided that such agencies are further categorized by Department of
Budget, CSC, and COA for purposes of the exercise and discharge of their respective
powers, functions and responsibilities. [E.O. No. 292, Administrative Code of 1987, II. Municipal Corporations
Sec. 2 (13)]

Elements of a GOCC
1. Requisites for creation, conversion, division, merger or dissolution
1. Any agency organized as a stock or non-stock corporation;
A local government unit may be created, divided, merged, abolished, or its boundaries
2. Vested with functions relating to public needs whether governmental or substantially altered either by law enacted by Congress in the case of a province, city,
proprietary in nature; municipality or any other political subdivision, or by ordinance passed by the
sangguniang panlalawigan or sangguniang panlungsod concerned in the case of a
3. Owned by the government directly or through its instrumentalities either barangay located within its territorial jurisdiction, subject to such limitations and
wholly, or, where applicable as in the case of stock corporations, to the extent requirements prescribed in the Local Government Code [Sec. 6, R.A. 7160]
of at least fifty-one (51) of its capital stock. (Leyson, Jr. v. Office of the
Ombudsman, G.R. No. 134990, April 27, 2000) In Section 19, R.A. 9054, Congress delegated to the Autonomous Region in Muslim
Mindanao (ARMM) the power to create provinces, cities, municipalities and barangays
Basis Public Corporation GOCC within the ARMM. Challenged as unconstitutional in Sema v. Comelec, G.R. No.
177597, July 16, 2008, the Supreme Court said: There is no provision in the Constitution
As to Administration of Performance of
Purpose local government or functions relating to that conflicts with the delegation to regional legislative bodies of the power to create
municipalities and barangays, provided Section 10, Article X of the Constitution is
rendering service for public needs, whether
the public interest. Governmental or followed. However, the creation of provinces and cities is another matter. Section 5 (3),
Article VI of the Constitution provides that each city with a population of at least two
Proprietary in nature.
hundred fifty thousand, or each province, shall have at least one representative. Section
As to Who Created by the state, Created by Congress or
3 of the Ordinance appended to the Constitution provides that any province that may
Creates either by general act by incorporators.
hereafter be created, or any city whose population may hereafter increase to more than
or special act.
two hundred fifty thousand shall be entitled in the immediately following election to at
As to How Through legislation. 1) Original charters or

M.R.A.D.C. LUMBRE 259


CONSTITUTIONAL LAW REVIEW

least one Member (in the House of Representatives). Pursuant to these provisions, a iic) City : 150,000
province cannot be created without creating a legislative district; nor can a city with a
population of 250,000 or more be created without a legislative district. Thus, the power iid) Highly urbanized city: 200,000
to create a province or a city with a population of 250,000 or more requires the power iie) Province: 250,000
to create a legislative district. Accordingly, the delegation granted by Congress to the
ARMM to create provinces and cities is unconstitutional, because Congress cannot iii. Land Area. It must be contiguous, unless it comprises two or more
validly delegate the power to create legislative districts for the House of islands or is separated by a local government unit independent of the
Representatives, since the power to increase the allowable membership in the House of others; properly identified by metes and bounds with technical
Representatives and to reapportion legislative districts, is vested exclusively in descriptions and sufficient to provide for such basic services and facilities
Congress. to meet the requirements of its populace. Area requirements are:

Requisites/Limitations on Creation or Conversion iiia) Municipality: 50 square kilometers

1. Sec. 10. Art. X. Constitution: No province, city, municipality or barangay may iiib) City : 100 square kilometers
be created, divided, merged, abolished, or its boundary substantially altered,
iiic) Province: 2,000 square kilometers
except in accordance with the criteria established in the local government code and
subject to approval by a majority of the votes cast in a plebiscite in the political c. Other Constitutional Limitations. Provisions of the Bill of Rights affording
units directly affected. protection to rights, property, and contracts of inhabitants.
a. Plebiscite requirement: No creation, division, merger, abolition or NOTE: Compliance with the foregoing indicators shall be attested to by:
substantial alteration of boundaries of local government units shall take effect
unless approved by a majority of the votes cast in a plebiscite called for the 1. The Department of Finance (Income requirement);
purpose in the political unit or units directly affected. Said plebiscite shall be 2. NSO (Population requirement); and
conducted by the Comelec within 120 days from the date of effectivity of the
law or ordinance effecting such action, unless said law or ordinance fixes 3. The Lands Management Bureau of DENR (Land requirement) [LGC, Sec. 7(c)]
another date [Sec. 10, R.A. 7160].
NOTE: Creation pertains to the creation of a new local government unit, whereas in
b. Section 7, R.A. No. 7160. Based verifiable indicators of viability and conversion, one local government unit shall cease to exist and a new one will arise, e.g.
projected capacity to provide services, to wit: component city converted into a highly urbanized city, or a municipality into a city.

i. Income – must be sufficient, based on acceptable standards, to provide Beginning of Corporate Existence
for all essential government facilities and services and special functions
Upon the election and qualification of its chief executive and a majority of the members
commensurate with the size of its population, as expected of the local
of its Sanggunian, unless some other time is fixed therefor by the law or ordinance
government unit concerned. Average annual income for the last two
creating it. (Section 14, R.A. No. 7160)
consecutive years based on 1991 constant prices should be at least:
Division and Merger, Abolition of Local Government Units
ia) Municipality: P2,500,000.00
1. Division and Merger. Shall comply with the same requirements, provided that:
ib) City: P100,000,000.00 (Year 2000 constant prices, amended by
R.A. 9009) a. such division shall not reduce the income, population or land area of the
local government unit or units concerned to less than the minimum
ic) Highly urbanized city: P50,000,000.00
requirements prescribed;
id) Province : P20,000,000.00
b. provided further that the income classification of the original local
ii. Population. It shall be determined as the total number of inhabitants government unit or units shall not fall below its current income classification
within the territorial jurisdiction of the local government unit concerned. prior to the division [Sec. 8, R.A. 7160];

Required minimum population for: c. Plebiscite be held in LGUs affected [Sec. 10, R.A. 7160)

iia) Barangay : 2,000 inhabitants (except in Metro Manila and other d. Assets and liabilities of the municipality/ies affected by such organization or
metropolitan political subdivisions or in highly urbanized cities, where creation of a new municipality shall be equitably distributed between the LGUs
the requirement is 5,000 inhabitants] affected and the new LGU. (Section 1(3), R.A. 688)

iib) Municipality: 25,000

M.R.A.D.C. LUMBRE 260


CONSTITUTIONAL LAW REVIEW

NOTE: When a municipal district of other territorial divisions is converted or fused General Rule: No collateral attack shall lie; an inquiry into the legal existence of
into a municipality, all property rights vested in original territorial organization shall a municipal corporation is reserved to the State in a proceeding for quo warranto
become vested in the government of the municipality. (Section 1(3), R.A. 688) or other direct proceeding. But this rule applies only when the municipal corporation
is, at least, a de facto municipal corporation.
b. Abolition. A local government unit may be abolished when its income, population
or land area has been irreversibly reduced to less than the minimum standards Exception: Where the challenge was made nearly 30 years after the executive
prescribed for its creation, as certified by the national agencies mentioned. The law order creating the municipality was issued, or where the municipality has been in
or ordinance abolishing a local government unit shall specify the province, city, existence for all of 16 years before the ruling in Pelaez v. Auditor General was
municipality or barangay with which the local government unit sought to be promulgated, and various governmental acts throughout the years all indicate the
abolished will be incorporated or merged [Sec. 9, R.A. 7160]. May be abolished by: State’s recognition and acknowledgment of the existence of the municipal
corporation, then the municipal corporation should be considered as a regular, de
i. Congress – in case of provinces, city, municipality, or any other political jure municipality.
subdivision;
The Local Government Code (R.A. No. 7160)
ii. Sangguniang Panlalawigan or Sangguniang Panlungsod – in case of
a barangay. Effectivity: January 1, 1992, unless otherwise provided herein, after its complete
publication in at least one newspaper of general circulation. (Section 536)
Exception: Metropolitan Manila area and in cultural communities.
Scope of Application: The Code shall apply to all provinces, cities, municipalities,
Required vote on creation, division, merger, abolition, or substantial alteration barangays and other political subdivisions as may be created by law, and, to the extent
of boundaries of LGUs herein provided, to officials, offices or agencies of the National Government (Section
Majority of the votes cast in a plebiscite called for the purpose in the political unit or 4)
units directly affected. a. Navarro vs. Executive Secretary Ermita, Min. Res., G.R. No. 180050, 11
NOTE: Said plebiscite shall be conducted by the COMELEC within one hundred twenty September 2012
(120) days from the date of effectivity of the law or ordinance effecting such action, It must be borne in mind that the central policy considerations in the creation of
unless said law or ordinance fixes another date. (LGC, Sec. 10) local government units are economic viability, efficient administration, and
A barangay may officially exist on record and the fact that nobody resides in the place capability to deliver basic services to their constituents. The criteria prescribed by
does not result in its automatic cessation as a unit of local government. the LGC: (a) income, (b) population and (c) land area, are all designed to
accomplish these results. In this light, Congress, in its collective wisdom, has
Under the LGC of 1991, the abolition of an LGU may be done by Congress in the case debated on the relative weight of each of these three criteria, placing emphasis on
of a province, city, municipality, or any other political subdivision. In the case of a which of them should enjoy preferential consideration.
barangay, except in Metropolitan Manila area and in cultural communities, it may be
done by the Sangguniang Panlalawigan or Sangguniang Panglungsod concerned subject NOTE: This is an excerpt of the en banc resolution of the SC dated April 12, 2011,
to the mandatory requirement of a plebiscite conducted for the purpose in the political which granted the motion for reconsideration. Here, the Court set aside its ruling
units affected. (Sarangani v. COMELEC, G.R. No. 135927, June 26, 2000) of February 10, 2010, declaring the provision in Article 9(2) of the Rules and
Regulations Implementing the Local Government Code of 1991 stating, "The land
De Facto Municipal Corporations; Requisites area requirement shall not apply where the proposed province is composed of one
(1) or more islands," as VALID. Accordingly, Republic Act No. 9355 (An Act
1. Valid law authorizing incorporation;
Creating the Province of Dinagat Islands) is declared as VALID and
2. Attempt in good faith to organize under it; CONSTITUTIONAL, and the proclamation of the Province of Dinagat Islands and the
election of the officials thereof are declared VALID. I could not find the full text of
3. Colorable compliance with the law; and the Resolution dated September 11, 2012, but the said Resolution denied with
4. Assumption of corporate powers. finality the motion for reconsideration on the Resolution dated April 12, 2011.

In Pelaez v. Auditor General, 15 SCRA 569, the Supreme Court declared as Constitution, Article X—Local Government Section 10. No province, city,
unconstitutional Sec. 68 of the Revised Administrative Code which authorized the municipality, or barangay may be created, divided, merged, abolished, or its
President to create municipalities through Executive Order. With this declaration, boundary substantially altered, except in accordance with the criteria
municipalities created by Executive Order could not claim to be de facto municipal established in the local government code and subject to the approval by a majority
corporations, because there was no valid law authorizing incorporation.’ of the votes cast in a plebiscite in the political units directly affected.

M.R.A.D.C. LUMBRE 261


CONSTITUTIONAL LAW REVIEW

SECTION. 10. Plebiscite Requirement.—No creation, division, merger, (b) The territorial jurisdiction of the new barangay shall be properly identified by
abolition, or substantial alteration of boundaries of local government units metes and bounds or by more or less permanent natural boundaries. The territory
shall take effect unless approved by a majority of the votes cast in a plebiscite need not be contiguous if it comprises two (2) or more islands.
called for the purpose in the political unit or units directly affected. Said plebiscite
shall be conducted by the Commission on Elections (COMELEC) within one hundred (c) The governor or city mayor may prepare a consolidation plan for barangays,
twenty (120) days from the date of effectivity of the law or ordinance effecting such based on the criteria prescribed in this Section, within his territorial jurisdiction.
action, unless said law or ordinance fixes another date. The plan shall be submitted to the sangguniang panlalawigan or sangguniang
panlungsod concerned for appropriate action. In the case of municipalities within
Creation and Conversion the Metropolitan Manila area and other metropolitan political subdivisions, the
SEC. 7. Creation and Conversion.—As a general rule, the creation of a local barangay consolidation plan can be prepared and approved by the sangguniang
government unit or its conversion from one level to another level shall be based on bayan concerned.
verifiable indicators of viability and projected capacity to provide services, to wit:
LGC-IRR: ARTICLE 14. Barangays.—
(a) Income.—It must be sufficient, based on acceptable standards, to provide for (a) Creation of barangays by the sangguniang panlalawigan shall require prior
all essential government facilities and services and special functions commensurate recommendation of the sangguniang bayan.
with the size of its population, as expected of the local government unit concerned;
(b) New barangays in the municipalities within MMA shall be created only by Act
(b) Population.—It shall be determined as the total number of inhabitants within of Congress, subject to the limitations and requirements prescribed in this Article.
the territorial jurisdiction of the local government unit concerned; and
(c) Notwithstanding the population requirement, a barangay may be created in the
(c) Land area.—It must be contiguous, unless it comprises two (2) or more islands indigenous cultural communities by Act of Congress upon recommendation
or is separated by a local government unit independent of the others; properly of the LGU or LGUs where the cultural community is located.
identified by metes and bounds with technical descriptions; and sufficient to provide
for such basic services and facilities to meet the requirements of its populace. (d) A barangay shall not be created unless the following requisites are present:
(1) Population—which shall not be less than two thousand (2,000) inhabitants,
It must be borne in mind that the central policy considerations in the creation of except in municipalities and cities within MMA and other metropolitan political
local government units are (1) economic viability, (2) efficient subdivisions as may be created by law, or in highly-urbanized cities where such
administration, and (3) capability to deliver basic services to their territory shall have a population of at least five thousand (5,000) inhabitants, as
constituents. The criteria prescribed by the LGC, i.e., income, population and land certified by the NSO. The creation of a barangay shall not reduce the population of
area, are all designed to accomplish these results. Without doubt, the primordial the original barangay or barangays to less than the prescribed minimum.
criterion in the creation of local government units, particularly of a province, is (2) Land Area—which must be contiguous, unless comprised by two (2) or more
economic viability. This is the clear intent of the framers of the LGC. islands. The territorial jurisdiction of a barangay sought to be created shall be
properly identified by metes and bounds or by more or less permanent natural
Also worthy of note are the requisites in the creation of a barangay, a boundaries.
municipality, a city, and a province as provided both in the LGC and the
LGC-IRR, viz.:— Municipality: LGC: SEC. 442. Requisites for Creation.—
(a) A municipality may be created if it has an average annual income, as certified
For a Barangay: LGC: SEC. 386. Requisites for Creation.— by the provincial treasurer, or at least Two million five hundred thousand pesos
(P2,500,000.00) for the last two (2) consecutive years based on the 1991 constant
(a) A barangay may be created out of a contiguous territory which has a prices; a population of at least twenty-five thousand (25,000) inhabitants as
population of at least two thousand (2,000) inhabitants as certified by the certified by the National Statistics Office; and a contiguous territory of at least fifty
National Statistics Office except in cities and municipalities within Metro Manila (50) square kilometers as certified by the Lands Management Bureau: Provided,
and other metropolitan political subdivisions or in highly urbanized cities where That the creation thereof shall not reduce the land area, population or income of
such territory shall have a certified population of at least five thousand (5,000) the original municipality or municipalities at the time of said creation to less than
inhabitants: Provided, That the creation thereof shall not reduce the population of the minimum requirements prescribed herein.
the original barangay or barangays to less than the minimum requirement
prescribed herein. To enhance the delivery of basic services in the indigenous (b) The territorial jurisdiction of a newly-created municipality shall be properly
cultural communities, barangays may be created in such communities by an Act of identified by metes and bounds. The requirement on land area shall not apply where
Congress, notwithstanding the above requirement. the municipality proposed to be created is composed of one (1) or more islands.
The territory need not be contiguous if it comprises two (2) or more islands.

M.R.A.D.C. LUMBRE 262


CONSTITUTIONAL LAW REVIEW

proposed to be created is composed of one (1) or more islands. The territory need
(c) The average annual income shall include the income accruing to the general not be contiguous if it comprises two (2) or more islands.
fund of the municipality concerned, exclusive of special funds, transfers and non-
recurring income. (c) The average annual income shall include the income accruing to the general
(d) Municipalities existing as of the date of effectivity of this Code shall continue to fund, exclusive of special funds, transfers, and non-recurring income.
exist and operate as such. Existing municipal districts organized pursuant to
presidential issuances or executive orders and which have their respective set of LGC-IRR: ARTICLE 11. Cities.—(a) Requisites for creation—A city shall not be
elective municipal officials holding office at the time of the effectivity of this Code created unless the following requisites on income and either population or land area
shall henceforth be considered regular municipalities. LGC- are present:
(1) Income—An average annual income of not less than Twenty Million Pesos
IRR: ARTICLE 13. Municipalities.—(a) Requisites for Creation —A (P20,000,000.00), for the immediately preceding two (2) consecutive years based
municipality shall not be created unless the following requisites are present: on 1991 constant prices, as certified by DOF. The average annual income shall
include the income accruing to the general fund, exclusive of special funds, special
(i) Income—An average annual income of not less than Two Million Five Hundred accounts, transfers, and nonrecurring income; and
Thousand Pesos (P2,500,000.00), for the immediately preceding two (2)
consecutive years based on 1991 constant prices, as certified by the provincial (2) Population or land area—Population which shall not be less than one hundred
treasurer. The average annual income shall include the income accruing to the fifty thousand (150,000) inhabitants, as certified by the NSO; or land area which
general fund, exclusive of special funds, special accounts, transfers, and must be contiguous with an area of at least one hundred (100) square kilometers,
nonrecurring income; as certified by LMB. The territory need not be contiguous if it comprises two (2) or
more islands or is separated by a chartered city or cities which do not contribute to
(ii) Population—which shall not be less than twenty five thousand (25,000) the income of the province. The land area requirement shall not apply where the
inhabitants, as certified by NSO; and proposed city is composed of one (1) or more islands. The territorial jurisdiction of
a city sought to be created shall be properly identified by metes and bounds.
(iii) Land area—which must be contiguous with an area of at least fifty (50) square
kilometers, as certified by LMB. The territory need not be contiguous if it comprises The creation of a new city shall not reduce the land area, population, and income
two (2) or more islands. The requirement on land area shall not apply where the of the original LGU or LGUs at the time of said creation to less than the prescribed
proposed municipality is composed of one (1) or more islands. The territorial minimum requirements. All expenses incidental to the creation shall be borne by
jurisdiction of a municipality sought to be created shall be properly identified by the petitioners.
metes and bounds. The creation of a new municipality shall not reduce the land
area, population, and income of the original LGU or LGUs at the time of said creation Provinces: LGC: SEC. 461. Requisites for Creation.—
to less than the prescribed minimum requirements. All expenses incidental to the (a) A province may be created if it has an average annual income, as certified by
creation shall be borne by the petitioners. the Department of Finance, of not less than Twenty million pesos (P20,000,000.00)
based on 1991 prices and either of the following requisites:
City: LGC: SEC. 450. Requisites for Creation.—
(a) A municipality or a cluster of barangays may be converted into a component (i) a contiguous territory of at least two thousand (2,000) square kilometers, as
city if it has an average annual income, as certified by the Department of Finance, certified by the Lands Management Bureau; or,
of at least Twenty million pesos (P20,000,000.00) for the last two (2) consecutive (ii) a population of not less than two hundred fifty thousand (250,000) inhabitants
years based on 1991 constant prices, and if it has either of the following requisites: as certified by the National Statistics Office: Provided, That the creation thereof
shall not reduce the land area, population, and income of the original unit or units
(i) a contiguous territory of at least one hundred (100) square kilometers, as at the time of said creation to less than the minimum requirements prescribed
certified by the Lands Management Bureau; or, herein.
(ii) a population of not less than one hundred fifty thousand (150,000)
inhabitants, as certified by the National Statistics Office: Provided, That, the (b) The territory need not be contiguous if it comprises two (2) or more
creation thereof shall not reduce the land area, population, and income of the islands or is separated by a chartered city or cities which do not contribute to
original unit or units at the time of said creation to less than the minimum the income of the province.
requirements prescribed herein. (c) The average annual income shall include the income accruing to the general
fund, exclusive of special funds, trust funds, transfers, and non-recurring income.
(b) The territorial jurisdiction of a newly-created city shall be properly identified by
metes and bounds. The requirement on land area shall not apply where the city

M.R.A.D.C. LUMBRE 263


CONSTITUTIONAL LAW REVIEW

LGC-IRR: ARTICLE 9. Provinces.—(a) Requisites for creation—A province shall be in order for the Court to uphold the validity of Article 9(2) of the LGC-IRR. This
not be created unless the following requisites on income and either population or interpretation finds merit when we consider the basic policy considerations
land area are present: underpinning the principle of local autonomy.

(1) Income—An average annual income of not less than Twenty Million pesos b. Umali vs. COMELEC, G.R. No. 203974. April 22, 2014 (NOTE: Asked in the
(P20,000,000.00) for the immediately preceding two (2) consecutive years based 2014 Bar Exams.)
on 1991 constant prices, as certified by DOF. The average annual income shall The general principle is that legislative power cannot be delegated. Nonetheless, the
include the income accruing to the general fund, exclusive of special funds, special general rule barring delegation is subject to certain exceptions allowed in the
accounts, transfers, and non-recurring income; and Constitution, namely:

(2) Population or land area—Population which shall not be less than two hundred (1) Delegation by Congress to the President of the power to fix "tariff rates,
fifty thousand (250,000) inhabitants, as cer tified by NSO; or land area which must import and export quotas, tonnage and wharfage dues, and other duties or
be contiguous with an area of at least two thousand (2,000) square kilometers, as imposts within the framework of the national development program of the
certified by LMB. The territory need not be contiguous if it comprises two (2) or Government" under Section 28(2) of Article VI of the Constitution; and
more islands or is separated by a chartered city or cities which do not contribute to
(2) Delegation of emergency powers by Congress to the President "to exercise
the income of the province. The land area requirement shall not apply where the
powers necessary and proper to carry out a declared national policy" in times
proposed province is composed of one (1) or more islands. The territorial
of war and other national emergency under Section 23(2) of Article VI of the
jurisdiction of a province sought to be created shall be properly identified by metes
Constitution.
and bounds.
The power to create, divide, merge, abolish or substantially alter boundaries of
The creation of a new province shall not reduce the land area, population, and provinces, cities, municipalities or barangays, which is pertinent in the case at bar, is
income of the original LGU or LGUs at the time of said creation to less than the essentially legislative in nature.5 The framers of the Constitution have, however, allowed
prescribed minimum requirements. All expenses incidental to the creation shall be for the delegation of such power in Sec. 10, Art. X of the Constitution as long as (1) the
borne by the petitioners.” (Emphasis supplied.) criteria prescribed in the LGC is met and (2) the creation, division, merger, abolition or
the substantial alteration of the boundaries is subject to the approval by a majority vote
It bears scrupulous notice that from the above cited provisions, with respect to the in a plebiscite.
creation of barangays, land area is not a requisite indicator of viability. However,
with respect to the creation of municipalities, component cities, and True enough, Congress delegated such power to the Sangguniang Panlalawigan or
provinces, the three (3) indicators of viability and projected capacity to Sangguniang Panlungsod to create barangays pursuant to Sec. 6 of the LGC, which
provide services, i.e., income, population, and land area, are provided for. provides:
But it must be pointed out that when the local government unit to be created Section 6. Authority to Create Local Government Units. - A local government unit
consists of one (1) or more islands, it is exempt from the land area requirement as may be created, divided, merged, abolished, or its boundaries substantially altered
expressly provided in Section 442 and Section 450 of the LGC if the local either by law enacted by Congress in the case of a province, city, municipality, or
government unit to be created is a municipality or a component city, respectively. any other political subdivision, or by ordinance passed by the sangguniang
This exemption is absent in the enumeration of the requisites for the creation of a panlalawigan or sangguniang panlungsod concerned in the case of a barangay
province under Section 461 of the LGC, although it is expressly stated under Article located within its territorial jurisdiction, subject to such limitations and
9(2) of the LGC-IRR. requirements prescribed in this Code."

There appears neither rhyme nor reason why this exemption should apply The guidelines for the exercise of this authority have sufficiently been outlined by the
to cities and municipalities, but not to provinces. In fact, considering the various LGC provisions detailing the requirements for the creation of barangays 6,
physical configuration of the Philippine archipelago, there is a greater likelihood municipalities7, cities8, and provinces9. Moreover, compliance with the plebiscite
that islands or group of islands would form part of the land area of a newly-created requirement under the Constitution has also been directed by the LGC under its Sec.
province than in most cities or municipalities. 10, which reads:
Section 10. Plebiscite Requirement. – No creation, division, merger,
It is, therefore, logical to infer that the genuine legislative policy decision
abolition, or substantial alteration of boundaries of local government units
was expressed in Section 442 (for municipalities) and Section 450 (for
shall take effect unless approved by a majority of the votes cast in a
component cities) of the LGC, but was inadvertently omitted in Section 461
plebiscite called for the purpose in the political unit or units directly
(for provinces). Thus, when the exemption was expressly provided in Article 9(2)
affected."
of the LGC-IRR, the inclusion was intended to correct the congressional oversight
in Section 461 of the LGC—and to reflect the true legislative intent. It would, then,

M.R.A.D.C. LUMBRE 264


CONSTITUTIONAL LAW REVIEW

With the twin criteria of standard and plebiscite satisfied, the delegation to LGUs It was determined in the case that the changes that will result from the conversion are
of the power to create, divide, merge, abolish or substantially alter boundaries has too substantial that there is a necessity for the plurality of those that will be affected to
become a recognized exception to the doctrine of non-delegation of legislative powers. approve it. Similar to the enumerated acts in the constitutional provision, conversions
were found to result in material changes in the economic and political rights of the
Likewise, legislative power was delegated to the President under Sec. 453 of the LGC people and LGUs affected. Given the far-reaching ramifications of converting the status
quoted earlier, which states: of a city, we held that the plebiscite requirement under the constitutional provision
Section 453. Duty to Declare Highly Urbanized Status. – It shall be the duty of the should equally apply to conversions as well.
President to declare a city as highly urbanized within thirty (30) days after it shall Second, while conversion to an HUC is not explicitly provided in Sec. 10, Art. X of the
have met the minimum requirements prescribed in the immediately preceding Constitution we nevertheless observe that the conversion of a component city into an
Section, upon proper application therefor and ratification in a plebiscite by the HUC is substantial alteration of boundaries.
qualified voters therein.
As the phrase implies, "substantial alteration of boundaries" involves and
In this case, the provision merely authorized the President to make a determination on necessarily entails a change in the geographical configuration of a local
whether or not the requirements under Sec. 452 of the LGC are complied with. The government unit or units. However, the phrase "boundaries" should not be
provision makes it ministerial for the President, upon proper application, to declare a limited to the mere physical one, referring to the metes and bounds of the LGU,
component city as highly urbanized once the minimum requirements, which are based but also to its political boundaries. It also connotes a modification of the
on certifiable and measurable indices under Sec. 452, are satisfied. The mandatory demarcation lines between political subdivisions, where the LGU’s exercise of
language "shall" used in the provision leaves the President with no room for discretion. corporate power ends and that of the other begins. And as a qualifier, the
In so doing, Sec. 453, in effect, automatically calls for the conduct of a plebiscite for alteration must be "substantial" for it to be within the ambit of the constitutional
purposes of conversions once the requirements are met. No further legislation is provision.
necessary before the city proposed to be converted becomes eligible to become an HUC Pertinent is Art. 12(c) of the LGC’s Implementing Rules and Regulations, which reads:
through ratification, as the basis for the delegation of the legislative authority is the
very LGC. Art. 12. Conversion of a Component City into a Highly Urbanized City. –

In view of the foregoing considerations, the Court concludes that the source of the xxxx
delegation of power to the LGUs under Sec. 6 of the LGC and to the President under
Sec. 453 of the same code is none other than Sec. 10, Art. X of the Constitution. (c) Effect of Conversion – The conversion of a component city into a highly-
urbanized city shall make it independent of the province where it is geographically
In Miranda, the rationale behind the afore-quoted constitutional provision and its located.
application to cases of conversion were discussed thusly:
Verily, the upward conversion of a component city, in this case Cabanatuan City, into
A close analysis of the said constitutional provision will reveal that the creation, division, an HUC will come at a steep price. It can be gleaned from the above-cited rule that the
merger, abolition or substantial alteration of boundaries of local government units province will inevitably suffer a corresponding decrease in territory brought about by
involve a common denominator - - - material change in the political and economic Cabanatuan City’s gain of independence. With the city’s newfound autonomy, it will be
rights of the local government units directly affected as well as the people free from the oversight powers of the province, which, in effect, reduces the territorial
therein. It is precisely for this reason that the Constitution requires the approval of the jurisdiction of the latter. What once formed part of Nueva Ecija will no longer be subject
people "in the political units directly affected." It is not difficult to appreciate the to supervision by the province. In more concrete terms, Nueva Ecija stands to lose
rationale of this constitutional requirement. The 1987 Constitution, more than any of 282.75 sq. km. of its territorial jurisdiction with Cabanatuan City’s severance from its
our previous Constitutions, gave more reality to the sovereignty of our people for it was mother province. This is equivalent to carving out almost 5% of Nueva Ecija’s 5,751.3
borne out of the people power in the 1986 EDSA revolution. Its Section 10, Article X sq. km. area. This sufficiently satisfies the requirement that the alteration be
addressed the undesirable practice in the past whereby local government units were "substantial."
created, abolished, merged or divided on the basis of the vagaries of politics and not of
the welfare of the people. Thus, the consent of the people of the local government unit Needless to stress, the alteration of boundaries would necessarily follow Cabanatuan
directly affected was required to serve as a checking mechanism to any exercise of City’s conversion in the same way that creations, divisions, mergers, and abolitions
legislative power creating, dividing, abolishing, merging or altering the boundaries of generally cannot take place without entailing the alteration. The enumerated acts, after
local government units. It is one instance where the people in their sovereign capacity all, are not mutually exclusive, and more often than not, a combination of these acts
decide on a matter that affects them - - - direct democracy of the people as opposed to attends the reconfiguration of LGUs.
democracy thru people’s representatives. This plebiscite requirement is also in accord In light of the foregoing disquisitions, the Court rules that conversion to an HUC is
with the philosophy of the Constitution granting more autonomy to local government substantial alternation of boundaries governed by Sec. 10, Art. X and resultantly, said
units. provision applies, governs and prevails over Sec. 453 of the LGC.

M.R.A.D.C. LUMBRE 265


CONSTITUTIONAL LAW REVIEW

The entire province of Nueva Ecija will be directly affected by Cabanatuan City’s (i) a contiguous territory of at least two thousand (2,000) square
conversion. kilometers, as certified by the Lands Management Bureau; or

After the Court has resolved the seeming irreconcilability of Sec. 10, Art. X of the (ii) a population of not less than two hundred fifty thousand (250,000)
Constitution and Sec. 453 of the LGC, it is now time to elucidate the meaning of the inhabitants as certified by the National Statistics Office:
phrase "political units directly affected" under Sec. 10, Art. X.
Provided, That, the creation thereof shall not reduce the land area, population, and
a. "Political units directly affected" defined income of the original unit or units at the time of said creation to less than the minimum
requirements prescribed herein.
In identifying the LGU or LGUs that should be allowed to take part in the plebiscite,
what should primarily be determined is whether or not the unit or units that desire to A component city’s conversion into an HUC and its resultant autonomy from the province
participate will be "directly affected" by the change. is a threat to the latter’s economic viability. Noteworthy is that the income criterion for
a component city to be converted into an HUC is higher than the income requirement
In the more recent case of Miranda, the interpretation in Tan and Padilla was modified for the creation of a province. The ensuing reduction in income upon separation would
to include not only changes in economic but also political rights in the criteria for clearly leave a crippling effect on the province’s operations as there would be less
determining whether or not an LGU shall be considered "directly affected." funding to finance infrastructure projects and to defray overhead costs. Moreover, the
Nevertheless, the requirement that the plebiscite be participated in by the plurality of quality of services being offered by the province may suffer because of looming austerity
political units directly affected remained. measures. These are but a few of the social costs of the decline in the province’s
b. Impact on Economic Rights economic performance, which Nueva Ecija is bound to experience once its most
progressive city of Cabanatuan attains independence.
To recall, it was held in Miranda that the changes that will result in the downgrading of
an LGU from an independent component city to a component city cannot be categorized c. Impact on Political Rights
as insubstantial, thereby necessitating the conduct of a plebiscite for its ratification. In Aside from the alteration of economic rights, the political rights of Nueva Ecija and those
a similar fashion, herein petitioner Umali itemized the adverse effects of Cabanatuan of its residents will also be affected by Cabanatuan’s conversion into an HUC.
City’s conversion to the province of Nueva Ecija to justify the province’s participation in
the plebiscite to be conducted. Duties, privileges and obligations appertaining to HUCs will attach to Cabanatuan City
if it is converted into an HUC. This includes the right to be outside the general
Often raised is that Cabanatuan City’s conversion into an HUC and its severance from supervision of the province and be under the direct supervision of the President. An
Nueva Ecija will result in the reduction of the Internal Revenue Allotment (IRA) to the HUC is not subject to provincial oversight because the complex and varied problems in
province based on Sec. 285 of the LGC. an HUC due to a bigger population and greater economic activity require greater
In our earlier disquisitions, we have explained that the conversion into an HUC carries autonomy. The provincial government stands to lose the power to ensure that the local
the accessory of substantial alteration of boundaries and that the province of Nueva government officials of Cabanatuan City act within the scope of its prescribed powers
Ecija will, without a doubt, suffer a reduction in territory because of the severance of and functions to review executive orders issued by the city mayor, and to approve
Cabanatuan City. The residents of the city will cease to be political constituencies of the resolutions and ordinances enacted by the city council. The province will also be divested
province, effectively reducing the latter’s population. Taking this decrease in territory of jurisdiction over disciplinary cases concerning the elected city officials of the new
and population in connection with the above formula, it is conceded that Nueva Ecija HUC, and the appeal process for administrative case decisions against barangay officials
will indeed suffer a reduction in IRA given the decrease of its multipliers’ values of the city will also be modified accordingly. Likewise, the registered voters of the city
will no longer be entitled to vote for and be voted upon as provincial officials.
Clear as crystal is that the province of Nueva Ecija will suffer a substantial reduction of
its share in IRA once Cabanatuan City attains autonomy. In view of the economic impact In cutting the umbilical cord between Cabanatuan City and the province of Nueva Ecija,
of Cabanatuan City’s conversion, petitioner Umali’s contention, that its effect on the the city will be separated from the territorial jurisdiction of the province, as earlier
province is not only direct but also adverse, deserves merit. explained. The provincial government will no longer be responsible for delivering basic
services for the city residents’ benefit. Ordinances and resolutions passed by the
Moreover, his claim that the province will lose shares in provincial taxes imposed in provincial council will no longer cover the city. Projects queued by the provincial
Cabanatuan City is well-founded. government to be executed in the city will also be suspended if not scrapped to prevent
the LGU from performing functions outside the bounds of its territorial jurisdiction, and
Section 461. Requisites for Creation.
from expending its limited resources for ventures that do not cater to its constituents.
(a) A province may be created if it has an average annual income, as certified
In view of these changes in the economic and political rights of the province of Nueva
by the Department of Finance, of not less than Twenty million pesos
Ecija and its residents, the entire province certainly stands to be directly affected by the
(₱20,000,000.00) based on 1991 constant prices and either of the following
conversion of Cabanatuan City into an HUC. Following the doctrines in Tan and Padilla,
requisites:

M.R.A.D.C. LUMBRE 266


CONSTITUTIONAL LAW REVIEW

all the qualified registered voters of Nueva Ecija should then be allowed to participate 7160, Sec. 16)
in the plebiscite called for that purpose.
Two branches of the General Welfare Clause
To limit the plebiscite to only the voters of the areas to be partitioned and seceded from
the province is as absurd and illogical as allowing only the secessionists to vote for the 1. General Legislative Power – Authorizes the municipal council to enact ordinances
secession that they demanded against the wishes of the majority and to nullify the basic and make regulations not repugnant to law, as may be necessary to carry into
principle of majority rule. effect and discharge the powers and duties conferred upon the municipal council
by law.
NOTE: Division of an LGU is allowed, provided that the creation of the new LGU does
not reduce the classification of the old one; that is, by reducing its income. 2. Police Power Proper – Authorizes the municipality to enact ordinances as may be
necessary and proper for the health and safety, prosperity, morals, peace, good
order, comfort, and convenience of the municipality and its inhabitants, and for the
protection of their property. (Rural Bank of Makati v. Municipality of Makati, July 2,
III. Powers (and Attributes) of Local Government Units (LGUs) 2004)
Requisites/Limitations

Sources 1. The interests of the public generally, as distinguished from those of a particular
class, require the interference of the state. (Equal Protection Clause)
1. Article II, Section 25; Article X, Sections 5, 6, 7;
2. The means employed are reasonably necessary for the attainment of the object
2. Statutes, e.g. R.A. No. 7610; sought to be accomplished and not duly oppressive. (Due Process Clause)
3. Charter (particularly those of cities; 3. Exercisable only within the territorial limits of the LGU, except for protection of
4. Doctrine of the right to self-government, but applies only in States which water supply. (LGC, Sec. 16)
adhere to the doctrine. 4. Must not be contrary to the Constitution and the laws.
Classification of Powers NOTE: There must be a concurrence of a lawful subject and lawful method.
1. Express, implied, inherent (powers necessary and proper for governance, e.g. (Lucena Grand Central v. JAC, G.R. No. 148339 February 23, 2005)
to promote health and safety, enhance prosperity, improve morals of inhabitants; Tests when police power is invoked as the rationale for the valid passage
2. Public or governmental, private or proprietary; of an ordinance

3. Intramural, extramural; 1. Rational relationship test – An ordinance must pass the requisites as discussed
above.
4. Mandatory, directory: ministerial, discretionary.
2. Strict scrutiny test – The focus is on the presence of compelling, rather than
1. Police power (general welfare clause) substantial, governmental interest and on the absence of less restrictive means for
achieving that interest. (Fernando v. St. Scholastica’s College, G.R. No. 161107,
Nature
March 12, 2013)
The police power of the LGU is not inherent. LGUs exercise the police power under
Governmental Powers
the general welfare clause. (LGC, Sec. 16)
1. General Welfare (Section 16) – Refer to discussion above.
General Welfare Clause
2. Basic Services and Facilities (Section 17) –
LGUs shall exercise powers that are necessary, appropriate, or incidental for its
efficient and effective governance, and those which are essential to the promotion a. Local government units shall endeavor to be self-reliant and shall continue
of general welfare. Within their respective territorial jurisdiction, LGUs shall ensure exercising the powers and discharging the duties and functions currently vested
and support, among other things, the preservation and enrichment of culture, upon them,
promote health and safety, enhance the right of the people to a balanced ecology,
b. They shall also discharge the functions and responsibilities of national agencies
encourage and support the development of appropriate and self-reliant scientific
and offices devolved to them pursuant to this Code [within six months after the
and technological capabilities, improve public morals, enhance economic prosperity
effectivity of this Code], They shall likewise exercise such other powers and
and social justice, promote full employment among its residents, maintain peace
discharge such other functions as are necessary, appropriate, or incidental to
and order, and preserve the comfort and convenience of their inhabitance. (RA

M.R.A.D.C. LUMBRE 267


CONSTITUTIONAL LAW REVIEW

efficient and effective provision of the basic services and facilities enumerated xxxx
herein.
(iv) Issue licenses and permits and suspend or revoke the same for any
Devolution - refers to the act by which the national government confers power and violation of the conditions upon which said licenses or permits had been issued,
authority upon the various local government units to perform specific functions and pursuant to law or ordinance.
responsibilities. This includes the transfer to the local government units of the
records, equipment and other assets and personnel of national agencies and offices. As Section 444(b)(3)(iv) so states, the power of the municipal mayor to issue
Regional offices of national agencies shall be phased out within one year from the licenses is pursuant to Section 16 of the Local Government Code of 1991, which
approval of this Code. Career regional directors who cannot be absorbed by the declares:
local government unit shall be retained by the national government, without SEC. 16. General Welfare. – Every local government unit shall exercise the
diminution in rank, salary or tenure. powers expressly granted, those necessarily implied therefrom, as well as
3. Power to Generate and Apply Resources (Section 18) - Local government units powers necessary, appropriate, or incidental for its efficient and effective
shall have the power and authority to establish an organization that shall be governance, and those which are essential to the promotion of the general
responsible for the efficient and effective implementation of their development welfare. Within their respective territorial jurisdictions, local government units
plans, program objectives and priorities; to create their own sources of revenue shall ensure and support, among other things, the preservation and
and to levy taxes, fees and charges which shall accrue exclusively to their use and enrichment of culture, promote health and safety, enhance the right of the
disposition and which shall be retained by them; to have a just share in the national people to a balanced ecology, encourage and support the development of
taxes which shall be automatically and directly released to them without need of appropriate and self-reliant scientific and technological capabilities, improve
any further action; to have an equitable share in the proceeds from the utilization public morals, enhance economic prosperity and social justice, promote full
and development of the national wealth and resources within their respective employment among their residents, maintain peace and order, and preserve
territorial jurisdictions including develop, lease, encumber, alienate or otherwise the comfort and convenience of their inhabitants.
dispose of real or personal property held by them in their proprietary capacity and Section 16, known as the general welfare clause, encapsulates the delegated police
to apply their resources and assets for productive, developmental or welfare power to local governments. Local government units exercise police power through
purposes, in the exercise or furtherance of their governmental or proprietary their respective legislative bodies. Evidently, the Local Government Code of 1991
powers and functions and thereby ensure their development into self-reliant is unequivocal that the municipal mayor has the power to issue licenses and permits
communities and active participants in the attainment of national goals. and suspend or revoke the same for any violation of the conditions upon which said
a. Rimando vs. Naguilian Emission Testing Center, Inc., 677 SCRA 343 licenses or permits had been issued, pursuant to law or ordinance. x x x
(2012) NOTE:
No, a mayor cannot be compelled by mandamus to issue a business permit since Q: Can a person compel an LGU via mandamus to clean the roads?
the exercise of the same is a delegated police power hence, discretionary in nature.
A: Yes, as it is a constitutional policy or mandate of the State to shall protect and
Section 444(b)(3)(iv) of the Local Government Code of 1991 is a manifestation of advance the right of the people to a balanced and healthful ecology in accord with
the delegated police power of a municipal corporation. Necessarily, the exercise the rhythm and harmony of nature.
thereof cannot be deemed ministerial. As to the question of whether the power is
validly exercised, the matter is within the province of a writ of certiorari, but b. Buklod nang Magbubukid sa Lupaing Ramos, Inc. vs. E. M. Ramos and
certainly, not of mandamus. Sons, Inc., 645 SCRA 401 (2011)

SEC. 444. The Chief Executive: Powers, Duties, Functions and Compensation. No, the subject properties are exempted from CARP. The Court reiterates that since
July 9, 1972, upon approval of Resolution No. 29-A by the Municipality of
(b) For efficient, effective and economical governance the purpose of which is Dasmarinas, the subject property had been reclassified from agricultural to
the general welfare of the municipality and its inhabitants pursuant to Section residential. The tax declarations covering the subject property, classifying the
16 of this Code, the municipal mayor shall: same as agricultural, cannot prevail over Resolution No. 29-A.
xxxx
Zoning classification is an exercise by the local government of police
3) Initiate and maximize the generation of resources and revenues, and apply power, not the power of eminent domain. A zoning ordinance is defined as
the same to the implementation of development plans, program objectives and a local city or municipal legislation which logically arranges, prescribes,
priorities as provided for under Section 18 of this Code, particularly those defines, and apportions a given political subdivision into specific land uses
resources and revenues programmed for agro-industrial development and as present and future projection of needs.
country-wide growth and progress, and relative thereto, shall:

M.R.A.D.C. LUMBRE 268


CONSTITUTIONAL LAW REVIEW

The regulation by local legislatures of land use in their respective To be considered as a valid police power measure, an ordinance must pass a two-
territorial jurisdiction through zoning and reclassification is an exercise of pronged test: the formal (i.e., whether the ordinance is enacted within the
police power. The police power is a governmental function, an inherent attribute corporate powers of the local government unit, and whether it is passed in
of sovereignty, which was born with civilized government. It is founded largely on accordance with the procedure prescribed by law); and the substantive (i.e.,
the maxims, "Sic utere tuo et alienum non laedas" and "Salus populi est suprema involving inherent merit, like the conformity of the ordinance with the limitations
lex" Its fundamental purpose is securing the general welfare, comfort and under the Constitution and the statutes, as well as with the requirements of fairness
convenience of the people. and reason, and its consistency with public policy).

Police power is inherent in the state but not in municipal corporations. The formalities in enacting an ordinance are laid down in Section 53 and Section
Before a municipal corporation may exercise such power, there must be a 54 of The Local Government Code. These provisions require the ordinance to be
valid delegation of such power by the legislature which is the repository passed by the majority of the members of the sanggunian concerned, and to be
of the inherent powers of the State. A valid delegation of police power may presented to the mayor for approval. With no issues regarding quorum during its
arise from express delegation, or be inferred from the mere fact of the creation of deliberation having been raised, and with its approval of by City Mayor Duterte not
the municipal corporation; and as a general rule, municipal corporations may being disputed, we see no reason to strike down Ordinance No. 0309-07 for non-
exercise police powers within the fair intent and purpose of their creation which are compliance with the formal requisites under the Local Government Code.
reasonably proper to give effect to the powers expressly granted, and statutes
conferring powers on public corporations have been construed as empowering them xxxxxxx
to do the things essential to the enjoyment of life and desirable for the safety of
the people. The corporate powers of the local government unit confer the basic authority to
enact legislation that may interfere with personal liberty, property, lawful
Police power is the power to prescribe regulations to promote the health, morals, businesses and occupations in order to promote the general welfare. Such
peace, education, good order or safety and general welfare of the people. It is the legislative powers spring from the delegation thereof by Congress through either
most essential, insistent, and illimitable of powers. In a sense it is the greatest and the Local Government Code or a special law. The General Welfare Clause in Section
most powerful attribute of the government. It is elastic and must be responsive to 16 of the Local Government Code embodies the legislative grant that enables the
various social conditions. local government unit to effectively accomplish and carry out the declared objects
of its creation, and to promote and maintain local autonomy.
The police power of a municipal corporation is broad, and has been said to be Section 16 comprehends two branches of delegated powers, namely: the general
commensurate with, but not to exceed, the duty to provide for the real needs of legislative power and the police power proper. General legislative power refers to
the people in their health, safely, comfort, and convenience as consistently as may the power delegated by Congress to the local legislative body, or the Sangguniang
be with private rights. It extends to all the great public needs, and, in a broad sense Panlungsod in the case of Dayao City, to enable the local legislative body to enact
includes all legislation and almost every function of the municipal government. It ordinances and make regulations. This power is limited in that the enacted
covers a wide scope of subjects, and, while it is especially occupied with whatever ordinances must not be repugnant to law, and the power must be exercised to
affects the peace, security, health, morals, and general welfare of the community, effectuate and discharge the powers and duties legally conferred to the local
it is not limited thereto, but is broadened to deal with conditions which exists so as legislative body. The police power proper, on the other hand, authorizes the local
to bring out of them the greatest welfare of the people by promoting public government unit to enact ordinances necessary and proper for the health and
convenience or general prosperity, and to everything worthwhile for the safety, prosperity, morals, peace, good order, comfort, and convenience of the local
preservation of comfort of the inhabitants of the corporation. Thus, it is deemed government unit and its constituents, and for the protection of their property.
inadvisable to attempt to frame any definition which shall absolutely indicate the
limits of police power. Section 458 of the Local Government Code explicitly vests the local government
unit with the authority to enact legislation aimed at promoting the general welfare,
NOTE: A city ordinance was passed in Manila reclassifying an area in Pandacan viz.:
from industrial to residential, which had the effect of disallowing further operations
in the Pandacan oil depots. The court upheld the validity of the ordinance when the Section 458. Powers, Duties, Functions and Compensation. (a) The
same was questioned. sangguniang panlungsod, as the legislative body of the city, shall enact
ordinances, approve resolutions and appropriate funds for the general welfare
c. Mosqueda vs. Pilipino Banana Growers & Exporters Association, Inc., of the city and its inhabitants pursuant to Section 16 of this Code and in the
G.R. No. 189185, 16 August 2016 proper exercise of the corporate powers of the city as provided for under
The Sangguniang Bayan of Davao City enacted Ordinance No. 0309-07 under its Section 22 of this Code. x x x
corporate powers In terms of the right of the citizens to health and to a balanced and healthful
ecology, the local government unit takes its cue from Section 15 and Section 16,

M.R.A.D.C. LUMBRE 269


CONSTITUTIONAL LAW REVIEW

Article II of the 1987 Constitution. Following the provisions of the Local Government government unit the blanket authority to legislate upon any subject that it finds
Code and the Constitution, the acts of the local government unit designed to ensure proper to legislate upon in the guise of serving the common good.
the health and lives of its constituents and to promote a balanced and healthful
ecology are well within the corporate powers vested in the local government unit. The function of pesticides control, regulation and development is within the
Accordingly, the Sangguniang Bayan of Davao City is vested with the requisite jurisdiction of the FPA under Presidential Decree No. 1144. The FPA was established
authority to enact an ordinance that seeks to protect the health and well-being of in recognition of the need for a technically oriented government entity that will
its constituents. protect the public from the risks inherent in the use of pesticides. To perform its
mandate, it was given under Section 6 of Presidential Decree No. 1144 the following
xxxxxxxxxx powers and functions with respect to pesticides and other agricultural
The impossibility of carrying out a shift to another mode of pesticide application chemicals, viz.:
within three months can readily be appreciated given the vast area of the affected
plantations and the corresponding resources required therefor. Evidently, the FPA was responsible for ensuring the compatibility between the
usage and the application of pesticides in agricultural activities and the demands
Respondent-appellee argues that the Ordinance merely banned an agricultural for human health and environmental safety. This responsibility includes not only
practice and did not actually prohibit the operation of banana plantations; hence, the identification of safe and unsafe pesticides, but also the prescription of the safe
it is not oppressive. While we agree that the measure did not impose a closure of modes of application in keeping with the standard of good agricultural practices.
a lawful enterprise, the proviso in Section 5, however, compels petitioners-
appellants to abandon aerial spraying without affording them enough time to On the other hand, the enumerated devolved functions to the local government
convert and adopt other spraying practices. This would preclude petitioners- units do not include the regulation and control of pesticides and other agricultural
appellants from being able to fertilize their plantations with essential vitamins and chemicals. The non-inclusion should preclude the Sangguniang Bayan of Davao City
minerals substances, aside from applying thereon the needed fungicides or from enacting Ordinance No. 0309-07, for otherwise it would be arrogating unto
pesticides to control, if not eliminate the threat of, plant diseases. Such an apparent itself the authority to prohibit the aerial application of pesticides in derogation of
eventuality would prejudice the operation of the plantations, and the economic the authority expressly vested in the FPA by Presidential Decree No. 1144.
repercussions thereof would just be akin to shutting down the venture.
In enacting Ordinance No. 0309-07 without the inherent and explicit authority to do so,
This Court, therefore, finds Section 5 of Ordinance No. 0309-07 an invalid provision the City of Davao performed an ultra vires act. As a local government unit, the City of
because the compulsion thereunder to abandon aerial spraying within an Davao could act only as an agent of Congress, and its every act should always conform
impracticable period of "three (3) months after the effectivity of this Ordinance" is to and reflect the will of its principal.
"unreasonable, oppressive and impossible to comply with."
For sure, every local government unit only derives its legislative authority from
xxxxxxxxx Congress. In no instance can the local government unit rise above its source of
authority. As such, its ordinance cannot run against or contravene existing laws,
The establishment of the buffer zone is required for the purpose of minimizing the
precisely because its authority is only by virtue of the valid delegation from
effects of aerial spraying within and near the plantations. Although Section 3(e) of
Congress.
the ordinance requires the planting of diversified trees within the identified buffer
zone, the requirement cannot be construed and deemed as confiscatory requiring
Moreover, Ordinance No. 0309-07 proposes to prohibit an activity already covered
payment of just compensation. A landowner may only be entitled to compensation
by the jurisdiction of the FPA, which has issued its own regulations under its
if the taking amounts to a permanent denial of all economically beneficial or
Memorandum Circular No. 02, Series of 2009, entitled Good Agricultural Practices
productive uses of the land. The respondents cannot be said to be permanently and
for Aerial Spraying of Fungicide in Banana Plantation. While Ordinance No. 0309-
completely deprived of their landholdings because they can still cultivate or make
07 prohibits aerial spraying in banana plantations within the City of Davao,
other productive uses of the areas to be identified as the buffer zones.
Memorandum Circular No. 02 seeks to regulate the conduct of aerial spraying in
xxxxxxxxx banana plantations pursuant to Section 6, Presidential Decree No. 1144, and in
conformity with the standard of Good Agricultural Practices (GAP). Memorandum
Although the Local Government Code vests the municipal corporations with Circular No. 02 covers safety procedures, handling and post-application, including
sufficient power to govern themselves and manage their affairs and activities, they the qualifications of applicators, storing of fungicides, safety and equipment of
definitely have no right to enact ordinances dissonant with the State's laws and plantation personnel, all of which are incompatible with the prohibition against
policy. The Local Government Code has been fashioned to delineate the specific aerial spraying under Ordinance No. 0309-07.
parameters and limitations to guide each local government unit in exercising its
delegated powers with the view of making the local government unit a fully Devoid of the specific delegation to its local legislative body, the City of Davao
functioning subdivision of the State within the constitutional and statutory exceeded its delegated authority to enact Ordinance No. 0309-07. Hence,
restraints. The Local Government Code is not intended to vest in the local

M.R.A.D.C. LUMBRE 270


CONSTITUTIONAL LAW REVIEW

Ordinance No. 0309-07 must be struck down for being an ultra vires act on the part 5. A valid and definite Offer has been previously made to the owner of the property
of the Sangguniang Bayan of Davao City. sought to be expropriated, but said offer was not accepted. (Municipality of
Paranaque v. V.M. Realty Corporation, G.R. No. 127820. July 20, 1998)
We must emphasize that our ruling herein does not seek to deprive the LGUs their
right to regulate activities within their jurisdiction. They are empowered under Due process requirements in eminent domain (PRP)
Section 16 of the Local Government Code to promote the general welfare of the Offer must be in writing specifying:
people through regulatory, not prohibitive, ordinances that conform with the policy
directions of the National Government. Ordinance No. 0309-07 failed to pass this 1. Property sought to be acquired;
test as it contravenes the specific regulatory policy on aerial spraying in banana
2. The reason for the acquisition; and
plantations on a nationwide scale of the National Government, through the FPA.
3. The price offered.
Finally, the unconstitutionality of the ban renders nugatory Ordinance No. 0309-07
in its entirety. NOTE:
1. If owner accepts offer: a contract of sale will be executed.
2. Eminent domain 2. If owner accepts but at a higher price: Local chief executive shall call a
Local government units have no inherent power of eminent domain. Local conference for the purpose of reaching an agreement on the selling price; If
governments can exercise such power only when expressly authorized by the agreed, contract of sale will be drawn. (Implenting Rules and Regulations of
Legislature. By virtue of the Local Government Code, Congress conferred upon local LGC, Art. 35)
government units the power to expropriate (Masikip v. City of Pasig, G.R. No. Elements for an authorized immediate entry
136349, January 23, 2006).
1. Filing of a complaint for expropriation which is sufficient in form and substance;
However, while the power of eminent may be validly delegated to LGUs, the and
exercise of such power by the delegated entities is not absolute. The scope of such
delegated power is narrower than that of the delegating authority and may be 2. Deposit of the amount equivalent to fifteen percent (15%) of the fair market
exercised only when authorized by Congress, subject to its control and the value of the property to be expropriated based on its current tax declaration.
restraints imposed through the law conferring the power. Strictly speaking, the
NOTE: Upon compliance, the issuance of writ of possession becomes
power of eminent domain delegated to an LGU is in reality not eminent but
ministerial. (City of Iloilo v. Legaspi, G.R. No. 154614, Nov. 25, 2004)
“inferior”. The national legislature is still the principal of the LGUs, and the latter
cannot go against the principal’s will or modify the same. (Beluso v. Municipality of Phases of Expropriation Proceedings
Panay, G.R. No. 153974, August 7, 2006)
1. The determination of the authority of the plaintiff to exercise the power of
NOTE: LGUs may, through its local chief executive and acting pursuant to an eminent domain and the propriety of its exercise in the context of the facts involved
ordinance, exercise power of eminent domain for public use, or purpose, or in the suit.
welfare for the benefit of the poor and the landless, upon payment of just
compensation. (LGC, Sec. 19) NOTE: It ends with an order, if not dismissal of action, of condemnation
declaring that the plaintiff has a lawful right to take the property sought to be
Requisites for the Valid Exercise condemned, for the public use or purpose described in the complaint, upon the
payment of just compensation to be determined as of the date of the filing of
1. An Ordinance is enacted by the local legislative council authorizing the local chief
the complaint.
executive, in behalf of the LGU, to exercise the power of eminent domain or pursue
expropriation proceeding; An order of dismissal, if this be ordained, would be a final one, since it finally
disposes of the action and leaves nothing more to be done by the Court on the
2. The property sought to be expropriated must be a private property;
merits. The order of condemnation shall be a final one, as the Rules expressly
3. It must be for Public use, purpose or welfare or for the benefit of the poor or state, in the proceedings before the Trial Court, no objection to the exercise of
landless; the right of condemnation (or the propriety thereof) shall be filed or heard.

NOTE: Property already devoted to public use may not be taken for another public 2. The determination by the RTC of the just compensation for the property sought
use. (City of Manila v. Chinese Community of Manila, G.R. No. L-14355, October to be taken. This is done by the Court with the assistance of not more than three
31, 1919) (3) commissioners. The order fixing the just compensation on the basis of the
evidence before, and findings of, the commissioners would be final. It would finally
4. There must be payment of just Compensation; dispose of the second stage of the suit, and leave nothing more to be done by the

M.R.A.D.C. LUMBRE 271


CONSTITUTIONAL LAW REVIEW

Court regarding the issue. (Brgy. San Roque, Talisay, Cebu v. Hrs. of Francisco to the limitations set forth under Section 133 of the LGC. (Batangas City v.
Pastor, G.R. No. 138896, June 20, 2000) Pilipinas Shell Petroleum Philippines, G.R. No. 187631, July 8, 2015)

NOTE: LGU’s prolonged occupation of private property without the benefit of Rationale for Local Taxation
expropriation proceedings entitles the landowner to damages. (City of Iloilo v.
Judge Contreras-Besana, G.R. No. 168967, February 12, 2010) The power of taxation is an essential and inherent attribute of sovereignty. It is a
Satisfaction of “public use” requirement power that is purely legislative and which the central legislative body cannot
delegate to either executive or judicial department without infringing upon the
In cases where only a few could actually benefit from the expropriation of the theory of separation of powers. The exception, however, lies in the case of
property does not diminish its public use character. It is simply not possible to municipal corporations, to which said theory does not apply. Legislative powers
provide for all at once, land and shelter, for all who need them. Corollary to the may be delegated to legislative governments in respect of matters of local concern.
expanded notion of public use, expropriation is not anymore confined to vast tracts This is sanctioned by immemorial practice. By necessary implication, legislative
of land and landed estates. It is therefore of no moment that the land sought to be power to create political corporations for purposes of local self-government carries
expropriated is less than half a hectare only. Through the years, the public use with it the power to confer on such local government agencies the power to tax.
requirement in eminent domain has evolved into a flexible concept, influenced by (Pepsi-Cola Bottling Co. v. Municipality of Tanauan, G.R. No. L-31156, Feb. 27,
changing conditions. 1976)
Public use now includes the broader notion of indirect public benefit or advantage
ARMM’s Taxing Power
including in particular, urban land reform and housing. (Philippine Columbian
Association v. Panis, G.R. No. L-106528, Dec. 21, 1993)
The ARMM has the legislative power to create sources of revenues within its
NOTE: The passage of RA 7279, the “Urban Development and Housing Act of 1992” territorial jurisdiction and subject to the provisions of the 1987 Constitution and
introduced a limitation on the size of the land sought to be expropriated for national laws. [1987 Constitution, Art. X, Sec. 20(2)]
socialized housing. The law expressly exempted “small property owners” from
expropriation of their land for urban land reform. (City of Mandaluyong v. Aguilar, Local Fiscal Autonomy
G.R. No. 137152, Jan. 29, 2001)
Fiscal autonomy means that local governments have the power to create their own
NOTE: sources of revenue in addition
Q: May an LGU take over private property even before the filing of a case for to their equitable share in the national taxes released by the national government,
expropriation? as well as the power to allocate their resources in accordance with their own
priorities. It extends to the preparation of their budgets, and local officials in turn
A: No, a case should be filed first, and a deposit of 15% of the amount of the have to work within the constraints thereof. They are not formulated at the national
property based on its tax declaration with an authorized depositary, after an offer level and imposed on local governments, whether they are relevant to local needs
was refused. and resources or not. Further, a basic feature of local fiscal autonomy is the
constitutionally mandated automatic release of the shares of local governments in
3. Taxing power
the national internal revenue. (Province of Batangas v. Romulo, G.R. No. 152774,
Nature May 27, 2004)

The power to tax is primarily vested in the Congress; however, in our jurisdiction, NOTE: A “no report, no release” policy may not be validly enforced against
it may be exercised by local legislative bodies, no longer merely by virtue of a valid offices vested with fiscal autonomy such as Constitutional Commissions and
delegation as before, but pursuant to direct authority conferred by Section 5, Article local governments. The automatic release provision found in the Constitution
V of the 1987 Constitution. The exercise of the power may be subject to such means these local governments units cannot be required to perform any act to
guidelines and limitations as the Congress may provide which, however, must be receive the “just share” accruing to them from the national coffers. (Civil
consistent with the basic policy of local autonomy. (MIAA v. Marcos, G.R. No. Service Commission v. Department of Budget and Management, G.R. No.
120082, Sept. 11, 1996) 158791, July 22, 2005)

NOTE: While the power to tax is inherent in the State, the same is not true for Main Sources of Revenues for LGUs
LGUs because although the mandate to impose taxes granted to LGUs is
categorical and long established in the 1987 Philippine Constitution, the same 1. Taxes, fees, and charges. (1987 Constitution Art. X, Sec. 5)
is not all encompassing as it is subject to limitations as explicitly stated in
Section 5, Article X of the 1987 Constitution. The LGUs’ power to tax is subject

M.R.A.D.C. LUMBRE 272


CONSTITUTIONAL LAW REVIEW

2. Internal Revenue Allotment (IRA) - Just share in the national taxes which shall
be automatically released to them. (1987 Constitution Art. X, Sec. 6) It is the Secretary of Justice who shall determine questions on the legality and
constitutionality of ordinances or revenue measures. Such questions shall be raised
NOTE: The current sharing is 40% local and 60% national. The share cannot on appeal within thirty days from the effectivity thereof to the Secretary of Justice
be reduced except if there is unmanageable public sector deficit. who shall render a decision within sixty days from the date of receipt of the appeal.

3. Equitable share in the proceeds of the utilization and development of the national NOTE: Such appeal shall not have the effect of suspending the effectivity of
wealth within their areas. (1987 Constitution Art. X, Sec. 7) the ordinance and the accrual and payment of the tax, fee, or charge levied
therein: Provided, finally, that within thirty days after receipt of the decision
Requirements for a valid tax ordinance (PUJ-NO) or the lapse of the sixty-day period without the Secretary of Justice acting
upon the appeal, the aggrieved party may file appropriate proceedings with a
1. The tax is for a public purpose; court of competent jurisdiction (RTC). (LGC, Sec. 187)

2. The rule on uniformity of taxation is observed; Tax Protest

3. Either the person or property taxed is within the jurisdiction of the government The formal statement, usually in writing, made by a person who is called upon by
levying the tax; and public authority to pay a sum of money, in which he declares that he does not
concede the legality or justice of the claim or his duty to pay it, or that he disputes
4. In the assessment and collection of certain kinds of taxes, notice and opportunity the amount demanded; the object being to save his right to recover or reclaim the
for hearing are provided. (Pepsi-Cola Bottling Co. v. Municipality of Tanauan, G.R. amount, which right would be lost by his acquiescence. Thus, taxes may be paid
No. L-31156, February 27, 1976) under "protest". (Black’s Law Dictionary)

Procedural requirements for a valid revenue ordinance Requisites of a valid tax protest in a LGU (PAP)

1. A prior public hearing on the measure to be conducted according to the 1. Taxpayer first pays the taxes
prescribed rules.
2. There shall be annotation on the tax receipts the words "paid under protest".
NOTE: An ordinance levying taxes, fees or charges shall not be enacted without
any prior public hearing conducted for the purpose. (Figuerres v. CA, G.R. No. 3. The protest in writing must be filed within thirty (30) days from payment of the
119172, March 25, 1999) tax to the provincial, city treasurer or municipal treasurer, in the case of a
municipality within Metropolitan Manila Area, who shall decide the protest within
2. Publication of the tax ordinance, within 10 days after their approval, for 3 sixty (60) days from receipt. (LGC, Sec. 252)
consecutive days in a newspaper of local circulation, provided that in provinces,
cities, and municipalities where there are no newspapers of local circulation, the NOTE: A claim for tax exemption, whether full or partial, does not deal with
same may be posted in at least two (2) conspicuous and publicly accessible places. the authority of local assessor to assess real property tax, but merely raises a
question of reasonableness of correctness of such assessment, which requires
NOTE: If the tax ordinance or revenue measure contains penal provisions as compliance with Sec. 252 of the LGC. (Camp John Hay Development
authorized in Art. 280 of this Rule, the gist of such tax ordinance or revenue Corporation v. Central Board of Assessment Appeals, G.R. No. 169234, October
measure shall be published in a newspaper of general circulation within the 2, 2013)
province where the sanggunian concerned belongs. (IRR of LGC, Art. 276)
Remedies available to the LGUs to enforce the payment of taxes
Effectivity of tax ordinance
1. Imposing penalties (surcharges and penalty interest) in case of delinquency
In case the effectivity of any tax ordinance or revenue measure falls on any date (LGC, Sec. 168)
other than the beginning of the quarter, the same shall be considered as falling at
the beginning of the next ensuing quarter and the taxes, fees, or charges due shall 2. Availing local government’s liens (LGC, Sec. 173)
begin to accrue therefrom. (IRR of LGC, Art. 276)
3. Administrative action through distraint of goods, chattels, and other personal
Authority to determine the legality or propriety of a local tax ordinance or property [LGC, Sec. 174(a)]
revenue measure

M.R.A.D.C. LUMBRE 273


CONSTITUTIONAL LAW REVIEW

4. Judicial action [LGC, Sec. 174(b)] (b) Documentary stamp tax;

Community tax (c) Taxes on estates, inheritance, gifts, legacies and other acquisitions mortis
Community tax is a poll or capitation tax which is imposed upon person who resides causa, except as otherwise provided herein;
within a specified territory. (d) Customs duties, registration fees of vessel and wharfage on wharves,
tonnage dues, and all other kinds of customs fees, charges and dues except
Exempted from the payment of community tax wharfage on wharves constructed and maintained by the local government unit
concerned;
1. Diplomatic and consular representatives;
(e) Taxes, fees, and charges and other impositions upon goods carried into or
2. Transient visitors when their stay in the Philippines does not exceed 3 months. out of, or passing through, the territorial jurisdictions of local government units
(LGC, Sec. 159) in the guise of charges for wharfage, tolls for bridges or otherwise, or other
Real property taxes taxes, fees, or charges in any form whatsoever upon such goods or
merchandise;
These are directly imposed on privilege to use real property such as land, building,
(f) Taxes, fees or charges on agricultural and aquatic products when sold by
machinery, and other improvements, unless specifically exempted.
marginal farmers or fishermen;
a. Film Development Council of the Philippines (FDCP) vs. Colon Heritage
(g) Taxes on business enterprises certified to by the Board of Investments as
Realty Corporation, G.R. No. 203754, 16 June 2015
pioneer or non-pioneer for a period of six (6) and four (4) years, respectively
RA 9167 violates local fiscal autonomy. from the date of registration;

It is beyond cavil that the City of Cebu had the authority to issue its City Ordinance (h) Excise taxes on articles enumerated under the national Internal Revenue
No. LXIX and impose an amusement tax on cinemas pursuant to Sec. 140 in relation Code, as amended, and taxes, fees or charges on petroleum products;
to Sec. 151 of the LGC. Sec. 140 states, among other things, that a "province may
(i) Percentage or value-added tax (VAT) on sales, barters or exchanges or
levy an amusement tax to be collected from the proprietors, lessees, or operators
similar transactions on goods or services except as otherwise provided herein;
of theaters, cinemas, concert halls, circuses, boxing stadia, and other places of
amusement at a rate of not more than thirty percent (30%) of the gross receipts (j) Taxes on the gross receipts of transportation contractors and persons
from admission fees." By operation of said Sec. 151, extending to them the engaged in the transportation of passengers or freight by hire and common
authority of provinces and municipalities to levy certain taxes, fees, and charges, carriers by air, land or water, except as provided in this Code;
cities, such as respondent city government, may therefore validly levy amusement
taxes subject to the parameters set forth under the law. Based on this authority, (k) Taxes on premiums paid by way or reinsurance or retrocession;
the City of Cebu passed, in 1993, its Revised Omnibus Tax Ordinance, Chapter XI, (l) Taxes, fees or charges for the registration of motor vehicles and for the
Secs. 42 and 43. issuance of all kinds of licenses or permits for the driving thereof, except
Then, after almost a decade of cities reaping benefits from this imposition, tricycles;
Congress, through RA 9167, amending Section 140 of the LGC, among others, (m) Taxes, fees, or other charges on Philippine products actually exported,
transferred this income from the cities and municipalities in Metropolitan Manila except as otherwise provided herein;
and highly urbanized and independent component cities, such as respondent City
of Cebu, to petitioner FDCP, which proceeds will ultimately be rewarded to the (n) Taxes, fees, or charges, on Countryside and Barangay Business Enterprises
producers of graded films. and cooperatives duly registered under R.A. No. 6810 and Republic Act
Numbered Sixty-nine hundred thirty-eight (R.A. No. 6938) otherwise known
A reading of the challenged provision reveals that the power to impose amusement as the "Cooperative Code of the Philippines" respectively; and
taxes was NOT removed from the covered LGUs, unlike what Congress did for the
taxes enumerated in Sec. 133, Article X of the LGC, which lays down the common (o) Taxes, fees or charges of any kind on the National Government, its agencies
limitations on the taxing powers of LGUs. Thus: and instrumentalities, and local government units.

Section 133. Common Limitations on the Taxing Powers of Local Government From the above, the difference between Sec. 133 and the questioned amendment
Units. -Unless otherwise provided herein, the exercise of the taxing powers of of Sec. 140 of the LGC by RA 9167 is readily revealed. In Sec. · 133, what Congress
provinces, cities, municipalities, and barangays shall not extend to the levy of did was to prohibit the levy by LGUs of the enumerated taxes. For RA 9167,
the following: however, the covered LGUs were deprived of the income which they will otherwise
be collecting should they impose amusement taxes, or, in petitioner's own words,
(a) Income tax, except when levied on banks and other financial institutions; "Section 14 of [RA 9167] can be viewed as an express and real intention on the

M.R.A.D.C. LUMBRE 274


CONSTITUTIONAL LAW REVIEW

part of Congress to remove from the LGU's delegated taxing power, all revenues 1. It must be approved by at least 2/3 of all the members of the Sanggunian
from the amusement taxes on graded films which would otherwise accrue to [them] and when necessary provide for an adequate substitute for the public facility
pursuant to Section 140 of the [LGC]."
2. Adequate provision for the public safety must be made
In other words, per RA 9167, covered LGUs still have the power to levy amusement 3. The property may be used or conveyed for any purpose for which other real
taxes, albeit at the end of the day, they will derive no revenue therefrom. The property may be lawfully used or conveyed. [LGC, Sec. 21(a)(b)]
same, however, cannot be said for FDCP and the producers of graded films since
the amounts thus levied by the LGUs which should rightfully accrue to them, they Note: No freedom park shall be closed permanently without provision for its
being the taxing authority-will be going to their coffers. As a matter of fact, it is transfer or relocation to a new site. [LGC, Sec. 21(a)(b)]
only through the exercise by the LGU of said power that the funds to be used for
the amusement tax reward can be raised. Without said imposition, the producers B. In case of temporary closure:
of graded films will receive nothing from the owners, proprietors and lessees of
cinemas operating within the territory of the covered LGU.
1. It must be for actual emergency, fiesta celebration, public rallies,
Taking the resulting scheme into consideration, it is apparent that what Congress agricultural or industrial fairs, or an undertaking of public works and highways,
did in this instance was not to exclude the authority to levy amusement taxes from telecommunications and water work projects
the taxing power of the covered LGUs, but to earmark, if not altogether confiscate, 2. Duration of which shall be specified
the income to be received by the LGU from the taxpayers in favor of and for
transmittal to FDCP, instead of the taxing authority. This, to Our mind, is in clear 3. Except for those activities not officially sponsored or approved by the LGU
contravention of the constitutional command that taxes levied by LGUs shall accrue concerned. [LGC, Sec. 21(c)]
exclusively to said LGU and is repugnant to the power of LGUs to apportion their NOTE: Any city, municipality or barangay may, by ordinance, temporarily close
resources in line with their priorities. and regulate the use of a local street, road, thoroughfare or any other public
place where shopping malls, Sunday, flea or night markets, or shopping areas
It is a basic precept that the inherent legislative powers of Congress, broad as they
may be established and where articles of commerce may be sold or dispensed
may be, are limited and confined within the four walls of the Constitution.37
with to the general public. [LGC, Sec. 21(d)]
Accordingly, whenever the legislature exercises its power to enact, amend, and
repeal laws, it should do so without going beyond the parameters wrought by the
Material factors to consider in closing a street
organic law.

In the case at bar, through the application and enforcement of Sec. 14 of RA 9167, The material factors which a municipality must consider in deliberating upon the
the income from the amusement taxes levied by the covered LGUs did not and will advisability of closing a street are:
under no circumstance accrue to them, not even partially, despite being the taxing
1. The topography of the property surrounding the street in the light of ingress and
authority therefor. Congress, therefore, clearly overstepped its plenary legislative
egress to other streets;
power, the amendment being violative of the fundamental law's guarantee on local
autonomy, as echoed in Sec. 130(d) of the LGC. 2. The relationship of the street in the road system throughout the subdivision;
4. Closure and opening of roads 3. The problem posed by the 'dead end' of the street; the width of the street;
The LGU may, pursuant to an ordinance, permanently or temporarily close or open 4. The cost of rebuilding and maintaining the street as contrasted to its ultimate
any local road, alley, park, or square falling within its jurisdiction; Provided, value to all of the property in the vicinity;
however, that in case of permanent closure, such ordinance must be approved by
at least two-thirds (2/3) of all the members of the sanggunian, and when 5. The inconvenience of those visiting the subdivision; and
necessary, an adequate substitute for the public facility that is subject to closure is 6. Whether the closing of the street would cut off any property owners from access
provided. [LGC, Sec. 21(a)] to a street. (Favis v. City of Baguio, G.R. No. L-29910, April 25, 1969)

NOTE: No permanent closure of any local road, street, alley, park, or square 5. Legislative power
shall be effected unless there exists a compelling reason or sufficient
Nature of local legislative powers
justification therefor such as, but not limited to, change in land use,
establishment of infrastructure facilities, projects, or such other justifiable
It is a fundamental principle that municipal ordinances are inferior in status and
reasons as public welfare may require. [IRR of LGC, Art. 44(a)]
subordinate to the laws of the State. An ordinance in conflict with a state law of
Limitations of permanent and temporary closure
general character and statewide application is universally held to be invalid. The
principle is frequently expressed in the declaration that municipal authorities, under
A. In case of permanent closure:

M.R.A.D.C. LUMBRE 275


CONSTITUTIONAL LAW REVIEW

a general grant of power, cannot adopt ordinances which infringe upon the spirit of The entire membership must be taken into account in computing the quorum.
a state law or repugnant to the general policy of the state. In every power to pass (Zamora v. Caballero, G.R. No. 147767, January 14, 2004)
ordinances given to a municipality, there is an implied restriction that the
ordinances shall be consistent with the general law (Batangas CATV v. Court of NOTE: The determination of the existence of quorum is based on the total number
Appeals, G.R. No. 138810, September 29, 2004). of members of the sanggunian without regard to filing of a leave of absence.
(Zamora v. Caballero, ibid.)
NOTE: The rule against undue delegation of legislative powers applies to LGUs.
In the case of Villegas v. Tsai Pao Ho (G.R. No. 29646, October 10, 1978), a Procedures to be taken by the presiding officer if there is a question on
city ordinance was declared void because it constituted undue delegation of quorum
legislative power to the Mayor. The ordinance did not lay down any standard
to guide the Mayor in the exercise of his discretion in the issuance or denial of Should there be a question of quorum raised during a session, the presiding officer
an alien employment permit. shall:

The Sanggunian 1. Immediately proceed to call the roll of the members; and
2. Announce the results. [LGC, Sec. 53 (a)]
A sanggunian is a collegial body. Legislation, which is the principal function of the
sanggunian, requires the participation of all its members so that they may not only Procedures to be taken by the presiding officer if there is no quorum
represent the interests of their respective constituents but also help in the making
of decisions, by voting upon every question put upon the body (Zamora v. The presiding officer may:
Caballero, G.R. No. 147767, January 14, 2004).
1. Declare a recess until such time that quorum is constituted
NOTE: A petition for certiorari filed against a Sangguniang Panlungsod assailing
the legality of an ordinance will not lie since the Sanggunian is not a tribunal, 2. Compel immediate attendance of the members who are absent without justifiable
board or officer exercising judicial or quasijudicial functions (Liga ng mga cause
Barangay National v. City Mayor of Manila, G.R. No. 154599, January 21, 3. Declare the session adjourned for lack of quorum and no business shall be
2004). transacted if there is still no quorum despite enforcement of attendance. [LGC, Sec.
53 (b)(c)]
No power to subpoena and hold persons in contempt
Guidelines in the conduct of a sanggunian session
The contempt power and the subpoena power cannot be deemed implied in the
1. It shall be open to public, unless it is a closed door session
delegation of certain legislative functions to local legislative bodies. These cannot
be presumed to exist in favor of the latter and must be considered an exception to 2. No two sessions, regular or special, may be held in a single day
Sec. 4 of BP Blg. 337 which provides for liberal rules of interpretation in favor of
local autonomy. Since the existence of these powers poses a potential derogation 3. Minutes of the session be recorded and each sanggunian shall keep a journal
of individual rights, the law cannot be liberally construed to have impliedly granted and record of its proceedings which may be published upon resolution of the
such powers to local legislative bodies. The intention of the people, through their sanggunian concerned.
representatives, to share these powers with the local legislative bodies must clearly 4. In case of special sessions:
appear in pertinent legislation. (Negros Oriental II Electric Cooperative Inc., v.
Sangguiang Panlungsod ng Dumaguete, G.R. No. L- 72492, November 5, 1987) a. Written notice to the members must be served personally at least 24 hours
before the special session is held
Quorum in the sanggunian
b. Unless otherwise concurred in by 2/3 votes of the sanggunian members
present, there being no quorum, no other matters may be considered at a
Quorum is defined as the number of members of a body which when legally
special session except those stated in the notice. (LGC, Sec. 52)
assembled in their proper places, will enable the body to transact its proper
business or that number which makes a lawful body and gives it power to pass Ordinance
upon a law, ordinance or any valid act. ‘Majority’, when required to constitute a
quorum, means the number greater than half or more than half of any total. The As a municipal statute, it is a rule of conduct or of action, laid down by the municipal
applicable rule on quorum of local legislative bodies is found in Section 53(a) of the authorities that must be obeyed by the citizens. It is drafted, prepared,
LGC which provides that a majority of all members of the sanggunian who have promulgated by such authorities for the information of all concerned, under and by
been elected and qualified shall constitute a quorum to transact official business.

M.R.A.D.C. LUMBRE 276


CONSTITUTIONAL LAW REVIEW

virtue of powers conferred upon them by law. (United States v. Pablo Trinidad, G.R. There is nothing in the LGC which prohibits the three readings of a proposed
No. L- 3023, January 16, 1907) ordinance from being held in just one session day. It is not the function of the
REQUISITES OF A VALID ORDINANCE courts to speculate that the councilors were not given ample time for reflection and
Requisites for a Valid Ordinance circumspection before the passage of the proposed ordinance by conducting three
readings in just one day. (Malonzo v. Zamora, G.R. No. 137718, July 27, 1999)
1. Must not contravene the constitution and any statute
Veto of the Local Chief Executive
2. Must not be unfair or oppressive
3. Must not be partial or discriminatory The Local Chief Executive may veto the ordinance only once on the ground that the
ordinance is ultra vires and prejudicial to public welfare. The veto must be
4. Must not prohibit, but may regulate trade communicated to the sanggunian within:
5. Must not be unreasonable
a. 15 days for a province
6. Must be general in application and Consistent with public policy [not-CUPPUn-
b. 10 days for a city or municipality (LGC, Secs. 54 and 55)
Gen]. (Magtajas v. Pryce Properties Corporation, Inc., July 20, 1994)
NOTE: While “to veto or not to veto involves the exercise of discretion,” a
NOTE: The mere fact that there is already a general statute covering an act or
mayor exceeded his/her authority in an arbitrary manner when he/she
omission is insufficient to negate the legislative intent to empower the municipality
vetoes a resolution where there exist sufficient municipal funds from which
to enact ordinances with reference to the same act or omission under the ‘general
the salary of the officer could be paid. The Mayor’s refusal in complying
welfare clause’ of the Municipal Charter. (United States v. Pascual Pacis, G.R. No.
with the directive of the Director of the Bureau of Local Government that
10363, September 29, 1915)
the salary could be provided for is oppressive. (Pilar v. Sangguniang Bayan
of Dasol, Pangasinan, G.R. No. L-63216, March 12, 1984)
Ordinance Resolution
Resolution Merely a declaration of the
Items that the local chief executive can veto
Law sentiment or opinion of a
lawmaking body on a
1. Item/s of an appropriation ordinance.
specific matter
General and Temporary in nature 2. Ordinance/resolution adopting local development plan and public
permanent character investment program
Third reading is GR: Third reading is not
necessary for an necessary in resolution. 3. Ordinance directing the payment of money or creating liability. (LGC, Sec.
ordinance. 55)
XPN: Unless decided
otherwise by a majority of NOTE: Ordinances enacted by the sangguniang barangay shall, upon
all the Sanggunian approval by a majority of all its members be signed by the punong
members. (Roble barangay. The latter has no veto power.
Arrastre, Inc. v. Villaflor,
G.R. No. 128509, August
22, 2006) Approval of ordinances

1. By affixing the signature of the local chief executive on each and every page
Note: It has been held that even where the statute or municipal charter
thereof if he approves the same
requires the municipality to act by an ordinance, if a resolution is passed in the
2. By overriding the veto of the local chief executive by 2/3 vote of all members of
manner and with the statutory formality required in the enactment of an
the sanggunian if the local chief executive vetoed the same. (LGC, Sec. 54)
ordinance, it will be binding and effective as an ordinance. Such resolution may
operate regardless of the name by which it is called. (Favis v. City of Baguio,
NOTE: A sanggunian may provide for a vote requirement different (not
G.R. No. L-29910, April 25, 1969)
majority vote) from that prescribed in the LGC for certain (but not all)
ordinances as in amending a zoning ordinance. (Casino v. Court of Appeals,
Three readings allowed in one day
G.R. No. 91192, Dec. 2, 1991)

Effectivity of ordinance or resolution

M.R.A.D.C. LUMBRE 277


CONSTITUTIONAL LAW REVIEW

NOTE: Any new corporate seal or changes on such shall be registered with the
GR: After 10 days from the date a copy is posted in a bulletin board at the entrance DILG.
of the capitol or city, municipal or barangay hall and in at least 2 conspicuous
spaces. [LGC, Sec. 59 (a)] 4. To a acquire and convey real or personal property;
5. To enter into contracts;
XPN: Unless otherwise stated in the ordinance or resolution. [LGC, Sec. 59 (a)]
NOTE: Unless otherwise provided in this Code, no contract may be entered into by
Effect of the enforcement of a disapproved ordinance or resolution the local chief executive in behalf of the LGU without prior authorization by the
sanggunian concerned. A legible copy of such contract shall be posted at a
It shall be a sufficient ground for the suspension or dismissal of the official or conspicuous place in the provincial capitol or the city, municipal or barangay hall.
employee. (LGC, Sec. 58)
6. To exercise such other powers as granted to corporations. (LGC, Sec. 22);
Ordinances requiring publication for its effectivity
1. To Sue and be sued
1. Ordinances that carry with them penal sanctions. [LGC, Sec. 59 (c)]
Proper officer to represent the city in court actions
2. Ordinances and resolutions passed by highly urbanized and independent
component cities. [LGC, Sec. 59 (d)]
GR: The city legal officer is supposed to represent the city in all civil actions and
NOTE: special proceedings wherein the city or any of its officials is a party.
NOTE: Only the Provincial Fiscal or the Municipal Attorney can represent a
Q: A city ordinance was passed in Manila prohibiting cat-calling. A copy of the province or municipality in lawsuits. This is mandatory. Hence, a private
ordinance was sent to the City Mayor. 2 weeks after its effectivity, the Secretary of attorney cannot represent a province or municipality.
the City Mayor sent a communication to the Secretary of the City Council stating
that the Legal Department of the City of Manila recommended the veto of the XPN: Where the position is as yet vacant, the City Prosecutor remains the city’s
Ordinance because of typographical errors and its provisions were inconsistent with legal adviser and officer for civil cases. (ASEAN Pacific Planners v. City of Urdaneta,
the draft Ordinance. Can it be considered a veto? G.R. No. 162525, Sept. 23, 2008)
A: No. The communication only cited the recommendation coming from the Legal
Department. There must be a categorical statement of vetoing the entire Ordinance NOTE: Suit is commenced by the local chief executive, upon authority of the
coming from the City Mayor or local Chief Executive, and in the absence of which, Sanggunian, except when the City Councilors, by themselves and as
does not operate as a veto message. representatives of or on behalf of the City bring the action to prevent unlawful
disbursement of City funds. (City Council of Cebu v. Cuizon, G.R. No. L-28972,
Q: Suppose the communication came from the City Mayor and signed by him? Oct. 31, 1972)

A: Still no. First, there was no categorical statement vetoing the entire Ordinance. Power of LGU to sue on behalf of community it represents
Second, the grounds relied upon by the City Mayor are not proper. There are only
two (2) grounds for veto: (1) the Ordinance is ultra vires; that is, the City Council A municipality prejudiced by the action of another municipality is vested with the
acted beyond its authority vested upon them by the Local Government Code; and character of a juridical entity, is a corporation of public interest endowed with the
(2) it is prejudicial to the public welfare. Outside those two grounds, a veto cannot personality to acquire and hold property, contract obligations, and bring civil and
be made by the City Mayor. Third, a veto message should be sent within the period criminal actions in accordance with the laws governing its organization, and it is entitled
of 10 days in the case of cities or municipalities. In provinces, the period is 15 days. to file claims for the purpose of recovering damages, losses and injuries caused to the
6. Corporate powers community it represents. (Municipality of Mangaldan v. Municipality of Manaoag, G.R.
No. L-11627, Aug. 10, 1918)
Corporate powers of LGUs
2. Power to Acquire and Sell Property
1. To have continuous succession in its corporate name;
Properties of municipalities not acquired by its own funds in its private capacity are
2. To sue and be sued;
public property held
3. To have and use a corporate seal; in trust for the State. Regardless of the source or classification of land in the possession
of a municipality, except those acquired with its own funds in its private or corporate
capacity, such property is held in trust for the State for the benefit of its inhabitants,

M.R.A.D.C. LUMBRE 278


CONSTITUTIONAL LAW REVIEW

whether it be for government or proprietary purposes. It holds such lands subject to 3. The contract must comply with certain substantive requirements: a. Actual
the paramount power of the legislature to dispose of the same, for after all it owes its appropriation; and b. Certificate of availability of funds
creation to it as an agent for the performance of a part of it public work, the municipality 4. The contract must comply with the formal requirements of written contracts. (e.g.
being but a subdivision or instrumentality thereof for the purposes of local Statue of Frauds)
administration. (Salas v. Jarencio, G.R. No. L-29788, Aug. 30, 1972)
NOTE: This includes the power to acquire and convey properties by the LGU through
Properties that can be alienated by LGUs written contracts.

Only properties owned in its private or proprietary capacity. (Province of Zamboanga Void contracts of LGUs do not require judicial declaration of nullity
del Norte v. City of Zamboanga, G.R. No. L-24440, March 28, 1968) Contracts entered into by a municipality, in violation of existing law, do not require
judicial action declaring their nullity. In the case of Bunye v. Sandiganbayan (G.R. No.
Art. 424 of the Civil Code lays down the basic principle that properties of public dominion 122058, May 5, 1999), the Supreme Court held that contracts which grant a 25-year
devoted to public use and made available to the public, in general, are outside the lease of the Public Market when the law at that time BP Blg. 337, limits such leases to
commerce of man and cannot be disposed of or leased by the LGU to private persons. a maximum of five years, are void.
(Macasiano v. Diokno, G.R. No. 97764, Aug. 10, 1992)
Conditions/Requisites under which a local chief executive may enter into a
Rules on LGU’s power to acquire and convey real or personal property contract in behalf of his government unit

1. In the absence of proof that the property was acquired through corporate or private 1. The contract must be within the power of the municipality
funds, the presumption is that it came from the State upon the creation of the 2. The contract must be entered into by an authorized officer (e.g. mayor with proper
municipality and, thus, is governmental or public property. (Salas v. Jarencio, G.R. No. resolution by the Sangguniang Bayan)
L-29788, Aug. 30, 1972; Rabuco v. Villegas, G.R. No. L-24661, Feb. 28, 1974) 3. There must be appropriation and certificate of availability of funds
4. The contract must conform with the formal requisites of a written contract as
2. Town plazas are properties of public dominion; they may be occupied temporarily, prescribed by law; and
but only for the duration of an emergency. (Espiritu v. Municipal Council of Pozorrubio, 5. In some cases the contract must be approved by the President and/or provincial
Pangasinan, G.R. No. L-11014, governor. (Revised Adm. Code, Sec. 2068 and Sec. 2196)
January 21, 1958)
Contracts validly entered into by previous chief executive bind successor-in-
3. Public plazas are beyond the commerce of man, and cannot be the subject of lease office
or other contractual undertaking. And, even assuming the existence of a valid lease of
the public plaza or part thereof, the municipal resolution effectively terminated the When there is a perfected contract executed by the former Governor, the succeeding
agreement, for it is settled that the police power cannot be surrendered or bargained governor cannot revoke or renounce the same without the consent of the other party.
away through the medium of a contract. (Villanueva v. Castaneda, G.R. No. L- 61311, The contract has the force of law between the parties and they are expected to abide
Sept. 21, 1987) in good faith by their respective contractual commitments. Just as nobody can be forced
to enter into a contract, in the same manner, once a contract is entered into, no party
4. Public streets or thoroughfares are property for public use, outside the commerce of can renounce it unilaterally or without the consent of the other. It is a general principle
man, and may not be the subject of lease or other contracts of law that no one may be permitted to change his/her mind or disavow and go back
upon his/her own acts, or to proceed contrary thereto, to the prejudice of the other
3. To Enter into Contracts party. (GSIS v. Province of Tarlac, G.R. No. 157860, Dec. 1, 2003)
Ultra vires contracts
Requisites
1. The LGU has the express, implied or inherent power to enter into particular contract Ultra vires contracts are those which:
2. The contract is entered into by the proper department board, committee, officer or
agent. a. are entered into beyond the express, implied or inherent powers of the LGU; and
NOTE: No contract may be entered into by the local chief executive on behalf of the b. do not comply with the substantive requirements of law e.g., when expenditure of
local government without prior authorization by the sanggunian concerned, unless public funds is to be made, there must be an actual appropriation and certificate of
otherwise provided. [LGC, Sec. availability of funds. (Land Bank of the Philippines v. Cacayuran, G.R. No. 191667, April
22(c)] 17, 2013)

M.R.A.D.C. LUMBRE 279


CONSTITUTIONAL LAW REVIEW

NOTE: Such are null and void and cannot be ratified or validated. a) All governors, city and municipal mayors are prohibited from practising their
profession or engaging in any occupation other than the exercise of their functions
Estoppel cannot be applied against a municipal corporation in order to validate a
as local chief executives.
contract which the municipal corporation has no power to make or which it is authorized
to make only under prescribed conditions, within prescribed limitations, or in a b) Sanggunian members may practice their professions, engage in any occupation,
prescribed mode or manner, although the corporation has accepted the benefits thereof or teach in schools except during session hours, Provided, that those who are also
and the other party has fully performed his part of the agreement, or has expended members of the Bar snail not:
large sums in preparation for performance. (Favis v. Municipality of Sabangan, G.R. No.
L-26522, Feb. 27, 1969) (i) appear as counsel before any court in any civil case wherein the local
government unit or any office, agency or instrumental of the government is
NOTE:
the adverse party;
Q: Suppose the Sangguniang Barangay passes an ordinance, and the same is signed
by the Punong Barangay. Is the ordinance subject to sound review? (ii) appear as counsel in any criminal case wherein an officer or employee of
the national or local government is accused of an offense committed in relation
A: Yes. The Sangguniang Bayan or Panlungsod may review. to his office;
Q: Is there a situation wherein the Sangguniang Bayan or Panlungsod invalidates an
(iii) collect any fee for their appearance in administrative proceedings involving
ordinance passed by the barangay? the local government unit of which he is an official; and
A: Yes. If the ordinance passed by the subordinate barangay is inconsistent with an
ordinance of the Sangguniang Bayan or Panlungsod, or the law. (iv) use property and personnel of the government except when the
sanggunian member concerned is defending the interest of the government.

c) Doctors of medicine may practice their profession even during hours of work only
IV. Local Officials on occasions of emergency, provided they do not derive monetary compensation
therefrom.
3. Prohibition against appointment [Sec. 94, R.A. 7160],
A. Provisions applicable to elective and appointive local officials
a) No elective or appointive local official shall be eligible for appointment or
1. Prohibited Business and Pecuniary Interest [Sec. 89, R.A. 7160]: It shall be unlawful designation in any capacity to any public office or position during his tenure.
for any local government official or employee, directly or indirectly, to:
Unless otherwise allowed by law or by the primary functions of his office, no local
a) Engage in any business transaction with the local government unit in which he official shall hold any other office or employment in the government or any
is an official or employee or over which he has the power of supervision, or with subdivision, agency or instrumentality thereof, including government-owned or -
any of its authorized boards, officials, agents or attorneys, whereby money is to be controlled corporations or their subsidiaries. Relate this to Sec. 7, Art. IX-B of the
paid, or property or any other thing of value is to be transferred, directly or Constitution. See Flores v. Drilon, supra.
indirectly, out of the resources of the local government unit to such person or firm:
b) Except for losing candidates in barangay elections, no candidate who lost in any
b) Hold such interests in any cockpit or other games licensed by the local election shall, within one year after such election, be appointed to any office in the
government unit; government or any government-owned or -controlled corporation or their
c) Purchase any real estate or other property forfeited in favor of the local subsidiaries. Relate this to Sec. 6, Art. IX-B, of the Constitution, which does not
government unit for unpaid taxes or assessment, or by virtue of a legal process at provide for an exception.
the instance of the local government unit; B. Elective Local Officials.
d) Be a surety for any person contracting or doing business with the local 1. Qualifications/Disqualifications.
government unit for which a surety is required; and
a) Qualifications [Sec. 39, R.A. 7160]: Citizen of the Philippines; a registered voter in
e) Possess or use any public property of the local government unit for private the barangay, municipality, city or province, or, in the case of a member of the
purposes. sangguniang panlalawigan, panlungsod or bayan, the district where he intends to be
f) The prohibitions and inhibitions prescribed in R.A. 6713 also apply. elected; a resident therein for at least one year immediately preceding the election;
able to read and write Filipino or any other local language or dialect; and, on election
2. Practice of Profession [Sec. 9C, R.A. 7160]. day, must be at least 23 years of age [for governor, vice-governor, member of the
sangguniang panlalawigan, mayor, vice mayor, or member of the sangguniang
panlungsod of highly urbanized cities], 21 years of age [for mayor or vice mayor of

M.R.A.D.C. LUMBRE 280


CONSTITUTIONAL LAW REVIEW

independent component cities, component cities, or municipalities], 18 years of age [for iii) Punong barangay, the highest-ranking sanggunian barangay member, or in
member of the sangguniang panlungsod or sangguniang bayan, or punong barangay or case of his permanent inability, the second highest ranking sanggunian barangay
member of the sangguniang barangay], or at least 15 but not more than 21 years of member. [Note: A tie between or among the highest ranking sanggunian
age [for candidates for the sangguniang kabataan] members shall be resolved by drawing of lots.]
The Local Government Code does not specify any particular date when the candidate iv) Sanggunian member, where automatic successions provided above do not
must possess Filipino citizenship. Philippine citizenship is required to ensure that no apply: filled by appointment by the President, through the Executive Secretary in
alien shall govern our people. An official begins to govern only upon his proclamation the case of the sanggunian panlalawigan or sanggunian panlungsod of highly
and on the day that his term begins. Since Frivaldo took his oath of allegiance on June urbanized cities and independent component cities; by the Governor in the case
30, 1995, when his application for repatriation was granted by the Special Committee of the sangguniang panlungsod of component cities and the sangguniang bayan;
on Naturalization created under PD 825, he was therefore qualified to be proclaimed. and by the city or municipal mayor in the case of sangguniang barangay upon
Besides, Sec. 39 of the Local Government Code speaks of qualifications of elective recommendation of the sangguniang barangay concerned.
officials, not of candidates. Furthermore, repatriation retroacts to the date of the filing
of his application on August 17, 1994 (Frivaldo v. COMELEC) However, except for the sangguniang barangay, only the nominee of the political
party under which the sanggunian member concerned had been elected and whose
b) Disqualifications [Sec. 40, R.A. 7160]: The following are disqualified from running elevation to the position next higher in rank created the last vacancy in the
for any elective local position: sanggunian shall be appointed. A nomination and a certificate of membership of
the appointee from the highest official of the political party concerned are
(i) Those sentenced by final judgment for an offense involving moral turpitude or conditions sine qua non, and any appointment without such nomination and
for an offense punishable by one year or more of imprisonment, within two years certificate shall be null and void and shall be a ground for administrative action
after serving sentence; against the official concerned. In case the permanent vacancy is caused by a
(ii) Those removed from office as a result of an administrative case; sangguniang member who does not belong to any political party, the local chief
executive shall upon the recommendation of the sanggunian concerned, appoint
(iii) Those convicted by final judgment for violating the oath of allegiance to the a qualified person to fill the vacancy.
Republic,
The reason behind the right given to a political party to nominate a replacement
(iv) Those with dual citizenship; when a permanent vacancy occurs in the Sanggunian is to maintain the party
representation as willed by the people in the election. In this case, with the
(v) Fugitives from justice in criminal or non-political cases here or abroad;
elevation of Tamayo, who belonged to Reforma-LM, to the position of Vice Mayor,
(vi) Permanent residents in a foreign country or those who have acquired the right a vacancy occurred in the Sanggunian that should be filled up with someone who
to reside abroad and continue to avail of the same right after the effectivity of the belongs to the political party of Tamayo. Otherwise, Reforma-LM’s representation
Code; and in the Sanggunian would be diminished. To argue that the vacancy created was
that formerly held by the 8th Sanggunian member, a Lakas-NUCDKampi member,
(vii) The insane or feeble-minded. would result in the increase in that party’s representation in the Sanggunian at
1. Succession of elective officials the expense of Reforma-LM [Navarro v. Court of Appeals, G.R. No. 141307, March
28, 2001].
a) Permanent vacancies: A permanent vacancy arises when an elective local official fills
a higher vacant office, refuses to assume office, fails to qualify, dies, is removed from The appointment to any vacancy caused by the cessation from office of a member
office, voluntarily resigns, or is permanently incapacitated to discharge the functions of of the sangguniang barangay must be made by the mayor upon recommendation
his office. If a permanent vacancy, occurs in the office of: of the sanggunian. The recommendation by the sanggunian takes the place of
nomination by the political party (since members of the sangguniang barangay
i) Governor or mayor, the vice governor or vice-mayor concerned shall become are prohibited to have party affiliations), and is considered as a condition sine qua
the governor or mayor. non for the validity of the appointment. In Farinas v. Barba, 256 SCRA 396, where
the vacancy to be filled was that of a member of the Sangguniang Bayan who did
ii) Vice-Governor or vice mayor, the highest ranking sanggunian member or, in
not belong to any political party, the Supreme Court held that neither the
case of his permanent inability, the second highest ranking sanggunian member,
petitioner nor the respondent was validly appointed. Not the petitioner, because
and subsequent vacancies shall be filled automatically by the other sanggunian
although he was appointed by the Governor, he was not recommended by the
members according to their ranking. Ranking in the sanggunian shall be
Sangguniang Bayan. Neither the respondent, because although he was
determined on the basis of the proportion of votes obtained by each winning
recommended by the Sangguniang Bayan, he was not appointed by the Governor.
candidate to the total number of registered voters in each district in the
immediately preceding election. v) Vacancy in the representation of the youth and the bar angay in the
sanggunian: filled automatically by the official next in rank of the organization

M.R.A.D.C. LUMBRE 281


CONSTITUTIONAL LAW REVIEW

concerned. In Garvida v. Sales, 271 SCRA 767, the Supreme Court pointed out (c) A tie between or among the highest ranking sanggunian members shall be
that under the Local Government Code, the member of the Sangguniang Kabataan resolved by the drawing of lots.
who obtained the next highest number of votes shall succeed the Chairman if the
latter refuses to assume office, fails to qualify, is convicted of a crime, voluntarily (d) The successors as defined herein shall serve only the unexpired terms of their
resigns, dies, is permanently incapacitated, is removed from office, or has been predecessors.
absent without leave for more than three consecutive months. Ineligibility is not For purposes of this Chapter, a permanent vacancy arises when an elective local official
one of the causes enumerated in the Local Government Code. Thus, to avoid a fills a higher vacant office, refuses to assume office, fails to qualify, dies, is removed
hiatus in the office of the Chairman, the vacancy should be filled by the member from office, voluntarily resigns, or is otherwise permanently incapacitated to discharge
of the Sangguniang Kabataan chosen by the incumbent SK members by simple the functions of his office.
majority from among themselves.
For purposes of succession as provided in the Chapter, ranking in the sanggunian shall
b) Temporary vacancies: be determined on the basis of the proportion of votes obtained by each winning
i) When the governor, city or municipal mayor, or punong barangay is temporarily candidate to the total number of registered voters in each district in the immediately
incapacitated to perform his duties for physical or legal reasons such as, but not preceding local election.
limited to, leave of absence, travel abroad and suspension from office, the vice Section 45. Permanent Vacancies in the Sanggunian. -
governor, city or municipal vice mayor, or the highest ranking sanggunian barangay
member shall automatically exercise the powers and perform the duties and (a) Permanent vacancies in the sanggunian where automatic succession provided
functions of the local chief executive concerned, except the power to appoint, above do not apply shall be filled by appointment in the following manner:
suspend, or dismiss employees which can be exercised only if the period of
(1) The President, through the Executive Secretary, in the case of the
temporary incapacity exceeds thirty working days. [Said temporary incapacity shall
sangguniang panlalawigan and the sangguniang panlungsod of highly
terminate upon submission to the appropriate sanggunian of a written declaration
urbanized cities and independent component cities;
that he has reported back to office. In case the temporary incapacity is due to legal
causes, the local chief executive concerned shall also submit necessary documents (2) The governor, in the case of the sangguniang panlungsod of component
showing that the legal causes no longer exists.] cities and the sangguniang bayan;
ii) When the local chief executive is travelling within the country but outside his (3) The city or municipal mayor, in the case of sangguniang barangay, upon
territorial jurisdiction for a period not exceeding three consecutive days, he may recommendation of the sangguniang barangay concerned.
designate in writing the officer-in-charge of the said office. Such authorization shall
specify the powers and functions that the local official shall exercise in the absence (b) Except for the sangguniang barangay, only the nominee of the political party
of the local chief executive, except the power to appoint, suspend or dismiss under which the sanggunian member concerned had been elected and whose
employees. [If the local chief executive fails or refuses to issue such authorization, elevation to the position next higher in rank created the last vacancy in the
the vice-governor, city or municipal vice-mayor, or the highest ranking sanggunian sanggunian shall be appointed in the manner hereinabove provided. The appointee
barangay member, as the case may be, shall have the right to assume the powers, shall come from the same political party as that of the sanggunian member who
duties and functions of the said office on the fourth day of absence of the local chief caused the vacancy and shall serve the unexpired term of the vacant office. In the
executive, except the power to appoint, suspend or dismiss employees.] appointment herein mentioned, a nomination and a certificate of membership of
the appointee from the highest official of the political party concerned are
Take note; may be asked: conditions sine qua non, and any appointment without such nomination and
certification shall be null and void ab initio and shall be a ground for administrative
Section 44. Permanent Vacancies in the Offices of the Governor, Vice-Governor, Mayor,
action against the official responsible therefore.
and Vice-Mayor. - If a permanent vacancy occurs in the office of the governor or mayor,
the vice-governor or vice-mayor concerned shall become the governor or mayor. If a (c) In case or permanent vacancy is caused by a sanggunian member who does
permanent vacancy occurs in the offices of the governor, vice-governor, mayor, or vice- not belong to any political party, the local chief executive shall, upon
mayor, the highest ranking sanggunian member or, in case of his permanent inability, recommendation of the sanggunian concerned, appoint a qualified person to fill the
the second highest ranking sanggunian member, shall become the governor, vice- vacancy.
governor, mayor or vice-mayor, as the case may be. Subsequent vacancies in the said
office shall be filled automatically by the other sanggunian members according to their (d) In case of vacancy in the representation of the youth and the barangay in the
ranking as defined herein. sanggunian, said vacancy shall be filled automatically by the official next in rank of
the organization concerned.
(b) If a permanent vacancy occurs in the office of the punong barangay, the highest
ranking sanggunian barangay member or, in case of his permanent inability, the Section 46. Temporary Vacancy in the Office of the Local Chief Executive. -
second highest ranking sanggunian member, shall become the punong barangay.

M.R.A.D.C. LUMBRE 282


CONSTITUTIONAL LAW REVIEW

(a) When the governor, city or municipal mayor, or punong barangay is temporarily v) Abuse of authority. In failing to share with the municipalities concerned the
incapacitated to perform his duties for physical or legal reasons such as, but not amount paid by the National Power Corporation for the redemption of the properties
limited to, leave of absence, travel abroad, and suspension from office, the vice- acquired by the Province of Albay at a public auction held for delinquent realty
governor, city or municipal vice-mayor, or the highest ranking sangguniang taxes, the Provincial Officials were held guilty of abuse of authority [Salalima v.
barangay member shall automatically exercise the powers and perform the duties Guingona, 257 SCRA 55].
and functions of the local chief executive concerned, except the power to appoint,
suspend, or dismiss employees which can only be exercised if the period of vi) Unauthorized absence for 15 consecutive working days, except in the case of
temporary incapacity exceeds thirty (30) working days. members of the sangguniang panlalawigan, panlungsod, bayan and barangay.

(b) Said temporary incapacity shall terminate upon submission to the appropriate vii) Application for, or acquisition of, foreign citizenship or residence or the status
sanggunian of a written declaration by the local chief executive concerned that he of an immigrant of another country.
has reported back to office. In cases where the temporary incapacity is due to legal viii) Such other grounds as may be provided in this Code and other laws.
causes, the local chief executive concerned shall also submit necessary documents
showing that said legal causes no longer exist. An elective local official may be removed from office on the grounds enumerated above
by order of the proper court.
(c) When the incumbent local chief executive is traveling within the country but
outside his territorial jurisdiction for a period not exceeding three (3) consecutive a) Doctrine of condonation - Conchita Carpio Morales vs. CA and Jejomar
days, he may designate in writing the officer-in-charge of the said office. Such Erwin S. Binay, Sr., G.R. Nos. 217126-27, 10 November 2015
authorization shall specify the powers and functions that the local official concerned
Generally speaking, condonation has been defined as "[a] victim's express or
shall exercise in the absence of the local chief executive except the power to
implied forgiveness of an offense, [especially] by treating the offender as if there
appoint, suspend, or dismiss employees.
had been no offense."
(d) In the event, however, that the local chief executive concerned fails or refuses
to issue such authorization, the vice-governor, the city or municipal vice-mayor, or The condonation doctrine - which connotes this same sense of complete
the highest ranking sangguniang barangay member, as the case may be, shall have extinguishment of liability as will be herein elaborated upon - is not based on
the right to assume the powers, duties, and functions of the said office on the fourth statutory law. It is a jurisprudential creation that originated from the 1959 case of
(4th) day of absence of the said local chief executive, subject to the limitations Pascual v. Hon. Provincial Board of Nueva Ecija, (Pascual), which was therefore
provided in subsection (c) hereof. decided under the 1935 Constitution.

(e) Except as provided above, the local chief executive shall in no case authorize That being said, this Court simply finds no legal authority to sustain the condonation
any local official to assume the powers, duties, and functions of the office, other doctrine in this jurisdiction. As can be seen from this discourse, it was a doctrine
than the vice-governor, the city or municipal vice-mayor, or the highest ranking adopted from one class of US rulings way back in 1959 and thus, out of touch from
sangguniang barangay member, as the case may be. - and now rendered obsolete by - the current legal regime. In consequence, it is
NOTE: Basis is the percentage of the garnered votes in proportion to the number of high time for this Court to abandon the condonation doctrine that
voters. originated from Pascual, and affirmed in the cases following the same,
such as Aguinaldo, Salalima, Mayor Garcia, and Governor Garcia, Jr. which
2. Discipline of local officials were all relied upon.
Grounds for disciplinary action: An elective local official may be disciplined, suspended, Thus, the doctrine of condonation was abandoned, but it is prospective in effect.
or removed from office on any of the following grounds:
i) Disloyalty to the Republic of the Philippines.
3. Term limits
ii) Culpable violation of the Constitution.
Three years, starting from noon of June 30, 1992, or such date as may be provided by
iii) Dishonesty, oppression, misconduct in office, gross negligence, or dereliction of law, except that of elective barangay officials.
duty. In Regidorv. Chiongbian, 173 SCRA 507, it was held that acts of
lasciviousness cannot be considered misconduct in office, and may not be the basis No local elective official shall serve for more than three consecutive terms in the same
of an order of suspension. To constitute a ground for disciplinary action, the mayor position. The term of office of barangay officials and members of the sangguniang
charged with the offense must be convicted in the criminal action. kabataan shall be for five (5) years, which shall begin after the regular election of
barangay officials pn the second Monday of May, 1997 [R.A. 8524].
iv) Commission of any offense involving moral turpitude or an offense punishable
by at least prision mayor. a. “Three-Term Limit” Rule

M.R.A.D.C. LUMBRE 283


CONSTITUTIONAL LAW REVIEW

i. Abundo vs. COMELEC, GR No. 201716, 8 January 2013 than twenty-five days from the time of the filing of the certificate of
candidacy. In this case, Tallada filed his COC for Governor of Camarines Norte
To constitute a disqualification to run for an elective local office pursuant to for the 2016 elections on October 16, 2015, and he had 25 days therefrom to
the constitutional and statutory provisions, the following requisites must file the petition for denial of due course or cancellation of COC on the ground
concur: (1) that the official concerned has been elected for three of violation of the three-term limit rule, which fell on November 10, 2015.
consecutive terms; and (2) that he has fully served three consecutive However, the petition was filed only on November 13, 2015 which was already
terms. beyond the period to file the same. To reiterate, a violation of the three-term
limit rule is not included among the grounds for disqualification, but a ground
The two-year period during which his opponent, Torres, was serving as mayor for a petition to deny due course to or cancel certificate of candidacy.
should be considered as an interruption, which effectively removed Abundo’s
case from the ambit of the three-term limit rule. Two conditions must concur for the application of the disqualification of a
candidate based on violation of the three-term limit rule, which are: (1) that
The consecutiveness of what otherwise would have been Abundo’s three the official concerned has been elected for three consecutive terms in the same
successive, continuous mayorship was effectively broken during the 2004- local government post, and (2) that he has fully served three consecutive
2007 term when he was initially deprived of title to, and was veritably terms.
disallowed to serve and occupy, an office to which he, after due proceedings,
was eventually declared to have been the rightful choice of the electorate. The constitutional provision fixes the term of a local elective office and limits
an elective official's stay in office to no more than three consecutive terms.
As is clearly provided in Sec. 8, Art. X of the Constitution as well as in Sec. Significantly, this provision refers to a "term" as a period of time - three years
43(b) of the LGC, voluntary renunciation of the office by the incumbent elective - during which an official has title to office and can serve. The word "term" in
local official for any length of time shall NOT, in determining service for three a legal sense means a fixed and definite period of time which the law describes
consecutive terms, be considered an interruption in the continuity of service that an officer may hold an office. According to Mechem, the term of office is
for the full term for which the elective official concerned was elected. In the period during which an office may be held. Upon expiration of the officer's
Aldovino, Jr., however, the Court stated the observation that the law “does not term, unless he is authorized by law to holdover, his rights, duties and
textually state that voluntary renunciation is the only actual interruption of authority as a public officer must ipso facto cease. In the law of public officers,
service that does not affect ‘continuity of service for a full term’ for purposes the most and natural frequent method by which a public officer ceases to be
of the three-term limit rule.” such is by the expiration of the terms for which he was elected or appointed.
The term means the time during which the officer may claim to hold office as
ii. Albania vs. COMELEC, G.R. No. 226792, 7 June 2017 of right, and fixes the interval after which the several incumbents shall succeed
one another.
No, respondent Tallado did not violate the three term limit rule under Section
43 of RA No 7160, otherwise known as the Local Government Code of 1991. In this case, while respondent ran as Governor of Camarines Norte in the 2007
elections, he did not win as such. It was only after he filed the petition for
Sections 12 and 68 of Batas Pambansa Blg. 881, as amended, otherwise known correction of manifest error that he was proclaimed as the duly-elected
as the Omnibus Election Code of the Philippines, as well as Section 40 of the Governor. He assumed the post and served the unexpired term of his opponent
Local Government Code, does not include the two grounds relied upon by from March 22, 2010 until June 30, 2010. Consequently, he did not hold the
Albania for a candidate's disqualification. office for the full term of three years to which he was supposedly entitled to.
Thus, such period of time that respondent served as Governor did not
Tallada's suspension from office is not a ground for a petition for constitute a complete and full service of his term. The period when he was out
disqualification as Section 40 (b) clearly speaks of removal from office as a of office involuntarily interrupted the continuity of his service as Governor. As
result of an administrative offense that would disqualify a candidate from he had not fully served the 2007-2010 term, and had not been elected for
running for any elective local position. In fact, the penalty of suspension cannot three consecutive terms as Governor, there was no violation of the three-term
be a bar to the candidacy of Tallada so suspended as long as he meets the limit rule when he ran again in the 2016 elections.
qualifications for the office.
In the instant case, since Tallado did not serve the full 2007-2010 term, it
While the alleged violation of the three-term limit rule is not a ground for a cannot be considered as one term for purposes of counting the three-term
petition for disqualification, however, it is an ineligibility which is a proper threshold. Consequently, Tallado cannot be said to have continuously served
ground for a petition to deny due course to or to cancel a Certificate of as Governor for three consecutive terms prior to the 2016 elections.
Candidacy under Section 78 of the OEC. As the petition filed is indeed a petition
under Section 78 of the OEC, the filing of the same must comply with the
period prescribed therein, i.e., the filing of the same must be made not later

M.R.A.D.C. LUMBRE 284


CONSTITUTIONAL LAW REVIEW

V. Inter-governmental Relations requirement, under Section 27 of the LGC, relative to a project within the territorial
jurisdiction of the SBMA under RA 7227.

SECTION 12. Subic Special Economic Zone. — Subject to the concurrence by


1. Hon. Paje vs. Hon. Casino, G.R. No. 207257, 3 February 2015
resolution of the sangguniang panlungsod of the City of Olongapo and the
The appellate court erred when it ruled that compliance with Section 27, in relation to sangguniang bayan of the Municipalities of Subic, Morong and Hermosa, there is
Section 26, of the LGC (i.e., approval of the concerned sanggunian requirement) is hereby created a Special Economic and Free-port Zone consisting of the City of
necessary prior to issuance of the subject ECC. The issuance of an ECC does not, by Olongapo and the Municipality of Subic, Province of Zambales, the lands occupied
itself, result in the implementation of the project. Hence, there is no necessity to secure by the Subic Naval Base and its contiguous extensions as embraced, covered, and
prior compliance with the approval of the concerned sanggunian requirement, and the defined by the 1947 Military Bases Agreement between the Philippines and the
issuance of the subject ECC without first complying with the aforesaid requirement does
United States of America as amended, and within the territorial jurisdiction of the
not render it invalid. The appellate court also erred when it ruled that compliance with
Municipalities of Morong and Hermosa, Province of Bataan, hereinafter referred to
the aforesaid requirement is necessary prior to the consummation of the LDA. By virtue
as the Subic Special Economic Zone whose metes and bounds shall be delineated
of the clear provisions of RA 7227, the project is not subject to the aforesaid
requirement and the SBMA’s decision to approve the project prevails over the apparent in a proclamation to be issued by the President of the Philippines. Within thirty (30)
objections of the concerned sanggunians. Thus, the LDA entered into between SBMA days after the approval of this Act, each local government unit shall submit its
and RP Energy suffers from no infirmity despite the lack of approval of the concerned resolution of concurrence to join the Subic Special Economic Zone to the office of
sanggunians. the President. Thereafter, the President of the Philippines shall issue a proclamation
defining the metes and bounds of the Zone as provided herein.
The prior approval of the concerned sanggunian requirement is an attribute and
implementation of the local autonomy granted to, and enjoyed by LGUs under the Subsequently, the aforesaid sanggunians submitted their respective resolutions of
Constitution. The LGU has the duty to protect its constituents and interests in the concurrence and the President issued Presidential Proclamation No. 532, Series of 1995,
implementation of the project. Hence, the approval of the concerned sanggunian is defining the metes and bounds of the SSEZ. To achieve the above-mentioned purposes,
required by law to ensure that local communities partake in the fruits of their own the law created SBMA to administer the SSEZ. In the process, SBMA was granted broad
backyard. and enormous powers as provided for under Section 13(b) of RA 7227.

For Section 27, in relation to Section 26, to apply, the following requisites must concur: The SBMA was given broad administrative powers over the SSEZ and these necessarily
(1) the planning and implementation of the project or program is vested in a national include the power to approve or disapprove the subject project, which is within its
agency or government-owned and-controlled corporation, i.e., national programs territorial jurisdiction. But, as previously discussed, the LGC grants the concerned
and/or projects which are to be implemented in a particular local community; and (2) sanggunians the power to approve and disapprove this same project. The SBMA asserts
the project or program may cause pollution, climatic change, depletion of non- that its approval of the project prevails over the apparent disapproval of the concerned
renewable resources, loss of cropland, rangeland, or forest cover, extinction of animal sanggunians. There is, therefore, a real clash between the powers granted under these
or plant species, or call for the eviction of a particular group of people residing in the two laws.
locality where the project will be implemented.
Which shall prevail? Section 12 of RA 7227 provides:
In the case at bar, the two requisites are evidently present: (1) the planning and
Sec. 12. Subic Special Economic Zone. x x x The abovementioned zone shall be
implementation of the subject project involves the Department of Energy, DENR, and
subjected to the following policies:
SBMA; and (2) the subject project may cause pollution, climatic change, depletion of
non-renewable resources, loss of cropland, rangeland, or forest cover, and extinction of (a) Within the framework and subject to the mandate and limitations of the
animal or plant species, or call for the eviction of a particular group of people residing Constitution and the pertinent provisions of the Local Government Code, the Subic
in the locality where the project will be implemented. Hence, Section 27 of the LGC Special Economic Zone shall be developed into a self-sustaining, industrial,
should ordinarily apply. commercial, financial and investment center to generate employment opportunities
in and around the zone and to attract and promote productive foreign investments;
It is not disputed that no approval was sought from the concerned sanggunians relative
xxxx
to the subject project. What is more, the affected LGUs have expressed their strong
oppositions to the project through various sanggunian resolutions. However, it is also (i) Except as herein provided, the local government units comprising the Subic
undisputed that the subject project is located within the SSEZ and, thus, under the Special Economic Zone shall retain their basic autonomy and identity. The cities
territorial jurisdiction of the SBMA pursuant to RA 7227. Thus, we are tasked to shall be governed by their respective charters and the municipalities shall operate
determine the applicability of the prior approval of the concerned sanggunian

M.R.A.D.C. LUMBRE 285


CONSTITUTIONAL LAW REVIEW

and function in accordance with Republic Act No. 7160, otherwise known as the operate transfer scheme or joint-venture the required utilities and infrastructure in
Local Government Code of 1991. coordination with local government units and appropriate government agencies
concerned and in conformity with existing applicable laws therefor;
This section sets out the basic policies underlying the creation of the SSEZ. Indeed, as
noted by the appellate court, Section 12(i) expressly recognizes the basic autonomy In sum, we find that the implementation of the project is not subject to the prior
and identity of the LGUs comprising the SSEZ. However, the clause "except as herein approval of the concerned sanggunians, under Section 27 of the LGC, and the SBMA’s
provided" unambiguously provides that the LGUs do not retain their basic autonomy decision to approve the project prevails over the apparent objections of the concerned
and identity when it comes to matters specified by the law as falling under the powers, sangguniansof the LGUs, by virtue of the clear provisions of RA 7227. Thus, there was
functions and prerogatives of the SBMA. no infirmity when the LDA was entered into between SBMA and RP Energy despite the
lack of approval of the concerned sanggunians.
In the case at bar, we find that the power to approve or disapprove projects within the
SSEZ is one such power over which the SBMA’s authority prevails over the LGU’s
autonomy. Hence, there is no need for the SBMA to secure the approval of the
concerned sanggunians prior to the implementation of the subject project.

This interpretation is based on the broad grant of powers to the SBMA over all
administrative matters relating to the SSEZ under Section 13 of RA 7227, as afore-
discussed. Equally important, under Section 14, other than those involving defense and
security, the SBMA’s decision prevails in case of conflict between the SBMA and the
LGUs in all matters concerning the SSEZ, viz.:

Sec. 14. Relationship with the Conversion Authority and the Local Government
Units.

(a) The provisions of existing laws, rules and regulations to the contrary
notwithstanding, the Subic Authority shall exercise administrative powers, rule-
making and disbursement of funds over the Subic Special Economic Zone in
conformity with the oversight function of the Conversion Authority.

(b) In case of conflict between the Subic Authority and the local government units
concerned on matters affecting the Subic Special Economic Zone other than
defense and security, the decision of the Subic Authority shall prevail.

Clearly, the subject project does not involve defense or security, but rather business
and investment to further the development of the SSEZ. Such is in line with the
objective of RA 7227 to develop the SSEZ into a self-sustaining industrial, commercial,
financial and investment center. Hence, the decision of the SBMA would prevail over
the apparent objections of the concerned sanggunians of the LGUs.

Significantly, the legislative deliberations on RA 7227, likewise, support and confirm the
foregoing interpretation. As earlier noted, Section 13 b(4) of RA 7227 provides:

Sec. 13. The Subic Bay Metropolitan Authority. –x x x x


(b) Powers and functions of the Subic Bay Metropolitan Authority - The Subic Bay
Metropolitan Authority, otherwise known as the Subic Authority, shall have the
following powers and function: x x x x

(4) To construct, acquire, own, lease, operate and maintain on its own or through
contract, franchise, license permits bulk purchase from the private sector and build-

M.R.A.D.C. LUMBRE 286


CONSTITUTIONAL LAW REVIEW

CHAPTER 6: PUBLIC INTERNATIONAL LAW ii. Dualism (or Pluralism)


a. International Law vs. National Law

The distinctions lie in that municipal law is issued by a political superior for
I. General Principles observance by those under its authority, while international law is not
imposed but adopted by States as a common rule of action.

International Law, Defined. Municipal law consists of enactments of the law-making authority, while
international law is derived from such sources as international customs,
Traditional: That branch of public law which regulates the relations of States and of conventions, or general principles of law.
other entities which have been granted international personality. [This definition focuses
on subjects, which are entities which possess international personality and with rights Municipal law regulates relations of individuals among themselves, while
and obligations recognized under international law, as against objects, which are international law applies to relations between states and international
persons or things in respect of which rights are held and obligations assumed by the persons.
subjects of international law.] Violations of municipal law are redressed through local judicial and
administrative processes, while in international law, they are resolved
Modern: The law that deals with the conduct of States and international organizations, through state-to-state transactions.
their relations with each other and, in certain circumstances, their relations with
persons, natural or juridical. Breaches of municipal law entail individual responsibility, while in
international law there is collective responsibility.

b. Approaches to Implement International Law in National Law


1. Theories of International Law
i. Doctrine of Transformation
a. Natural Law Theory/School
It requires the enactment by the legislative body of such international law
There is a natural and universal principle of right and wrong, independent of mutual principles as are sought to be part of municipal law.
intercourse or compact, which can be discovered and recognized by every
Individual through the use of his reason and conscience. Since individuals compose ii. Doctrine of Incorporation
the State whose will is but the collective will of the inhabitants, the State also
Expressed in Sec. 2, Art. II, Philippine Constitution, as follows: “The Philippines
becomes bound by the law of nature.
renounces war as an instrument of national policy, adopts the generally
b. Positive Law Theory/School accepted principles of international law as part of the law of the land, and
adheres to the policy of peace, equality, justice, freedom, cooperation and
The binding force of international law is derived from the agreement of the States amity with all nations”.
to be bound by it. In this context, international law is not a law of subordination
but of coordination.
See: Kuroda vs. Jalandoni, 83 Phil. 171 (although the Philippines was not a
c. Eclectic Theory/School (a.k.a. the Grotian School) signatory to the Hague and Geneva Conventions, international jurisprudence
is automatically incorporated in Philippine law, thus making war crimes
In so far as it conforms to the dictates of right reason, the voluntary law may be punishable in the Philippines).
said to blend with the natural law and be, indeed, an expression of it. In case of
conflict, the natural law prevails, being the more fundamental law. iii. What the Philippine Law Follows

Conflict between International Law and Municipal Law.


NOTE: In case of conflict, natural law theory shall prevail as it is acceptable to
human nature. a) On the domestic sphere, with a local court deciding:
2. Relationship Between International and National/Municipal Law
i) If the conflict is with the Constitution: uphold the Constitution. [See Sec.
a. Independence of International Law from National Law 5(2)(a), Art. VIII, Philippine Constitution, which provides that the
Supreme Court has the power to declare a treaty or executive agreement
i. Monism
unconstitutional.] In Secretary of Justice v. Judge Lantion, G.R. No.
There is no substantial distinction between international law and municipal law. 139465, January 18, 2000, it was held that in states where the
Constitution is the highest law of the land, such as the Republic of the

M.R.A.D.C. LUMBRE 287


CONSTITUTIONAL LAW REVIEW

Philippines, both statutes and treaties may be invalidated if they are in A norm accepted and recognizes by the international community as a norm
conflict with the Constitution. from which no derogation is permitted, and which can be modified only by a
subsequent norm of general international law having the same character.
ii) If the conflict is with a statute: The doctrine of incorporation, as applied
iii. Erga Omnes Obligation “towards all”
in most countries, decrees that rules of international law are given equal
standing with, but are not superior to, national legislative enactments. A An obligation of every State towards the international community as a whole.
treaty may repeal a statute, and a statute may repeal a treaty; thus, the
NOTE: Q: Suppose the old jus cogens norm against torture was abrogated in
principle of lex posterior derogat priori, that which comes last in time, will
that the same is now allowed. Is this proper?
usually be upheld by the municipal tribunal.
A: Yes. The old jus cogens norm was modified by a subsequent norm of the
b) On the international sphere, with an international tribunal deciding: same character.
international law is superior to municipal law, because international law
provides the standard by which to determine the legality of a State’s conduct. c. General Principles of Law

These are rules derived mainly from natural law, observed and recognized by
NOTE: civilized nations, e.g., res judicata, prescription, pacta sunt servanda and estoppel.
Q: Can a law void an international agreement?
To these may be added the principle of ex aequo et bono (what is good and just),
A: Yes, following lex posteriori, derogat priori. provided that the parties to the dispute agree thereto, as provided in Art. 38 (1),
Statute of the International Court of Justice.
Q: What doctrine does the Philippines follow?
NOTE: Q: General principles of international law shall apply in what cases?
A: As to treaties/conventions, the doctrine of transformation applies. As to
customary international law and general principles of international law, the doctrine A: If there is: (1) conflict between treaty and customary law; and (2) there is
of incorporation applies. (Bernas) conflict between two customary norms.
i. Non liquet in International Law “it is not clear”
II. Sources of International Law It refers to a situation in which a competent court or tribunal fails to decide
the merits of an admissible case for whatever reason, be it the absence of
On the domestic sphere, the constitution, legislative enactments and case law (stare suitable law, the vagueness or ambiguity of rules, inconsistencies in law, or
decisis). On the international plane, it is a bit complicated because there is no body the injustice of the legal consequences
likened to a national legislature, no fundamental law, and the doctrine of precedents is
not applicable. Reference to such principles is to both those which are inferred from municipal
laws and those which have no counterparts in municipal law and are have no
1. Primary Sources counterparts in municipal law and are inferred from the nature of the
a. International Treaties and Conventions international community. If there is no treaty relevant to a dispute, or if there
is no rule of customary international la that can be applied to it, the ICJ is
Whether general or particular, establishing rules expressly recognized by the directed, under Article 38 of its Statute, to apply general principles of
contesting states. international law.
b. Customary International Law Simply stated, such principles are gap-filler provisions, utilized by the ICJ in
reference to rules typically found in domestic courts and domestic legal
As evidence of a general practice accepted as binding law through persistent usage
systems in order to address procedural and other issues.
over a long period of time. It is necessary, however, that the custom be [i]
prevailing practice by a number of states; [ii] repeated over a considerable period The main objective of inserting the third source in Article 38 is to fill in gaps in
of time; and [iii] attended by opinio juris or a sense of legal obligation. treaty and customary law and to meet the possibility of a non liquet.

i. Opinio Juris “an opinion of law” Non liquet means the possibility that a court or tribunal could not decide a case
because of a ‘gap’ in law.
Recognition of a practice as a legal norm and therefore obligatory.
Examples: Burden of proof, admissibility of evidence, waiver, estoppel, unclean
ii. Jus cogens “compelling law” hands, necessity, and force majeure.

M.R.A.D.C. LUMBRE 288


CONSTITUTIONAL LAW REVIEW

2. Subsidiary Sources An object is a person or thing in respect of which rights are held and obligations
assumed by the subject; it is not directly governed by the rules of international law; its
a. Judicial Decisions and Highly Regarded Publicists rights are received, and its responsibilities imposed, indirectly through the
Judicial Decisions: Generally of international tribunals, the most authoritative being instrumentality of an international agency.
the International Court of Justice. They are not really sources, but “subsidiary 1. States
means” for finding what the law is, and whether a norm has been accepted as a
rule of international law. The decision of a national court may be used depending It is a community of persons, more or less numerous, permanently occupying a definite
upon the prestige and perceived impartiality of the domestic court, not being in portion of territory, independent of external control, and possessing an organized
conflict with the decisions of international tribunals, and its admissibility in the government to which the great body of inhabitants render habitual obedience.
forum where it is cited.
NOTE: Recognition is relative; most do not believe that this is required, except in the
Writings of Highly Regarded Publicists: Must be fair and unbiased representation of case of belligerents.
international law by acknowledged authorities in the field.
2. International Organizations
b. ICJ Advisories
Bodies created by sovereign states and whose functioning is regulated by international
An advisory opinion is legal advice provided to the United Nations or a specialized law, not the law of any given country. They have functional personality which is limited
agency by the International Court of Justice, in accordance with Article 96 of the to what is necessary to carry out their functions as found in the instruments of the
UN Charter. organization.
The General Assembly and the Security Council may request advisory opinions on
NOTE: The importance of knowing whether an entity is a subject of international law is
“any legal matter.” Other organs and the specialized agencies may request advisory
opinions on “legal questions arising within the scope of their activities.” that it can, by itself, sue before international tribunals without lodging its grievance with
the State. In order for international organizations to be considered a subject, the
NOTE: Subsidiary sources cannot be the sole basis of international law, or in the following requisites should concur:
absence of primary sources.
a. There must be a permanent association of Statees in that international
3. Hierarchy of Sources
organization; i.e. the group of States committed themselves to be associated within
Interpretation of Article 38 of the ICJ: the international organization
Although the provision is silent on the question of whether the three primary sources b. The States which created the international organization considers the latter as
have the same hierarchic value, although by practice, treaties take precedence over having a separate and distinct personality (similar to that in corporations);
customs, and customs over general principles of law, except:
c. The international organization possesses rights and duties.
a. Jus Cogens
3. Individuals
Supra. Examples: slave trade, piracy, and terrorism. See Human Rights Cases vs.
Marcos, where it was held that official torture of prisoners/ dissenters was a In the early development of international law human beings were exclusively under the
violation of the principle of jus cogens. control of states. In international law they were objects or at best “beneficiaries” of
international law. With the greater global awareness of human rights individuals have
now come to be recognized as possessing albeit limited rights and obligations in
III. Subjects and Objects of International Law international law.
A subject is an entity that has rights and responsibilities under international law; it can Among the obligations of individuals are those arising from the regulation of armed
be a proper party in transactions involving the application of the law of nations among
conflict. Violation of these rules can place individuals under criminal responsibility. There
members of the international community.
are also rules on international crimes to which individuals are subject such as crimes
The subjects of international law are: states, colonies and dependencies, mandates and against humanity, genocide, aggression, and terrorism.
trust territories, the Holy See (Vatican City), the United Nations, belligerent
When individual rights are violated, however, individuals still have to rely on the
communities, international administrative bodies, and, to a certain extent, individuals.
enforcement power of states. But some treaties have provided for the right of individuals

M.R.A.D.C. LUMBRE 289


CONSTITUTIONAL LAW REVIEW

to petition international bodies alleging that a contracting state has violated some of On the Kalayaan Islands: Between 1947 to 1956, Tomas Cloma, a Filipino,
their human rights. discovered the Kalayaan Islands (a 53-island group not part of the Spratlys).
Subsequently, Cloma ceded his rights to the Philippine Government. By virtue of
4. Others Presidential Decree No. 1596 (June 11, 19978), the Philippines formally laid claim
to the islands by virtue of occupation and exercise of jurisdiction. The Municipality
a. Non-Governments Organizations? was established as part of the Province of Palawan. On May 20, 1980, the
Those organizations founded by private individuals, which are independent of Philippines registered its claim with the United Nations Secretariat. The Philippine
States, oriented towards the rule of law, pursue public rather than private goals as claim to the islands is justified by reason of history, indispensable need, and
an objective, and possess a minimal organizational structure. Examples: ICRC or effective occupation and control. Thus, in accordance with international law, the
Kalayaan group is subject to the sovereignty of the Philippines. (By virtue of the
Human Rights Watch.
Manila Declaration of 1992, it was agreed that whatever conflicting claims there
NOTE: As a rule, they are not considered subjects of international law, except may be over the islands shall be resolved in a peaceful manner, through diplomatic
negotiations.)
where their functions are sanctioned by the United Nations or the international
community. 2. Prescription

b. Corporations (Multinational) Territory may also be acquired through continuous and uninterrupted possession
over a long period of time, just like in civil law. In international law, however, there
Those which operate in many countries. They are not considered subjects as there is no rule of thumb as to the length of time needed for acquisition of territory
is no provision which considers them as such. through prescription. In this connection, consider the Grotius doctrine of
immemorial prescription, which speaks of uninterrupted possession going beyond
memory.
IV. Fundamental Rights of States
3. Cession (By Treaty)

Cession may be voluntary, through a treaty of sale, e.g., the sale of Alaska by
1. The Right of Existence, Territorial integrity and Self-Preservation Russia to the U.S., or through a treaty of donation, e.g., the donation of Sabah by
Borneo to the Sultan of Sulu. Cession may also be involuntary, or forced, such as
Right of Existence and Self-Preservation: By far the most basic and important right. the treaty entered into by the U.S. and Spain after the Spanish-American War,
although the treaty was denominated one of sale, whereby Spain ceded the
Art. 51 of the UN Charter recognizes the right of the state to individual and collective Philippines, Puerto Rico, Marianas and Guam to the U.S. for $20M.
self-defense (through regional arrangements) if an armed attack occurs against such
state, until the Security Council has taken measures necessary to maintain international 4. Conquest and Subjugation
peace and security. However, the right may be resorted to only upon a clear showing Conquest is the acquisition of sovereignty of a country by force of arms, exercised
of grave and actual danger, and must be limited by necessity. It is eventually the
by an independent power, which reduces the vanquished to the submission of its
Security Council which determines whether or not an “armed attack” has taken place. empire. Physical conquest is not enough for the title to ripen to real ownership,
similar to discovery and occupation. Annexation or subjugation must follow, which
Right to Territorial Integrity: Principle under international law that prohibits states from takes place if a formal cession is made in the treaty of peace.
the use of force against the "territorial integrity or political independence" of another
state. It is enshrined in Article 2(4) of the UN Charter and has been recognized as This is not a legitimate mode of acquiring territory under the UN Charter [Article
customary international law. 2(4)].

5. Accretion
Modes of Acquisition:
A mode of acquiring property produced by or which is attached or united to a thing
1. Discovery and Occupation already owned by a person. In Roman Law, this is known as accession which may
either be: (1) accession continua, which occurs as a consequence of forces external
Territory not belonging to any State, or terra nulius, is placed under the sovereignty of the thing itself; and (2) accession discreta, which occurs as a consequence of
of the claiming State. “Discovery”, alone, merely creates an inchoate right; it must forces inherent in the thing itself.
be followed within a reasonable time by effective occupation and administration.
Thus, in the Palmas Island Arbitration case, the inchoate right flowing from 2. The Right of Sovereignty and Independence
discovery was deemed lost because administration was not undertaken within a
reasonable time.

M.R.A.D.C. LUMBRE 290


CONSTITUTIONAL LAW REVIEW

Sovereignty is defined as the supreme power in a State by which that State is governed, 3) acts of state;
or the supreme, absolute, uncontrollable power by which any State is governed. It has
two (2) aspects: internal and external. 4) foreign merchant vessels exercising rights of innocent passage or arrival
under stress;
In its internal aspect, sovereignty is the power inherent in the people or vested in its
ruler by the Constitution to govern the State. Such sovereignty does not, in any degree, 5) foreign armies passing through or stationed in its territories with its
depend upon its recognition by other States. permission; and

In its external aspect, sovereignty consists in the independence of one political society 6) such other persons or property, including organizations like the United
in respect to all other political societies. The external sovereignty of any State requires Nations, over which it may, by agreement, waive jurisdiction.
the recognition by other States in order to render it perfect and complete. This external Applicable to both criminal and civil cases.
manifestation is called independence.
ii. Nationality principle and statelessness
a. Self-Determination
A State may exercise jurisdiction over its nationals, with respect to their
People’s pursuit of its political, economic, social, and cultural development within conduct, whether within or outside its territory.
the framework of an existing State.
b. Decolonization and Uti Possidetis “as you possess” Instances when a person may be declared stateless:

Decolonization – a state which has been previously colonized regains its 1. Conflict of 2 laws;
sovereignty. 2. State deprived person of nationality;
i. official act;
Uti Possidetis – A principle of international law that allows retention of property or
ii. political event;
territory in the belligerent’s actual possession at the time of cessation of hostilities.
3. Person’s own renunciation;
3. The Right of Equality 4. Voluntary release;
Legal equality or equality before the law, which means that the rights of a State,
regardless of its size, population, power, degree of civilization, wealth, etc., must be Nationality of Foundling – where he or she may be found.
respected, and if a State has obligations, it has to respect them and comply with them.
iii. Protective principle
NOTE: An example would be that each Member State is given one vote, regardless of
Any State has the right to punish acts even if committed outside its territory,
its size, wealth, power, etc.
when such acts constitute attacks against its security, as long as that conduct
4. The Right to Property and Jurisdiction is generally recognized as criminal by states in the international community.

Three Domains of the State: iv. Universality principle

1. Terrestrial; Certain offenses are so heinous and so widely condemned that any state that
captures an offender may prosecute and punish that person on behalf of the
2. Fluvial or maritime; and international community regardless of the nationality of the offender or victim
3. Aerial. or where the crime was committed. Not just heinous, but universal crimes.

a. Bases of Jurisdiction a. Treaties Providing for ‘Universal Jurisdiction’: Aut Dedere Aut
Judicare
i. Territoriality principle
A state may prosecute a crime committed elsewhere if it against jus
A state has jurisdiction over all persons and property within its territory. The
cogens norms.
jurisdiction of the nation within its own territory is necessary, exclusive, and
absolute. However, there are a few exceptions on when a state cannot Aut Dedere Aut Judicare “either extradite or prosecute” – Legal
exercise jurisdiction even within its own territory, to wit: obligation of states under PIL to prosecute persons who commit serious
international crimes where no other state has requested extradition.
1) foreign states, head of states, diplomatic representatives, and consuls to a
certain degree; Extradition is given more preference than prosecution.
2) foreign state property; v. Passive personality principle

M.R.A.D.C. LUMBRE 291


CONSTITUTIONAL LAW REVIEW

It authorizes states to assert jurisdiction over offenses committed against their crime charged. As homicide is necessarily included in the crime of parricide,
citizens abroad. It recognizes that each state has a legitimate interest in then there is no violation of the principle of specialty.
protecting the safety of its citizens when they journey outside national
boundaries. ii. Dual criminality principle
a. Government of Hongkong Special Administrative Region
Not automatically applicable; applies only when other principles resorted to do
(HKSAR) vs. Munoz, G.R. No. 207342, 16 August 2016
not apply. It is considered as a last resort principle.
For purposes of the extradition of Munoz, the HK.SAR as the requesting
vi. Conflicts of jurisdiction state must establish the following six elements, namely: (1) there must
be an extradition treaty in force between the HK.SAR and the Philippines;
Modes of addressing conflicts of jurisdiction:
(2) the criminal charges· that are pending 'in the HK.SAR against the
person to be extradited;(3) the crimes for which the person to be
1. Balancing Test – It is a judicial doctrine whereby a court measures
extradited is charged are extraditable within the terms of the treaty; ( 4)
competing interest–as between individual rights and governmental powers, or
the individual before the court is the same person charged in the HKSAR;
between state authority and federal supremacy – and decides which interest
(5) the evidence submitted establishes probable cause to believe that the
should prevail. (Black’s Law Dictionary) person to be extradited committed the offenses charged; and (6) the
offenses are criminal in both the HK.SAR and the Philippines (double
2. International Comity – Even when a state has basis for exercising
criminality rule).
jurisdiction, it will refrain from doing so if its exercise will be unreasonable.
It was as to the sixth element that the CA took exception as not having
3. Forum Non Conveniens – It is the discretionary power that allows courts to
been established.
dismiss a case where another court, or forum, is much better suited to hear
the case. The CA ultimately concluded that the crime of accepting an advantage as
an agent did not have an equivalent in this jurisdiction considering that
b. Extradition when the unauthorized giving and receiving of benefits happened in the
The right of a foreign power, created by treaty, to demand the surrender of one private sector, the same was not a crime because there was no law that
accused or convicted of a crime within its territorial jurisdiction, and the correlative defined and· punished such act as criminal in this jurisdiction.
duty of the other State to surrender.
Under the rule of specialty in international law, a Requested State shall
i. Principle of specialty surrender to a Requesting State a person to be tried only for a criminal
offense specified in their treaty of extradition. Conformably with the dual
A fugitive who is extradited may be tried only for the crime specified in the criminality rule embodied in the extradition treaty between the Philippines
request for extradition and included in the list of offenses in the extradition and the Hong Kong Special Administrative Region (HKSAR), however, the
treaty, unless the requested State does not object to the trial of such person Philippines as the Requested State is not bound to extradite the
for the unlisted offense. respondent to the jurisdiction of the HKSAR as the Requesting State for
the offense of accepting an advantage as an agent considering that the
NOTE: extradition treaty is forthright in providing that surrender shall only be
granted for an offense coming within the descriptions of offenses in its
Q: Suppose X was charged with parricide by State A and the latter requests Article 2 insofar as the offenses are punishable by imprisonment or other
State B to extradite him. Parricide is also a crime punishable in State B, which form of detention for more than one year, or by a more severe
satisfies the requirement of dual criminality. State B extradites X. However, penalty according to the laws of both parties.
upon investigation, it was found that the victims of X were not related to him.
X was charged with homicide instead. Does this violate the principle of Considering that the transactions were entered into by and in behalf of the
specialty? Central Bank of the Philippines, an instrumentality of the Philippine
Government, Munoz should be charged for the offenses not as a regular
A: Sir is of the opinion that it does not. There is a general principle that a agent or one representing a private entity but as a public servant or
person may be charged not only of the crime he or she has committed, but employee of the Philippine Government. Yet, because the offense of
also with the crime which necessarily includes or is necessarily included in the accepting an advantage as an agent charged against him in the
HK.SAR is one that deals with private sector bribery, the conditions
for the application of the double criminality rule are obviously not

M.R.A.D.C. LUMBRE 292


CONSTITUTIONAL LAW REVIEW

met. Accordingly, the crime of accepting an advantage as an agent must of the principle that ambassadors do not have immunity from penalty, but only
be dropped from the request for extradition. Conformably with the immunity from local jurisdiction?
principle of specialty embodied in Article 1 7 of the RP-HK Agreement,
Munoz should be proceeded against only for the seven counts of A: Under the Vienna Convention on Diplomatic Relations, a diplomatic agent shall enjoy
conspiracy to defraud. As such, the HK.SAR shall hereafter arrange for immunity from criminal jurisdiction of the receiving State.
Munoz's surrender within the period provided under Article 15 of the RP-
HK Agreement. If there is only immunity from liability, it means that a person can be held liable if his
State waives his immunity. If the immunity covers not only immunity from jurisdiction
5. The Right of Legation (Right of Diplomatic Intercourse)
but also immunity from liability, then it cannot be waived.
It is the right of the state to send and receive diplomatic missions, which enables states
to carry on friendly intercourse. It is governed by the Vienna Convention on Diplomatic Q: Consul v. Ambassador
Relations (1961).
A: Consul – in charge of economic and commercial intercourse with receiving State.
The exercise of this right is one of the most effective ways of facilitating and promoting
Ambassador – has immunity from jurisdiction, which the consul does not have (this is
intercourse among nations. Through the active right of sending diplomatic
an important distinction).
representatives and the passive right of receiving them, States are able to deal more
directly and closely with each other in the improvement of their mutual intercourse.

NOTE: As the right of legation is purely consensual, the State is not obliged to maintain V. Fundamental Duties of States
diplomatic relations with other States.

If it wants to, a State may shut itself from the rest of the world, as Japan did until the 1. Doctrine of State Responsibility
close of the 19th century. Disadvantage: A policy of isolation would hinder the
progress of a State since it would be denying itself of the many benefits available from A State may be held responsible for an international delinquency, directly or indirectly,
the international community. imputable to it which causes injury to the national of another State. Liability will attach
to the State where its treatment of the alien falls below the international standard of
Q: May State A force State B to establish an embassy in State B’s territory? justice or where it is remiss in according him the protection or redress that is warranted
by the circumstances.
A: No, the basis of right of legation is the consent of the State parties. A State may not
Elements:
be forced to maintain diplomatic relations if it is not willing. Establishing a diplomatic
mission is a power solely vested in the President. 1. An act or omission in violation of international law;

Q: Active v. Passive 2. The same is attributable to the State;

3. The act or omission caused damage to a third State either directly or indirectly
A: Active Right Legation – the right of the State to send envoys or establish diplomatic
to a national of the third State.
missions.
Kinds:
Passive Right of Legation – the right to receive or accept envoys or missions.
1. Direct State Responsibility – where the international delinquency was committed
Q: Situations where the exercise of passive right of legation prevents the exercise of by superior government officials or organs of the State such as the Chief Executive
active right of legation? or the national legislature. Liability will attach immediately as their acts may not
be effectively prevented or reversed under the Constitution or laws of the State.
A: (1) If the diplomatic relations between the States are covered by a treaty, but the
2. Indirect State Responsibility – where the offense is committed by inferior
Senate does not ratify said treaty; (2) By not accepting the envoy of the other State; government officials or by private individuals. The State will be held liable only if
(3) Through legislation, if the diplomatic representative is declared a persona non grata; by reason of its indifference in preventing or punishing it, it can be considered to
(4) The receiving State orders the diplomatic representative to leave the country. have connived in effecting its commission.

Q: Ambassador Teehankee was convicted for profiteering in Geneva, but was neither a. The Rules of Attribution: Lex Specialis “law governing a specific
imprisoned nor fined because he invoked his diplomatic immunity. Immunity of subject matter”
ambassadors covers only jurisdiction, but not liability. What then, is the consequence

M.R.A.D.C. LUMBRE 293


CONSTITUTIONAL LAW REVIEW

It is a legal construct whereby an internationally unlawful conduct of state organ A: Visa is a permit to enter a country, while a passport is permit to exit a country. Visa-
acting in that capacity is regarded as the conduct of state itself, making that state free country? It is not actually visa-free, but that a visa was previously given in general
responsible for its own acts or omissions, but as an abstract entity it can physically to the public.
act only through individuals or group of individuals performing “act of the state” on
its behalf. Q: What is the Doctrine of Non-Refouement?

NOTE: A: Principle in international law which prohibits States from expelling from their
territories and returning refugees to places where their lives or freedom could be
Q: What are Secondary Rules? threatened.
A: These address basic issues of responsibility and remedies available for breach
of “primary” or substantive rules of international law, such as with respect to the Q: What are the kinds of Asylum?
use of armed force.
A: 1. Territorial Asylum - Refuge within the territory of the sheltering state, the
The espousing of a claim of an individual claimant is a political question, as in prosecution which a refugee obtains by escaping to, or remaining upon, the territory of
the case of Vinuya v. Executive Secretary. a State other than that the State that “wants” him, until the protection is terminated by
his extradition.
Q: What is the basis for a person to ask for relief directly from international courts?
2. Extraterritorial Asylum – Asylum in what are considered the “extensions” of a State’s
A: Some treaties allow individuals to claim relief directly from international courts. territory. It includes: asylum in foreign public ships; and diplomatic asylum is the
prosecution afforded by a State to a refugee by granting him an asylum in or upon its
Q: How to determine that State is attributable to the crime?
diplomatic buildings within the territory of the State that wants him.
A: (1) Acts/omissions by the State, through the Chief Executive or the executive
Exists only where there is an existing treaty between the parties, which allows States
department or other branches; (2) the injured party’s claim is espoused by the
to exercise its rights to extra-territorial asylum. It cannot exercise such right where
State.
there is no treaty to that effect.
Q: Suppose a private person commits genocide, is the State attributable to the
Q: Is asylum applicable in all cases?
crime?
A: No. It applies only to political offenses.
A: ICC defines the 4 heinous international crimes, one of which is genocide.
However, if there is a convention which defines act of genocide, then the latter shall
apply as it is lex specialis.
VI. Consequences of Rights
Q: Is the State responsible for the drug war?

A: Yes, because the President declared the same, but it may be countered by the 1. Entrance Into Treaties
principle of attribution.
a. Treaty Law - Vienna Convention on the law of treaties (VCLT)
2. Treatment of aliens Article 2 provides: “It is an international agreement concluded between states in
Flowing from its right to existence and as an attribute of sovereignty, no state is under written form and governed by international law whether embodied in a single
obligation to admit aliens. The state can determine in what cases and under what instrument or in two or more related instruments.
conditions it may admit aliens. NOTE: Under the VCLT, a treaty must be in written form. While contracting parties
may enter into an unwritten or oral treaty, the same cannot affect third parties.
NOTE: Q: Is a State obligate to accept aliens?
Internationally, treaty and executive agreement are essentially the same. The
A: No.
differences are immaterial. In the VCLT, both treaty and executive agreement have
Q: What is the difference between a visa and a passport? force and effect and must be complied with in good faith.

i. Preliminary Considerations in Treaty-Making

M.R.A.D.C. LUMBRE 294


CONSTITUTIONAL LAW REVIEW

1. Requisites of a Treaty
a. Entered into by parties having treaty-making capacity (only States or
international organizations can enter into treaties);

b. Through their authorized organs or representatives;

Q: What are full powers?

A: It is a document or proof attesting to a State representative’s


authority to negotiate on behalf of it, or to bind his or her government
or State.

c. Without attendance of duress, fraud, mistake or other vices of


consent; a. Commissioner of Customs v. Eastern Sea Trading, 3 SCRA
351 (1961)
Q: Before allowing the Vietnamese people to regain their
independence, China, as conquering nation, pressured Vietnam into The Court of Tax Appeals entertained doubts on the legality of the
signing a treaty prohibiting it to export rice except to China, and at a executive agreement sought to be implemented by Executive Order
very discounted price. Vietnam goes now to the ICJ to declare said No. 328, owing to the fact that our Senate had not concurred in the
treaty as invalid, considering that it was entered into with force or making of said executive agreement. The concurrence of said House
of Congress is required by our fundamental law in the making of
intimidation. Will the suit prosper?
"treaties" which are, however, distinct and different from "executive
agreements," which may be validly entered into without such
A: No. Generally, consent is absent if it is obtained through the use concurrence.
of force or intimidation, under the Doctrine of Unequal Treaties.
Internationally, however, the use of force, intimidation, or influence Treaties are formal documents which require ratification with the
to obtain consent is allowed. It was the trend or general practice in approval of two thirds of the Senate. Executive agreements become
World War II, even as the treaties were disadvantageous to the binding through executive action without the need of a vote by the
subjugated countries. Senate or by Congress.

d. Proper subject matter and object; International agreements involving political issues or changes of
national policy and those involving international arrangements of a
Q: Example of unlawful subject matter? permanent character usually take the form of treaties. But
international agreements embodying adjustments of detail carrying
A: Must not be beyond the commerce of man, or must not violate the jus out well-established national policies and traditions and those
cogens norms, e.g. torture, genocide, piracy, crimes against humanity, involving arrangements of a more or less temporary nature usually
or crimes of aggression. take form of executive agreements.

As to the treaty between China and NoKor related to nuclear weapons, it The validity of the executive agreement in question is thus patent.
is a mere violation of the States’ obligation under the Convention.
b. Saguisag v. Ochoa, G.R. No. 212426, January 12, 2016
e. Ratification in accordance with their respective constitutional processes; In the seminal case of US v. Curtiss-Wright Export Corp., the US
Supreme Court held that "[t]he President is the sole organ of the
There are States which do not consider ratification as necessary. nation in its external relations, and its sole representative with foreign
relations."
2. Treaty vs. Executive Agreement
The plain meaning of the Constitution prohibits the entry of foreign
military bases, troops or facilities, except by way of a treaty concurred
in by the Senate - a clear limitation on the President's dual role as
defender of the State and as sole authority in foreign relations.

M.R.A.D.C. LUMBRE 295


CONSTITUTIONAL LAW REVIEW

The President, however, may enter into an executive agreement on executive agreements under serious question for the main function of
foreign military bases, troops, or facilities, if (a) it is not the the Executive is to enforce the Constitution and the laws enacted by
instrument that allows the presence of foreign military bases, troops, the Legislature, not to defeat or interfere in the performance of these
or facilities; or (b) it merely aims to implement an existing law or rules. In turn, executive agreements cannot create new international
treaty. obligations that are not expressly allowed or reasonably implied in the
law they purport to implement.
The inapplicability to executive agreements of the requirements under
Section 21 was again recognized in Bayan v. Zamora and in Bayan Second, treaties are, by their very nature, considered superior to
Muna v. Romulo. These cases, both decided under the aegis of the executive agreements. Treaties are products of the acts of the
present Constitution, quoted Eastern Sea Trading in reiterating that Executive and the Senate unlike executive agreements, which are
executive agreements are valid and binding even without the solely executive actions. Because of legislative participation through
concurrence of the Senate. the Senate, a treaty is regarded as being on the same level as a
statute. If there is an irreconcilable conflict, a later law or treaty takes
Executive agreements may dispense with the requirement of Senate precedence over one that is prior. An executive agreement is treated
concurrence because of the legal mandate with which they are differently. Executive agreements that are inconsistent with either a
concluded. As culled from the afore-quoted deliberations of the law or a treaty are considered ineffective. Both types of international
Constitutional Commission, past Supreme Court Decisions, and works agreement are nevertheless subject to the supremacy of the
of noted scholars, executive agreements merely involve Constitution.
arrangements on the implementation of existing policies, rules, laws,
or agreements. They are concluded (1) to adjust the details of a This rule does not imply, though, that the President is given carte
treaty; (2) pursuant to or upon confirmation by an act of the blanche to exercise this discretion. Although the Chief Executive
Legislature; or (3) in the exercise of the President's independent wields the exclusive authority to conduct our foreign relations, this
powers under the Constitution. The raison d'etre of executive power must still be exercised within the context and the parameters
agreements hinges on prior constitutional or legislative set by the Constitution, as well as by existing domestic and
authorizations. international laws. There are constitutional provisions that restrict or
limit the President's prerogative in concluding international
The special nature of an executive agreement is not just a domestic agreements, such as those that involve the following:
variation in international agreements. International practice has
accepted the use of various forms and designations of international a. The policy of freedom from nuclear weapons within Philippine
agreements, ranging from the traditional notion of a treaty - which territory;
connotes a formal, solemn instrument - to engagements concluded in
modem, simplified forms that no longer necessitate ratification. An b. The fixing of tariff rates, import and export quotas, tonnage and
international agreement may take different forms: treaty, act, wharfage dues, and other duties or imposts, which must be pursuant
protocol, agreement, concordat, compromis d'arbitrage, convention, to the authority granted by Congress;
covenant, declaration, exchange of notes, statute, pact, charter, c. The grant of any tax exemption, which must be pursuant to a law
agreed minute, memorandum of agreement, modus vivendi, or some concurred in by a majority of all the Members of Congress;
other form. Consequently, under international law, the distinction
between a treaty and an international agreement or even an d. The contracting or guaranteeing, on behalf of the Philippines, of
executive agreement is irrelevant for purposes of determining foreign loans that must be previously concurred in by the Monetary
international rights and obligations. Board;

However, this principle does not mean that the domestic law e. The authorization of the presence of foreign military bases, troops,
distinguishing treaties, international agreements, and executive or facilities in the country must be in the form of a treaty duly
agreements is relegated to a mere variation in form, or that the concurred in by the Senate;
constitutional requirement of Senate concurrence is demoted to an
f. For agreements that do not fall under paragraph 5, the concurrence
optional constitutional directive. There remain two very important
of the Senate is required, should the form of the government chosen
features that distinguish treaties from executive agreements and
be a treaty.
translate them into terms of art in the domestic setting.
The President had the choice to enter into EDCA by way of an
First, executive agreements must remain traceable to an express or
executive agreement or a treaty.
implied authorization under the Constitution, statutes, or treaties. The
absence of these precedents puts the validity and effectivity of

M.R.A.D.C. LUMBRE 296


CONSTITUTIONAL LAW REVIEW

No court can tell the President to desist from choosing an executive A: No. If there is an existing treaty already allowed, the presence of
agreement over a treaty to embody an international agreement, foreign military troops may be allowed via an executive agreement.
unless the case falls squarely within Article VIII, Section 25. What is important in this case are the three (3) exceptions cited by
xxx the Supreme Court where international agreements may be just in
the form of an executive agreement:
Indeed, in the field of external affairs, the President must be given a
larger measure of authority and wider discretion, subject only to the 1. Only to adjust the details of an existing treaty;
least amount of checks and restrictions under the Constitution. The
rationale behind this power and discretion was recognized by the 2. If it is pursuant or upon confirmation by act of Congress (there
Court in Vinuya v. Executive Secretary, cited earlier. is already a law citing a national policy);
Section 9 of Executive Order No. 459, or the Guidelines in the 3. If it is in the exercise of independent powers of the President
Negotiation of International Agreements and its Ratification, thus, under the Constitution.
correctly reflected the inherent powers of the President when it stated
that the DFA "shall determine whether an agreement is an executive The IPAP v. Ochoa case (cited below) listed the proper subjects of an
agreement or a treaty." executive agreement. However, this is not wise. It is more proper to
Accordingly, in the exercise of its power of judicial review, the Court understand the three (3) principles listed under Bayan v. Zamora.
does not look into whether an international agreement should be in
the form of a treaty or an executive agreement, save in cases in which c. Intellectual Property Association of the Philippines (IPAP)
the Constitution or a statute requires otherwise. Rather, in view of vs. Ochoa, G.R. No. 204605, 19 July 2016
the vast constitutional powers and prerogatives granted to the Before going further, we have to distinguish between treaties and
President in the field of foreign affairs, the task of the Court is to international agreements, which require the Senate's concurrence, on
determine whether the international agreement is consistent with the one hand, and executive agreements, which may be validly entered
applicable limitations. into without the Senate's concurrence. Executive Order No, 459,
xxxx Series of 1997, notes the following definitions, to wit:

Executive agreements may cover the matter of foreign military forces Sec. 2. Definition of Terms.
if it merely involves detail adjustments.
a. International agreement - shall refer to a contract or
Basically, treaties should be concurred by at least 2/3 of the Members understanding, regardless of nomenclature, entered into
of the Senate. Here comes Section 25, Article XVIII regarding foreign between the Philippines and another government in written form
bases, troops, and facilities, which requires three (3) elements: and governed by international law, whether embodied in a single
instrument or in two or more related instruments.
1. It is a treaty duly concurred by the Senate;
b. Treaties - international agreements entered into by the
2. If Congress so requires, a majority of votes in a national Philippines which require legislative concurrence after executive
referendum; ratification. This term may include compacts like conventions,
declarations, covenants and acts.
3. It is recognized as treaty by the other contracting Party;
c. Executive Agreements - similar to treaties except that they
Hence, generally, the treaty requires only 2/3 concurrence, but if it
do not require legislative concurrence.
pertains to foreign bases, troops, the three (3) elements must concur.
Since the Senate vote requirement was silent, the default vote must The Court has highlighted the difference between treaties and
be followed, which is the majority vote. executive agreements in Commissioner of Customs v. Eastern Sea
Trading, thusly:
NOTE:
International agreements involving political issues or changes of
Q: Does EDCA have to comply with the 3 requisites in order for it to national policy and those involving international arrangements of
be valid? a permanent character usually take the form of treaties. But
international agreements embodying adjustments of detail

M.R.A.D.C. LUMBRE 297


CONSTITUTIONAL LAW REVIEW

carrying out well-established national policies and traditions and In this connection, Francis B. Sayre, former U.S. High Commissioner
those involving arrangements of a more or less temporary nature to the Philippines, said in his work on "The Constitutionality of Trade
usually take the form of executive agreements. Agreement Acts":

In the Philippines, the DFA, by virtue of Section 9, Executive Order Agreements concluded by the President which fall short of treaties are
No. 459, is initially given the power to determine whether an commonly referred to as executive agreements and are no less
agreement is to be treated as a treaty or as an executive agreement. common in our scheme of government than are the more formal
To determine the issue of whether DFA Secretary Del Rosario gravely instruments - treaties and conventions. They sometimes take the
abused his discretion in making his determination relative to form of exchanges of notes and at other times that or more formal
the Madrid Protocol, we review the jurisprudence on the nature of documents denominated 'agreements' or 'protocols'. The point where
executive agreements, as well as the subject matters to be covered ordinary correspondence between this and other governments ends
by executive agreements. and agreements - whether denominated executive agreements or
exchanges of notes or otherwise - begin, may sometimes be difficult
The pronouncement in Commissioner of Customs v. Eastern Sea of ready ascertainment. It would be useless to undertake to discuss
Trading is instructive, to wit: here the large variety of executive agreements as such, concluded
from time to time, Hundreds of executive agreements, other than
x x x The concurrence of said House of Congress is required by those entered into under the trade-agreements act, have been
our fundamental law in the making of "treaties" (Constitution of negotiated with foreign governments, x x x It would seem to be
the Philippines, Article VII, Section 10[7]), which are, however, sufficient, in order to show that the trade agreements under the act
distinct and different from "executive agreements," which may of 1934 are not anomalous in character, that they are not treaties,
be validly entered into without such concurrence. and that they have abundant precedent in our history, to refer to
certain classes of agreements heretofore entered into by the
"Treaties are formal documents which require ratification with the Executive without the approval of the Senate. They cover such
approval of two thirds of the Senate. Executive agreements subjects as the inspection of vessels, navigation dues, income
become binding through executive action without the need of a tax on shipping profits, the admission of civil aircraft, customs
vote by the Senate or by Congress. matters, and commercial relations generally, international
claims, postal matters, the registration of trademarks and
xxxx copyrights, etcetera. Some of them were concluded not by
specific congressional authorization but in conformity with
“x x x The right of the Executive to enter into binding agreements policies declared in acts of Congress with respect to the
without the necessity of subsequent Congressional approval has general subject matter, such as tariff acts; while still others,
been confirmed by long usage.” particularly those with respect of the settlement of claims against
foreign governments, were concluded independently of any
From the earliest days of our history we have entered into legislation.
executive agreements covering such subjects as commercial and
consular relations, most-favored-nation rights, patent Accordingly, DFA Secretary Del Rosario's determination and
rights, trademark and copyright protection, postal and treatment of the Madrid Protocol as an executive agreement, being in
navigation arrangements and the settlement of claims, The apparent contemplation of the express state policies on intellectual
validity of these has never been seriously questioned by our property as well as within his power under Executive Order No. 459,
courts. are upheld. We observe at this point that there are no hard and fast
rules on the propriety of entering into a treaty or an executive
xxxx agreement on a given subject as an instrument of international
relations. The primary consideration in the choice of the form of
Agreements with respect to the registration of agreement is the parties' intent and desire to craft their international
trademarks have been concluded by the Executive with various agreement in the form they so wish to further their respective
countries under the Act of Congress of March 3, 1881. x x x interests. The matter of form takes a back seat when it comes to
effectiveness and binding effect of the enforcement of a treaty or an
xxxx executive agreement, inasmuch as all the parties, regardless of the
form, become obliged to comply conformably with the time-honored
principle of pacta sunt servanda.

M.R.A.D.C. LUMBRE 298


CONSTITUTIONAL LAW REVIEW

d. Sec. 20, Art. VII of the Constitution instalments necessary to comply with such Agreements constitute a
ratification thereof, which places the question of validity out of the
The President may contract or guarantee foreign loans on behalf Court’s reach, no constitutional principle having been invoked to
of the Republic of the Philippines with the prior concurrence of restrict Congress’ plenary power to appropriate funds — loan or no
the Monetary Board, and subject to such limitations as may be loan.
provided by law. The Monetary Board shall, within thirty days
from the end of every quarter of the calendar year, submit to the f. Importance of Knowing the Distinction Between Executive
Congress a complete report of its decision on applications for Agreement and Treaty - Bayan v. Zamora, G.R. No. 138570,
loans to be contracted or guaranteed by the Government or October 10, 2000
government-owned and controlled corporations which would
have the effect of increasing the foreign debt, and containing The VFA is not unconstitutional.
other matters as may be provided by law.
Section 25, Article XVIII disallows foreign military bases, troops, or
NOTE: This may be in the form of an executive agreement. It facilities in the country, unless the following conditions are sufficiently
empowers the President to enter into loan contract, with met, viz: (a) it must be under a treaty; (b) the treaty must be duly
recommendation by the Monetary Board, but the power alone concurred in by the Senate and, when so required by congress,
rests in the Executive. ratified by a majority of the votes cast by the people in a national
referendum; and (c) recognized as a treaty by the other contracting
Article II, Section 12 – also applicable, because the state.
exploration/exploitation agreement may be entered into with
another State (or privately owned companies). There is no dispute as to the presence of the first two requisites in the
e. Two Classes of Executive Agreements - USAFFE Veterans case of the VFA. The concurrence handed by the Senate through
Association, Inc. v. The Treasurer of the Philippines, G.R. No. Resolution No. 18 is in accordance with the provisions of the
10500, June 30, 1959 Constitution . . . the provision in [in §25, Article XVIII] requiring
ratification by a majority of the votes cast in a national referendum
Executive Agreements fall into two classes: (1) agreements made being unnecessary since Congress has not required it.
purely as executive acts affecting external relations and
independent of or without legislative authorization, which may be This Court is of the firm view that the phrase “recognized as a
termed as presidential agreements, and (2) agreements entered into treaty” means that the other contracting party accepts or
in pursuance of acts of Congress, which have been designated as acknowledges the agreement as a treaty. To require the other
Congressional-Executive Agreements. contracting state, the United States of America in this case, to
submit the VFA to the United States Senate for concurrence pursuant
The Romulo-Snyder Agreement may fall under any of these two
to its Constitution, is to accord strict meaning to the phrase.
classes, for precisely on September 18, 1946, Congress of the
Philippines specifically authorized the President of the Philippines to
Well-entrenched is the principle that the words used in the
obtain such loans or incur such indebtedness with the Government of
Constitution are to be given their ordinary meaning except where
the United States, its agencies or instrumentalities. even granting,
technical terms are employed, in which case the significance thus
arguendo, that there was no legislative authorization, it is hereby
attached to them prevails. Its language should be understood in the
maintained that the Romulo-Snyder Agreement was legally and
sense they have in common use.
validly entered into to conform to the second category, namely,
‘agreements entered into purely as executive acts without legislative
Moreover, it is inconsequential whether the United States treats the
authorization.’ This second category usually includes money
VFA only as an executive agreement because, under international law,
agreements relating to the settlement of pecuniary claims of citizens.
an executive agreement is as binding as a treaty. To be sure, as long
It may be said that this method of settling such claims has come to
as the VFA possesses the elements of an agreement under
be the usual way of dealing with matters of this kind.
international law, the said agreement is to be taken equally as a
Indeed, the Agreement was not submitted to the U.S. Senate either; treaty.
but we do not stop to check the authorities above listed nor test the
conclusions derived therefrom in order to render a definite The records reveal that the United States Government, through
pronouncement, for the reason that our Senate Resolution No. 15 3 Ambassador Thomas C. Hubbard, has stated that the United States
practically admits the validity and binding force of such Agreement. government has fully committed to living up to the terms of the
Furthermore, the acts of Congress Appropriating funds for the yearly VFA. For as long as the United States of America accepts or

M.R.A.D.C. LUMBRE 299


CONSTITUTIONAL LAW REVIEW

acknowledges the VFA as a treaty, and binds itself further to comply 2. The doctrine cannot operate Retroactively, i.e., it must
with its obligations under the treaty, there is indeed marked not adversely affect provisions which have already been
compliance with the mandate of the Constitution. complied with prior to the vital change in the situation;

NOTE: The distinction between a treaty or international agreement 3. The change must have been Unforeseen or
and an executive agreement is of great significance in the Philippines unforeseeable at the time of the perfection of the treaty;
because the procedure followed in the process of ratification is
different. 4. The doctrine must be invoked within a Reasonable time;

If what is involved is a treaty, the concurrence by at least two-thirds 5. The duration of the treaty must be Indefinite;
of all the Members of the Senate is required. On the other hand, if
6. The change must be so Substantial that the foundation
what is involved is an executive, there is no such requirement.
of the treaty must have altogether disappeared;
Treaties have similar principles to contracts:
Limitation: The principle of rebus sic stantibus cannot be
1. Consensual; invoked as a ground for terminating or withdrawing from a
2. Relativity; treaty if:
3. Non-retroactivity, unless the parties so provide;
1. The treaty establishes a boundary; or
4. Object;
5. Obligatoriness – pacta sunt servanda 2. The fundamental change is the result of a breach by
the party invoking it of an obligation under the treaty or
3. Principles Governing Treaties of any other obligation owed to any other party to the
treaty.
a. General Rule: Pacta Sunt Servanda “treaties shall be
complied with” This doctrine does not operate automatically. There is
Every Treaty in force is binding between the parties to it and must be necessity for a formal act of rejection, usually by the Head
performed by them in good faith (Art. 26, Vienna Convention on the of State, with the statement of the reasons why compliance
Law of Treaties) with the treaty is no longer required.

i. Exceptions: 2. Treaty in Violation of the Constitution

1. Rebus Sic Stantibus “things thus standing” A State may not invoke the fact that its consent to be bound
by a treaty has been expressed in violation of a provision of
It states that a fundamental change of circumstances which its internal law regarding competence to conclude treaties as
determined the parties to accept a treaty, if it has resulted invalidating its consent unless that violation was manifest
in a radical transformation of the extent of the obligations and concerned a rule of its internal law of fundamental
imposed by it, may under certain conditions, afford the party importance.
affected a ground to invoke the termination of the treaty. Generally, the State cannot invoke its own laws to escape
liability or from complying from its treaty obligations, the
The change must have increased the burden of the
same being a violation of pacta sunt servanda.
obligations to be executed to the extent of rendering
performance essentially different from the original intention. However, a treaty which violates the provisions of the
Constitution is an exception to pacta sunt servanda.
Requisites (Not-IR, Must-URIS): Remember the principle that the doctrine of incorporation
applies to customary international law and general principles
1. The change must not have been caused by the party of international law, and the doctrine of transformation
Invoking the doctrine; applies to treaties/conventions. Also, international law has
the same level as domestic laws.

M.R.A.D.C. LUMBRE 300


CONSTITUTIONAL LAW REVIEW

As such, a treaty can be subsequently amended or abrogated 1. A right arises for a third State from a provision of a treaty
by local legislation, as it is of the same level as domestic law. if the parties to the treaty intend the provision to accord
A domestic law also cannot violate the Constitution. that right either to the third State, or to a group of States
At the risk of being sanctioned internationally, the Philippine to which it belongs, or to all States, and the third State
courts must decide in favor of upholding Constitutional assents thereto. Its assent shall be presumed so long as
provisions, even as the State should violate treaty the contrary is not indicated, unless the treaty otherwise
obligations, because what the doctrine of pacta sunt provides.
servanda prohibits is the invoking of local laws, but not the
Constitution. 2. A State exercising a right in accordance with paragraph 1
shall comply with the conditions for its exercise provided
It is actually the Senate, who violates the Constitution by
for in the treaty or established in conformity with the
concurring with the treaty.
treaty.
ii. General Rule: Principle of Autonomy
Stipulation pour autrui is an exception to the relativity of
The parties are free to stipulate or choose the subject matter of
the contract, but it cannot be outside the commerce of man. contracts. It applies only to giving rights to third parties. If
however, the treaty involves the conferment of obligations on a
In governmental parlance, autonomy refers to self-governance. third state, the latter has to give its consent, thereby making
An example of an autonomous jurisdiction was the former United said third state a party to the treaty, in effect making the treaty
States governance of the Philippine Islands. The Philippine a multi-lateral one.
Autonomy Act of 1916 provided the framework for the creation
of an autonomous government under which the Filipino people NOTES:
had broader domestic autonomy than previously, although it
reserved certain privileges to the United States to protect its Q: Can a third State send a statement declaring its non-assent
sovereign rights and interests. to the agreement?

1. Exception: Jus cogens A: Yes. In this case, the party may. It is important to
remember, however, is that assent is presumed, unless the
See discussion above. party declares otherwise.
iii. General Rule: Non-retroactivity
Q: Can it cancel its obligation or rights to the contract, as it is
1. Exception: VCLT, Art. 28 not a party to the contract?

Unless a different intention appears from the treaty or is A: Assent is only presumed. However, the party may send his
otherwise established, its provisions do not bind a party in non-assent or refusal of the benefit given.
relation to any act or fact which took place or any situation
which ceased to exist before the date of the entry into force Q: Suppose after it sends its assent, can it subsequently
of the treaty with respect to that party. revoke?

iv. General Rule: Pacta tertiis nec nocent nec prosunt A: No, as it is a multi-lateral treaty, the party’s withdrawal must
“agreement does not benefit or hurt a third person” be with the consent of the contracting parties because of the
principle of pacta sunt servanda.
A treaty binds only the parties and does not create obligations for
a third state. Q: A treaty is entered into between State A and State B
1. Exceptions: conferring an obligation on the part of State C. Is this valid?

a. Treaties which have become part of Customary A: Stipulations pour autrui pertains to conferment of rights, not
International Law obligations.
b. Stipulation Pour Autrui (VCLT, Art. 36) 4. Steps in Treaty-Making

M.R.A.D.C. LUMBRE 301


CONSTITUTIONAL LAW REVIEW

a. Negotiation Note: It should be emphasized that under the Constitution the


power to ratify is vested in the President subject to the
Conducted by the parties to reach an agreement on the terms.
concurrence of the Senate.
b. Signing
The President has the discretion even after the signing of the
The signing of the text of the instrument agreed upon by the parties.
treaty by the Philippine representative whether or not to ratify a
c. Consent to be Bound treaty. The signature of the representative does not signify final
consent, it is ratification that binds the state to the provisions of
1. Definitive signature
the treaty and renders it effective. Senate is limited only to
NOTES: giving or withholding its consent, concurrence to the ratification.
It is within the President to refuse to submit a treaty to the
Q: Difference between simple v. definitive signature?
Senate or having secured its consent for its ratification, refuse
A: Simple signature involves a reservation that the same is to ratify it. Such decision is within the competence of the
subject to ratification by the State or concurrence by the Senate. President alone, which cannot be encroached by this court via
writ of mandamus. (Pimentel v. Executive Secretary, G.R.
Definitive signature does not require ratification. In the PH, we No.158088, July 6, 2005)
cannot give definitive signature.
a. Ratification in the Philippines
2. Exchange of instruments constituting a treaty
The act by which the provisions of a treaty are formally
Applies only to bilateral treaties. After adoption of the treaty, a confirmed and approved by the State.
contracting party, if it wants to be bound by the provisions of
the treaty, shall send to the other contracting party its signed Negotiation: The authority of the Secretary of the Foreign Affairs
copy of the treaty, thereby giving its consent to be bound. is always presumed. The document of full powers is only required
when the representative is not the DFA Secretary or President.
3. Acceptance and approval
Adoption of the Text: First agreement refers to the adoption of
Applies only in cases where the domestic law of the country does
the treaty. If it is a multi-lateral treaty, the same will be opened
not require ratification. Only upon acceptance and approval of the
Head of State, arises the consent to be bound. to signing by all, but the date of opening shall be agreed upon.

4. Accession i. Pimentel v. Executive Secretary, G.R. No. 158088, July


6, 2005
A State can accede to a treaty only if invited or permitted to do
so by the contracting parties. Such invitation or permission is Justice Isagani Cruz, in his book on International Law, describes
usually given in the accession clause of the treaty itself. the treaty-making process in this wise: The usual steps in the
treaty-making process are: negotiation, signature,
5. Ratification ratification, and exchange of the instruments of
Ratification is the act by which the provisions of a treaty are ratification. The treaty may then be submitted for
formally confirmed and approved by a State. By ratifying a treaty registration and publication under the U.N. Charter,
although this step is not essential to the validity of the
signed in its behalf, a State expresses its willingness to be bound
agreement as between the parties.
by the provisions of such treaty.
Negotiation may be undertaken directly by the head of state
Note: A State may ratify a treaty only when it is a signatory to but he now usually assigns this task to his authorized
it. There is no moral duty on the part of the States to ratify a representatives. These representatives are provided with
treaty notwithstanding that its plenipotentiaries have signed the credentials known as full powers, which they exhibit to the other
same. This step, however, should not be taken lightly. A treaty negotiators at the start of the formal discussions. It is standard
may provide that it shall not be valid even if ratified but shall be practice for one of the parties to submit a draft of the proposed
valid only after the exchange or deposit of ratification has treaty which, together with the counter-proposals, becomes the
transpired. basis of the subsequent negotiations. The negotiations may be

M.R.A.D.C. LUMBRE 302


CONSTITUTIONAL LAW REVIEW

brief or protracted, depending on the issues involved, and may the President for ratification. After the President has ratified the
even “collapse” in case the parties are unable to come to an treaty, the Department of Foreign Affairs shall submit the same
agreement on the points under consideration. to the Senate for concurrence. Upon receipt of the concurrence
of the Senate, the Department of Foreign Affairs shall comply
If and when the negotiators finally decide on the terms of the with the provisions of the treaty to render it effective. Section 7
treaty, the same is opened for signature. This step is primarily of Executive Order No. 459 reads:
intended as a means of authenticating the instrument and for the
purpose of symbolizing the good faith of the parties; but, Sec. 7. Domestic Requirements for the Entry into Force of
significantly, it does not indicate the final consent of the a Treaty or an Executive Agreement. The domestic
state in cases where ratification of the treaty is required. requirements for the entry into force of a treaty or an
The document is ordinarily signed in accordance with the executive agreement, or any amendment thereto, shall be
alternat, that is, each of the several negotiators is allowed to sign as follows:
first on the copy which he will bring home to his own state.
A. Executive Agreements.
Ratification, which is the next step, is the formal act by which
a state confirms and accepts the provisions of a treaty concluded i. All executive agreements shall be transmitted to the
by its representatives. The purpose of ratification is to enable Department of Foreign Affairs after their signing for the
the contracting states to examine the treaty more closely preparation of the ratification papers. The transmittal
and to give them an opportunity to refuse to be bound by shall include the highlights of the agreements and the
it should they find it inimical to their interests. It is for this benefits which will accrue to the Philippines arising from
reason that most treaties are made subject to the scrutiny them.
and consent of a department of the government other than ii. The Department of Foreign Affairs, pursuant to the
that which negotiated them. endorsement by the concerned agency, shall transmit the
agreements to the President of the Philippines for his
The last step in the treaty-making process is the exchange of ratification. The original signed instrument of ratification
the instruments of ratification, which usually also signifies the shall then be returned to the Department of Foreign
effectivity of the treaty unless a different date has been agreed Affairs for appropriate action.
upon by the parties. Where ratification is dispensed with and no
effectivity clause is embodied in the treaty, the instrument is B. Treaties.
deemed effective upon its signature.
i. All treaties, regardless of their designation, shall comply
Petitioners’ arguments equate the signing of the treaty by the with the requirements provided in sub-paragraph[s] 1
Philippine representative with ratification. It should be and 2, item A (Executive Agreements) of this Section. In
underscored that the signing of the treaty and the addition, the Department of Foreign Affairs shall submit
ratification are two separate and distinct steps in the the treaties to the Senate of the Philippines for
treaty-making process. As earlier discussed, the signature is concurrence in the ratification by the President. A
primarily intended as a means of authenticating the instrument certified true copy of the treaties, in such numbers as
and as a symbol of the good faith of the parties. It is usually may be required by the Senate, together with a certified
performed by the state’s authorized representative in the true copy of the ratification instrument, shall accompany
diplomatic mission. Ratification, on the other hand, is the formal the submission of the treaties to the Senate.
act by which a state confirms and accepts the provisions of a
treaty concluded by its representative. ii. Upon receipt of the concurrence by the Senate, the
Department of Foreign Affairs shall comply with the
Thus, Executive Order No. 459 issued by President Fidel V. Ramos provision of the treaties in effecting their entry into force.
on November 25, 1997 provides the guidelines in the negotiation
of international agreements and its ratification. It mandates that It should be emphasized that under our Constitution, the power
after the treaty has been signed by the Philippine representative, to ratify is vested in the President, subject to the concurrence of
the same shall be transmitted to the Department of Foreign the Senate. The role of the Senate, however, is limited only to
Affairs. The Department of Foreign Affairs shall then prepare the giving or withholding its consent, or concurrence, to the
ratification papers and forward the signed copy of the treaty to ratification. Hence, it is within the authority of the President to

M.R.A.D.C. LUMBRE 303


CONSTITUTIONAL LAW REVIEW

refuse to submit a treaty to the Senate or, having secured its The Constitution also regulates the foreign relations
consent for its ratification, refuse to ratify it. Although the refusal powers of the Chief Executive when it provides that "no
of a state to ratify a treaty which has been signed in its behalf is treaty or international agreement shall be valid and
a serious step that should not be taken lightly, such decision is effective unless concurred in by at least two-thirds of all
within the competence of the President alone, which cannot be the members of the Senate." Even more pointedly, the
encroached by this Court via a writ of mandamus. SC has no Transitory Provisions state:
jurisdiction over actions seeking to enjoin the President in the
performance of his official duties and therefore, cannot issue the
Sec. 25. After the expiration in 1991 of the Agreement
writ of mandamus prayed for by the petitioners as it is beyond
its jurisdiction to compel the executive branch of the government between the Republic of the Philippines and the United
to transmit the signed text of Rome Statute to the Senate. States of America concerning Military Bases, foreign military
bases, troops or facilities shall not be allowed in the
ii. Lim v. Executive Secretary, G.R. No. 151445, April 11, Philippines except under a treaty duly concurred in by the
2002 Senate and, when the Congress so requires, ratified by a
In our considered opinion, neither the Mutual Defense Treaty majority of the votes cast by the people in a national
(MDT) nor the Visiting Forces Agreement (VFA) allow foreign referendum held for that purpose, and recognized as a treaty
troops to engage in an offensive war on Philippine territory. We by the other contracting state.
bear in mind the salutary proscription stated in the Charter of the
United Nations. A rather recent formulation of the relation of international law vis-
a-vis municipal law was expressed in Philip Morris, Inc. v.
In the same manner, both the Mutual Defense Treaty and the
Court of Appeals, to wit:
Visiting Forces Agreement, as in all other treaties and
international agreements to which the Philippines is a party, must xxx Withal, the fact that international law has been made
be read in the context of the 1987 Constitution. In particular, the part of the law of the land does not by any means imply the
Mutual Defense Treaty was concluded way before the present
primacy of international law over national law in the
Charter, though it nevertheless remains in effect as a valid source
municipal sphere. Under the doctrine of incorporation as
of international obligation.
applied in most countries, rules of international law are given
The present Constitution contains key provisions useful in a standing equal, not superior, to national legislation.
determining the extent to which foreign military troops are
allowed in Philippine territory. Thus, in the Declaration of This is not exactly helpful in solving the problem at hand since in
Principles and State Policies, it is provided that: trying to find a middle ground, it favors neither one law nor the
other, which only leaves the hapless seeker with an unsolved
SEC. 2. The Philippines renounces war as an instrument of dilemma. Other more traditional approaches may offer valuable
national policy, adopts the generally accepted principles of insights.
international law as part of the law of the land and adheres
to the policy of peace, equality, justice, freedom, From the perspective of public international law, a treaty is
cooperation, and amity with all nations. favored over municipal law pursuant to the principle of pacta sunt
servanda. Hence, "every treaty in force is binding upon the
SEC. 7. The State shall pursue an independent foreign policy. parties to it and must be performed by them in good
In its relations with other states the paramount consideration faith." Further, a party to a treaty is not allowed to "invoke the
shall be national sovereignty, territorial integrity, national provisions of its internal law as justification for its failure to
interest, and the right to self- determination. perform a treaty."

SEC. 8. The Philippines, consistent with the national interest, Our Constitution espouses the opposing view. Witness our
adopts and pursues a policy of freedom from nuclear jurisdiction as I stated in section 5 of Article VIII:
weapons in the country.
The Supreme Court shall have the following powers:
xxx

M.R.A.D.C. LUMBRE 304


CONSTITUTIONAL LAW REVIEW

(2) Review, revise, reverse, modify, or affirm on appeal positive duty, or a virtual refusal to perform the duty enjoined or
or certiorari, as the law or the Rules of Court may provide, act in contemplation of law, or where the power is exercised in
final judgments and order of lower courts in: an arbitrary and despotic manner by reason of passion and
personal hostility."
(A) All cases in which the constitutionality or validity of any
treaty, international or executive agreement, law, In this connection, it will not be amiss to add that the Supreme
presidential decree, proclamation, order, instruction, Court is not a trier of facts.
ordinance, or regulation is in question.
Under the expanded concept of judicial power under the
xxx Constitution, courts are charged with the duty "to determine
whether or not there has been a grave abuse of discretion
In Ichong v. Hernandez, we ruled that the provisions of a treaty
amounting to lack or excess of jurisdiction on the part of any
are always subject to qualification or amendment by a
branch or instrumentality of the government." From the facts
subsequent law, or that it is subject to the police power of the
State. obtaining, we find that the holding of "Balikatan 02-1"
joint military exercise has not intruded into that penumbra
The foregoing premises leave us no doubt that US forces of error that would otherwise call for correction on our
are prohibited / from engaging in an offensive war on part. In other words, respondents in the case at bar have
Philippine territory. not committed grave abuse of discretion amounting to
lack or excess of jurisdiction.
Yet a nagging question remains: are American troops
actively engaged in combat alongside Filipino soldiers d. Entry into Force
under the guise of an alleged training and assistance A treaty enters into force in such manner and upon such date as it
exercise? Contrary to what petitioners would have us do, we may provide or as the negotiating States may agree. Failing any such
cannot take judicial notice of the events transpiring down south, provision or agreement, a treaty enters into force as soon as consent
as reported from the saturation coverage of the media. As a rule, to be bound by the treaty has been established for all the negotiating
we do not take cognizance of newspaper or electronic reports per States.
se, not because of any issue as to their truth, accuracy, or
impartiality, but for the simple reason that facts must be e. Registration with the UN
established in accordance with the rules of evidence. As a result,
1. Charter of the United Nations, Art. 102 and Art. 6
we cannot accept, in the absence of concrete proof, petitioners'
allegation that the Arroyo government is engaged in Article 102
"doublespeak" in trying to pass off as a mere training exercise an
offensive effort by foreign troops on native soil. The petitions “1. Every treaty and every international agreement entered into by
invite us to speculate on what is really happening in Mindanao, any Member of the United Nations after the present Charter comes
to issue I make factual findings on matters well beyond our into force shall as soon as possible be registered with the Secretariat
immediate perception, and this we are understandably loath to and published by it.
do.
2. No party to any such treaty or international agreement which has
It is all too apparent that the determination thereof involves not been registered in accordance with the provisions of paragraph 1
basically a question of fact. On this point, we must concur with of this Article may invoke that treaty or agreement before any organ
the Solicitor General that the present subject matter is not a fit of the United Nations.”
topic for a special civil action for certiorari. We have held in too
Article 6
many instances that questions of fact are not entertained in such
a remedy. The sole object of the writ is to correct errors of “A Member of the United Nations who has persistently violated the
jurisdiction or grave abuse of discretion: The phrase "grave abuse Organization by the General Assembly upon the recommendation of
of discretion" has a precise meaning in law, denoting abuse of
discretion "too patent and gross as to amount to an evasion of a

M.R.A.D.C. LUMBRE 305


CONSTITUTIONAL LAW REVIEW

the Security Council.” the Principles contained in the present Charter which is given when the non-contracting party gives its consent to be bound
may be expelled from ICJ or ICC. by the jurisdiction of the ICJ.

It is not one of the processes in treaty-making, as registration is not ii. Forum Prorogatum “prorogated jurisdiction”
required for the validity of a treaty. However, this step is important Prorogated jurisdiction, which occurs when a power is conferred – by the
as the party cannot invoke the treaty before any organ of the United consent of the parties and following the initiation of proceedings – upon the
Nations if it did not register the same to the UN Secretariat. International Court of Justice, which otherwise would not have adjudicated.
Such consent can be indicated in an implied or informal way or by a succession
b. Peaceful and Forcible Sanctions
of acts (Anglo-Iranian Oil Case [1952] ICJ Rep 93).
Peaceful Sanctions:
This principle has been applied by the Permanent Court of International Justice
1. Arbitration; (PCIJ) and the ICJ as a flexible way of founding the jurisdiction of the court
over the merits of the dispute.
2. Embargo;
3. Severance of Diplomatic Ties; VII. Specialist Areas of International Law
4. Referral of the case to the ICJ.
Forcible Sanctions: 1. The International Law of the Sea (UNCLOS)

1. Retorsion; a. Baselines

2. Reprisals; It is a line from which the breadth of the territorial sea, the contiguous zone and
the exclusive economic zone is measured in order to determine the maritime
3. War. boundary of the coastal State.
i. Concept of ex aequo et bono “according to the right and good”
b. Archipelagic states
It is a judgment based on considerations of fairness, not on considerations of
existing law; that is, to simply decide the case based upon a balancing of the A state constituted wholly by one or more archipelagos and may include other
equities. islands. (Article 46, UNCLOS)

Article 38 i. Two kinds of archipelago according to the UNCLOS

1. The Court, whose function is to decide in accordance with It means a group of islands, including parts of islands, interconnecting waters
international law such disputes as are submitted to it, shall apply: and other natural features which are so closely interrelated that such
islands, waters and other natural features form an intrinsic geographical,
a. international conventions, whether general or particular, economic and political entity, or which historically have been regarded as
establishing rules expressly recognized by the contesting states; such (UNCLOS, Art. 46).
b. international custom, as evidence of a general practice accepted ii. Straight archipelagic baselines
as law;
An archipelagic State may draw straight archipelagic baselines by joining the
c. the general principles of law recognized by civilized nations; outermost points of the outermost islands and drying reefs of the archipelago
provided that within such baselines are included the main islands and an area
d. subject to the provisions of Article 59, judicial decisions and the
in which the ration of the water to the area of the land, including atolls, is
teachings of the most highly qualified publicists of the various
between 1 to 1 and 9 to 1 (UNCLOS, Art. 47).
nations, as subsidiary means for the determination of rules of law.
iii. Archipelagic waters
2. This provision shall not prejudice the power of the Court to decide
a case ex aequo et bono, if the parties agree thereto. These are waters enclosed by the archipelagic baselines, regardless of their
depth or distance from the coast (UNCLOS, Art. 49[1]).
NOTE: This principle can only be applied if the parties agree or consent
thereto. There are two (2) kinds of consent: (1) Prior Consent, wherein all iv. Archipelagic sea lanes passage
signatories to the ICJ give their prior consent; and (2) Subsequent Consent,

M.R.A.D.C. LUMBRE 306


CONSTITUTIONAL LAW REVIEW

GR: As a rule, ships of all States enjoy the right of innocent passage through part of the internal waters of the coastal state. However, in the case of archipelagic
archipelagic waters (UNCLOS, Art. 52[1]). states, waters landward of the baseline other than those of rivers, bays, and lakes,
are archipelagic waters (UNCLOS, Art. 8 [1]).
XPN: Right of Innocent Passage may be suspended in some areas of its
archipelagic waters. But such suspension must be: d. Territorial sea
1. Without discrimination in form or in fact among foreign ships; Every State has the right to establish the breadth of the territorial sea up to a limit
not exceeding 12 nautical miles, measured from baselines (UNCLOS, Art. 3).
2. Essential for the protection of its security; and
i. Arigo vs. Swift, G.R. No. 206501, 16 September 2014
3. Shall take effect only after having been duly published (UNCLOS, Art.
52[2]). During the deliberations, Senior Associate Justice Antonio T. Carpio took the
position that the conduct of the US in this case, when its warship entered a
v. Regime of Islands restricted area in violation of R.A. No. 10067 and caused damage to the TRNP
1. An island is a naturally formed area of land, surrounded by water, which is reef system, brings the matter within the ambit of Article 31 of the United
above water at high tide. Nations Convention on the Law of the Sea (UNCLOS). He explained that while
historically, warships enjoy sovereign immunity from suit as extensions of their
2. Except as provided for in paragraph 3, the territorial sea, the contiguous flag State, Art. 31 of the UNCLOS creates an exception to this rule in cases
zone and the continental shelf of an island are determined in accordance with where they fail to comply with the rules and regulations of the coastal State
the provisions of the Convention applicable to other land territory. regarding passage through the latter's internal waters and the territorial sea.
3. Rocks which cannot sustain human habitation or economic life of their own According to Justice Carpio, although the US to date has not ratified the
shall have no exclusive economic zone or continental shelf. (UNCLOS, Article UNCLOS, as a matter of long-standing policy the US considers itself bound by
121) customary international rules on the "traditional uses of the oceans" as codified
in UNCLOS, as can be gleaned from previous declarations by former Presidents
NOTE: Islands can be very important because of the possibility of exploiting oil
Reagan and Clinton, and the US judiciary in the case of United States v. Royal
and gas resources around them. This explains the controversy over Spratleys.
Caribbean Cruise Lines, Ltd.
It is noteworthy that islands can have their own territorial sea, exclusive
economic zone and continental shelf. The international law of the sea is generally defined as "a body of treaty rules
arid customary norms governing the uses of the sea, the exploitation of its
However, rocks “which cannot sustain human habitation or economic life” only
resources, and the exercise of jurisdiction over maritime regimes. It is a branch
have a territorial sea. But there is no clear international law definition of
of public international law, regulating the relations of states with respect to the
“economic life” referred to in n. 3. (Bernas, Introduction to Public International
uses of the oceans."28 The UNCLOS is a multilateral treaty which was opened
Law 2009, p. 129)
for signature on December 10, 1982 at Montego Bay, Jamaica. It was ratified
Artificial islands or installations are not “islands” in the sense of Article 121 of by the Philippines in 1984 but came into force on November 16, 1994 upon the
the UNCLOS. However, coastal states may establish safety zones around submission of the 60th ratification.
artificial islands and prescribe safety measures around them. [ibid, citing
The UNCLOS is a product of international negotiation that seeks to balance
UNCLOS, Article 60(4) and (5)]
State sovereignty (mare clausum) and the principle of freedom of the high
The baseline in the following areas over which the Philippines likewise exercises seas (mare liberum).29 The freedom to use the world's marine waters is one
sovereignty and jurisdiction shall be determined as "Regime of Islands" under of the oldest customary principles of international law.30 The UNCLOS gives to
the Republic of the Philippines consistent with Article 121 of the United Nations the coastal State sovereign rights in varying degrees over the different zones
Convention on the Law of the Sea (UNCLOS): of the sea which are: 1) internal waters, 2) territorial sea, 3) contiguous zone,
4) exclusive economic zone, and 5) the high seas. It also gives coastal States
a) The Kalayaan Island Group as constituted under Presidential Decree No. more or less jurisdiction over foreign vessels depending on where the vessel is
1596; and located.
Insofar as the internal waters and territorial sea is concerned, the Coastal State
b) Bajo de Masinloc, also known as Scarborough
exercises sovereignty, subject to the UNCLOS and other rules of international
Shoal. (Republic Act No. 9522, Sect. 2)ERS
law. Such sovereignty extends to the air space over the territorial sea as well
c. Internal waters as to its bed and subsoil.

These are waters of lakes, rivers and bays landward of the baseline of the territorial In the case of warships, as pointed out by Justice Carpio, they continue to
sea. Waters on the landward side of the baseline of the territorial sea also form enjoy sovereign immunity subject to the following exceptions:

M.R.A.D.C. LUMBRE 307


CONSTITUTIONAL LAW REVIEW

Article 30 While UNCLOS cleared the Senate Foreign Relations Committee (SFRC) during
the 108th and 110th Congresses, its progress continues to be hamstrung by
Non-compliance by warships with the laws and regulations of the significant pockets of political ambivalence over U.S. participation in
coastal State international institutions. Most recently, 111 th Congress SFRC Chairman
If any warship does not comply with the laws and regulations of the Senator John Kerry included "voting out" UNCLOS for full Senate consideration
coastal State concerning passage through the territorial sea and among his highest priorities. This did not occur, and no Senate action has been
disregards any request for compliance therewith which is made to it, taken on UNCLOS by the 112th Congress.
the coastal State may require it to leave the territorial sea Justice Carpio invited our attention to the policy statement given by President
immediately. Reagan on March 10, 1983 that the US will "recognize the rights of the other ,
Article 31 states in the waters off their coasts, as reflected in the convention [UNCLOS],
so long as the rights and freedom of the United States and others under
Responsibility of the flag State for damage caused by a warship or international law are recognized by such coastal states", and President
other government ship operated for non-commercial purposes Clinton's reiteration of the US policy "to act in a manner consistent with its
[UNCLOS] provisions relating to traditional uses of the oceans and to
The flag State shall bear international responsibility for any loss or
encourage other countries to do likewise." Since Article 31 relates to the
damage to the coastal State resulting from the non-compliance by a
"traditional uses of the oceans," and "if under its policy, the US 'recognize[s]
warship or other government ship operated for non-commercial
the rights of the other states in the waters off their coasts,"' Justice Carpio
purposes with the laws and regulations of the coastal State
postulates that "there is more reason to expect it to recognize the rights of
concerning passage through the territorial sea or with the provisions
other states in their internal waters, such as the Sulu Sea in this case."
of this Convention or other rules of international law.
As to the non-ratification by the US, Justice Carpio emphasizes that "the US'
Article 32
refusal to join the UN CLOS was centered on its disagreement with UN CLOS'
Immunities of warships and other government ships operated for regime of deep seabed mining (Part XI) which considers the oceans and deep
non-commercial purposes seabed commonly owned by mankind," pointing out that such "has nothing to
do with its [the US'] acceptance of customary international rules on
With such exceptions as are contained in subsection A and in articles 30 and navigation."
31, nothing in this Convention affects the immunities of warships and other
government ships operated for non-commercial purposes. (Emphasis It may be mentioned that even the US Navy Judge Advocate General's Corps
supplied.) A foreign warship's unauthorized entry into our internal waters with publicly endorses the ratification of the UNCLOS, as shown by the following
resulting damage to marine resources is one situation in which the above statement posted on its official website:
provisions may apply. But what if the offending warship is a non-party to the
The Convention is in the national interest of the United States because it
UNCLOS, as in this case, the US?
establishes stable maritime zones, including a maximum outer limit for
An overwhelming majority - over 80% -- of nation states are now members of territorial seas; codifies innocent passage, transit passage, and archipelagic
UNCLOS, but despite this the US, the world's leading maritime power, has not sea lanes passage rights; works against "jurisdictional creep" by preventing
ratified it. coastal nations from expanding their own maritime zones; and reaffirms
sovereign immunity of warships, auxiliaries and government aircraft.
While the Reagan administration was instrumental in UNCLOS' negotiation and
drafting, the U.S. delegation ultimately voted against and refrained from xxxx
signing it due to concerns over deep seabed mining technology transfer
Economically, accession to the Convention would support our national interests
provisions contained in Part XI. In a remarkable, multilateral effort to induce
by enhancing the ability of the US to assert its sovereign rights over the
U.S. membership, the bulk of UNCLOS member states cooperated over the
resources of one of the largest continental shelves in the world. Further, it is
succeeding decade to revise the objectionable provisions. The revisions
the Law of the Sea Convention that first established the concept of a maritime
satisfied the Clinton administration, which signed the revised Part XI
Exclusive Economic Zone out to 200 nautical miles, and recognized the rights
implementing agreement in 1994. In the fall of 1994, President Clinton
of coastal states to conserve and manage the natural resources in this Zone.
transmitted UNCLOS and the Part XI implementing agreement to the Senate
requesting its advice and consent. Despite consistent support from President We fully concur with Justice Carpio's view that non-membership in the UNCLOS
Clinton, each of his successors, and an ideologically diverse array of does not mean that the US will disregard the rights of the Philippines as a
stakeholders, the Senate has since withheld the consent required for the Coastal State over its internal waters and territorial sea. We thus expect the
President to internationally bind the United States to UNCLOS. US to bear "international responsibility" under Art. 31 in connection with the
USS Guardian grounding which adversely affected the Tubbataha reefs.

M.R.A.D.C. LUMBRE 308


CONSTITUTIONAL LAW REVIEW

Indeed, it is difficult to imagine that our long-time ally and trading partner, The UNCLOS unifies the continental and the extended continental shelves into one
which has been actively supporting the country's efforts to preserve our vital by providing that the continental shelf extends to the breadth of either shelf,
marine resources, would shirk from its obligation to compensate the damage whichever is the farthest (UNCLOS, Art. 76[1][4]).
caused by its warship while transiting our internal waters. Much less can we
comprehend a Government exercising leadership in international affairs, i. Extended continental shelf
unwilling to comply with the UNCLOS directive for all nations to cooperate in It is that portion of the continental shelf that lies beyond the 200 nautical miles
the global task to protect and preserve the marine environment as provided in limit in the juridical/legal continental Shelf (Ibid).
Article 197, viz:
Benham Plateau – It is also known as the Benham Rise. The Philippines lodged
Article 197 its claim on the area with the United Nations Commission on the Limits of the
Cooperation on a global or regional basis Continental Shelf on April 8, 2009. The UNCLOS approved the claim of the
Philippines that the Benham Plateau is part of Philippine Territory on April 12,
States shall cooperate on a global basis and, as appropriate, on a 2012.
regional basis, directly or through competent international
organizations, in formulating and elaborating international rules, Sovereign rights of a coastal State over the continental shelf
standards and recommended practices and procedures consistent 1. Right to explore and exploit its natural resources (UNCLOS, Art. 77[1]);
with this Convention, for the protection and preservation of the
marine environment, taking into account characteristic regional NOTE: This right is exclusive. Should the coastal State not explore or exploit
features. the natural resources, no one may undertake these activities without the
express consent of the coastal State (UNCLOS, Art. 77[2]). Natural resources
In fine, the relevance of UNCLOS provisions to the present controversy is include mineral and other non-living resources of the seabed and subsoil
beyond dispute. Although the said treaty upholds the immunity of warships together with living organisms belonging to sedentary species (UNCLOS, Art.
from the jurisdiction of Coastal States while navigating the latter's territorial 77[4]).
sea, the flag States shall be required to leave the territorial sea immediately if
they flout the laws and regulations of the Coastal State, and they will be liable g. In the Matter of the South China Sea Arbitration (Philippines v. China),
for damages caused by their warships or any other government vessel PCA Case No. 2013-19, 12 July 2016 (You can read online summaries)
operated for non-commercial purposes under Article 31.
The South China Sea Arbitration was conducted between the Republic of the
e. Exclusive economic zone Philippines and the People’s Republic of China by the Permanent Court of Arbitration
(PCA), under the 1982 United Nations Convention on the Law of the Sea (UNCLOS).
It gives the coastal State sovereign rights overall economic resources of the sea, The arbitration is related to disputes between the Parties regarding the legal basis
sea-bed and subsoil in an area extending not more than 200 nautical miles beyond of maritime rights and entitlements, the status of certain geographic features, and
the baseline from which the territorial sea is measured (UNCLOS, Arts. 55 & 57). the lawfulness of certain actions taken by China in the South China Sea; in
f. Continental shelf particular, the following four issues, as raised by Philippines:

Otherwise known as archipelagic or insular shelf for archipelagos, refers to a) the To resolve a dispute between the parties regarding the source of maritime rights
seabed and subsoil of the submarine areas adjacent to the meters or, beyond that and entitlements in the South China Sea;
limit, to where the depth allows exploitation, and b) the seabed and subsoil of areas To resolve a dispute between the parties concerning the entitlements to maritime
adjacent to islands. zones that would be generated under the Convention by Scarborough Shoal and
It comprises the sea-bed and subsoil of the submarine areas that extend beyond certain maritime features in the Spratly Islands that are claimed by both the
its territorial sea throughout the natural prolongation of its land territory to the parties;
outer edge of the continental margin or to a distance of 200 nautical miles beyond To resolve a series of disputes concerning the lawfulness of China’s actions in the
the baselines from which the breadth of the territorial sea is measured if the edge South China Sea, vis-à-vis interfering with Philippine’s rights, failing to protect and
of the continental margin does not extend up to that distance (UNCLOS, Art. 76[1]). preserve the marine environment, and inflicting harm on the marine environment
NOTE: The rights of the coastal state over the continental shelf do not depend on (through land reclamation and construction of artificial islands);
occupation, effective or notional, or on any express proclamation (UNCLOS, Art. To find that China has aggravated and extended the disputes between the Parties
77[3]). by restricting access to a detachment of Philippines Marines stationed at Second
Thomas Shoal.

M.R.A.D.C. LUMBRE 309


CONSTITUTIONAL LAW REVIEW

While China and Philippines are both parties to the UNCLOS, China specifically made supersedes any treaties in force before its coming into force. It questioned China’s
a declaration in 2006 to exclude maritime boundary delimitation from its claim to historical rights in the region, and established that China’s state practice
acceptance of compulsory dispute settlement. In addition, China has shown does not show that China had been enjoying any historical rights in the South China
disagreement with Philippines’ decision to take the matter to arbitration and has Sea; rather, it was enjoying the freedom of the high seas and since it did not create
decided neither to agree with the decision of the Tribunal nor to participate in the bar to other states’ usage of the same, it could not be understood as being a
proceedings. historical right. Furthermore, since China’s publishing of the same in its Notes
Verbales in 2009, many states have objected to its claim as well. “The Tribunal
The Tribunal, on its end, has taken cognizance of these factors and has purported concludes that the Convention superseded any historic rights or other sovereign
to not deal with delimiting maritime boundaries. Furthermore, the Tribunal did not rights or jurisdiction in excess of the limits imposed therein.” However, the Tribunal
bar the proceedings, on the basis of Article 9 of Annex VII of UNCLOS. In addition, also concluded that its jurisdiction was limited to the claims of historic rights on the
the Tribunal also noted that despite China’s absence from the proceedings, since it maritime region and not to the land masses in the South China Sea, i.e. if it can
is a party to the UNCLOS, the decision of the Tribunal would, in fact, be binding claim historic rights on any of the islands, then it may also be able to claim maritime
upon it, pursuant to Article 296 (1) and Article 11 of Annex VII. zones (as per the Convention) on the basis of these islands.
China’s Foreign Ministry, further, stated its position with regard to the proceedings Next, the Tribunal looked at Philippines’ submissions 3 to 7, concerning the nature
by publishing a Position Paper in 2014. It claimed that the Tribunal lacks jurisdiction of the features in the South China Sea. It differentiates between low-tide
over the matter because: elevations, high-tide features and rocks. In its Award on Jurisdiction, the Tribunal
The essence of the subject-matter of the arbitration is the territorial sovereignty clarified that:
over the relevant maritime features in the South China Sea; This is not a dispute concerning sovereignty over the features, notwithstanding any
China and the Philippines have agreed, through bilateral instruments and the possible question concerning whether low-tide elevations may be subjected to a
Declaration on the Conduct of Parties in the South China Sea, to settle their relevant claim of territorial sovereignty. Nor is this a dispute concerning sea boundary
disputes through negotiations; delimitation: the status of a feature as a “low-tide elevation”, “island”, or a “rock”
relates to the entitlement to maritime zones generated by that feature, not to the
Philippines’ disputes would constitute an integral part of maritime delimitation delimitation of such entitlements in the event that they overlap.
between the two countries.
The Philippines put forward three categories for classifying low-tide elevations:
The Tribunal considered China’s Position Paper as a plea on jurisdiction, and where a low-tide elevation is located within 12 miles of a high-tide feature, where
conducted a separate hearing on the issue of jurisdiction and admissibility. the low-tide elevation is beyond 12 miles but within the state’s exclusive economic
Additionally, the Tribunal also declared that it would honour China’s declaration of zone or continental shelf, and where the low-tide elevation is located beyond the
2006 and the UNCLOS and would neither delve into issues of maritime boundary areas of natural jurisdiction.
delimitation or questions of sovereignty. The Philippines also stated that it, “does
not seek in this arbitration a determination of which Party enjoys sovereignty over For the purpose of identifying the nature of the features in the South China Sea,
the islands claimed by both of them. Nor does it request a delimitation of any the Tribunal relied upon satellite imagery that had been conducted on the area and
maritime boundaries.” direct surveys that had been carried out, by navies or otherwise, in the area, and
relied upon maps that were sufficiently detailed. They chose a certain tidal height
Pursuant to this, the Tribunal issued its Award on Jurisdiction in October 2015, in to maintain uniformity across the features, and decided to rely, in cases where
which it concluded that it did indeed have jurisdiction in the case, as per Philippines’ there had been significant man-made changes, alterations or construction on the
Final Submissions, and that China’s lack of participation would not prove to be a features, upon maps/imagery/surveys that depicted the features as they had been
bar to its proceedings. It, further, concluded that the treaties China was relying on in their original form.
were either political in nature and not legally binding, or that they did were legally
binding and yet did not bar either Party from alternative means of dispute Again the Tribunal relied upon statements previously made by China to obtain their
resolution. In accordance with Article 283 of the UNCLOS, the Tribunal found that stance on the nature of the features, since China had neither submitted any
this requirement was met in the diplomatic communications between the Parties document to the Tribunal nor had it discussed these in its Position Paper.
and that Philippines’ initiation of proceedings under the UNCLOS did not constitute The Tribunal concluded that Scarborough Shoal, Cuarteron Reef, Fiery Cross Reef,
an abuse of of process as claimed by China. Johnson Reef, McKennan Reef and Gaven Reef (North) were all found to be high-
The Tribunal, proceeding with the first two submissions made by the Philippines, tide features. The Tribunal further noted that for the purposes of Article 121(3),
considered the validity of China’s claim to historic rights in the maritime region of the high-tide features at Scarborough Shoal and the reefs were rocks that cannot
the South China Sea and the ‘Nine-Dash Line’. Through a lengthy analysis of the sustain human human habitation or economic life of their own and so have no
text and context of the Convention, in line with the principles set out in the Vienna exclusive economic zone or continental shelf. The Tribunal found the same to be
Convention on the Law of Treaties, the Tribunal established that the Convention true of the Spratly Islands and so concluded that China, therefore, has no
entitlement to any maritime zone in the area of Mischief Reef or Second Thomas

M.R.A.D.C. LUMBRE 310


CONSTITUTIONAL LAW REVIEW

Shoal; they do, however, form part of the exclusive economic zone and continental independent experts on coral reef biology, expert briefs and satellite imagery,
shelf of the Philippines as they lie within 200 nautical miles of the Philippines’ coast found that China was in breach of the Convention for failing to stop the fishing
and there are no overlapping entitlements in the area with respect to China. vessels from engaging in harmful harvesting practices and also for its island-
building activities. The Tribunal further opined that China’s construction on Mischief
On the contrary, Hughes Reef, Gaven Reef (South), Subi Reef, Mischief Reef and Reef, without authorization from Philippines was in violation of Philippines’
Second Thomas Shoal were all found to be low-tide elevations, of which Hughes sovereign rights in its exclusive economic zone and continental shelf and a breach
Reef lay within 12 miles of McKennan Reef and Sin Cowe Island, Gaven Reef of the Convention.
(South) lay within 12 miles of Gaven Reef (North) and Namyit Island, and Subi
Reef lay within 12 miles of the high-tide feature of Sandy Cay on the reefs to the The next consideration before the Tribunal was the demeanour of China’s law
west of Thitu. enforcement vessels at Scarborough Shoal and the lawfulness of these actions. The
Philippines also raised the issue under the relevant provisions of the Convention on
In the issue of Chinese interference with the living and non-living resources the International Regulations for Preventing of Collisions at Sea, 1972 (COLREGS).
(primarily concerned with fishing practices in the South China Sea and oil and gas The Tribunal found that China, through the actions of its law enforcement vessels,
exploration and exploitation) of the Philippines, the Tribunal considered diplomatic endangered Philippine vessels and personnel and created a serious risk of collision
statements from China to the Philippines and regulations related to the matter that and found China in breach of Article 94 of the Convention.
China had passed domestically. The Philippines put forward four contentions related
to living resources: China’s prevention of fishing by Philippine vessels at Mischief The Tribunal, in response to Submission 14 of the Philippines, opined that China
Reef since 1995, and at Second Thomas Shoal since 1995, China’s revision of the had, in the course of the proceedings of this arbitration, aggravated and extended
Hainan Regulation and China’s moratorium on fishing in the South China Sea in its disputes with Philippines, through its actions of dredging, artificial island-
2012. The Tribunal finds that China had breached Articles 77 and 56 of the building and construction activities.
Convention through the operation of its marine surveillance vessels (which
interfered with Philippines’ oil and gas exploration) and through its moratorium on Lastly, the Tribunal did not find it necessary to make any further declaration, owing
fishing which interfered with the exclusive economic zone of the Philippines, to the fact that both the parties are already parties to the Convention and are
respectively. already obliged to comply with it.

The Tribunal also found China in breach of Article 58 (3) of the Convention, due to Source: http://rsilpak.org/case-brief-on-the-south-china-sea-arbitration/
its failure to prevent fishing by Chinese flagged ships in the exclusive economic
zone of the Philippines, failing to respect the sovereign rights of the Philippines over
its fisheries in its exclusive economic zone. 2. International Humanitarian Law

Submission 10 of the Philippines related to China’s interference with Philippines’ a. R.A. 9851 (Philippine Act on Crimes Against International Humanitarian
fishing vessels and practices in the Scarborough Shoal. While both the states had Law, Genocide, and Other Crimes Against Humanity)
conflicting views on the situation (China believed that it was Philippines who was
causing the interference) and both claimed historic rights (Philippines distinguished CHAPTER III
this by clarifying that it only referred to historic fishing rights) to the region, the CRIMES AGAINST INTERNATIONAL HUMANITARIAN LAW,
Tribunal opined that China was, in fact, in contravention of the Convention by GENOCIDE AND OTHER CRIMES AGAINST HUMANITY
interfering with the traditional fishing practice of the Philippines in its exclusive
Section 4. War Crimes. - For the purpose of this Act, "war crimes" or "crimes against
economic zone through the deployment of its official ships in the region. The
International Humanitarian Law" means:
Tribunal also noted that this decision does not depend on the question of
sovereignty, and that the Tribunal once again refrained from commenting on the (a) In case of an international armed conflict, grave breaches of the Geneva Conventions of
matter. 12 August 1949, namely, any of the following acts against persons or property protected
under provisions of the relevant Geneva Convention:
Philippines’ successive contention related to China’s activities on the reefs in the
South China Sea, with regards the practices it had adopted for the purpose of large-
(1) Willful killing;
scale construction and reclamation at seven locations in the Spratly Islands, and
(2) Torture or inhuman treatment, including biological experiments;
its practices with regards to fishing in the South China Sea. Philippines claimed that (3) Willfully causing great suffering, or serious injury to body or health;
China had been harming and causing damage to the marine environment of the (4) Extensive destruction and appropriation of property not justified by military necessity
South China Sea through these practices and despite objections from the and carried out unlawfully and wantonly;
surrounding states, China had not ceased its actions. It was also noted that while (5) Willfully depriving a prisoner of war or other protected person of the rights of fair and
some of the fishing ships were not state-appointed ships and were being manned regular trial;
by non-state actors, the Chinese government had neither condemned their actions (6) Arbitrary deportation or forcible transfer of population or unlawful confinement;
nor made any efforts to stop them from proceeding. The Tribunal, assisted by three (7) Taking of hostages;

M.R.A.D.C. LUMBRE 311


CONSTITUTIONAL LAW REVIEW

(8) Compelling a prisoner of war or other protected person to serve in the forces of a whether such building or place has been used to make an effective contribution to
hostile power; and military action, it shall be presumed not to be so used;
(9) Unjustifiable delay in the repatriation of prisoners of war or other protected persons. (11) Subjecting persons who are in the power of an adverse party to physical mutilation
or to medical or scientific experiments of any kind, or to removal of tissue or organs for
(b) In case of a non-international armed conflict, serious violations of common Article 3 to transplantation, which are neither justified by the medical, dental or hospital treatment
the four (4) Geneva Conventions of 12 August 1949, namely, any of the following acts of the person concerned nor carried out in his/her interest, and which cause death to or
committed against persons taking no active part in the hostilities, including member of the seriously endanger the health of such person or persons;
armed forces who have laid down their arms and those placed hors de combat by sickness, (12) Killing, wounding or capturing an adversary by resort to perfidy;
wounds, detention or any other cause; (13) Declaring that no quarter will be given;
(14) Destroying or seizing the enemy's property unless such destruction or seizure is
(1) Violence to life and person, in particular, willful killings, mutilation, cruel treatment imperatively demanded by the necessities of war;
and torture; (15) Pillaging a town or place, even when taken by assault;
(2) Committing outrages upon personal dignity, in particular, humiliating and degrading (16) Ordering the displacements of the civilian population for reasons related to the
treatment; conflict, unless the security of the civilians involved or imperative military reasons so
(3) Taking of hostages; and demand;
(4) The passing of sentences and the carrying out of executions without previous (17) Transferring, directly or indirectly, by the occupying power of parts of its own civilian
judgment pronounced by a regularly constituted court, affording all judicial guarantees population into the territory it occupies, or the deportation or transfer of all or parts of
which are generally recognized as indispensable. the population of the occupied territory within or outside this territory;
(18) Committing outrages upon personal dignity, in particular, humiliating and degrading
(c) Other serious violations of the laws and customs applicable in armed conflict, within the treatments;
established framework of international law, namely: (19) Committing rape, sexual slavery, enforced prostitution, forced pregnancy, enforced
sterilization, or any other form of sexual violence also constituting a grave breach of the
(1) Internationally directing attacks against the civilian population as such or against Geneva Conventions or a serious violation of common Article 3 to the Geneva
individual civilians not taking direct part in hostilities; Conventions;
(2) Intentionally directing attacks against civilian objects, that is, object which are not (20) Utilizing the presence of a civilian or other protected person to render certain points,
military objectives; areas or military forces immune from military operations;
(3) Intentionally directing attacks against buildings, material, medical units and (21) Intentionally using starvation of civilians as a method of warfare by depriving them
transport, and personnel using the distinctive emblems of the Geneva Conventions or of objects indispensable to their survival, including willfully impeding relief supplies as
Additional Protocol III in conformity with intentional law; provided for under the Geneva Conventions and their Additional Protocols;
(22) In an international armed conflict, compelling the nationals of the hostile party to
(4) Intentionally directing attacks against personnel, installations, material, units or take part in the operations of war directed against their own country, even if they were
vehicles involved in a humanitarian assistance or peacekeeping mission in accordance in the belligerent's service before the commencement of the war;
with the Charter of the United Nations, as ling as they are entitled to the protection given (23) In an international armed conflict, declaring abolished, suspended or inadmissible
to civilians or civilian objects under the international law of armed conflict; in a court of law the rights and actions of the nationals of the hostile party;
(5) Launching an attack in the knowledge that such attack will cause incidental loss of (24) Committing any of the following acts:
life or injury to civilians or damage to civilian objects or widespread, long-term and
severe damage to the natural environment which would be excessive in relation to the (i) Conscripting, enlisting or recruiting children under the age of fifteen (15) years
concrete and direct military advantage anticipated; into the national armed forces;
(6) Launching an attack against works or installations containing dangerous forces in the (ii) Conscripting, enlisting or recruiting children under the age of eighteen (18) years
knowledge that such attack will cause excessive loss of life, injury to civilians or damage into an armed force or group other than the national armed forces; and
to civilian objects, and causing death or serious injury to body or health. (iii) Using children under the age of eighteen (18) years to participate actively in
(7) Attacking or bombarding, by whatever means, towns, villages, dwellings or buildings hostilities; and
which are undefended and which are not military objectives, or making non-defended (25) Employing means of warfare which are prohibited under international law, such
localities or demilitarized zones the object of attack; as:
(8) Killing or wounding a person in the knowledge that he/she is hors de combat, (i) Poison or poisoned weapons;
including a combatant who, having laid down his/her arms or no longer having means of (ii) Asphyxiating, poisonous or other gases, and all analogous liquids, materials or
defense, has surrendered at discretion; devices;
(9) Making improper use of a flag of truce, of the flag or the military insignia and uniform (iii) Bullets which expand or flatten easily in the human body, such as bullets with
of the enemy or of the United Nations, as well as of the distinctive emblems of the Geneva hard envelopes which do not entirely cover the core or are pierced with incisions;
Conventions or other protective signs under International Humanitarian Law, resulting and
in death, serious personal injury or capture; (iv) Weapons, projectiles and material and methods of warfare which are of the
(10) Intentionally directing attacks against buildings dedicated to religion, education, art, nature to cause superfluous injury or unnecessary suffering or which are inherently
science or charitable purposes, historic monuments, hospitals and places where the sick indiscriminate in violation of the international law of armed conflict.
and wounded are collected, provided they are not military objectives. In case of doubt

M.R.A.D.C. LUMBRE 312


CONSTITUTIONAL LAW REVIEW

Any person found guilty of committing any of the acts specified herein shall suffer the penalty b. Principle 21 of the Stockholm Declaration
provided under Section 7 of this Act.
States have, in accordance with the Charter of the United Nations and the principles
Section 5. Genocide - (a) For the purpose of this Act, "genocide" means any of the following of international law, the sovereign right to exploit their own resources pursuant to
acts with intent to destroy, in whole or in part, a national, ethnic, racial, religious, social or their own environmental policies, and the responsibility to ensure that activities
any other similar stable and permanent group as such: within their jurisdiction or control do not cause damage to the environment of other
States or of areas beyond the limits of national jurisdiction.
(1) Killing members of the group;
(2) Causing serious bodily or mental harm to members of the group; NOTE
(3) Deliberately inflicting on the group conditions of life calculated to bring about its
Q: Suppose South Korea files a complaint for violation of international
physical destruction in whole or in part;
environmental laws against China, can the latter invoke as defense the fact that it
(4) Imposing measures intended to prevent births within the group; and
(5) Forcibly transferring children of the group to another group. operates under its own environmental laws?

A: No, under the above principle.


(b) It shall be unlawful for any person to directly and publicly incite others to commit
genocide. 4. International Criminal Law

Any person found guilty of committing any of the acts specified in paragraphs (a) and (b) of a. Rome Statute of the International Criminal Court (take special attention
this section shall suffer the penalty provided under Section 7 of this Act. to Arts. 5 and 127)

Section 6. Other Crimes Against Humanity. - For the purpose of this act, "other crimes against Art. V
humanity" means any of the following acts when committed as part of a widespread or The jurisdiction of the Court shall be limited to the most serious crimes of
systematic attack directed against any civilian population, with knowledge of the attack: concern to the international community as a whole. The Court has
jurisdiction in accordance with this Statute with respect to the following
(a) Willful killing;
crimes:
(b) Extermination;
(c) Enslavement;
(a) The crime of genocide;
(d) Arbitrary deportation or forcible transfer of population;
(e) Imprisonment or other severe deprivation of physical liberty in violation of
(b) Crimes against humanity;
fundamental rules of international law;
(f) Torture;
(c) War crimes;
(g) Rape, sexual slavery, enforced prostitution, forced pregnancy, enforced sterilization,
or any other form of sexual violence of comparable gravity;
(h) Persecution against any identifiable group or collectivity on political, racial, national,
(d) The crime of aggression.
ethnic, cultural, religious, gender, sexual orientation or other grounds that are
universally recognized as impermissible under international law, in connection with any
The Court shall exercise jurisdiction over the crime of aggression once a
act referred to in this paragraph or any crime defined in this Act; provision is adopted in accordance with articles 121 and 123 defining the
(i) Enforced or involuntary disappearance of persons; crime and setting out the conditions under which the Court shall exercise
(j) Apartheid; and jurisdiction with respect to this crime. Such a provision shall be consistent
(k) Other inhumane acts of a similar character intentionally causing great suffering, or with the relevant provisions of the Charter of the United Nations.
serious injury to body or to mental or physical health.
Art. CXXVII
Any person found guilty of committing any of the acts specified herein shall suffer the penalty
provided under Section 7 of this Act. 1. A State Party may, by written notification addressed to the Secretary-
General of the United Nations, withdraw from this Statute. The withdrawal
3. International Environmental Law shall take effect one year after the date of receipt of the notification,
a. Precautionary principle unless the notification specifies a later date.

This principle is expressed in the Rio Declaration, which stipulates that, where there 2. A State shall not be discharged, by reason of its withdrawal, from the
are “threats of serious or irreversible damage, lack of full scientific certainty shall obligations arising from this Statute while it was a Party to the Statute,
not be used as a reason for postponing cost-effective measures to prevent including any financial obligations which may have accrued. Its
environmental degradation.” withdrawal shall not affect any cooperation with the Court in connection
with criminal investigations and proceedings in relation to which the

M.R.A.D.C. LUMBRE 313


CONSTITUTIONAL LAW REVIEW

withdrawing State had a duty to cooperate and which were commenced A: No. The State still has the obligation to coordinate with the investigation.
prior to the date on which the withdrawal became effective, nor shall it
prejudice in any way the continued consideration of any matter which was
already under consideration by the Court prior to the date on which the
withdrawal became effective.
~~~~~~~ FIN ~~~~~~~

NOTES: NOTE: Based on Atty. Enan’s observations on Bar exam trends, 70-80% of the political
law questions are from case laws covering the last 4 years, while 20-30% of the
Q: A person files a complaint against the President with the ICC. Can the President
questions are answerable by codal provisions.
escape from liability by withdrawing from the ICC?
A: No, Article 127 of the ICC.
Q: Can the President just ignore the summonses of the ICC or not comply with the
orders of the ICC as it has withdrawn from the ICC?

M.R.A.D.C. LUMBRE 314


CONSTITUTIONAL LAW REVIEW

EXTENT and DEFINITION RIGHTS and POWERS OF STATES


1. Rights under existing agreement on the part of the
third states should be respected.

2. The traditional fishing rights and other legitimate


activities of the immediately adjacent neighboring
These are waters enclosed by the
States.
Internal Waters archipelagic baselines, regardless of their depth
or distance from the coast.
3. Existing submarine cables laid by other States and
“passing through its waters without making a windfall”
as well as the maintenance and
replacement of such cables upon being notified of their
location and the intention to repair or replace them.
Territorial Seas are defined by historic right or
treaty limits.
Coastal states exercise sovereignty over Territorial sea
Territorial Seas As defined in the Convention on the Law of the and it extends to the airspace over the territorial sea and
Sea, it has a uniform breadth of 12 miles to its seabed and subsoil.
measured from the lower water mark of the
coast.
The coastal state does not have sovereignty over the
contiguous zone because the contiguous zone is a zone
of jurisdiction for a particular purpose, not of
It is the zone adjacent to the territorial sea. The sovereignty.
contiguous zone may not extend more than 24
Contiguous nautical miles beyond the baseline from which the State may exercise control as is necessary to:
Zone breadth of the territorial sea is measured 12
nautical miles from the 1. Prevent infringement of its customs, fiscal,
territorial sea. immigration, or sanitary laws within its territory or its
territorial sea, or

2. Punish such infringement.


States may exercise:
It gives the coastal State sovereign
rights overall economic resources of 1. Sovereign rights;
the sea, sea-bed and subsoil in an 2. Jurisdictional rights;
Exclusive
area extending not more than 200 3. Other rights and duties provided for in the Law of the
Economic Zone
nautical miles beyond the Sea Convention.
baseline from which the territorial
sea is measured (Please see discussion on rights of the coastal state in
the EEZ, p. 50)

They are beyond the jurisdiction and sovereign rights of


The waters, which do not constitute
state.
the internal waters, archipelagic
High Seas
waters, territorial sea and exclusive
It is treated as res communes or res nullius, and thus,
economic zone of a state.
are not part of the territory of a particular State.

M.R.A.D.C. LUMBRE 315


CONSTITUTIONAL LAW REVIEW

Latin Maxims Under International Law obligations imposed by it, may under certain conditions, afford the party affected
a ground to invoke the termination of the treaty.

10. Pacta tertiis nec nocent nec prosunt “agreement does not benefit or hurt a
1. Opinio Juris “an opinion of law” third person”

Recognition of a practice as a legal norm and therefore obligatory. A treaty binds only the parties and does not create obligations for a third state.

2. Jus cogens “compelling law” 11. Ex aequo et bono “according to the right and good”

A norm accepted and recognizes by the international community as a norm from It is a judgment based on considerations of fairness, not on considerations of
which no derogation is permitted, and which can be modified only by a subsequent existing law; that is, to simply decide the case based upon a balancing of the
norm of general international law having the same character. equities. (Art. 38, Rome Statute)

3. Erga Omnes Obligation “towards all” 12. Forum Prorogatum “prorogated jurisdiction”

An obligation of every State towards the international community as a whole. Prorogated jurisdiction, which occurs when a power is conferred – by the consent
of the parties and following the initiation of proceedings – upon the International
4. Non liquet “it is not clear” Court of Justice, which otherwise would not have adjudicated. Such consent can be
indicated in an implied or informal way or by a succession of acts (Anglo-Iranian
Non liquet means the possibility that a court or tribunal could not decide a case
Oil Case [1952] ICJ Rep 93).
because of a ‘gap’ in law.

5. Uti Possidetis “as you possess”

A principle of international law that allows retention of property or territory in the


belligerent’s actual possession at the time of cessation of hostilities.

6. Aut Dedere Aut Judicare (either extradite or prosecute)

Legal obligation of states under PIL to prosecute persons who commit serious
international crimes where no other state has requested extradition.

7. Lex Specialis “law governing a specific subject matter”

It is a legal construct whereby an internationally unlawful conduct of state organ


acting in that capacity is regarded as the conduct of state itself, making that state
responsible for its own acts or omissions, but as an abstract entity it can physically
act only through individuals or group of individuals performing “act of the state” on
its behalf.

8. Pacta Sunt Servanda “treaties shall be complied with”

Every Treaty in force is binding between the parties to it and must be performed
by them in good faith (Article 26, VCLT)

9. Rebus Sic Stantibus “things thus standing”

It states that a fundamental change of circumstances which determined the parties


to accept a treaty, if it has resulted in a radical transformation of the extent of the

M.R.A.D.C. LUMBRE 316


CONSTITUTIONAL LAW REVIEW

REFERENCES:
1987 Constitution of the Philippines and statutes
Related Treaties/Conventions
Nachura, Antonio E.B., 2014 and 2016 ed. Outline Reviewer in Political Law.

Suarez, Rolando, 2018 ed. Political Law Reviewer.


Supreme Court Decisions

2017 UST Golden Notes


Atty. Enan’s Comments :D

M.R.A.D.C. LUMBRE 317

You might also like